Sunteți pe pagina 1din 499

American Board of Family Medicine

2016 IN-TRAINING EXAMINATION

CRITIQUE BOOK

This book contains the answers to each question in the In-Training Examination, as well as a critique that
provides a rationale for the correct answer. Bibliographic references are included at the end of each critique
to facilitate any further study you may wish to do in a particular area.

Copyright 2016 The American Board of Family Medicine, Inc. All rights reserved.
Item 1

ANSWER: A

GLP-1 receptor agonists are the most likely of these medications to lead to weight loss. They work by
activating the GLP-1 receptors, which increases insulin secretion, decreases glucagon secretion, slows
gastric emptying, and increases satiety. Other diabetes medications that may promote weight loss include
metformin, amylin mimetics, and SGLT-2 inhibitors. The other medications listed have all been shown to
cause weight gain. DPP-4 inhibitors are felt to be weight neutral.

Ref: American Diabetes Association: Standards of medical care in diabetes2015:7. Approaches to glycemic treatment. Diabetes
Care 2015;38(Suppl 1):S41-S48.

Item 2

ANSWER: B

The hepatitis C virus is spread through contact with infected blood (usually unscreened blood transfusions,
which were the norm before 1992), intravenous drug use, or high-risk sexual activity. However, studies
have shown that screening only patients with high-risk medical or clinical histories will identify just
20%33% of infected patients. As a different strategy, both the CDC and the U.S. Preventive Services
Task Force have recommended one-time screening of patients born between 1945 and 1965. It is estimated
that this cohort includes 75% of all patients who have chronic hepatitis C.

Ref: Chou R, Cottrell EB, Wasson N, et al: Screening for hepatitis C virus infection in adults: A systematic review for the US
Preventive Services Task Force. Ann Intern Med 2013;158(2):101-108. 2) Smith BD, Yartel AK, Krauskopf K, et al:
Hepatitis C virus antibody positivity and predictors among previously undiagnosed adult primary care outpatients:
Cross-sectional analysis of a multisite retrospective cohort study. Clin Infect Dis 2015;60(8):1145-1152.

Item 3

ANSWER: D

Family physicians are frequently consulted by patients and institutions regarding outbreaks of head lice.
Optimal treatment involves home eradication measures and medication. Permethrin 1% shampoo remains
the first-line treatment. Ivermectin appears effective for treatment but is not FDA approved and is not
considered first-line treatment. Lindane carries an increased risk of toxicity and should not be used in
children weighing <50 kg. Topical benzyl alcohol is FDA-approved for treatment but is expensive. The
current recommendation from the American Academy of Pediatrics Council on School Health and
Committee on Infectious Diseases is to return children to class despite the presence of nits.

Ref: Frankowski BL, Bocchini JA Jr; Council on School Health and Committee on Infectious Diseases: Head lice. Pediatrics
2010;126(2):392-403. 2) Gunning K, Pippitt K, Kiraly B, Sayler M: Pediculosis and scabies: Treatment update. Am Fam
Physician 2012;86(6):535-541.

1
Item 4

ANSWER: B

This patient most likely has ischemic colitis, given his abdominal pain, bloody diarrhea, and cardiovascular
risk factors. Peptic ulcer disease is unlikely because the nasogastric aspirate was negative. Diverticular
bleeding and angiodysplasia are painless. Infectious colitis is associated with fever.

Ref: Wilkins T, Baird C, Pearson AN, Schade RR: Diverticular bleeding. Am Fam Physician 2009;80(9):977-983. 2) Kasper
DL, Fauci AS, Hauser SL, et al (eds): Harrisons Principles of Internal Medicine, ed 19. McGraw-Hill, 2015, p 1957.

Item 5

ANSWER: C

Overdiagnosis is the diagnosis of a disease that will not produce symptoms during a patients lifetime. It
tends to occur with cancers that have very slow rates of growth. Prostate cancer is most often a
slow-growing cancer and is often present without symptoms in older men. The introduction of
prostate-specific antigen (PSA) screening was accompanied by a marked rise in the rate of diagnosis of
prostate cancer while mortality decreased much less significantly, and this decrease was probably largely
attributable to improved treatment. The problem of overdiagnosis remains a significant problem with PSA
screening and was a factor in the U.S. Preventive Services Task Forces recommendation against routine
PSA-based screening for prostate cancer (D recommendation).

Ref: US Preventive Services Task Force: Screening for cervical cancer: Recommendation statement. Am Fam Physician
2012;86(6):555-559. 2) Tirona MT: Breast cancer screening update. Am Fam Physician 2013;87(4):274-278. 3) Gates
TJ: Screening for cancer: Concepts and controversies. Am Fam Physician 2014;90(9):625-631. 4) Mulhem E, Fulbright
N, Duncan N: Prostate cancer screening. Am Fam Physician 2015;92(8):683-688.

Item 6

ANSWER: B

This patient most likely has patellofemoral pain syndrome, as evidenced by her recent increased activity
and anterior knee pain with increased lateral patellar tracking. The most effective treatment for
patellofemoral pain syndrome is physical therapy and activity modification. Glucosamine and chondroitin
have been used to treat osteoarthritis in the past but should not be used to treat symptomatic osteoarthritis
of the knee, according to the American Academy of Orthopedic Surgeons. Intra-articular joint injections
can be used for knee arthritis but are not indicated for patellofemoral pain syndrome. Knee arthroscopy
would not be indicated for patellofemoral syndrome without evidence of internal knee derangement. The
evidence for patellar taping is inconsistent, but it may be helpful as an adjunct treatment.

Ref: Dixit S, DiFiori JP, Burton M, Mines B: Management of patellofemoral pain syndrome. Am Fam Physician
2007;75(2):194-202. 2) Jones BQ, Covey CJ, Sineath MH Jr: Nonsurgical management of knee pain in adults. Am Fam
Physician 2015;92(10):875-883.

2
Item 7

ANSWER: C

In general, all patients with hypertension, regardless of age, benefit from treatment. Patients 80 years of
age who are healthy and functionally independent should be treated according to current recommendations
for patients >65 years old. Available guidelines recommend a target blood pressure of <150/90 mm Hg
unless the patient is frail or has numerous comorbidities. In these cases, the target blood pressure should
be determined on a case-by-case basis (SOR B).

Ref: Benetos A, Rossignol P, Cherubini A, et al: Polypharmacy in the aging patient: Management of hypertension in
octogenarians. JAMA 2015;314(2):170-180.

Item 8

ANSWER: D

The likelihood of azithromycin inducing an arrhythmia is small, but given its widespread use this possibility
must be kept in mind. The arrhythmia results from prolongation of the QT interval and is also more
common in patients with a prior cardiac history. The other three antibiotics have not been implicated in an
increase in cardiac deaths.

Ref: Ray WA, Murray KT, Hall K, et al: Azithromycin and the risk of cardiovascular death. N Engl J Med
2012;366(20):1881-1890.

Item 9

ANSWER: A

This child most likely has transient synovitis, based on the fact that he is well appearing and afebrile, and
his WBC count, erythrocyte sedimentation rate, C-reactive protein level, and radiographs are all normal.
This is a self-limited inflammatory condition, and he will likely respond to NSAIDs such as ibuprofen and
relative rest. Physical therapy and orthopedic consultation are not required, as this condition lasts less than
a week. Septic arthritis would require treatment with intravenous antibiotics, but there is no evidence that
he has this condition.

Ref: Naranje S, Kelly DM, Sawyer JR: A systematic approach to the evaluation of a limping child. Am Fam
Physician2015;92(10):908-916.

3
Item 10

ANSWER: E

Calcium channel blockers such as verapamil or diltiazem can be used acutely to decrease heart rate and
terminate supraventricular tachycardia (SVT) and chronically to prevent SVT recurrence. Adenosine,
amiodarone, esmolol, and procainamide all can be used to treat SVT acutely, but they are not suitable for
long-term therapy.

Ref: Helton MR: Diagnosis and management of common types of supraventricular tachycardia. Am Fam Physician
2015;92(9):793-800.

Item 11

ANSWER: D

This presentation is consistent with $-thalassemia minor trait in a generally asymptomatic patient.
Hemoglobin electrophoresis will be abnormal, with HbA2 increased and HbA decreased. The free T4 level
is likely to be normal in a patient with a normal TSH level. A ferritin level is also likely to be normal given
the normal levels of hemoglobin and hematocrit. A fluorescent spot test is used to screen for G-6-PD
deficiency, which would be associated with bite cells and Heinz bodies. A vitamin B12 level would be useful
for evaluating macrocytosis, which is not present in this patient.

Ref: Wang M: Iron deficiency and other types of anemia in infants and children. Am Fam Physician 2016;93(4):270-278.

Item 12

ANSWER: B

A diagnosis of delirium based on the Confusion Assessment Method (CAM) algorithm requires the presence
of an acute onset and a fluctuating course, inattention, and either an altered level of consciousness or
disorganized thinking. The patient described in this question exhibits an acute onset, fluctuation, inattention,
and an altered level of consciousness.

This patients presentation is more consistent with delirium than encephalitis, as patients with encephalitis
frequently have signs of systemic illness such as fever, lethargy, seizures, and neurologic deficits, as well
as a nonspecific rash in some cases. Furthermore, the fluctuations in the level of consciousness seen in
delirium do not occur with encephalitis. Vascular dementia and Alzheimers disease develop over years,
not acutely as in this case. Stroke, while a consideration and a potential cause of delirium, would not be
the most likely diagnosis in an older patient hospitalized with pneumonia.

Ref: Inouye SK, van Dyck CH, Alessi CA, et al: Clarifying confusion: The confusion assessment method. A new method for
detection of delirium. Ann Intern Med 1990;113(12):941-948. 2) Barr J, Fraser GL, Puntillo K, et al: Clinical practice
guidelines for the management of pain, agitation, and delirium in adult patients in the intensive care unit. Crit Care Med
2013;41(1):263-306. 3) Inouye SK, Westendorp RG, Saczynski JS: Delirium in elderly people. Lancet 2014;383(9920):
911-922.

4
Item 13

ANSWER: D

Amyotrophic lateral sclerosis is a pure motor disorder involving anterior horn cells in the spinal cord and
their brainstem homologues. It may begin as lower motor or bulbar palsy, and the median survival is 35
years. Pseudoclaudication is associated with spinal stenosis, and paresthesias are caused by sensory
problems. A bulls-eye rash is associated with Lyme disease. Charcot joint is seen with conditions that
involve sensory loss, such as diabetes mellitus or syphilis.

Ref: Kasper DL, Fauci AS, Hauser SL, et al (eds): Harrisons Principles of Internal Medicine, ed 19. McGraw-Hill, 2015, pp
2631-2635.

Item 14

ANSWER: A

Causes of pleuritic chest pain include pneumonia, chest wall trauma, pulmonary embolus, and vasculitis.
If these conditions are deemed unlikely based on the history, physical examination, and limited laboratory
studies, a chest radiograph is obtained. If this is within normal limits then viral pleuritic pain is most likely,
and can be treated with an NSAID. Given that the history and physical findings are not suspicious for
thromboembolism and a D-dimer is negative, anticoagulation is inappropriate. With no other systemic
symptoms or findings of collagen-vascular disease, corticosteroids are not indicated. Since there is no rib
tenderness and no radiographic findings of an acute rib fracture, a rib belt is not indicated.

Ref: Kass SM, Williams PM, Reamy BV: Pleurisy. Am Fam Physician 2007;75(9):1357-1364. 2) Marx JA, Hockberger RS,
Walls RM (eds): Rosens Emergency Medicine: Concepts and Clinical Practice, ed 8. Elsevier Saunders, 2014, pp 992-996.

Item 15

ANSWER: C

Methadone has a widely variable half-life (772 hours) and bioavailability, as well as an inactive sedating
metabolite. It is metabolized and cleared by the liver, and should therefore not be used in patients with
severe hepatic impairment. It is, however, a reasonable option for patients with severe renal impairment
(SOR C). Morphine, oxycodone, fentanyl, and hydromorphone can be used cautiously in patients with
hepatic impairment, with reductions in the initial dosage and close monitoring (SOR C).

Ref: Nicholson AB: Methadone for cancer pain. Cochrane Database Syst Rev 2007;(4):CD003971. 2) Groninger H, Vijayan
J: Pharmacologic management of pain at the end of life. Am Fam Physician 2014;90(1):26-32.

5
Item 16

ANSWER: D

This patient is obese and has an isolated elevation of ALT. The initial history and workup, including testing
for chronic hepatitis, a medication review, and alcohol screening, have all been conducted. Current
guidelines suggest iron studies to screen for hemochromatosis as part of the initial evaluation (SOR C).
Antismooth-muscle antibody and ceruloplasmin levels can detect more rare causes of hepatocyte
inflammation (autoimmune hepatitis and copper deposition from Wilsons disease). These tests can be
considered in persistent transaminitis based on clinical judgment (SOR C). Hepatitis D is a superinfection
seen in patients with hepatitis B, and is not a consideration in this patient. A liver biopsy may also be
considered in persistent transaminitis, especially in the setting of clinical or serologic evidence of advanced
liver fibrosis, but is not recommended at this stage (SOR C). Patients with fatty liver disease are at
increased risk for developing liver cirrhosis.

Ref: Chalasani N, Younossi Z, Lavine JE, et al: The diagnosis and management of non-alcoholic fatty liver disease: Practice
guideline by the American Association for the Study of Liver Diseases, American College of Gastroenterology, and the
American Gastroenterological Association. Hepatology 2012;55(6):2005-2023. 2) Grover M, Rutkowski R, Nashelsky
J: FPINs Clinical Inquiries: Evaluation of elevated serum transaminase levels. Am Fam Physician 2012;86(8):1-2.

Item 17

ANSWER: A

Cats that are infected with Toxoplasma gondii can shed the virus for weeks when newly infected. Cleaning
the cats litter box is a high-risk activity, and transmission to the fetus can occur. Toxocara cati infection
is less common and usually results from ingesting contaminated cat meat or soil containing cat feces.
Chagas disease is spread by insect bites. Cysticercosis is generally acquired by eating contaminated pork.
Aspergillosis usually occurs in immunocompromised people who inhale the spores found in soil (SOR A).

Ref: Woodhall D, Jones JL, Cantey PT, et al: Neglected parasitic infections: What every family physician needs to know. Am
Fam Physician 2014;89(10):803-811.

Item 18

ANSWER: A

A review of multiple studies has revealed that continuous daily macrolide antibiotic use for 1 year decreased
the number of exacerbations in older patients with COPD (SOR A). Quality of life did not show
improvement, however, and resistance to macrolide and quinolone antibiotics rose. There was no increase
in conduction abnormalities or tinnitus, but the incidence of hearing loss increased with the use of
azithromycin.

Ref: Herath SC, Poole P: Prophylactic antibiotic therapy in chronic obstructive pulmonary disease. JAMA
2014;311(21):2225-2226.

6
Item 19

ANSWER: C

Patients with new health insurance plans or policies purchased under the Patient Protection and Affordable
Care Act are expected to be covered for certain services. Preventive services with a U.S. Preventive
Services Task Force grade of A or B, such as colorectal cancer screening, are covered without cost-sharing
when performed by an in-network provider. Although coverage for vaccines such as diphtheria, tetanus,
and influenza is required with these plans, those that are needed only for travel are not covered. In addition,
although consumers can purchase vision and dental coverage in addition to health coverage through the
marketplace, they are not a required part of coverage. While coverage for obesity counseling is required,
coverage for bariatric surgery still varies by state.

Ref: US Preventive Services Task Force: Guide to Clinical Preventive Services 2014. Agency for Healthcare Research and
Quality, 2014. 2) National Conference of State Legislatures. Health reform and health mandates for obesity. 2016. 3)
Health benefits and coverage: What marketplace health insurance plans cover. US Centers for Medicare and Medicaid
Services, HealthCare.gov.

Item 20

ANSWER: C

The transjugular intrahepatic portosystemic shunt (TIPS) procedure, by shunting blood destined for the liver
into the systemic circulation, lowers pressure in the portal veins, thereby decreasing portal system
hypertension and making variceal bleeding and portal hypertensive gastropathy less likely. TIPS may
decrease the likelihood of variceal bleeding by as much as 90%. TIPS also reduces the pressure that leads
to ascites and lower extremity edema, or the massive edema of anasarca. The diversion of blood from the
liver circulation compromises the livers role in removing toxins, including the serum marker ammonia.
This may make patients more vulnerable to episodes of hepatic encephalopathy.

Ref: Fidelman N, Kwan SW, LaBerge JM, et al: The transjugular intrahepatic portosystemic shunt: An update. AJR Am J
Roentgenol 2012;199(4):746-755. 2) Goldman L, Schafer AI (eds): Goldmans Cecil Medicine, ed 25. Elsevier Saunders,
2016, pp 1023-1031.

Item 21

ANSWER: A

In elderly patients, detrusor instability is the most common cause of urinary incontinence in both men and
women. Incontinence may actually become worse after surgical relief of obstructive prostatic hypertrophy.

Infection is unlikely to be the cause of persistent incontinence in this patient in the absence of fever or
symptoms of urinary tract infection. Overflow is unlikely in the absence of residual urine. Impaction is a
relatively rare cause of urinary incontinence, and associated findings would be present on the rectal
examination. Normalization of the urinary stream and the absence of residual urine reduce the likelihood
of recurrent obstruction. The prostate would be expected to remain enlarged on rectal examination after
transurethral resection of the prostate (TURP).

7
Ref: Gibbs CF, Johnson TM II, Ouslander JG: Office management of geriatric urinary incontinence. Am J Med
2007;120(3):211-220. 2) Khandelwal C, Kistler C: Diagnosis of urinary incontinence. Am Fam Physician
2013;87(8):543-50.

Item 22

ANSWER: A

Individuals with impaired fasting glucose or impaired glucose tolerance have a higher risk of developing
diabetes mellitus in the future. Randomized, controlled trials have shown that taking metformin,
"-glucosidase inhibitors (acarbose), orlistat, or thiazolidinediones significantly reduces the risk of
developing diabetes mellitus. The U.S. Diabetes Prevention Program Outcomes Study demonstrated a 34%
reduction in the development of diabetes mellitus at 10 years. In addition to medications, it is also
recommended that patients be counseled about weight loss and engaging in moderate physical activity for
at least 150 minutes per week. The other medications listed are indicated for the treatment of diabetes but
have not been shown to be effective for prevention.

Ref: Chiasson JL, Josse RG, Gomis R, et al; STOP-NIDDM Trial Research Group: Acarbose treatment and the risk of
cardiovascular disease and hypertension in patients with impaired glucose tolerance: The STOP-NIDDM Trial. JAMA
2003;290(4):486-494. 2) American Diabetes Association: Standards of Medical Care in Diabetes2016. 4. Prevention
or delay of type 2 diabetes. Diabetes Care 2016;39(Suppl 1):S36-S38.

Item 23

ANSWER: A

Desloratadine, fexofenadine, levocetirizine, and loratadine are among the second-generation antihistamines
that have a better adverse effect profile and cause less sedation than first-generation antihistamines (SOR
A). Second-generation antihistamines have more complex chemical structures that decrease their movement
across the blood-brain barrier, reducing central nervous system adverse effects such as sedation. Although
cetirizine is generally classified as a second-generation antihistamine and a more potent histamine
antagonist, it is known to cause sedation (SOR A).

Ref: Bender BG, Berning S, Dudden R, et al: Sedation and performance impairment of diphenhydramine and second-generation
antihistamines: A meta-analysis. J Allergy Clin Immunol 2003;111(4):770-776. 2) Sur DK, Plesa ML: Treatment of
allergic rhinitis. Am Fam Physician 2015;92(11):985-992.

Item 24

ANSWER: C

Trismus is almost universally present with peritonsillar abscess, while voice changes, otalgia, and
odynophagia may or may not be present. Pharyngotonsillitis and peritonsillar cellulitis may also be
associated with these complaints. Otalgia is common with peritonsillar abscess, otitis media,
temporomandibular joint disorders, and a variety of other conditions. Peritonsillar abscess is rarely found
in patients who do not have at least a 3-day history of progressive sore throat.

Ref: Galioto NJ: Peritonsillar abscess. Am Fam Physician 2008;77(2):199-202. 2) Kasper DL, Fauci AS, Hauser SL, et al
(eds): Harrisons Principles of Internal Medicine, ed 19. McGraw-Hill, 2015, p 235.

8
Item 25

ANSWER: D

There are estimated to be 1.63.8 million sports-related concussions each year in the United States, and
appropriate management can prevent many of the long-term complications. In this scenario, the patient
meets the criteria for a concussion, with headache, nausea, and imbalance following an injury. She was
appropriately removed from the game. The next step in management is complete physical and cognitive rest
until her symptoms resolve without medication (SOR C). Complete physical rest includes avoidance of any
physical activity that exacerbates symptoms. Cognitive rest means avoidance of activities that require
concentration or attention, such as schoolwork and electronics use.

Once the patients symptoms have resolved without medication, she should be reevaluated and can start a
graded return-to-play protocol (SOR C). Reevaluation may include repeat neurocognitive testing and
postural stability testing (SOR C). Both types of testing can be important in the evaluation but there is
insufficient evidence that either affects outcomes.

In the initial evaluation of concussion, imaging is not indicated in the absence of focal neurologic findings,
loss of consciousness for more than 60 seconds, or evidence of skull fracture (SOR C). If imaging criteria
are met, CT of the head is indicated. Plain films are not indicated in the evaluation.

There is no evidence that protective gear reduces the incidence of concussion (SOR C). Even though some
protective gear can prevent other types of injuries, it is not indicated for concussion prevention. Eliminating
dangerous behaviors such as heading the ball in soccer can help, however.

Ref: Scorza KA, Raleigh MF, OConnor FG: Current concepts in concussion: Evaluation and management. Am Fam Physician
2012;85(2):123-132. 2) Armstrong C: Evaluation and management of concussion in athletes: Recommendations from the
AAN. Am Fam Physician 2014;89(7):585-587.

Item 26

ANSWER: C

The 2013 U.S. Preventive Services Task Force guidelines recommend annual lung cancer screening with
low-dose CT for asymptomatic adults 5580 years of age who have a 30-pack-year smoking history and
currently smoke or have quit smoking within the past 15 years (grade B recommendation). Screening should
be discontinued when the patient has not smoked for 15 years or develops a health problem that
substantially limits life expectancy or the ability or willingness to have curative lung surgery.

Ref: Final Recommendation Statement: Lung Cancer Screening. US Preventive Services Task Force, 2014.

9
Item 27

ANSWER: A

The fetal fibronectin test is used to help determine the risk of early delivery. Unfortunately, the positive
predictive value for delivery within the next week is <20% in patients who have clinical indications of
preterm labor. In a patient who does not have any features of preterm labor, it does not affect management.
A negative result is reassuring, on the other hand, because the negative predictive value is >95%. In
studies that tested gravidas every week, positive results had a mild correlation with preterm delivery, but
not enough to guide management. False-positive test results can be caused by maternal blood or amniotic
fluid, but fetal fibronectin does not indicate rupture of the membranes when other tests are negative.

Ref: Lowe MP, Zimmerman B, Hansen W: Prospective randomized controlled trial of fetal fibronectin on preterm labor
management in a tertiary care center. Am J Obstet Gynecol 2004;190(2):358-362. 2) Fetal fibronectin does not affect
outcomes of preterm labor. J Fam Pract 2004;53(6):442. 3) Sayres WG: Preterm labor. Am Fam Physician
2010;81(4):477-84. 4) Abbott DS, Hezelgrave NL, Seed PT, et al: Quantitative fetal fibronectin to predict preterm birth
in asymptomatic women at high risk. Obstet Gynecol 2015;125(5):1168-1176.

Item 28

ANSWER: D

Fifth disease, caused by parvovirus B19, is a common childhood infection that typically presents with a
prodrome of low-grade fever, malaise, sore throat, nausea, and headache, followed a few days later by an
erythematous facial rash on the cheeks. The rash fades in a few days but a lacy, reticular rash develops on
the extremities. Once the rash appears, patients are no longer considered contagious. NSAIDs and
antihistamines are often used for symptom relief (SOR A).

Ref: Allmon A, Deane K, Martin KL: Common skin rashes in children. Am Fam Physician 2015;92(3):211-216.

Item 29

ANSWER: B

Idarucizumab has been approved for the urgent reversal of the anticoagulant effect of dabigatran. The dose
is two consecutive 2.5-g infusions and the cost is $3500. Apixaban, edoxaban, and rivaroxabanall factor
Xa inhibitorsare not affected by this medication and they do not currently have a reversal agent.

Ref: Idarucizumab (Praxbind)An antidote for dabigatran. Med Lett Drugs Ther 2015;57(1482):157-158.

10
Item 30

ANSWER: B

The American Academy of Family Physicians and the American College of Obstetricians and Gynecologists
collaborated on the Choosing Wisely campaign recommendation regarding scheduled inductions of labor
or cesarean deliveries. These organizations noted that delivery prior to 39 weeks, 0 days is associated with
an increased risk of learning disabilities and may increase morbidity and mortality. While delivery prior
to 39 weeks and 0 days may be indicated in certain situations, a mature fetal lung test by itself is not an
indication for delivery (SOR C).

Ref: Main E, Oshiro B, Chagolla B, et al; (California Maternal Quality Care Collaborative): Elimination of non-medically
indicated (elective) deliveries before 39 weeks gestational age. California: March of Dimes; 2010. 2) American Academy
of Family Physicians: Fifteen things physicians and patients should question. ABIM Foundation, Choosing Wisely
campaign, 2013. 3) Quinlan JD, Murphy NJ: Cesarean delivery: Counseling issues and complication management. Am
Fam Physician2015;91(3):178-184. 4) American College of Obstetricians and Gynecologists: Ten things physicians and
patients should question. ABIM Foundation, Choosing Wisely campaign, 2016.

Item 31

ANSWER: B

Benzodiazepines, such as chlordiazepoxide, can prevent alcohol withdrawal seizures. Anticonvulsants such
as carbamazepine, gabapentin, and phenytoin have less abuse potential than benzodiazepines but do not
prevent seizures. Clonidine, an "-adrenergic agonist, reduces the adrenergic symptoms associated with
withdrawal but does not prevent seizures.

Ref: Muncie HL Jr, Yasinian Y, Oge L: Outpatient management of alcohol withdrawal syndrome. Am Fam Physician
2013;88(9):589-595.

Item 32

ANSWER: C

This patients pulmonary function test (PFT) findings are consistent with a restrictive defect with a low
DLCO. Idiopathic pulmonary fibrosis, asbestosis, hypersensitivity pneumonitis, and sarcoidosis are
restrictive lung diseases with a low diffusion capacity due to alveolar damage. COPD and asthma are both
obstructive lung diseases, but can be associated with an abnormal DLCO. Morbid obesity causes a
restrictive lung disease associated with a normal DLCO. The presence of chronic pulmonary emboli is
associated with a low DLCO due to pulmonary vascular disease, but PFTs are normal.

Ref: Johnson JD, Theurer WM: A stepwise approach to the interpretation of pulmonary function tests. Am Fam Physician
2014;89(5):359-366.

11
Item 33

ANSWER: B

An intranasal corticosteroid alone should be the initial treatment for allergic rhinitis with symptoms
affecting quality of life (SOR A). Intranasal corticosteroids act by decreasing the influx of inflammatory
cells and inhibiting the release of cytokines, thereby reducing inflammation of the nasal mucosa. Intranasal
corticosteroids are more effective than oral and intranasal antihistamines in the treatment of persistent or
more severe allergic rhinitis (SOR A). Intranasal antihistamines also have more adverse effects than
intranasal corticosteroids (SOR C). Subcutaneous and sublingual immunotherapy are not considered
first-line treatments but should be considered for moderate or severe persistent allergic rhinitis that is not
responsive to usual treatments (SOR A).

Ref: Price D, Bond C, Bouchard J, et al: International Primary Care Respiratory Group (IPCRG) guidelines: Management of
allergic rhinitis. Prim Care Respir J 2006;15(1):58-70. 2) Sur DK, Plesa ML: Treatment of allergic rhinitis. Am Fam
Physician 2015;92(11):985-992.

Item 34

ANSWER: B

Acute pericarditis is a common cause of chest pain in young adults. It presents with an acute to subacute
onset of severe retrosternal and left precordial chest pain that is characteristically alleviated by leaning
forward or sitting up and worsened with lying supine. Viral and idiopathic acute pericarditis are most
common and occur at all ages, but are more common in young adults. The pain, which is inflammatory in
origin, typically responds to an NSAID such as indomethacin. High-dose aspirin (24 g/day) has also been
shown to be effective. Patients with pain resulting from myocardial ischemia would benefit from
nitroglycerin, while those with gastroesophageal reflux disease would benefit from a proton pump inhibitor
such as omeprazole. Hydrocodone can be used to relieve moderate to severe pain but lacks
anti-inflammatory properties (SOR C).

Ref: Snyder MJ, Bepko J, White M: Acute pericarditis: Diagnosis and management. Am Fam Physician 2014;89(7):553-560.
2) Kasper DL, Fauci AS, Hauser SL, et al (eds): Harrisons Principles of Internal Medicine, ed 19. McGraw-Hill, 2015,
pp 1571-1573.

Item 35

ANSWER: D

The differential diagnosis of acute nonlocalized abdominal pain is broad. CT is typically the imaging
modality of choice if there is significant concern about serious pathology or if the diagnosis is unclear from
the history, physical examination, and laboratory testing. In this instance, the American College of
Radiology recommends CT of the abdomen and pelvis, preferably with contrast. One prospective study of
patients with nontraumatic abdominal pain in an emergency department setting found that CT results
changed the leading diagnosis in 49% of patients and the management plan in 42% of patients.

Ref: Cartwright SL, Knudson MP: Diagnostic imaging of acute abdominal pain in adults. Am Fam Physician
2015;91(7):452-459.

12
Item 36

ANSWER: A

Family physicians are faced with many treatment options for managing chronic low back pain. Both
supervised walking and physical therapy resulted in decreased disability and pain scores at 6 months in a
2015 randomized, controlled trial. A walking program is the best choice because it is much more affordable
than physical therapy. Cochrane reviews found that neither epidural corticosteroid injections nor
transcutaneous electrical nerve stimulation were better than placebo, and spinal manipulation showed no
benefit after 6 months. Spinal cord stimulators may help patients with severe pain after failed back surgery,
but lack moderate or better evidence to support their widespread use.

Ref: Khadilkar A, Odebiyi DO, Brosseau L, Wells GA: Transcutaneous electrical nerve stimulation (TENS) versus placebo for
chronic low-back pain. Cochrane Database Syst Rev 2008;(4):CD003008. 2) Kwan G, Balady GJ: Cardiac rehabilitation
2012: Advancing the field through emerging science. Circulation 2012;125(7):e369-e373. 3) Hurley DA, Tully MA,
Lonsdale C, et al: Supervised walking in comparison with fitness training for chronic back pain in physiotherapy: Results
of the SWIFT single-blinded randomized controlled trial (ISRCTN17592092). Pain 2015;156(1):131-147. 4) Herndon CM,
Zoberi KS, Gardner BJ: Common questions about chronic low back pain. Am Fam Physician 2015;91(10):708-714.

Item 37

ANSWER: A

Agranulocytosis occurs in about 1 in 500 patients who are taking methimazole. Prior to starting therapy
the patient should be advised to be alert for a fever and severe sore throat. A recent survey showed that
61% of patients were unaware of potential adverse reactions. Another serious reaction is hepatotoxicity.

Ref: Burch HB, Cooper DS: Management of Graves disease: A review. JAMA 2015;314(23):2544-2554.

Item 38

ANSWER: C

Medications are effective for alcohol use disorder and should be offered in conjunction with psychosocial
interventions. Oral naltrexone is the most effective medication to prevent relapse. Acamprosate is
moderately effective but is contraindicated in patients with renal disease. A recent meta-analysis indicated
that disulfiram is not effective for preventing relapse. Bupropion is used for smoking cessation.

Ref: Jonas DE, Amick HR, Feltner C, et al: Pharmacotherapy for adults with alcohol use disorders in outpatient settings: A
systematic review and meta-analysis. JAMA 2014;311(18):1889-1900. 2) Centers for Disease Control and Prevention. Fact
sheetsAlcohol use and your health. 3) Hendry S, Mounsey A: PURLS: Consider these medications to help patients stay
sober. J Fam Pract 2015;64(4):238-240.

13
Item 39

ANSWER: D

Geriatric patients are at greater risk of adverse drug reactions compared to their younger counterparts.
Guidelines recommend several antidepressant agents as good first-line options, including venlafaxine,
bupropion, and mirtazapine. Among the SSRIs, citalopram, escitalopram, and sertraline are all good
choices, but paroxetine is associated with more anticholinergic effects and should be avoided (SOR C).

Ref: Kovich H, DeJong A: Common questions about the pharmacologic management of depression in adults. Am Fam Physician
2015;92(2):94-100.

Item 40

ANSWER: A

Since early intensive behavioral therapy can improve functional outcomes for children with autism and
autism spectrum disorders, early diagnosis is critical. The American Academy of Pediatrics recommends
screening all children with an autism-specific screening instrument at 18 and 24 months in addition to
surveillance of developmental issues at all well child visits.

Ref: Carbone PS, Farley M, Davis T: Primary care for children with autism. Am Fam Physician 2010;81(4):453-460. 2)
Zwaigenbaum L, Bauman ML, Fein D, et al: Early screening of autism spectrum disorder: Recommendations for practice
and research. Pediatrics 2015;136(Suppl 1):S41-59.

Item 41

ANSWER: E

Women between the ages of 21 and 29 at average risk for cervical cancer should be screened with cytology
every 3 years (USPSTF A recommendation). Because of the high prevalence of HPV infection in this age
group and because there are no clear benefits to HPV testing, testing is not recommended (USPSTF grade
D). However, most clinicians will order reflex testing for high-risk HPV types if the Papanicolaou smear
shows atypical squamous cells of uncertain significance (ASCUS), based on a recommendation by the
American Society for Colposcopy and Cervical Pathology.

Women 3065 years of age at average risk for cervical cancer may be screened with cytology with HPV
cotesting every 5 years or with cytology alone every 3 years (USPSTF grade A). HPV DNA typing and
colposcopy are not screening tests and are used for further evaluation of cytologic abnormalities. HPV
antibody results have no role in screening. Women with a history of HPV immunization should continue
to be screened according to usual guidelines.

Ref: Final Recommendation Statement: Cervical Cancer: Screening. US Preventive Services Task Force, 2012. 2) Saslow D,
Solomon D, Lawson HW, et al: American Cancer Society, American Society for Colposcopy and Cervical Pathology, and
American Society for Clinical Pathology screening guidelines for the prevention and early detection of cervical cancer. J
Low Genit Tract Dis 2012;16(3):175-204. 3) ACOG releases guideline on cervical cancer screening. Am Fam Physician
2013;88(11):776-777. 4) Lambert M: ACP releases best practice advice on screening for cervical cancer. Am Fam
Physician 2015;92(12):1107-1110.

14
Item 42

ANSWER: D

Third degree atrioventricular (AV) block is characterized by the inability of atrial impulses to reach the
ventricles. Hence, the EKG will show both atrial activity (P waves) and ventricular escape activity (QRS
complexes) with no correlation between them. Hemodynamically unstable patients with third degree AV
block should be emergently treated with atropine and temporary pacing. With Mobitz type I AV block there
is progressive PR interval prolongation preceding a nonconducted P wave. With Mobitz type II AV block
the PR interval remains the same but there is a sudden dropped P wave. Second and third degree AV blocks
are seen in patients with advanced heart disease. The EKG in atrial fibrillation lacks distinct P waves.
Additionally, fibrillary waves and irregularly irregular intervals between QRS complexes are characteristic
(SOR C).

Ref: Link MS: Evaluation and initial treatment of supraventricular tachycardia. N Engl J Med 2012;367(15):1438-1448. 2)
Goldberger AL, Goldberger ZD, Shvilkin A: Goldbergers Clinical Electrocardiography: A Simplified Approach, ed 8.
Elsevier Saunders, 2012.

Item 43

ANSWER: C

Neurologic complications related to Pagets disease of the bone can result from bony compression of the
cranial nerves, spinal cord, and/or nerve roots, resulting in pain or loss of function, or from the creation
of a vascular steal of blood supplying CNS structures, which causes paralysis that is often reversible. In
rare cases it is caused by basilar invagination of the C2 vertebra compressing the brain stem and leading
to hydrocephalus. Hearing loss, the most common neurologic complication of Pagets disease, was initially
believed to be caused by compression of the vestibulocochlear nerve but is now thought to be the result of
cochlear damage.

Ref: Singer FR, Bone HG 3rd, Hosking DJ, et al: Pagets disease of bone: An Endocrine Society clinical practice guideline. J
Clin Endocrinol Metab 2014;99(12):44084422.

Item 44

ANSWER: C

This patients presentation and mechanism of injury suggest radial head subluxation, or nursemaids
elbow. This is a clinical diagnosis that usually does not require imaging prior to attempted reduction,
either by hyperpronation or the supination and flexion method. There is no evidence of a fracture requiring
immobilization of the joint or the arm, but the injury does require treatment.

Ref: Fleisher GR, Ludwig S (eds): Textbook of Pediatric Emergency Medicine, ed 6. Lippincott Williams & Wilkins, 2010,
pp 1832-1833. 2) Tintinalli JE, Kelen GD, Stapczynski JS (eds): Emergency Medicine: A Comprehensive Study Guide,
ed 7. McGraw-Hill, 2011, p 983.

15
Item 45

ANSWER: B

The most common cardiac abnormality associated with Lyme disease is fluctuating degrees of
atrioventricular (AV) block. This may be first degree, second degree, or complete heart block. The
condition is usually temporary, lasting from 3 to 42 days, with more advanced block lasting longer than
first degree AV block. The degree of AV block can fluctuate within minutes. The highest risk of AV block
is in patients with a PR interval >300 msec. Other cardiac abnormalities such as myocarditis, left
ventricular failure, and pericarditis are less common.

Ref: Kasper DL, Fauci AS, Hauser SL, et al (eds): Harrisons Principles of Internal Medicine, ed 19. McGraw-Hill, 2015, pp
1150-1151.

Item 46

ANSWER: E

This patients test results suggest a moderate anemia that is associated with a mild reduction of platelets.
The mean corpuscular volume is solidly in the normal range, so this is a normocytic anemia and is unlikely
to be due to iron or folate deficiency, which usually result in microcytic and macrocytic anemias,
respectively. A normal ferritin level also suggests a diagnosis other than iron deficiency.

Acute blood loss and hemolysis are two common causes of normocytic anemia. However, with both of
these conditions a high reticulocyte count would be expected. The fact that the reticulocyte count is in the
normal range despite significant anemia suggests that this patient has decreased bone marrow production
of red blood cells, which is at least contributing to his anemia. Bone marrow response to anemia is often
evaluated by using the reticulocyte index, which is calculated by multiplying the measured reticulocyte
percentage (1.5 in this patient) by the ratio of the patients hematocrit to a normal hemotocrit based on the
persons age and sex (26.9 divided by 45 in this case). If the patient has a normal bone marrow, the
reticulocyte index should be 2%3% or higher. In this case the result is 0.9%, suggesting a profound
suppression of bone marrow.

Ref: Hoffman R, Benz EJ, Silberstein LE, et al: Hematology: Basic Principles and Practice, ed 6. Elsevier Inc, 2013, pp
418-426.

Item 47

ANSWER: E

This patient is more likely to have bacterial vaginosis than other causes of vaginitis, based on the presence
of a foul-smelling discharge in the absence of pruritus or dyspareunia. The odor of bacterial vaginitis is
more likely to appear in the presence of semen because of the increase in vaginal alkalinity. Bacterial
vaginosis is associated with a vaginal pH >4.5. Vulvovaginal candidiasis is associated with vulvar
erythema, excoriation of the vulva, and a normal vaginal pH (4.04.5). Vaginal atrophy is seen in women
with estrogen deficiency, and atrophic vaginitis is unlikely in this patient due to her age and lack of pruritus
and dyspareunia.

16
Ref: ACOG Practice Bulletin. Clinical management guidelines for obstetrician-gynecologists, number 72, May 2006: Vaginitis.
Obstet Gynecol 2006;107(5):1195-1206. 2) Hainer BL, Gibson MV: Vaginitis: Diagnosis and treatment. Am Fam
Physician2011;83(7):807-815.

Item 48

ANSWER: C

Pregnant women should receive a dose of Tdap, optimally between 27 and 36 weeks of gestation, regardless
of when they last received it. MMR vaccine and varicella vaccine are both live attenuated vaccines and
should not be given to women known to be pregnant. Influenza vaccine is recommended for all women who
are or will be pregnant during influenza season, but pregnant women should receive the inactivated
influenza vaccine.

Ref: National Center for Immunization and Respiratory Diseases: General recommendations on
immunizationRecommendations of the Advisory Committee on Immunization Practices (ACIP). MMWR Recomm Rep
2011;60(2):1-64. 2) Centers for Disease Control and Prevention: Guidelines for Vaccinating Pregnant Women. 2013.

Item 49

ANSWER: E

According to a Cochrane review, routine preoperative testing prior to cataract surgery does not decrease
intraoperative or postoperative complications (SOR A). The American Heart Association recommends
against routine preoperative testing in asymptomatic patients undergoing low-risk procedures, since the
cardiac risk associated with such procedures is less than 1%.

Ref: Keay L, Lindsley K, Teisch J, et al: Routine preoperative medical testing for cataract surgery. Cochrane Database Syst Rev
2012;(3):CD007293.

Item 50

ANSWER: D

This patient has gout, based on his history and examination. NSAIDs, colchicine, or corticosteroids may
be used as first-line treatment of gout (SOR B). The best initial treatment in this patient would be
prednisone, due to his advanced chronic kidney disease. Allopurinol may be a good choice to prevent future
episodes if his uric acid level is elevated, but should not be started as an acute treatment. Probenecid would
also not be an acute treatment but can be used in some patients for prevention of future attacks.

Ref: Hainer BL, Matheson E, Wilkes RT: Diagnosis, treatment, and prevention of gout. Am Fam Physician
2014;90(12):831-836.

17
Item 51

ANSWER: B

Severe hyponatremia with symptoms of confusion and seizures requires raising the serum sodium level until
symptoms improve. Symptomatic hyponatremia occurs when sodium levels decrease over less than 24
hours. Once symptoms resolve, the cause should be determined. The rate of sodium correction should be
612 mEq/L in the first 24 hours and 18 mEq/L or less in the first 48 hours. An increase of 46 mEq/L
is usually sufficient to reduce symptoms of acute hyponatremia. Rapid correction of sodium levels can
result in osmotic demyelination (previously called central pontine myelinolysis).

Infusion of normal saline or 0.45% saline will not correct the sodium as rapidly as 3% saline in acute,
severe hyponatremia. Desmopressin, 12 :g every 46 hours, can be used concurrently with 3% saline.

Ref: Braun MM, Barstow CH, Pyzocha NJ: Diagnosis and management of sodium disorders: Hyponatremia and hypernatremia.
Am Fam Physician 2015;91(5):299-307.

Item 52

ANSWER: C

After cleansing with saline or tap water, application of a moist, nonadhesive bandage is the preferred way
to manage a pressure ulcer. A moist wound environment assists in healing and aids in autolytic
debridement. Wet-to-dry dressings may impede healing by causing pain and unnecessary debridement when
a fully dry dressing is removed. Chlorine-based and povidone-iodine solutions should be avoided because
they may impede granulation tissue formation. While it is important to debride necrotic tissue and slough,
a dry, intact eschar over the heels without any sign of infection should be left in place as a natural biologic
cover.

Ref: Raetz JG, Wick KH: Common questions about pressure ulcers. Am Fam Physician 2015;92(10):888-894.

Item 53

ANSWER: B

Anticoagulant medications have been consistently identified as the most common cause of adverse drug
events across health care settings in the United States. The top three categories responsible for adverse drug
events are anticoagulants, opioids, and diabetic agents.

Ref: US Department of Health and Human Services Office of Disease Prevention and Health Promotion: National Action Plan
for Adverse Drug Event Prevention, 2014. 2) Harris Y, Hu DJ, Lee C, et al: Advancing medication safety: Establishing
a national action plan for adverse drug event prevention. Jt Comm J Qual Patient Saf 2015;41(8):351-360.

18
Item 54

ANSWER: C

Shoulder pain is the third most common musculoskeletal reason patients consult primary care physicians,
and rotator cuff disease is the most common cause of shoulder pain. Pain and restricted active and passive
range of motion, accompanied by pain and joint stiffness, are diagnostic of adhesive capsulitis. Range of
motion would not likely be affected with a partial rotator cuff tear, subdeltoid bursitis, active myositis, or
osteoarthritis. Limitations that occur only with active motion suggest impairment of rotator cuff muscles.

Ref: Hermans J, Luime JL, Meuffels DE, et al: Does this patient with shoulder pain have rotator cuff disease?: The Rational
Clinical Examination Systematic Review. JAMA 2013;310(8):837-847.

Item 55

ANSWER: A

An incidentally discovered adrenal mass is a common finding on abdominal CT and MRI, occurring in
approximately 3%4% of scans. However, only about 1% of these are malignant, and malignancies rarely
occur in lesions <5 cm in size. Metastatic lesions are rare in patients without a history of cancer. An
incidentally discovered adrenal mass 14 cm in size is most likely to be a benign adenoma, although
follow-up is often indicated to ensure stability of the lesion.

Ref: Willatt JM, Francis IR: Radiologic evaluation of incidentally discovered adrenal masses. Am Fam Physician
2010;81(11):1361-1366. 2) Hitzeman N, Cotton E: Incidentalomas: Initial management. Am Fam Physician
2014;90(11):784-789.

Item 56

ANSWER: C

There are multiple risk factors for uncomplicated cases of cystitis. Sexual intercourse is the most common.
Others include spermicide use, previous urinary tract infection, a new sex partner, and a family history of
urinary tract infections in a first degree female relative.

Various studies have shown no relationship between cystitis and water consumption, urinating after
intercourse, patterns of wiping after urination, use of hot tubs, type of underwear, or obesity.

Ref: Hooton TM: Uncomplicated urinary tract infection. N Engl J Med 2012;366(11):1028-1037.

19
Item 57

ANSWER: D

Infants less than 12 months of age have higher rates of pertussis infection and have the largest proportion
of pertussis-related deaths. The majority of pertussis cases, admissions, and deaths occur in children under
2 months of age before they receive their first vaccines. The Advisory Committee on Immunization
Practices recommends that all unvaccinated family members get a dose of Tdap to help protect infants from
pertussis. Both 13-valent and 23-valent pneumococcal vaccine are indicated for someone over 65 years of
age, but there is not a demonstrated benefit for the health of the infant in this case. Meningococcal and
varicella vaccines are not routinely given to adults over age 65 and have not been shown to help protect the
infants they have contact with.

Ref: Centers for Disease Control and Prevention (CDC): Updated recommendations for use of tetanus toxoid, reduced diphtheria
toxoid and acellular pertussis vaccine (Tdap) in pregnant women and persons who have or anticipate having close contact
with an infant aged <12 monthsAdvisory Committee on Immunization Practices (ACIP), 2011. MMWR Morb Mortal
Wkly Rep 2011;60(41):1424-1426.

Item 58

ANSWER: E

The U.S. Preventive Services Task Force recommends against screening for asymptomatic carotid artery
stenosis (grade D recommendation), citing with moderate certainty that risks outweigh benefits. Although
carotid artery stenosis is a risk factor for stroke, which is a major cause of death and disability, screening
tests were not found to improve patient outcomes.

Asymptomatic carotid artery stenosis has a low prevalence (0.5%1%) and carotid ultrasonography has a
high rate of false-positives, exposing patients to harm from unnecessary treatment. Surgical treatments for
carotid artery stenosis have a 30-day risk of stroke and mortality of 2.2%3.8%. Carotid auscultation has
not been found to be accurate or beneficial, and screening has not been shown to help optimize medical
therapy.

Ref: Final Recommendation Statement: Carotid Artery Stenosis Screening. US Preventive Services Task Force, 2014.

Item 59

ANSWER: B

In spite of good evidence that antibiotics are ineffective for the treatment of acute bronchitis, and that 90%
of cases are caused by viruses, rates of antibiotic prescription for acute bronchitis remain in the 60%80%
range. Several strategies have been shown to reduce the rate of antibiotic prescribing for this condition.
These include careful use of nonmedical terminology such as referring to the problem as a chest cold,
providing pocket prescriptions with advice to fill the prescription only if the patient does not improve
in a defined period of time, and educating patients about the natural history of bronchitis, informing them
that symptoms may persist for 3 weeks. Specialists are not less likely than primary care physicians to
prescribe antibiotics. Sinus films would not provide evidence to confirm that the infection is viral.

20
Ref: Zoorob R, Sidani MA, Fremont RD, Kihlberg C: Antibiotic use in acute upper respiratory tract infections. Am Fam
Physician 2012;86(9):817-822. 2) Aring AM, Chan MM: Current concepts in adult acute rhinosinusitis. Am Fam Physician
2016;94(2):97-105.

Item 60

ANSWER: D

NSAIDs should be used as first-line treatment for primary dysmenorrhea (SOR A). A Cochrane review that
included 73 randomized, controlled trials demonstrated strong evidence to support NSAIDs as the first-line
treatment for primary dysmenorrhea. Since no NSAID has been proven more effective than others, the
choice of NSAID should be based on effectiveness and tolerability for each patient. The medication should
be taken 12 days before the expected onset of the menstrual period and continued on a fixed schedule for
23 days. Oral contraceptives may be effective for relieving symptoms of primary dysmenorrhea but the
evidence is limited. Hydrocodone, acetaminophen, and medroxyprogesterone acetate are not appropriate
choices.

Ref: Osayande AS, Mehulic S: Diagnosis and initial management of dysmenorrhea. Am Fam Physician 2014;89(5):341-346.

Item 61

ANSWER: D

Less than 1% of patients started on an ACE inhibitor develop angioedema, but some studies have reported
that up to 10% of these patients require intubation. This type of angioedema is due to increased bradykinin
rather than histamine, and antihistamines, anticholinergics, corticosteroids, and epinephrine would not be
effective. Icatibant is a bradykinin receptor type 2 blocker and is recommended in patients with laryngeal
angioedema compromising airway function (level 2 evidence). Angiotensin receptor blockers, although
probably not harmful, would not be helpful.

Ref: Bezalel S, Mahlab-Guri K, Asher I, et al: Angiotensin-converting enzyme inhibitor-induced angioedema. Am J Med
2015;128(2):120-125.

Item 62

ANSWER: E

The euthyroid sick syndrome refers to alterations in thyroid function tests seen frequently in hospitalized
patients, and abnormal thyroid function tests may be seen early in sepsis. These changes are statistically
much more likely to be secondary to the euthyroid sick syndrome than to unrecognized pituitary or
hypothalamic disease (SOR C). Graves disease generally is a hyperthyroid condition associated with low
TSH and elevated free T4. Subclinical hypothyroidism is diagnosed by high TSH and normal free T4 levels.
Subacute thyroiditis most often is a hyperthyroid condition.

Ref: Sakharova OV, Inzucchi SE: Endocrine assessments during critical illness. Crit Care Clin 2007;23(3):467-490. 2) Adler
SM, Wartofsky L: The nonthyroidal illness syndrome. Endocrinol Metab Clin North Am 2007;36(3):657-672. 3) Melmed
S, Polonsky KS, Larsen PR, Kronenberg HM (eds): Williams Textbook of Endocrinology, ed 13. Elsevier Saunders, 2016,
pp 354-355.

21
Item 63

ANSWER: B

Endoscopic screening results in the detection of Barretts esophagus in 6%12% of patients with prolonged
gastroesophageal reflux disease symptoms. Barretts esophagus, in which specialized intestinal columnar
epithelium replaces the normal esophageal lining in response to chronic inflammation, is a precursor of
esophageal adenocarcinoma. The annual cancer risk for patients with nondysplastic Barretts esophagus is
0.12%0.4%, with a significant increase in risk if dysplasia is present. Surveillance with endoscopy every
3 years is recommended for patients with Barretts esophagus without dysplasia. Patients with
adenocarcinoma of the esophagus found during surveillance endoscopy are more likely to have early-stage,
curable cancer than those whose cancer is found during a diagnostic endoscopy for evaluation of symptoms.

Ref: Rustgi AK, El-Serag HB: Esophageal carcinoma. N Engl J Med 2014;371(26):2499-2509.

Item 64

ANSWER: A

If TSH is suppressed in this patient it indicates that the nodule is producing thyroid hormone and further
evaluation with a radionuclide scan is indicated. If the TSH is normal or elevated the next step is to
determine whether the nodule needs to be biopsied. Thyroid ultrasonography can determine the size and
characteristics of the nodule to help determine whether to refer the patient for a fine-needle aspiration
biopsy. With a large, firm lesion that is highly suspicious for malignancy, it may be appropriate to refer
directly for a fine-needle aspiration biopsy. However, for this patient the lesion did not appear suspicious.
If the patient were hyperthyroid it might be appropriate to check antithyroid antibodies to look for Graves
disease.

Ref: Knox MA: Thyroid nodules. Am Fam Physician 2013;88(3):193-196.

Item 65

ANSWER: C

Keloids are overgrowths of scar tissue seen more commonly in individuals with dark skin. The best initial
treatment is intralesional corticosteroid injections. If this does not produce acceptable results, other
treatment modalities include surgery, laser therapy, and bleomycin injection.

Ref: Kundu RV, Patterson S: Dermatologic conditions in skin of color: Part II. Disorders occurring predominately in skin of
color. Am Fam Physician 2013;87(12):859-865.

22
Item 66

ANSWER: D

The dissociative reactions (flashbacks) in this patient are consistent with the diagnosis of posttraumatic
stress disorder (PTSD). The first-line medications for this disorder are SSRIs and SNRIs. Paroxetine and
sertraline have FDA approval for PTSD. Other antidepressants such as mirtazapine would be second-line
therapy. The effectiveness of central "2-agonists such as clonidine are unknown, and even though
benzodiazepines might help with hyperarousal symptoms, they can worsen other symptoms. Atypical
antipsychotics such as risperidone are not recommended.

Ref: Warner CH, Warner CM, Appenzeller GN, Hoge CW: Identifying and managing posttraumatic stress disorder. Am Fam
Physician 2013;88(12):827-834.

Item 67

ANSWER: B

Crohns disease typically spares the rectum. Ulcerative colitis usually has rectal involvement, progresses
proximally, and rarely has perianal or systemic manifestations (SOR A). Both conditions may cause bloody
diarrhea as well as inflammatory arthropathies, eye inflammation such as uveitis, and skin findings such
as erythema nodosum.

Ref: Cheifetz AS: Management of active Crohn disease. JAMA 2013;309(20):2150-2158.

Item 68

ANSWER: A

Many drugs can cause lung disease. Amiodarone has been known to cause both bronchiolitis obliterans
organizing pneumonia (BOOP) and interstitial pneumonitis. BOOP, also known as cryptogenic organizing
pneumonia, is characterized by interstitial inflammation superimposed on the dominant background of
alveolar and ductal fibrosis. This is a very distinctive pattern of lung response to exposure to several drugs,
including amiodarone, bleomycin, gold, penicillamine, sulfasalazine, radiation, interferons, methotrexate,
mitomycin C, cyclophosphamide, and cocaine.

Interstitial pneumonitis is the most common manifestation of drug-induced lung disease. Drugs that can
cause this include amiodarone, azathioprine, bleomycin, chlorambucil, methotrexate, phenytoin, statins,
and sulfasalazine.

Ref: Broaddus VC, Mason RJ, Ernst JD, et al (eds): Murray & Nadels Textbook of Respiratory Medicine, ed 6. Elsevier
Saunders, 2016, pp 71, 1275-1294.

23
Item 69

ANSWER: B

This patient demonstrates an antalgic gait without a clear etiology. The evaluation of a limping child begins
with a thorough history, observation of the childs gait, and a physical examination. If the history is not
contributory and the physical examination demonstrates no focal source of pain, radiographs of both lower
extremities should be the first step in the workup (SOR C). If there is a focal source of pain, radiographs
of the affected joint would be appropriate (SOR C). If the patient demonstrates systemic signs of illness
such as fever or anorexia, a laboratory evaluation (CBC, erythrocyte sedimentation rate, and C-reactive
protein) should be performed in addition to radiographs. Ultrasonography is useful subsequently if there
is concern about joint effusion. If no source of the problem is found, additional testing should include a
bone scan (SOR C) and MRI.

Ref: Naranje S, Kelly DM, Sawyer JR: A systematic approach to the evaluation of a limping child. Am Fam Physician
2015;92(10):908-916.

Item 70

ANSWER: D

Men who have sex with men but are in a monogamous relationship need not be offered preexposure or
postexposure HIV prophylaxis, unlike men with multiple or anonymous sexual partners. Meningococcal
vaccine is not indicated unless there are other risk factors. Since this patient is in a monogamous
relationship, screening for sexually transmitted infections once a year is considered adequate. Screening
for hepatitis C at this visit is recommended, as well as testing for hepatitis B infection.

Ref: Workowski KA, Bolan GA: Sexually transmitted diseases treatment guidelines, 2015. MMWR Recomm Rep 2015;64
(RR-3):13-16. 2) Final Recommendation Statement: Hepatitis B Virus Infection: Screening, 2014. US Preventive Services
Task Force, October 2014. 3) Knight DA, Jarrett D: Preventive health care for men who have sex with men. Am Fam
Physician 2015;91(12):844-851.

Item 71

ANSWER: A

Based on the patients age it is appropriate for her to be screened. Her bone density is consistent with
osteopenia and she has no identified secondary causes of osteopenia. Because her estimated 10-year risk
of hip fracture is <3% and she has not had any fractures to date, prescription medications such as
bisphosphonates or calcitonin are not indicated. Estrogen does increase bone density but it is not indicated
for osteoporosis prevention or treatment, due to associated cardiovascular risks. Professional organizations
vary on the daily calcium and vitamin D intake recommended for postmenopausal women, but 1200 mg
of dietary calcium and supplementation with 800 IU of vitamin D3 are reasonable recommendations. The
National Osteoporosis Foundation suggests treatment of osteoporosis if the 10-year risk of major
osteoporotic fracture is >20%.

24
Ref: Final Recommendation Statement: Osteoporosis Screening. US Preventive Services Task Force, 2011. 2) Bauer DC:
Calcium supplements and fracture prevention. N Engl J Med 2013;369(16):1537-1543. 3) Jeremiah MP, Unwin BK,
Greenawald MH, Casiano VE: Diagnosis and management of osteoporosis. Am Fam Physician 2015;92(4):261-268. 4)
Black DM, Rosen CJ: Postmenopausal osteoporosis. N Engl J Med 2016;374(3):254-262.

Item 72

ANSWER: E

Meniscal tears are a common source of knee pain in acute knee injuries, occurring in approximately 10%
of cases presenting with acute pain after an injury. Although the McMurray test (passive extension of the
knee while applying valgus and varus stresses to the knee) has historically been used to detect meniscus
injuries, the Thessaly test has superior positive and negative predictive value for meniscus injuries
compared with the McMurray test. The Thessaly test is performed by having the patient stand on the
affected leg while it is flexed 20 and internally and externally rotate the knee three times while holding
the examiners hands for support. Locking, catching, or joint-line pain constitutes a positive test. The
pivot-shift, Lachman, and anterior drawer tests are used to detect injuries to the anterior cruciate ligament,
not meniscal injuries. The valgus stress test detects injuries to the medial collateral ligament.

Ref: Grover M: Evaluating acutely injured patients for internal derangement of the knee. Am Fam Physician
2012;85(3):247-252.

Item 73

ANSWER: E

This patient has low energy, menstrual irregularities, and a history of stress fractures, which is consistent
with the female athlete triad. She is at risk for altered bone density and needs a bone mineral density test.
This condition is also associated with disordered eating and low body mass. Echocardiography may be
indicated if there is a personal or family history of cardiac problems. Radiographs of the feet are not
sensitive for osteoporosis evaluation. A stress test is not indicated.

Ref: Payne JM, Kirchner JT: Should you suspect the female athlete triad? J Fam Pract 2014;63(4):187-192. 2) Weiss Kelly AK,
Hecht S; Council on Sports Medicine and Fitness: The female athlete triad. Pediatrics 2016;138(2):pii:e20160922.

Item 74

ANSWER: E

Over 85% of cases of infectious endocarditis are caused by gram-positive cocci. In patients suspected of
having acute infectious endocarditis, empiric antibiotic treatment should be started immediately after
obtaining initial blood cultures and should include coverage against gram-positive cocci with vancomycin.
For patients with prosthetic heart valves, initial coverage should include vancomycin plus rifampin.
Clindamycin, levofloxacin, and trimethoprim/sulfamethoxazole have no role in the initial treatment of
infectious endocarditis.

Ref: Pierce D, Calkins BC, Thornton K: Infectious endocarditis: Diagnosis and treatment. Am Fam Physician
2012;85(10):981-986.

25
Item 75

ANSWER: C

Vaccination rates for younger children (46 years of age) generally surpass 90%, but rates are much lower
in older children. Only 34% of boys receive HPV vaccine at age 11 or 12, and only 40% receive Tdap.
Family physicians need to be familiar with routine immunizations in this age group so they can be
recommended at the appropriate time.

This patient was up to date with immunizations at age 6 years, so the only catch-up vaccines needed are
those in the routine 11- to 12-year-old set, which includes influenza, meningococcal, HPV, and Tdap
vaccines. DTap and varicella vaccine are not routinely recommended after the age of 6 years.

Ref: Ackerman LK, Serrano JL: Update on routine childhood and adolescent immunizations. Am Fam Physician
2015;92(6):460-468. 2) Centers for Disease Control and Prevention. Birth18 Years & Catch-up Immunization
Schedules. 2016.

Item 76

ANSWER: A

Early studies of ischemic heart disease included mostly male subjects. More recently there has been a
determined effort to understand the special considerations associated with this problem and its management
in women. In 2014 the American Heart Association published a consensus statement summarizing the
research on how to best evaluate women with suspected ischemic heart disease. Its recommendations
focused on the level of pretest risk for ischemic heart disease (low, intermediate, and high), a normal or
abnormal resting EKG, the ability of the subject to exercise, and potential risks of radiation exposure. The
patient in this scenario would be considered low to intermediate risk for ischemia due to her age and risk
factors, along with a history of atypical chest pain. Since her resting EKG is normal and she is physically
fit, she should undergo an exercise treadmill test without imaging.

Ref: Mieres JH, Gulati M, Bairey Merz N; American Heart Association Cardiac Imaging Committee of the Council on Clinical
Cardiology; Cardiovascular Imaging and Intervention Committee of the Council on Cardiovascular Radiology and
Intervention: Role of noninvasive testing in the clinical evaluation of women with suspected ischemic heart disease: A
consensus statement from the American Heart Association. Circulation 2014;130(4):350-379.

26
Item 77

ANSWER: A

Gastroesophageal reflux is very common in infants. Reflux in infants peaks at 4 months of age, and
two-thirds of infants regurgitate at least once a day at this age. The incidence declines dramatically in the
next few months, and by 1 year of age less than 5% of infants regurgitate on a daily basis. If the infant is
healthy and growing normally, reassurance is appropriate (SOR C). Conservative measures are
recommended if the reflux causes distress (SOR C). Such measures include placing the infant on her side
or prone while awake to reduce reflux. However, infants should not be placed prone while sleeping, to
prevent SIDS. Appropriate conservative measures also include smaller, more frequent feedings, the addition
of thickening agents such as rice cereal to formula, and changing to amino acid formulas if infants are
allergic to cows milk protein.

If conservative measures and time do not alleviate the symptoms, a 4-week trial of H2-blockers or proton
pump inhibitors can be tried. The American Academy of Pediatrics and the Society of Hospital Medicine
(Pediatric) recommend not routinely treating reflux with medication. Imaging, such as
esophagogastroduodenoscopy, is not routinely used in the initial workup of reflux in infants. It is reserved
for recalcitrant cases, atypical symptoms, or complications (SOR C).

Ref: Baird DC, Harker DJ, Karmes AS: Diagnosis and treatment of gastroesophageal reflux in infants and children. Am Fam
Physician 2015;92(8):705-714.

Item 78

ANSWER: A

Henoch-Schnlein purpura (HSP) presents most often in children but not infrequently in adults. The
purpuric rash is classically seen on the waist and extends to the legs, sparing the proximal trunk and arms.
Orchitis with testicular swelling occurs in 35% of men with HSP and is often complicated by abdominal
pain, arthritis, and renal insufficiency.

Kawasaki disease is a pediatric disease presenting with fever, conjunctivitis, and lesions of the lips.
Polyarteritis nodosa often presents with fever and multisystem symptoms and findings. There are most often
abnormalities on the CBC and chemistry profile. Rocky Mountain spotted fever is associated with a
petechial rash that involves the proximal trunk and extremities, including the palms and soles.
Thrombocytopenic purpura, by definition, is associated with a low platelet count.

Ref: Reamy BV, Williams PM, Lindsay TJ: Henoch-Schnlein purpura. Am Fam Physician 2009;80(7):697-704. 2) Kasper DL,
Fauci AS, Hauser SL, et al (eds): Harrisons Principles of Internal Medicine, ed 19. McGraw-Hill, 2015, pp 2190-2192.

27
Item 79

ANSWER: D

Opioids, given either orally or intravenously, are the treatment of choice for dyspnea and have been studied
thoroughly in patients with COPD and patients with cancer. They have been found to be effective in
alleviating dyspnea and, when used carefully, do not have serious side effects such as respiratory
depression. When the patient is experiencing anxiety, which regularly occurs in association with
breathlessness, benzodiazepines can be added, although there is no evidence that they improve the dyspnea.
Patients are regularly given supplemental oxygen for dyspnea, but systematic reviews have found no benefit
for patients with cancer or heart failure who do not have hypoxemia. However, oxygen may provide some
relief for patients with COPD who do not have hypoxemia. Prednisone and albuterol are not indicated for
this patient.

Ref: Blinderman CD, Billings JA: Comfort care for patients dying in the hospital. N Engl J Med 2015;373(26):2549-2561.

Item 80

ANSWER: B

Core features of the behavioral variant frontotemporal dementia (FTD) include an insidious onset and
gradual progression, an early decline in social and interpersonal conduct, early impairment in regulation
of personal conduct, early emotional blunting, and early loss of insight. Common initial symptoms include
apathy, lack of initiation, diminished interest, and inactivity. Common features also include disinhibition
and impulsivity. Examples include socially inappropriate remarks, including sexual comments.

These types of symptoms are less common in early phases of other types of dementia. FTD is frequently
misdiagnosed as a primary psychiatric disorder such as depression. Alzheimers disease presents with
memory and visuospatial loss. Lewy body dementia tends to cause memory loss, fluctuating cognition,
visual hallucinations, and spontaneous parkinsonian motor features. Vascular dementia patients usually have
a history of cerebrovascular events. Mixed dementias generally are a combination of Alzheimers and other
types of dementias.

Ref: Cardarelli R, Kertesz A, Knebl JA: Frontotemporal dementia: A review for primary care physicians. Am Fam Physician
2010;82(11):1372-1377. 2) Warren JD, Rohrer JD, Rossor MN: Frontotemporal dementia. BMJ 2013;347:f4827.

Item 81

ANSWER: A

The mainstay of treatment for osteoarthritis of the knee is active rehabilitation and exercise (SOR A).
Active rehabilitation, such as stretching and strengthening, is more effective than passive rehabilitation,
such as taping, heat, electrostimulation, or therapeutic ultrasound (SOR B). Lateral wedge insoles and
glucosamine and chondroitin supplements are unlikely to significantly improve pain in patients with knee
arthritis (SOR B).

Ref: Jones BQ, Covey CJ, Sineath MH Jr: Nonsurgical management of knee pain in adults. Am Fam Physician
2015;92(10):875-883.

28
Item 82

ANSWER: D

This child meets the criteria for acute bacterial sinusitis (ABS) and should be treated with antibiotics. He
exemplifies the concept of double sickening, in which a child initially has typical symptoms of a viral
upper respiratory infection and improves initially only to worsen later, with daytime cough, persistent nasal
discharge, and/or new fever. Other criteria for ABS include persistence of URI symptoms without
improvement after 710 days and severe onset ABS with a high fever and purulent nasal discharge for
at least 3 days. Evidence shows that treatment with antibiotics in these situations improves outcomes (SOR
B). The first-line antibiotic is amoxicillin with or without clavulanate. The length of treatment can range
from 10 to 28 days. Depending on risks, patients may be treated with either high-dose amoxicillin or
amoxicillin/clavulanate, with an amoxicillin dosage of 90 mg/kg/day. Many of the bacteria causing ABS
have been shown to be resistant to azithromycin and trimethoprim/sulfamethoxazole and these antibiotics
should be avoided. For patients allergic to penicillins, cephalosporins should be used.

The diagnosis of ABS is based on the history (SOR C). The physical examination is not particularly helpful
and findings such as sinus tenderness, mucosal swelling, and transillumination of the sinuses do not help
differentiate ABS from a viral URI. Laboratory studies are not indicated in the diagnosis. Imaging studies
are likewise not indicated for the initial diagnosis, as they are often abnormal in both viral URIs and ABS.
If complications such as orbital cellulitis or neurologic compromise are a concern, then CT may be
indicated.

There is no good evidence to support adjuvant care for ABS. Saline nasal irrigation, decongestants, or
intranasal corticosteroids may be helpful but cannot replace antibiotic therapy in children who meet the
criteria for ABS.

Ref: Wald ER, Applegate KE, Bordley C, et al: Clinical practice guideline for the diagnosis and management of acute bacterial
sinusitis in children aged 1 to 18 years. Pediatrics 2013;132(1):e262-e280. 2) Marom T, Alvarez-Fernandez PE, Jennings
K, et al: Acute bacterial sinusitis complicating viral upper respiratory tract infection in young children. Pediatr Infect Dis
J 2014;33(8):803-808. 3) Hauk L: AAP releases guideline on diagnosis and management of acute bacterial sinusitis in
children one to 18 years of age. Am Fam Physician 2014;89(8):676-681.

Item 83

ANSWER: E

The 2014 evidence-based guideline from the JNC 8 panel recommends that in the general African-American
population, including those with diabetes mellitus, initial antihypertensive treatment should include a
thiazide-type diuretic or calcium channel blocker (for general African-American population: SOR B; for
African-American patients with diabetes: SOR C).

Ref: James PA, Oparil S, Carter BL, et al: 2014 Evidence-based guideline for the management of high blood pressure in adults:
Report from the panel members appointed to the Eighth Joint National Committee (JNC 8). JAMA 2014;311(5):507-520.

29
Item 84

ANSWER: D

Asthmatic symptoms during exercise are common. These can occur as exacerbations of underlying airway
inflammation or as bronchospasm in otherwise normal airways. This patient seems to have bronchospasm
that would be best managed by albuterol prior to exercise. If she finds that she needs her inhaler frequently,
the addition of an anti-inflammatory agent such as inhaled corticosteroids or oral montelukast would be
reasonable.

Ref: Krafczyk MA, Asplund CA: Exercise-induced bronchoconstriction: Diagnosis and management. Am Fam Physician
2011;84(4):427-434. 2) Spiro SG, Silvestri GA, Agusti A: Clinical Respiratory Medicine, ed 4. Elsevier Saunders, 2012,
pp 501-520. 3) Boulet LP, OByrne PM: Asthma and exercise-induced bronchoconstriction in athletes. N Engl J Med
2015;372(7):641-648.

Item 85

ANSWER: B

Ankylosing spondylitis is an inflammatory condition that affects the axial skeleton primarily, but other
joints may be involved. The pain begins insidiously in the lower back and gluteal region. The symptoms
improve with exercise and activity but they worsen at night. The stiffness is most prominent in the morning
and may last up to a few hours.

Ref: Kasper DL, Fauci AS, Hauser SL, et al (eds): Harrisons Principles of Internal Medicine, ed 19. McGraw-Hill, 2015, pp
2169-2173.

Item 86

ANSWER: C

Radiation exposure in humans is quantified by the sievert (Sv), which equals 1 joule of radiation energy/kg
of human tissue. Most clinical diagnostic test exposures measure in the millisievert (mSv) range. Natural
background exposure in the United States averages 3 mSv/year. Although exact exposures will vary
according to patient size, type of equipment used, and operator expertise, typical radiation doses for
common radiographic studies include the following:

Posteroanterior chest0.02 mSv


Skull0.1 mSv
Lumbar spine1.5 mSv
CT head2 mSv
CT abdomen8 mSv

30
This list shows that CT of the abdomen provides a radiation dose 400 times that of the typical
posteroanterior chest radiograph. A small increased relative risk of cancer mortality was demonstrated in
Japanese survivors of atomic bombs receiving doses in the range of 520 mSv, suggesting that consideration
of the risk/benefit ratio of some radiographic studies is warranted.

Ref: US Food and Drug Administration: What are the radiation risks from CT? 2016.

Item 87

ANSWER: B

Patients with recurrent nephrolithiasis should first try to increase fluid intake to achieve a daily urine output
2 L. Increasing fluid intake decreases the recurrence of stones by at least 50%. Reducing soft drink intake
may also help but this seems to be limited to those who drink colas, which are acidified by phosphoric acid.
Soft drinks acidified with citric acid, such as fruit-flavored drinks, do not appear to have the same effect.
There is little evidence that dietary changes help significantly. If a patients fluid increase is not sufficient,
treatment with a thiazide diuretic, citrate, or allopurinol is recommended (SOR C).

Ref: Qaseem A, Dallas P, Forciea MA, et al: Dietary and pharmacologic management to prevent recurrent nephrolithiasis in
adults: A clinical practice guideline from the American College of Physicians. Ann Intern Med 2014;161(9):659-667. 2)
Hauk L: Prevention of recurrent nephrolithiasis: Dietary and pharmacologic options recommended by the ACP. Am Fam
Physician 2015;92(4):311.

Item 88

ANSWER: A

The 2013 American College of Cardiology/American Heart Association guideline for reducing
atherosclerotic cardiovascular disease (ASCVD) risk in adults recommended several significant changes
in the management of hyperlipidemia. This guideline recommends looking at overall risk as estimated by
a tool, the ASCVD Risk Estimator, which considers not only lipid parameters but also age, sex, ethnicity,
systolic blood pressure, and the presence or absence of diabetes mellitus, treated hypertension, and
smoking. Four major risk groups were identified for treatment, one of which was adults 40 years of age
with an estimated 10-year risk of ASCVD 7.5%.

Therapy is graded by intensity (low-, moderate-, or high-intensity statin therapy), and therapeutic targets
for LDL-cholesterol were abandoned since there is no demonstrable benefit from achieving a certain level
of LDL-cholesterol in treated patients. This patient has an estimated 10-year ASCVD risk well above 7.5%
and is a candidate for moderate- or high-intensity statin therapy under the guideline. Niacin and fibrate
therapy do not have a demonstrable impact on cardiovascular outcomes and are not recommended in the
guideline. PCSK9 inhibitors are also not yet recommended in any guideline.

Ref: ACC/AHA release updated guideline on the treatment of blood cholesterol to reduce ASCVD risk. Am Fam Physician
2014;90(4):260-265. 2) Stone NJ, Robinson JG, Lichtenstein AH, et al: 2013 ACC/AHA guideline on the treatment of
blood cholesterol to reduce atherosclerotic cardiovascular risk in adults: A report of the American College of
Cardiology/American Heart Association Task Force on Practice Guidelines. Circulation 2014;129(25 Suppl 2):S1-S45.

31
Item 89

ANSWER: B

Major risk factors for acute mountain sickness include a history of previous mountain sickness, fast ascent,
and lack of acclimatization. Slow ascent with frequent stops at various levels is the safest way to prevent
altitude sickness. Females are at increased risk and good physical fitness is not protective. Persons <46
years of age are at increased risk.

Ref: Brtsch P, Swenson ER: Acute high-altitude illnesses. N Engl J Med 2013;368(24):2294-2302.

Item 90

ANSWER: A

Thiazide diuretics such as chlorthalidone are considered a first-line therapy for hypertension. $-Blockers,
aldosterone antagonists, and other antihypertensive medications may be used as add-on therapy to reach
blood pressure goals.

Ref: James PA, Oparil S, Carter BL, et al: 2014 evidence-based guidelines for the management of high blood pressure in adults:
Report from the panel members appointed to the Eighth Joint National Committee (JNC 8). JAMA 2014;311(5):507-520.

Item 91

ANSWER: D

Prompt recognition and treatment of sepsis increases the chances of survival. Aggressive fluid resuscitation
is the initial treatment for hypotension in patients in septic shock. Antibiotic therapy should be administered
within 1 hour of suspecting sepsis. If fluid resuscitation is not successful in restoring blood pressure,
norepinephrine is the currently recommended first-line vasopressor. The use of hydroxyethyl starch is not
recommended because the mortality rate is higher in sepsis patients. Previously, dopamine was
recommended, but low-dose dopamine for renal perfusion has now been shown to be ineffective. For
patients who are vasopressor dependent, low-dose corticosteroids can be considered.

Ref: Gauer RL: Early recognition and management of sepsis in adults: The first six hours. Am Fam Physician 2013;88(1):44-53.
2) Seymour CW, Rosengart MR: Septic shock: Advances in diagnosis and treatment. JAMA 2015;314(7):708-717.

32
Item 92

ANSWER: B

In spite of an estimated 50%80% reduction in cases of invasive pneumococcal disease (IPD) as a result
of vaccination with the 23-valent pneumococcal polysaccharide vaccine (PPSV23), IPD remains a
significant problem in the United States, with approximately 40,000 cases in 2010 resulting in about 4000
deaths. The introduction of the 13-valent pneumococcal conjugate vaccine (PCV13) has been shown to
further reduce vaccine-type cases of IPD by as much as 75%, leading to the 2014 recommendation from
the Advisory Committee on Immunization Practices to administer PCV13 to adults age 65 and older. In
immunocompetent adults age 65 and older who are pneumococcal vaccinenaive, administering PPSV23
1 year or more after PCV13 results in a better immune response than giving PPSV23 first, and reduces
local reactions to the vaccines compared to simultaneous administration or administering PPSV23 first. For
some immunocompromised patients the recommendation is to wait a minimum of 8 weeks after giving
PCV13 before administering PPSV23.

Ref: Shah AA, Fields H, Wallace MR: Navigating the changes in pneumococcal immunizations for adults. Am Fam Physician
2015;92(6):456-458. 2) Kobayashi M, Bennett NM, Gierke R, et al: Intervals between PCV13 and PPSV23 vaccines:
Recommendations of the Advisory Committee on Immunization Practices (ACIP). MMWR Morb Mortal Wkly Rep
2015;64(34):944-947.

Item 93

ANSWER: B

Average-risk adults should be screened for colorectal cancer starting at age 50. People who are at higher
risk for developing colon cancer should be screened at either age 40 or 10 years earlier than the age at
which the youngest affected family member was diagnosed, whichever is earliest. Risk factors for colon
cancer include age, ethnicity, and family history. There is good evidence that screening identifies
premalignant lesions, which allows for early treatment and reduced mortality.

Ref: Randel A: ACP releases best practice advice on colorectal cancer screening. Am Fam Physician 2012;86(12):1153-1154.

Item 94

ANSWER: B

In the past, rubella was a common cause for reduced hearing. With the advent of vaccination, genetic
inheritance has become the most frequent cause for deafness. Aminoglycosides are rarely a reason for
hearing loss. The well-known association of aminoglycosides with hearing loss has reduced the exposure
risk from these drugs. Head trauma and prematurity remain important causes for deafness but are still very
small risk factors compared to simple inheritance. Prematurity risk has diminished with improvements in
the care of premature infants.

Ref: Morton CC, Nance WE: Newborn hearing screeningA silent revolution. N Engl J Med 2006;354(20):2151-2164. 2)
Rakel RE, Rakel DP (eds): Textbook of Family Medicine, ed 9. Elsevier Saunders, 2016, p 322.

33
Item 95

ANSWER: C

Treatment of genital warts should be based on patient preference and cost (SOR C). There is no treatment
that is more effective than others, and the recurrence rate for any treatment is relatively high. Cryotherapy
is not more effective than other treatments. Treatment in pregnancy has not been found to decrease the risk
of transmission.

Ref: Karnes JB, Usatine RP: Management of external genital warts. Am Fam Physician 2014;90(5):312-318.

Item 96

ANSWER: E

Serotonin syndrome is a potentially life-threatening condition caused by excessive serotonergic activity, and
certain medications are more likely to precipitate it. Early recognition of symptoms is important, as most
cases can be managed on an outpatient basis with discontinuation of the precipitating medication/agent and
supportive care. The Hunter Serotonin Toxicity Criteria can be used to diagnose serotonin syndrome (SOR
C).

The differential diagnosis includes anticholinergic syndrome, malignant hyperthermia, and neuroleptic
malignant syndrome. Anticholinergic syndrome is associated with tachycardia, tachypnea, and
hyperthermia. Malignant hyperthermia and neuroleptic malignant syndrome are associated with
hypertension, tachycardia, tachypnea, and hyperthermia. These conditions are precipitated by other classes
of medications. Central cord syndrome is a spinal cord disease caused by spinal trauma, syringomyelia,
and intrinsic cord tumors, and presents with arm weakness greater than leg weakness.

Ref: Ables AZ, Nagubilli R: Prevention, recognition, and management of serotonin syndrome. Am Fam Physician
2010;81(9):1139-1142. 2) Kasper DL, Fauci AS, Hauser SL, et al (eds): Harrisons Principles of Internal Medicine, ed
19. McGraw-Hill, 2015, pp 2650-2660.

Item 97

ANSWER: C

Only subacute thyroiditis is associated with a painful and tender thyroid. It often follows an upper
respiratory viral illness that triggers inflammatory destruction of thyroid tissue. Thyroid hormone is
increased and TSH is suppressed. Treatment consists of corticosteroids or NSAIDs. The condition is usually
self-limited.

Ref: Sweeney LB, Stewart C, Gaitonde DY: Thyroiditis: An integrated approach. Am Fam Physician 2014;90(6):389-396.

34
Item 98

ANSWER: E

This patient has a high risk of pulmonary embolism based on her presentation and the elevated D-dimer
assay. CT angiography (CTA) of the chest would be the next step in the evaluation of this patient (SOR A).
The other tests may be helpful but should not delay chest CTA. If chest CTA is negative a venous duplex
study would be helpful in ruling out a DVT.

Ref: Qaseem A, Snow V, Barry P, et al; Joint American Academy of Family Physicians/American College of Physicians Panel
on Deep Venous Thrombosis/Pulmonary Embolism: Current diagnosis of venous thromboembolism in primary care: A
clinical practice guideline from the American Academy of Family Physicians and the American College of Physicians. Ann
Fam Med 2007;5(1):57-62. 2) Fesmire FM, Brown MD, Espinosa JA, et al; American College of Emergency Physicians:
Critical issues in the evaluation and management of adult patients presenting to the emergency department with suspected
pulmonary embolism. Ann Emerg Med 2011;57(6):628-652.

Item 99

ANSWER: C

Retinal detachments can be caused by a break in the retina, exudate or leakage from beneath the retina, or
traction on the retina. Retinal detachments are often preceded by a posterior vitreous detachment, which
can lead to a break in the retina, and patients may experience an increase in floaters but not light flashes
or loss of vision. With a retinal detachment, patients typically experience floaters followed within 1 week
by flashes of light as the retina tears. If the retinal tear becomes large enough vision is impaired. Any
patient with the typical history and loss of visual acuity or peripheral field should be urgently referred to
an ophthalmologist, as urgent surgery may be required.

Risk factors for retinal detachment include age 5075, ocular trauma, previous cataract surgery, family
history, and a past history of retinal detachment. A patient with a previous retinal detachment has a 25%
risk of developing a retinal detachment in the other eye. Myopia is the other significant risk factor,
associated with a tenfold increased risk in patients with >3 diopters of refractive error.

Central retinal artery occlusion is manifested by painless complete loss of vision in the affected eye. Acute
angle-closure glaucoma is associated with eye pain, photophobia, headache, nausea and vomiting, and
cloudy loss of vision, and the onset is not typically associated with coughing or sneezing.

Ref: Gelston CD: Common eye emergencies. Am Fam Physician 2013;88(8):515-519.

35
Item 100

ANSWER: A

The 2014 Global Initiative for Chronic Obstructive Lung Disease (GOLD) guideline recommends
administration of prednisone, 40 mg daily for 5 days, for COPD exacerbations. The 2011 GOLD guideline
had recommended the same dosage of prednisone for 1014 days. The guideline does not recommend a
methylprednisolone dose pack or intravenous methylprednisolone sodium succinate. A recent multicenter
study showed that the shorter duration of low-dose prednisone was equivalent to the longer treatment.
Corticosteroids are associated with elevated blood glucose; the development of cataracts, diabetes mellitus,
and osteopenia; and thromboembolic complications.

Ref: Sonstein L, Clark C, Seidensticker S, et al: Improving adherence for management of acute exacerbation of chronic
obstructive pulmonary disease. Am J Med 2014;127(11):1097-1104.

Item 101

ANSWER: B

Von Willebrand disease (vWD) is a common coagulation disorder generally due to a hereditary reduction
in the quality or quantity of a protein complex required for platelet adhesion, known as von Willebrand
factor (vWF). The extent of deficiency varies greatly, resulting in vWD subtypes ranging from
asymptomatic to serious. A common problem associated with vWD is menorrhagia, and the diagnosis
should always be entertained in women who experience excessive menstrual blood loss. Although tests
measuring vWF are easily obtained, interpretation of the results can be challenging since vWF levels can
be affected by blood type, inflammation, infection, trauma, and emotional stress. Confirmation of vWD
often requires the expertise of a hematologist. Although the results for all laboratory tests listed can fall
within their reference ranges in a patient with vWD, the finding most suggestive of this diagnosis is an
isolated prolonged partial thromboplastin time.

Ref: Yawn B, Nichols WL, Rick ME: Diagnosis and management of von Willebrand disease: Guidelines for primary care. Am
Fam Physician 2009;80(11):1261-1268. 2) Committee on Adolescent Health Care, Committee on Gynecologic Practice:
Von Willebrand disease in women. American College of Obstetricians and Gynecologists, Committee Opinion no 580, 2013
(reaffirmed 2015).

36
Item 102

ANSWER: C

This patient has slight elevations of her alkaline phosphatase along with a deficiency of vitamin D and
normal calcium levels. This constellation of findings is most consistent with secondary hyperparathyroidism
related to vitamin D deficiency. Vitamin D storage is best reflected by the serum 25-hydroxyvitamin D
level. While there is some disagreement regarding normal levels, a level <10 ng/mL is clearly deficient.
This would put the patient at risk for osteomalacia but not rickets, which is a clinical diagnosis based on
the effects of insufficient bone mineralization secondary to low vitamin D activity before the closure of
growth plates. Supplementation with activated vitamin D (calcitriol) is generally only necessary in patients
with renal failure or other conditions associated with inadequate activation of the storage forms of vitamin
D. Supplementation with vitamin D should decrease PTH activity and thus bone turnover in this patient,
which would likely normalize the alkaline phosphatase.

Ref: Kasper DL, Fauci AS, Hauser SL, et al (eds): Harrisons Principles of Internal Medicine, ed 19. McGraw-Hill, 2015, pp
2454-2466.

Item 103

ANSWER: D

This patient presents with a classic description of torsion of the left testicle, and not an infectious process
requiring antibiotics. Surgical exploration is the immediate priority, with ischemic damage starting between
4 and 8 hours after onset. Attempts at manual detorsion should not delay surgical exploration and often
require analgesia or sedation. If surgery is not an immediate option, manual detorsion is performed by
rotating the testicle from medial to lateral, like opening a book. Doppler ultrasonography is the imaging
modality of choice, but it delays surgical exploration and should be used only when the history and physical
examination make the diagnosis questionable.

Ref: Sharp VJ, Kieran K, Arlen AM: Testicular torsion: Diagnosis, evaluation, and management. Am Fam Physician
2013;88(12):835-840.

Item 104

ANSWER: B

The Patient Protection and Affordable Care Act amended the Federal Labor Standards Act of 1938 to
require employers of >50 employees to provide nursing mothers reasonable break time to express milk
for up to 1 year following the birth of their children. The employer is not required to compensate nursing
mothers during the breaks and is not required to provide refrigerated storage for the expressed milk. The
law also requires that the employer provide a place to express milk, other than a bathroom, that is shielded
from view and free from intrusion from coworkers and the public.

Ref: US Department of Labor, Wage and Hour Division: Section 7(r) of the Fair Labor Standards Act of 1938 (29 U.S.C. 207):
Break time for nursing mothers provision.

37
Item 105

ANSWER: A

Laboratory testing for Clostridium difficile should be done only on symptomatic patients. A diagnosis of
C. difficile infection requires the presence of diarrhea ( 3 unformed stools in a 24-hour period) or
radiographic evidence of ileus and toxic megacolon. In addition, the diagnosis requires a positive stool test
for toxigenic C. difficile or its toxins, or colonoscopic or histopathologic findings showing
pseudomembranous colitis. Laboratory testing cannot distinguish between asymptomatic colonization and
symptomatic infection. Test of cure is not recommended after C. difficile treatment. Probiotics may prevent
antibiotic-associated diarrhea and may also reduce C. difficile diarrhea in children and adults younger than
65, but are not specifically recommended for preventing or treating C. difficile infection.

Ref: Bagdasarian N, Rao K, Malani PN: Diagnosis and treatment of Clostridium difficile in adults: A systematic review. JAMA
2015;313(4):398-408.

Item 106

ANSWER: D

Symptoms consistent with proctalgia fugax occur in 13%19% of the general population. These consist of
episodic, sudden, sharp pains in the anorectal area lasting several seconds to minutes. The diagnosis is
based on a history that fits the classic picture in a patient with a normal examination. All the other
diagnoses listed would be evident from the physical examination, except for sacral nerve neuralgia, which
would not be intermittent for years and the pain would not be transitory.

Ref: Feldman M, Friedman LS, Brandt LJ (eds): Sleisenger and Fordtrans Gastrointestinal and Liver Disease, ed 9. Saunders
Elsevier, 2010, pp 2272-2273. 2) Fargo MV, Latimer KM: Evaluation and management of common anorectal conditions.
Am Fam Physician 2012;85(6):624-630.

Item 107

ANSWER: A

This is a cutaneous horn, sometimes referred to as a hypertrophic actinic keratosis. It is a horn-like


projection of keratin on a slightly raised base. These usually arise in areas subject to photoaging, including
the forearms. The differential diagnosis often includes keratoacanthoma, which occurs most commonly on
the face, grows very rapidly, and often is more nodular with a central pit. Pyogenic granulomas are fleshy
appearing, and a wart or seborrheic keratosis would both look slightly different but would rarely become
so large in just months. Cutaneous horns should be removed due to the possible development of in situ or
invasive squamous cell carcinoma (SOR A).

Ref: Habif TP: Clinical Dermatology: A Color Guide to Diagnosis and Therapy, ed 6. Mosby Elsevier, 2015, p 794.

38
Item 108

ANSWER: A

Undertreatment of osteoporosis occurs frequently after a hip fracture. Unless a contraindication exists,
patients should be treated with a bisphosphonate after a hip fracture, regardless of bone mineral density
(SOR C). Controlled gastroesophageal reflux is not a contraindication to bisphosphonate therapy. Proton
pump inhibitor use, but not H2-blocker use, is also a modifiable risk factor for osteoporosis. Raloxifene and
teriparatide are not bisphosphonates or first-line therapy for prevention of recurrent hip fracture. Raloxifene
has not been shown to reduce the rate of nonvertebral fractures.

Ref: LeBlanc KE, Muncie HL Jr, LeBlanc LL: Hip fracture: Diagnosis, treatment, and secondary prevention. Am Fam Physician
2014;89(12):945-951.

Item 109

ANSWER: C

All currently available influenza vaccines, with the exceptions of recombinant and cell-culturebased
inactivated influenza vaccines, are prepared using embryonated egg culture and can potentially provoke
allergic and anaphylactic reactions. Large studies of influenza vaccine administration to egg-allergic patients
have resulted in a few mild reactions but no documented occurrences of anaphylaxis, although there are
reports of serious reactions. The data collected from these studies provided sufficient confidence for the
Advisory Committee on Immunization Practices to develop guidelines for administration of influenza
vaccine in individuals with egg allergy.

For those who report that they can eat lightly cooked scrambled eggs, vaccination can proceed without
precaution or observation. Those who have experienced only hives can also receive any influenza vaccine
appropriate for their age and health status. In the past the CDC recommended observing these patients for
30 minutes afterward, but this recommendation was changed in 2016. People who have experienced
symptoms such as hypotension, wheezing, nausea, or vomiting, or reactions requiring emergency attention
or epinephrine after eating eggs or egg-containing foods can also receive any influenza vaccine appropriate
for their age and health status and also do not need to be observed. However, the vaccine should be
administered by a provider who can recognize and manage severe allergic reactions. Withholding
vaccination because of egg-induced hives is not recommended.

Ref: Grohskopf LA, Sokolow LZ, Broder KR, et al: Prevention and control of seasonal influenza with vaccines:
Recommendations of the Advisory Committee on Immunization PracticesUnited States, 201617 influenza season.
MMWR Recomm Rep 2016;65(RR-5):1-54.

39
Item 110

ANSWER: D

Patients with low back pain should be evaluated for the presence of neurologic deficits. Urinary retention
is the most frequent finding in cauda equina syndrome (90% sensitivity), caused by compression of nerve
roots from the lower cord segments. This is usually due to a massive, centrally herniated disc, which can
result in urinary retention or incontinence from loss of sphincter function, bilateral motor weakness of the
lower extremities, and saddle anesthesia. This problem should be addressed urgently. In patients without
urinary retention, the probability of the cauda equina syndrome is approximately 1 in 10,000.

Ref: Chou R, Qaseem A, Snow V, et al: Diagnosis and treatment of low back pain: A joint clinical practice guideline from the
American College of Physicians and the American Pain Society. Ann Intern Med 2007;147(7):478-491. 2) Casazza BA:
Diagnosis and treatment of acute low back pain. Am Fam Physician 2012;85(4):343-350.

Item 111

ANSWER: A

This patient has a low risk of cancer, based upon his age and medical history, and no suggestion of a lower
respiratory infection. With this presentation, a chest radiograph is recommended as the first step in the
workup, and if findings are normal he should be observed for 26 weeks (SOR A). If there is a recurrence
of hemoptysis further evaluation is indicated, which should include an interval history, a repeat
examination, and CT of the chest.

If the initial presentation had suggested a lower respiratory infection, treatment with oral antibiotics rather
than observation would have been appropriate.

Ref: Earwood JS, Thompson TD: Hemoptysis: Evaluation and management. Am Fam Physician 2015;91(4):243-249.

Item 112

ANSWER: A

The Timed Up and Go test is the most frequently recommended screening test for mobility. It takes less
than a minute to perform and involves asking the patient to rise from a chair, walk 10 feet, turn, return to
the chair, and sit down. Any unsafe or ineffective movement with this test suggests balance or gait
impairment and an increased risk of falling. If the test is abnormal, referral to physical therapy for complete
evaluation and assessment should be considered. Other interventions should also be considered, such as a
medication review for factors related to the risk of falling.

Ref: Tinetti ME, Kumar C: The patient who falls: Its always a trade-off. JAMA 2010;303(3):258-266. 2) Herman T, Giladi
N, Hausdorff JM: Properties of the Timed Up and Go test: More than meets the eye. Gerontology 2011;57(3):203-210.

40
Item 113

ANSWER: D

Hemoglobin A1c (HbA1c) testing measures the percentage of glycosylation of the HbA1c chain, and correlates
to average blood glucose levels over the previous 23 months. However, hypertriglyceridemia,
hyperbilirubinemia, iron deficiency anemia, splenectomy, renal failure, and aplastic anemia can all falsely
elevate HbA1c levels. The other factors listed can all falsely lower HbA1c levels, as can vitamins C and E,
and acute blood loss.

Ref: Pippitt K, Li M, Gurgle HE: Diabetes mellitus: Screening and diagnosis. Am Fam Physician 2016;93(2):103-109.

Item 114

ANSWER: A

The U.S. Preventive Services Task Force has a grade I recommendation for routine screening for
osteoporosis in men, meaning there is insufficient evidence to recommend for or against routine screening.
Men older than 50 with a minimal-trauma fracture and men with conditions associated with bone loss could
be considered for screening. The National Osteoporosis Foundation recommends screening all men age 70
and above for osteoporosis.

Ref: US Preventive Services Task Force: Screening for osteoporosis: US Preventive Services Task Force recommendation
statement. Ann Intern Med 2011;154(5):356-364. 2) Jeremiah MP, Unwin BK, Greenawald MH, Casiano VE: Diagnosis
and management of osteoporosis. Am Fam Physician 2015;92(4):261-268.

Item 115

ANSWER: C

Although this patient has a family history of coronary artery disease, she is under the age of 50 and thus
aspirin therapy as primary prevention is not recommended and may increase the risk for gastrointestinal
bleeding, regardless of the dosage. The U.S. Preventive Services Task Force found insufficient evidence
for screening for chronic kidney disease even in individuals with a positive family history. Neither a resting
nor exercise EKG is recommended for asymptomatic individuals to detect or prevent coronary artery
disease.

Ref: Moyer VA; US Preventive Services Task Force: Screening for coronary heart disease with electrocardiography: US
Preventive Services Task Force recommendation statement. Ann Intern Med 2012;157(7):512-518. 2) Vandvik PO, Lincoff
AM, Gore JM, et al; American College of Chest Physicians: Primary and secondary prevention of cardiovascular disease:
Antithrombotic therapy and prevention of thrombosis, 9th ed: American College of Chest Physicians evidence-based clinical
practice guidelines. Chest 2012;141(2 Suppl):e637S-668S. 3) Jumbo M, Dorsch MP, Ealovega MW, et al: Essential
hypertension. University of Michigan Health System Guidelines for Clinical Care, 2014.

41
Item 116

ANSWER: B

This patient has signs and symptoms of acute salpingitis. This condition is commonly confused with
appendicitis, ectopic pregnancy, and other pelvic pathology. In this case the findings are clearly pelvic in
origin. Endocervical inflammation with a mucopurulent discharge is noted in almost every case of acute
salpingitis. The acute nature of this presentation and the adnexal fullness suggest gonorrhea rather than
chlamydial infection, although the antibiotic regimen should probably cover Chlamydia as well. Admission
to the hospital and treatment with parenteral antibiotics is most appropriate in this case because of the
severity of the illness, the desire to maintain reproductive function, and the adnexal fullness.

Routine laparoscopy for every case of salpingitis is considered too costly and dangerous. The choice of
intravenous antibiotic may vary, but usually consists of a $-lactam antibiotic (cefoxitin or ceftriaxone) plus
doxycycline, or gentamicin and clindamycin. Ceftriaxone, 250 mg intramuscularly, is appropriate for
uncomplicated gonococcal infection.

Ref: Workowski KA, Bolan G; Centers for Disease Control and Prevention (CDC): Sexually transmitted diseases treatment
guidelines, 2015. MMWR Recomm Rep 2015;64(RR-03):78-82.

Item 117

ANSWER: B

Mite larvae called chiggers cause itchy bites. The chiggers crawl on skin until they reach constrictive
clothing like belts or socks and then bite there. Flea bites are usually at ankle height because fleas jump.
Mosquito bites would be diffuse on exposed areas. Bedbugs tend to bite on the upper body and neck. This
is not a typical presentation for Lyme disease, which has an initial rash that is localized and not pruritic
(SOR C).

Ref: Juckett G: Arthropod bites. Am Fam Physician 2013;88(12):841-847.

Item 118

ANSWER: C

It is important for health care professionals to be familiar with the various options for screening and testing
for latent or active tuberculosis. In the United States the tuberculin skin test (TST) is the traditional
screening test for tuberculosis. The interferon-gamma release assay (IGRA) is a blood test that can also aid
in the diagnosis of latent tuberculosis. Advantages of IGRA include the ability to get results without
follow-up and the fact that prior bacille Calmette-Gurin (BCG) vaccination does not cause a false-positive
test. With TST testing, prior BCG vaccination, especially if given within the last 10 years, can result in
a false-positive test.

As with the TST, conversion (i.e., a positive test) of IGRA may not occur within the first 812 weeks
following exposure to an individual with active tuberculosis. In addition, neither the TST nor the IGRA can
distinguish between latent and active tuberculosis.

42
There are two IGRA tests available for use in the United States and both tests need to be processed within
830 hours, depending on the specific test used. Although there is now a test available that can detect
Mycobacterium tuberculosis complex (MTBC) and resistance to rifampin, it is a separate test called the
Xpert MTB/RIF assay.

Ref: Tuberculosis: Testing & diagnosis. Centers for Disease Control and Prevention, 2013. 2) Hartman-Adams H, Clark K,
Juckett G: Update on latent tuberculosis infection. Am Fam Physician 2014;89(11):889-896.

Item 119

ANSWER: E

This patients symptoms are consistent with the DSM-5 criteria for generalized anxiety disorder. First-line
treatments for this condition are SSRIs, SNRIs, and tricyclic antidepressants. Quetiapine and gabapentin
are considered second-line medications for anxiety if control cannot be obtained with more traditional
agents. Benzodiazepines such as alprazolam and clonazepam are sometimes necessary for short-term control
of anxiety symptoms but are generally discouraged due to sedating side effects, the potential for abuse or
diversion, and gradual tolerance.

Ref: American Psychiatric Association: Diagnostic and Statistical Manual of Mental Disorders, ed 5. American Psychiatric
Association, 2013, pp 189-264. 2) Combs H, Markman J: Anxiety disorders in primary care. Med Clin North Am
2014;98(5):1007-1023. 3) Locke AB, Kirst N, Shultz CG: Diagnosis and management of generalized anxiety disorder and
panic disorder in adults. Am Fam Physician 2015;91(9):617-624.

Item 120

ANSWER: E

Bariatric surgery has been shown to reduce all-cause mortality in patients with morbid obesity, mostly from
reduced myocardial infarctions. Although orlistat and phentermine, along with other weight loss drugs,
have been shown to be associated with moderate weight loss, there is no evidence that any of these agents
reduce morbidity or mortality. A low-carbohydrate diet has been associated with increased HDL-cholesterol
levels and decreased triglyceride levels when compared to a low-fat diet, which may indicate a reduction
in cardiac risk. No particular diet strategy has been shown to be more effective for weight loss than any
other strategy.

Ref: Sjstrm L, Narbro K, Sjstrm CD, et al: Effects of bariatric surgery on mortality in Swedish obese subjects. N Engl J
Med 2007;357(8):741-752. 2) Shai I, Schwarzfuchs D, Henkin Y, et al: Weight loss with a low-carbohydrate,
Mediterranean, or low-fat diet. N Engl J Med 2008;359(3):229-241. 3) Bazzano LA, Hu T, Reynolds K, et al: Effects of
low-carbohydrate and low-fat diets: A randomized trial. Ann Intern Med 2014;161(5):309-318. 4) Yanovski SZ, Yanovski
JA: Long-term drug treatment for obesity: A systematic and clinical review. JAMA 2014;311(1):74-86.

43
Item 121

ANSWER: A

The combination of antipsychotic medication and psychosocial treatments, including cognitive-behavioral


therapy, family therapy, and social skills training, is associated with the best outcomes in patients with
schizophrenia (SOR B). Antipsychotic medications should not be combined. Hospitalization, especially for
the first episode of schizophrenia, is also recommended for the best outcome (SOR C). Antidepressant
medication will treat depression associated with schizophrenia but will not necessarily improve the
symptoms of schizophrenia.

Ref: Holder SH, Wayhs A: Schizophrenia. Am Fam Physician 2014;90(11):775-782.

Item 122

ANSWER: D

Inpatient video-electroencephalography (vEEG) monitoring is the preferred test for the diagnosis of
psychogenic nonepileptic seizures (PNES), and is considered the gold standard (SOR B). Video-EEG
monitoring combines extended EEG monitoring with time-locked video acquisition that allows for analysis
of clinical and electrographic features during a captured event. Many other types of evidence have been
used, including the presence or absence of self-injury and incontinence, the ability to induce seizures by
suggestion, psychologic tests, and ambulatory EEG. While useful in some cases, these alternatives have
been found to be insufficient for the diagnosis of PNES.

Elevated postictal prolactin levels (at least two times the upper limit of normal) have been used to
differentiate generalized and complex partial seizures from PNES but are not reliable (SOR B). While
prolactin levels are often elevated after an epileptic seizure, they do not always rise, and the timing of
measurement is crucial, making this a less sensitive test than was previously believed. Other serum markers
have also been used to help distinguish PNES from epileptic seizures, including creatine phosphokinase,
cortisol, WBC counts, lactate dehydrogenase, pCO2, and neuron-specific enolase. These also are not
reliable, as threshold levels for abnormality, sensitivity, and specificity have not been determined.

MRI is not reliable because abnormal brain MRIs have been documented in as many as one-third of patients
with PNES. In addition, patients with epileptic seizures often have normal brain MRIs.

Ref: Alsaadi TM, Marquez AV: Psychogenic nonepileptic seizures. Am Fam Physician 2005;72(5):849-856. 2) Ropper AH,
Samuels MA, Klein JP: Adams and Victors Principles of Neurology, ed 10. McGraw-Hill, 2014, p 332. 3) Goldman L,
Schafer AI (eds): Goldmans Cecil Medicine, ed 25. Elsevier Saunders, 2016, p 2403.

44
Item 123

ANSWER: D

In an asplenic patient, fever might be an initial manifestation of a catastrophic infection and must be treated
immediately with a parenteral antibiotic agent. Quick administration of antibiotics might prevent sepsis.
Mortality can be as high as 50% among patients with postsplenectomy sepsis. Intravenous or intramuscular
ceftriaxone is recommended for patients who have normal laboratory test results and who do not appear
ill. If the patient lives more than 2 hours from a medical facility that can administer parenteral antibiotic
therapy, oral antibiotics should be given.

Ref: Rubin LG, Schaffner W: Care of the asplenic patient. N Engl J Med 2014;371(4):349-356.

Item 124

ANSWER: D

This child has atypical Kawasaki disease. The diagnostic criteria for classic Kawasaki disease is fever for
at least 5 days and at least four of five principal clinical features. The clinical features include:

Changes of the oral cavity and lips: cracked erythematous lips, strawberry tongue
Polymorphous rash: maculopapular, erythema multiformelike or scarlitiniform rash, involving the
extremities, trunk, and perineal regions
Bilateral nonpurulent conjunctivitis
Changes in the extremities (erythema of the hands and feet, desquamation of the hands and toes in
weeks 2 and 3)
Cervical lymphadenopathy (>1.5 cm in diameter and generally unilateral)

Some patients do not meet the classic criteria but are labeled as having incomplete or atypical disease.
While Kawasaki disease is generally a clinical diagnosis and there are no specific diagnostic tests,
supplemental laboratory testing can help in the diagnosis of these atypical cases. The supplemental
laboratory criteria include:

Anemia
Cerebrospinal fluid pleocytosis
Elevated C-reactive protein and erythrocyte sedimentation rate
Elevated liver enzymes
Hypoalbuminemia
Hyponatremia
Platelets >450,000/mm3 after 5 weeks
Sterile pyuria
WBCs 15,000/mm3

45
Patients who have a fever for 5 days or more and two or three of the classic criteria should be treated for
atypical Kawasakis disease if the C-reactive protein level is elevated and they have three or more
associated laboratory abnormalities. The treatment of choice is IVIG and high-dose aspirin to reduce the
risk of coronary abnormalities. Corticosteroids have been used as an adjunct in refractory cases or with
IVIG, but not alone.

Ref: Saguil A, Fargo M, Grogan S: Diagnosis and management of Kawasaki disease. Am Fam Physician 2015;91(6):365-371.

Item 125

ANSWER: D

This patient has an acute migraine headache that did not respond to NSAIDs. Metoclopramide is an
effective treatment for migraine beyond its antiemetic benefit (SOR B) and intravenous administration may
be helpful for the patient unable to tolerate oral medications. Acetaminophen has not been proven to be
effective for migraine. Opiates and barbiturate-containing medications should only be used for patients who
have failed multiple other treatments (SOR C). Supplemental oxygen has shown efficacy in the treatment
of cluster headaches. Triptan medications would be another evidence-based choice for abortive therapy of
migraine.

Ref: Gilmore B, Michael M: Treatment of acute migraine headache. Am Fam Physician 2011;83(3):271-280.

Item 126

ANSWER: D

This patient presents with mild symptoms of hyperandrogenism. Her initial laboratory results rule out
pregnancy, thyroid disorders, and primary ovarian failure. The follow-up laboratory evaluation indicates
significant hyperandrogenism. The rapid onset and high testosterone level suggest an ovarian or adrenal
tumor that should be evaluated by abdominal/pelvic imaging.

MRI of the brain is not helpful in evaluating hyperandrogenism. It would be appropriate in evaluating
hypothalamic and pituitary causes of secondary amenorrhea such as the female athlete triad or other causes
of stress and malnutrition that are associated with weight loss. A karyotype would be helpful in identifying
the cause of primary amenorrhea. An estradiol level is not helpful in evaluating hyperandrogenism. A
dexamethasone suppression test aids in the diagnosis of Cushing syndrome, which generally does not cause
amenorrhea and is usually associated with stigmata of hypercortisolism, which this patient does not have.

Ref: Rotterdam ESHRE/ASRM-Sponsored PCOS Consensus Workshop Group: Revised 2003 consensus on diagnostic criteria
and long-term health risks related to polycystic ovary syndrome. Fertil Steril 2004;81(1):19-25. 2) Azziz R, Carmina E,
Dewailly D, et al: Positions statement: Criteria for defining polycystic ovary syndrome as a predominantly hyperandrogenic
syndrome: An Androgen Excess Society guideline. J Clin Endocrinol Metab 2006;91(11):4237-4245. 3) Klein DA, Poth
MA: Amenorrhea: An approach to diagnosis and management. Am Fam Physician 2013;87(11):781-788.

46
Item 127

ANSWER: A

Recent studies suggest that financial exploitation is emerging as the most prevalent form of elder abuse.
By the time cases are detected the older adults financial resources have often been drastically reduced, so
early detection and intervention are critical. Financial exploitation of older adults, which was explored only
minimally in previous studies, has recently been identified as a virtual epidemic and is a problem that may
be detected or suspected by an alert physician.

Ref: Lachs MS, Pillemer KA: Elder abuse. N Engl J Med 2015;373(20):1947-1956.

Item 128

ANSWER: B

The diagnosis of myalgic encephalomyelitis/chronic fatigue syndrome (ME/CFS) is made by determining


that a patients symptoms are consistent with a standard definition of this disorder and by ruling out the
presence of other systemic diseases that can result in similar symptoms. Currently suggested criteria for
ME/CFS require at least 6 months of pathologic fatigue, along with a combination of other symptoms
including post-exertional fatigue, impaired focus/concentration, orthostatic intolerance, and unrefreshing
sleep. Other symptoms frequently encountered in patients with this condition include headache, diffuse
myalgias, tender lymph nodes, and gastrointestinal or genitourinary impairments. The diagnosis of ME/CFS
should not be considered in patients with red flag symptoms of a potentially more severe condition, such
as chest pain, focal neurologic deficits, joint erythema/swelling, enlarged lymph nodes, or shortness of
breath.

Ref: Yancey JR, Thomas SM: Chronic fatigue syndrome: Diagnosis and treatment. Am Fam Physician 2012;86(8):741-746.
2) IOM (Institute of Medicine) Committee on the Diagnostic Criteria for Myalgic Encephalitis/Chronic Fatigue Syndrome:
Beyond Myalgic Encephalomyelitis/Chronic Fatigue Syndrome: Redefining an Illness. The National Academies Press, 2015,
pp 213-225.

Item 129

ANSWER: C

The American Academy of Pediatrics (AAP) recommends nebulized hypertonic saline for infants and
children hospitalized with bronchiolitis (SOR B). They do not recommend nebulized hypertonic saline for
infants in the emergency department with a diagnosis of bronchiolitis.

The AAP guideline also recommends that clinicians do not administer albuterol or epinephrine to infants
and children with a diagnosis of bronchiolitis (SOR B). In addition, clinicians should not administer
systemic corticosteroids to infants with a diagnosis of bronchiolitis in any setting (SOR A).

Ref: Ralston SL, Lieberthal AS, Meissner HC, et al: Clinical practice guideline: The diagnosis, management, and prevention
of bronchiolitis. Pediatrics 2014;134(5):e1474-e1502.

47
Item 130

ANSWER: E

Pituitary adenomas are the most common disorder of the pituitary gland and are responsible for 10%15%
of all intracranial masses. They present with symptoms of hormone secretion or a neurologic mass effect,
or as an incidental finding on CT/MRI. Premenopausal women often experience amenorrhea, while the
most common neurologic symptoms are headache and vision changes (classically bitemporal hemianopsia)
due to compression of the optic chiasm.

Tumors secreting prolactin (lactotrophs) are the most common, comprising 40%57% of all pituitary
adenomas. Nonsecreting tumors make up 28%37%, growth hormonesecreting adenomas (somatotrophs)
11%13%, and ACTH-secreting adenomas (corticotrophs) 1%2%. FSH-, LH-, and TSH-secreting tumors
are rare. ACTH-secreting tumors result in an increase in circulating cortisol, and the diagnosis is confirmed
by 24-hour urine cortisol, late-night salivary cortisol, or overnight dexamethasone suppression testing.

Gonadotrophs cause a resulting elevation of FSH and LH. Lactotrophs increase serum prolactin, and
somatotrophs increase serum insulin-like growth factor. Thyrotrophs result in normal TSH with low free
T4.

Ref: Lake MG, Krook LS, Cruz SV: Pituitary adenomas: An overview. Am Fam Physician 2013;88(5):319-327. 2) Hitzeman
N, Cotton E: Incidentalomas: Initial management. Am Fam Physician 2014;90(11):784-789.

Item 131

ANSWER: B

Multiple studies, including randomized, controlled trials, have compared standard access to emergency
contraception (EC) with advance provision in which the patient is given a prescription for the EC and
encouraged to have it filled in order to have it immediately available in case of unprotected intercourse. In
multiple populations in the United States and Europe advance provision compared to standard access has
been shown to increase the rate of use of EC and to reduce the interval between intercourse and use of EC.

However, no change has been shown for several outcomes when advance provision was compared to
standard access to EC. These include rates of sexually transmitted infections, unprotected intercourse, use
of routine contraceptives prior to sexual activity, and pregnancy within the population studied.

Ref: American Academy of Pediatrics Committee on Adolescence: Emergency contraception. Pediatrics 2012;130(6):1174-1182.
2) ACOG Committee Opinion Number 542: Access to emergency contraception. Obstet Gynecol 2012;120(5):1250-1253.

48
Item 132

ANSWER: E

Age-related physiologic differences such as wider pulse pressure should be a consideration when treating
hypertension in the elderly. These patients also have lower cardiac output, higher peripheral resistance,
lower intravascular volume, and lower renal blood flow compared with younger patients. Pulse pressure
(the difference between systolic blood pressure and diastolic blood pressure) is a measure of the degree of
age-related vascular stiffness and is a risk factor for coronary artery disease events.

With increasing age the strongest predictor of coronary artery disease gradually shifts from diastolic blood
pressure to systolic blood pressure, and then to pulse pressure. Systolic blood pressure rises gradually
throughout adult life, whereas diastolic blood pressure peaks and plateaus in late middle age, and declines
slightly thereafter. Diastolic hypertension occurs in <10% of all patients with hypertension after age 70.

Ref: Aronow WS, Fleg JL, Pepine CJ, et al: ACCF/AHA 2011 expert consensus document on hypertension in the elderly: A
report of the American College of Cardiology Foundation Task Force on Clinical Expert Consensus documents developed
in collaboration with the American Academy of Neurology, American Geriatrics Society, American Society for Preventive
Cardiology, American Society of Hypertension, American Society of Nephrology, Association of Black Cardiologists, and
European Society of Hypertension. J Am Coll Cardiol 2011;57(20):2037-2114. 2) Chaudhry KN, Chavez P, Gasowski
J, et al: Hypertension in the elderly: Some practical considerations. Cleve Clin J Med 2012;79(10):694-704.

Item 133

ANSWER: D

Asymptomatic microscopic hematuria is defined by the American Urological Association (AUA) as 3


RBCs/hpf in the absence of an obvious cause such as menstruation, infection, vigorous exercise, renal
disease, trauma, a recent urologic procedure, or a viral illness. Urine microscopy is required to confirm
hematuria found on a dipstick examination.

This patient has risk factors for urothelial cancer, including smoking, his age, and his sex. In a patient with
no obvious cause for hematuria, the AUA does NOT recommend repeating the urinalysis or treating
empirically with antibiotics, as this may delay the diagnosis of cancer. In addition, assuming that benign
prostatic hyperplasia (BPH) is the cause for his hematuria is inadvisable; patients with BPH usually also
have risk factors for malignancy. The recommended initial workup includes renal function testing, CT
urography, and cystoscopy.

Ref: Sharp VJ, Lee DK, Askeland EJ: Urinalysis: Case presentations for the primary care physician. Am Fam Physician
2014;90(8):542-547.

49
Item 134

ANSWER: A

Moderate evidence of lower risk from randomized, controlled trials supports the use of low molecular
weight heparin over unfractionated heparin for treatment of venous thromboembolism (VTE) in pregnancy.
Aspirin is not a first-line treatment for VTE, as it crosses the placenta and has a weak association with
miscarriage. Clopidogrel is not indicated for treatment of VTE. Rivaroxaban and other new nonvitamin
K oral anticoagulants are not recommended because of the lack of data regarding their use in pregnancy.
Warfarin crosses the placenta and is associated with fetal hemorrhage and loss.

Ref: Bates SM, Greer IA, Middeldorp S, et al: VTE, thrombophilia, antithrombotic therapy, and pregnancy: Antithrombotic
Therapy and Prevention of Thrombosis, 9th ed: American College of Chest Physicians Evidence-Based Clinical Practice
Guidelines. Chest 2012;141(2 Suppl):e691S-e736S. 2) Marshall AL: Diagnosis, treatment, and prevention of venous
thromboembolism in pregnancy. Postgrad Med 2014;126(7):25-34.

Item 135

ANSWER: A

S-adenosyl-L-methionine is found in all human cells and assists in producing a wide range of compounds
such as cartilage and neurotransmitters. It has been shown to be as effective as celecoxib in relieving joint
pain, but may take up to 2 months to take effect. Glucosamine and chondroitin have not been shown to be
effective. B-complex vitamins may have some benefit in diabetic and alcoholic neuropathy. "-Lipoic acid
and gamma-linolenic acid have also been shown to improve symptoms of neuropathy.

Ref: Chang K-L, Fillingham R, Hurley RW, Schmidt S: Chronic Pain Management. FP Essentials, no 432, May 2015, pp
21-26. 2) Onysko M, Legerski P, Potthoff J, Erlandson M: Targeting neuropathic pain: Consider these alternatives. J Fam
Pract 2015;64(8):470-475.

Item 136

ANSWER: E

This patient requires hospitalization based on her CURB-65 score of 3 (age >65, diastolic blood pressure
<60 mm Hg, respiratory rate >30/min, BUN <19 mg/dL, no confusion), which places her mortality risk
at 14%. Although azithromycin has been associated with an increased risk of myocardial infarction in
elderly patients hospitalized with community-acquired pneumonia, the combination of azithromycin with
a $-lactam has been associated with decreased mortality in this population. Azithromycin alone is acceptable
treatment in the outpatient setting, but not when the patient requires hospitalization. A macrolide plus a
$-lactam antibiotic has been shown to have a lower 30-day mortality rate than a $-lactam alone. This
combination also results in a greater proportion of hospitalized patients achieving clinical stability at 7 days
(defined as stable vital signs and oxygen saturation >90% on room air) when compared with $-lactam
therapy alone.

50
Ref: Aujesky D, Auble TE, Yealy DM, et al: Prospective comparison of three validated prediction rules for prognosis in
community-acquired pneumonia. Am J Med 2005;118(4):384-392. 2) Rodrigo C, Mckeever TM, Woodhead M, Lim WS;
British Thoracic Society: Single versus combination antibiotic therapy in adults hospitalized with community acquired
pneumonia. Thorax 2013;68(5):493-495. 3) Garin N, Genn D, Carballo S, et al: $-Lactam monotherapy vs $-lactam-
macrolide combination treatment in moderately severe community-acquired pneumonia: A randomized noninferiority trial.
JAMA Intern Med 2014;174(12):1894-1901. 4) Mortensen EM, Halm EA, Pugh MJ, et al: Association of azithromycin
with mortality and cardiovascular events among older patients hospitalized with pneumonia. JAMA
2014;311(21):2199-2208.

Item 137

ANSWER: C

Pertussis has been increasing in incidence. The initial presentation usually involves nonspecific symptoms
including malaise, lacrimation, and rhinorrhea, which is referred to as the catarrhal stage. The following
stage, known as the paroxysmal stage, is manifested by severe coughing that may lead to the characteristic
high-pitched whooping sound when the patient tries to catch his or her breath. Coughing to the point of
emesis is also characteristic of pertussis, and the cough can be severe enough to actually result in rib
fractures. The cough may last several weeks before it begins to wane during the convalescent phase, which
usually lasts 23 weeks.

Pertussis can be diagnosed clinically if there is a coughing illness of 2 weeks duration with one of the
classic signs of pertussis (post-tussive emesis, respiratory whoop, or paroxysmal cough) and there is no
other apparent cause. The CDC also recommends the use of both cultures and polymerase chain reaction
testing to confirm the diagnosis. Culture is not the best choice, however, as it is often done improperly,
and culture results may not be available for several days. Polymerase chain reaction has the advantage of
providing results in 12 days. It has good specificity and the sensitivity is much higher than that of a
culture; its sensitivity is highest during the first 2 weeks of symptoms.

Treatment with antibiotics usually does not improve clinical symptoms, although it does decrease
transmission. The CDC continues to recommend antibiotics for pertussis. Either azithromycin or
clarithromycin is currently recommended.

Ref: Kline JM, Lewis WD, Smith EA, et al: Pertussis: A reemerging infection. Am Fam Physician 2013;88(8):507-514.

Item 138

ANSWER: B

Screening all infants for developmental dysplasia of the hip (DDH) has been a mainstay of care for many
years. DDH is an abnormality of the acetabulum or femoral head and their congruence, presenting in
infancy. The prevalence of instability on examination ranges from 1.628.5 per 1000 births. While many
screening measures may be helpful in identifying DDH, none has been found to improve long-term clinical
outcomes (SOR C). The U.S. Preventive Services Task Force found insufficient evidence to recommend
routine screening to prevent poor outcomes. However, the American Academy of Pediatrics and the
Pediatric Orthopedic Society of North America both recommend that the physical examination of all
newborns include screening for DDH. There are no studies recommending the abandonment of this
screening.

51
Ortolani (reducing a dislocated hip) and Barlow (dislocating an unstable hip) maneuvers are commonly
performed early in infancy. By 23 months of age these are less useful and limited hip abduction assessment
is more acceptable (SOR C). A clunk denotes a positive finding. Leg length measurement is useful in the
evaluation of a child presenting with a limp but is not indicated in DDH.

More than 50% of patients with positive physical findings have been found to have normal hips within 1
month on follow-up ultrasonography. Universal ultrasonography resulted in a higher rate of detected DDH
with subsequent treatment but did not reduce the need for surgery. Most of these abnormalities resolve
spontaneously, and ultrasonography should not be used as a universal screening measure (SOR C). Plain
films are inappropriate in infancy because the hip is primarily cartilaginous and the associated radiation
exposure is unacceptable. It may be useful in older children to track progress after treatment.

Ref: Ewald E, Kiesel E: Screening for developmental dysplasia of the hip in newborns. Am Fam Physician 2013;87(1):10-11.
2) Lewis ML: A comprehensive newborn examination: Part II. Skin, trunk, extremities, neurologic. Am Fam Physician
2014;90(5):297-302. 3) Jackson JC, Runge MM, Nye NS: Common questions about developmental dysplasia of the hip.
Am Fam Physician 2014;90(12):843-850. 4) Naranje S, Kelly DM, Sawyer JR: A systematic approach to the evaluation
of a limping child. Am Fam Physician 2015;92(10):908-918.

Item 139

ANSWER: A

The American Academy of Pediatrics now recommends early introduction of peanut-containing products
for most children to reduce the incidence of peanut allergy. For children at high risk for an allergic reaction
(e.g., those with severe eczema or a first degree relative with peanut allergy) allergy testing can be
considered first. For this child, it should be recommended that the mother introduce peanut-containing
products soon. There is no evidence that testing a parent prior to introducing the child to peanut-containing
products is beneficial.

Ref: Fleischer DM, Sicherer S, Greenhawt M, et al: Consensus communication on early peanut introduction and the prevention
of peanut allergy in high-risk infants. Pediatrics 2015;136(3):600-604.

Item 140

ANSWER: A

Smoking is the strongest modifiable risk factor for the development of an abdominal aortic aneurysm.
Nonmodifiable risk factors include older age, male sex, and a family history of the problem. Other less
prominent risk factors include hypertension, an elevated cholesterol level, obesity, and preexisting
atherosclerotic occlusive disease.

Ref: Kent KC: Abdominal aortic aneurysms. N Engl J Med 2014;371(22):2101-2108.

52
Item 141

ANSWER: B

Prolongation of the QT interval is an important medication adverse effect to consider. This is particularly
true in patients taking multiple medications, because this effect can be additive and increases the risk of
life-threatening arrhythmias such as torsades de pointes. Among commonly used antidepressants, citalopram
and escitalopram may prolong the QT interval. Other SSRIs, as well as bupropion, venlafaxine, and
mirtazapine, do not have this effect. Both tricyclic antidepressants and antipsychotics, commonly used in
patients also taking SSRIs, can cause QT prolongation, making their combined use problematic.

Ref: Muench J, Hamer AM: Adverse effects of antipsychotic medications. Am Fam Physician 2010;81(5):617-622. 2) Price
AL, Marzani-Nissen GR: Bipolar disorders: A review. Am Fam Physician 2012;85(5):483-493. 3) Kovich H, DeJong A:
Common questions about the pharmacologic management of depression in adults. Am Fam Physician 2015;92(2):94-100.

Item 142

ANSWER: B

Antibiotics shorten the course of moderate to severe travelers diarrhea. Azithromycin is recommended as
self-treatment for moderate to severe travelers diarrhea in South and Southeast Asia, where Campylobacter
species are a more common cause of the illness than anywhere else. Campylobacter species are resistant
to fluoroquinolones. Ciprofloxacin is recommended for travel to South and Central America and to Africa.
When symptoms are mild (13 loose bowel movements per 24 hours without limiting activities), travelers
diarrhea can be treated with loperamide or bismuth subsalicylate. Rifampin and doxycycline are not
commonly prescribed for presumptive treatment of travelers diarrhea.

Ref: Steffen R, Hill DR, DuPont HL: Travelers diarrhea: A clinical review. JAMA 2015;313(1):71-80. 2) Freedman DO,
Chen LH, Kozarsky PE: Medical considerations before international travel. N Engl J Med 2016;375(3):247-260.

Item 143

ANSWER: B

Secondary hyperparathyroidism is most likely due to low vitamin D intake or low serum vitamin D, often
in the setting of renal disease. However, the serum vitamin D level and estimated glomerular filtration rates
are normal in this patient. This rules out secondary hyperparathyroidism, as well as vitamin D deficiency
and renal insufficiency. Primary hyperparathyroidism is more likely in the presence of hypercalcemia and
inappropriately normal or high parathyroid hormone levels. Familial hypocalciuric hypercalcemia is ruled
out by a normal 24-hour urine calcium level.

Ref: Marcocci C, Cetani F: Primary hyperparathyroidism. N Engl J Med 2011;365(25):2389-2397. 2) Michels TC, Kelly KM:
Parathyroid disorders. Am Fam Physician 2013;88(4):249-257.

53
Item 144

ANSWER: B

In order to facilitate the most effective interview, the interpreter should be as inconspicuous as possible.
This is best achieved by having the interpreter seated next to or slightly behind the patient.

Ref: Juckett G, Unger K: Appropriate use of medical interpreters. Am Fam Physician 2014;90(7):476-480.

Item 145

ANSWER: A

Phimosis is the inability of the foreskin (prepuce) to retract over the glans and it can be physiologic or
pathologic. Nonretractile foreskin is very common in young boys, and is seen in up to 10% of
uncircumcised 3-year-old boys. This physiologic phimosis is part of normal development and over time the
foreskin will become retractile due to intermittent erections and keratinization of the inner foreskin.
Pathologic phimosis is due to distal scarring and on examination typically appears as a white, contracted
fibrotic ring around the preputial outlet. Pathologic phimosis, painful erections with a tight foreskin,
recurrent bouts of balanitis, and recurrent urinary tract infections in conjunction with phimosis are
indications for urologic consultation and consideration of circumcision. This childs examination is
consistent with physiologic phimosis.

A short course of a topical corticosteroid (28 weeks of 0.05% betamethasone twice daily) applied to the
preputial outlet may result in accelerated resolution of physiologic phimosis. However, given this childs
age and the absence of complications, reassurance and continued good foreskin hygiene are recommended.
The foreskin should not be forcibly retracted, as this may lead to microtears and resultant scarring. In the
absence of infection, neither antibacterial nor antifungal ointment is indicated.

Ref: McGregor TB, Pike JG, Leonard MP: Pathologic and physiologic phimosis: Approach to the phimotic foreskin. Can Fam
Physician 2007;53(3):445-448. 2) Drake T, Rustom J, Davies M: Phimosis in childhood. BMJ 2013;346:f3678.

Item 146

ANSWER: C

This patient has sarcoidosis. Extrapulmonary manifestations are common in patients with sarcoidosis and
erythema nodosum is a common cutaneous sign. Ocular symptoms usually include uveitis, not
conjunctivitis. Xerostomia is associated with Sjgrens syndrome. A malar rash and alopecia are
dermatologic findings associated with lupus erythematosus.

Ref: Soto-Gomez N, Peters JI, Nambiar AM: Diagnosis and management of sarcoidosis. Am Fam Physician
2016;93(10):840-848.

54
Item 147

ANSWER: A

The treatment of diabetes mellitus in frail elderly patients, especially nursing home residents, can be less
stringent than with other patients. Sliding-scale insulin and diabetic diets should both be avoided in nursing
home residents. Lowering LDL-cholesterol levels and aggressive blood pressure control are not indicated
for frail elderly patients. The acceptable levels of hemoglobin A1c can also be liberalized, with levels of
8%9% being acceptable.

Ref: Sinclair A, Morley JE: How to manage diabetes mellitus in older persons in the 21st century: Applying these principles
to long term diabetes care. J Am Med Dir Assoc 2013;14(11):777-780.

Item 148

ANSWER: B

Monoclonal gammopathy of undetermined significance (MGUS) is present in approximately 2%3% of the


white population older than 50. It is associated with a risk of progression to multiple myeloma at a rate of
1% per year. Most patients diagnosed with MGUS should be reevaluated in 6 months with a medical
history, physical examination, CBC, calcium and creatinine levels, and serum electrophoresis, and then
annually thereafter.

Ref: Rajkumar SV, Kyle RA: Protein electrophoresis and immunofixation for the diagnosis of monoclonal gammopathies. JAMA
2014;312(20):2160-2161.

Item 149

ANSWER: D

Isolated elevated calcium levels should be confirmed before pursuing further testing. After calcium
elevation is confirmed, immediate treatment should be undertaken if hypercalcemia is severe. Otherwise,
a history and physical examination would be appropriate, as well as 25-hydroxyvitamin D, magnesium,
creatinine, and PTH levels. In patients with a normal or elevated PTH level, 24-hour urine calcium and
creatinine levels can help to differentiate between primary hyperparathyroidism and familial hypocalciuric
hypercalcemia. If the PTH level is low, 25-hydroxyvitamin D, 1,25-dihydroxyvitamin D, and parathyroid
hormonerelated peptide levels should be checked to evaluate possible causes of hypercalcemia independent
of the parathyroid. Sestamibi scintigraphy is indicated only after confirmation of hyperparathyroidism, and
typically in anticipation of surgical treatment.

Ref: Michels TC, Kelly KM: Parathyroid disorders. Am Fam Physician2013;88(4):249-257.

55
Item 150

ANSWER: E

Phalangeal fractures of the second to fifth toes can usually be managed with buddy taping and a rigid-sole
shoe for 3 weeks followed by buddy taping for another 3 weeks. If the fracture is significantly displaced,
closed reduction in the office using local anesthesia may be appropriate.

However, because of its importance in weight bearing and balance, fractures of the first toe (hallux) have
a higher potential for negative outcomes. Specifically, fractures of the hallux can be managed nonsurgically
if they are not displaced and involve less than 25% of the articular surface of the joint. Nonsurgical
management involves use of a short leg walking cast with a toe plate (extending past the end of the great
toe) for 3 weeks and then progression to a rigid-sole shoe with buddy taping after that.

Since this patients fracture involves more than 25% of the articular surface of the joint of the first toe,
early referral for surgical pinning by an orthopedic surgeon would lead to the best outcome.

Ref: Marx JA, Hockberger RS, Walls RM (eds): Rosens Emergency Medicine: Concepts and Clinical Practice, ed 8. Elsevier
Saunders, 2014, pp 723-750. 2) Bica D, Sprouse RA, Armen J: Diagnosis and management of common foot fractures.
Am Fam Physician 2016;93(3):183-191.

Item 151

ANSWER: E

Sedative hypnotics such as zolpidem are associated with increased delirium symptoms and should not be
used as first-line therapy (SOR C). Behavioral interventions such as early and recurrent mobilization,
continuous observation, improving sensory perception (such as with hearing aids), initial reorientation, and
removal of unnecessary irritations such as intravenous lines and catheters have been shown to decrease
symptoms of delirium (SOR B). While opioids can affect mental status, untreated pain is also associated
with delirium, and opioids are likely indicated in this postoperative patient. Antipsychotic medications such
as haloperidol, as well as physical restraints, should be used as last resorts and only after an appropriate
medical evaluation (SOR A).

Ref: Kalish VB, Gillham JE, Unwin BK: Delirium in older persons: Evaluation and management. Am Fam Physician
2014;90(3):150-158.

Item 152

ANSWER: A

This child has monosymptomatic enuresis, meaning there are no other symptoms except nighttime bed
wetting. His physical examination is normal. In this setting, the recommended initial workup is a urinalysis
alone. If the results are normal and there are no other symptoms suggestive of underlying behavioral or
medical conditions, reassurance and instruction on possible behavioral interventions is appropriate without
further evaluation.

Ref: Baird DC, Seehausen DA, Bode DV: Enuresis in children: A case-based approach. Am Fam Physician 2014;90(8):560-568.

56
Item 153

ANSWER: B

The American College of Physicians recently published clinical practice guidelines for the treatment of
pressure ulcers. Many modalities were evaluated, but good studies and evidence were available only for
a few treatments. Several good-quality studies indicate that protein supplementation improves wound
healing and also appears to have some benefit in prevention as well. Vitamin C supplementation made no
difference in wound healing. Dextranomer paste applied to wounds actually worsened wound healing
compared to standard wound dressings. Topical collagen dressings and negative-pressure wound therapy
yielded no improvement in wound healing compared to usual care.

Ref: Qaseem A, Humphrey LL, Forciea MA, et al: Treatment of pressure ulcers: A clinical practice guideline from the
American College of Physicians. Ann Intern Med 2015;162(5):370-379.

Item 154

ANSWER: A

A simple abscess with no overlying cellulitis that is treated with incision and drainage does not require
antibiotics. If there were overlying cellulitis, antibiotic use would be appropriate. Purulent infections have
a higher likelihood of being caused by MRSA, so coverage with trimethoprim/sulfamethoxazole or
clindamycin would be appropriate. In nonpurulent skin infections, amoxicillin/clavulanate or cephalexin
would be appropriate choices.

Ref: Ramakrishnan K, Salinas RC, Agudelo Higuita NI: Skin and soft tissue infections. Am Fam Physician 2015;92(6):474-483.

Item 155

ANSWER: A

Pain in fibromyalgia is thought to be due to a centralized pain state. Medications that reduce the activity
of neurotransmitters or increase the activity of inhibitory neurotransmitters such as norepinephrine and
serotonin work best, and tricyclic antidepressants appear to be most effective (SOR A). Older SSRIs have
limited benefit. Oral analgesics only work in about one-third of patients, and opioids may increase
hyperalgesia. Corticosteroids are ineffective.

Ref: Clauw DJ: Fibromyalgia: A clinical review. JAMA 2014;311(15):1547-1555.

Item 156

ANSWER: E

Systolic heart failure, or heart failure with a reduced left ventricular ejection fraction (<40%), should be
managed with an ACE inhibitor and a $-blocker. If there is volume overload, a diuretic should be added.
For those who cannot tolerate an ACE inhibitor, an angiotensin receptor blocker can be used. However,
an ACE inhibitor should not be given with an angiotensin receptor blocker, as this combination increases
mortality.

57
Digoxin will decrease symptoms and lessen the chance of hospitalization, but mortality is not reduced.
Mortality and hospitalization can be reduced by the addition of an aldosterone antagonist such as
spironolactone. In African-American patients, using a combination of hydralazine and isosorbide dinitrate
improves both symptoms and mortality for those with New York Heart Association class III or IV heart
failure with a reduced ejection fraction.

Ref: Drugs for chronic heart failure. JAMA 2015;313(10):1052-1053.

Item 157

ANSWER: B

Healthy full-term infants receive 60%80% of their iron stores from their mothers during the third trimester
of pregnancy. Thus, even an exclusively breastfed infant will not typically be at risk of developing iron
deficiency anemia until the age of 4 months. Iron supplementation should then be started and continued until
the child is eating foods containing sufficient dietary iron. Preterm babies who are exclusively breastfed
should begin iron supplementation at 1 month of age due to a foreshortened third trimester.

Ref: Wang M: Iron deficiency and other types of anemia in infants and children. Am Fam Physician 2016;93(4):270-278.

Item 158

ANSWER: D

Nephrotic syndrome is the triad of edema, proteinuria, and hypoalbuminemia, and it usually is also
associated with hyperlipidemia. The initial evaluation of this patient should include testing for
hyperlipidemia. Nephrotic syndrome is associated with an increased risk of deep vein thrombosis but does
not cause a coagulopathy. Diabetes mellitus can be a secondary cause of nephrotic syndrome but is not a
finding in primary nephrotic syndrome. Relative hypocalcemia may be found in patients with nephrotic
syndrome due to low protein, but hypercalcemia is not associated with nephrotic syndrome. Children and
patients with relapses of nephrotic syndrome have an increased risk of infection.

Ref: Kodner C: Diagnosis and management of nephrotic syndrome in adults. Am Fam Physician 2016;93(6):479-485.

Item 159

ANSWER: C

Low vitamin B12 is seen with prolonged use of metformin, H2-blockers, and proton pump inhibitors. The
mechanism is unclear, and it is not known whether prophylactic supplementation prevents deficiency.
Metformin reduces absorption of vitamin B12 in 30% of patients and reduces vitamin B12 concentrations in
5%10% of those taking it. The dosage and duration of metformin therapy is correlated with the risk of
vitamin B12 deficiency. Given this risk, it seems prudent to monitor vitamin B12 levels periodically in
patients taking metformin.

Ref: Langan RC, Zawistoski KJ: Update on vitamin B12 deficiency. Am Fam Physician 2011;83(12):1425-1430.

58
Item 160

ANSWER: B

Clinicians who care for pregnant women need to be familiar with the diagnosis and monitoring parameters
for gestational diabetes mellitus, as these help to determine the need for management strategies outside of
diet and exercise. The goal is 95 mg/dL for fasting blood glucose, 140 mg/dL for 1-hour postprandial
glucose, and 120 mg/dL for 2-hour postprandial glucose.

Ref: Garrison A: Screening, diagnosis, and management of gestational diabetes mellitus. Am Fam Physician
2015;91(7):460-467.

Item 161

ANSWER: B

There is an increasing prevalence of hypertension in children and adolescents. BMI is the strongest risk
factor for hypertension in this age group. Other risk factors include a family history of hypertension, low
birth weight, male sex, and certain ethnic backgrounds. The prevalence of hypertension in obese children
is 11%, which is more than double that of the general pediatric population (reported at 1%5%).

Ref: Moyer VA; US Preventive Services Task Force: Screening for primary hypertension in children and adolescents: US
Preventive Services Task Force recommendation statement. Pediatrics 2013;132(5):907-914.

Item 162

ANSWER: E

Popliteal cysts cause posterior knee pain, and are thought to be a result of increased intra-articular pressure
forcing fluid into the bursa and causing expansion and subsequent pain. Risk factors include rheumatoid
arthritis or osteoarthritis and advanced age. With the patient lying down, a mass can be palpated in the
medial popliteal fossa while the knee is extended. When the knee is flexed to 45 the pain subsides and the
mass is reduced in size or disappears.

Imaging is generally not needed for the diagnosis. Plain radiographs are obtained if there is a possible
fracture. Ultrasonography can be used if deep vein thrombosis is a consideration. If significant internal
derangement of the knee is suspected, MRI may be useful. An arteriogram would be an option if aneurysm
of the popliteal artery were suspected.

Treatment is generally conservative, although addressing the underlying knee pathology will prevent the
cyst from recurring. Pain is often decreased by keeping the knee in flexion and the use of ice and NSAIDs.
If this is not successful, intra-articular corticosteroids or cyst aspiration is often effective.

Ref: Covey CJ, Hawks MK: Nontraumatic knee pain: A diagnostic & treatment guide. J Fam Pract 2014;63(12):720-728.

59
Item 163

ANSWER: C

Overscreening has become increasingly common. This inappropriate care costs the health care system while
providing no benefit and possibly causing harm to patients. All of the listed screenings are appropriate with
the exception of cervical cancer screening for a patient who has had a hysterectomy. The U.S. Preventive
Services Task Force recommends against screening for cervical cancer in women over 65 who have had
adequate recent screening, and in women who have had a hysterectomy for benign disease (SOR A).

Ref: Ebell M, Herzstein J: Improving quality by doing less: Overscreening. Am Fam Physician 2015;91(1):22-24.

Item 164

ANSWER: E

Evidence-based guidelines for the treatment of patients with acute coronary syndrome support several
medications in the subacute period. Dual antiplatelet therapy, such as clopidogrel combined with aspirin,
has been shown to reduce cardiovascular mortality (SOR B). ACE inhibitors and statins should be initiated
immediately after a myocardial infarction and continued indefinitely to reduce mortality and the risk of
repeat infarction (SOR A). $-Blockers have been shown to improve mortality in patients with a left
ventricular ejection fraction <40% (SOR A). Nitroglycerin is often used to manage angina but has no
demonstrated mortality benefit.

Ref: Mercado MG, Smith DK, McConnon ML: Myocardial infarction: Management of the subacute period. Am Fam Physician
2013;88(9):581-588.

Item 165

ANSWER: A

Somatic disorders usually require a multifaceted approach to treatment. It is important to schedule regular
visits at short intervals to establish a collaborative relationship with the patient. It is also important to limit
diagnostic testing and reassure the patient that serious diseases have already been ruled out, and to screen
for other mental illnesses. Referral for counseling using cognitive-behavioral therapy and mindfulness-based
therapies is also effective. SSRIs and tricyclic antidepressants have been found to be the most effective
pharmacotherapy (SOR B) for somatic symptom disorders. Amitripityline is the most studied tricyclic
antidepressant, and trials have shown that it has a greater likelihood of success compared to SSRIs. Other
antidepressants, anticonvulsants, and antipsychotic medications are ineffective and should be avoided (SOR
B).

Ref: Kurlansik SL, Maffei MS: Somatic symptom disorder. Am Fam Physician 2016;93(1):49-54.

60
Item 166

ANSWER: A

This case can be best described as acute lumbar pain with radiculopathy. There are no red flags that would
suggest the presence of a time-sensitive condition such as cauda equina syndrome, cancer, infection, or
fracture. In cases like this, consensus agreements recommend against any imaging studies. Instead, a
6-week course of pain medication, manipulation, physical therapy, and self-care should be undertaken.

Ref: Chou R, Qaseem A, Snow V, et al: Diagnosis and treatment of low back pain: A joint clinical practice guideline from the
American College of Physicians and the American Pain Society. Ann Intern Med 2007;147(7):478-491. 2) Patel ND,
Broderick DF, Burns J, et al; Expert Panel on Neurologic Imaging: ACR Appropriateness Criteria: Low Back Pain.
American College of Radiology, 2015.

Item 167

ANSWER: C

Nausea is a common symptom near the end of life and can have many causes. One of the first objectives
in the palliative care of nausea is to identify and address the underlying cause. This patient is taking a
long-acting opiate that predisposes her to constipation. Furthermore, the family seems to accept the fact
that she is not having any bowel movements because she is not eating very much, which is an incorrect
understanding. There are no signs of dehydration to warrant subcutaneous hydration, nor are there signs
of bowel obstruction that would warrant a nasogastric tube.

Ref: Walsh TD, Caraceni AT, Fainsinger R, et al (eds): Palliative Medicine. Saunders Elsevier, 2009, pp 921-930. 2) Clary
PL, Lawson P: Pharmacologic pearls for end-of-life care. Am Fam Physician 2009;79(12):1059-1065. 3) Rakel RE, Rakel
DP (eds): Textbook of Family Medicine, ed 9. Elsevier Saunders, 2016, pp 61-62.

Item 168

ANSWER: A

Frailty is an age-related, multidimensional state of decreased physiologic reserves. Frail patients are at
increased risk of decline as a result of illness or stressors such as surgery. The definition and assessment
of frailty are still not uniformly agreed upon, but gait speed is recognized as a highly reliable single
measurement tool. The other options listed are useful components of a geriatric assessment but are not used
as a single test for the evaluation of frailty.

Ref: Elsawy B, Higgins KE: The geriatric assessment. Am Fam Physician 2011;83(1):48-56. 2) Robinson TN, Walston JD,
Brummel NE, et al: Frailty for surgeons: Review of a National Institute on Aging Conference on Frailty for Specialists.
J Am Coll Surg 2015;221(6):1083-1092.

61
Item 169

ANSWER: A

Benzoyl peroxide is the only topical antibiotic without evidence to suggest that its use contributes to
bacterial antimicrobial resistance. For this reason it is recommended as first-line therapy in mild to
moderate inflammatory acne and as an adjunctive agent with other topical antibiotics that can induce
bacterial resistance, such as clindamycin and erythromycin. Dapsone is an antibiotic but is thought to
primarily improve acne via its anti-inflammatory effect. Metronidazole is used chiefly in acne rosacea
rather than acne vulgaris and it may contribute to antimicrobial resistance.

Ref: Titus S, Hodge J: Diagnosis and treatment of acne. Am Fam Physician2012;86(8):734-740.

Item 170

ANSWER: A

Peppermint oil is an antispasmodic that has moderate evidence of benefit in the relief of abdominal
cramping in patients with irritable bowel syndrome (IBS). Probiotics as a group have some evidence of
benefit in improving global IBS symptoms, but the studies have used different probiotic preparations so it
is difficult to draw definite conclusions about their effectiveness or to recommend a specific preparation.
Polyethylene glycol is an osmotic laxative that has evidence of benefit for bowel symptoms, including
improvement in stool frequency and consistency, but it does not improve abdominal pain. Wheat bran is
an insoluble fiber that contains fructans that can exacerbate IBS symptoms and should be avoided in patients
with IBS.

Ref: Chey WD, Kurlander J, Eswaran S: Irritable bowel syndrome: A clinical review. JAMA 2015;313(9):949-958.

Item 171

ANSWER: A

The American Academy of Pediatrics recommends that medical care for children with an urgent or
emergent condition never be withheld or delayed because of problems obtaining consent. All efforts to
obtain consent should be documented. In the case of a 4-year-old male who is suspected to have meningitis,
even a brief delay in treatment could result in a bad outcome for the patient.

Ref: Committee on Pediatric Emergency Medicine and Committee on Bioethics: Consent for emergency medical services for
children and adolescents. Pediatrics 2011;128(2):427-433.

62
Item 172

ANSWER: C

This patient presents with the classic symptoms and signs of rosacea. FDA-approved first-line agents for
treating rosacea include metronidazole 0.75% lotion, 0.75% cream, and 1% gel; azelaic acid 15% gel;
sulfacetamide 10%/sulfur 5% cream, foam, lotion, and suspension; brimonidine 0.33% gel; and ivermectin
1% cream (SOR A).

Ref: Oge LK, Muncie HL, Phillips-Savoy AR: Rosacea: Diagnosis and treatment. Am Fam Physician 2015;92(3):187-196.

Item 173

ANSWER: B

Bulimia nervosa is an eating disorder associated with recurrent binge eating and induced vomiting. There
is also a history of excessive physical activity in many cases. Because of vomiting, the teeth are stained and
there is destruction of the enamel from stomach acid. Parotid swelling is also noted frequently. With
mumps and bacterial parotitis the patient feels sick and parotid glands are tender. Cat scratch disease and
mononucleosis affect lymph glands.

Ref: Harrington BC, Jimerson M, Haxton C, Jimerson DC: Initial evaluation, diagnosis, and treatment of anorexia nervosa and
bulimia nervosa. Am Fam Physician 2015;91(1):46-52.

Item 174

ANSWER: C

A new treatment may not be more effective than available treatments but it may have lower costs, fewer
adverse effects, or greater convenience compared to standard treatment. A treatment with other advantages
might be preferred to standard treatment for some patients. A non-inferiority trial rigorously evaluates a
treatment against accepted and effective treatments, with the goal of demonstrating that it is at least almost
as good. Single- and double-blind studies (with or without crossover of the control and experimental
groups) can be done to demonstrate the drug is effective, but showing that a new treatment is not inferior
to standard treatments can be rigorously demonstrated by a non-inferiority trial. A non-inferiority trial does
not distinguish between a new treatment that is not inferior and one that is truly superior, and cannot
demonstrate equivalence. A case-control trial is not appropriate to compare one medication to another; its
retrospective design is better for evaluating risk factors or exposures as possible causes or contributors to
a disease or illness, as it involves no intervention in either the control or the experimental group.

Ref: Kaji AH, Lewis RJ: Noninferiority trials: Is a new treatment almost as effective as another? JAMA
2015;313(23):2371-2372.

63
Item 175

ANSWER: D

Streptococcus species and Staphylococcus aureus are the most common causes of skin and soft-tissue
infections. With a puncture wound caused by an object penetrating the sole of the shoe, another organism
that should be considered as a possible cause of the infection is Pseudomonas aeruginosa. Klebsiella is not
a common cause of these infections. Pasteurella multocida is a common cause of infection from a cat bite,
and can also be seen in dog bites. Corynebacterium is also associated with infections from animal bites.
Vibrio vulnificus should be considered when there is a skin infection after exposure to salt water.

Ref: Marx JA, Hockberger RS, Walls RM (eds): Rosens Emergency Medicine: Concepts and Clinical Practice, ed 8. Elsevier
Saunders, 2014, pp 1851-1863. 2) Ramakrishnan K, Salinas RC, Agudelo Higuita NI: Skin and soft tissue infections. Am
Fam Physician 2015;92(6):474-483.

Item 176

ANSWER: D

If isolated ear anomalies such as cup ears, preauricular skin tags, or ear pits are associated with maternal
gestational diabetes mellitus, ultrasonography is warranted to screen for renal abnormalities. Ear
malformations are also associated with renal abnormalities in children with other malformations,
dysmorphic features, a family history of deafness, or teratogenic exposures (SOR C).

Ref: Lewis ML: A comprehensive newborn exam: Part I. General, head and neck, cardiopulmonary. Am Fam Physician
2014;90(5):289-296.

Item 177

ANSWER: D

According to the U.S. Preventive Services Task Force, screening for major depression is recommended
if adequate treatment and follow-up can be provided. Sexually active females should be screened for
Chlamydia infection. Cervical cancer screening begins at age 21. Because of undue harm from unnecessary
treatment, scoliosis screening is not recommended. There is insufficient evidence to recommend for or
against routine screening for alcohol use (SOR A, B).

Ref: Ham P, Allen C: Adolescent health screening and counseling. Am Fam Physician 2012;86(12):1109-1116.

Item 178

ANSWER: C

Depression can be treated with counseling and/or medication. The choice of medication depends on many
factors including side effects, dosing schedule, cost, patient preference, and comorbidities. In a patient with
significant depression and no comorbidities, a second-generation SSRI should be the initial medication
choice. Fluoxetine is the only SSRI choice listed.

64
Duloxetine is an SNRI that may lead to sweating, tachycardia, urinary retention, and elevation of blood
pressure. It is indicated if the patient has fibromyalgia. Trazodone is associated with somnolence,
orthostatic hypotension, and priapism. It may be used in low doses as an adjuvant in patients with insomnia.
Mirtazapine can cause sedation, weight gain, increased appetite, dry mouth, dizziness, and constipation.
It is helpful for patients with significant insomnia and loss of appetite. Amitriptyline is a tricyclic
antidepressant and has significant anticholinergic side effects. It is also associated with conduction
abnormalities that can lead to arrhythmias.

Ref: Drugs for psychiatric disorders. Treat Guidel Med Lett 2013;11(130):53-64. 2) Kovich H, DeJong A: Common questions
about the pharmacologic management of depression in adults. Am Fam Physician 2015;92(2):94-100.

Item 179

ANSWER: A

An intertriginous rash of brownish-red macules that often coalesce is characteristic of a bacterial infection
caused by Corynebacterium minutissimum known as cutaneous erythrasma. Coral-red fluorescence under
a Woods lamp confirms the diagnosis. The best treatment for erythrasma is oral or topical erythromycin,
with the oral form being most effective. Topical clindamycin and antibacterial soaps may also be of some
benefit.

Ref: Kalra MG, Higgins KE, Kinney BS: Intertrigo and secondary skin infections. Am Fam Physician 2014;89(7):569-573.

Item 180

ANSWER: D

Flow cytometry and immunophenotyping of peripheral blood demonstrate clonality of the circulating
lymphocytes in chronic lymphocytic leukemia (CLL). Absolute lymphocytosis is defined as >5000
cells/mm3.

Ninety percent of CLL patients are asymptomatic at the time of diagnosis. Imaging procedures are not
necessary for establishing the diagnosis. While a bone marrow biopsy or aspiration and/or a lymph node
biopsy may be necessary as the disease progresses, they are not routinely needed to establish a CLL
diagnosis.

Ref: Nabhan C, Rosen ST: Chronic lymphocytic leukemia: A clinical review. JAMA 2014;312(21):2265-2276.

65
Item 181

ANSWER: B

Illicit substance use is the most common medical cause or secondary cause of acute psychosis. Conditions
diagnosed by brain imaging, blood chemistry, and HIV tests (intracranial mass, paraneoplastic syndrome,
and HIV infection, respectively) are less common medical causes of acute psychosis. Major depression with
psychosis would be considered a psychiatric or primary cause of psychosis.

Ref: Griswold KS, Del Regno PA, Berger RC: Recognition and differential diagnosis of psychosis in primary care. Am Fam
Physician 2015;91(12):856-863.

Item 182

ANSWER: B

In the evaluation of a solitary pulmonary nodule, a risk assessment is used to determine the diagnostic
evaluation of the nodule. Clinical risk factors for malignancy include older age, a history of smoking, and
a previous history of cancer. Radiologic features indicative of malignancy include a nodule diameter >10
mm, an upper lung location, irregular or spiculated borders, no calcifications, a ground-glass appearance,
and increasing size over time. Radiologic characteristics suggesting benign disease include a size <5 mm,
central calcifications, smooth borders, a solid appearance, and a stable size over 1 year.

Ref: Kikano GE, Fabien A, Schilz R: Evaluation of the solitary pulmonary nodule. Am Fam Physician 2015;92(12):1084-1091.

Item 183

ANSWER: E

Rapid administration of systemic corticosteroids can reduce the frequency of hospitalization in patients with
severe asthma who present with an exacerbation (SOR A). There is no clear benefit from parenteral versus
oral administration (SOR B). The same benefit is not seen with inhaled corticosteroids during an
exacerbation. Montelukast and cromolyn are not appropriate treatments for an asthma exacerbation.
Nebulized epinephrine is used in the treatment of croup.

Ref: Pollart SM, Compton RM, Elward KS: Management of acute asthma exacerbations. Am Fam Physician 2011;84(1):40-47.

66
Item 184

ANSWER: D

The safest and most effective treatment for postmenopausal atrophic vaginitis is local estrogen such as the
estradiol vaginal ring, estradiol vaginal tablets, or estrogen cream. All of these therapies are effective, and
while there is a theoretical risk of small amounts of systemic absorption, there is no evidence to suggest
that there is an increased risk of endometrial cancer or thromboembolic disease related to this type of
estrogen use. These therapies are therefore preferred over systemic estrogens such as estradiol or a
combination of conjugated estrogens and medroxyprogesterone. Raloxifene, a synthetic estrogen
agonist/antagonist, has not been found to be effective for the treatment of atrophic vaginitis. The vaginal
ring is approved for use as a contraceptive and is not intended for use in postmenopausal patients.

Ref: ACOG Practice Bulletin No. 141: Management of menopausal symptoms. Obstet Gynecol 2014;123(1):202-216.

Item 185

ANSWER: B

Down syndrome is the most common chromosomal abnormality. The 5-year survival rate is >90%, with
most individuals living past 60 years of age, and there has been significant progress in improving quality
of life for these patients. Infants with Down syndrome present with a wide variety of symptoms and signs,
however, and there are certain health complications that can increase the risk of hospitalization.

All infants with Down syndrome should be screened for cardiac, feeding, vision, hearing, thyroid, and
hematologic abnormalities. Up to 50% manifest congenital cardiac defects, and a cardiology consultation
and echocardiography are warranted. Hypotonia may affect feeding and should be noted for assessment.
Congenital cataracts are common, so a red reflex examination is imperative. Cervical spine films, hip
ultrasonography, and polysomnography are not recommended in a newborn with Down syndrome.
However, all children with Down syndrome should have polysomnography between the ages of 1 and 4
years. Due to concerns about atlantoaxial instability parents should be advised to monitor for any signs of
spinal cord impingement.

Screening for subclinical thyroid disease should also be performed, as well as a CBC to evaluate for
transient myeloproliferative disorder, leukemoid reaction, and polycythemia, which all occur more
frequently in Down syndrome patients.

It is important to congratulate the parents on their newborn and carefully explain the rationale for each
recommended test and discuss results and their meaning clearly. An Individual Family Service Plan is also
recommended, with a review every 6 months to evaluate specific measures indicated for each age group.

Ref: Bunt CW, Bunt SK: Role of the family physician in the care of children with Down syndrome. Am Fam Physician
2014;90(12):851-858.

67
Item 186

ANSWER: B

Evidence strongly supports the use of dual antiplatelet therapy with aspirin and a P2Y12 inhibitor such as
clopidogrel for a minimum of 612 months in patients with a drug-eluting stent. Aspirin at a dosage of 81
mg is as effective as 325 mg, if not more effective. It is also associated with a lower bleeding risk.
Prasugrel is a second-line antiplatelet agent and can be used when there is a contraindication to clopidogrel
or if the patient is resistant to clopidogrel. When it is used, it should be used with an aspirin dosage of 81
mg daily. Aspirin/dipyridamole is effective for secondary stroke prevention but does not have a role in the
treatment of ischemic cardiovascular disease.

Ref: Montalescot G, Brieger D, Dalby AJ, et al: Duration of dual antiplatelet therapy after coronary stenting: A review of the
evidence. J Am Coll Cardiol 2015;66(7):832-847.

Item 187

ANSWER: D

Secondary centrally mediated hypothyroidism is caused by hypothalamic or pituitary disease in which TSH
is not being produced, resulting in failure to stimulate the thyroid gland to make thyroid hormone. Standard
treatment for this type of hypothyroidism is levothyroxine, but determining the response to therapy requires
measuring free T4 rather than TSH. Desiccated thyroid USP is a combination of T3 and T4 and response
to treatment is usually assessed with TSH levels. T3 levels can fluctuate in patients treated with desiccated
thyroid, as well as those who are on liothyronine (T3), and are therefore generally not used. TSH is also
used to monitor treatment in patients with surgical hypothyroidism. Graves disease is an autoimmune
disorder commonly causing hyperthyroidism, but patients usually become hypothyroid after treatment. TSH
is used to monitor these patients as well.

Ref: Drugs for hypothyroidism. Med Lett Drugs Ther 2015;57(1480):147-150.

Item 188

ANSWER: C

This patient has severe hyperkalemia and needs urgent treatment. Intravenous insulin followed by glucose
will shift potassium intracellularly and is an effective treatment for severe hyperkalemia (SOR B). Sodium
polystyrene sulfonate is not recommended as an urgent treatment and would not be an initial treatment for
severe hyperkalemia. Intravenous calcium gluconate solution does not lower serum potassium but is
indicated to prevent arrhythmias in patients with hyperkalemia and EKG changes. Intravenous furosemide
is not a treatment for hyperkalemia, although hypokalemia is a common side effect. Hemodialysis is a
treatment for severe hyperkalemia but is not considered a first-line treatment.

Ref: Viera AJ, Wouk N: Potassium disorders: Hypokalemia and hyperkalemia. Am Fam Physician 2015;92(6):487-495.

68
Item 189

ANSWER: B

Croup is a common, self-limited illness caused by viral infection of the upper respiratory tract. The
diagnosis is based primarily on the clinical history and examination findings. A history of the abrupt onset
of a barking cough, inspiratory stridor, and hoarseness in a 2-year-old child is typical of croup, although
it can present at any age between 6 months and 12 years. Low-grade fever, a barking cough, and varying
degrees of respiratory distress (nasal flaring, retractions, or stridor) are typically present on examination.
Findings such as a toxic appearance, wheezing, drooling, and difficulty swallowing are not consistent with
the diagnosis.

Treatment of mild disease with a corticosteroid has proven benefit (SOR A) even when administered as a
single oral dose (SOR B). Nebulized epinephrine has been shown to improve outcomes in children with
moderate to severe croup (SOR A). Humidification therapy in the emergency department setting provides
no benefit (SOR A). This child has mild croup and a single dose of dexamethasone (0.150.60 mg/kg,
usually given orally) followed by close observation is the most appropriate treatment.

Ref: Zoorob R, Sidani M, Murray J: Croup: An overview. Am Fam Physician2011;83(9):1067-1073.

Item 190

ANSWER: C

Intensive glycemic control in type 2 diabetes mellitus results in a reduced rate of progression to
macroalbuminuria, which is an indicator of the development of diabetic nephropathy. Intensive glycemic
control does not reduce the likelihood of dialysis, nor does it improve blood pressure control. Intensive
control does increase the incidence of hypoglycemic events, which is particularly hazardous in the elderly.
As was demonstrated in the Action to Control Cardiovascular Risk in Diabetes (ACCORD) trial, intensive
glycemic control actually increases all-cause mortality.

Ref: Roett MA, Liegl S, Jabbarpour Y: Diabetic nephropathyThe family physicians role. Am Fam Physician
2012;85(9):883-889.

Item 191

ANSWER: C

Porphyria cutanea tarda (PCT) should be suspected in patients who present with blistering lesions on
sun-exposed skin. Factors associated with increased susceptibility to PCT include those that cause liver
damage, such as alcohol use and hepatitis C, as well as smoking, estrogen use, HIV infection, and HFE
mutations. The initial test in suspected PCT should be total serum, plasma, or spot urine porphyrins. The
clinical presentation of this patient is less consistent with cellulitis, which usually presents with erythema,
pain, and warmth, and when severe may lead to fever and leukocytosis.

Ref: Habif TP: Clinical Dermatology: A Color Guide to Diagnosis and Therapy, ed 6. Elsevier, 2016, pp 407-410.

69
Item 192

ANSWER: D

Psychostimulants are the most effective medication option for treating attention-deficit/hyperactivity
disorder (ADHD) and are widely accepted as first-line therapy. Behavioral therapy is recommended before
considering medications for children under the age of 6, and may be considered for older children but may
not add much benefit in this age group, particularly when there is a good response to medication.
Atomoxetine and the "2-receptor agonist guanfacine are considered second-line agents because of lower
effectiveness against core ADHD symptoms and a greater frequency of side effects. Bupropion has been
used off-label for ADHD but is not FDA approved for this indication.

Ref: Felt BT, Biermann B, Christner JG, et al: Diagnosis and management of ADHD in children. Am Fam Physician
2014;90(7):456-464.

Item 193

ANSWER: C

This patient has ruptured the flexor digitorum profundus (FDP) tendon as a result of a sudden force applied
to the distal phalanx while in flexion. This happens when catching the finger in an opponents jersey or belt
loop while making a tackle, hence the name jersey finger. This injury is also seen in wrestlers.

The FDP attaches at the base of the distal phalanx while the flexor digitorum sublimis (FDS) attaches at
the base of the middle phalanx. Thus, it can be determined by physical examination which tendon is
injured, providing for early and accurate diagnosis and proper treatment. With the noninvolved fingers held
in full extension, if the patient can flex the proximal interphalangeal (PIP) joint the FDS is intact. With
rupture of the FDP, distal interphalangeal (DIP) joint flexion may still be weakly present, but the patient
is unable to hold flexion against resistance and this maneuver elicits pain. With rupture of both flexor
tendons, the patient is unable to flex either the PIP or the DIP joint. In the scenario presented, rupture of
both flexor tendons is unlikely. Rupture of the extensor digitorum tendons may cause mallet finger at the
DIP joint, or a boutonnire deformity of the PIP joint. Since many of these flexor tendon injuries require
surgical exploration and repair, early orthopedic consultation is recommended.

Ref: Borchers JR, Best TM: Common finger fractures and dislocations. Am Fam Physician 2012;85(8):805-810. 2) Brukner
P, Khan K: Clinical Sports Medicine, ed 4. McGraw Hill, 2012, pp 435-448.

Item 194

ANSWER: B

Anticoagulant therapy is the treatment for pulmonary embolism and should be started immediately. A
rapid-acting parenteral anticoagulant (heparin, enoxaparin) or a direct orally active factor Xa inhibitor
(rivaroxaban) should be started initially. After the initial treatment, the factor Xa inhibitor should be
continued or the parenteral anticoagulant should be transitioned to an oral anticoagulant. Patients who
develop a pulmonary embolism as a complication of a reversible factor have a low risk of recurrence when
the anticoagulant therapy is stopped. A 3-month course of anticoagulant is adequate treatment.

Ref: Goldman L, Schafer AI (eds): Goldmans Cecil Medicine, ed 25. Elsevier Saunders, 2016, pp 620-627.

70
Item 195

ANSWER: A

Proper sleep position is highly important in reducing the risk of sudden infant death syndrome (SIDS). Side
sleeping and prone sleeping increase the risk for SIDS (SOR A). Bed sharing also increases the risk. The
Back to Sleep campaign of the American Academy of Pediatrics (AAP) was associated with a 53%
decrease in the incidence of SIDS in the United States in the 1990s.

The use of pacifiers during sleep decreases the risk for SIDS, as does exclusive breastfeeding. The AAP
recommends offering a pacifier for use during sleep for infants in whom breastfeeding is well established.

There are also multiple invariable risk factors for SIDS. These include male sex, Native American
ethnicity, birth weight <2500 g, and birth before 37 weeks gestation. Large for gestational age infants are
not at increased risk for SIDS, and infants of Asian or Pacific Islander ethnicity actually have a lower than
average risk of SIDS. Apparent life-threatening events are not a risk factor for SIDS and are not related
to SIDS (SOR B).

Ref: Adams SM, Ward CE, Garcia KL: Sudden infant death syndrome. Am Fam Physician 2015;91(11):778-783.

Item 196

ANSWER: B

BNP is a sensitive marker for heart failure and procalcitonin is a reasonable marker for acute infection.
Serum lactate is used to help assess the severity of hypoperfusion, or shock. The erythrocyte sedimentation
rate and C-reactive protein level are not specific and would not be good discriminators in this case. Serial
troponin and creatine phosphokinase can help rule out acute myocardial ischemia but would not help
discriminate between heart failure and pneumonia. D-dimer is a marker for thromboembolism and is not
relevant in this situation.

Ref: Okorie ON, Dellinger P: Lactate: Biomarker and potential therapeutic target. Crit Care Clin 2011;27(2):299-326. 2) Maisel
A, Neath SX, Landsberg J, et al: Use of procalcitonin for the diagnosis of pneumonia in patients presenting with a chief
complaint of dyspnoea: Results from the BACH (Biomarkers in Acute Heart Failure) trial. Eur J Heart Fail
2012;14(3):278-286. 3) Chowdhury P, Choudhary R, Maisel A: The appropriate use of biomarkers in heart failure. Med
Clin North Am 2012;96(5):901-913. 4) Vincent JL (ed): Annual Update in Intensive Care and Emergency Medicine 2013.
Springer, 2013, pp 77-86.

71
Item 197

ANSWER: D

The diagnosis and treatment of left ventricular hypertrophy (LVH) in patients with hypertension is
important, as it is associated with increased morbidity and mortality from heart failure, arrhythmias, and
sudden cardiac death. LVH is often seen in patients with long-standing uncontrolled hypertension. If the
sum of the S wave in lead V1 and the R wave in lead V5 or V6 is 35 mm or the R wave in aVL is 11
mm, it suggests the presence of LVH (Sokolow-Lyon indices). The sensitivity of an EKG for LVH ranges
from 7% to 50%, however, so echocardiography is the test of choice to make the diagnosis.
Echocardiography helps to directly visualize and quantify left ventricle wall size, in addition to providing
other valuable information such as cardiac wall motion abnormalities, ejection fraction, and cardiac
chamber dimensions. There is no primary role for the other tests listed in this setting (SOR B).

Ref: Drazner MH: The progression of hypertensive heart disease. Circulation 2011;123(3):327-334. 2) Goldberger AL,
Goldberger ZD, Shvilkin A: Goldbergers Clinical Electrocardiography: A Simplified Approach, ed 8. Elsevier Saunders,
2012.

Item 198

ANSWER: B

This patient has gestational hypertension. She has no findings consistent with a diagnosis of preeclampsia,
although she is now at risk for this condition. Current evidence suggests that, in the absence of
preeclampsia with severe features (blood pressure >160/110 mm Hg, thrombocytopenia, impaired liver
function, renal insufficiency, pulmonary edema, cerebral edema, or visual disturbance), pregnancy can
safely be continued until 37 weeks gestation. At that point delivery is recommended in order to avoid the
risks associated with gestational hypertension, which include abruption, intrauterine growth restriction, and
progression to preeclampsia and eclampsia.

Ref: American College of Obstetricians and Gynecologists; Task Force on Hypertension in Pregnancy: Hypertension in
pregnancy. Report of the American College of Obstetricians and Gynecologists Task Force on Hypertension in Pregnancy.
Obstet Gynecol 2013;122(5):1122-1131.

Item 199

ANSWER: D

This patient has signs and symptoms of carpal tunnel syndrome. Her new task is likely causing
inflammation and/or hypertrophy of the flexor tendons of the wrist and hand, which in turn are placing
pressure on the median nerve within the closed space of the carpal tunnel. The use of a wrist splint and
NSAIDs has been shown to reduce pain and should be tried along with exercises of the wrist and hand, and
activity modification if possible. Nerve conduction studies are not recommended unless conservative
management fails and carpal tunnel release is being considered.

Ref: Rakel RE, Rakel DP (eds): Textbook of Family Medicine, ed 9. Elsevier Saunders, 2016, pp 659-660.

72
Item 200

ANSWER: D

The term Achilles tendinopathy reflects the chronic, noninflammatory, degenerative nature of this disorder
as opposed to the traditional term tendinitis. Eccentric strengthening programs of the gastrocnemius and
soleus muscles have demonstrated 60%90% improvement in pain and function and are considered the
first-line treatment for chronic midsubstance Achilles tendinopathy. Oral NSAIDs and oral corticosteroids
are ineffective, and local corticosteroid injection in or near a load-bearing tendon such as the Achilles
tendon increases the risk of tendon rupture. Surgical tendinoplasty is inconsistently beneficial and carries
added risk, and is not a first-line treatment.

Ref: Childress MA, Beutler A: Management of chronic tendon injuries. Am Fam Physician 2013;87(7):486-490.

Item 201

ANSWER: A

Drug therapy aimed at increasing HDL-cholesterol levels when added to a statin treatment does not
decrease a patients cardiovascular risk. Such agents have no effect on all-cause mortality, cardiovascular
mortality, or the risk of stroke (SOR B). Current guidelines for treatment of hyperlipidemia are based on
individual cardiovascular risk stratification rather than LDL-cholesterol levels, and recommend low-,
medium-, or high-intensity therapy with statins, based on an individual patients risk.

Ref: Ebell MH, Grad R: Top 20 research studies of 2014 for primary care physicians. Am Fam Physician 2015;92(5):377-383.

Item 202

ANSWER: E

Patients with diverticulosis should increase dietary fiber intake or take fiber supplements to slow
progression of the diverticular disease. Avoiding nuts, corn, popcorn, and small seeds has not been shown
to prevent complications of diverticular disease. Limiting intake of dairy products, spicy foods, and wheat
flour would be appropriate for other gastrointestinal problems such as lactose intolerance, gastroesophageal
reflux disease (GERD), and celiac disease.

Ref: Wilkins T, Embry K, George R: Diagnosis and management of acute diverticulitis. Am Fam Physician 2013;87(9):612-620.

Item 203

ANSWER: B

Necrobiosis lipoidica diabeticorum is seen in 0.3% of patients with diabetes mellitus or impaired glucose
tolerance. The lesions may precede the diagnosis of diabetes mellitus by several years. The sharply
demarcated reddish-brown plaque with central yellow deposits in the pretibial area is characteristic and a
biopsy is not always necessary. Topical corticosteroids are sometimes helpful.

73
Granuloma annulare and sarcoidosis are unlikely on the leg. Early lesions of necrobiosis lipoidica
diabeticorum can be confused with granuloma annulare or sarcoidosis, however, and a biopsy may be
helpful. While xanthomas can be flat plaques up to several centimeters in size, they usually occur on flexor
surfaces of the limbs along with the trunk and face. They lack the reddish-brown outer portion of the
lesions described here.

Ref: Lee SY, Shen L, Keimig EL: A solitary red patch. JAMA 2014;311(22):2328-2329.

Item 204

ANSWER: B

Thiazolidinediones (TZDs) improve hyperglycemia by improving insulin resistance and by maintaining or


improving $-cell secretory function. One of the side effects of TZDs is that they can cause fluid retention,
especially in patients with cardiac and renal disease. This may lead to weight gain and peripheral edema.
Because of this effect, TZDs are contraindicated in patients with New York Heart Association class III or
IV heart failure. They can also reduce bone mineral density and are associated with a higher risk of
non-osteoporotic bone fractures. In addition to their ability to decrease glucose levels, however, there have
been some other favorable effects noted with their use. Clinical trials have shown that treatment with
pioglitazone resulted in a significant reduction in the composite outcome of nonfatal acute myocardial
infarction, stroke, and all-cause mortality. They may also help prevent central nervous system insulin
resistancerelated cognitive dysfunction. TZDs are also useful in patients with nonalcoholic steatohepatitis.

Ref: Handelsman Y, Bloomgarden ZT, Grunberger G, et al: American Association of Clinical Endocrinologists and American
College of EndocrinologyClinical practice guidelines for developing a diabetes mellitus comprehensive care plan2015.
Endocr Pract 2015;21(Suppl 1):1-87. 2) Garber AJ, Abrahamson MJ, Barzilay JI, et al: Consensus statement by the
American Association of Clinical Endocrinologists and American College of Endocrinology on the comprehensive type 2
diabetes management algorithm2016 Executive Summary. Endocr Pract 2016;22(1):84-113.

Item 205

ANSWER: A

Proton pump inhibitor use has been shown to increase the risk for Clostridium difficile and other enteric
infections, and elderly patients and those with significant comorbid conditions may already be at increased
risk. Studies have not shown an increased risk for iron deficiency. There is no increased risk for
hypothyroidism, Helicobacter pylori infection, or type 2 diabetes mellitus.

Ref: Harnik IG: Gastroesophageal reflux disease. Ann Intern Med 2015;163(1):ITC1.

74
Item 206

ANSWER: A

Nonbullous impetigo is most often caused by Streptococcus pyogenes and methicillin-sensitive


Staphylococcus aureus. Cephalexin is the most appropriate option, with good coverage for both of these
bacteria. Penicillin VK has been found to be no more effective than placebo in the treatment of impetigo.
Macrolide resistance limits the use of erythromycin. Tetracycline should not be used in children <8 years
old as it may cause staining of permanent teeth. Trimethoprim/sulfamethoxazole has coverage against both
methicillin-sensitive and methicillin-resistant Staphylococcus aureus, but may have inadequate coverage for
Streptococcus.

Ref: Hartman-Adams H, Banvard C, Juckett G: Impetigo: Diagnosis and treatment. Am Fam Physician 2014;90(4):229-235.

Item 207

ANSWER: C

Dopamine antagonists, such as metoclopramide, block dopamine stimulation in the chemoreceptor trigger
zone, thereby limiting emetic input to the medullary vomiting center (SOR C). SSRIs, such as ondansetron,
also work in the chemoreceptor trigger zone. They inhibit serotonin at the 5-HT3 receptor in the small
bowel, vagus nerve, and chemoreceptor trigger zone. Antihistamines and anticholinergics limit stimulation
of the vomiting center through inhibition of the H1 receptor and acetylcholine, respectively. These
medications are particularly beneficial in vestibular-mediated nausea, such as motion sickness.

Ref: Flake ZA, Linn BS, Hornecker JR: Practical selection of antiemetics in the ambulatory setting. Am Fam
Physician2015;91(5):293-296.

Item 208

ANSWER: C

Autosomal dominant polycystic kidney disease (ADPCKD) is the most common genetic kidney disease and
accounts for 4.7% of end-stage kidney disease cases in America. Many patients with ADPCKD are
asymptomatic, but early symptoms can include flank pain, gross hematuria, or recurrent urinary tract
infections. The most common extrarenal manifestation of ADPCKD is hypertension, which can precipitate
cardiovascular dysfunction, including left ventricular hypertrophy. Thus, early diagnosis and management
of hypertension is crucial.

The goal blood pressure should be <140/90 mm Hg in patients under the age of 60. All ADPCKD patients
eventually develop a loss of renal function, and approximately 80% develop end-stage renal disease by age
70. An ACE inhibitor is the recommended first-line therapy (SOR C), so lisinopril is the best choice for
this patient. Angiotensin receptor blockers are acceptable in patients who cannot tolerate ACE inhibitors.

Ref: Grantham JJ: Autosomal dominant polycystic kidney disease. N Engl J Med 2008;359(14):1477-1485. 2) Srivastava A,
Patel N: Autosomal dominant polycystic kidney disease. Am Fam Physician 2014;90(5):303-307.

75
Item 209

ANSWER: E

Polymyalgia rheumatica (PMR) without concurrent giant cell arteritis is treated with a slow taper of
low-dose corticosteroids (SOR C). Disease-modifying antirheumatic drugs and antimalarial medications
have no role in the treatment of PMR. Aspirin therapy is recommended as adjuvant therapy for giant cell
arteritis to decrease stroke risk (SOR C). Colchicine may alleviate acute symptoms of gout and pseudogout.

Ref: Wasserman AM: Diagnosis and management of rheumatoid arthritis. Am Fam Physician 2011;84(11):1245-1252. 2) Caylor
TL, Perkins A: Recognition and management of polymyalgia rheumatica and giant cell arteritis. Am Fam Physician
2013;88(10):676-684. 3) Hainer BL, Matheson E, Wilkes RT: Diagnosis, treatment, and prevention of gout. Am Fam
Physician 2014;90(12):831-836.

Item 210

ANSWER: A

Chronic kidney disease appears to be overdiagnosed in the older population. Stage 3A kidney disease is
defined as an estimated glomerular filtration rate (GFR) of 4559 mL/min/1.73 m2 and is predominantly
seen in older patients. It is seldom progressive in the absence of significant proteinuria. Older patients with
chronic kidney disease are less likely to develop end-stage renal disease than to die of complications related
to aging and cardiovascular disease. There is a decline in estimated GFR with normal aging, and the
likelihood of patients progressing to end-stage renal disease and dialysis is minimal if they have a GFR of
4559 mL/min/1.73 m2.

Ref: Glassock R, Delanaye P, El Nahas M: An age-calibrated classification of chronic kidney disease. JAMA
2015;314(6):559-560.

Item 211

ANSWER: D

In contrast to Little League elbow, which is a complex of possible injuries, Little League shoulder refers
to one entity, proximal humeral epiphysitis, most often developing as an overuse injury in baseball pitchers
age 1116.

Ref: Saltzman BM, Chalmers PN, Mascarenhas R, et al: Upper extremity physeal injury in young baseball pitchers. Phys
Sportsmed 2014;42(3):100-111.

76
Item 212

ANSWER: B

The most common cause of hypoglycemia in previously stable, well-controlled diabetic patients who have
not changed their diet or insulin dosage is diabetic renal disease. A reduction in physical activity or the
appearance of insulin antibodies (unlikely after 20 years of therapy) would increase insulin requirements
and produce hyperglycemia. Spontaneous improvement of $-cell function after 20 years would be very rare.

Ref: Fauci AS, Braunwald E, Kasper DL, et al (eds): Harrisons Principles of Internal Medicine, ed 17. McGraw-Hill, 2008,
pp 1751-1752. 2) Alsahli M, Gerich JE: Hypoglycemia, chronic kidney disease, and diabetes mellitus. Mayo Clin Proc
2014;89(11):1564-1571.

Item 213

ANSWER: A

Common causes of red eye include infectious conjunctivitis, allergies, corneal abrasion,
keratoconjunctivitis, subconjunctival hemorrhage, uveitis, blepharitis, iritis, acute angle-closure glaucoma,
and herpes zoster ophthalmicus.

Viral infections typically cause conjunctivitis with mild pain and no loss of vision. The problem is usually
unilateral in the beginning and a watery to serous discharge may be noted. Adenovirus is the most common
cause. Acute bacterial conjunctivitis has a similar presentation and may include eyelid edema and a purulent
discharge. Allergic conjunctivitis is usually bilateral and painless, with intense itching, and a stringy or
ropy watery discharge.

Herpes zoster ophthalmicus is associated with a vesicular rash, keratitis, and uveitis. The rash is preceded
by pain and a tingling sensation. Findings include conjunctivitis and dermatomal involvement, which are
usually unilateral.

With corneal abrasion there is usually a history of an injury involving a foreign object. Signs and symptoms
include severe eye pain; red, watery eyes; photophobia; and a foreign body sensation. Vision is usually
normal and pupils are equal and reactive to light.

Symptoms of uveitis include a red eye, loss of vision, and photophobia. It is associated with many
autoimmune diseases, including reactive arthritis, ankylosing spondylitis, and inflammatory bowel disease.

Acute angle-closure glaucoma causes a significant loss of vision, with dilated pupils that dont react
normally to light. Symptoms include severe pain and watery eyes, with halos around lights. Patients may
have nausea and vomiting. This form of glaucoma often has an acute onset.

Ref: Cronau H, Kankanala RR, Mauger T: Diagnosis and management of red eye in primary care. Am Fam Physician
2010;81(2):137-144. 2) Goldman L, Schafer AI (eds): Goldmans Cecil Medicine, ed 25. Elsevier Saunders, 2016, pp
2556-2568.

77
Item 214

ANSWER: D

The U.S. Preventive Services Task Force recommends against the use of $-carotene or vitamin E
supplementation for the prevention of cardiovascular disease or cancer. This is a class D recommendation
(do not recommend). Overall there is no beneficial effect on cancer or heart disease from these vitamin
supplements. In one study vitamin E appeared to increase the risk of hemorrhagic stroke, and $-carotene
has been found to increase the risk of lung cancer in persons already at higher risk for lung cancer.

Ref: Moyer VA; US Preventive Services Task Force: Vitamin, mineral, and multivitamin supplements for the primary prevention
of cardiovascular disease and cancer: US Preventive Services Task Force recommendation statement. Ann Intern Med
2014;160(8):558-564.

Item 215

ANSWER: B

The U.S. Preventive Services Task Force recommends that primary care clinicians prescribe oral fluoride
supplementation starting at 6 months of age for children whose water supply is deficient in fluoride (<0.6
ppm) (B recommendation). The task force found evidence of moderate benefit of oral fluoride
supplementation for the prevention of dental caries in this group.

Ref: Prevention of dental caries in children from birth through five years of age: Recommendation statement. Am Fam Physician
2015;91(3):190A-190E.

Item 216

ANSWER: B

For all infants born before 29 weeks gestation, palivizumab is recommended for the first year of life during
respiratory syncytial virus season to reduce the likelihood of hospitalization. Immunization against pertussis
and rotavirus is not recommended until the 2-month visit. Influenza vaccine is not recommended for any
infant until 6 months of age. There is no indication for immunoglobulin in this infant.

Ref: Joffe S, Escobar GJ, Black SB, et al: Rehospitalization for respiratory syncytial virus among premature infants. Pediatrics
1999;104(4 Pt 1):894-899. 2) American Academy of Pediatrics Committee on Infectious Diseases, American Academy
of Pediatrics Bronchiolitis Guidelines Committee: Updated guidance for palivizumab prophylaxis among infants and young
children at increased risk of hospitalization for respiratory syncytial virus infection. Pediatrics 2014;134(2):415-420. 3)
Gauer RL, Burket J, Horowitz E: Common questions about outpatient care of premature infants. Am Fam Physician
2014;90(4):244-251.

78
Item 217

ANSWER: D

The treatment goal of narrow QRS complex tachycardia is to slow down the heart rate and to convert to
normal sinus rhythm by blocking, or increasing the refractoriness of, the atrioventricular node. In a
hemodynamically stable patient vagal maneuvers are a good first-line treatment, followed by adenosine.
If those do not work verapamil or diltiazem can be used. DC cardioversion is used in narrow QRS complex
tachycardia if the patient becomes hemodynamically unstable. Amiodarone, procainamide, and sotalol are
all used for the treatment of wide QRS complex tachycardia but not for narrow-complex tachycardia.

Ref: Helton MR: Diagnosis and management of common types of supraventricular tachycardia. Am Fam Physician
2015;92(9):793-800.

Item 218

ANSWER: E

This patient displays most of the criteria for borderline personality disorder. This is a maladaptive
personality type that is present from a young age, with a strong genetic predisposition. It is estimated to
be present in 1% of the general population and involves equal numbers of men and women; women seek
care more often, however, leading to a disproportionate number of women being identified by medical
providers.

Borderline personality disorder is defined by high emotional lability, intense anger, unstable relationships,
frantic efforts to avoid a feeling of abandonment, and an internal sense of emptiness. Nearly every patient
with this disorder engages in self-injurious behavior (cutting, suicidal gestures and attempts), and about 1
in 10 patients eventually succeeds in committing suicide. However, 90% of patients improve despite having
made numerous suicide threats. Suicidal gestures and attempts peak when patients are in their early 20s,
but completed suicide is most common after age 30 and usually occurs in patients who fail to recover after
many attempts at treatment. In contrast, suicidal actions such as impulsive overdoses or superficial cutting,
most often seen in younger patients, do not usually carry a high short-term risk, and serve to communicate
distress.

Inpatient hospitalization may be an appropriate treatment option if the person is experiencing extreme
difficulties in living and daily functioning, and pharmacotherapy may offer a mild degree of symptom
relief. While these modalities have a role in certain patients, psychotherapy is considered the mainstay of
therapy, especially in a relatively stable patient such as the one described.

Ref: Dean L, Falsetti SA: Treating patients with borderline personality disorder in the medical office. Am Fam Physician
2013;88(2):140-141.

79
Item 219

ANSWER: C

Absorption and metabolism of vitamin D is known to be affected by interaction with other medications.
Isoniazid and thiazide diuretics can lead to increased blood levels or activity of vitamin D. Estrogen
replacement therapy can also increase levels of vitamin D in the blood, although this potential benefit seems
to diminish when progesterone is added. Vitamin D absorption through the gut can be reduced by mineral
oil, cholestyramine, and certain antacid preparations, leading to lower blood levels. The metabolism of
vitamin D is accelerated by anticonvulsant drugs such as phenobarbital and phenytoin, which can also result
in lower than desired levels of vitamin D. Statins are not reported to have any known effect on vitamin D
levels.

Ref: NIH Office of Dietary Supplements. Vitamin D: Fact sheet for health professionals. 2) Hawkins EB, Ehrlich SD:
University of Maryland Medical Center. Possible interactions with: Vitamin D. 3) Grber U, Kisters K: Influence of drugs
on vitamin D and calcium metabolism. Dermatoendocrinol 2012;4(2):158-166.

Item 220

ANSWER: D

This case presents a child with an insidious onset of pain and mild limitation of range of motion of one hip
joint. The case suggests involvement of the femoroacetabular joint, and all of the listed options can affect
this joint. Idiopathic osteonecrosis of the hip (Legg-Calv-Perthes disease) occurs most commonly in
children at 212 years of age and has a male predominance. Symptom onset is insidious, as in this case,
with symptom severity and functional limitations dependent on the level of disease progression. Slipped
capital femoral epiphysis occurs much more commonly in adolescents. Transient synovitis and septic
arthritis have a more acute onset, and typically cause fever. Osteoarthritis typically occurs in older adults.

Ref: Wilson JJ, Furukawa M: Evaluation of the patient with hip pain. Am Fam Physician 2014;89(1):27-34.

Item 221

ANSWER: D

A number of alternative therapies have been used for problems related to pregnancy, although vigorous
studies are not always possible. For nausea and vomiting, however, vitamin B6 is considered first-line
therapy, sometimes combined with doxylamine. Other measures that have been found to be somewhat
useful include ginger and acupressure.

Cranberry products can be useful for preventing urinary tract infections, and could be recommended for
patients if this is a concern. Blue cohosh has been used as a partus preparator, but there are concerns about
its safety. Fenugreek has been used to increase milk production in breastfeeding mothers, but no rigorous
trials have been performed.

Ref: Low Dog T: The use of botanicals during pregnancy and lactation. Altern Ther Health Med 2009;15(1):54-58. 2) Herrell
HE: Nausea and vomiting of pregnancy. Am Fam Physician 2014;89(12):965-970.

80
Item 222

ANSWER: D

Most of the symptoms of foodborne illness are not specific to the causative organism. However, the onset
of vomiting and diarrhea within hours of consumption of contaminated food results from the ingestion of
preformed toxins, most often from Staphylococcus aureus or Bacillus cereus in the United States.
Campylobacter jejuni, Escherichia coli O157:H7, Shigella species, and hepatitis A typically produce
symptoms more than a day after ingestion.

Ref: Switaj TL, Winter KJ, Christensen SR: Diagnosis and management of foodborne illness. Am Fam Physician
2015;92(5):358-365.

Item 223

ANSWER: D

For functional constipation in older adults, behavioral changes should be first-line management. These
include scheduled toileting with proper positioning, increased intake of fiber and fluids, and avoiding
bedpan use. If there is not an adequate response to behavioral interventions, osmotic laxatives should be
initiated. Polyethylene glycol is more effective and has fewer side effects compared to lactulose.
Magnesium salts, including magnesium citrate, do not have strong evidence for safety or efficacy, and
magnesium toxicity is a concern with long-term use. Due to possible adverse effects of stimulant laxatives
in the long term, particularly with older adults, these drugs should be used only if fiber and osmotic
laxatives are unsuccessful.

Ref: Bharucha AE, Pemberton JH, Locke GR 3rd: American Gastroenterological Association technical review on constipation.
Gastroenterology 2013;144(1):218-238. 2) Mounsey A, Raleigh M, Wilson A: Management of constipation in older adults.
Am Fam Physician 2015;92(6):500-504.

Item 224

ANSWER: C

SSRIs and SNRIs are both effective in reducing depressive symptoms, but SNRIs have been shown to be
superior to SSRIs for management of neuropathic pain (SOR A). Bupropion would effectively treat the
patients depression and could cause weight loss, but it is contraindicated in patients with seizure disorders
(SOR A). Tricyclic antidepressants such as nortriptyline could also help with the pain but might also worsen
the patients obesity and fatigue (SOR A).

Ref: Kovich H, DeJong A: Common questions about the pharmacologic management of depression in adults. Am Fam Physician
2015;92(2):94-100.

81
Item 225

ANSWER: B

An ulnar neuropathy most commonly presents with sensory changes in the fourth and fifth digits and usually
does not involve weakness in hand grip. There is usually no specific injury, but any activity that results in
repetitive or prolonged wrist extension, as with cycling or playing catcher, may increase the risk of this
problem. It is important to examine the neck for cervical disc disease and to examine the shoulder to see
if motion elicits the pain, which would indicate a brachial plexus problem. If symptoms are reproduced by
compressing the ulnar nerve at the elbow this could be the site of entrapment. Clinical tests may include
a positive Tinel sign on percussion of the ulnar nerve over Guyons canal. Also, there may be a positive
Phalen sign, with maximum passive flexion of the wrist for 1 minute inducing paresthesias in the fourth
and fifth fingers.

With ulnar neuropathy, plain radiographs are usually normal. Ultrasonography of the peripheral nerves may
be helpful in identifying compression etiologies. However, electromyelography and nerve conduction
velocities may be required to identify the area of entrapment.

Ref: Shehab R, Mirabelli MH: Evaluation and diagnosis of wrist pain: A case-based approach. Am Fam Physician
2013;87(8):568-573.

Item 226

ANSWER: D

Self-measured blood pressure monitoring, with or without additional support (e.g., education, counseling,
telemedicine, home visits, Web-based logging), lowers blood pressure when compared with usual care,
although the benefits beyond 12 months are not clear (SOR A). Limiting sodium intake to 2400 mg/day
(approximately 1 teaspoon of table salt) is recommended to lower blood pressure. Additional benefit occurs
with a limit of 1500 mg/day (SOR B). A diet that emphasizes vegetables, fruits, and whole grains is also
recommended to lower blood pressure (SOR A), as well as limiting alcohol consumption to no more than
2 drinks/day for men, and 1 drink/day for women (SOR C). Because of mixed results from therapeutic
trials and many limitations to the trials, the American Heart Association does not recommend either yoga
or acupuncture to lower blood pressure.

Ref: Oza R, Garcellano M: Nonpharmacologic management of hypertension: What works? Am Fam Physician
2015;91(11):772-776.

82
Item 227

ANSWER: D

Daytime sleepiness is the clinically relevant symptom of obstructive sleep apnea (OSA) that is most
responsive to treatment. Other associated symptoms such as snoring, insomnia, and fatigue are either less
clinically relevant or less responsive to treatment. Hypertension, diabetes mellitus, and coronary artery
disease are associated with OSA, but evidence is insufficient that continuous positive airway pressure
(CPAP) improves outcomes for these conditions, especially if they are not associated with daytime
sleepiness.

Ref: Qaseem A, Dallas P, Owens DK, et al: Diagnosis of obstructive sleep apnea in adults: A clinical practice guideline from
the American College of Physicians. Ann Intern Med 2014;161(3):210-220.

Item 228

ANSWER: D

A baseline EKG is currently not recommended in an asymptomatic patient to screen for cardiovascular
disease. The American Academy of Family Physicians (AAFP) specifically advises against performing a
baseline EKG.

The AAFP does endorse performing a detailed past medical history to exclude a history of hypertension,
chest pain, prior heart murmur, or syncope. It also recommends obtaining a family history that includes
questions about any family members with prolonged QT syndrome, Marfan syndrome, or sudden death
before the age of 50. A physical examination is recommended, including palpation of the femoral pulses
simultaneously to detect coarctation of the aorta, as well as heart auscultation performed with the patient
both supine and standing, and with the Valsalva maneuver, to detect a heart murmur suggestive of cardiac
disease.

Ref: Siwek J: Choosing wisely: Top interventions to improve health and reduce harm, while lowering costs. Am Fam Physician
2012;86(2):128-133. 2) Final Recommendation Statement: Coronary Heart Disease: Screening with Electrocardiography.
US Preventive Services Task Force, 2012. 3) Mirabelli MH, Devine MJ, Singh J, Mendoza M: The preparticipation sports
evaluation. Am Fam Physician 2015;92(5):371-376.

Item 229

ANSWER: B

This patient has transient tachypnea of the newborn, which typically occurs within 2 hours of birth. The
chest radiograph usually shows hyperexpansion with perihilar densities and fluid within the fissures.
Respiratory distress syndrome of the newborn is most often seen in premature infants, and the chest
radiograph shows a classic diffuse ground-glass appearance. With pneumothorax a chest radiograph would
typically show a partial or complete lung collapse. Meconium aspiration syndrome occurs in the setting of
meconium-stained fluid and is usually apparent immediately after delivery. The chest radiograph typically
shows fluffy densities with hyperinflation.

Ref: Hermansen CL, Mahajan A: Newborn respiratory distress. Am Fam Physician 2015;92(11):994-1002.

83
Item 230

ANSWER: A

In critically ill adult patients, the only deep vein thrombosis prophylaxis that decreases mortality risk is
anticoagulation therapy. Mechanical device prophylaxis does not lower the mortality risk compared to no
prophylaxis. Data suggests that patients managed with both prophylactic anticoagulation and mechanical
device prophylaxis have a higher mortality risk than those managed by prophylactic anticoagulation alone.

Ref: Lilly CM, Liu X, Badawi O, et al: Thrombosis prophylaxis and mortality risk among critically ill adults. Chest
2014;146(1):51-57.

Item 231

ANSWER: E

Panic disorder typically presents in late adolescence or early adulthood with unpredictable episodes of
palpitations, sweating, gastrointestinal distress, dizziness, and paresthesias. The attacks are sporadic and
last 1060 minutes. Generalized anxiety disorder is more common, and common symptoms include
restlessness, fatigue, muscle tension, irritability, difficulty concentrating, and sleep disturbance.
Pheochromocytoma is associated with headache and hypertension, and usually occurs in thin patients.
Paroxysmal supraventricular tachycardia is usually not associated with gastrointestinal distress or
paresthesias. While mitral valve prolapse can be associated with anxiety and panic disorder, the physical
examination would not be normal.

Ref: Kasper DL, Fauci AS, Hauser SL, et al (eds): Harrisons Principles of Internal Medicine, ed 19. McGraw-Hill, 2015, pp
2708-2709.

Item 232

ANSWER: C

The U.S. Preventive Services Task Force recommends one-time conventional abdominal duplex
ultrasonography for screening patients who are at risk of abdominal aortic aneurysm (males 6575 years
of age who have smoked a total of 100 cigarettes or more during their lifetime). This imaging modality has
high sensitivity and specificity, and it is noninvasive, easy to use, and low cost. None of the other imaging
modalities have been formally evaluated in clinical trials.

Ref: LeFevre ML; US Preventive Services Task Force: Screening for abdominal aortic aneurysm: US Preventive Services Task
Force recommendation statement. Ann Intern Med 2014;161(4):281-290. 2) Croswell J, Chu K: Screening for abdominal
aortic aneurysm. Am Fam Physician 2015;91(8):563-564.

84
Item 233

ANSWER: E

Repetitive high-intensity training places an individual at risk for developing a stress fracture. A plain
radiograph is the best initial test for a suspected stress fracture. If this is negative, then the study should
be repeated in 23 weeks. MRI is now considered the procedure of choice if there is an urgent need for a
diagnosis, although triple-phase bone scintigraphy has a similar sensitivity.

Treatment of stress fractures usually consists of decreasing activity or, in some instances, such as
involvement of the anterior tibial cortex (where there is a risk of a complete fracture), nonweight bearing
with immobilization. Using a walker boot for tibial stress fractures reduces the time to resumption of full
activity. Most stress fractures should not be treated with a bone stimulator. Supplementation with vitamin
D and calcium has shown some benefit in prevention, whereas bisphosphonates have not. NSAIDs are
relatively contraindicated, as limited studies have shown that they may actually inhibit healing of traumatic
fractures.

Ref: Patel DS, Roth M, Kapil N: Stress fractures: Diagnosis, treatment, and prevention. Am Fam Physician 2011;83(1):39-46.
2) Davis BJ, Estes AR: An adolescent with right shin pain. JAMA 2015;313(15):1566-1567.

Item 234

ANSWER: B

Up to 28% of patients may be defined as having resistant hypertension (not controlled on three drugs or
controlled on four or more drugs). Primary aldosteronism is present in up to 5%10% of all hypertensive
patients and 7%20% of those with resistant hypertension. This may be due to bilateral adrenal hyperplasia
or a unilateral aldosterone-secreting adenoma, which can be diagnosed if there is elevated serum
aldosterone in the presence of suppressed renin levels.

A cortisol level and a dexamethasone suppression test are appropriate tests for Cushing syndrome. A
17-hydroxyprogesterone level tests for congenital adrenal hyperplasia. Renal ultrasonography will not
adequately screen for any of these conditions.

Ref: Viera AJ, Neutze DM: Diagnosis of secondary hypertension: An age-based approach. Am Fam Physician
2010;82(12):1471-1478. 2) Vongpatanasin W: Resistant hypertension: A review of diagnosis and management. JAMA
2014;311(21):2216-2224.

Item 235

ANSWER: A

This patient has molluscum contagiosum, which can be easily treated by curetting the lesions and inducing
an inflammatory reaction. The lesions may disappear spontaneously in a few months; however, the best
management of this condition in a patient who is bothered by it is to induce resolution.

Ref: Habif TP: Clinical Dermatology: A Color Guide to Diagnosis and Therapy, ed 6. Elsevier, 2016, pp 219-223.

85
Item 236

ANSWER: B

The patient has a mallet fracture, an avulsion fracture of the distal phalanx with a bone fragment on the
terminal extensor tendon, resulting in unopposed flexion and the inability to actively extend the distal
interphalangeal (DIP) joint. Conservative treatment consists of immobilization of the DIP joint in extension
for 8 weeks and is recommended for most cases of mallet fracture. If the joint is allowed to flex at any time
during that period, then the treatment period must be extended. Immobilization of the proximal
interphalangeal (PIP) joint is not required. Surgical pinning may be indicated in more complicated fractures
or with failure of conservative therapy.

Ref: Borchers JR, Best TM: Common finger fractures and dislocations. Am Fam Physician 2012;85(8):805-810.

Item 237

ANSWER: D

Pulmonary rehabilitation should be considered in patients with COPD who are optimally medically managed
and continue to have symptoms, particularly dyspnea. Pulmonary rehabilitation improves exercise capacity,
dyspnea, and health-related quality of life outcomes in patients with COPD. Supplemental oxygen use has
been shown to decrease mortality in patients with COPD who have severe hypoxemia. Short-acting
anticholinergics such as ipratropium should be used for patients with mild disease requiring only as-needed
medications. The long-acting anticholinergic tiotropium has been shown to improve quality-of-life scores.

Ref: Nici L, Lareau S, ZuWallack R: Pulmonary rehabilitation in the treatment of chronic obstructive pulmonary disease. Am
Fam Physician2010;82(6):655-660. 2) Lee H, Kim J, Tagmazyan K: Treatment of stable chronic obstructive pulmonary
disease: The GOLD guidelines. Am Fam Physician 2013;88(10):655-663. 3) McCarthy B, Casey D, Devane D, et al:
Pulmonary rehabilitation for chronic obstructive pulmonary disease. Cochrane Database Syst Rev 2015;(2):CD003793.

Item 238

ANSWER: D

Nonverbal communication is important for identifying issues that a patient may be hiding or be unwilling
to divulge. Some nonverbal clues, however, are culturally based. Many older or less-educated
Mexican-Americans consider direct eye contact to be disrespectful. Because a physician is held in high
regard, these patients will often either look down or look at another, more equal person in the room
while being interviewed. Many Americans, on the other hand, may consider a lack of eye contact to be
negative, and that it indicates that a patient is unsure of the information they are providing, has poor
self-esteem, or is hiding something.

Ref: Purnell LD, Paulanka BJ: Guide to Culturally Competent Health Care. FA Davis Company, 2005, pp 339-351. 2) Rakel
RE, Rakel DP (eds): Textbook of Family Medicine, ed 9. Elsevier Saunders, 2016, pp 149-150.

86
Item 239

ANSWER: E

In a female 35 years old, smoking 15 or more cigarettes per day poses an unacceptable health risk with
the use of combined oral contraceptives. Patients with varicose veins are not at increased risk for deep vein
thrombosis (DVT)/pulmonary embolism, which would be an unacceptable health risk with the use of
combined oral contraceptives. Combined oral contraceptives actually lower the risk of ovarian cancer, and
women may continue to use them while awaiting treatment for ovarian cancer. Women with a BMI 30.0
kg/m2 who take oral contraceptives are more likely to develop DVTs than those who do not use them, but
the advantages of oral contraceptives are considered to be greater than the disadvantages in these patients,
and obesity is not an absolute contraindication. A patient with sickle cell disease is at a higher risk of
adverse events from an unintended pregnancy than from the use of combined oral contraceptives.

Ref: Curtis KM, Tepper NK, Jatlaoui TC, et al: US medical eligibility criteria for contraceptive use, 2016. MMWR Recomm
Rep 2016;65(RR-3):1-103.

Item 240

ANSWER: C

An understanding of the anatomy and function of the components of the knee, coupled with a clear
description of the traumatic event, is essential for making an accurate initial clinical assessment of
sports-related knee injuries. The posterior cruciate ligament (PCL) connects the medial femoral condyle
to the posterior intercondylar area of the tibia and is affixed in such a way that the anterolateral section is
taut in flexion and the posteromedial section is taut in extension, helping to maintain the correct anatomic
relationship between the femur and tibia. The PCL alone provides almost all of the resistance to posterior
displacement of the tibia and so is appropriately the strongest of the cruciate ligaments. Tearing or rupture
of the PCL can occur with hyperextension, hyperflexion, or rotation applied with a force that is so great
that other knee components are also generally injured.

The most common mechanism leading to an isolated injury of the PCL is a direct blow to the anterior tibia
with the knee in flexion, like that experienced when the proximal tibia impacts the dashboard in an
automobile crash or when an athlete is hit or kicked in the proximal tibia while the knee is in flexion.
Athletes with a PCL injury frequently complain of posterior knee pain and pain when kneeling. The
presence of painful limitation of flexion and a posterior sag sign (posterior drawer sign) on examination
strongly supports a diagnosis of isolated PCL injury.

Ref: Grover M: Evaluating acutely injured patients for internal derangement of the knee. Am Fam Physician
2012;85(3):247-252.

87
American Board of Family Medicine

2015 IN-TRAINING EXAMINATION

CRITIQUE BOOK

This book contains the answers to each question in the In-Training Examination, as well as a critique that
provides a rationale for the correct answer. Bibliographic references are included at the end of each
critique to facilitate any further study you may wish to do in a particular area.

Copyright 2015 The American Board of Family Medicine, Inc. All rights reserved.
Item 1

ANSWER: B

Chlamydial pneumonia is usually seen in infants 316 weeks of age, and these patients frequently have
been sick for several weeks. The infant appears nontoxic and is afebrile, but is tachypneic with a prominent
cough. The physical examination will reveal diffuse crackles with few wheezes, and conjunctivitis is
present in about 50% of cases. A chest film will show hyperinflation and diffuse interstitial or patchy
infiltrates.

Staphylococcal pneumonia has a sudden onset. The infant appears very ill and has a fever, and initially
may have an expiratory wheeze simulating bronchiolitis. Signs of abdominal distress, tachypnea, dyspnea,
and localized or diffuse bronchopneumonia or lobar disease may be present. The WBC count will show
a prominent leukocytosis.

Respiratory syncytial virus infections start with rhinorrhea and pharyngitis, followed in 13 days by a
cough and wheezing. Auscultation of the lungs will reveal diffuse rhonchi, fine crackles, and wheezes, but
the chest film is often normal. If the illness progresses, coughing and wheezing increase, air hunger and
intercostal retractions develop, and evidence of hyperexpansion of the chest is seen. In some infants the
course of the illness may be similar to that of pneumonia. Rash or conjunctivitis may occur occasionally,
and fever is an inconsistent sign. The WBC count will be normal or elevated, and the differential may be
normal or shifted either to the right or left. Chlamydial infections can be differentiated from respiratory
syncytial virus infections by a history of conjunctivitis, the subacute onset and absence of fever, and the
mild wheezing. There may also be eosinophilia.

Parainfluenza virus infection presents with typical cold symptoms. Eight percent of infections affect the
upper respiratory tract. In children hospitalized for severe respiratory illness, parainfluenza viruses account
for about 50% of the cases of laryngotracheitis and about 15% each of the cases of bronchitis,
bronchiolitis, and pneumonia.

Ref: Kliegman RM, Stanton BF, Geme JW III, et al (eds): Nelson Textbook of Pediatrics, ed 19. Elsevier Saunders, 2011, p
1037. 2) Mishori R, McClaskey EL, WinklerPrins VJ: Chlamydia trachomatis infections: Screening, diagnosis, and
management. Am Fam Physician 2012;86(12):1127-1132.

Item 2

ANSWER: C

This patient has polycystic ovary syndrome, which is characterized by hyperandrogenism on clinical and
laboratory evaluations, polycystic ovaries on pelvic ultrasonography, and ovulatory dysfunction.
Hyperandrogenism and either polycystic ovaries or ovulatory dysfunction are necessary to make the
diagnosis. The first-line recommendation in obese patients is lifestyle modification, but metformin may
improve abnormal menstruation (SOR A). Low-dose combined oral contraceptives are more frequently
used to reduce the risk of endometrial cancer in patients with chronic anovulation and the resulting
unopposed estrogen secretion. This patient does not have thyroid dysfunction, so levothyroxine is not
indicated.

Ref: Master-Hunter T, Heiman DL: Amenorrhea: Evaluation and treatment. Am Fam Physician 2006;73(8):1374-1382. 2) Klein
DA, Poth MA: Amenorrhea: An approach to diagnosis and management. Am Fam Physician 2013;87(11):781-788.

1
Item 3

ANSWER: B

The recommended treatment for a mallet fracture is splinting the distal interphalangeal (DIP) joint in
extension (SOR B). The usual duration of splinting is 8 weeks. It is important that extension be maintained
throughout the duration of treatment because flexion can affect healing and prolong the time needed for
treatment. If the finger fracture involves >30% of the intra-articular surface, referral to a hand or
orthopedic surgeon can be considered. However, conservative therapy appears to have outcomes similar
to those of surgical treatment and therefore is generally preferred.

Ref: Borchers JR, Best TM: Common finger fractures and dislocations. Am Fam Physician 2012;85(8):805-810.

Item 4

ANSWER: A

Lithium, valproate, lamotrigine, and some antipsychotics (including quetiapine) are effective treatments
for both acute depression and maintenance therapy of bipolar disorders. Haloperidol is an effective
treatment for acute mania in bipolar disorders, but not for maintenance therapy or acute depression.

Ref: Price AL, Marzani-Nissen GR: Bipolar disorders: A review. Am Fam Physician 2012;85(5):483-493.

Item 5

ANSWER: D

This patient has physical findings consistent with a necrotizing skin and soft-tissue infection, or necrotizing
fasciitis. Severe pain and skin changes outside the realm of cellulitis, including bullae and deeper
discoloration, are strong indications of necrotizing fasciitis. Antimicrobial therapy is essential but is not
sufficient by itself; aggressive surgical debridement within 12 hours reduces the risk of amputation and
death.

Ref: Headley AJ: Necrotizing soft tissue infections: A primary care review. Am Fam Physician 2003;68(2):323-328. 2) Usatine
RP, Sandy N: Dermatologic emergencies. Am Fam Physician 2010;82(7):773-780.

Item 6

ANSWER: E

Patients on amiodarone can develop either hyperthyroidism or hypothyroidism. It is recommended that a


patient on amiodarone have baseline thyroid function tests (free T4, TSH) with follow-up testing every 6
months to monitor for these conditions. Hyperadrenalism and hypoadrenalism are not associated with
amiodarone treatment.

Ref: Siddoway LA: Amiodarone: Guidelines for use and monitoring. Am Fam Physician 2003;68(11):2189-2196. 2) Vassallo
P, Trohman RG: Prescribing amiodarone: An evidence-based review of clinical indications. JAMA
2007;298(11):1312-1322. 3) Goldschlager N, Epstein AE, Naccarelli GV, et al: A practical guide for clinicians who treat
patients with amiodarone: 2007. Heart Rhythm 2007;4(9):1250-1259. 4) Padmanabhan H: Amiodarone and thyroid
dysfunction. South Med J 2010;103(9):922-930.

2
Item 7

ANSWER: D

Radial head subluxation, or nursemaids elbow, is the most common orthopedic condition of the elbow in
children 14 years of age, although it can be encountered before 1 year of age and in children as old as
9 years of age. The mechanism of injury is partial displacement of the radial head when the childs arm
undergoes axial traction while in a pronated and fully extended position. The classic history includes a
caregiver picking up (or pulling) a toddler by the arm. In half of all cases, however, no inciting event is
recalled.

As long as there are no outward signs of fracture or abuse it is considered safe and appropriate to attempt
reduction of the radial head before moving on to imaging studies. With the childs elbow in 90 of flexion,
the hand is fully supinated by the examiner and the elbow is then brought into full flexion. Usually the
child will begin to use the affected arm again within a couple of minutes. If ecchymosis, significant
swelling, or pain away from the joint is present, or if symptoms do not improve after attempts at reduction,
then a plain radiograph is recommended.

Ref: Sarwark JF (ed): Essentials of Musculoskeletal Care, ed 4. American Academy of Orthopaedic Surgeons, 2010, p 1004.
2) Marx JA, Hockberger RS, Walls RM (eds): Rosens Emergency Medicine: Concepts and Clinical Practice, ed 8. Elsevier
Saunders, 2013, pp 596-617. 3) Irie T, Sono T, Hayama Y, et al: Investigation on 2331 cases of pulled elbow over the last
10 years. Pediatr Rep 2014;6(2):5090.

Item 8

ANSWER: A

This patient has moderate persistent asthma. Although many parents are concerned about corticosteroid
use in children with open growth plates, inhaled corticosteroids have not been proven to prematurely close
growth plates and are the most effective treatment with the least side effects. Scheduled use of a
short-acting bronchodilator has been shown to cause tachyphylaxis, and is not recommended. The same
is true for long-acting bronchodilators. Leukotriene use may be beneficial, but compared to those using
inhaled corticosteroids, patients using leukotrienes are 65% more likely to have an exacerbation requiring
systemic corticosteroids.

Ref: Scow DT, Luttermoser GK, Dickerson KS: Leukotriene inhibitors in the treatment of allergy and asthma. Am Fam
Physician 2007;75(1):65-70. 2) Expert Panel Report 3: Guidelines for the Diagnosis and Management of Asthma. National
Asthma Education and Prevention Program, 2007, pp 74, 328-339. 3) Kliegman RM, Stanton BF, Geme JW III, et al (eds):
Nelson Textbook of Pediatrics, ed 19. Elsevier Saunders, 2011, pp 791-796.

Item 9

ANSWER: E

The first-line treatment for primary dysmenorrhea should be NSAIDs (SOR A). They should be started
at the onset of menses and continued for the first 12 days of the menstrual cycle. Combined oral
contraceptives may be effective for primary dysmenorrhea, but there is a lack of high-quality randomized,
controlled trials demonstrating pain improvement (SOR B). They may be a good choice if the patient also
desires contraception. Although combined oral contraceptives and intramuscular and subcutaneous

3
progestin-only contraceptives are effective treatments for dysmenorrhea caused by endometriosis, they are
not first-line therapy for primary dysmenorrhea.

Ref: Osayande AS, Mehulic S: Diagnosis and initial management of dysmenorrhea. Am Fam Physician 2014;89(5):341-346.

Item 10

ANSWER: C

Thyroid nodules >1 cm that are discovered incidentally on examination or imaging studies merit further
evaluation. Nodules <1 cm should also be fully evaluated when found in patients with a family history
of thyroid cancer, a personal history of head and neck irradiation, or a finding of cervical node
enlargement. Reasonable first steps include measurement of TSH or ultrasound examination. The American
Thyroid Associations guidelines recommend that TSH be the initial evaluation (SOR A) and that this be
followed by a radionuclide thyroid scan if results are abnormal. Diagnostic ultrasonography is
recommended for all patients with a suspected thyroid nodule, a nodular goiter, or a nodule found
incidentally on another imaging study (SOR A). Routine measurement of serum thyroglobulin or calcitonin
levels is not currently recommended.

Ref: American Thyroid Association (ATA) Guidelines Taskforce on Thyroid Nodules and Differentiated Thyroid Cancer, Cooper
DS, Doherty GM, et al: Revised American Thyroid Association management guidelines for patients with thyroid nodules
and differentiated thyroid cancer. Thyroid 2009;19(11):1167-1214.

Item 11

ANSWER: C

Lamotrigine is an anti-epileptic medication that is often used in bipolar disorder. It can cause
Stevens-Johnson syndrome, which is a severe disorder of the skin and mucous membranes. This most
commonly occurs in children or when the drug is initiated at a high dosage, and is also more likely to occur
in patients taking divalproex. To decrease the risk of Stevens-Johnson syndrome, it is recommended that
lamotrigine therapy be started at a dosage of 25 mg daily and titrated every 2 weeks until the goal dosage
is reached.

Ref: Brenner CJ, Shyn SI: Diagnosis and management of bipolar disorder in primary care: A DSM-5 update. Med Clin North
Am 2014;98(5):1025-1048.

Item 12

ANSWER: A

The occurrence of two or more laboratory-confirmed cases of influenza A is considered an outbreak in a


long-term care facility. The CDC has specific recommendations for managing an outbreak, which include
chemoprophylaxis with an appropriate medication for all residents who are asymptomatic and treatment
for all residents who are symptomatic, regardless of laboratory confirmation of infection or vaccination
status. All staff should be considered for chemoprophylaxis regardless of whether they have had direct
patient contact with an infected resident or have received the vaccine. Requesting restriction of visitation
is recommended; however, it cannot be strictly enforced due to residents rights.

Ref: Fiore AE, Fry A, Shay D, et al: Antiviral agents for the treatment and chemoprophylaxis of influenzaRecommendations
of the Advisory Committee on Immunization Practices (ACIP). MMWR Recomm Rep 2011;60(1):1-24.

4
Item 13

ANSWER: A

In general, the strongest evidence for treatment, screening, or prevention strategies is found in systematic
reviews, meta-analyses, randomized controlled trials (RCTs) with consistent findings, or a single
high-quality RCT. Second-tier levels of evidence include poorer quality RCTs with inconsistent findings,
cohort studies, or case-control studies. The lowest quality of evidence comes from sources such as expert
opinion, consensus guidelines, or usual practice recommendations.

Ref: Ebell MH, Siwek J, Weiss BD, et al: Strength of recommendation taxonomy (SORT): A patient-centered approach to
grading evidence in the medical literature. Am Fam Physician 2004;69(3):548-556. 2) Kasper DL, Fauci AS, Hauser SL,
et al (eds): Harrisons Principles of Internal Medicine, ed 19. McGraw-Hill, 2015, pp 24-26.

Item 14

ANSWER: E

This infant has findings consistent with erythema toxicum neonatorum, which usually resolves in the first
week or two of life (SOR A). No testing is usually necessary because of the distinct appearance of the
lesions. The cause is unknown.

Ref: Lewis ML: A comprehensive newborn exam: Part II. Skin, trunk, extremities, neurologic. Am Fam Physician
2014;90(5):297-302.

Item 15

ANSWER: B

The American Urological Association guidelines define asymptomatic microscopic hematuria (AMH) as
3 RBCs/hpf on a properly collected urine specimen in the absence of an obvious benign cause (SOR C).
A positive dipstick does not define AMH, and evaluation should be based solely on findings from
microscopic examination of urinary sediment and not on a dipstick reading. A positive dipstick reading
merits microscopic examination to confirm or refute the diagnosis of AMH.

Ref: Davis R, Jones JS, Barocas DA, et al: Diagnosis, evaluation and follow-up of asymptomatic microhematuria (AMH) in
adults: AUA guideline. J Urol 2012;188(6 Suppl):2473-2481.

Item 16

ANSWER: A

Dyspnea is a frequent and distressing symptom in terminally ill patients. In the absence of hypoxia, oxygen
is not likely to be helpful. Opiates are the mainstay of symptomatic treatment and other measures may be
appropriate in specific circumstances. For example, inhaled bronchodilators or glucocorticoids may be
helpful in patients with COPD, and diuresis may be helpful in patients with heart failure. The evidence for
oxygen in patients with hypoxemia is not clear, but there is no benefit from oxygen for nonhypoxemic
patients.

Ref: Ekstrm MP, Abernethy AP, Currow DC: The management of chronic breathlessness in patients with advanced and
terminal illness. BMJ 2015;349:g7617. 2) Kasper DL, Fauci AS, Hauser SL, et al (eds): Harrisons Principles of Internal
Medicine, ed 19. McGraw-Hill, 2015, p 62.

5
Item 17

ANSWER: B

A number of medications can cause or exacerbate prolonged QT syndrome, which can lead to torsades de
pointes. This can be associated with syncope or degenerate into a sustained ventricular tachycardia or
ventricular fibrillation. Clarithromycin interferes with the delayed rectifier potassium current, which results
in the accumulation of potassium ions in cardiac myocytes and thereby delays cardiac repolarization. This
leads to prolongation of the QT interval and therefore the risk of fatal arrhythmia. Clarithromycin is
metabolized by the cytochrome P450 3A enzyme. When using clarithromycin it is important to avoid any
other medications that may inhibit this enzyme, leading to higher clarithromycin levels. The other
antibiotics listed do not have this effect.

Ref: Zipes DP, Jalife J (eds): Cardiac Electrophysiology: From Cell to Bedside, ed 6. Elsevier Saunders, 2014, pp 1001-1008.
2) Svanstrm H, Pasternak B, Hviid A: Use of clarithromycin and roxithromycin and risk of cardiac death: Cohort study.
BMJ 2014;349:g4930.

Item 18

ANSWER: B

Inhaled corticosteroids increase the risk of bruising, candidal infection of the oropharynx, and pneumonia.
They also have the potential for increasing bone loss and fractures. They decrease the risk of COPD
exacerbations but have no benefit on mortality and do not improve FEV1 on a consistent basis.

Ref: Hamilton T, Miller JB, Vincent EC, St Anna L: What are the benefits and risks of inhaled corticosteroids for COPD? J
Fam Pract 2014;63(5):276-278.

Item 19

ANSWER: B

Depression affects up to 9% of U.S. patients and can cause significant disability. The U.S. Preventive
Services Task Force recommends screening for depression in adults in practices that have systems in place
to ensure accurate diagnosis and treatment with followup. Brief validated depression screening tools are
readily available to assist in the diagnosis of depressed patients.

In his history, this patient gave the equivalent of positive answers to the two-question Patient Health
Questionnaire (PHQ-2), a screening instrument that is specific for depression. In other words, depression
can be ruled out when the responses are negative. Because the PHQ-2 questions are positive in this patient,
the next step is confirmation with the PHQ-9, a questionnaire that includes the two questions in the PHQ-2
plus seven additional questions.

Cardiovascular testing may be indicated in the future for this patient, but not for these symptoms. The
patients sleep disturbance, viewed in the context of his other depressive symptoms and positive PHQ-2,
is not likely to be due to a sleep disorder, so polysomnography is not indicated at this point. Untreated
depression is associated with worse outcomes in coronary artery disease, so postponing further evaluation
would be inappropriate for this patient.

Ref: US Preventive Services Task Force: Screening for depression in adults: US Preventive Services Task Force
Recommendation Statement. Ann Intern Med 2009;151(11):784-792. 2) Maurer DM: Screening for depression. Am Fam
Physician 2012;85(2):139-144.

6
Item 20

ANSWER: E

This patient has benign nocturnal limb pains of childhood (previously known as growing pains). These
crampy pains often occur in the thigh, calf, or shin, occur in up to 35% of children 46 years of age, and
may continue up to age 19. The pathology of these pains is unknown. The pain is nocturnal, without
limping or other signs of inflammatory processes. The erythrocyte sedimentation rate and CBC are normal
in this condition but testing is indicated in patients with chronic joint pain to rule out malignancy or
infection (SOR C). Rheumatoid factor and ANA have a low predictive value in primary care settings and
are not indicated in the pediatric population without evidence of an inflammatory process (SOR C). Plain
radiographs are more useful for excluding certain conditions such as cancer than for making a diagnosis
of arthritis in children (SOR C). Reassurance of the parents is indicated in this situation, along with
instruction on supportive care and over-the-counter analgesics as necessary.

Ref: Junnila JL, Cartwright VW: Chronic musculoskeletal pain in children: Part I. Initial evaluation. Am Fam Physician
2006;74(1):115-122. 2) Kliegman RM, Stanton BF, Geme JW III, et al (eds): Nelson Textbook of Pediatrics, ed 19.
Elsevier Saunders, 2011, p 878.

Item 21

ANSWER: D

In 2014 new evidence-based guidelines for blood pressure management were published by the panel
members of the Eighth Joint National Committee (JNC 8). They looked only at randomized, controlled
trials that compared one class of antihypertensive agent to another to develop the treatment
recommendations. ACE inhibitors, angiotensin receptor blockers (ARBs), calcium channel blockers, and
thiazide-type diuretics all yielded comparable effects on overall mortality and cardiovascular,
cerebrovascular, and kidney outcomes. They are all recommended for initial treatment of high blood
pressure in the nonblack population, including patients with diabetes mellitus. $-Blockers were not
recommended for the initial treatment of hypertension because one study found there was a higher rate of
the primary composite outcome of cardiovascular death, myocardial infarction, or stroke with use of these
drugs compared to the use of an ARB.

Ref: James PA, Oparil S, Carter BL, et al: 2014 evidence-based guideline for the management of high blood pressure in adults:
Report from the panel members appointed to the Eighth Joint National Committee (JNC 8). JAMA 2014;311(5):507-520.

Item 22

ANSWER: C

This patient has characteristic features of polymyalgia rheumatica, a disease whose prevalence increases
with age in older adults but is almost never seen before age 50. Most people will have accompanying
systemic symptoms including fatigue, weight loss, low-grade fever, a decline in appetite, and depression.
There are no validated diagnostic criteria available to assist in the diagnosis. The treatment response to 15
mg of prednisone daily is dramatic, often within 2448 hours, and if this response is not seen, alternative
diagnoses must be considered. NSAIDs are not useful in the management of polymyalgia rheumatica and,
in fact, are associated with high drug morbidity. Ultrasonography may be useful in making the diagnosis,
with typical findings of subdeltoid bursitis and tendon synovitis of the shoulders, but synovitis of the
glenohumeral joint is less common.

7
Ref: Michet CJ, Matteson EL: Polymyalgia rheumatica. BMJ 2008;336(7647):765-769. 2) Weyand CM, Goronzy JJ: Giant-cell
arteritis and polymyalgia rheumatica. N Engl J Med 2014;371(1):50-57.

Item 23

ANSWER: E

In patients who are euvolemic but have hyponatremia, decreased serum osmolality, and elevated urine
osmolality, the syndrome of inappropriate secretion of antidiuretic hormone (SIADH) is likely. Other
causes to rule out include thyroid disorders, adrenal insufficiency, and diuretic use. Renal function has to
be normal as well. Common drugs that cause SIADH include SSRIs (particularly in patients over 65),
chlorpropamide, barbiturates, carbamazepine, opioids, tolbutamide, vincristine, diuretics, and NSAIDs.
Treatment of the problem consists of discontinuing the offending drug. Temporary fluid restriction may
also be required.

Ref: Braun MM, Barstow CH, Pyzocha NJ: Diagnosis and management of sodium disorders: Hyponatremia and hypernatremia.
Am Fam Physician 2015;91(5):299-307.

Item 24

ANSWER: E

Although the American Academy of Family Physicians (AAFP) states that physicians are not compelled
to perform any act that violates their moral principles, the AAFP also states that physicians do have a
responsibility to provide resources on how to access a safe and legal abortion for women who are
considering that option. Induced abortion is safer than live childbirth. Between 1998 and 2005 in the
United States, mortality was 8.8 per 100,000 live births among women who delivered live neonates and
0.6 per 100,000 abortions among women who had legal abortions. Physicians should not broker adoptions,
either by matching pregnant women with prospective parents or by offering to adopt children from their
patients. Physicians should also not advocate or argue their personal moral position to patients.
Conscientious refusal does not excuse a physician from providing appropriate medical care, including
providing unbiased, medically accurate information regarding options and either having a referral process
for transfer of care or identifying resources where such information can be obtained.

Ref: Moss DA, Snyder MJ, Lu L: Options for women with unintended pregnancy. Am Fam Physician 2015;91(8):544-549.

Item 25

ANSWER: E

This patient has a severe diabetic foot ulcer. It appears to be infected and there are signs of a systemic
inflammatory response. This is an indication for intravenous antibiotics. Piperacillin/tazobactam and
vancomycin would be the most appropriate choice of antibiotics because together they cover the most
common pathogens in diabetic foot ulcers, as well as MRSA, which is present in 10%32% of diabetic
foot ulcers. This patient has recently been hospitalized and would thus be at high risk for a MRSA
infection. Moderate to severe diabetic foot ulcers are often polymicrobial and can include gram-positive
cocci, gram-negative bacilli, and anaerobic pathogens.

Ref: Gemechu FW, Seemant F, Curley CA: Diabetic foot infections. Am Fam Physician 2013;88(3):177-184.

8
Item 26

ANSWER: A

Unintentional injuries account for 40% of childhood deaths. Motor vehicle accidents are the most frequent
cause of these deaths (58.2% of unintentional deaths). The proper use of child restraints is the most
effective way to prevent injury or death, and the American Academy of Family Physicians and the
American Academy of Pediatrics strongly recommend that physicians actively promote the proper use of
motor vehicle restraints for all patients. Drowning accounts for 10.9% of all unintentional deaths in
children, poisoning for 7.7%, fires 5.7%, and falls 1.4%.

Ref: Theurer WM, Bhavsar AK: Prevention of unintentional childhood injury. Am Fam Physician 2013;87(7):502-509.

Item 27

ANSWER: E

Based on the results of pulmonary function testing, this patient has a pure restrictive pattern with a low
diffusing capacity for carbon monoxide. Pulmonary fibrosis is compatible with this pattern. A patient with
any of the other listed diagnoses would be expected to have an obstructive pattern on testing.

Ref: Johnson JD, Theurer WM: A stepwise approach to the interpretation of pulmonary function tests. Am Fam Physician
2014;89(5):359-366.

Item 28

ANSWER: D

Patients with acute pericarditis should be treated empirically with colchicine and/or NSAIDs for the first
episode of mild to moderate pericarditis. $-Blockers would only be appropriate if the cause of the patients
chest pain were an infarction or ischemia. Nitrates do not relieve the pain of pericarditis. Glucocorticoids
are typically reserved for use in patients with severe or refractory cases or in cases where the likely cause
of the pericarditis is connective tissue disease, autoreactivity, or uremia (SOR C).

Ref: Snyder MJ, Bepko J, White M: Acute pericarditis: Diagnosis and management. Am Fam Physician 2014;89(7):553-560.

Item 29

ANSWER: B

The recommended terminology for weight classification in children is based on age and either BMI (for
children ages 218 years) or weight-for-length ratio (for children ages 02 years). Children under the age
of 2 years are identified as being overweight when their weight-for-length ratio exceeds the 95th percentile
for their sex. The term obese is not used for children under the age of 2 years. Children age 218 years
are appropriately classified as underweight when their BMI falls below the 5th percentile, healthy weight
when their BMI is between the 5th and 85th percentile, overweight when their BMI is between the 85th
and 94th percentile, and obese when their BMI is in the 95th or greater percentile. There is currently no
standard definition of childhood morbid obesity, but obesity is sometimes classified as severe or extreme
when a childs BMI is at the 99th percentile or greater.

9
Ref: Division of Nutrition, Physical Activity and Obesity, National Center for Chronic Disease Prevention and Health
Promotion: Basics about childhood obesity. Centers for Disease Control and Prevention, 2012. 2) Fitch A, Fox C, Bauerly
K, et al: Prevention and Management of Obesity for Children and Adolescents. Institute for Clinical Systems Improvement,
2013.

Item 30

ANSWER: E

Primary amenorrhea is defined as a history of no menses in a female 13 years of age or older with no
pubertal development, or 5 years after initial breast development, or in a patient older than 15 years.
Primary amenorrhea is typically due to chromosomal problems that lead to primary ovarian insufficiency
or anatomic abnormalities. If the patient has dysmorphic features such as short stature, a low hairline, or
a webbed neck, the suspicion for Turners syndrome should be high. While FSH and LH levels may be
elevated, the definitive diagnosis would be made from a karyotype.

Ref: Klein DA, Poth MA: Amenorrhea: An approach to diagnosis and management. Am Fam Physician 2013;87(11):781-788.

Item 31

ANSWER: A

There is no indication for cardiac testing in a low-risk asymptomatic person, and testing may lead to harm
resulting from false positives. The U.S. Preventive Services Task Force does not recommend resting or
stress EKG testing for asymptomatic low-risk patients (D recommendation). Asymptomatic patients should
be risk stratified to assess the risk of chronic heart disease, and this patient should have a lipid profile for
risk stratification. Low-risk patients do not benefit from nontraditional risk assessments, including
high-sensitivity C-reactive protein or coronary artery calcium assessment.

Ref: Final Recommendation Statement: Coronary Heart Disease: Screening Using Non-Traditional Risk Factors. US Preventive
Services Task Force, 2009. 2) Final Recommendation Statement: Coronary Heart Disease: Screening with
Electrocardiography. US Preventive Services Task Force, 2012. 3) Goff DC Jr, Lloyd-Jones DM, Bennett G, et al;
American College of Cardiology/American Heart Association Task Force on Practice Guidelines: 2013 ACC/AHA guideline
on the assessment of cardiovascular risk: A report of the American College of Cardiology/American Heart Association Task
Force on Practice Guidelines. Circulation 2014;129(25 Suppl 2):S49-S73.

Item 32

ANSWER: D

Respiratory syncytial virus (RSV) is a common cause of respiratory tract infections in children. The
infections are usually self-limited and are rarely associated with bacterial co-infection, but in very young
infants, prematurely born infants, or those with pre-existing heart/lung conditions, the infection can be
severe. In North America, RSV season is November to April. Treatment is primarily supportive, including
a trial of bronchodilators, with continued use only if there is an immediate response. Corticosteroids and
antibiotics are not routinely indicated (SOR B). Routine laboratory and radiologic studies should not be
used in making the diagnosis, as it is based on the history and physical examination (SOR C).

Ref: Dawson-Caswell M, Muncie HL Jr: Respiratory syncytial virus infection in children. Am Fam Physician
2011;83(2):141-146.

10
Item 33

ANSWER: A

This patient is most likely suffering from a drug-induced myopathy caused by simvastatin, which is
associated with elevated creatine kinase. Polymyalgia rheumatica is usually associated with an elevated
erythrocyte sedimentation rate. Guillain-Barr syndrome is associated with depressed deep tendon reflexes.
This case has no clinical features or laboratory findings that suggest ketoacidosis.

Ref: Saguil A: Evaluation of the patient with muscle weakness. Am Fam Physician 2005;71(7):1327-1336. 2) Ahmad Z: Statin
intolerance. Am J Cardiol 2014;113(10):1765-1771.

Item 34

ANSWER: A

Amoxicillin is the recommended first-line treatment for previously healthy infants and school-age children
with mild to moderate community-acquired pneumonia (CAP) (strong recommendation; moderate-quality
evidence). The most prominent bacterial pathogen in CAP in this age group is Streptococcus pneumoniae,
and amoxicillin provides coverage against this organism. Azithromycin would be an appropriate choice
in an older child because Mycoplasma pneumoniae would be more common. Moxifloxacin should not be
used in children. Ceftriaxone and cefdinir can both be used to treat CAP, but they are broader spectrum
antibiotics and would not be a first-line choice in this age group.

Ref: Bradley JS, Byington CL, Shah SS, et al; Pediatric Infectious Diseases Society and the Infectious Diseases Society Of
America: The management of community-acquired pneumonia in infants and children older than 3 months of age: Clinical
practice guidelines by the Pediatric Infectious Diseases Society and the Infectious Diseases Society of America. Clin Infect
Dis 2011;53(7):e25-e76.

Item 35

ANSWER: A

When compared to a figure-of-eight dressing, a sling has been shown to have similar fracture healing rates
in patients with a nondisplaced midshaft clavicular fracture. In addition, a figure-of-eight dressing is
uncomfortable and difficult to adjust, and patients have reported increased satisfaction when treated with
a sling. Long and short arm casts are not appropriate options to manage a patient with a clavicular fracture.
Operative treatment is an option to treat displaced midshaft fractures (SOR B).

It should be noted that a Cochrane review of interventions for clavicle fracture pointed out that the studies
of this problem were done in the 1980s and did not meet current standards. One of the conclusions of this
review was that further research should be done.

Ref: Pecci M, Kreher JB: Clavicle fractures. Am Fam Physician 2008;77(1):65-70. 2) Lenza M, Belloti JC, Andriolo RB,
Faloppa F: Conservative interventions for treating middle third clavicle fractures in adolescents and adults. Cochrane
Database Syst Rev 2014;(5):CD007121.

11
Item 36

ANSWER: E

In children, obstructive sleep apnea (OSA) is most often due to enlarged tonsils and adenoids. OSA onset
is usually between 2 and 8 years of age, coinciding with peak tonsil growth. Adenotonsillectomy is the
primary treatment for most non-obese children with OSA (SOR B). SSRIs are sometimes effective in
treating nightmares because these medications can suppress rapid eye movement sleep. Benzodiazepines
are an option for treating sleep terrors. Methylphenidate is a stimulant used to treat
attention-deficit/hyperactivity disorder and has no benefit for OSA. The use of a mouthguard at night is
recommended for management of temporomandibular joint syndrome to reduce excessive teeth grinding
during sleep. It is not a treatment for OSA.

Ref: American Academy of Sleep Medicine: International Classification of Sleep Disorders: Diagnostic and Coding Manual, ed
2nd. American Academy of Sleep Medicine, 2005. 2) Marcus CL, Brooks LJ, Draper KA, et al: Diagnosis and
management of childhood obstructive sleep apnea syndrome. Pediatrics 2012;130(3):e714-e755. 3) Carter KA, Hathaway
NE, Lettieri CF: Common sleep disorders in children. Am Fam Physician 2014;89(5):368-377.

Item 37

ANSWER: D

Chronic kidney disease (CKD) is now divided into five stages of progressively worsening function based
on the glomerular filtration rate (GFR). Stage 1 is defined as a GFR >90 mL/min/1.73 m2, while the fifth
stage, kidney failure, is defined as a GFR <15 mL/min/1.73 m2. Anemia is associated with not only stage
5 disease, where it is universal, but also with earlier stages. The National Kidney Foundation Guidelines
define anemia as a hemoglobin level 13.5 g/dL in men or 12.0 g/dL in women.

Anemia due to CKD is diagnosed by excluding other etiologies. Anemia in CKD is due to decreased
production of erythropoietin, but testing for levels is not needed, nor is a bone marrow biopsy. The
indicated tests include a CBC, reticulocyte count, ferritin level, vitamin B12 level, folate level, and
transferrin saturation (serum iron to total iron binding capacity ratio). Usually the CBC will demonstrate
a normochromic, normocytic anemia, but can show microcytosis (mean corpuscular volume <80). A
serum ferritin level <25 ng/mL is indicative of low iron stores. Some patients have a combination of iron
deficiency and anemia of chronic disease due to the kidney disease.

Patients with depleted iron stores will benefit from replenishment, which serves to correct an isolated iron
deficiency or improve the response to erythropoiesis-stimulating agents. Iron therapy is generally initiated
orally with ferrous sulfate, 325 mg 3 times a day. The effectiveness of this therapy can be monitored by
checking hemoglobin, transferrin saturation, and ferritin levels at 1 and 3 months after beginning
treatment. If the goals have not been achieved by 3 months, intravenous iron therapy should be considered.

For patients who do not respond to iron replacement, erythropoiesis-stimulating agents such as epoetin alfa
or darbepoetin alfa should be used. The goal should be to relieve symptoms such as fatigue and to achieve
a hemoglobin level of 1112 g/dL. Levels >13 g/dL increase the mortality rate, particularly from
cardiovascular disease.

Ref: Bross MH, Soch K, Smith-Knuppel T: Anemia in older persons. Am Fam Physician 2010;82(5):480-487. 2) Rivera JA,
OHare AM, Harper GM: Update on the management of chronic kidney disease. Am Fam Physician 2012;86(8):749-754.
3) Fiore DC, Fox CL: Urology and Nephrology Update. American Academy of Family Physicians, FP Essentials
monograph series, no 416, 2014, pp 22-25.

12
Item 38

ANSWER: B

Heart failure due to diastolic dysfunction occurs in the older population. The criteria for diastolic heart
failure include symptoms and signs consistent with heart failure (including dyspnea), a nondilated left
ventricle with a preserved ejection fraction (50%), and evidence of structural heart disease such as
diastolic dysfunction on echocardiography (SOR C).

Ref: Argulian E, Messerli FH: Misconceptions and facts about diastolic heart failure. Am J Med 2014;127(12):1144-1147.

Item 39

ANSWER: C

Slipped capital femoral epiphysis (SCFE) occurs most commonly during the adolescent growth spurt
(1113 years of age for girls, 1315 years of age for boys). While the cause is unknown, associated factors
include anatomic variables such as femoral retroversion or steeper inclination of the proximal femoral
physis, in addition to being overweight. African-Americans are affected more commonly as well.

The patient may present with pain in the groin or anterior thigh, but also may present with pain referred
to the knee. That is also the case for Legg-Calv-Perthes disease, also known as avascular or aseptic
necrosis of the femoral head. This condition most commonly occurs in boys 48 years of age. In addition
to hip (or knee) pain, limping is a prominent feature.

Upper thigh numbness in an adolescent female is a classic symptom of meralgia paresthetica, which is
attributed to impingement of the lateral femoral cutaneous nerve in the groin, often associated with obesity
or wearing clothing that is too tight in the waist or groin. Developmental dysplasia of the hip is identified
by a click during a provocative hip examination of the newborn, using both the Barlow and Ortolani
maneuvers to detect subluxation or dislocation.

Ref: Sarwark JF (ed): Essentials of Musculoskeletal Care, ed 4. American Academy of Orthopaedic Surgeons, 2010, pp
1050-1053, 1122-1126, 1180-1183.

Item 40

ANSWER: A

Patients with symptomatic peripheral arterial disease should be started on a daily dose of either aspirin or
clopidogrel to prevent cardiovascular events such as acute myocardial infarction or stroke (SOR B).
Cilostazol is a phosphodiesterase inhibitor with both antiplatelet and arterial vasodilatory activity. It has
been shown to improve claudication symptoms by 50% compared to placebo. Likewise, pentoxifylline is
also used in the treatment of claudication symptoms but is less effective than cilostazol and is reserved as
a second-line agent. Neither agent has been shown to decrease cardiovascular events in patients with
symptomatic peripheral artery disease. Neither enoxaparin nor warfarin is indicated for symptomatic
peripheral artery disease.

Ref: US Preventive Services Task Force: Screening for peripheral arterial disease: Recommendation statement. Am Fam
Physician 2006;73(3):497-500. 2) Hauk L: ACCF/AHA update peripheral artery disease management guideline. Am Fam
Physician 2012;85(10):1000-1001. 3) Hennion DR, Siano KA: Diagnosis and treatment of peripheral arterial disease. Am
Fam Physician 2013;88(5):306-310.

13
Item 41

ANSWER: D

This patient has signs and symptoms of a hip labral tear. This causes dull or sharp groin pain, which in
some patients radiates to the lateral hip, anterior thigh, or buttock. The pain usually has an insidious onset,
but occasionally begins acutely after a traumatic event. Half of patients also have mechanical symptoms,
such as catching or painful clicking with activity. The FADIR and FABER tests are effective for detecting
intra-articular pathology (the sensitivity is 75%96% for the FADIR test and 88% for the FABER test),
although neither test has high specificity. Magnetic resonance arthrography is considered the diagnostic
test of choice for labral tears, as it has a sensitivity of 90% and an accuracy of 91%. However, if a labral
tear is not suspected, less invasive imaging modalities such as plain radiography and conventional MRI
should be used first to assess for other causes of hip and groin pain.

This patient has no history of trauma or risk factors to suggest a fracture. A femoral hernia would typically
present as pain that is worse with straining or lifting, associated with a palpable bulge in the upper thigh.
Trochanteric bursitis typically causes lateral hip pain with point tenderness over the greater trochanter of
the femur.

Ref: Wilson JJ, Furukawa M: Evaluation of the patient with hip pain. Am Fam Physician 2014;89(1):27-34.

Item 42

ANSWER: D

Shared decision-making should include a discussion of risks and benefits that are meaningful to the
individual patient. It is an important component of patient-centered care, but published studies often report
intermediate endpoints. Patient-oriented outcomes typically include data on mortality (especially all-cause
mortality because changes in disease-specific mortality may be offset by changes in other causes of
mortality), morbidity, symptoms, and quality of life. Intermediate endpoints typically involve
disease-oriented data, including histologic, physiologic, or clinical measurements such as blood pressure,
carotid intimal thickness, hemoglobin A1c, and risk scores such as the Framingham score.

Ref: Sanders AR, van Weeghel I, Vogelaar M, et al: Effects of improved patient participation in primary care on health-related
outcomes: A systematic review. Fam Pract 2013;30(4):365-378.

Item 43

ANSWER: B

Fever and febrile seizures may occur after administration of several vaccines. Postimmunization seizures,
especially febrile seizures, occur at a higher rate in children who have a past history of seizures or a
first-degree relative with a history of seizures. The benefits of the vaccines outweigh the risks, so they are
not contraindicated in this situation, although the parents need to be cautioned about the increased risk of
seizure.

14
Of the vaccines listed, the only one likely to put the child at risk for a seizure up to 2 weeks after
administration is the MMR vaccine. Specifically, it is the measles component of the vaccine that is the
potential culprit. A temperature of 39.4C (103F) or higher develops in approximately 5%15% of
susceptible vaccine recipients, usually 612 days after receipt of MMR vaccine. The fever generally lasts
12 days but may last up to 5 days.

Ref: Possible side-effects from vaccines. Centers for Disease Control and Prevention website, 2013.

Item 44

ANSWER: D

This patient has injured his ulnar collateral ligament (UCL). The UCL is the primary restraint to valgus
stress on the elbow during overhead throwing. These injuries often occur in athletes participating in sports
that require overhead throwing, such as baseball, javelin, and volleyball. Patients often report a pop
followed by immediate pain and bruising around the medial elbow. The moving valgus stress test has 100%
sensitivity and 75% specificity for diagnosing UCL injuries.

Medial epicondylitis usually presents with an insidious onset of pain related to a recent increase in
occupational or recreational activities. Patients also often report weakened grip strength. The point of
maximal tenderness is 510 mm distal to and anterior to the medial epicondyle. It is most often a
tendinopathy of the flexor carpi radialis and the pronator teres.

Biceps tendinopathy usually presents with a history of vague anterior elbow pain and a history of repeated
elbow flexion with forearm supination and pronation, such as dumbbell curls. Resisted supination produces
pain deep in the antecubital fossa.

Cubital tunnel syndrome is a neuropathy of the ulnar nerve caused by compression or traction as it passes
through the cubital tunnel of the medial elbow. The onset of pain is more insidious than UCL injury,
occurring with repetitive activity, and is usually accompanied by numbness and tingling in the ulnar border
of the forearm and hand. If it has existed for some time, the intrinsic hand muscle may become weak.

Tendinopathy of the triceps insertion is more common in weight lifters or athletes who repetitively extend
their elbows against resistance. Pain occurs at the posterior elbow with resisted extension, and tenderness
is located over the triceps insertion.

Ref: Kane SF, Lynch JH, Taylor JC: Evaluation of elbow pain in adults. Am Fam Physician 2014;89(8):649-657.

Item 45

ANSWER: B

The live attenuated intranasal influenza vaccine is recommended for healthy nonpregnant persons 249
years of age. It is more effective than the inactivated vaccine in children 26 years of age; for patients
649 years of age either the live attenuated intranasal or the inactivated vaccine is recommended. The live
intranasal vaccine is contraindicated in pregnancy and in patients with asthma or COPD. Patients older than
49 years should receive the inactivated vaccine.

Ref: Influenza vaccine for 20142015. Med Lett Drugs Ther 2014;56(1453):97-99.

15
Item 46

ANSWER: E

All of the pathogens listed can cause pneumonia in any patient. However, in patients with chronic lung
disease who are taking corticosteroids, Pseudomonas is more common than in those with otherwise healthy
lungs. The antibiotics chosen empirically should cover this pathogen.

Ref: Musher DM, Thorner AR: Community-acquired pneumonia. N Engl J Med 2014;371(17):1619-1628.

Item 47

ANSWER: D

This patient scores in the moderate range for withdrawal severity and is a candidate for pharmacotherapy,
based on the Clinical Institute Withdrawal Assessment for Alcohol Scale, Revised, and the Short Alcohol
Withdrawal Scale. He also has no known contraindications to outpatient treatment, such as abnormal
laboratory results, absence of a support network, acute illness, high risk for delirium tremens, history of
alcohol withdrawal seizure, recent long-term intake of large amounts of alcohol, poorly controlled chronic
medical conditions, a serious psychiatric condition, severe withdrawal symptoms, or a positive urine drug
screen.

Benzodiazepines are the preferred medication for treating alcohol withdrawal (SOR A) and preventing
alcohol withdrawal seizures. There is no evidence that indicates that any particular medication is superior,
but long-acting benzodiazepines are preferred. Neither fixed nor symptom-triggered dosing of
benzodiazepines has been shown to be superior to the other. Although anticonvulsants have less abuse
potential than benzodiazepines, they do not prevent seizures or delirium tremens. Clonidine and $-blockers
can help reduce adrenergic symptoms but do not prevent alcohol withdrawal seizures. Thiamine or
magnesium may be appropriate to address nutritional deficiencies resulting from alcoholism but would not
reduce withdrawal symptoms.

Ref: Muncie HL Jr, Yasinian Y, Oge L: Outpatient management of alcohol withdrawal syndrome. Am Fam Physician
2013;88(9):589-595.

Item 48

ANSWER: D

Whole cows milk does not supply infants with enough vitamin E, iron, and essential fatty acids, and
overburdens them with too much protein, sodium, and potassium. Skim and low-fat milk lead to the same
problems as whole milk, and also fail to provide adequate calories for growth. For these reasons cows
milk is not recommended for children under 12 months of age. Human breast milk or iron-fortified
formula, with introduction of certain solid foods and juices after 46 months of age if desired, is
appropriate for the first year of life.

Ref: Allen RE, Myers AL: Nutrition in toddlers. Am Fam Physician 2006;74(9):1527-1532. 2) Kliegman RM, Stanton BF,
Geme JW III, et al (eds): Nelson Textbook of Pediatrics, ed 19. Elsevier Saunders, 2011, p 164.

16
Item 49

ANSWER: A

It is usually simple to reduce a lateral patellar dislocation, and these injuries rarely require acute surgical
management. The proper technique is to have the patient sit or lie with the leg in a flexed position and then
apply gentle medial pressure to the patella until the most lateral edge is over the femoral condyle. The leg
should then be gently extended and the knee brought into full extension. This should cause the patella to
slip back into place, and the knee should then be immobilized.

Ref: Thomsen TW, Setnik GS: Dislocation reduction of the patella (Orthopedics). Procedures Consult 2012. 2)
Subluxation/dislocation of the patella. Wheeless Textbook of Orthopaedics. Duke Orthopaedics, 2015.
www.wheelessonline.com/ortho/subluxation_dislocation_of_the_patella

Item 50

ANSWER: D

The U.S. Selected Practice Recommendations for Contraceptive Use, 2013, focuses on optimizing the use
of contraceptive methods. Perceived obstacles to obtaining guidance and prescription contraceptives are
identified, and are addressed by recommendations that facilitate the ease of obtaining both. In the absence
of coexisting medical conditions that may require additional evaluation or limit contraceptive options, the
only medical evaluation identified as essential before prescription of combined hormonal contraception is
measurement of blood pressure, as severe hypertension is a contraindication to oral contraception.
Ascertaining the likelihood of pregnancy based on the sexual history and/or phase of the menstrual cycle
is also necessary before determining a start date. Each of the listed examination options is ideal, but none
has been found to contribute substantially to the safe and effective use of combined hormonal
contraceptives. A baseline weight is useful in determining excessive weight gain on reevaluation following
the initiation of oral contraception, but it is not a prerequisite.

Ref: Division of Reproductive Health, National Center for Chronic Disease Prevention and Health Promotion, Centers for
Disease Control and Prevention (CDC): U.S. Selected Practice Recommendations for Contraceptive Use, 2013: Adapted
from the World Health Organization Selected Practice Recommendations for Contraceptive Use, 2nd edition. MMWR
Recomm Rep 2013;62(RR-05):1-60.

Item 51

ANSWER: B

Opioid-induced hyperalgesia is characterized by a paradoxical increase in sensitivity to pain despite an


increase in the opioid dosage. It is seen in patients who are receiving high doses of parenteral opioids such
as morphine. Patients report the development of diffuse pain away from the site of the original pain.
Allodynia, a perception of pain in the absence of a painful stimulus, is also typical in opioid-induced
hyperalgesia. Strategies to manage this condition include reducing the current opioid dosage, and
occasionally eliminating the current opioid and starting another opioid. The addition of non-opioid pain
medications should also be considered. The addition of an anxiolytic is not likely to improve this patients
pain (SOR C).

Ref: Groninger H, Vijayan J: Pharmacologic management of pain at the end of life. Am Fam Physician 2014;90(1):26-32.

17
Item 52

ANSWER: D

Corticosteroids, either orally or parenterally, are the first-line treatment for acute exacerbations of multiple
sclerosis (MS) (SOR A). A Cochrane review found no significant differences in outcomes based on the
route of administration. Disease-modifying agents such as interferon beta, glatiramer, and
immunosuppressants such as fingolimod may decrease the frequency of exacerbations and slow the
progression of MS but are not the agents of first choice for treatment of acute flareups. Pramipexole does
not have a primary role in the treatment of MS, although it might be used to treat certain specific
symptoms as an adjunct therapy.

Ref: Saguil A, Kane S, Farnell E: Multiple sclerosis: A primary care perspective. Am Fam Physician 2014;90(9):644-652.

Item 53

ANSWER: E

Serotonin syndrome is a result of increased serotonergic activity in the central nervous system and may
be life-threatening. It is usually a combination of autonomic hyperactivity, neuromuscular abnormality,
and mental status changes. The most common group of medications that may cause this is the SSRIs.
Serotonin syndrome most commonly occurs in the first 24 hours of treatment. Patients often present with
agitation and confusion, tachycardia, and elevated blood pressure, as well as a dry mouth. While there are
usually no focal neurologic findings, hyperreflexia and even spontaneous clonus may be seen. The finding
of slow, horizontal movement of the eyes is also helpful in making the diagnosis.

The initial management is to discontinue the offending agent, begin supportive care, and attempt to calm
the patient verbally. Many times medication is needed, and the drug of choice is an intravenous
benzodiazepine such as lorazepam or diazepam. If treatment for tachycardia or hypertension is needed,
propranolol should not be used due to its longer activity. Haloperidol should be avoided, as it may actually
increase anticholinergic activity. Flumazenil is rarely used, although it has been used for tricyclic
antidepressant overdosage, and it carries a significant risk of inducing seizures. If the patient does not
respond to calming with benzodiazepines, the antidote would be cyproheptadine.

Ref: Tintinalli JE, Kelen GD, Stapczynski JS (eds): Emergency Medicine: A Comprehensive Study Guide, ed 7. McGraw-Hill,
2011, pp 1202-1203. 2) Bienvenu OJ, Neufeld KJ, Needham DM: Treatment of four psychiatric emergencies in the
intensive care unit. Crit Care Med 2012;40(9):2662-2670.

Item 54

ANSWER: E

Diagnostic criteria for sepsis include leukocytosis. Diagnostic criteria for severe sepsis (sepsis plus organ
dysfunction) include an increase in the serum creatinine level >0.5 mg/dL, thrombocytopenia, and
hyperbilirubinemia. A diagnosis of septic shock requires either hyperlactatemia or hypotension refractory
to intravenous fluids.

Ref: Angus DC, van der Poll T: Severe sepsis and septic shock. N Engl J Med 2013;369(9):840-851.

18
Item 55

ANSWER: C

Positive predictive value refers to the percentage of patients with a positive test for a disease who actually
have the disease. The negative predictive value of a test is the proportion of patients with negative test
results who do not have the disorder.

The percentage of patients with a disorder who have a positive test for that disorder is a tests sensitivity.
The percentage of patients without a disorder who have a negative test for that disorder is a tests
specificity.

Ref: Goldman L, Schafer AI (eds): Goldmans Cecil Medicine, ed 25. Elsevier Saunders, 2016, pp 37-41.

Item 56

ANSWER: C

Gilberts syndrome is a hereditary condition associated with unconjugated hyperbilirubinemia (usually with
a bilirubin level <5.0 mg/dL). The bilirubin level increases with infection, exertion, and fasting. Patients
are asymptomatic and have otherwise normal liver function studies. The differential diagnosis includes
hemolytic anemias, which cause a decrease in serum haptoglobin, an increase in lactate dehydrogenase,
and/or CBC abnormalities, particularly on the peripheral smear.

Ref: VanWagner LB, Green RM: Evaluating elevated bilirubin levels in asymptomatic adults. JAMA 2015;313(5):516-517.

Item 57

ANSWER: A

The U.S. Preventive Services Task Force recommends against screening for cervical cancer for women
younger than 21, for women over the age of 65 who have had adequate screening in the recent past and
are not at high risk, and for women who have had a hysterectomy with removal of the cervix and no
history of CIN 2 or 3 or cervical cancer (USPSTF D recommendation). Women between the ages of 21
and 65 can be screened every 3 years with cytology alone, or the interval can be increased to 5 years after
age 30 by using a combination of cytology and HPV testing (USPSTF A recommendation). The history
of HPV vaccination is not a factor in screening decisions. Other organizations such as the American
Cancer Society and the American College of Obstetricians and Gynecologists have similar guidelines.

Ref: Fontaine PL, Saslow D, King VJ: ACS/ASCCP/ASCP guidelines for the early detection of cervical cancer. Am Fam
Physician 2012;86(6):501, 506-507. 2) Final Recommendation Statement. Cervical Cancer: Screening. US Preventive
Services Task Force, 2014.

19
Item 58

ANSWER: E

Lidocaine buffered with sodium bicarbonate decreases the pain associated with the injection. This effect
is enhanced when the solution is warmed to room temperature (SOR B). Rapidly inserting the needle
through the skin, injecting the solution slowly and steadily while withdrawing the needle, and injecting into
the subcutaneous tissue also minimize the pain of injection.

Ref: Latham JL, Martin SN: Infiltrative anesthesia in office practice. Am Fam Physician 2014;89(12):956-962.

Item 59

ANSWER: B

Asplenic patients who develop a fever should be given antibiotics immediately. Due to the increased risk
of pneumococcal sepsis in asplenic patients, vaccinations against these particular bacteria are specifically
recommended. Since pneumococcal conjugate vaccine (PCV13) and pneumococcal polysaccharide vaccine
(PPSV23) can interact with each other they should be given at least 8 weeks apart. Prophylactic penicillin
given orally twice a day is particularly important in children under 5 years of age who are asplenic, and
may be considered for 12 years post splenectomy in older patients. Lifelong daily antibiotics may be
considered following post-splenectomy sepsis. The risk for Haemophilus influenzae type b infection is not
increased in asplenic patients, so additional vaccine is not needed for those who have already been
vaccinated. Live attenuated influenza vaccine may be used in asplenic patients, unless they have sickle cell
disease.

Ref: Rubin LG, Schaffner W: Care of the asplenic patient. N Engl J Med 2014;371(4):349-356.

Item 60

ANSWER: B

This patient has de Quervains tenosynovitis. Finkelsteins test has good sensitivity and specificity (SOR
C) in patients with a negative grind test. A positive grind test would be more consistent with scaphoid
fracture. A hand radiograph with secondary thumb spica splinting would be appropriate for a suspected
scaphoid fracture, but the insidious onset as opposed to overt trauma makes this diagnosis unlikely in this
case. A short arm cast is not indicated in de Quervains tenosynovitis but may be appropriate for
forearm/wrist fractures.

Ref: Tallia AF, Cardone DA: Diagnosis and therapeutic injection of the wrist and hand region. Am Fam Physician
2003;67(4):745-750. 2) Shehab R, Mirabelli MH: Evaluation and diagnosis of wrist pain: A case-based approach. Am Fam
Physician 2013;87(8):568-573.

20
Item 61

ANSWER: E

Methicillin-resistant Staphylococcus aureus (MRSA) is the predominant cause of suppurative skin and
soft-tissue infection. While community-acquired strains have been susceptible to many antibiotics,
clindamycin is associated with Clostridium difficile enterocolitis, trimethoprim/sulfamethoxazole is usually
used orally only for outpatient treatment, and doxycycline and minocycline are often effective clinically
but seldom used for serious infections. Resistance to quinolones is increasing and may emerge during
treatment. Vancomycin given parenterally is generally still the drug of choice for hospitalized patients.

Ref: Drugs for MRSA skin and soft-tissue infections. Med Lett Drugs Ther 2014;56(1442):39-40.

Item 62

ANSWER: D

Patients with chronic illness, diabetes mellitus, cerebrospinal fluid leaks, chronic bronchopulmonary
dysplasia, cyanotic congenital heart disease, or cochlear implants should receive one dose of pneumococcal
polysaccharide vaccine after 2 years of age, and at least 2 months after the last dose of pneumococcal
conjugate vaccine. Revaccination with polysaccharide vaccine is not recommended for these patients.
Individuals with sickle cell disease, those with anatomic or functional asplenia, immunocompromised
persons with renal failure or leukemia, and HIV-infected persons should receive polysaccharide vaccine
on this same schedule and should also be revaccinated at least 3 years after the first dose.

Ref: Nuorti JP, Whitney CG; Centers for Disease Control and Prevention (CDC): Prevention of pneumococcal disease among
infants and childrenUse of 13-valent pneumococcal conjugate vaccine and 23-valent pneumococcal polysaccharide
vaccineRecommendations of the Advisory Committee on Immunization Practices (ACIP). MMWR Recomm Rep
2010;59(RR-11):1-18. 2) Centers for Disease Control and Prevention (CDC): Use of 13-valent pneumococcal conjugate
vaccine and 23-valent pneumococcal polysaccharide vaccine among children aged 618 years with immunocompromising
conditions: Recommendations of the Advisory Committee on Immunization Practices (ACIP). MMWR Morb Mortal Wkly
Rep 2013;62(25):521-524.

Item 63

ANSWER: E

Spironolactone, an aldosterone antagonist, can bind to androgen and progesterone receptors, in addition
to the mineralocorticoid receptors, resulting in breast tenderness and gynecomastia. Eplerenone, another
aldosterone antagonist, has greater specificity for the mineralocorticoid receptors and is therefore less
likely to cause breast tenderness and gynecomastia than spironolactone. While there have been case reports
of gynecomastia with ACE inhibitors and digoxin, it is noted to be rare. The side effect profile of
hydralazine does not include gynecomastia.

Ref: Bowman JD, Kim H, Bustamante JJ: Drug-induced gynecomastia. Pharmacotherapy 2012;32(12):1123-1140. 2) Morcos
RN, Kizy T: Gynecomastia: When is treatment indicated? J Fam Pract 2012;61(12):719-725.

21
Item 64

ANSWER: E

Although infertility issues may be very complex, the primary care physician can initiate an appropriate
workup. For women who are having regular menstrual cycles, ovulation is very likely. Ovulation can be
confirmed by a progesterone level 5 ng/mL on day 21 of the cycle. If this is the case, tubal patency
should be confirmed with hysterosalpingography or laparoscopy. Obstruction or adhesions would require
surgical correction, but if there are none, referral for assisted reproductive technology would be
appropriate.

Should the progesterone level be <5 ng/mL, anovulation should be investigated with TSH, estradiol, FSH,
and prolactin levels. Treatment can be initiated if findings reveal the cause of the problem, but if they are
unremarkable it is reasonable to try clomiphene to induce ovulation. If this is unsuccessful, referral would
be the next step.

Ref: Lindsay TJ, Vitrikas KR: Evaluation and treatment of infertility. Am Fam Physician 2015;91(5):308-314.

Item 65

ANSWER: D

U.S. Preventive Services Task Force (USPSTF) guidelines recommend that asymptomatic adults with
sustained blood pressure >135/80 mm Hg be screened for type 2 diabetes mellitus using fasting plasma
glucose, a 2-hour glucose tolerance test, or hemoglobin A1c measurements (USPSTF B recommendation).
Screening for colon cancer with either annual high-sensitivity fecal occult blood testing, sigmoidoscopy
every 5 years, or colonoscopy every 10 years is also recommended for adults between the ages of 50 and
75 years (USPSTF A recommendation). Men who have ever smoked (defined as 100 or more cigarettes)
should be screened once for abdominal aortic aneurysm (USPSTF B recommendation) between the ages
of 65 and 75. Similar screening is recommended in men who have never smoked, but this is a USPSTF
grade C recommendation. No recommendation has been made with regard to screening for peripheral
vascular disease, and the recommendation on screening for hemochromatosis is listed as inactive on the
USPSTF website.

Ref: Final Recommendation Statement: Diabetes Mellitus (Type 2) in Adults: Screening. US Preventive Services Task Force,
2008. 2) Final Recommendation Statement: Colorectal Cancer: Screening. US Preventive Services Task Force, 2008. 3)
Final Recommendation Statement: Peripheral Arterial Disease (PAD) and CVD in Adults: Risk Assessment with
Ankle/Brachial Index. US Preventive Services Task Force, 2013. 4) Final Recommendation Statement: Abdominal Aortic
Aneurysm: Screening. US Preventive Services Task Force, 2013.

Item 66

ANSWER: B

As is true for the general population in the United States, coronary artery disease is the leading cause of
death in patients with rheumatoid arthritis (RA). RA patients have accelerated atherosclerosis related to
a chronic inflammatory state. It is thus particularly important to address modifiable risk factors for
coronary disease in these patients, including tobacco use, hypertension, and dyslipidemia. Patients with
RA also have an increased risk of lymphoma, lung cancer, and thromboembolic disease, but these are not
as common as coronary disease. Infections are a concern for patients on disease-modifying agents but are
not the leading cause of death.

22
Ref: Wasserman AM: Diagnosis and management of rheumatoid arthritis. Am Fam Physician 2011;84(11):1245-1252. 2) Bacani
AK, Gabriel SE, Crowson CS, et al: Noncardiac vascular disease in rheumatoid arthritis: Increase in venous
thromboembolic events? Arthritis Rheum 2012;64(1):5361.

Item 67

ANSWER: D

An FVC that falls below the lower limit of normal (LLN), defined as the fifth percentile of spirometry data
obtained from the Third National Health and Nutrition Examination Survey, is consistent with a restrictive
pattern of pulmonary function. An FEV1/FVC less than the LLN is consistent with an obstructive defect.
A mixed pattern exists when both values are below the LLN, as in this case. The patient should now be
referred for full pulmonary function testing, including diffusing capacity of the lungs for carbon monoxide
(DLCO).

DLCO is a quantitative measure of gas transfer in the lungs. Diseases that decrease blood flow to the lungs
or that damage alveoli will lead to less efficient gas exchange and result in a lower DLCO value.
Bronchoprovocation (a methacholine challenge, a mannitol inhalation challenge, or exercise testing) should
be performed if pulmonary function test results are normal but exercise- or allergen-induced asthma is
suspected.

Ref: Johnson JD, Theurer WM: A stepwise approach to the interpretation of pulmonary function tests. Am Fam Physician
2014;89(5):359-366.

Item 68

ANSWER: A

There is little evidence that femoral anteversion causes long-term functional problems. Studies have shown
that shoe wedges, torque heels, and twister cable splints are not effective. Surgery should be reserved for
children 810 years of age who still have cosmetically unacceptable, dysfunctional gaits. Major
complications of surgery occur in approximately 15% of cases, and can include residual in-toeing,
out-toeing, avascular necrosis of the femoral head, osteomyelitis, fracture, valgus deformity, and loss of
position. Thus, observation alone is appropriate for a 5-year-old with uncomplicated anteversion.

Ref: Mooney JF 3rd: Lower extremity rotational and angular issues in children. Pediatr Clin North Am 2014;61(6):1175-1183.
2) Kliegman RM, Stanton BF, Geme JW III, et al (eds): Nelson Textbook of Pediatrics, ed 19. Elsevier Saunders, 2011,
pp 2344-2346. 3) Talley W, Goodemote P, Henry SL: Managing intoeing in children. Am Fam Physician
2011;84(8):937-944.

Item 69

ANSWER: E

In obsessive-compulsive disorder (OCD), intrusive thoughts cause anxiety, which patients suppress with
recurring behaviors. Various types of psychotherapy have been tried, but repeated exposure to fearful
stimuli has been the best. Repeated and prolonged exposure to stimuli that elicit fear, combined with strict
avoidance of any compulsive behaviors, seems to be the most effective method for controlling the
obsessive-compulsive behaviors. Tricyclic antidepressants and SSRIs are also effective for treating OCD.

23
Freudian analysis is ineffective for relieving the anxiety associated with OCD. Benzodiazepines can help
with anxiety but do little for long-term control, while amphetamines aggravate anxiety and are not helpful.
Atypical antipsychotics may help with other mental disorders associated with obsessive-compulsive
behavior but do not treat the disorder itself.

Ref: Grant JE: Obsessive-compulsive disorder. N Engl J Med 2014;371(7):646-653.

Item 70

ANSWER: B

Antiviral drugs are useful for treatment of acute herpes zoster but not for treatment of postherpetic
neuralgia. Herpes zoster vaccine can prevent postherpetic neuralgia by reducing the incidence of herpes
zoster but it has no role in the treatment of neuralgia. Neither acupuncture nor epidural corticosteroid
injections are helpful in treating postherpetic neuralgia. Topical agents such as lidocaine patches and
capsaicin cream or patches have been shown to reduce symptoms of postherpetic neuralgia, as have the
oral agents gabapentin, pregabalin, and amitriptyline.

Ref: Johnson RW, Rice AS: Postherpetic neuralgia. N Engl J Med 2014;371(16):1526-1533.

Item 71

ANSWER: E

Early diagnosis of a nontraumatic subarachnoid hemorrhage is paramount for achieving a good outcome
when a patient presents with a headache that is unusually severe and feels different than other headaches.
Risk factors include smoking, hypertension, heavy alcohol use, and a family history of aneurysm or
hemorrhagic stroke. The initial evaluation should consist of noncontrast CT of the head (SOR C). If it is
negative or equivocal the next step would be to perform a lumbar puncture to determine whether or not
the cerebrospinal fluid is xanthochromic. The absence of xanthochromia rules out subarachnoid
hemorrhage (SOR C).

Ref: Cohen-Gadol AA, Bohnstedt BN: Recognition and evaluation of nontraumatic subarachnoid hemorrhage and ruptured
cerebral aneurysm. Am Fam Physician 2013;88(7):451-456. 2) Perry JJ, Stiell IG, Sivilotti ML, et al: Clinical decision
rules to rule out subarachnoid hemorrhage for acute headache. JAMA 2013;310(12):1248-1255.

Item 72

ANSWER: B

The Advisory Committee on Immunization Practices advises that the 13-valent pneumococcal vaccine be
given in addition to the 23-valent vaccine, preferably before the 23-valent vaccine. Only one dose of
influenza vaccine is recommended per season. A single dose of 23-valent pneumococcal vaccine is all that
is required.

Ref: In brief: PCV13 for adults 65 years and older. Med Lett Drugs Ther 2014;56:102.

24
Item 73

ANSWER: D

Proton pump inhibitors (PPIs), including omeprazole, are generally safe and effective for peptic ulcer
disease, gastroesophageal reflux disease, and stress ulcer prevention in critically ill patients. As use has
increased, however, risks of long-term use of PPIs have emerged. Currently known risks include increased
fractures of the hip, wrist, and spine (SOR B), community-acquired pneumonia (SOR B), Clostridium
difficile and other enteric infections (SOR C), hypomagnesemia (SOR B), and cardiac events when
coadministered with clopidogrel (SOR B). PPIs may also affect the absorption of vitamins and minerals,
including iron, vitamin B12, and folate (SOR C). There is no known association of PPIs with nephrolithiasis
or urinary tract infections.

Ref: Gill JM, Player MS, Metz DC: Balancing the risks and benefits of proton pump inhibitors. Ann Fam Med
2011;9(3):200202. 2) Ament PW, Dicola DB, James ME: Reducing adverse effects of proton pump inhibitors. Am Fam
Physician 2012;86(1):66-70.

Item 74

ANSWER: D

This patient has symptoms typical of a mild concussion without loss of consciousness. In such cases
standard neuroimaging can be expected to be normal. The evaluation should include a standard concussion
assessment tool, and if concussion is suspected the athlete should be removed from play. Complete physical
and cognitive rest are required for the first 12 days, but return to normal activity must be individualized
depending on the course of symptoms and response to gradually increasing activity. Athletes should be
completely free of symptoms before returning to sports activities.

Ref: Scorza KA, Raleigh MF, OConnor FG: Current concepts in concussion: Evaluation and management. Am Fam Physician
2012;85(2):123-132. 2) Valentine V, Logan K: Cognitive rest in concussion management. Am Fam Physician
2012;85(2):100-101. 3) Armstrong C: Evaluation and management of concussion in athletes: Recommendations from the
AAN. Am Fam Physician 2014;89(7):585-587.

Item 75

ANSWER: A

ACE inhibitors or angiotensin receptor blockers should be used in all patients with a history of myocardial
infarction and reduced ejection fraction. Aldosterone receptor antagonists are indicated in patients who
have a left ventricular ejection fraction 35%. Nondihydropyridine calcium channel blockers with negative
inotropic effects (verapamil and diltiazem) may be harmful in patients with low left ventricular ejection
fractions. Statin therapy is recommended in all patients with a history of myocardial infarction.
Evidence-based $-blockers (carvedilol or metoprolol succinate) should be used in all patients with a history
of myocardial infarction.

Ref: Yancy CW, Jessup M, Bozkurt B, et al: 2013 ACCF/AHA guideline for the management of heart failure: Executive
summary: A report of the American College of Cardiology Foundation/American Heart Association Task Force on Practice
Guidelines. Circulation 2013;128(16):1810-1852.

25
Item 76

ANSWER: B

Oral osmotics such as polyethylene glycolbased solutions are recommended as an appropriate initial
approach to constipation in children because they are effective, easy to administer, noninvasive, and well
tolerated (SOR C). Rectal therapies are similar in terms of effectiveness but are more invasive and less
commonly used as first-line treatment (SOR A). Oral stimulants and bisacodyl rectal suppositories are not
recommended for children under 2 years of age. Enemas are sometimes used as second-line therapy, but
the addition of enemas to oral laxative regimens does not improve outcomes in children with severe
constipation (SOR B). Manual disimpaction is a more invasive option and is not recommended as first-line
treatment in young children.

Ref: Candy D, Belsey J: Macrogol (polyethylene glycol) laxatives in children with functional constipation and faecal impaction:
A systematic review. Arch Dis Child 2009;94(2):156-160. 2) Bekkali NL, van den Berg MM, Dijkgraaf MG, et al: Rectal
fecal impaction treatment in childhood constipation: Enemas versus high doses oral PEG. Pediatrics
2009;124(6):e1108-e1115. 3) Nurko S, Zimmerman LA: Evaluation and treatment of constipation in children and
adolescents. Am Fam Physician 2014;90(2):82-90.

Item 77

ANSWER: A

Head lice are a common and easily treated inconvenience in school-aged children that, unlike body lice,
are not associated with significant illnesses. Transmission generally requires head-to-head contact, as lice
cannot survive when separated from their host for more than 24 hours and do not fly or hop. Visible nits
are generally present at the time of diagnosis, confirming that the infestation has been present for some
time, so immediate isolation from other children would not be expected to change the natural course of
events. The American Academy of Pediatrics (AAP) recommends that children found to be infested with
lice remain in class but be discouraged from close contact with others until treated appropriately with a
pediculicide. The AAP position also recommends abandonment of no nits school policies, which prohibit
attendance until no visible nits are identified. Nits can be found long after their deposition at the scalp level
and generally have already hatched by the time they are easily noted at some distance from the scalp.

Ref: Frankowski BL, Bocchini JA Jr; Council on School Health and Committee on Infectious Diseases: Head lice. Pediatrics
2010;126(2):392-403. 2) Albrecht S: The prevention and treatment of head lice in children. US Pharm 2012;37(3):32-36.

Item 78

ANSWER: B

The first step in the evaluation of nonmassive hemoptysis is to obtain a chest radiograph. If this is normal
and there is a high risk of malignancy (patient age 40 years or older with at least a 30-pack year smoking
history), chest CT should be ordered. Bronchoscopy should also be considered in the workup of high-risk
patients. If a chest radiograph shows an infiltrate, treatment with antibiotics is warranted. If the chest
radiograph is normal the patient is at low risk for malignancy, and if the history does not suggest lower
respiratory infection and hemoptysis does not recur, observation can be considered.

Ref: Earwood JS, Thompson TD: Hemoptysis: Evaluation and management. Am Fam Physician 2015;91(4):243-249.

26
Item 79

ANSWER: B

Symptoms of acute urticaria are best managed first with an H1-antihistamine. Second-generation
H1-blockers are usually preferred because they have a longer duration of action and are less likely to cause
drowsiness than first-generation H1-blockers. Patients with associated laryngeal swelling and respiratory
symptoms require urgent treatment with injectable epinephrine before anything else is given. Topical
corticosteroids would not be helpful. H2-blockers are modestly beneficial as an adjunct to H1-blockers.
Leukotriene-receptor antagonists may also be added if H1-blockers are not sufficient.

Ref: Schaefer P: Urticaria: Evaluation and treatment. Am Fam Physician 2011;83(9):1078-1084.

Item 80

ANSWER: C

Many patients with uveitis have an associated systemic disease. Some medications may cause secondary
uveitis, and conditions such as ocular lymphoma and bloodborne infection may masquerade as primary
uveitis. In North America, the most common conditions associated with uveitis are the seronegative
spondyloarthropathies, sarcoidosis, syphilis, rheumatoid arthritis, and reactive arthritis. All of the
conditions listed may be associated with uveitis, but given the chest radiograph findings and clinical
scenario in this case, sarcoidosis is most likely.

Ref: Harman LE, Margo CE, Roetzheim RG: Uveitis: The collaborative diagnostic evaluation. Am Fam Physician
2014;90(10):711-716.

Item 81

ANSWER: D

Vitamin D supplementation helps prevent falls in community-dwelling adults age 65 and older, although
the mechanism is not clearly understood. Supplementation is recommended by the U.S. Preventive
Services Task Force (SOR B).

Ref: US Preventive Services Task Force: Prevention of Falls in Community-Dwelling Older Adults: US Preventive Services
Task Force Recommendation Statement. AHRQ pub no 11-05150-EF-2, 2012.

Item 82

ANSWER: D

Esophageal adenocarcinoma has become the predominant type of esophageal cancer in North America and
Europe, and gastroesophageal reflux and obesity are the main risk factors. Helicobacter pylori infection,
aspirin therapy, NSAID use, and Crohns disease are not significant risk factors.

Ref: Rustgi AK, El-Serag HB: Esophageal carcinoma. N Engl J Med 2014;371(26):2499-2509.

27
Item 83

ANSWER: E

When symptoms begin to appear in a patient with aortic stenosis the prognosis worsens. It is therefore
important to be aware of systolic murmurs in older patients presenting with exertional dyspnea, chest pain,
or dizziness. This can be the first presentation of a downward spiral and the need for rapid valve
replacement. Weight loss, frequent urination, jaundice, and worsening headache are not as closely
associated with a generally worse outlook for patients with aortic stenosis.

Ref: Otto CM, Prendergast B: Aortic-valve stenosisfrom patients at risk to severe valve obstruction. N Engl J Med
2014;371(8):744-756.

Item 84

ANSWER: B

Acute pyelonephritis is a common bacterial infection of the renal pelvis and kidney most often seen in
young adult women. It is most commonly caused by Escherichia coli. Outpatient treatment with oral
antibiotics is safe in most adults with mild or moderate pyelonephritis (SOR B). An oral fluoroquinolone
such as ciprofloxacin is usually the first-line therapy in mild and moderate cases in areas where the rate
of fluoroquinolone resistance in E. coli is <10% (SOR A). If the community fluoroquinolone resistance
rate exceeds 10%, a one-time dose of a parenteral antimicrobial such as ceftriaxone or a consolidated dose
of an aminoglycoside should be given, followed by an oral fluoroquinolone regimen (SOR B).

Alternative oral agents include trimethoprim/sulfamethoxazole and $-lactam antibiotics; however, these
are not first-line empiric agents, due to high levels of resistance (SOR A), and should not be used for
treatment until the uropathogen is confirmed to be susceptible. Amoxicillin and nitrofurantoin are
sometimes used to treat uncomplicated cystitis but these agents are less effective than other available agents
for treatment of pyelonephritis (SOR B). Erythromycin and metronidazole are not appropriate for treating
pyelonephritis.

Ref: Colgan R, Williams M, Johnson JR: Diagnosis and treatment of acute pyelonephritis in women. Am Fam Physician
2011;84(5):519-526. 2) Gupta K, Hooton TM, Naber KG, et al: International clinical practice guidelines for the treatment
of acute uncomplicated cystitis and pyelonephritis in women: A 2010 update by the Infectious Diseases Society of America
and the European Society for Microbiology and Infectious Diseases. Clin Infect Dis 2011;52(5):e103-e120.

Item 85

ANSWER: B

Asymptomatic patients in excellent health often present with this characteristic chest radiograph pattern,
which is usually due to histoplasmosis infection, especially if the patient has been in the midwestern United
States. Exposure to bird or bat excrement is a common cause, and treatment is usually not needed. This
pattern is not characteristic of the other infections listed, although miliary tuberculosis is a remote
possibility despite the negative PPD skin test.

Ref: Mandell GL, Bennett JE, Dolin R (eds): Mandell, Douglas, and Bennetts Principles and Practice of Infectious Diseases,
ed 7. Churchill Livingstone, 2010, pp 3305-3318.

28
Item 86

ANSWER: B

Immune (idiopathic) thrombocytopenic purpura is an acquired immune-mediated disorder defined as


isolated thrombocytopenia not found to have another cause. Treatment is usually restricted to severe
thrombocytopenic cases (platelet count <50,000/mm3) unless there is evidence of acute bleeding.
Corticosteroids are considered the first-line therapy (SOR C). Intravenous immunoglobulin and rituximab
have also been used as first-line agents. Second-line therapies include thrombopoietin-receptor agonists
and splenectomy. Further evaluation, including a bone marrow biopsy, to rule out myelodysplastic
syndrome and lymphoproliferative disorders is indicated in patients over the age of 60 (SOR C). Platelet
transfusion is not indicated in the absence of hemorrhage or a need for surgery.

Ref: Sharma S, Sharma P, Tyler LN: Transfusion of blood and blood products: Indications and complications. Am Fam
Physician 2011;83(6):719-724. 2) Gauer RL, Braun MM: Thrombocytopenia. Am Fam Physician 2012;85(6):612-622.

Item 87

ANSWER: B

Azithromycin should be considered the preferred agent for the treatment and prophylaxis of pertussis (SOR
A). Trimethoprim/sulfamethoxazole is an alternative in cases of allergy or intolerance to macrolides.
Because of the possibility of treatment benefit, and because of the potential of antibiotics to decrease
transmission, the CDC continues to recommend antibiotics for the treatment of pertussis. In order to
prevent transmission of the infection, treatment should be initiated within 6 weeks of the onset of cough
in patients younger than 12 months, and within 3 weeks in all other patients.

Ref: Kline JM, Lewis WD, Smith EA, et al: Pertussis: A reemerging infection. Am Fam Physician 2013;88(8):507-514.

Item 88

ANSWER: D

Plaque psoriasis is characterized by silvery-white scales adhered to well demarcated erythematous papules
and/or plaques, typically on the scalp, extensor surfaces of the elbows and knees, or buttocks, and often
extending to other exposed areas of the body. When limited to skin folds or the genital region, psoriasis
can easily be confused with other conditions such as bacterial or fungal intertrigo. The lesions in this
variant, known as flexural or inverse psoriasis, usually appear smooth and moist to the point of maceration,
often with minimal to no scaling. Affected patients may report significant pruritus and an unpleasant odor
in the involved area. Evidence-based data for treatment options is limited but supports topical application
of mild corticosteroid creams, vitamin D preparations, or coal tar products. Medium- or higher-potency
corticosteroid creams are best avoided, as the affected areas are either delicate, occlusive, or both, and
susceptible to corticosteroid-induced atrophy.

Ref: Kalb RE, Bagel J, Korman NJ, et al: Treatment of intertriginous psoriasis: From the Medical Board of the National
Psoriasis Foundation. J Am Acad Dermatol 2009;60(1):120-124. 2) Meeuwis KA, de Hullu JA, Massuger LF, et al:
Genital psoriasis: A systematic literature review on this hidden skin disease. Acta Derm Venereol 2011;91(1):511. 3)
Wilmer EN, Hatch RL: Resistant candidal intertrigo: Could inverse psoriasis be the true culprit? J Am Board Fam Med
2013;26(2):211-214.

29
Item 89

ANSWER: B

The JNC 8 panel recommends a goal blood pressure of 150/90 mm Hg in patients age 60 and older with
no comorbidities (SOR A). For those younger than 60 with no comorbidities the recommended goal is
<140/90 mm Hg. For patients with diabetes mellitus or chronic renal disease the goal is <140/90 mm
Hg for patients age 18 or older (SOR C).

Ref: Mahvan TD, Mlodinow SG: JNC 8: Whats covered, whats not, and what else to consider. J Fam Pract
2014;63(10):574-584.

Item 90

ANSWER: B

Although family physicians do not prescribe chemotherapy, they are often called upon by families to help
navigate the choices specialists offer. Patients who receive palliative chemotherapy for end-stage cancers
are less likely to die at home, more likely to undergo CPR, and more likely to undergo mechanical
ventilation. In addition, these patients are referred to hospice later and there is no survival benefit.

Ref: Wright AA, Zhang B, Keating NL, et al: Associations between palliative chemotherapy and adult cancer patients end of
life care and place of death: Prospective cohort study. BMJ 2014;348:g1219.

Item 91

ANSWER: C

Meningococcal disease remains a leading cause of sepsis and meningitis. Those in close contact with
patients who have presumptive meningococcal disease are at heightened risk. While secondary cases have
been reported, they are rare because of prompt chemoprophylaxis of household members and anyone
directly exposed to the index patients oral secretions. The risk for secondary disease among close contacts
is highest during the first few days after the onset of illness in the index patient, mandating immediate
chemoprophylaxis of those exposed. There is no need to isolate family members. The delay in immunity
post vaccination makes it necessary to use other preventive measures instead.

Ref: Bilukha OO, Rosenstein N; National Center for Infectious Diseases, Centers for Disease Control and Prevention (CDC):
Prevention and control of meningococcal disease: Recommendations of the Advisory Committee on Immunization Practices
(ACIP). MMWR Recomm Rep 2005;54(RR-7):1-21. 2) Goldman L, Schafer AI (eds): Goldmans Cecil Medicine, ed 25.
Elsevier Saunders, 2016, pp 2485-2489.

Item 92

ANSWER: B

This patient most likely has a mild to moderate COPD exacerbation. His vital signs do not indicate a
serious condition at this time, so he can be treated as an outpatient. Since he is already on a reasonable
dose of an inhaled bronchodilator/anticholinergic combination, he should be treated with an oral antibiotic
and an oral corticosteroid. Intravenous corticosteroids offer no advantages over oral therapy, provided
there are no gastrointestinal tract limitations such as poor motility or absorption.

30
Oral corticosteroid therapy initiated early in a COPD exacerbation reduces the rate of treatment failure,
decreases hospitalization rates, improves hypoxia and pulmonary function, and shortens the length of stay
for patients requiring hospitalization. Short courses of oral corticosteroids (57 days) are as effective as
longer ones (SOR A). Inhaled corticosteroids are ineffective in the treatment of a COPD exacerbation.
Intramuscular dexamethasone has no role in treating COPD.

Ref: Evensen AE: Management of COPD exacerbations. Am Fam Physician 2010;81(5):607-613, 616. 2) Global Strategy for
the Diagnosis, Management, and Prevention of Chronic Obstructive Pulmonary Disease. Global Initiative for Chronic
Obstructive Lung Disease, 2015. 3) Slawson D: Five-day steroid treatment effective for acute COPD exacerbation. Am
Fam Physician 2013;88(11):744A.

Item 93

ANSWER: D

Diarrhea has several causes, requiring different management. In many cases the diarrhea is caused by a
viral or bacterial infection that is self-limited and requires only supportive measures. In some cases,
however, antibiotic treatment may be needed and it is important to determine the cause of the diarrhea.

Patients who have recently been hospitalized for antibiotic treatment are susceptible to infection with
Clostridium difficile, and should be treated with metronidazole. Travelers to less developed countries often
develop travelers diarrhea from ingesting contaminated food or water. This is most often due to
enterotoxigenic Escherichia coli, although travelers can also have Norovirus infections. The most
appropriate antibiotic choice in this situation is ciprofloxacin.

Patients who become ill after an event where food is served and several attendees have similar symptoms
should be suspected of having a Campylobacter infection if the symptoms include bloody diarrhea. This
should also be treated with ciprofloxacin.

Daycare workers are susceptible to giardiasis, with symptoms including bloating, flatulence, and
foul-smelling stools. This can be treated with metronidazole.

Ref: Barr W, Smith A: Acute diarrhea. Am Fam Physician 2014;89(3):180-189.

Item 94

ANSWER: B

The U.S. Preventive Services Task Force recommends that the selective estrogen receptor modulators
tamoxifen and raloxifene be offered to women at high risk for breast cancer and low risk for adverse
medication effects (B recommendation). This reduces the incidence of invasive breast cancer by 79 events
per 1000 women over 5 years. Tamoxifen has been shown to be more beneficial than raloxifene.

Potential harms include an increase of 47 events of venous thromboembolism per 1000 women over 5
years. Tamoxifen increases the risk more than raloxifene. Tamoxifen also reduces bone fractures but
increases the incidence of endometrial cancer, leg cramps, bladder control issues, vasomotor symptoms,
and vaginal dryness, itching, and discharge.

Ref: Nelson HD, Smith ME, Griffin JC, Fu R: Use of medications to reduce risk for primary breast cancer: A systematic review
for the US Preventive Services Task Force. Ann Intern Med 2013;158(8):604-614.

31
Item 95

ANSWER: D

The protein and lactate dehydrogenase (LDH) levels in pleural fluid can help differentiate between
transudative and exudative effusions. Lights criteria (pleural fluid protein to serum protein ratio >0.5,
pleural fluid LDH to serum LDH ratio >0.6, and/or pleural LDH >0.67 times the upper limit of normal
for serum LDH) are 99.5% sensitive for diagnosing exudative effusions and differentiate exudative from
transudative effusions in 93%96% of cases. Of the listed pleural effusion etiologies, only pulmonary
embolism is exudative. The remainder are all transudative.

Ref: Saguil A, Wyrick K, Hallgren J: Diagnostic approach to pleural effusion. Am Fam Physician 2014;90(2):99-104.

Item 96

ANSWER: D

The intrauterine device (IUD) is a safe and effective method of contraception. There are two main classes
of IUDs: the copper T 380A IUD and the levonorgestrel-releasing IUD (14 or 20 :g). There are few
contraindications to their use but in certain conditions one class is preferred over the other (SOR C).

Women with severe cirrhosis or liver cancer should not use the levonorgestrel-releasing IUD, and the
copper T is preferred. Hormonal contraceptives in general should be avoided in women with severe liver
disease, as there is a known association between oral contraceptive use and the growth of hepatocellular
adenoma, and this risk is thought to extend to other types of hormonal contraceptives (SOR C). Breast
cancer is another contraindication to use of the levonorgestrel-releasing IUD, and the copper T would be
preferred.

There is no difference in risk between the copper T and levonorgestrel-releasing IUD with regard to deep
vein thrombosis/pulmonary embolism. However, the IUD is preferable to contraceptives containing
estrogen.

IUDs can be used in nulliparous women and either type may be used, although there is some evidence that
there are fewer complications with the levonorgestrel-releasing IUD.

Smoking does not preclude the use of either type of IUD. Patients with heart failure may use either type
of IUD as well. Women with controlled hypertension may use either form, but there is a slight risk from
use of the levonorgestrel-releasing IUD in women with uncontrolled hypertension, although the benefits
outweigh the risks.

Ref: Centers for Disease Control and Prevention (CDC): US medical eligibility criteria for contraceptive use, 2010. MMWR
Recomm Rep 2010;59(RR-4):1-86. 2) Tepper NK, Steenland MW, Marchbanks PA, Curtis KM: Laboratory screening
prior to initiating contraception: A systematic review. Contraception 2013;87(5):645-649. 3) Hardeman J, Weiss BD:
Intrauterine devices: An update. Am Fam Physician 2014;89(6):445-450.

32
Item 97

ANSWER: E

A teaspoon of honey, given alone or in a noncaffeinated liquid before bed, has been shown to reduce the
severity and frequency of coughing. It improves the sleep of both the child and the parents. Placebo was
also effective in one study, but not as effective as honey. Honey should not be given to children younger
than 12 months of age because of the risk of botulism, although this risk is very small.

Ref: Ebell MH, Grad R: Top 20 research studies of 2012 for primary care physicians. Am Fam Physician 2013;88(6):380-386.

Item 98

ANSWER: A

Long-term medication-assisted treatment for narcotic addiction is more successful than detoxification
programs (SOR A). One study reported 49% of patients with minimal or no opiate use after 12 weeks of
buprenorphine/naloxone treatment as opposed to only 7% of those undergoing a brief taper. Methadone
is the drug of choice for pregnant women, with no long-term harmful effects noted. Diversion is reported
but is not frequent. Treatment with opioid agonists does not clearly diminish cocaine abuse (SOR C).

Ref: Hill KP, Rice LS, Connery HS, Weiss RD: Diagnosing and treating opioid dependence. J Fam Pract 2012;61(10):588-597.

Item 99

ANSWER: C

The umbilicus normally contains 2 arteries and 1 vein. A single artery is found in up to 1% of newborns,
and may be associated with renal abnormalities.

Ref: Fuloria M, Kreiter S: The newborn examination: Part II. Emergencies and common abnormalities involving the abdomen,
pelvis, extremities, genitalia, and spine. Am Fam Physician 2002;65(2):265-270. 2) Gleason CA, Devaskar SU (eds):
Averys Diseases of the Newborn, ed 9. Elsevier Saunders, 2012, p 50.

Item 100

ANSWER: E

It is estimated that 3%10% of infants and toddlers refuse to eat, according to their caregivers. Unlike
other feeding problems such as colic, this problem tends to persist without intervention. It is recommended
that caregivers establish routines for healthy scheduled meals and snacks, and follow them consistently.
Parents should control what, when, and where children are being fed, whereas children should control how
much they eat at any given time in accordance with physiologic signals of hunger and fullness. No food
or drinks other than water should be offered between meals or snacks. Food should not be offered as a
reward or present. Parents can be reassured that a normal child will learn to eat enough to prevent
starvation. If malnutrition does occur, a search for a physical or mental abnormality should be sought.

Ref: Bernard-Bonnin AC: Feeding problems of infants and toddlers. Can Fam Physician 2006;52(10):1247-1251. 2) Kerzner
B, Milano K, MacLean WC Jr, et al: A practical approach to classifying and managing feeding difficulties. Pediatrics
2015;135(2):344-353.

33
Item 101

ANSWER: E

Trochanteric bursitis develops insidiously after repetitive use, and the patient may report morning stiffness
and pain when lying on the affected side. Palpation of the greater trochanter elicits tenderness, and
occasionally swelling may be noted as well. Early injection with a corticosteroid usually produces a
satisfactory response.

Ref: Cardone DA, Tallia AF: Diagnostic and therapeutic injection of the hip and knee. Am Fam Physician
2003;67(10):2147-2152. 2) Wilson JJ, Furukawa M: Evaluation of the patient with hip pain. Am Fam Physician
2014;89(1):27-34.

Item 102

ANSWER: E

Renal artery stenosis may be present in as many as 5% of patients with hypertension. It is often seen in
those who have coronary artery disease and/or peripheral vascular disease. Hypertension requiring four
or five drugs to control, abdominal bruits, and development of hyperkalemia or renal insufficiency after
initiating therapy with an ACE inhibitor can all point toward renal artery stenosis as a diagnosis.

For patients with renal artery stenosis who have good control, no testing is necessary other than monitoring
renal function, particularly if an ACE inhibitor or ARB is part of the regimen. Screening tests
recommended by clinical guidelines include duplex ultrasonography, CT angiography, or MR cystography
(SOR B). Captopril renography was used in the past but is no longer recommended.

In the 1990s uncontrolled studies were done that suggested that either stenting or angioplasty resulted in
significant blood pressure reduction and reduced renal failure. However, a clinical trial has shown that
stenting did not benefit patients when added to comprehensive multifactorial medical therapy.

Ref: Viera AJ, Neutze DM: Diagnosis of secondary hypertension: An age-based approach. Am Fam Physician
2010;82(12):1471-1478. 2) Cooper CJ, Murphy TP, Cutlip DE, et al: Stenting and medical therapy for atherosclerotic
renal-artery stenosis. N Engl J Med 2014;370(1):13-22. 3) Weber BR, Dieter RS: Renal artery stenosis: Epidemiology
and treatment. Int J Nephrol Renovasc Dis 2014;7:169-181.

Item 103

ANSWER: A

In patients with community-acquired pneumonia it is necessary to decide on both the antibiotic regimen
and the treatment setting. The decision regarding site of care is based on the severity of illness, which can
be assessed with tools such as the CURB-65 score, which take into account factors such as respiratory rate,
blood pressure, uremia, confusion, and age.

34
Patients who have only mild symptoms can be treated with azithromycin on an outpatient basis if there is
a low level of macrolide resistance in the community. If there is a high level of resistance in the
community, if the patient has comorbidities such as diabetes mellitus or COPD, or if there is a history of
use of an immunosuppressing drug or recent use of an antibiotic, the patient can still be treated as an
outpatient but should be treated with levofloxacin. Patients with more severe symptoms, such as an
elevated pulse rate or respiratory rate, should be treated on an inpatient basis with ceftriaxone or
azithromycin. Patients who have more severe symptoms along with bronchiectasis should be treated with
piperacillin/tazobactam plus levofloxacin.

Patients with the most severe symptoms, including hypotension, a more elevated pulse rate, low oxygen
saturation, and confusion, should be treated in the intensive-care unit with levofloxacin and vancomycin.

Ref: Mandell LA, Wunderink RG, Anzueto A, et al; Infectious Diseases Society of America; American Thoracic Society:
Infectious Diseases Society of America/American Thoracic Society consensus guidelines on the management of
community-acquired pneumonia in adults. Clin Infect Dis 2007;44(Suppl 2):S27-S72.

Item 104

ANSWER: C

In children, neck masses usually fall into one of three categories: developmental, inflammatory/reactive,
or neoplastic. The history and physical examination can help narrow the diagnosis, with location of the
mass being particularly helpful.

Branchial cleft cysts make up approximately 20% of neck masses in children. They commonly present in
late childhood or adulthood, when a previously unrecognized cyst becomes infected. They are most
frequently found anterior to the sternocleidomastoid muscle, but can also be preauricular.

Thyroglossal duct cysts are located in the midline over the hyoid bone. Frequently, they elevate when the
patient swallows. Dermoid cysts are usually mobile, moving with the overlying skin. They can be located
in the submental or midline region. Thyroid tumors are also usually located in the midline. Malignant
masses are usually hard, irregular, nontender, and fixed.

Ref: Bluestone CD, Stool SE, Alper CM, et al (eds): Pediatric Otolaryngology, ed 4. WB Saunders, 2003, pp 1629-1647. 2)
Acierno SP, Waldhausen JH: Congenital cervical cysts, sinuses and fistulae. Otolaryngol Clin North Am
2007;40(1):161-176. 3) Meier JD, Grimmer JF: Evaluation and management of neck masses in children. Am Fam
Physician 2014;89(5):353-358.

Item 105

ANSWER: E

The treatment of rhabdomyolysis includes rapid large infusions of isotonic saline to prevent and treat acute
kidney injury, which occurs in 10%60% of patients. Sodium bicarbonate administration is unnecessary
and is not better than normal saline diuresis and increasing urine pH. Loop diuretics and mannitol have
little human evidence to support their use. Corticosteroid use is not recommended.

Ref: Zimmerman JL, Shen MC: Rhabdomyolysis. Chest 2013;144(3):1058-1065.

35
Item 106

ANSWER: C

Antibiotic prophylaxis should be used for high-risk bite wounds. Factors associated with a high risk include
a bite on an extremity with underlying venous and/or lymphatic compromise, a bite involving the hand,
a bite near or in a prosthetic joint, cat bites, crush injuries, delayed presentation, puncture wounds,
underlying diabetes mellitus, and immunosuppression. A Cochrane review of nine trials showed no
statistical difference in infection rates between prophylaxis and no treatment, except when the bite wound
was on the hand. The role of tetanus and rabies prophylaxis should be considered on a case-by-case basis.
The other factors listed do not influence whether or not an antibiotic should be prescribed (SOR B).

Ref: Ellis R, Ellis C: Dog and cat bites. Am Fam Physician 2014;90(4):239-243.

Item 107

ANSWER: D

Hypertension is the most common manifestation of autosomal dominant polycystic kidney disease and it
also contributes to worsening renal function and an increased risk for cardiovascular disease and death.
ACE inhibitors such as lisinopril are first-line agents because they have renal protective benefits in addition
to their effects on blood pressure. Some studies have suggested they help slow the decline in renal function
and help to prevent left ventricular hypertrophy (more so than diuretics or calcium channel blockers).
Angiotensin receptor blockers should be reserved for those who cannot tolerate ACE inhibitors.

Ref: Torres VE, Harris PC, Pirson Y: Autosomal dominant polycystic kidney disease. Lancet 2007;369(9569):12871301. 2)
Srivastava A, Patel N: Autosomal dominant polycystic kidney disease. Am Fam Physician 2014;90(5):303-307.

Item 108

ANSWER: A

The American College of Rheumatology has defined diagnostic criteria for fibromyalgia based on the
patients symptoms (SOR A). Previously, tender points on examination were the diagnostic criterion.
Laboratory testing, muscle biopsies, and electromyography can be used to rule out other conditions.

Ref: Kodner C: Common questions about the diagnosis and management of fibromyalgia. Am Fam Physician
2015;91(7):472-478.

36
Item 109

ANSWER: C

Sympathomimetic agents can elevate blood pressure and intraocular pressure, may worsen existing urinary
obstruction, and adversely interact with $-blockers, methyldopa, tricyclic antidepressants, oral
hypoglycemic agents, and MAOIs. They also speed up the heart rate. First-generation nonprescription
antihistamines can enhance the anticholinergic and sedative effects of other medications.

Ref: Brunton LL (ed): Goodman & Gilmans The Pharmacological Basis of Therapeutics, ed 12. McGraw-Hill, 2011, p 302.
2) Young SS: Appropriate Use of Common OTC Analgesics and Cough and Cold Medications. American Academy of
Family Physicians monograph, 2008.

Item 110

ANSWER: B

Taking oral iron with vitamin C or a meal high in meat protein increases iron absorption. Calcium and
coffee both decrease iron absorption, but not as much as tea, which can reduce absorption of oral iron by
as much as 90%.

Ref: DeLoughery TG: Microcytic anemia. N Engl J Med 2014;371(14):1324-1331.

Item 111

ANSWER: B

The hemoglobin A1c (HbA1c) blood test provides information regarding average glucose levels over the past
3 months. Any condition that shortens erythrocyte survival or decreases mean erythrocyte age, such as
recent acute blood loss or hemolytic anemia, will falsely lower HbA1c levels. Hemoglobin variants and iron
deficiency, kidney failure, and liver disease can also affect HbA1c results. Heart failure, COPD, and
hypothyroidism do not influence HbA1c values.

Ref: The A1c test and diabetes. National Diabetes Information Clearinghouse, NIH pub no 14-7816, 2014. 2) Sickle cell trait
and other hemoglobinopathies and diabetes: Important information for providers. National Diabetes Information
Clearinghouse, NIH pub no 14-6287, 2014. 3) For people of African, Mediterranean, or Southeast Asian heritage:
Important information about diabetes blood tests. National Diabetes Information Clearinghouse, NIH pub no 12-6283, 2011.
4) National Glycohemoglobin Standardization Program (NGSP): Factors that interfere with HbA1c test results. NGSP, 2015.

37
Item 112

ANSWER: E

The physiologic changes that accompany aging result in altered pharmacokinetics. In older persons there
is a relative increase in body fat and a relative decrease in lean body mass, which causes increased
distribution of fat-soluble drugs such as diazepam. This also increases the elimination half-life of such
medications. The volume of distribution of water-soluble compounds such as digoxin is decreased in older
patients, which means a smaller dose is required to reach a given target plasma concentration. There is also
a predictable reduction in glomerular filtration rate and tubular secretion with aging, which causes
decreased clearance of medications in the geriatric population. The absorption of drugs changes little with
advancing age. All of these changes are important to consider when choosing dosages of medications for
the older patient.

Ref: Goldman L, Schafer AI (eds): Goldmans Cecil Medicine, ed 25. Elsevier Saunders, 2016, pp 129-130.

Item 113

ANSWER: E

NSAIDs cause an elevation of blood pressure due to their salt and water retention properties. This effect
can also lead to edema and worsen underlying heart failure. In addition, all NSAIDs can have a deleterious
effect on kidney function and can worsen underlying chronic kidney disease, in addition to precipitating
acute kidney injury. Celecoxib, ibuprofen, meloxicam, and diclofenac are associated with an increased risk
of cardiovascular adverse effects and myocardial infarction, compared with placebo. However, naproxen
has not been associated with an increased risk of myocardial infarction and is therefore preferred over
other NSAIDs in patients with underlying coronary artery disease risk factors (SOR B).

Ref: Fogleman CD: Analgesics for osteoarthritis. Am Fam Physician 2013;87(5):354-356.

Item 114

ANSWER: B

This patient has examination findings that strongly suggest a peritonsillar abscess, which is the most
common deep infection of the head and neck in young adults. Although antibiotics are indicated in this
case, the cornerstone of management is drainage of the abscess either by needle drainage or by incision
and drainage. Immediate tonsillectomy is less favored, as it is a less cost-effective option.

Ref: Galioto NJ: Peritonsillar abscess. Am Fam Physician 2008;77(2):199-202. 2) Tintinalli JE, Kelen GD, Stapczynski JS
(eds): Emergency Medicine: A Comprehensive Study Guide, ed 7. McGraw-Hill, 2011, pp 795-796.

Item 115

ANSWER: A

This patient meets the clinical criteria for Henoch-Schnlein purpura (HSP), an immune-mediated vasculitis
found commonly in children under the age of 10. The clinical triad of purpura, abdominal pain, and
arthritis is classic.

38
Almost 95% of children with HSP spontaneously improve, so supportive therapy is the main intervention.
Acetaminophen or ibuprofen can be used for the arthritic pain. However, ibuprofen should be avoided in
those with abdominal pain or known renal involvement. Prednisone has been found to help in those with
renal involvement or other complications of the disease such as significant abdominal pain, scrotal
swelling, or severe joint pains (SOR B). However, it is not effective for preventing renal disease or
reducing the severity of renal involvement, as was once thought (SOR A).

Immunosuppressants such as cyclophosphamide and cyclosporine have been suggested for treating patients
with severe renal involvement, but there is insufficient evidence to support their use. Amoxicillin is
appropriate for patients with a bacterial infection, such as streptococcal pharyngitis, which has led to HSP.
In this patient, however, there is no indication of pharyngitis or another bacterial focus. Patients with renal
involvement and resultant hypertension with HSP should be treated with calcium channel blockers such
as amlodipine. This patient exhibits neither renal involvement nor hypertension.

Ref: Chartapisak W, Opastirakul S, Hodson EM, et al: Interventions for preventing and treating kidney disease in
Henoch-Schnlein purpura (HSP). Cochrane Database Syst Rev 2009;(3):CD005128. 2) Reamy BV, Williams PM, Lindsay
TJ: Henoch-Schnlein purpura. Am Fam Physician 2009;80(7):697-704. 3) Fleisher GR, Ludwig S (eds): Textbook of
Pediatric Emergency Medicine, ed 6. Lippincott Williams & Wilkins, 2010, pp 1117-1118.

Item 116

ANSWER: D

Acute laryngitis most often has a viral etiology and symptomatic treatment is therefore most appropriate.
A Cochrane review concluded that antibiotics appear to have no benefit in treating acute laryngitis. Proton
pump inhibitors such as omeprazole can be of benefit in treating chronic laryngitis caused by acid reflux,
but not for an acute problem such as the one described.

Ref: Reveiz L, Cardona AF: Antibiotics for acute laryngitis in adults. Cochrane Database Syst Rev 2013;(3):CD004783.

Item 117

ANSWER: A

Low back pain is one of the most common reasons for visits to physicians. The workup should start with
a thorough history and physical examination to determine whether the patient has nonspecific back pain,
back pain possibly related to radiculopathy or spinal stenosis, or back pain due to some other specific
cause. Nonspecific back pain does not require imaging (SOR B). An initial plain film would be appropriate
if there were a history of recent significant trauma, or even a history of minor trauma in an elderly patient.
Immediate MRI would be appropriate in the presence of other red flags such as bladder dysfunction,
areflexia, saddle anesthesia, progressive motor weakness, a history of cancer, or the presence of fever,
unexplained weight loss, or night sweats.

Ref: Chou R, Qaseem A, Snow V, et al; Clinical Efficacy Assessment Subcommittee of the American College of Physicians;
American College of Physicians; American Pain Society Low Back Pain Guidelines Panel: Diagnosis and treatment of low
back pain: A joint clinical practice guideline from the American College of Physicians and the American Pain Society. Ann
Intern Med 2007;147(7):478-491. 2) Last AR, Hulbert K: Chronic low back pain: Evaluation and management. Am Fam
Physician 2009;79(12):1067-1074.

39
Item 118

ANSWER: A

Hyperaldosteronism, usually caused by a hyperaldosterone-secreting adrenal mass, has to be considered


in a middle-aged patient with resistant hypertension and hypokalemia. Peripheral aldosterone concentration
(PAC) and peripheral renin activity (PRA), preferably after being upright for 2 hours, are the preferred
screening tests for hyperaldosteronism. A PAC >15 ng/dL and a PAC/PRA ratio >20 suggest an adrenal
cause. Abdominal CT may miss adrenal hyperplasia or a microadenoma. Renal CT angiography is useful
for detecting renal artery stenosis. If the PAC/PRA is abnormal, an aldosterone suppression test should
be ordered.

Ref: Viera AJ, Neutze DM: Diagnosis of secondary hypertension: An age-based approach. Am Fam Physician
2010;82(12):1471-1478.

Item 119

ANSWER: C

A patient with known adrenal insufficiency secondary to hypopituitarism who is undergoing a period of
stress such as illness or surgery should be given intravenous corticosteroids. For moderate-risk procedures
such as vascular or orthopedic operations, 50 mg of hydrocortisone is recommended. For major surgery,
such as open heart surgery or an esophagectomy, 100 mg of hydrocortisone would be needed. These doses
can be repeated every 8 hours until the patient is stable and is able to take his usual oral maintenance dose.

Thyroid replacement is not required for short-term situations, and ACTH is not recommended. If the
patient becomes hypotensive a bolus of normal saline may be indicated. However, in a stable patient
undergoing elective surgery, only routine hydration is indicated.

Ref: Marik PE, Varon J: Requirement of perioperative stress doses of corticosteroids: A systematic review of the literature. Arch
Surg 2008;143(12):1222-1226. 2) Kohl BA, Schwartz S: Surgery in the patient with endocrine dysfunction. Med Clin North
Am 2009;93(5):1031-1047.

Item 120

ANSWER: C

Periodic developmental screening is essential for the early recognition of neuromuscular disorders and
motor delays in children. Multiple developmental screening tools are available for primary care physicians
to use. Motor development should progress throughout infancy and childhood. Either failure to adequately
progress or signs of regression should be cause for concern and raise the suspicion for a neuromuscular
disorder such as muscular dystrophy.

Infants should roll from prone to supine by 4 months of age and supine to prone by 6 months of age. They
should be able to get themselves into a sitting position by 9 months of age. While low muscular tone in an
infant suggests muscular dystrophy, high muscle tone is concerning for an upper motor neuron condition
and should be evaluated with MRI.

40
A 15-month-old who is unable to rise to a standing position without using his hands should have a creatine
kinase (CK) level obtainedthis is the classic Gowers sign. Although many children walk unassisted by
12 months, CK levels should not be obtained (unless indicated for other reasons) unless a male child is not
walking by 18 months of age.

Ref: Bushby K, Finkel R, Birnkrant DJ, et al: Diagnosis and management of Duchenne muscular dystrophy, part 1: Diagnosis,
and pharmacological and psychosocial management. Lancet Neurol 2010;9(1):77-93. 2) Noritz GH, Murphy NA;
Neuromotor Screening Expert Panel: Motor delays: Early identification and evaluation. Pediatrics
2013;131(6):e2016-e2027. 3) Lurio JG, Peay HL, Mathews KD: Recognition and management of motor delay and muscle
weakness in children. Am Fam Physician 2015;91(1):38-44.

Item 121

ANSWER: B

This patient meets the criteria for Kawasaki disease, also known as mucocutaneous lymph node syndrome.
It is an acute type of vasculitis that predominantly affects small and medium-size vessels and is the most
common cause of acquired coronary artery disease in childhood.

Diagnostic criteria include fever for at least 5 days and at least 4 of the 5 principal clinical features:

changes of the oral cavity and lips


polymorphous rash
bilateral nonpurulent conjunctivitis
changes in the extremities (erythema followed by desquamation)
cervical lymphadenopathy

Coronary abnormalities, including coronary aneurysms, are the most concerning sequelae of Kawasaki
disease and may occur in the first week. For this reason early cardiac evaluation is recommended, with
transthoracic echocardiography being the preferred initial imaging. Radionuclide imaging can be useful
in assessing cardiac perfusion in patients found to have persisting echocardiographic findings. MR
coronary angiography can be used to assess response to treatment over time. Intravenous immunoglobulin
and corticosteroids reduce the risk of coronary abnormalities and should be administered as soon as the
disease is suspected.

Ref: Saguil A, Fargo M, Grogan S: Diagnosis and management of Kawasaki disease. Am Fam Physician 2015;91(6):365-371.

Item 122

ANSWER: C

Electronic cigarettes (e-cigarettes) are not currently regulated by the FDA. The amount of particulate
matter released into the air by low-nicotine e-cigarettes is comparable to that released by tobacco
cigarettes, while higher-nicotine e-cigarettes release more particulate matter. E-cigarettes have various
concentrations of nicotine in solution. While the number of tobacco cigarettes smoked per day is decreased
in e-cigarette users, this has not yet been shown to lead to smoking cessation. There are no studies of
e-cigarettes in pregnant women, and nicotine is contraindicated during pregnancy.

Ref: Grana R, Benowitz N, Glantz SA: E-cigarettes: A scientific review. Circulation 2014;129(19):1972-1986.

41
Item 123

ANSWER: A

Cardiovascular disease, in particular ischemic heart disease, has now become the leading cause of human
deaths worldwide. It was once considered a disease of the wealthy, but now more than 80% of deaths from
noncommunicable diseases occur in low- to middle-income countries. The other conditions listed remain
among the top 10 causes of human deaths worldwide, along with stroke, lower respiratory infections,
COPD, diabetes mellitus, and road deaths.

Ref: Fuster VF, Narula J, Vedanthan R, Kelly BB (eds): Promoting Global Cardiovascular Health: Perspective on the 12
Recommendations of the Institute of Medicine. Scientific American Custom Media, 2014, p 74. 2) The top 10 causes of
death. World Health Organization, 2014, Fact Sheet no 310.

Item 124

ANSWER: D

This patient has symptoms consistent with hyperthyroidism, which could be caused by any of the options
listed. TSH is suppressed and free T4 and free T3 are elevated in all of these conditions. Only Graves
disease, however, will cause high radioactive iodine uptake on a thyroid scan. Uptake will be low in the
other conditions.

Ref: Sweeney LB, Stewart C, Gaitonde DY: Thyroiditis: An integrated approach. Am Fam Physician 2014;90(6):389-396.

Item 125

ANSWER: D

This patient meets the criteria for severe inflammatory response syndrome (SIRS) (fever >38.5C, heart
rate >90 beats/min, respiratory rate >20/min, WBC count >12,000/mm3). He also meets the criteria
for severe sepsis, with a positive chest radiograph and evidence of organ hypoperfusion (mental status
changes), as well as septic shock (mean arterial pressure <60 mm Hg). The most appropriate initial
treatment for patients with hypotension in septic shock is fluid resuscitation (SOR A). While vasopressor
therapy is certainly appropriate in septic shock, it should be initiated only after fluid resuscitation fails to
restore mean arterial pressure (>65 mm Hg) or when there is evidence of continued organ hypoperfusion.
Appropriate antibiotics to cover community-acquired pneumonia are recommended during the first hour
of presentation in sepsis (SOR B) but will likely have little effect on acute hypotension. Packed red blood
cell transfusion is not indicated in this scenario, as the patients hemoglobin is above 7 g/dL.

Ref: Gauer RL: Early recognition and management of sepsis in adults: The first six hours. Am Fam Physician 2013;88(1):44-53.

Item 126

ANSWER: E

This patient has symptoms and findings consistent with acute compartment syndrome, which is an
emergency. The diagnostic test is tissue pressure studies. This condition can occur after a severe injury
to the extremity, although it can also develop after a relatively minor injury. Associated problems include
fractures, a badly bruised muscle, crush injuries, constricting bandages, and bites with swelling.

42
Ref: Wall CJ, Lynch J, Harris IA, et al: Clinical practice guidelines for the management of acute limb compartment syndrome
following trauma. ANZ J Surg 2010;80(3):151-156.

Item 127

ANSWER: B

It is of critical importance that patients have a documented negative HIV antibody test (from serum or
point-of-care fingerstick) prior to starting pre-exposure prophylaxis (PrEP) to avoid inadvertent treatment
of HIV infection with emtricitabine/tenofovir. This is the only medication currently approved in the United
States for PrEP, but it is inadequate for HIV treatment. Using this treatment by itself in HIV-positive
patients increases the risk of HIV strains developing resistance to these antiviral agents.

Other recommended testing prior to PrEP use includes creatinine clearance calculation, hepatitis B antibody
testing, screening for sexually transmitted diseases, and pregnancy testing in females capable of pregnancy.
The CDC does not recommend testing liver function, hemoglobin, or platelet levels prior to PrEP use in
otherwise healthy individuals.

Ref: US Public Health Service: Preexposure Prophylaxis for the Prevention of HIV Infection in the United States2014. A
Clinical Practice Guideline. Centers for Disease Control and Prevention, 2014.

Item 128

ANSWER: A

The 2012 American College of Chest Physicians evidenced-based clinical practice guidelines recommend
early ambulation over initial bed rest in patients with acute DVT of the leg (SOR C). If edema and pain
are severe, ambulation may need to be deferred. Several studies and meta-analyses have shown there is
no statistically significant difference between ambulation and bed rest for development of a pulmonary
embolus, a new thrombus, or progression of a thrombus. Therefore, based on the evidence and the
well-recognized benefits of mobility, the current recommendation is to consider early ambulation as soon
as effective anticoagulation has been achieved.

Ref: Guyatt GH, Akl EA, Crowther M, et al; American College of Chest Physicians Antithrombotic Therapy and Prevention
of Thrombosis Panel: Executive summary: Antithrombotic therapy and prevention of thrombosis, 9th ed: American College
of Chest Physicians evidence-based clinical practice guidelines. Chest 2012;141(2 Suppl):7S-47S.

Item 129

ANSWER: A

Of the bioterrorism agents listed, only anthrax requires 60 days of antibiotic treatment (SOR B). If used
in an intentional attack, anthrax spores would be released into the air to be inhaled by the target population.
The full incubation period of the bacterium is 60 days and treatment should cover the entire period.
Appropriate antibiotics include oral fluoroquinolones and doxycycline.

Smallpox is a result of infection with the variola virus. Potential treatments include postexposure treatment
with the smallpox vaccine and two compounds currently in development. Pneumonic plague, caused by
Yersinia pestis, can be provoked by inhalation of the released bacterium or by contact with an infected
individual. Treatment consists of a 10-day course of an aminoglycoside or doxycycline.

43
Inhalational botulism is treated with an antitoxin (equine-derived heptavalent antitoxin). Tularemia could
be caused by an intentional release of the bacterium Francisella tularensis, which would cause a
pneumonia. Treatment is a 10-day course of an aminoglycoside, ciprofloxacin, or doxycycline.

Ref: Adalja AA, Toner E, Inglesby TV: Clinical management of potential bioterrorism-related conditions. N Engl J Med
2015;372(10):954-962.

Item 130

ANSWER: B

A single dose of dexamethasone (0.150.60 mg/kg, usually given orally) is recommended in all patients
with croup, including those with mild disease. Humidification therapy has not been proven beneficial.
Nebulized epinephrine is an accepted treatment in patients with moderate to severe croup. Subcutaneous
epinephrine, diphenhydramine, and ceftriaxone are not recommended treatments.

Ref: Zoorob R, Sidani M, Murray J: Croup: An overview. Am Fam Physician 2011;83(9):1067-1073.

Item 131

ANSWER: A

H1 and H2 histamine blockers and corticosteroids may be useful, but they are not first-line treatments for
an anaphylactic reaction to a Hymenoptera sting. Intravenous normal saline may also be necessary for fluid
resuscitation, but the first treatment should be immediate administration of intramuscular epinephrine.

Ref: Casale TB, Burks AW: Hymenoptera-sting hypersensitivity. N Engl J Med 2014;370(15):1432-1439.

Item 132

ANSWER: E

Reptiles, including snakes, lizards, and turtles, cause both isolated cases of Salmonella infection and local
and widespread outbreaks. While the sale of small pet turtles was outlawed in 1975, the law is not widely
enforced and pet turtles are often a source of Salmonella infection in small children. The infection can also
be spread by other reptiles and amphibians, including snakes and frogs. At a Colorado zoo in 1996, a total
of 65 children were infected by touching a wooden barrier around a Komodo dragon exhibit. Pasteurella
multocida is a common cause of infection as a result of dog or cat bites. Yersinia pestis, the organism of
plague, is transmitted to humans from rodents or their fleas. Hantavirus is also transmitted by rodents, and
psittacosis by certain bird species.

Ref: National Center for Emerging and Zoonotic Infectious Diseases: Reptiles, amphibians, and Salmonella. Centers for Disease
Control and Prevention, 2013. 2) National Association of State Public Health Veterinarians, Inc. (NASPHV); Centers for
Disease Control and Prevention (CDC): Compendium of measures to prevent disease associated with animals in public
settings, 2011: National Association of State Public Health Veterinarians, Inc. MMWR Recomm Rep 2011;60(RR-04):1-24.

44
Item 133

ANSWER: B

Although breast milk is the ideal source of nutrition for healthy term infants, supplementation with 400
IU/day of vitamin D is recommended beginning in the first few days of life and continuing until the child
is consuming at least 500 mL/day of formula or milk containing vitamin D (SOR B). The purpose of
supplementation is to prevent rickets. Unless the baby is anemic or has other deficiencies, neither iron nor
a multivitamin is necessary at this age. For exclusively breastfed infants, iron supplementation should begin
at 4 months of age. Parents often mistakenly think babies need additional water, which can be harmful
because it decreases milk intake and can cause electrolyte disturbances. The introduction of cereal is
recommended at 6 months of age.

Ref: Hagan JF Jr, Shaw JS, Duncan PM (eds): Bright Futures Guidelines for Health Supervision of Infants, Children, and
Adolescents, ed 3. American Academy of Pediatrics, 2008, pp 121-145. 2) Golden NH, Abrams SA; Committee on
Nutrition: Optimizing bone health in children and adolescents. Pediatrics 2014;134(4):e1229-1243.

Item 134

ANSWER: A

Many older drivers have physiologic or cognitive impairment that may affect mobility and driving safety,
and older drivers have an increased crash rate per mile driven. However, older individuals who stop
driving are at higher risk for isolation and depression and there is also an increased cost to the family and
society in general for transportation assistance.

A comprehensive assessment of the ability to drive begins with a driving history from both the patient and
a passenger. Any mention of close calls, mishaps, disorientation, or becoming lost in familiar locations
is an important hint at unsafe driving. It is important to ask the passenger, Do you feel safe riding with
this individual?

In this patient a medication review would also be appropriate. His tricyclic antidepressant, benzodiazepine,
and insulin could all contribute to unsafe driving. Medical illnesses should also be considered in this
history. Dementia, peripheral neuropathy, and retinopathy can all increase the risk for unsafe driving.

Further testing such as a Snellen eye test, audiometry, timed gait, range of motion, muscle strength, clock
drawing, and a Mini-Mental State Examination may also be indicated, but a thorough driving history
should be obtained first. If a concern arises about unsafe driving, referral to a rehabilitative driving center
or the appropriate government agency for further written/road testing may be warranted.

Ref: Carr DB: The older adult driver. Am Fam Physician 2000;61(1):141-146, 148. 2) Carr DB, Duchek JM, Meuser TM,
Morris JC: Older adult drivers with cognitive impairment. Am Fam Physician 2006;73(6):1029-1034. 3) Coln-Emeric
CS, Whitson HE, Pavon J, Hoenig H: Functional decline in older adults. Am Fam Physician 2013;88(6):388-394.

45
Item 135

ANSWER: B

Keloids may arise from scars that result from any cause of skin trauma or infection. Keloids, unlike
hypertrophic scars, are not confined to the margins of the primary injury. Hypertrophic scars are more
likely on extensor surfaces of the body, typically develop soon after the inciting injury, and are more likely
to regress with time.

Ref: Kundu RV, Patterson S: Dermatologic conditions in skin of color: Part II. Disorders occurring predominately in skin of
color. Am Fam Physician 2013;87(12):859-865.

Item 136

ANSWER: D

Testosterone replacement therapy can cause erythrocytosis, so monitoring hematocrit at regular intervals
is recommended. Testosterone replacement therapy does not significantly affect lipid levels, and additional
monitoring of these levels is not recommended. Although there have been anecdotal reports of testosterone
replacement therapy being associated with sleep apnea, current recommendations do not advise routine
testing with overnight polysomnography for patients on testosterone replacement. There is inconsistent
evidence of the effects of testosterone replacement therapy on depression, and thus no recommendation
for monitoring of mood symptoms related to testosterone therapy. Low testosterone levels have been
associated with insulin resistance, but testosterone replacement therapy is not recommended as treatment
for hyperglycemia. Monitoring of serum glucose while on testosterone therapy is not routinely
recommended.

Ref: Bhasin S, Cunningham GR, Hayes FJ, et al: Testosterone therapy in men with androgen deficiency syndromes: An
Endocrine Society clinical practice guideline. J Clin Endocrinol Metab 2010;95(6):2536-2559. 2) Basaria S: Male
hypogonadism. Lancet 2014;383(9924):1250-1263.

Item 137

ANSWER: C

Osteoporosis in males can be caused by multiple conditions, including multiple myeloma, AIDS,
hyperparathyroidism, and hypotestosteronism. In this patient, multiple myeloma is unlikely because of the
normal erythrocyte sedimentation rate, AIDS is unlikely given his past history, and a parathyroid
abnormality is unlikely since his blood chemistry results were normal. Hypotestosteronism is not an
infrequent cause of osteoporosis in men.

Ref: Rao SS, Budhwar N, Ashfaque A: Osteoporosis in men. Am Fam Physician 2010;82(5):503-508.

46
Item 138

ANSWER: B

Physicians should avoid reacting to laboratory values without considering the clinical scenario. This patient
presented with mild dehydration and normal laboratory values. Although he is improving clinically, his
laboratory values show multiple unexpected results. The most noticeable is the severely elevated glucose,
because he has no history of diabetes mellitus or use of medications that could cause this effect. Similarly,
the elevated potassium and decreased sodium suggest profound electrolyte abnormalities. Most likely, the
laboratory technician drew blood from the patients indwelling port without discarding the first several
milliliters. Thus, the blood was contaminated with intravenous fluids, leading to erroneous results. A
repeat blood test from a peripheral vein should give more accurate results.

Ref: Wu AHB: Tietz Clinical Guide to Laboratory Tests, ed 4. Saunders Elsevier, 2006, p 5.

Item 139

ANSWER: A

Obstructive sleep apnea is found in 30%40% of hypertensive patients and 60%70% of patients with
resistant hypertension, whereas primary aldosteronism is present in only 7%20% of patients with resistant
hypertension. Renal artery stenosis is seen in 2%24% of cases of resistant hypertension in various studies,
renal parenchymal disease in 2%4%, and thyroid disease in less than 1%.

Ref: Vongpatanasin W: Resistant hypertension: A review of diagnosis and management. JAMA 2014;311(21):2216-2224.

Item 140

ANSWER: E

Reduced visual acuity may be a symptom of acute angle-closure glaucoma and requires immediate referral
to an ophthalmologist. Copious mucopurulent drainage from the eye is a sign of infectious conjunctivitis,
most likely bacterial, and bilateral eye redness is typically seen with allergic conjunctivitis. Allergic or
infectious conjunctivitis and small corneal abrasions can be managed by the family physician. Bright red
blood under the conjunctiva is consistent with a subconjunctival hemorrhage that will typically resolve
without intervention.

Ref: Cronau H, Kankanala RR, Mauger T: Diagnosis and management of red eye in primary care. Am Fam Physician
2010;81(2):137-144.

Item 141

ANSWER: B

Advance directives, including a living will and durable power of attorney for health care, are used so that
the desires of the individual will be followed in the event he or she lacks the capacity to participate in
health care decisions. This ability refers to decision making capacity. The standards for decision making
capacity vary from state to state but generally include four abilities: patients must (1) have the ability to
understand the relevant information about proposed diagnostic tests or treatment, (2) appreciate their
situation (including their values and current medical situation), (3) use reason to make a decision, and (4)
communicate their choice.

47
A patients capacity is both temporal and situational and capacity evaluations should occur in the context
of the specific health care decision that needs to be made. Some patients lack capacity for specific periods
of time, such as when critically ill, but not permanently. Although some people are completely
incapacitated, many have limited capacity. Those with limited capacity may be able to make some
diagnostic and treatment decisions (generally less risky decisions) but not others. Physicians commonly
hold patients to higher standards when judging capacity for more serious medical decisions.

There is a relationship between capacity and cognition but a patient with dementia can still have
decision-making capacity. With Mini-Mental State Examination scores <20 (maximum score = 30) there
is an increased likelihood of incapacity, but this varies from case to case and is situation dependent.

Competence is a legal term and is a judicial decision made by a court. Any licensed physician can make
a determination of capacity, and a psychiatrist is not required.

Ref: Leo RJ: Competency and the capacity to make treatment decisions: A primer for primary care physicians. Prim Care
Companion J Clin Psychiatry 1999;1(5):131-141. 2) Sessums LL, Zembrzuska H, Jackson JL: Does this patient have
medical decision-making capacity? JAMA 2011;306(4):420-427. 3) Spoelhof GD, Elliott B: Implementing advance
directives in office practice. Am Fam Physician 2012;85(5):461-466. 4) Ethics and Advance Planning for End-of-Life Care.
Advance planning for health care decisions. American Academy of Family Physicians, 2013.

Item 142

ANSWER: B

Injection of glucocorticoids (usually mixed with a local anesthetic) into the subacromial space may be
considered in patients with rotator cuff tendinitis if the pain is significant enough to interfere with sleep
and/or function despite adequate analgesia.

An intra-articular injection is appropriate for a patient with severe shoulder osteoarthritis. A corticosteroid
injection into the biceps or deltoid insertions is not appropriate. An acromioclavicular injection is
appropriate for acromioclavicular arthritis but not for rotator cuff tendinitis.

Ref: Whittle S, Buchbinder R: Rotator cuff disease. Ann Intern Med 2015;162(1):ITC1-ITC15.

Item 143

ANSWER: C

The U.S. Preventive Services Task Force found that the most effective behavioral counseling interventions
for obesity management were comprehensive and of high intensity (1226 sessions in a year), and involved
multiple behavioral management activities, such as group and individual sessions, setting weight-loss goals,
addressing barriers to change, and active use of self-monitoring. Low-carbohydrate diets are minimally
effective over the long term without behavioral interventions.

Ref: Rao G: Office-based strategies for the management of obesity. Am Fam Physician 2010;81(12):1449-1456. 2) Croswell
J, Luger S: Screening for and management of obesity in adults. Am Fam Physician 2012;86(10):947-948.

48
Item 144

ANSWER: D

Posterior or posteromedial rib fractures are secondary to child abuse until proven otherwise, justifying
notification of Child Protective Services or referral to an emergency department familiar with the
appropriate workup. Osteogenesis imperfecta can cause continuous beading of the ribs and crumpled long
bones such as accordina femora, and is often associated with blue sclerae, skin fragility, or brittle teeth.
A skeletal survey is appropriate in a child 2 years of age or younger suspected of being physically abused.
It is not thought to be necessary in children 4 years of age or older, especially in a case where suspicious
fractures have already been discovered. Rickets usually is associated with long bone bowing deformities.
In the chest it can cause prominence of the costochondral junctions (rachitic rosary) and indentation of the
lower ribs where the diaphragm attaches (Harrisons grooves).

Ref: McDonald KC: Child abuse: Approach and management. Am Fam Physician 2007;75(2):221-228. 2) Orient JM: Sapiras
Art and Science of Bedside Diagnosis, ed 4. Lippincott Williams & Wilkins, 2010, p 492. 3) Kodner C, Wetherton A:
Diagnosis and management of physical abuse in children. Am Fam Physician 2013;88(10):669-675.

Item 145

ANSWER: B

The U.S. Preventive Services Task Force recommends opt-out HIV screening for all adolescents and adults
1565 years of age (SOR A). The Centers for Disease Control recommends routine HIV screening in
patients age 1364 (SOR A). Opt-out screening is preferred to opt-in screening, as opt-in screening based
on demographic, behavioral, or clinical subpopulations only identifies approximately 75% of patients with
HIV. Rapid screening tests are highly accurate; however, subsequent conventional testing is necessary to
confirm an HIV diagnosis.

Ref: Branson BM, Handsfield HH, Lampe MA, et al; Centers for Disease Control and Prevention (CDC): Revised
recommendations for HIV testing of adults, adolescents, and pregnant women in health-care settings. MMWR Recomm
Rep 2006;55(RR-14):117. 2) Moyer VA; US Preventive Services Task Force: Screening for HIV: US Preventive Services
Task Force recommendation statement. Ann Intern Med 2013;159(1):5160. 3) Sherin K, Klekamp BG, Beal J, Martin
N: What is new in HIV infection? Am Fam Physician 2014;89(4):265-272.

Item 146

ANSWER: A

The 2013 ACOG guideline recommends induction of labor for gestational hypertension after 37 weeks.
Identifying elevated urine protein is not required for this decision, as gestational hypertension and
preeclampsia without severe features are managed in the same way at 39 weeks gestation. Twice-weekly
office visits with assessment of blood pressure and the other tests mentioned may be appropriate for
patients at less than 37 weeks gestation. Bed rest is no longer recommended for control of hypertension
in pregnancy. Oral antihypertensives are used only at higher blood pressure readings in the setting of
chronic hypertension.

Ref: American College of Obstetricians and Gynecologists; Task Force on Hypertension in Pregnancy: Hypertension in
pregnancy. Report of the American College of Obstetricians and Gynecologists Task Force on Hypertension in Pregnancy.
Obstet Gynecol 2013;122(5):1122-1131.

49
Item 147

ANSWER: A

The clinical history and laboratory findings presented are consistent with a diagnosis of narcolepsy. In
addition to the sleepiness, the patient also has cataplexy, which is manifested in this case by episodes of
sudden weakness when laughing and is almost pathognomonic for narcolepsy. Some patients may also have
vivid hallucinations when falling asleep or waking up. Treatment involves improving both the quantity and
quality of sleep during the night, which can be accomplished with sodium oxybate. This improves daytime
alertness and cataplexy. Scheduling naps is the second important aspect of managing narcolepsy. The third
important step is the use of stimulants such as methylphenidate to improve function during the day.
Periodic daytime naps may also help to reduce symptoms. Since there is no evidence of obstructive sleep
apnea in this patient, weight reduction would not be expected to address his sleep problem. In general,
sedatives, hypnotics, and alcohol should be avoided.

Ref: Goldman L, Schafer AI (eds): Goldmans Cecil Medicine, ed 25. Elsevier Saunders, 2016, p 2418. 2) Ramar K, Olson
EJ: Management of common sleep disorders. Am Fam Physician 2013;88(4):231-238.

Item 148

ANSWER: C

This patient meets the American College of Rheumatology's criteria for rheumatoid arthritis. The criteria
use an algorithm giving a weighted score to joints involved, rheumatoid serology, acute phase reactants
and duration of symptoms. Of the medications listed, oral methotrexate is the only disease-modifying
antirheumatic drug, which should be part of the treatment for rheumatoid arthritis. Prednisone, aspirin,
and NSAIDs are not disease-modifying and should not be used as sole agents, although they can be used
for symptom relief. Doxycycline could be considered in the treatment of Lyme disease but this patient
meets the criteria for rheumatoid arthritis and has negative Lyme serologies.

Ref: Wasserman AM: Diagnosis and management of rheumatoid arthritis. Am Fam Physician 2011;84(11):1245-1252.

Item 149

ANSWER: E

Interstitial lung disease is a consideration in patients with chronic dyspnea. It is often accompanied by a
chronic nonproductive cough. Office spirometry is useful in detecting whether the problem is restrictive
or obstructive. If the FVC is normal or decreased and the FEV1 is decreased, an FEV1/FVC ratio <0.7
means there is an obstructive ventilatory impairment. If the FVC is decreased and the FEV1 is normal or
decreased the ratio would be >0.7, indicating a restrictive impairment.

Diffuse parenchymal lung disease may be idiopathic, but there are a number of identified causes such as
environmental or occupational exposures. Many collagen vascular diseases and medications used to treat
them can induce interstitial lung disease. Common offenders also include amiodarone and nitrofurantoin,
which can induce a pneumonitis. In this patient, lisinopril might explain the cough but not the dyspnea,
crackles, or abnormal spirometry.

Ref: Karnani NG, Reisfield GM, Wilson GR: Evaluation of chronic dyspnea. Am Fam Physician 2005;71(8):1529-1537. 2)
Johnson JD, Theurer WM: A stepwise approach to the interpretation of pulmonary function tests. Am Fam Physician
2014;89(5):359-366.

50
Item 150

ANSWER: D

All pregnant women should be screened for asymptomatic bacteriuria between 11 and 16 weeks gestation
and should be appropriately treated if the urine culture is positive. Asymptomatic bacteriuria is a known
contributor to recurrent urinary tract infections, pyelonephritis, and preterm labor. TSH levels should be
checked in patients with a history of thyroid disease or symptoms of disease, but universal testing is not
recommended. Although treatment of bacterial vaginosis decreases the risk of low birth weight and
premature rupture of membranes, universal screening is not recommended. This patient should be screened
for both group B Streptococcus (GBS) and diabetes mellitus, but not at this point in her pregnancy. GBS
screening should be done between 35 and 37 weeks gestation, and diabetes screening should be performed
with a 50-g glucose load between 24 and 28 weeks gestation.

Ref: Zolotor AJ, Carlough MC: Update on prenatal care. Am Fam Physician 2014;89(3):199-208.

Item 151

ANSWER: C

Evidence shows that early treatment of autism is beneficial (SOR B), and the American Academy of
Pediatrics recommends screening with a validated autism-specific tool such as the MCHAT at 18 and 24
months (SOR C). Delayed social development is typically the first sign of autism. Language delay can be
another finding, but it is less specific. Of the behaviors listed, only abnormal sensitivity to sound is
consistent with autism. Gesturing, pretend play, mimicking, and attempting to attract caregiver attention
all suggest other diagnoses.

Ref: Carbone PS, Farley M, Davis T: Primary care for children with autism. Am Fam Physician 2010;81(4):453-460.

Item 152

ANSWER: B

Stress fractures are common in teenage athletes. Because this patient has a normal physical examination
and can walk without pain, she can return to basketball as long as her symptoms do not return.

Most stress fractures heal in 610 weeks with conservative management such as nonweight bearing and
activity limitation. Athletes can return to play once they are pain free and have a normal physical
examination, even if the time since diagnosis is less than 6 weeks. However, they should refrain from all
high-impact activities such as running and jumping until they can walk without pain. Repeat radiographs
are rarely indicated. Calcium and vitamin D supplementation are recommended as part of the management
of stress fractures, but checking blood levels of vitamin D is not necessary either at the time of the injury
or prior to return to play. Most stress fractures in low-risk locations such as the tibia can be managed in
a primary care office without consulting a sports medicine or orthopedic physician. Fractures in high-risk
locations are at increased risk for malunion and thus are often managed by specialists. This patient has a
low-risk stress fracture.

Ref: Patel DR: Stress fractures: Diagnosis and management in the primary care setting. Pediatr Clin North Am
2010;57(3):819-827. 2) Behrens SB, Deren ME, Matson A, et al: Stress fractures of the pelvis and legs in athletes: A
review. Sports Health 2013;5(2):165174.

51
Item 153

ANSWER: C

In males younger than 40, hematospermia is usually benign and self-limited. Examination of the testes and
prostate is warranted but findings are usually normal. If the patient is sexually active a screen for STDs
is reasonable. Imaging of the genitourinary tract, a serum PSA level, and urology referral are unnecessary
in this age group unless the history or physical examination suggests an unusual cause.

Ref: Stefanovic KB, Gregg PC, Soung M: Evaluation and treatment of hematospermia. Am Fam Physician
2009;80(12):1421-1427.

Item 154

ANSWER: A

Tricyclic antidepressants (TCAs) such as amitriptyline have shown benefit in patients with irritable bowel
syndrome (IBS), as have SSRIs. Because of the anticholinergic properties of TCAs it is thought that TCAs
may be more beneficial than SSRIs in patients with diarrhea-predominant IBS, such as this patient.

Unfortunately, studies have not shown a significant benefit from increasing either insoluble or soluble fiber
to the diet of patients with IBS. Although increasing fiber may help improve constipation in patients with
constipation-predominant IBS, this does not improve abdominal pain. In some studies adding insoluble
fiber resulted in either worsening of symptoms or no change in symptoms.

Clarithromycin was studied in a single randomized, controlled trial and found not to be effective compared
with placebo. Loperamide has not been successful for reducing abdominal pain compared with placebo in
patients with IBS.

Ref: American College of Gastroenterology Task Force on Irritable Bowel Syndrome, Brandt LJ, Chey WD, et al: An
evidence-based position statement on the management of irritable bowel syndrome. Am J Gastroenterol 2009;104(Suppl
1):S1-S35. 2) Saha L: Irritable bowel syndrome: Pathogenesis, diagnosis, treatment, and evidence-based medicine. World
J Gastroenterol 2014;20(22):6759-6773.

Item 155

ANSWER: A

This patient presents with classic symptoms of Addisons disease, which is an autoimmune adrenalitis in
which the adrenal cortex is destroyed. This results in the loss of mineralocorticoid, glucocorticoid, and
adrenal androgen hormone production. Common symptoms of Addisons disease include anorexia,
weakness, fatigue, gastrointestinal symptoms, hypotension, salt cravings, postural dizziness, vitiligo,
muscle pain, and joint pain. Hyperpigmentation is the most common physical finding and is generally
distributed diffusely over the entire body. It can also be seen in the palmar creases, at the vermillion border
of the lips, on the buccal mucosa, around the nipples, and around scars.

Low serum cortisol measured at 8 a.m. suggests adrenal insufficiency. Hyponatremia may also be seen,
due to cortisol and mineralocorticoid deficiencies, and hyperkalemia may occur as a result of the lack of
mineralocorticoids. If cortisol is low, a cosyntropin stimulation test is the first-line test for diagnosing
adrenal insufficiency.

52
People with Addisons disease require lifelong hormone therapy with glucocorticoids and
mineralocorticoids. They also require stress-dose glucocorticoids for illnesses and before surgical
procedures because they are unable to mount an adequate response to stress. Generally, the treatment will
be prednisone or hydrocortisone along with fludrocortisone. Men with Addisons disease do not need
testosterone replacement because their testes will produce adequate levels. Women may benefit from
testosterone replacement because the adrenal glands are their primary source of testosterone.

Ref: Michels A, Michels N: Addison disease: Early detection and treatment principles. Am Fam Physician 2014;89(7):563-568.

Item 156

ANSWER: B

Recombinant influenza vaccine is formulated without using eggs. Live attenuated influenza vaccine comes
only in a trivalent formulation. The other vaccines listed are all prepared using eggs.

Ref: Grohskopf LA, Olsen SJ, Sokolow LZ, et al; Centers for Disease Control and Prevention: Prevention and control of
seasonal influenza with vaccines: Recommendations of the Advisory Committee on Immunization Practices (ACIP)United
States, 201415 influenza season. MMWR Morb Mortal Wkly Rep 2014;63(32):691-697.

Item 157

ANSWER: D

Arthroscopic partial meniscectomy is the most common orthopedic procedure performed in the United
States. For patients without osteoarthritis of the knee, studies show meniscectomy for a tear of the
meniscus is no more beneficial than conservative therapy in terms of functional status at 6 months. In a
high-quality randomized, controlled trial involving patients with a medial meniscus tear but no
osteoarthritis, meniscectomy and sham surgery were equally effective (SOR B). The optimal approach in
patients with a degenerative tear of the meniscus is a physical therapy and exercise regimen.

Ref: McClester Brown M, Mounsey A: Surgery for persistent knee pain? No so fast. J Fam Pract 2014;63(9):534-536.

Item 158

ANSWER: D

Analysis of 78 randomized clinical trials has shown an increase in all-cause mortality associated with
supplementation with vitamin E, vitamin A, and $-carotene. No benefits or reductions in all-cause
mortality were demonstrated for vitamin C or selenium (SOR A).

Ref: Bjelakovic G, Nikolova D, Gluud C: Antioxidant supplements to prevent mortality. JAMA 2013;310(11):1178-1179.

53
Item 159

ANSWER: A

The best initial test for the diagnosis of male hypogonadism is measurement of total testosterone in serum
in a morning sample. Low concentrations of testosterone in serum should be confirmed by repeat
measurement. If abnormalities in concentrations of sex hormonebinding globulin are suspected,
measurement of free or bioavailable testosterone is indicated. Examples of conditions associated with
altered sex hormonebinding globulin include liver disease, obesity, and diabetes mellitus.

Ref: Basaria S: Male hypogonadism. Lancet 2014;383(9924):1250-1263.

Item 160

ANSWER: C

When investigating presumed hyper- or hypothyroidism, TSH is the first-line test (SOR A). If the patient
is found to have an abnormal TSH level, free T4 is the next test to order. A free T3 test can also be helpful,
but the free T4 assay is not affected by changes in iodothyronine-binding proteins, and T3 is often a
peripheral product and can be abnormal due to nonthyroid diseases or medications. Occasionally, free T4
and T3 tests are performed as second-line tests, even if the TSH is normal, if the results do not match the
clinical picture. Other second- and third-line tests include measurement of thyroid antibodies, such as
antithyroid peroxidase and antithyroglobulin. Routine thyroid screening tests are not indicated for
asymptomatic adults (SOR A).

Ref: Gaitonde DY, Rowley KD, Sweeney LB: Hypothyroidism: An update. Am Fam Physician 2012;86(3):244-251. 2) Tessier
J, Downen M, Engel-Brower J, et al: Pitfalls and pearls for 8 common lab tests. J Fam Pract 2014;63(4):198-205. 3) Final
Recommendation Statement: Thyroid Dysfunction: Screening. US Preventive Services Task Force, 2015.

Item 161

ANSWER: A

A 2013 update from the American College of Obstetricians and Gynecologists on hypertension in
pregnancy summarizes the evidence regarding prevention of preeclampsia. The only medication with
sufficient evidence to support its routine use is aspirin at dosages of 6080 mg daily. In a high-risk
population, defined as women with a history of preeclampsia in two or more pregnancies or a history of
preeclampsia with delivery at <34 weeks, the risk of preeclampsia is sufficiently high to justify the use
of aspirin, with a number needed to treat of 50 to prevent one case of preeclampsia. Calcium
supplementation may help prevent preeclampsia in women with a very low calcium intake, but in the
United States and other developed countries routine calcium supplementation has not been found to provide
a benefit with regard to preeclampsia. Vitamin E has also been studied and found to be of no benefit.
Antihypertensive agents such as labetalol and nifedipine may be used to control blood pressure in pregnant
patients but they have not been shown to reduce the risk of preeclampsia.

Ref: American College of Obstetricians and Gynecologists; Task Force on Hypertension in Pregnancy: Hypertension in
pregnancy. Report of the American College of Obstetricians and Gynecologists Task Force on Hypertension in Pregnancy.
Obstet Gynecol 2013;122(5):1122-1131.

54
Item 162

ANSWER: A

Most patients with asymptomatic gallstones can be managed expectantly with no treatment unless symptoms
of biliary colic develop (SOR B). Only about 2% of such patients will develop symptoms. Once symptoms
start, recurrence of pain, obstruction of the biliary or pancreatic duct, and the potential for attendant
complications such as pancreatitis or ascending cholangitis become significantly more likely. In selected
patients, oral dissolution therapy, ERCP, or lithotripsy may be effective alternative therapies, but
laparoscopic cholecystectomy is clearly the treatment of choice for symptomatic cholelithiasis (SOR A).

Ref: Abraham S, Rivero HG, Erlikh IV, et al: Surgical and nonsurgical management of gallstones. Am Fam Physician
2014;89(10):795-802.

Item 163

ANSWER: B

This patient has primary monosymptomatic enuresis, the most common type of nocturnal enuresis. Primary
refers to a child who has never achieved 6 months of continuous dry nights. Monosymptomatic refers to
the absence of daytime symptoms such as dysuria or urinary frequency. Children with daytime urinary
symptoms have a higher incidence of urinary tract pathology and require further diagnostic evaluation.

Primary monosymptomatic enuresis has a spontaneous annual remission rate of about 15% and does not
require treatment unless the patient (not just the parent) is concerned about the issue. Treatment requires
participation from both the child and the parents, so ensuring interest from both parties is key. Bed alarms
have the best evidence for long-term success in that they train children via classical conditioning to awaken
at the onset of urination and get up to finish voiding into the toilet.

Reward systems for achieving dry nights have some evidence of benefit but it is difficult to determine if
they are superior to the spontaneous remission rate. Medications such as desmopressin, imipramine, and
oxybutynin have a role in addressing nocturnal enuresis if bed alarm use is unsuccessful or if parents and
children are not willing to engage in the activities necessary to implement the therapy. Medications may
work well while they are used, but enuresis commonly recurs when they are stopped.

Ref: Baird DC, Seehusen DA, Bode DV: Enuresis in children: A case-based approach. Am Fam Physician 2014;90(8):560-568.

Item 164

ANSWER: C

The 2013 ACC/AHA cholesterol guidelines outline four major groups in whom statin therapy is beneficial:
(1) individuals with clinical atherosclerotic cardiovascular disease (ASCVD), (2) those with primary
elevations of LDL-C >190 mg/dL, (3) patients age 4075 with diabetes mellitus, an LDL-C level of
70189 mg/dL, and no clinical ASCVD, (4) patients age 4075 without clinical ASCVD or diabetes, an
LDL-C level of 70189 mg/dL, and an estimated 10-year ASCVD risk >7.5%. For patients age 4075
with diabetes, an LDL-C level of 70189 mg/dL, and no clinical ASCVD, a moderate-intensity statin is
recommended.

55
Ref: Stone NJ, Robinson JG, Lichtenstein AH, et al: 2013 ACC/AHA guideline on the treatment of blood cholesterol to reduce
atherosclerotic cardiovascular risk in adults: A report of the American College of Cardiology/American Heart Association
Task Force on practice guidelines. Circulation 2014;129(25 Suppl 2):S1-S45.

Item 165

ANSWER: A

Fentanyl is one of the preferred narcotics in patients with end-stage renal disease. Fentanyls elimination
is 99% hepatic and it has a long history of safe use in patients with renal failure. Morphine,
hydromorphone, and hydrocodone can be used in these patients, but these drugs require close monitoring
for side effects and indications for dosage reduction because they have active metabolites that accumulate
in patients with renal failure. Meperidine, codeine, and propoxyphene are all contraindicated in chronic
kidney disease because of the accumulation of toxic metabolites.

Ref: O'Connor NR, Corcoran AM: End-stage renal disease: symptom management and advance care planning. Am Fam
Physician 2012;85(7):705-710.

Item 166

ANSWER: E

Aseptic olecranon bursitis is often preceded by minor trauma to the elbow followed by a nontender, boggy
mass over the olecranon. Septic olecranon bursitis causes not just swelling, but also erythema, warmth,
and pain. Half of affected individuals will have a fever. If septic bursitis is suspected, aspiration with
bursal fluid analysis should be done and antibiotic therapy should be initiated. Aspiration is not
recommended for the initial treatment of aseptic bursitis, as complications such as infection may occur.
Management initially is with ice, compression dressings, and avoidance of activities that aggravate the
problem. If conservative therapy is unsuccessful the problem can be managed by aspiration followed by
compression dressings for 2 weeks. The bursa may be injected with a corticosteroid, but this could cause
skin atrophy or infection. Surgical bursectomy can be offered for refractory cases lasting over 3 months.

Ref: Kane SF, Lynch JH, Taylor JC: Evaluation of elbow pain in adults. Am Fam Physician 2014;89(8):649-657.

Item 167

ANSWER: C

Bicuspid aortic valve is the most likely cause of heart valve disease in this family. It is the most common
congenital heart defect in the United States, with a prevalence of approximately 1%2%. This valve
disorder appears to have a genetic basis, with an autosomal dominant pattern of inheritance and incomplete
penetrance. The children of a patient with a bicuspid aortic valve have about a 10% chance of having this
condition; it is therefore recommended to screen first degree relatives of affected patients with
echocardiography. Most patients with a bicuspid aortic valve will eventually have significant aortic valve
dysfunction (stenosis or insufficiency) and/or aortopathy such as aortic root dilation.

Rheumatic heart disease can also cause valve disease but its incidence is low in the United States,
especially with appropriate treatment for streptococcal pharyngitis. Endocarditis is unlikely in the absence
of systemic symptoms. Hypertension and coronary atherosclerosis are unlikely in this patient because of
his normal blood pressure. These conditions also are more likely to cause obstructive coronary disease and
heart failure than valve disease.

56
Ref: Losenno KL, Chu MW: Bicuspid aortic valve disease. CMAJ 2013;185(18):1599. 2) Otto CM, Prendergast B: Aortic-valve
stenosisFrom patients at risk to severe valve obstruction. N Engl J Med 2014;371(8):744-756.

Item 168

ANSWER: E

An ankle-brachial index (ABI) is considered normal between 1.00 and 1.40, borderline from 0.91 to 0.99,
and abnormal if 0.90. The lower the ABI, the more severe peripheral artery disease is likely to be.
Values greater than 1.40 indicate incompressible vessels and are not reliable. Incompressible vessels may
be found in patients with long-standing diabetes mellitus, or in older persons. A toe-brachial index
measurement may be used in persons with incompressible arteries of the more proximal lower extremity.

Ref: Hauk L: ACCF/AHA update peripheral artery disease management guideline. Am Fam Physician 2012;85(10):1000-1001.

Item 169

ANSWER: E

The U.S. Preventive Services Task Force recommends one-time screening for hepatitis C for individuals
born between the years 1945 and 1965 (USPSTF B recommendation). As far as screening for the other
problems listed, there is no significant evidence to determine whether this should be done on a widespread
basis.

Ref: US Preventive Services Task Force: Screening for Hepatitis C Virus Infection in Adults: Final Recommendation Statement.
AHRQ pub no 12-05174-EF-2, 2013.

Item 170

ANSWER: D

While all of the antibiotics listed have been used to treat Lyme disease, the only antibiotic that has been
shown to be effective for chemoprophylaxis is doxycycline. A randomized, controlled trial showed that
a single 200-mg dose of doxycycline was 87% effective for preventing Lyme disease if given within 72
hours after removal of a deer tick. Nevertheless, a meta-analysis showed that the number needed to treat
to prevent one case of erythema migrans was 50, and routine prophylaxis is not recommended. It may be
indicated, however, after removal of an engorged nymphal deer tick.

Ref: Shapiro ED: Lyme disease. N Engl J Med 2014;370(18):1724-1731.

Item 171

ANSWER: C

Burns can be classified based on the depth and area of the burn. Only superficial and deep-thickness burns
are included in the calculation of the burn area. Minor burns cover less than 10% of the body for patients
1050 years old and <5% of the body for patients <10 or >50 years old. Any burn involving the face,
hands, or a major joint may be more complicated and should be promptly evaluated.

57
Superficial burns involve the epidermis and appear as painful patches of erythema and dry skin. Superficial
partial-thickness burns involve part of the dermis and all of the epidermis. They cause painful blanching
erythema with small blisters and weeping skin. This patient has a superficial burn but in a high-risk area.

Immediate management of a minor burn may include cooling with water but should not involve ice water
as this may lead to further injury (SOR C). All wounds should be cleaned with sterile water but not a
cleansing agent such as povidone iodine (SOR C). The skin should remain intact if possible and small
blisters should not be debrided. Topical corticosteroids should be avoided, as they do not reduce
inflammation.

Superficial burns do not require antibiotics or wound dressings. They can be treated with aloe vera, lotion,
antibiotic ointment, or honey (SOR B). There is evidence that these treatments promote skin repair and
prevent drying. Aloe vera may also decrease pain. There is also evidence that honey heals partial thickness
wounds more quickly than conventional dressings.

Ref: Lloyd EC, Rodgers BC, Michener M, Williams MS: Outpatient burns: Prevention and care. Am Fam Physician
2012;85(1):25-32. 2) Jull AB, Cullum N, Dumville JC, et al: Honey as a topical treatment for wounds. Cochrane Database
Syst Rev 2015;(3):CD005083.

Item 172

ANSWER: D

The diagnosis of hereditary hemochromatosis requires a random measurement of serum ferritin and
calculation of transferrin saturation. The transferrin saturation is calculated by dividing the serum iron level
by the total iron binding capacity. If the serum ferritin level is elevated (>200 ng/mL in women) or the
transferrin saturation is 45% the HFE gene should be checked. Measurement of liver transaminases plays
a role in determining liver disease but is not helpful in the diagnosis.

Ref: Crownover BK, Covey CJ: Hereditary hemochromatosis. Am Fam Physician 2013;87(3):183-190.

Item 173

ANSWER: E

The classic symptoms of hypoparathyroidism are those of insufficient calcium. Typically these include
refractory heart failure, tetany, seizures, altered mental status, and stridor. Refractory heart failure is
related to the low calcium interfering with the normal contractility of myocytes. Low vitamin D can cause
hypocalcemia but is not caused by it. Patients are not at risk for hyperkalemia if they have
hypoparathyroidism. Seizures, not somnolence, and muscle twitching, not flaccidity, are symptoms of low
calcium.

Ref: Shoback D: Hypoparathyroidism. N Engl J Med 2008;359(4):391-403. 2) Michels TC, Kelly KM: Parathyroid disorders.
Am Fam Physician 2013;88(4):249-257.

58
Item 174

ANSWER: A

Vagal maneuvers and administration of adenosine are useful in the diagnosis and treatment of
narrow-complex supraventricular tachycardias. Adenosine, a very short-acting endogenous nucleotide that
blocks atrioventricular nodal conduction, terminates nearly all atrioventricular nodal reentrant tachycardias
and atrioventricular reciprocating tachycardias, as well as up to 80% of atrial tachycardias. Although
intravenous verapamil and diltiazem, which also block the atrioventricular node, have a potential diagnostic
and therapeutic use in narrow-complex tachycardia, they may cause hypotension and thus are not a first
choice in the emergency setting. Electrical cardioversion is reserved for patients who do not respond to
adenosine. Antiarrhythmic agents are rarely necessary in the early management of supraventricular
tachycardias, with the exception of the management of arrhythmias that have caused hemodynamic
instability and that have not responded to electrical cardioversion. In these cases, procainamide and
ibutilide can be used.

Ref: Link MS: Evaluation and initial treatment of supraventricular tachycardia. N Engl J Med 2012;367(15):1438-1448.

Item 175

ANSWER: A

It is important to be able to tell children and their families that many small umbilical hernias resolve
without surgical repair and that the rate of both incarceration prior to surgery and complications from
surgery are very low. Surgical repair of pediatric umbilical hernias is indicated if the hernia has not
resolved by 35 years of age or for incarcerated hernias at any age. The primary care physician may
observe younger children who are asymptomatic, limiting the need for surgical referral. Smaller hernias
(<1.01.5 cm in diameter) typically resolve more quickly than larger hernias. The surgery is usually done
on an outpatient basis, which is often reassuring to parents. Less than 1% of patients experience
incarceration. Imaging studies are not routinely required and applying pressure over the defect has no
benefit.

Ref: Kaiser GL: Symptoms and Signs in Pediatric Surgery. Springer, 2012. 2) Holcomb GW III, Murphy JP, Ostlie DJ:
Ashcrafts Pediatric Surgery, ed 6. Saunders Elsevier, 2014.

Item 176

ANSWER: D

In April 2005 the FDA issued a boxed warning for second-generation antipsychotics, including quetiapine,
after a meta-analysis demonstrated a 1.6- to 1.7-fold increase in the risk of death associated with their use
in elderly patients with dementia, related in part to sudden cardiac death and also to stroke. In June 2008,
after two large cohort studies showed a similar risk with first-generation antipsychotics, boxed warnings
were added to this class as well. The other medications listed do not have this association or warning.

Ref: Schneider LS, Dagerman KS, Insel P: Risk of death with atypical antipsychotic drug treatment for dementia: Meta-analysis
of randomized placebo-controlled trials. JAMA 2005;294(15):1934-1943. 2) Gill SS, Rochon PA, Herrmann N, et al:
Atypical antipsychotic drugs and risk of ischaemic stroke: Population based retrospective cohort study. BMJ
2005;330(7489):445. 3) Schneeweiss S, Setoguchi S, Brookhart A, et al: Risk of death associated with the use of
conventional versus atypical antipsychotic drugs among elderly patients. CMAJ 2007;176(5):627-632. 4) Muench J, Hamer
AM: Adverse effects of antipsychotic medications. Am Fam Physician 2010;81(5):617-622.

59
Item 177

ANSWER: C

If recommended prior to surgery, $-blockers should be started several weeks beforehand and carefully
titrated. They may be harmful if initiated in the immediate perioperative period. Statins are recommended
in the perioperative period for vascular surgery regardless of other cardiac risk factors; a statin would
ideally have been initiated previously in this case, but may still be started in the immediate preprocedural
period. There is no specific indication in this case for an ACE inhibitor.

Ref: Holt NF: Perioperative cardiac risk reduction. Am Fam Physician 2012;85(3):239-246.

Item 178

ANSWER: A

Allergic contact dermatitis is secondary to a trigger that incites a delayed (type IV) hypersensitivity
reaction. The most common sensitizers include plants (poison ivy, poison oak, and poison sumac), metals
(nickel found in jewelry or belt buckles), and fragrances. Patch testing data has shown that out of 3700
known contact allergens, nickel caused contact dermatitis in 14.3% of patients, fragrance mix in 14%,
neomycin in 11.6%, balsam of Peru in 10.4%, and thimerosal in 10.4%. The rash is limited to the area
of exposure and is characterized by an intensely pruritic papular eruption with erythema. Herpes simplex
is characterized by a vesicular eruption surrounded by erythema and associated with localized burning and
tingling. Tinea corporis presents as a pruritic circular or oval erythematous lesion with superficial scaling
and erythema. Multiple oval or circular pruritic salmon-colored scaly lesions preceded by a herald patch
are typical of pityriasis rosea (SOR C).

Ref: Usatine RP, Riojas M: Diagnosis and management of contact dermatitis. Am Fam Physician 2010;82(3):249-255.

Item 179

ANSWER: D

The most common serum tumor marker used for pancreatic ductal adenocarcinoma is cancer antigen 19-9,
which is expressed in pancreatic and hepatobiliary disease. In symptomatic patients it can help confirm the
diagnosis and aid in assessing the prognosis and predicting the likelihood of recurrence after resection.

CA-125 may be a useful marker with ovarian carcinoma, and "-fetoprotein may be followed as a marker
of hepatoma. Neither serum amylase nor "-1-antitrypsin is useful as a tumor marker.

Ref: De La Cruz MS, Young AP, Ruffin MT: Diagnosis and management of pancreatic cancer. Am Fam Physician
2014;89(8):626-632.

60
Item 180

ANSWER: C

Chest pain is a common presenting complaint in children and certainly can result from serious cardiac
pathology. However, the majority of chest pain in children is benign, and determining clinically which
patients need a cardiac workup is therefore paramount. Patients and families overestimate the prevalence
of cardiac causes of chest pain and underestimate the prevalence of more benign causes. The most common
cause of chest pain in children is musculoskeletal (50%60%) followed by psychogenic (10%30%) and
respiratory causes (3%12%). Cardiac conditions account for 0%5% of cases of chest pain in children.
Red flags that suggest a cardiac etiology include a patient history of palpitations with the chest pain, an
abnormal cardiac physical examination (rubs or gallops), exertional chest pain without another more likely
etiology such as asthma, and a positive family history. When any of the red flags is present, the patient
should be referred to a pediatric cardiologist. This patient has no red flags and the most likely etiology of
her chest pain is therefore musculoskeletal.

Ref: Friedman KG, Alexander ME: Chest pain and syncope in children: A practical approach to the diagnosis of cardiac disease.
J Pediatr 2013;163(3):896-901.

Item 181

ANSWER: C

The patient has EKG findings suggestive of an acute ST-elevation myocardial infarction. This is
demonstrated on the EKG by the presence of ST-segment elevation in contiguous leads I, aVL, and V6.
Troponin, a cardiac biomarker released from damaged myocardial cells, is elevated in patients with an
acute myocardial infarction. Elevated D-dimer suggests thromboembolism, but normal levels have a high
negative predictive value for ruling out pulmonary embolism. Hyperkalemia is associated with peaked
T-waves in multiple leads. EKG findings in patients with a pulmonary embolism include sinus tachycardia
with an S1Q3T3 pattern (T-wave inversion in III), incomplete right bundle branch block, and right
precordial T-wave inversions. Thyroid hormone abnormalities can be associated with nonspecific EKG
findings, but tachyarrhythmias (including atrial fibrillation) are more common in hyperthyroidism, whereas
bradycardia is more common in hypothyroidism (SOR C).

Ref: Kasper DL, Fauci AS, Hauser SL, et al (eds): Harrisons Principles of Internal Medicine, ed 19. McGraw-Hill, 2015, pp
1455-1466, 1600-1601.

Item 182

ANSWER: D

The U.S. Preventive Services Task Force (USPSTF) recommends screening smokers for lung cancer with
low-dose CT. Patients should be age 5580 and healthy. They should be current smokers or have quit
within the past 15 years, and have a 30-pack-year history of smoking. The screening test is low-dose CT
of the chest. Abdominal ultrasonography to screen for abdominal aneurysms is recommended for any male
age 6575 who has ever smoked (USPSTF B recommendation). A bone density test screens for
osteoporosis and is recommended for women age 65 or older or in younger women at increased risk. The
USPSTF recommends against PSA testing (D recommendation) for prostate cancer, as well as screening
for carotid artery stenosis.

61
Ref: Final Recommendation Statement: Osteoporosis: Screening. US Preventive Services Task Force, 2011. 2) Final
Recommendation Statement: Prostate Cancer: Screening. US Preventive Services Task Force, 2012. 3) Final
Recommendation Statement: Lung Cancer: Screening. US Preventive Services Task Force, 2013. 4) Final Recommendation
Statement: Abdominal Aortic Aneurysm: Screening. US Preventive Services Task Force, 2014. 5) Final Recommendation
Statement: Carotid Artery Stenosis: Screening. US Preventive Services Task Force, 2014.

Item 183

ANSWER: D

This patient has hypotonic hyponatremia, manifested by low serum osmolality. She is asymptomatic and
has no signs of hypovolemia on her laboratory tests or physical examination. Her urine sodium is high and
her urine osmolality is low, which indicates the syndrome of inappropriate secretion of antidiuretic
hormone (SIADH). This is most likely related to her pneumonia, which is improving. The initial treatment
for mild euvolemic hyponatremia is fluid restriction. Intravenous isotonic saline would be indicated for
mild hypovolemic hyponatremia. Intravenous hypertonic saline would be indicated for severe hyponatremia
with symptoms. Intravenous diuretics would be indicated for hypervolemic hyponatremia, such as in heart
failure, along with fluid and sodium restriction.

Ref: Braun MM, Barstow CH, Pyzocha NJ: Diagnosis and management of sodium disorders: Hyponatremia and hypernatremia.
Am Fam Physician 2015;91(5):299-307.

Item 184

ANSWER: D

Hyperplastic polyps <10 mm in size in the rectum and sigmoid colon carry a low risk for developing into
colon cancer. If they are the only finding, colonoscopy may be repeated in 10 years.

Ref: Short MW, Layton MC, Teer BN, Domagalski JE: Colorectal cancer screening and surveillance. Am Fam Physician
2015;91(2):93-100.

Item 185

ANSWER: E

Worldwide, hepatitis B is a common cause of liver failure, cirrhosis, and hepatocellular carcinoma. The
disease characteristically is asymptomatic before such complications develop. Although routine infant
vaccination against hepatitis B has greatly decreased the incidence of this infection in the United States,
it remains a significant cause of morbidity and mortality both in the United States and globally. Identifying
persons infected with hepatitis B allows vaccination of their household contacts and sexual partners,
thereby preventing further transmission. It also allows for medical treatment of infected individuals,
including antiviral therapy and monitoring for the development of cirrhosis or hepatocellular carcinoma.

The CDC recommends screening for hepatitis B in patients on hemodialysis, household contacts of
individuals with chronic hepatitis B, patients on immunosuppressive therapy, and all pregnant women.
Other individuals who should be screened include anyone exposed to bodily fluids of infected individuals,
such as sexual partners or infants of infected mothers. Behavioral risks such as intravenous drug use are
also an indication for screening. Patients from areas where HBsAg prevalence is >2% should also be
screened.

62
Ref: Weinbaum CM, Williams I, Mast EE, et al; Centers for Disease Control and Prevention (CDC): Recommendations for
identification and public health management of persons with chronic hepatitis B virus infection. MMWR Recomm Rep
2008;57(RR-8):1-20. 2) Peters MG, Weinbaum C, Tan L, et al: Recommendations for prevention, screening, and diagnosis
of HBV and HCV infections. J Fam Pract 2010;59(4 Suppl):S29-S35.

Item 186

ANSWER: C

Neurologic symptoms may develop with low-normal vitamin B12 levels in serum. In true vitamin B12
deficiency, methylmalonic acid and homocysteine levels are typically quite elevated, and these return to
normal with treatment. Gastrin levels may be abnormal in pernicious anemia, but are not diagnostic alone.
High ferritin levels are seen with increased iron stores in the liver, and ferritin levels are used to screen
for hemochromatosis.

Ref: Stabler SP: Vitamin B12 deficiency. N Engl J Med 2013;368(2):149-160.

Item 187

ANSWER: D

Neglect is the most common form of child abuse and is the most common type of abuse in children who
die as a result of abuse. In 2011, 79% of abused children suffered from neglect, 18% from physical abuse,
and 9% from sexual abuse. Among abused children who died, 71% suffered from neglect, 48% from
physical abuse, and less than 1% from sexual abuse. Neglect is defined as the failure of caregivers to
provide needed, age-appropriate care, even though the caregiver was financially able to do so or was
offered financial or other assistance to provide appropriate care.

Ref: Administration on Children, Youth and Families, Childrens Bureau. Child Maltreatment 2011. US Dept of Health and
Human Services, 2012. 2) Fan T, Pham A: Primary care interventions to prevent child maltreatment. Am Fam Physician
2014;90(4):255-256.

Item 188

ANSWER: D

Because hyperkalemia can have deleterious effects on the myocardium, an EKG is the first diagnostic test
in the workup of a patient with hyperkalemia. Although not all patients with hyperkalemia will have an
abnormal EKG, those who do need to be given intravenous calcium immediately to prevent arrhythmias
and cardiac arrest. A urinalysis, blood pH, and CBC are part of the workup to determine the etiology of
the hyperkalemia, but an EKG is the top priority. There is not a defined threshold for treatment of
hyperkalemia with intravenous calcium in asymptomatic patients without EKG changes. Likewise, renal
ultrasonography may be indicated for other reasons but is not part of the initial workup for hyperkalemia.

Ref: Medford-Davis L, Rafique Z: Derangements of potassium. Emerg Med Clin North Am 2014;32(2):329-347.

63
Item 189

ANSWER: E

Metformin is an inexpensive first-line oral agent for type 2 diabetes mellitus. Its mechanism of action is
to increase the sensitivity of the liver and peripheral tissues to insulin. This assists the patient with weight
loss efforts and, unlike insulin secretagogues, has been proven to reduce mortality with long-term use.
When metformin is used as monotherapy it is not associated with episodes of hypoglycemia. For many
years there has been a concern that metformin can increase the risk for lactic acidosis. This risk has been
assumed to be greater in conditions that can lead to tissue hypoperfusion, such as heart failure or
hypovolemia, or with renal impairment. The FDA has historically recommended against the use of
metformin for any patient with even mild renal impairment (creatinine >1.4 mg/dL for women and >1.5
mg/dL for men). However, a recent meta-analysis did not find supportive evidence for such restrictions.
Newer evidence suggests that the use of metformin is safe even with mild to moderate renal impairment
(eGFR >30 mL/min) (SOR A).

Ref: Salpeter SR, Greyber E, Pasternak GA, Salpeter EE: Risk of fatal and nonfatal lactic acidosis with metformin use in type
2 diabetes mellitus. Cochrane Database Syst Rev 2010;(4):CD002967. 2) Inzucchi SE, Lipska KJ, Mayo H, et al:
Metformin in patients with type 2 diabetes and kidney disease: A systematic review. JAMA 2014;312(24):2668-2675. 3)
AHFS Drug Information 2015. American Society of Health-System Pharmacists, 2015.

Item 190

ANSWER: D

Approximately 1.7 million breast, colon, prostate, and other carcinomas are diagnosed in the United States
each year. More than 2.5 million basal cell carcinomas will be diagnosed. Most of these will be treated,
including more than 100,000 in the patients last year of life. These are very slow growing tumors that
rarely metastasize, and asymptomatic basal cell carcinomas rarely need treatment in frail older patients.

Ref: Linos E, Schroeder SA, Chren MM: Potential overdiagnosis of basal cell carcinoma in older patients with limited life
expectancy. JAMA 2014;312(10):997-998.

Item 191

ANSWER: A

Behavioral therapy should be the primary treatment for attention-deficit/hyperactivity disorder (ADHD)
in children younger than 6 years, and it may be helpful at older ages (SOR B). Treatment of ADHD in
children 6 years and older should start with medication (SOR B).

Ref: Felt BT, Biermann B, Christner JG, et al: Diagnosis and management of ADHD in children. Am Fam Physician
2014;90(7):456-464.

64
Item 192

ANSWER: D

Hoarding disorder is included in the DSM-5. It is more common than previously realized, affecting
between 2% and 6% of adults. It is characterized by excessive, often dangerous, clutter and disorganized
living spaces. The items collected or saved often are worthless, such as old newspapers and paperwork,
but may also be valuable items. Opposed to this is normal collecting, which is organized and pleasurable,
and does not lead to dangerous or chaotic living spaces. Both could involve collecting unusual or seemingly
bizarre items. However, with hoarders, disposing of the items causes extreme anxiety and emotional
distress.

Ref: Mataix-Cols D: Hoarding disorder. N Engl J Med 2014;370(21):2023-2030.

Item 193

ANSWER: E

A nontreponemal test, such as the rapid plasma reagin (RPR) test or Venereal Disease Research Laboratory
(VDRL) test, is the initial step for evaluating a patient with suspected syphilis. These tests become positive
within 3 weeks of the appearance of the primary chancre, so they may be negative in patients with an early
infection. Darkfield microscopy of material obtained from a swab of the lesion is often useful in this
situation, but it requires special equipment and experienced technicians. If there is a strong suspicion of
syphilis, a repeat nontreponemal test in 2 weeks is indicated. Patients with a positive nontreponemal test
should be tested with a specific treponemal test for confirmation. These tests may lack reactivity in early
primary syphilis, however, and are not indicated for use in the initial evaluation. Spinal fluid analysis is
used only for the evaluation of tertiary syphilis.

Ref: Mattei PL, Beachkofsky TM, Gilson RT, Wisco OJ: Syphilis: A reemerging infection. Am Fam Physician
2012;86(5):433-440.

Item 194

ANSWER: C

Trials have evaluated various antidepressant medications as aids in tobacco cessation. Both bupropion and
nortriptyline have been found to increase smoking cessation success rates. Because this patient does not
want to try bupropion, nortriptyline would be a reasonable option. Studies have shown similar efficacy for
these two medications, although there is a lack of evidence for increased efficacy when these medications
are added to nicotine replacement therapy. Studies have not shown a benefit for promoting tobacco
cessation with SSRIs such as fluoxetine, monoamine oxidase inhibitors such as selegiline, opioid
antagonists such as naltrexone, or St. Johns wort.

Ref: Hughes JR, Stead LF, Hartmann-Boyce J, et al: Antidepressants for smoking cessation. Cochrane Database Syst Rev
2014;(1):CD000031. 2) Hartmann-Boyce J, Stead LF, Cahill K, Lancaster T: Efficacy of interventions to combat tobacco
addiction: Cochrane update of 2013 reviews. Addiction 2014;109(9):1414-1425.

65
Item 195

ANSWER: E

A screening tests specificity is the proportion of persons without the condition who test negative for that
condition. In other words, it is a measure of the tests ability to properly identify those who do not have
the disease. Conversely, the sensitivity of a screening test is the proportion of those with the condition who
test positive. The other options listed describe false-negatives, false-positives, and prevalence.

Ref: Goldman L, Schafer AI (eds): Goldmans Cecil Medicine, ed 25. Elsevier Saunders, 2016, p 37.

Item 196

ANSWER: B

Recommendations for the treatment of hospitalized infants with bronchiolitis include nasal suctioning via
bulb or neosucker to clear the upper airway. Deep suction (beyond the nasopharynx) is not recommended.
Oxygen is recommended for infants with a persistent oxygen saturation <90%. Bronchodilators should
not be used routinely in the management of bronchiolitis, and corticosteroids, antibiotics, nasal
decongestants, and chest physiotherapy are not recommended. A single trial of inhaled epinephrine or
albuterol for respiratory distress may be considered, but only if there is a history of asthma, atopy, or
allergy.

Ref: Mittal V, Darnell C, Walsh B, et al: Inpatient bronchiolitis guideline implementation and resource utilization. Pediatrics
2014;133(3):e730-e737.

Item 197

ANSWER: D

The use of bisphosphonates is associated with a small increase in the risk of atypical femoral shaft
fractures. The risk increases with the duration of use (SOR B). These drugs are also associated with an
increased risk of osteonecrosis of the jaw, esophagitis, and esophageal ulceration, as well as hypocalcemia.
In fact, bisphosphonates are used as a treatment for hypercalcemia. They do not affect phosphorus or
vitamin D levels.

Ref: Scott MA, Meadowcraft L, Skolnik D: Risks of bisphosphonate use. Am Fam Physician 2013;88(10):697, 702.

66
Item 198

ANSWER: A

Carbon monoxide (CO) exposure most commonly results from fuel combustion in heaters, stoves, or
automobiles, so it is most often seen during cold periods when people are in closed quarters. Symptoms
include headache, nausea, vomiting, and weakness, and patients have a flushed complexion, so symptoms
are commonly attributed to viral flu-like illnesses. CO poisoning results in the formation of
carboxyhemoglobin, which does not carry oxygen. All oxygen-carrying sites are occupied by CO, which
has such a high affinity for hemoglobin that oxygen cannot displace it. If a patient has a
carboxyhemoglobin level of 25%, and their hemoglobin level is 12 mg/dL, their effective hemoglobin level
is only 9 mg/dL since 25% of their hemoglobin is not carrying oxygen. If the carboxyhemoglobin level
is 25%, then the maximum oxygen saturation that can be attained is 75%. However, a pulse oximeter will
show an oxygen saturation of 100% because the color of carboxyhemoglobin is bright red, which is what
the pulse oximeter is detecting. Thus, pulse oximetry is not reliable in patients with CO poisoning.

Similarly, arterial blood gas measurements are based on oxygen gas tension (pO2) and not oxygen content
or true oxygen saturation. The only arterial blood gas abnormality in CO poisoning may be metabolic
acidosis, which is a consequence of inadequate oxygen delivery to the peripheral tissues. This causes an
anaerobic metabolism and lactic acid production, but is not seen early in CO poisoning. Serious cases of
pneumonia, ARDS, or methemoglobinemia would produce abnormalities on pulse oximetry or arterial
blood gas measurements. To detect CO poisoning it would be necessary to order either a CO level or a
co-oximetry test.

Ref: Tintinalli JE, Kelen GD, Stapczynski JS (eds): Emergency Medicine: A Comprehensive Study Guide, ed 7. McGraw-Hill,
2011, pp 1410-1413.

Item 199

ANSWER: A

The Centers for Disease Control and Prevention (CDC) provides specific recommendations for backup
contraception after IUD insertion. According to the CDC guidelines, this patient does not need to use
backup contraception if her IUD is inserted today because it was inserted within 7 days after menstrual
bleeding started. If the levonorgestrel IUD is inserted more than 7 days after menstrual bleeding starts,
the patient needs to abstain from sexual intercourse or use additional contraceptive protection for the next
7 days.

Ref: Division of Reproductive Health, National Center for Chronic Disease Prevention and Health Promotion, Centers for
Disease Control and Prevention (CDC): US selected practice recommendations for contraceptive use, 2013: Adapted from
the World Health Organization selected practice recommendations for contraceptive use, 2nd edition. MMWR Recomm
Rep 2013;62(RR-05):1-60.

67
Item 200

ANSWER: A

SGLT2 inhibitors inhibit SGLT2 in the proximal nephron. This blocks glucose reabsorption by the kidney,
increasing glucosuria. The advantages of this medication include no hypoglycemia, decreased weight,
decreased blood pressure, and effectiveness at all stages of type 2 diabetes mellitus. Disadvantages are that
it increases the risk of genitourinary infections, polyuria, and volume depletion and increases
LDL-cholesterol and creatinine levels. GLP-1 receptor agonists work by activating the GLP-1 receptors,
causing an increase in insulin secretion, a decrease in glucagon secretion, slowing of gastric emptying, and
increasing satiety. DPP-4 inhibitors inhibit DPP-4 activity, which increases postprandial active incretin
concentration. This increases insulin secretion and decreases glucagon secretion. Meglitinides act by
closing the ATP-sensitive K+ channels on the B-cell plasma membranes, which increases insulin secretion.
"-Glucosidase inhibitors inhibit intestinal "-glucosidase, which slows intestinal carbohydrate digestion and
absorption.

Ref: American Diabetes Association: Standards of medical care in diabetes2015: 7. Approaches to glycemic treatment.
Diabetes Care 2015;38(Suppl):S41-S48.

Item 201

ANSWER: C

Eccentric exercise should be the first-line treatment for chronic midsubstance Achilles tendinopathy.
Corticosteroid injections, bracing, and NSAIDs are not effective for providing long-term relief for chronic
degenerative tendon injuries. Therapeutic ultrasonography is a reasonable second-line alternative.

Ref: Childress MA, Beutler A: Management of chronic tendon injuries. Am Fam Physician 2013;87(7):486-490.

Item 202

ANSWER: C

Vancomycin, 125 mg orally 4 times daily for 1014 days, is recommended for the first severe episode of
Clostridium difficile colitis (SOR B). If the first episode is mild to moderate, oral metronidazole, 500 mg
3 times daily for 1014 days, would be preferred. Intravenous vancomycin is not effective in the treatment
of colitis. Rifaximin is not well studied and is not recommended in any current guidelines.

Ref: Cohen SH, Gerding DN, Johnson S, et al: Clinical practice guidelines for Clostridium difficile infection in adults: 2010
update by the Society for Healthcare Epidemiology of America (SHEA) and the Infectious Diseases Society of America
(IDSA). Infect Control Hosp Epidemiol 2010;31(5):431-455.

Item 203

ANSWER: C

Structural non-atherosclerotic heart disease is the predominant cause of sudden death in young athletes.
Hypertrophic cardiomyopathy, an autosomal dominant condition with variable expression, accounts for
more than one-third of these cases. Coronary artery abnormalities are second in frequency as a cause of
sudden cardiac death in this population, with idiopathic ventricular hypertrophy third.

68
Ref: Chandra N, Bastiaenen R, Papadakis M, Sharma S: Sudden cardiac death in young athletes: Practical challenges and
diagnostic dilemmas. J Am Coll Cardiol 2013;61(10):1027-1040.

Item 204

ANSWER: D

Most thoracic aortic aneurysms are asymptomatic, but symptoms can be produced by distortion,
compression, or erosion of adjacent structures by the aneurysm. Resulting symptoms include cough,
hemoptysis, chest pain, hoarseness, and dysphagia. A chest radiograph showing widening of the
mediastinum and prominence of the aortic arch and thoracic aorta suggests a thoracic aortic aneurysm.
Contrast-enhanced CT, MRI, and aortography are sensitive and specific tests for assessment of thoracic
aneurysms and involvement of branch vessels. Echocardiography (especially transesophageal) helps in
further evaluating the proximal ascending and descending thoracic aorta.

A pulmonary cavitary lesion, seen in pulmonary tuberculosis, is typically located in the upper lung lobe
and is often associated with mediastinal lymphadenopathy. The presence of a retrocardiac gas-filled
structure suggests the presence of a hiatal hernia. The chest radiograph may show a water bottle
configuration of the cardiac silhouette in a patient with pericardial effusion (SOR C).

Ref: Kasper DL, Fauci AS, Hauser SL, et al (eds): Harrisons Principles of Internal Medicine, ed 19. McGraw-Hill, 2015, pp
1638-1639.

Item 205

ANSWER: E

The patient has typical signs and symptoms of testicular torsion despite inconclusive ultrasonography.
Surgical exploration is necessary because the testicle can be salvaged if the torsion is repaired within 6
hours of symptom development (SOR C).

Ref: Crawford P, Crop JA: Evaluation of scrotal masses. Am Fam Physician 2014;89(9):723-727.

Item 206

ANSWER: E

Patients who are diagnosed with celiac disease are at increased risk of osteoporosis due to bone loss from
decreased calcium and vitamin D absorption. These patients are at higher risk for fractures. Patients with
celiac disease are not at increased risk for inflammatory bowel disease, diverticulitis, or colon cancer.

Ref: Leffler D: Celiac disease diagnosis and management: A 46-year-old woman with anemia. JAMA 2011;306(14):1582-1592.
2) Pelkowski TD, Viera AJ: Celiac disease: Diagnosis and management. Am Fam Physician 2014;89(2):99-105.

69
Item 207

ANSWER: A

Acute stress disorder (ASD) lies on a spectrum of trauma-related disorders between adjustment disorder
and posttraumatic stress disorder (PTSD). ASD is differentiated from PTSD primarily by duration, with
PTSD requiring the presence of similar symptoms (intrusion, negative mood, dissociation, avoidance, and
arousal) for longer than 1 month. Conversely, adjustment disorder is a less severe condition than ASD that
involves either a less traumatic or threatening inciting event and/or less severe symptoms that do not meet
DSM-5 criteria for acute stress disorder.

Ref: Kavan MG, Elsasser GN, Barone EJ: The physicians role in managing acute stress disorder. Am Fam Physician
2012;86(7):643-649. 2) American Psychiatric Association: Diagnostic and Statistical Manual of Mental Disorders, ed 5.
American Psychiatric Association, 2013, pp 265-290.

Item 208

ANSWER: A

Enteral nutrition is preferred over parenteral nutrition for patients with severe pancreatitis who have been
on prolonged bowel rest, and it is associated with lower complication rates and shorter hospitalizations
(SOR A). Prophylactic antibiotics should only be used when there is significant necrosis (SOR C).
Similarly, surgical debridement is indicated only if there is infected necrosis or persistent fluid collections
(SOR C).

Ref: Quinlan JD: Acute pancreatitis. Am Fam Physician 2014;90(9):632-639.

Item 209

ANSWER: B

Endometriosis is caused by menstrual tissue in the pelvic peritoneal cavity. Infertility, dysmenorrhea, and
dyspareunia with postcoital bleeding are common. Although laparoscopy with histology is the definitive
test, transvaginal ultrasonography is the noninvasive test of choice. CA-125 will often be elevated but is
nonspecific. CT and MRI also have low specificity, and colonoscopy is of no value in the evaluation of
endometriosis.

Ref: Schrager S, Falleroni J, Edgoose J: Evaluation and treatment of endometriosis. Am Fam Physician 2013;87(2):107-113.

Item 210

ANSWER: B

Chagas disease is caused by Trypanosoma cruzi, and is estimated to infect some 300,000 persons in the
United States. Potential consequences include cardiomyopathy, heart failure, and fatal cardiac arrhythmias.
The CDC has designated Chagas disease as a neglected parasitic infection, based on the number of people
estimated to be infected in the United States, the potential severity of the illness, and the ability to prevent
and treat this disease. This infection is considered neglected because relatively little attention has been
devoted to its surveillance, prevention, and/or treatment. It is most common in those who live in rural,
impoverished areas in Mexico or central America, where the vector of the disease, the kissing bug, is
found.

70
Trichomoniasis can lead to infertility and poor birth outcomes. Toxocariasis and toxoplasmosis cause
developmental defects in children. Cysticercosis can lead to epilepsy in young adults. Some of these
sequelae develop years after an initial mild infection.

Ref: Woodhall D, Jones JL, Cantey PT, et al: Neglected parasitic infections: What every family physician needs to know. Am
Fam Physician 2014;89(10):803-811.

Item 211

ANSWER: A

The Ottawa Ankle Rules are widely accepted guidelines for appropriate evaluation of ankle and midfoot
injuries occurring in adults age 19 or older presenting for the first time in a clinical setting. The guidelines
utilize the historical and physical findings to determine which radiographic studies, if any, are indicated.
Patients who were able to bear weight immediately following their injury and who can take 4 steps
independently in a clinical setting require radiographic study only when the following criteria are met: pain
is present in the malleolar zone and bony tenderness of the posterior edge or tip of either malleolus is
elicited (ankle radiograph), or pain is present in the midfoot zone and bony tenderness of either the base
of the fifth metatarsal or the navicular region is present.

Ref: Tiemstra JD: Update on acute ankle sprains. Am Fam Physician 2012;85(12):1170-1176.

Item 212

ANSWER: C

A D recommendation means the U.S. Preventive Services Task Force (USPSTF) recommends against
the service. There is moderate or high certainty that the service has no net benefit or that the harms
outweigh the benefits. An I recommendation means the USPSTF concludes that the evidence is lacking,
of poor quality, or conflicting, and the balance of benefits and harms cannot be determined. A C
recommendation means the USPSTF recommends selectively offering or providing this service to
individual patients based on professional judgment and patient preferences. There is at least moderate
certainty that the net benefit is small. A B recommendation means the USPSTF recommends the service.
There is high certainty that the net benefit is moderate or there is moderate certainty that the net benefit
is moderate to substantial. An A recommendation means the USPSTF recommends the service and there
is high certainty that the net benefit is substantial. The highest levels of evidence and most recent evidence
available are used by the USPSTF in making all of its recommendations.

Ref: Grade definitions. US Preventive Services Task Force, 2014.

Item 213

ANSWER: C

According to the DSM-5, the level of severity of anorexia nervosa is based on the patients body mass
index (BMI). Mild is a BMI >17.0 kg/m2, moderate is a BMI of 16.016.99 kg/m2, severe is a BMI of
15.015.9 kg/m2, and extreme is a BMI <15.0 kg/m2. Recurrent episodes of binge eating or purging
behavior help differentiate restricting type from binge-eating/purging type, but do not indicate severity.
Orthostatic changes in pulse or blood pressure and refusal to eat are criteria for inpatient hospitalization,
but are not part of the classification of severity according to the DSM-5. Amenorrhea can be a clinical sign
of anorexia nervosa but is not part of the classification of severity.

71
Ref: Harrington BC, Jimerson M, Haxton C, Jimerson DC: Initial evaluation, diagnosis, and treatment of anorexia nervosa and
bulimia nervosa. Am Fam Physician 2015;91(1):46-52.

Item 214

ANSWER: C

The radiograph shown depicts a right-sided spontaneous pneumothorax. Primary spontaneous


pneumothorax, which results from the rupture of subpleural apical blebs, typically affects young men who
are smokers with no underlying history of lung disease. The recommended treatment is needle aspiration
of air from the pleural space (SOR B). In a reliable patient with a small (<15% of a hemithorax), stable
spontaneous primary pneumothorax, observation alone may be appropriate. There is no role for
intravenous heparin or corticosteroids in the management of pneumothorax. The Valsalva maneuver could
potentially expand an underlying tension pneumothorax.

Ref: Kasper DL, Fauci AS, Hauser SL, et al (eds): Harrisons Principles of Internal Medicine, ed 19. McGraw-Hill, 2015, p
1719.

Item 215

ANSWER: B

Risk factors for progression from latent to active tuberculosis include lung cancer, diabetes mellitus,
alcoholism, recent contact with a person who has an active tuberculosis infection, any condition treated
with immunosuppressive therapy, and lung parenchymal diseases such as COPD, silicosis, or lung cancer.
The medically underserved and those in low-income groups are also more at risk of progression, as well
as children under age 5 and individuals weighing less than 90% of their ideal minimum body weight.

Ref: Hartman-Adams H, Clark K, Juckett G: Update on latent tuberculosis infection. Am Fam Physician 2014;89(11):889-896.

Item 216

ANSWER: C

Influenza vaccine is indicated for all pregnant women, and there are no known deleterious effects on the
course of pregnancy or the fetus. Women are advised to avoid pregnancy for 28 days after receiving MMR
or varicella vaccines. HPV vaccine is not recommended during pregnancy.

Ref: Guidelines for vaccinating pregnant women. Centers for Disease Control and Prevention, 2013.

Item 217

ANSWER: C

Nearly 75% of pregnant women are affected by nausea and vomiting of pregnancy. Though dietary
modifications are often recommended, there is little evidence to support their use. Vitamin B6 is
recommended as first-line therapy. It is safe to use in the first trimester and is associated with less
drowsiness compared with other medications.

72
Scopolamine is effective for nausea and vomiting of pregnancy but should be avoided in the first trimester
due to the possibility of causing trunk and limb deformities. Likewise, methylprednisolone is also effective
but should be avoided in the first trimester as it is associated with an increased risk of cleft palate if used
before 10 weeks of gestation. Auricular acupressure has been found to be ineffective.

Ref: Puangsricharern A, Mahasukhon S: Effectiveness of auricular acupressure in the treatment of nausea and vomiting in early
pregnancy. J Med Assoc Thai 2008;91(11):1633-1638. 2) Herrell HE: Nausea and vomiting of pregnancy. Am Fam
Physician 2014;89(12):965-970.

Item 218

ANSWER: E

According to the American Heart Associations 2007 guidelines, prophylaxis to prevent bacterial
endocarditis associated with dental, gastrointestinal, or genitourinary procedures is now indicated only for
high-risk patients with prosthetic valves, a previous history of endocarditis, unrepaired cyanotic congenital
heart disease (CHD), or CHD repaired with prosthetic material, and for cardiac transplant recipients who
develop valvular disease.

Based on a risk-benefit analysis in light of available evidence for and against antibiotic prophylaxis, these
recommendations specifically exclude mitral valve prolapse and acquired valvular disease, even if they are
associated with mitral regurgitation. The American Dental Association has endorsed this guideline.

Ref: Wilson W, Taubert KA, Gewitz M, et al: Prevention of infective endocarditis: Guidelines from the American Heart
Association. Circulation 2007;116(15):1736-1754.

Item 219

ANSWER: E

Treatment of hypertriglyceridemia depends on its severity. Contributing factors include a sedentary


lifestyle, being overweight, excessive alcohol intake, type 2 diabetes mellitus, and genetic disorders.
Triglyceride levels of 150199 mg/dL are considered mild hypertriglyceridemia, levels of 200999 mg/dL
are moderate, 10001999 mg/dL are severe, and levels >2000 mg/dL are considered very severe. Patients
with hypertriglyceridemia in the mild to moderate range may be at risk for cardiovascular disease, but
those who have severe or very severe hypertriglyceridemia have a significant risk of pancreatitis.

In addition to having the patient exercise, reduce intake of fat and carbohydrates, and lose weight, she
should also be counseled to avoid alcohol. For patients at risk for pancreatitis, fibrates are recommended
as the initial treatment for pancreatitis. It should be noted that statins may have a modest
triglyceride-lowering effect and may be helpful in decreasing cardiovascular risk in those who have
moderately elevated triglycerides. However, they should not be used alone in patients who have severe
hypertriglyceridemia. Studies have also shown that while omega-3 fatty acids decrease triglycerides and
very low density lipoprotein cholesterol levels, they may increase LDL-cholesterol levels. Treatment with
omega-3 fatty acids does not decrease total mortality or cardiovascular events, and therefore is not
recommended.

Niacin does seem to have the advantage of raising HDL cholesterol and lowering LDL cholesterol, but it
has never been proven in clinical trials to have benefit with regard to the primary outcome of
cardiovascular disease, and some trials have shown significant increases in adverse events.

73
Ref: Stone NJ, Robinson JG, Lichtenstein AH, et al; American College of Cardiology/American Heart Association Task Force
on Practice Guidelines: 2013 ACC/AHA guideline on the treatment of blood cholesterol to reduce atherosclerotic
cardiovascular risk in adults: A report of the American College of Cardiology/American Heart Association Task Force on
Practice Guidelines. J Am Coll Cardiol 2014;63(25 Pt B):2889-2934. 2) Armstrong C: Endocrine Society releases
guidelines on diagnosis and management of hypertriglyceridemia. Am Fam Physician 2013;88(2):142-144. 2) Ebell MH:
Niacin does not improve clinical outcomes in patients with vascular disease. Am Fam Physician 2014;90(9):660-661. 3)
Narla R, Peck SB, Qiu KM: Fish oil for treatment of dyslipidemia. Am Fam Physician 2014;89(4):288, 290.

Item 220

ANSWER: D

Approximately 0.3% of patients taking methimazole develop agranulocytosis, usually within the first 60
days of starting therapy. Other rare complications of methimazole include serum sickness, cholestatic
jaundice, alopecia, nephrotic syndrome, hypoglycemia, and loss of taste. It is associated with an increased
risk of fetal anomalies, so propylthiouracil (PTU) is preferred in pregnancy. The other medications listed
are not known to cause the combination of agranulocytosis and cholestatic jaundice that this patient has.

Ref: Papadakis MA, McPhee SJ (eds): Current Medical Diagnosis & Treatment. McGraw-Hill, 2014, p 1074.

Item 221

ANSWER: B

Although there is no specific recommendation about when to initiate pharmacotherapy for the treatment
of gestational diabetes mellitus (GDM), many women do require specific treatment beyond diet and
exercise. Insulin has traditionally been used but oral medications are becoming increasingly common
despite the lack of long-term safety data. Many outcomes for both the mother and infant are improved with
pharmacologic management of GDM. These include a decreased risk for operative delivery,
large-for-gestational-age infants, shoulder dystocia, and maternal preeclampsia. Although a significant
percentage of women with GDM subsequently develop type 2 diabetes mellitus after delivery,
pharmacologic treatment of GDM has not been shown to decrease that risk. In addition, neither perinatal
death nor the likelihood of small-for-gestational-age infants is significantly affected. The risk of neonatal
hypoglycemia has also not consistently been shown to be affected by treatment.

Ref: Horvath K, Koch K, Jeitler K, et al: Effects of treatment in women with gestational diabetes mellitus: Systematic review
and meta-analysis. BMJ 2010;340:c1395. 2) Committee on Practice BulletinsObstetrics: Practice bulletin no. 137:
Gestational diabetes mellitus. Obstet Gynecol 2013;122(2 Pt 1):406-416. 3) Balsells M, Garcia-Patterson A, Sol I, et al:
Glibenclamide, metformin, and insulin for the treatment of gestational diabetes: A systematic review and meta-analysis.
BMJ 2015;350:h102.

Item 222

ANSWER: B

Motor side effects of the antipsychotic drugs can be separated into five general categories: dystonias,
parkinsonism, akathisia, withdrawal dyskinesias, and tardive dyskinesia. Akathisia is a syndrome marked
by motor restlessness. Affected patients commonly complain of being inexplicably anxious, of being unable
to sit still or concentrate, and of feeling comfortable only when moving. Hysteria is no longer considered
a useful term.

74
Ref: Hales RE, Yudofsky SC, Roberts LW (eds): Textbook of Psychiatry, ed 6. American Psychiatric Publishing, 2014, pp
944-945. 2) Goldman L, Schafer AI (eds): Goldmans Cecil Medicine, ed 25. Elsevier Saunders, 2016, pp 2467-2468.
3) Muench J, Hamer AM: Adverse effects of antipsychotic medications. Am Fam Physician 2010;81(5):617-622.

Item 223

ANSWER: C

All of the diagnoses listed are intertriginous rashes but only erythrasma fluoresces with Woods light.
Erythrasma is a superficial gram-positive bacterial infection caused by Corynebacterium minutissimum.
The fluorescence is caused by porphyrins. Erythrasma is most often seen between the toe web spaces,
followed by the groin and axillae. There are multiple treatments, including topical and oral erythromycins
and clindamycins (level of evidence 3, strength of evidence 1).

Ref: Vary JC, OConnor KM: Common dermatologic conditions. Med Clin North Am 2014;98(3):445-485.

Item 224

ANSWER: A

All of the drugs listed are appropriate for uterine atony and postpartum hemorrhage. Carboprost should
not be used in this patient, however, as it is contraindicated in patients with asthma. Methylergonovine is
contraindicated in hypertensive patients but may be used in patients with asthma.

Ref: Alexander JM, Wortman AC: Intrapartum hemorrhage. Obstet Gynecol Clin North Am 2013;40(1):15-26.

Item 225

ANSWER: A

This case is typical for acute parotitis, which is commonly caused by dehydration and can be diagnosed
from the history and examination. Empiric treatment is directed toward gram-positive and anaerobic
organisms, with the most common pathogen being Staphylococcus. These are often penicillin resistant so
a $-lactamase inhibitor is the agent of choice. Treatment should be followed up with cultures.
Administration of sialagogues such as lemon drops may be helpful, as well as parotid gland massage.

CT or MRI may help confirm the diagnosis but imaging is usually not necessary. The history and clinical
examination are most important for making the diagnosis. Incision and drainage would be appropriate only
for an abscess, and surgical removal of the parotid gland is not indicated.

Ref: Mandel L: Salivary gland disorders. Med Clin North Am 2014;98(6):1407-1449. 2) Wilson KF, Meier JD, Ward PD:
Salivary gland disorders. Am Fam Physician 2014;89(11):882-888.

Item 226

ANSWER: A

Any child younger than 29 days old with a fever and any child who appears toxic, regardless of age,
should undergo a complete sepsis workup and be admitted to the hospital for observation until culture
results are known or the source of the fever is found and treated (SOR C).

75
Observation only, with close follow-up, is recommended for nontoxic infants 336 months of age with a
temperature <39.0C (102.2F) (SOR C). Children 2990 days old who appear to be nontoxic and have
negative screening laboratory studies, including a CBC and urinalysis, can be sent home with precautions
and with follow-up in 24 hours (SOR B). Testing for neonatal herpes simplex virus infection should be
considered in patients with risk factors, including maternal infection at the time of delivery, use of fetal
scalp electrodes, vaginal delivery, cerebrospinal fluid pleocytosis, or herpetic lesions. Testing also should
be considered when a child does not respond to antibiotics (SOR C).

Ref: Hamilton JL, John SP: Evaluation of fever in infants and young children. Am Fam Physician 2013;87(4):254-260.

Item 227

ANSWER: C

Family physicians see many patients with aortic stenosis (AS) and it is important to know when and if
further workup is indicated for asymptomatic patients. Although aortic stenosis can result in adverse
cardiac events, most of these events occur in patients who are symptomatic. Thus, the American Heart
Association and the American College of Cardiology recommend that asymptomatic patients with mild
aortic stenosis undergo repeat echocardiography every 35 years. Further workup or treatment is not
indicated for patients who have mild AS and are asymptomatic. Exercise treadmill testing may be indicated
in patients with severe AS based on echocardiography even if they are asymptomatic.

Use of statin drugs has not been shown to slow or stop progression of AS. Right and left heart
catheterization can be used in an attempt to resolve discrepancies between symptoms and echocardiographic
findings. Because this patient is asymptomatic and her echocardiogram shows only mild AS, left and/or
right heart catheterization is not indicated. An ACE inhibitor would be indicated in patients who have a
reduced ejection fraction.

Ref: Bonow RO, Carabello BA, Chatterjee K, et al: 2008 focused update incorporated into the ACC/AHA 2006 guidelines for
the management of patients with valvular heart disease: A report of the American College of Cardiology/American Heart
Association Task Force on Practice Guidelines (Writing Committee to Revise the 1998 Guidelines for the Management of
Patients With Valvular Heart Disease). Endorsed by the Society of Cardiovascular Anesthesiologists, Society for
Cardiovascular Angiography and Interventions, and Society of Thoracic Surgeons. J Am Coll Cardiol 2008;52(13):e1-e142.

Item 228

ANSWER: C

This patient has postoperative delirium, which is associated with an increased mortality rate. Reorientation
and pain management are important management strategies. Benzodiazepines, antipsychotics,
antidepressants, and restraints are not helpful and may make the situation worse. Imaging modalities are
not helpful in the absence of localizing signs.

Ref: Clinical Practice Guideline for Postoperative Delirium in Older Adults. American Geriatrics Society, 2014.

76
Item 229

ANSWER: B

Torus palatinus is an exostosis, or benign bony overgrowth. It is usually located on the midline of the hard
palate, and occurs in 12%27% of the population. Since these are usually not symptomatic many people
are not even aware of their presence.

Torus palatinus is easily diagnosed from the history and physical examination. Imaging studies are usually
unnecessary. These growths typically enlarge gradually throughout life but have no potential for malignant
transformation.

Ref: Ladizinski B, Lee KC: A nodular protuberance on the hard palate. JAMA 2014;311(15):1558-1559.

Item 230

ANSWER: E

Undiagnosed vitamin D deficiency is not uncommon, and 25-hydroxyvitamin D is the barometer for
vitamin D status. Although there is no consensus on optimal levels of 25-hydroxyvitamin D as measured
in serum, vitamin D deficiency is defined by most experts as a 25-hydroxyvitamin D level <20 ng/mL
(50 nmol/L).

Ref: Rosen CJ: Vitamin D insufficiency. N Engl J Med 2011;364(3):248-54. 2) Goldman L, Schafer AI (eds): Goldmans Cecil
Medicine, ed 25. Elsevier Saunders, 2016, p 1446.

Item 231

ANSWER: A

Sharply demarcated lesions with raised borders surrounding a paler region containing a darker center
(target or iris lesions) are characteristic of erythema multiforme. The lesions of erythema multiforme
usually appear on the distal extremities, are often accompanied by burning and pruritus, and may progress
centrally. Usually the rash resolves spontaneously within 46 weeks but some patients experience frequent
recurrences. Erythema multiforme results from a hypersensitivity reaction to any number of medications,
vaccine preparations, or infections, the most commonly identified being herpes simplex virus (HSV)
infection. In a minority of those harboring HSV infection, recurrent outbreaks of erythema multiforme are
often associated with HSV reactivations, even those that may occur unnoticed. Continuous antiviral
treatment using acyclovir, valacyclovir, or famciclovir has been shown to be effective in reducing or
eliminating the frequency of recurrent outbreaks in these patients (SOR A). In patients not helped by daily
antiviral suppressive therapy, treatment with dapsone, azathioprine, cyclosporine, and thalidomide have
been used with some success, but evidence-based data supporting the use of these drugs is limited.

Ref: Lamoreux MR, Sternbach MR, Hsu WT: Erythema multiforme. Am Fam Physician 2006;74(11):1883-1888. 2) Wetter
DA, Davis MD: Recurrent erythema multiforme: Clinical characteristics, etiologic associations, and treatment in a series
of 48 patients at Mayo Clinic, 2000 to 2007. J Am Acad Dermatol 2010;62(1):45-53.

77
Item 232

ANSWER: A

Fluoxetine is the only medication with consistent evidence showing that it improves depression symptoms
in children and adolescents, including a Cochrane review of three randomized trials. Escitalopram is
licensed for treatment of depression in children 12 and over, and consensus guidelines also recommend
the use of citalopram and sertraline as first-line treatment in children and adolescents. However, these
drugs do not have the same level of evidence for their effectiveness as fluoxetine.

Tricyclic antidepressants have not been shown to be more effective than placebo and should not be used
(SOR A). All antidepressants carry a black box warning about an increased risk of suicide with their use
in younger patients. It is recommended that children and adolescents be monitored closely, including
weekly contact. Psychotherapy should be used in conjunction with pharmacologic treatment.

Aripiprazole, a second-generation antipsychotic medication, would not be indicated. Paroxetine should not
be used in young people because of its association with increased suicide risk.

Ref: Clark MS, Jansen KL, Cloy JA: Treatment of childhood and adolescent depression. Am Fam Physician 2012;86(5):442-448.

Item 233

ANSWER: E

Lice, scabies, and secondary bacterial infections are endemic in the homeless. Body lice transmit
Bartonella quintana, which causes trench fever. This disease got its name in World War I, when soldiers
in the trenches were often infested with body lice. This is a serious disease that can be treated with
antibiotics.

Ref: National Center for Emerging and Zoonotic Infectious Diseases: Bartonella infection (cat scratch disease, trench fever, and
Carrins disease). Centers for Disease Control and Surveillance, 2012. 2) Maness DL, Khan M: Care of the homeless:
An overview. Am Fam Physician 2014;89(8):634-640.

Item 234

ANSWER: D

This patients clinical picture is most concerning for giant cell arteritis (also known as temporal arteritis).
This condition is a type of vasculitis and in its most serious form can lead to blindness. It is most common
in the elderly and is twice as common in women as in men. Because of its inflammatory nature, patients
commonly have systemic symptoms, including fever. The temporal artery may be thickened, tender, or
lacking pulsation, although a normal artery does not rule out the diagnosis. Jaw claudication is a fairly
specific but nonsensitive finding.

78
The laboratory finding most classically associated with giant cell arteritis is an elevated erythrocyte
sedimentation rate (ESR). Only 4% of patients with biopsy-proven giant cell arteritis have a normal ESR.
However, a high ESR is nonspecific and may be caused by other conditions. Because the treatment for
giant cell arteritis involves high-dose corticosteroids, which may cause significant morbidity, most
clinicians favor confirmation of the diagnosis with a temporal artery biopsy prior to committing a patient
to full treatment. MRI and CT would be used in the evaluation of other causes of headaches, including a
cerebral hemorrhage or mass. A lumbar puncture would identify benign intracranial hypertension or
meningitis, and an EEG would be helpful for evaluating seizures.

Ref: Caylor TL, Perkins A: Recognition and management of polymyalgia rheumatica and giant cell arteritis. Am Fam Physician
2013;88(10):676-684. 2) Weyand CM, Goronzy JJ: Giant-cell arteritis and polymyalgia rheumatica. N Engl J Med
2014;371(1):50-57.

Item 235

ANSWER: B

The American Academy of Pediatrics (AAP) recommends antibiotic therapy for children 6 months of age
or older with severe signs and symptoms of acute otitis media (AOM), including moderate or severe otalgia
or otalgia for more than 48 hours, or a temperature 39C (102F), whether the AOM is unilateral or
bilateral (SOR B). Children younger than 24 months without severe symptoms should receive antibiotic
therapy for bilateral AOM, whereas older children or those with unilateral AOM can be offered the option
of observation and follow-up.

The usual treatment for AOM is amoxicillin, but an antibiotic with additional $-lactamase coverage, such
as amoxicillin/clavulanate, should be given if the child has received amoxicillin within the past 30 days,
has concurrent purulent conjunctivitis, or has a history of AOM unresponsive to amoxicillin (SOR C).
Penicillin-allergic patients should be treated with an alternative antibiotic such as cefdinir, cefuroxime,
cefpodoxime, or ceftriaxone.

Ref: Lieberthal AS, Carroll AE, Chonmaitree T, et al: The diagnosis and management of acute otitis media. Pediatrics
2013;131(3):e964-e999.

Item 236

ANSWER: B

An inhaled daily low-dose corticosteroid plus occasional use of as-needed inhaled albuterol is the best
regimen for the treatment of exercised-induced bronchospasm. Daily use of short-acting $2-agonists can
lead to overuse and tolerance. Long-acting $2-agonists should not be used without the concomitant use of
an inhaled corticosteroid. Chronic oral corticosteroids are not indicated in this situation, and may require
a therapeutic use exemption by the sports authority overseeing athletic competitions. Immunotherapy has
limited benefit for the treatment of asthma.

Ref: Boulet LP, OByrne PM: Asthma and exercise-induced bronchoconstriction in athletes. N Engl J Med 2015;372(7):641-648.

79
Item 237

ANSWER: B

Ultrasonography is the preferred initial imaging modality for suspected acute cholecystitis or cholelithiasis
(SOR C). If ultrasound findings are equivocal, contrast CT, cholescintigraphy, or contrast MRI can be
used as second-line imaging modalities (SOR C). While useful in evaluating abdominal pain in some cases,
a plain radiograph would not be an appropriate first-line evaluation when cholecystitis or cholelithiasis is
suspected.

Ref: Crownover BK, Bepko JL: Appropriate and safe use of diagnostic imaging. Am Fam Physician 2013;87(7):494-501. 2)
Yarmish GM, Smith MP, Rosen MP, et al: ACR appropriateness criteria right upper quadrant pain. J Am Coll Radiol
2014;11(3):316-322.

Item 238

ANSWER: E

The recommended management for patients who have non-severe Salmonella infection and are otherwise
healthy is no treatment. Patients with high-risk conditions that predispose to bacteremia, and those with
severe diarrhea, fever, and systemic toxicity or positive blood cultures should be treated with levofloxacin,
500 mg once daily for 710 days (or another fluoroquinolone in an equivalent dosage), or with a slow
intravenous infusion of ceftriaxone, 12 g once daily for 710 days (14 days in patients with
immunosuppression).

Ref: DuPont HL: Acute infectious diarrhea in immunocompetent adults. N Engl J Med 2014;370(16):1532-1540.

Item 239

ANSWER: E

This child has cervical lymphadenitis, characterized by systemic symptoms, unilateral lymphadenopathy,
skin erythema, node tenderness, and a node that is 23 cm in size. The most common organisms associated
with lymphadenitis are Staphylococcus aureus and group A Streptococcus. Empiric antibiotic therapy with
observation for 4 weeks is acceptable for children with presumed reactive lymphadenopathy (SOR C). If
symptoms do not resolve, or if the mass increases in size during antibiotic treatment, further evaluation
is appropriate.

When imaging is indicated, ultrasonography is the preferred initial study for most children with a neck
mass. CT with intravenous contrast media is the preferred study for evaluating a malignancy or a suspected
retropharyngeal or deep neck abscess that may require surgical drainage. If the initial mass is suspicious
for malignancy (>3.0 cm in size, hard, firm, immobile, and accompanied by type B symptoms such as
fever, malaise, weight loss, or night sweats) immediate referral to a surgeon for evaluation and possible
biopsy is appropriate.

Ref: Meier JD, Grimmer JF: Evaluation and management of neck masses in children. Am Fam Physician 2014;89(5):353-358.

80
Item 240

ANSWER: B

This normotensive diabetic patient, appropriately screened for microalbuminuria, should have this finding
confirmed on at least one of two additional spot tests, since temporary factors other than nephropathy can
also result in microalbuminuria. Once a diagnosis of chronic kidney disease is confirmed, renal
ultrasonography should be ordered to detect potentially reversible causes.

A 24-hour urine is not necessary since the urine microalbumin/creatinine ratio correlates well with a
24-hour urine for albumin. Metformin is not contraindicated in the presence of microalbuminuria alone
without a decline in the glomerular filtration rate. The patient is already on high-intensity statin therapy
and there is no specific indication to increase the statin dosage based on his current LDL-cholesterol level
since treatment to the target LDL-cholesterol goal has fallen out of favor.

Ref: KDOQI: KDOQI clinical practice guidelines and clinical practice recommendations for diabetes and chronic kidney disease.
Am J Kidney Dis 2007;49(2 Suppl 2):S12S154. 2) Roett MA, Liegl S, Jabbarpour Y: Diabetic nephropathyThe family
physicians role. Am Fam Physician 2012;85(9):883-889. 3) Stone NJ, Robinson JG, Lichtenstein AH, et al: 2013
ACC/AHA guideline on the treatment of blood cholesterol to reduce atherosclerotic cardiovascular risk in adults: A report
of the American College of Cardiology/American Heart Association Task Force on Practice Guidelines. J Am Coll Cardiol
2014;63(25 Pt B):2889-2934.

81
American Board of Family Medicine

2014 IN-TRAINING EXAMINATION

CRITIQUE BOOK

This book contains the answers to each question in the In-Training Examination, as well as a critique that
provides a rationale for the correct answer. Bibliographic references are included at the end of each
critique to facilitate any further study you may wish to do in a particular area.

Copyright 2014 The American Board of Family Medicine, Inc. All rights reserved.
Item 1

ANSWER: E

Nonalcoholic fatty liver disease is characterized by the accumulation of fat in hepatocytes. It is associated
with insulin resistance, central adiposity, increased BMI, hypertension, and dyslipidemia. An incidentally
discovered elevated AST level in the absence of alcohol or drug-induced liver disease strongly suggests
the presence of nonalcoholic fatty liver disease. The goal of therapy is to prevent or reverse hepatic injury
and fibrosis. Diabetes mellitus, hypertension, dyslipidemia, and other comorbid conditions should be
appropriately managed.

A healthy diet, weight loss, and exercise are first-line therapeutic measures to reduce insulin resistance in
patients with nonalcoholic fatty liver disease. Weight loss has been shown to both normalize AST levels
and improve hepatic histology. Vitamin E has been shown to improve AST levels but has no impact on
liver histology, and pentoxifylline, simvastatin, and L-carnitine have not been shown to consistently
improve either AST levels or liver histology (SOR B).

Ref: Wilkins T, Tadkod A, Hepburn I, Schade RR: Nonalcoholic fatty liver disease: Diagnosis and management. Am Fam
Physician 2013;88(1):35-42.

Item 2

ANSWER: C

The only FDA-approved oral treatment for acne rosacea is doxycycline at a subantimicrobial dosage (40
mg daily). This does not contribute to antibiotic resistance, even when used over several months, and is
better tolerated than higher dosages. Other antibiotics have limited and low-quality supporting evidence
of efficacy and may lead to antibiotic resistance.

Ref: May D, Kelsberg G, Safranek S: What is the most effective treatment for acne rosacea? J Fam Pract 2011;60(2):108a-100c.
2) Del Rosso JQ, Thiboutot D, Gallo R, et al: Consensus recommendations from the American Acne and Rosacea Society
on the management of rosacea, part 3: A status report on systemic therapies. Cutis 2014;93(1):18-28.

Item 3

ANSWER: A

The American Academy of Dermatology recommends against the routine use of topical antibiotics for clean
surgical wounds, based on randomized, controlled trials. Topical antibiotics have not been shown to reduce
the rate of infection in clean surgical wounds compared to the use of nonantibiotic ointment or no ointment.
Studies have shown that white petrolatum ointment is as effective as antibiotic ointment in postprocedure
care.

Topical antibiotics can aggravate open wounds, hindering the normal wound-healing process. In addition,
there is a significant risk of developing contact dermatitis, as well as a potential for antibiotic resistance.
Antibiotic treatment should be reserved for wounds that show signs of infection.

Ref: Forsch RT: Essentials of skin laceration repair. Am Fam Physician 2008;78(8):945-951. 2) Levender MM, Davis SA,
Kwatra SG, et al: Use of topical antibiotics as prophylaxis in clean dermatologic procedures. J Am Acad Dermatol
2012;66(3):445-451. 3) American Academy of Dermatology: Five things physicians and patients should question. ABIM
Foundation Choosing Wisely campaign, 2014.

1
Item 4

ANSWER: B

The American Academy of Asthma, Allergy, and Immunology recommends that asthma not be diagnosed
or treated without spirometry. Once the diagnosis is confirmed, treatment should commence with a
short-acting $-agonist as needed, followed by stepwise treatment based on the severity of asthma.

Ref: Weiss LN: The diagnosis of wheezing in children. Am Fam Physician 2008;77(8):1109-1114. 2) Pollart SM, Elward KS:
Overview of changes to asthma guidelines: Diagnosis and screening. Am Fam Physician 2009;79(9):761-767. 3) Johnson
JD, Theurer WM: A stepwise approach to the interpretation of pulmonary function tests. Am Fam Physician
2014;89(5):359-366.

Item 5

ANSWER: E

This patient has diabetes insipidus, which is caused by a deficiency in the secretion or renal action of
arginine vasopressin (AVP). AVP, also known as antidiuretic hormone, is produced in the posterior
pituitary gland and the route of secretion is generally regulated by the osmolality of body fluid stores,
including intravascular volume. Its chief action is the concentration of urine in the distal tubules of the
kidney. Both low secretion of AVP from the pituitary and reduced antidiuretic action on the kidney can
be primary or secondary, and the causes are numerous.

Patients with diabetes insipidus present with profound urinary volume, increased frequency of urination,
and thirst. The urine is very dilute, with an osmolality <300 mOsm/L. Further workup will help
determine the specific type of diabetes insipidus and its cause, which is necessary for appropriate
treatment.

Low levels of aldosterone, plasma renin activity, or angiotensin would cause abnormal blood pressure,
electrolyte levels, and/or renal function. Insulin deficiency results in diabetes mellitus.

Ref: Longo DL, Fauci AS, Kasper DL, et al (eds): Harrisons Principles of Internal Medicine, ed 18. McGraw-Hill, 2012, pp
2904-2907.

Item 6

ANSWER: B

A reduction of the pain caused by abdominal palpation when the abdominal muscles are tightened is known
as Carnetts sign. If the cause of the pain is visceral, the taut abdominal muscles may protect the locus of
pain. In contrast, intensification of pain with this maneuver points to a source of pain within the abdominal
wall itself.

Ref: Ortiz DD: Chronic pelvic pain in women. Am Fam Physician 2008;77(11):1535-1542.

2
Item 7

ANSWER: B

The concept of a medical home was first suggested by the American Academy of Pediatrics in 1967 to
describe the ideal care of children with disabilities. In 2004 the Future of Family Medicine Project adapted
this concept to describe how primary care should be based on continuous, relationship-centered,
whole-system, comprehensive care for communities. In 2007 all of the major primary care organizations
collaborated to define the foundational principles of the patient-centered medical home (PCMH). These
principles include the following:

Comprehensiveness: Most preventive, acute, and chronic care for individual patients can be performed
at the PCMH.
Patient Centered: The PCMH provides care that is relationship-based, with an orientation toward the whole
person.
Coordination: The PCMH coordinates care for patients across all elements of the health care system.
Accessibility: The PCMH works to provide patients with timely access to providers.
Quality: The PCMH continuously works to improve care quality and safety.

Ref: Rakel RE, Rakel DP (eds): Textbook of Family Medicine, ed 8. Elsevier Saunders, 2011, pp 17-23. 2) Patient Centered
Medical Home Resource Center. Agency for Healthcare Research and Quality. http://pcmh.ahrq.gov

Item 8

ANSWER: E

Mild cognitive impairment is an intermediate stage between normal cognitive function and dementia. Motor
function remains normal. The presence of the APO E4 allele is a risk factor, but is not necessary for a
diagnosis. Patients have essentially normal functional activities but there is objective evidence of memory
impairment, and the patient may express concerns about cognitive decline.

Ref: Roberts R, Knopman DS: Classification and epidemiology of MCI. Clin Geriatr Med 2013;29(4):753-772.

Item 9

ANSWER: A

The Centers for Disease Control and Prevention (CDC) assembled a panel of national health experts to
develop evidence-based guidelines for evaluating and treating adults with acute respiratory disease.
According to these guidelines, the most reliable clinical predictors of streptococcal pharyngitis are the
Centor criteria. These include tonsillar exudates, tender anterior cervical lymphadenopathy, absence of
cough, and history of fever. The presence of three or four of these criteria has a positive predictive value
of 40%60%, and the absence of three or four of these criteria has a negative predictive value of 80%.

Patients with four positive criteria should be treated with antibiotics, those with three positive criteria
should be tested and treated if positive, and those with 01 positive criteria should be treated with
analgesics and supportive care only. This patient has only one of the Centor criteria, and should therefore
not be tested or treated with antibiotics.

Ref: McIsaac WJ, Kellner JD, Aufricht P, et al: Empirical validation of guidelines for the management of pharyngitis in children
and adults. JAMA 2004;291(13):1587-1595. 2) Choby BA: Diagnosis and treatment of streptococcal pharyngitis. Am Fam
Physician 2009;79(5):383-390.

3
Item 10

ANSWER: C

Conjugate vaccines against Haemophilus influenzae type b and Streptococcus pneumoniae have been highly
effective in reducing the incidence of bacterial meningitis in young children and are now routinely
recommended for infants and older patients who fall into appropriate risk groups (SOR B). Escherichia
coli and Listeria monocytogenes also cause meningitis in young children, but there is not currently a
routine vaccine for these pathogens. Likewise, Borrelia burgdorferi and Mycoplasma pneumoniae can
cause aseptic meningitis, but there is no routine vaccine.

Ref: Schuchat A, Robinson K, Wenger JD, et al: Bacterial meningitis in the United States in 1995. Active Surveillance Team.
N Engl J Med 1997;337(14):970-976. 2) Kupila L, Vuorinen T, Vainionp R, et al: Etiology of aseptic meningitis and
encephalitis in an adult population. Neurology 2006;66(1):75-80. 3) Hsu HE, Shutt KA, Moore MR, et al: Effect of
pneumococcal conjugate vaccine on pneumococcal meningitis. N Engl J Med 2009;360(3):244-256. 4) Bamberger DM:
Diagnosis, initial management, and prevention of meningitis. Am Fam Physician 2010;82(12):1491-1498.

Item 11

ANSWER: A

Targets for uric acid levels in patients with gout vary according to published guidelines but range from 5
to 6 mg/dL. Patients may be symptom-free at higher levels but risk joint damage even without acute
episodes (SOR A).

Ref: Shmerling RH: Management of gout: A 57-year-old man with a history of podagra, hyperuricemia, and mild renal
insufficiency. JAMA 2012;308(20):2133-2141.

Item 12

ANSWER: A

After confirmation of anemia and microcytosis on a CBC, a serum ferritin level is recommended (SOR
C). If the ferritin level is consistent with iron deficiency anemia, identifying the underlying cause of the
anemia is the priority. A common cause of iron deficiency anemia in premenopausal adult women is
menstrual blood loss. If the serum ferritin level is not consistent with iron deficiency anemia, the next stage
of the evaluation should include a serum iron level, total iron-binding capacity (TIBC), and transferrin
saturation (SOR C). Iron deficiency anemia is still probable if the serum iron level and transferrin
saturation are decreased and TIBC is increased. It is more likely anemia of chronic disease if the serum
iron level is decreased and the TIBC and transferrin saturation are decreased or normal. Other laboratory
tests that may help in differentiating the cause of microcytosis include hemoglobin electrophoresis, a
reticulocyte count, and peripheral blood smears.

Ref: Galloway MJ, Smellie WS: Investigating iron status in microcytic anaemia. BMJ 2006;333(7572):791-793. 2) Knovich
MA, Storey JA, Coffman LG, et al: Ferritin for the clinician. Blood Rev 2009;23(3):95-104. 3) Van Vranken M:
Evaluation of microcytosis. Am Fam Physician 2010;82(9):1117-1122. 4) Short MW, Domagalski JE: Iron deficiency
anemia: Evaluation and management. Am Fam Physician 2013;87(2):98-104.

4
Item 13

ANSWER: D

It is recommended that an implanted cardioverter-defibrillator be deactivated when it is inconsistent with


the care goals of the patient and family. In about one-quarter of patients with an implanted
cardioverter-defibrillator, the defibrillator delivers shocks in the weeks preceding death. For patients with
advanced irreversible disease, defibrillator shocks rarely prevent death, may be painful, and are distressing
to caregivers and family members. Advance care planning discussions should include the option of
deactivating the implanted cardioverter-defibrillator when it no longer supports the patients goals.

Ref: Goldstein NE, Lampert R, Bradley E, et al: Management of implantable cardioverter defibrillators in end-of-life care. Ann
Intern Med 2004;141(11):835-838. 2) Berger JT: The ethics of deactivating implanted cardioverter defibrillators. Ann
Intern Med 2005;142(8):631-634. 3) American Academy of Hospice and Palliative Medicine: Five things physicians and
patients should question. ABIM Foundation Choosing Wisely campaign, 2014.

Item 14

ANSWER: E

Except for persistent pubertal gynecomastia, medication use and substance use are the most common
causes of nonphysiologic gynecomastia. Common medication-related causes include the use of
antipsychotic agents, antiretroviral drugs, or prostate cancer therapies. Spironolactone also has a high
propensity to cause gynecomastia; other mineralocorticoid receptor antagonists, such as eplerenone, have
not been associated with similar effects. Discontinuing the contributing agent often results in regression
of breast tissue within 3 months.

Ref: Dickson G: Gynecomastia. Am Fam Physician 2012;85(7):716-722.

Item 15

ANSWER: E

The Advisory Committee on Immunization Practices (ACIP) periodically makes recommendations for
routine or postexposure immunization for a number of preventable diseases, including tetanus. Since 2005,
the recommendation for tetanus prophylaxis has included coverage not only for diphtheria (Td) but also
pertussis, due to waning immunity in the general population. The current recommendation for adults who
require a tetanus booster (either as a routine vaccination or as part of treatment for a wound) is to use the
pertussis-containing Tdap unless it has been less than 5 years since the last booster in someone who has
completed the primary vaccination series.

In this scenario, no additional vaccination is needed at this time, since the patient is certain of completing
the primary vaccinations and received a tetanus booster within the previous 5 years. Had the interval been
longer than 5 years, then a single dose of Tdap would be appropriate unless his previous booster was Tdap.
Tetanus immune globulin is recommended in addition to tetanus vaccine for wounds that are tetanus-prone
due to contamination and tissue damage in persons with an uncertain primary vaccine history. Plain tetanus
toxoid (TT) is usually indicated only when the diphtheria component is contraindicated, which is
uncommon.

5
Ref: Kretsinger K, Broder KR, Cortese MM, et al: Preventing tetanus, diphtheria, and pertussis among adults: Use of tetanus
toxoid, reduced diphtheria toxoid and acellular pertussis vaccine recommendations of the Advisory Committee on
Immunization Practices (ACIP) and recommendation of ACIP, supported by the Healthcare Infection Control Practices
Advisory Committee (HICPAC), for use of Tdap among health-care personnel. MMWR Recomm Rep
2006;55(RR-17):1-37. 2) Centers for Disease Control and Prevention (CDC): Updated recommendations for use of tetanus
toxoid, reduced diphtheria toxoid and acellular pertussis (Tdap) vaccine from the Advisory Committee on Immunization
Practices, 2010. MMWR Morb Mortal Wkly Rep 2011;60(1):13-15. 3) Bader MS, McKinsey DS: Postexposure
prophylaxis for common infectious diseases. Am Fam Physician 2013;88(1):25-32.

Item 16

ANSWER: D

Patients with repeated EKGs showing a QTc interval >480 ms with a syncopal episode, or >500 ms in
the absence of symptoms, are diagnosed with long QT syndrome if no secondary cause such as medication
use is present. This syndrome occurs in 1 in 2000 people and consists of cardiac repolarization defects.
It is associated with polymorphic ventricular tachycardia, including torsades de pointes, and sudden cardiac
death. It may be treated with $-blockers and implanted cardioverter defibrillators.

Ref: Longo DL, Fauci AS, Kasper DL, et al (eds): Harrisons Principles of Internal Medicine, ed 18. McGraw-Hill, 2012, pp
1897-1900.

Item 17

ANSWER: D

The initial management of hypercalcemic crisis involves volume repletion and hydration. The combination
of inadequate fluid intake and the inability of hypercalcemic patients to conserve free water can lead to
calcium levels >1415 mg/dL. Because patients often have a fluid deficiency of 45 liters, delivering
1000 mL of normal saline during the first hour, followed by 250300 mL/hour, may decrease the
hypercalcemia to less than critical levels (<13 mg/dL). If the clinical status is not satisfactory after
hydration alone, then renal excretion of calcium can be enhanced by saline diuresis using furosemide.

Intravenous pamidronate, a bisphosphonate, reduces the hypercalcemia of malignancy and is best used in
the semi-acute setting, since calcium levels do not start to fall for 24 hours. Glucocorticoids are useful in
the treatment of hypercalcemia associated with certain malignancies (multiple myeloma, leukemia, several
lymphomas, and breast cancer) or with vitamin D intoxication. The onset of action, however, takes several
days, with the effect lasting days to weeks.

Ref: Longo DL, Fauci AS, Kasper DL, et al (eds): Harrisons Principles of Internal Medicine, ed 18. McGraw-Hill, 2012, pp
3111-3113.

6
Item 18

ANSWER: B

Quadriceps tendon rupture can be partial or complete. When complete, as in this case, the patient has no
ability to straighten the leg actively. A similar pattern is seen with patellar tendon rupture, but in this
situation the patella is retracted superiorly by the quadriceps. Quadriceps rupture often produces a sulcus
sign, a painful indentation just above the patella. If the patient is not examined soon after the injury, the
gap in the quadriceps can fill with blood so that it is no longer palpable. The clinical examination is usually
diagnostic for this condition, but this patients radiograph shows some interesting findings, especially on
the lateral view. A small shard of the patella has been pulled off and has migrated superiorly with the
quadriceps. The hematoma filling the gap in the quadriceps is the same density as the muscle, but
wrinkling of the fascia over the distal quadriceps provides a clue that it is no longer attached to the superior
margin of the patella.

Tibial plateau fractures are intra-articular, so they produce a large hemarthrosis. They are evident on a
radiograph in almost all cases. Pain inhibits movement of the knee, but the extreme weakness evident in
this case would not be seen.

Patellar subluxation is obvious acutely, when the patella is displaced laterally. More often, the patient
comes in after the patella has relocated. Findings then include tenderness along the medial retinaculum,
sometimes a joint effusion, and a positive apprehension sign when the patella is pushed gently laterally.

Lumbar radiculopathy can cause weakness of the quadriceps if it involves the third lumbar root, but
complete paralysis would not occur. Other findings would include lumbar pain radiating to the leg, possibly
with paresthesias and fasciculations if there were significant neurologic impairment.

Ref: Ilan DI, Tejwani N, Keschner M, Leibman M: Quadriceps tendon rupture. J Am Acad Orthop Surg 2003;11(3):192-200.
2) Goldman L, Schafer AI (eds): Goldmans Cecil Medicine, ed 24. Elsevier Saunders, 2011, pp 1680-1681.

Item 19

ANSWER: B

Transvaginal ultrasonography is the preferred initial test for a patient with painless postmenopausal
bleeding, although endometrial biopsy is an option if transvaginal ultrasonography is not available.
Transvaginal ultrasonography showing an endometrial thickness <34 mm would essentially rule out
endometrial carcinoma (SOR C). An endometrial biopsy is invasive and has low sensitivity for focal
lesions. Saline infusion hysterography should be considered if the endometrial thickness is greater than the
threshold, or if an adequate measurement cannot be obtained by ultrasonography. If hysterography shows
a global process, then a histologic diagnosis can usually be obtained with an endometrial biopsy, but if a
focal lesion is present hysteroscopy should be considered as the next diagnostic step. Colposcopy is not
indicated given the patients normal Papanicolaou smear.

Ref: Apgar BS, Kaufman AJ, Bettcher C, Parker-Featherstone E: Gynecologic procedures: Colposcopy, treatments for cervical
intraepithelial neoplasia and endometrial assessment. Am Fam Physician 2013;87(12):836-843.

7
Item 20

ANSWER: C

Systemic corticosteroid therapy reduces the hospital length of stay in patients with acute COPD
exacerbations (SOR A). Oral therapy has been shown to be as effective as the intravenous route in patients
who can tolerate oral intake (SOR B). A randomized, controlled trial has demonstrated that 5-day courses
of systemic corticosteroid therapy are at least as effective as 14-day courses (SOR A). Inhaled
corticosteroids are beneficial in some COPD patients but nebulizers generally do not offer significant
advantages over metered-dose inhalers in most patients.

Ref: Evensen AE: Management of COPD exacerbations. Am Fam Physician 2010;81(5):607-613. 2) Slawson D: Five-day
steroid treatment effective for acute COPD exacerbation. Am Fam Physician 2013;88(11). 3) Leuppi JD, Schuetz P,
Bingisser R, et al: Short-term vs conventional glucocorticoid therapy in acute exacerbations of chronic obstructive
pulmonary disease: The REDUCE randomized clinical trial. JAMA 2013;309(21):2223-2231.

Item 21

ANSWER: E

Irritable bowel syndrome (IBS) symptoms improve with several different medications and alternative
therapies. Exercise, probiotics, antibiotics, antispasmodics, antidepressants, psychological treatments, and
peppermint oil all have evidence that they may improve IBS symptoms (SOR B). A Cochrane review of
15 studies involving 922 patients found a beneficial effect from antidepressants with regard to improvement
in pain and overall symptom scores compared to placebo. SSRIs used in these trials included citalopram,
fluoxetine, and paroxetine, and tricyclic antidepressants included amitriptyline, desipramine, and
imipramine. Buspirone, clonazepam, divalproex sodium, and risperidone have not been shown to be
effective for symptom relief in IBS patients.

Ref: Ruepert L, Quartero AO, de Wit NJ, et al: Bulking agents, antispasmodics and antidepressants for the treatment of irritable
bowel syndrome. Cochrane Database Syst Rev 2011;(8):CD003460. 2) Wilkins T, Pepitone C, Alex B, Schade RR:
Diagnosis and management of IBS in adults. Am Fam Physician 2012;86(5):419-426. 3) Ford AC, Talley NJ: Irritable
bowel syndrome. BMJ 2012;345:e5836.

Item 22

ANSWER: D

While anyone, even previously healthy individuals, may benefit from treatment of symptomatic clinical
influenza infection with antiviral agents, not everyone who has been exposed but is asymptomatic requires
chemoprophylaxis. However, persons at higher risk for complications from influenza should be considered
for preventive treatment. Those at highest risk include children under the age of 2 years, pregnant women
(including women less than 2 weeks post partum), adults over the age of 65, the morbidly obese (BMI
>40 kg/m2), and Native or Alaskan Americans. If persons at high risk for influenza complications are not
treated prophylactically with antiviral agents after exposure, then they should receive prompt treatment as
soon as possible after developing signs and symptoms of influenza infection.

Ref: Antiviral drugs for influenza 20132014. Med Lett Drugs Ther 2014;56(1434):6-8.

8
Item 23

ANSWER: E

The Infectious Diseases Society of America recommends that penicillin remain the treatment of choice for
group A streptococcal pharyngitis because of its proven efficacy, safety, narrow spectrum, and low cost.
Penicillin-resistant group A Streptococcus has never been documented. Amoxicillin is often used in place
of penicillin V as oral therapy for young children, primarily because of acceptance of the taste of the
suspension. The other options listed are all possible regimens for group A streptococcal pharyngitis but
penicillin is still considered the treatment of choice.

Ref: Shulman ST, Bisno AL, Clegg HW, et al: Clinical practice guideline for the diagnosis and management of group A
streptococcal pharyngitis: 2012 update by the Infectious Diseases Society of America. Clin Infect Dis
2012;55(10):1279-1282.

Item 24

ANSWER: C

While all of the conditions listed are in the differential diagnosis, the most likely in this patient is Rocky
Mountain spotted fever (RMSF) (SOR C). It is transmitted by ticks and occurs throughout the United
States, but is primarily found in the South Atlantic and South Central states. It is most common in the
summer and with exposure to tall vegetation from activities such as camping, hiking, or gardening. The
diagnosis is based on clinical criteria that include fever, hypotension, rash, myalgia, vomiting, and
headache (sometimes severe). The rash associated with RMSF usually appears 24 days after the onset of
fever and begins as small, pink, blanching macules on the ankles, wrists, or forearms that evolve into
maculopapules. It can occur anywhere on the body, including the palms and soles, but the face is usually
spared.

Mucocutaneous lymph node syndrome is a similar condition in children (usually <2 years old), but
symptoms include changes in the lips and oral cavity, such as strawberry tongue, redness and cracking of
the lips, and erythema of the oropharyngeal mucosa. Leptospirosis is usually accompanied by severe
cutaneous hyperesthesia. The patient with scarlet fever usually has prominent pharyngitis and a fine,
papular, erythematous rash. Toxic shock syndrome may present in a similar fashion, but usually in
postmenarchal females.

Ref: Huntzinger A: Guidelines for the diagnosis and treatment of tick-borne rickettsial diseases. Am Fam Physician
2007;76(1):137-139. 2) Longo DL, Fauci AS, Kasper DL, et al (eds): Harrisons Principles of Internal Medicine, ed 18.
McGraw-Hill, 2012, pp 1407-1410.

Item 25

ANSWER: D

Morphine should be avoided in patients with renal insufficiency because the toxic metabolites
morphine-3-glucuronide and morphine-6-glucuronide are not eliminated by the kidneys. Accumulation of
these metabolites causes neuroexcitatory effects, including confusion, sedation, respiratory depression, and
myoclonus.

9
Fentanyl and methadone are considered the safest opioids to use in patients with end-stage renal disease,
but they require careful titration, dosage adjustments as necessary, continued monitoring, and an awareness
of possible interactions with other medications that patients may be taking.

Ref: Moryl N, Coyle N, Foley KM: Managing an acute pain crisis in a patient with advanced cancer. JAMA
2008;299(12):1457-1467. 2) OConnor NR, Corcoran AM: End-stage renal disease: Symptom management and advance
care planning. Am Fam Physician 2012;85(7):705-710.

Item 26

ANSWER: A

Viruses cause 80% of infectious conjunctivitis cases and viral conjunctivitis usually requires no treatment.
Bacterial conjunctivitis is associated with mattering and adherence of the eyelids. Topical antibiotics reduce
the duration of bacterial conjunctivitis but have no effect on viral conjunctivitis. Allergic conjunctivitis
would be more likely if the patient reported itching. Antibiotics or corticosteroids would not be helpful in
this patient, and would not prevent complications.

The majority of cases of viral conjunctivitis are caused by adenoviruses, which cause pharyngeal
conjunctival fever and epidemic keratoconjunctivitis. Pharyngeal conjunctival fever is characterized by high
fever, pharyngitis, and bilateral eye inflammation. Keratoconjunctivitis occurs in epidemics, and is
associated with a watery discharge, hyperemia, and ipsilateral lymphadenopathy in >50% of cases.

Ref: Azari AA, Barney NP: Conjunctivitis: A systematic review of diagnosis and treatment. JAMA 2013;310(16):1721-1729.

Item 27

ANSWER: A

The Society of Hospital Medicine recommends that urinary catheters not be placed or left in place for
managing incontinence or for staff convenience, or for monitoring output in patients who are not critically
ill. The Infectious Diseases Society of America recommends using patient weight to monitor diuresis.
Acceptable indications for an indwelling catheter include critical illness, obstruction, hospice care, and
perioperatively for <2 days for urologic procedures.

Ref: Saint S, Meddings JA, Calfee D, et al: Catheter-associated urinary tract infection and the Medicare rule changes. Ann Intern
Med 2009;150(12):877-884. 2) Hooton TM, Bradley SF, Cardenas DD, et al: Diagnosis, prevention, and treatment of
catheter-associated urinary tract infection in adults: 2009 International Clinical Practice Guidelines from the Infectious
Diseases Society of America. Clin Infect Dis 2010;50(5):625-663. 3) Society of Hospital Medicine: Five things physicians
and patients should question. ABIM Foundation Choosing Wisely campaign, 2014.

10
Item 28

ANSWER: E

Statin drugs are effective for preventing stroke, which should be the key goal in this high-risk patient. They
may stabilize the intimal wall. Rapid lowering of blood pressure could cause brain injury by reducing blood
flow in patients with carotid stenosis. Any evidence of hypoperfusion needs to be corrected immediately.
Combination therapy with aspirin and clopidogrel is associated with an increased risk of bleeding and is
not recommended for stroke prevention. Patients over age 70 have worse outcomes with carotid stenting
than with endarterectomy. Occult patent ductus arteriosus has not been shown to be a significant risk factor
for stroke.

Ref: Grotta JC: Carotid stenosis. N Engl J Med 2013;369(12):1143-1150.

Item 29

ANSWER: D

Common features of acute adrenal insufficiency include fatigue and lack of energy, weight loss,
hypotension, loss of appetite, nausea, and vomiting. Other features such as dry skin, hyperpigmentation,
and abdominal pain are seen to varying degrees. Common laboratory findings include electrolyte
disturbances, hyponatremia, hyperkalemia, hypercalcemia, azotemia, anemia, and eosinophilia. Patients
can also have unexplained hypoglycemia. Patients with advanced-stage cancer (especially of the lung or
breast) may develop acute adrenal insufficiency from metastatic infiltration of the adrenal glands.
Intravenous hydrocortisone is the treatment of choice in the management of adrenal crisis. For managing
hypotension, dopamine is recommended for patients with sepsis, dobutamine for those in cardiogenic
shock, and packed RBCs for those with hemorrhagic shock. Broad-spectrum antibiotics are part of the
therapy for sepsis, but are not first-line agents for hypotension (SOR B).

Ref: Chou SH: Adrenal insufficiency. Hosp Med Clin 2012;1(1):e97-e108.

Item 30

ANSWER: D

Osteomyelitis is a serious complication of diabetic foot infections and is present in up to 20% of mild to
moderate infections and in 50%60% of severe infections. While a bone biopsy and/or bone cultures are
definitive for making the diagnosis, radiologic studies can also be helpful. Plain radiography may show
bony destruction but has a sensitivity for osteomyelitis ranging from 28% to 75%, depending on the timing
of the examination and the severity of the infection. It may take weeks for these infections to become
apparent on plain radiographs. The sensitivity of triple-phase technetium bone scans is up to 90% but they
have low specificity for osteomyelitis. The 90% sensitivity and 80% specificity of MRI is superior to all
other imaging modalities.

Ref: Gemechu FW, Seemant F, Curley CA: Diabetic foot infections. Am Fam Physician 2013;88(3):177-184.

11
Item 31

ANSWER: A

Initial testing with serum ferritin levels and transferrin saturation is indicated when hereditary
hemochromatosis is suspected. Normal values for these tests exclude iron-mediated organ dysfunction.
Genetic testing is indicated if the serum ferritin level is >300 ng/mL in men or >200 ng/mL in women,
or if transferrin saturation is 45%. A liver biopsy would be indicated to determine hepatic iron content
and histopathology if the ferritin level were 1000 ng/mL or liver transaminases were elevated in a patient
who is homozygous for C282Y. An "-fetoprotein level and hepatic ultrasonography would be indicated
to detect hepatocellular carcinoma if the condition has already advanced to cirrhosis.

Ref: Crownover BK, Covey CJ: Hereditary hemochromatosis. Am Fam Physician 2013;87(3):183-190.

Item 32

ANSWER: D

A full set of pulmonary function tests consists of spirometry, helium lung volume measurements, and the
measurement of diffusing capacity of the lung for carbon monoxide (DLCO). A bronchodilator challenge
will allow assessment of reversible airway obstruction. A methacholine challenge test can also be used to
look for airway hyperreactivity. A reduced FVC with either a normal or increased FEV1/FVC ratio is
consistent with restrictive lung disease. There are three basic categories of restrictive lung disease: intrinsic
lung disease, chest wall deformities, and neuromuscular disorders. A reduced FEV1 and decreased
FEV1/FVC ratio is seen in obstructive lung disease (asthma, COPD). The DLCO is the measure of the
diffusion of carbon monoxide across the alveolar-capillary membrane. Reduced values are obtained when
interstitial fibrosis is extensive, or when the capillary surface is compromised by vascular obstruction or
nonperfusion, or is destroyed (as in emphysema).

Ref: Goldman L, Schafer AI (eds): Goldmans Cecil Medicine, ed 24. Elsevier Saunders, 2011, pp 524-527.

Item 33

ANSWER: A

According to the American Academy of Pediatrics, preschool-age children with ADHD should receive
behavioral therapy alone, administered by a parent and/or teacher. Initially prescribing behavioral therapy
alone is supported by strong overall evidence and also by a study finding that many preschool-age children
with moderate to severe dysfunction had improved symptoms with behavioral therapy alone. If significant
improvement is not observed, then methylphenidate can be added. Medications combined with behavioral
therapy should be prescribed in elementary schoolage children. Evidence for the use of stimulants is
strong, and evidence for the use of atomoxetine is sufficient, but not as strong as for the stimulants.

Ref: Subcommittee on Attention-Deficit/Hyperactivity Disorder; Steering Committee on Quality Improvement and Management;
Wolraich M, Brown L, Brown RT, et al: ADHD: Clinical practice guideline for the diagnosis, evaluation, and treatment
of attention-deficit/hyperactivity disorder in children and adolescents. Pediatrics 2011;128(5):1007-1022.

12
Item 34

ANSWER: C

This patient has lichen simplex chronicus, consisting of lichenified plaques and excoriations that result
from excessive scratching. Treatment focuses on stopping the itch-scratch cycle. Topical corticosteroids
under an occlusive dressing or intralesional corticosteroids can be helpful. Scabies lesions are small,
erythematous papules that are frequently excoriated. Contact dermatitis is usually associated with direct
skin exposure to an allergen or irritant and is typically localized to the area of exposure. Chronic urticaria
causes a typical circumscribed, raised, erythematous lesion with central pallor.

Ref: Moses S: Pruritus. Am Fam Physician 2003;68(6):1135-1142. 2) Habif TP: Clinical Dermatology: A Color Guide to
Diagnosis and Therapy, ed 5. Mosby Elsevier, 2010, pp 115-118.

Item 35

ANSWER: A

Amlodipine is metabolized by the cytochrome P450 3A4 enzyme. Clarithromycin is a strong 3A4 inhibitor
that can slow the metabolism of calcium channel blockers metabolized by this enzyme, thus increasing their
levels. This can lead to hypotension, edema, and acute kidney injury due to decreased renal perfusion. It
is preferable to choose a different antibiotic regimen for patients on a dihydropyridine calcium channel
blocker such as amlodipine, but if another antibiotic cannot be used, either temporarily stopping the
calcium channel blocker or empirically lowering the dosage should be considered.

Ref: Gandhi S, Fleet JL, Bailey DG, et al: Calcium-channel blockerClarithromycin drug interactions and acute kidney injury.
JAMA 2013;310(23):2544-2553.

Item 36

ANSWER: C

Basal cell carcinoma is the most common invasive malignant cutaneous neoplasm in humans. The tumor
rarely metastasizes but it can advance by direct extension and can destroy normal tissue. Approximately
85% of all basal cell carcinomas occur on the head and neck, with 25%30% on the nose. Lesions on the
nose, eyelid, chin, jaw, and ear have higher recurrence rates than lesions in other locations. A biopsy is
necessary to make a definitive diagnosis prior to treatment. Excision is preferred for larger tumors with
well-defined borders, but wide margins are not necessary. It is very difficult to perform this surgery with
a primary closure around the nose. For lesions around the nose, especially those >1 cm, Mohs
micrographic surgery is the preferred treatment. This is a microscopically controlled technique that
facilitates removal of the entire lesion with the least amount of tissue removed.

Imiquimod is an immune response modifier that can be used on superficial basal cell carcinomas but should
not be used for a site with a high risk of recurrence. Electrodesiccation and curettage is effective for
smaller nodular basal cell carcinomas. Cryotherapy is not recommended.

Ref: Habif TP: Clinical Dermatology: A Color Guide to Diagnosis and Therapy, ed 5. Mosby Elsevier, 2010, pp 801-811. 2)
Kundu RV, Patterson S: Dermatologic conditions in skin of color: Part I. Special considerations for common skin disorders.
Am Fam Physician 2013;87(12):850-856.

13
Item 37

ANSWER: A

Lateral epicondylitis is a common condition characterized by degeneration of the extensor carpi radialis
muscle tendon originating in the lateral epicondyle. It is a self-limited condition and usually resolves within
1218 months without treatment. It is not an inflammatory condition and anti-inflammatory agents have
not been found to be beneficial. Corticosteroid injections have been found to be associated with poor
long-term outcomes, as well as high recurrence rates. Neither physical therapy, bracing, nor splinting is
proven to provide long-term pain relief. Approximately 90%95% of all patients with lateral epicondylitis
show improvement at 1 year despite the type of therapy utilized (SOR A).

Ref: Coombes BK, Bisset L, Brooks P, et al: Effect of corticosteroid injection, physiotherapy, or both on clinical outcomes in
patients with unilateral lateral epicondylalgia: A randomized controlled trial. JAMA 2013;309(5):461-469. 2) Sims SEG,
Miller K, Elfar JC, Hammert WC: Non-surgical treatment of lateral epicondylitis: A systematic review of randomized
controlled trials. Hand May 2014.

Item 38

ANSWER: D

In March of 2013 the FDA issued a safety warning regarding azithromycin and its potential to lead to
serious and even fatal arrhythmias, particularly in at-risk patients. Risk factors include hypokalemia,
hypomagnesemia, a prolonged QT interval, and the use of certain medications to treat abnormal heart
rhythms. The mechanism of action is prolongation of the QT interval, leading to torsades de pointes (level
of evidence 2, SOR A).

The FDA recommends that physicians consider the risk of torsades de pointes and fatal heart rhythms
associated with azithromycin when considering antibiotic treatment options, particularly in patients who
are already at risk for cardiovascular events.

Ref: FDA Drug Safety Communication: Azithromycin (Zithromax or Zmax) and the risk of potentially fatal heart rhythms, 2013.

Item 39

ANSWER: C

This patient continues to have an elevated hemoglobin A1c and bedtime hyperglycemia. The addition of a
rapid-acting insulin at dinner would be the next step in management. For patients exhibiting blood glucose
elevations before dinner, the addition of rapid-acting insulin at lunch is preferred. For patients with
elevations before lunch, rapid-acting insulin with breakfast would most likely improve glucose control.
Increasing or splitting the insulin glargine would be unlikely to improve management.

Ref: Nathan DM, Buse JB, Davidson MB, et al: Medical management of hyperglycemia in type 2 diabetes: A consensus
algorithm for the initiation and adjustment of therapy: A consensus statement of the American Diabetes Association and
the European Association for the Study of Diabetes. Diabetes Care 2009;32(1):193-203.

14
Item 40

ANSWER: A

This patient exhibits signs of possible sepsis, including fever, altered mental status, tachycardia, and
tachypnea. Confirmation of a documented infection would establish the diagnosis, but treatment should be
started before the infection is confirmed. Initial management includes respiratory stabilization. This patient
responded to oxygen supplementation, but if he had not, mechanical ventilation would be indicated. The
next appropriate step is fluid resuscitation. A bolus of intravenous fluids at 20 mL/kg over 30 minutes or
less is recommended (SOR A). Vasopressors should be started if a patient does not respond to intravenous
fluids as evidenced by an adequate increase in mean arterial pressure and organ perfusion (SOR B).
First-line agents include dopamine and norepinephrine. Vasopressin may be added but has not been shown
to improve mortality. Bicarbonate therapy is not usually recommended to improve hemodynamic status.
Hydrocortisone may be used in patients who do not respond to fluids and vasopressors.

Ref: Gauer RL: Early recognition and management of sepsis in adults: The first six hours. Am Fam Physician 2013;88(1):44-53.

Item 41

ANSWER: D

This case is consistent with physiologic genu valgus, and the parents should be reassured. Toddlers under
2 years of age typically have a varus angle at the knee (bowlegs). This transitions to physiologic genu
valgus, which gradually normalizes by around 6 years of age. As this condition is physiologic, therapies
such as surgical intervention, special bracing, and exercise programs are not indicated.

Ref: Kliegman RM, Stanton BF, Geme JW III, et al (eds): Nelson Textbook of Pediatrics, ed 19. Elsevier Saunders, 2011, pp
2344-2351.

Item 42

ANSWER: E

This patient has documented bradycardia on an EKG and a diagnosis of sick sinus syndrome. She has
symptomatic end-organ hypoperfusion resulting from her slow heart rate. In addition to lightheadedness
and fatigue, other manifestations can include palpitations, angina, heart failure, oliguria, TIA, or stroke.
In a symptomatic patient with documented bradycardia, permanent pacemaker placement is recommended
(SOR C).

If a patient is symptomatic but bradycardia is not evident on the EKG, prolonged monitoring is
recommended with a 48-hour Holter monitor. The next step would be longer monitoring with an event
monitor.

Evidence of sick sinus syndrome may be seen with exercise treadmill testing. Patients with chronotropic
incompetence may be unable to achieve target heart rates with exercise. However, the exercise treadmill
test is not standardized to diagnose sick sinus syndrome.

15
Patients with sick sinus syndrome may have problems resulting from anesthesia during surgery. Clearing
these patients for surgery may not be in their best interest.

Ref: Semelka M, Gera J, Usman S: Sick sinus syndrome: A review. Am Fam Physician 2013;87(10):691-696.

Item 43

ANSWER: A

Topical intranasal glucocorticoids are currently believed to be the most efficacious medications for the
treatment of allergic rhinitis. They are far superior to oral preparations in terms of safety. Cromolyn
sodium is also an effective topical agent for allergic rhinitis; however, it is more effective if started prior
to the season of peak symptoms. Because of the high risk of rhinitis medicamentosa with chronic use of
topical decongestants, these agents have limited usefulness in the treatment of allergic rhinitis.

Azelastine, an intranasal antihistamine, is effective for controlling symptoms but can cause somnolence
and a bitter taste. Oral antihistamines are not as useful for congestion as for sneezing, pruritus, and
rhinorrhea. Overall, they are not as effective as topical glucocorticoids.

Ref: Lambert M: Practice guidelines for managing allergic rhinitis. Am Fam Physician 2009;80(1):79. 2) Longo DL, Fauci AS,
Kasper DL, et al (eds): Harrisons Principles of Internal Medicine, ed 18. McGraw-Hill, 2012, pp 2716-2718.

Item 44

ANSWER: D

There is substantial evidence that small (<10 mm) hyperplastic polyps found in the rectum or sigmoid
colon are not neoplastic. Data obtained from numerous studies provides considerable evidence of moderate
quality that individuals with no significant findings other than rectal or sigmoid hyperplastic polyps of this
size should be included in the same low-risk cohort as those who have an unremarkable colonoscopy. For
patients at low risk the recommended interval between screening colonoscopies is 10 years. Reductions
in this interval are recommended for patients with one or two small tubular adenomas (510 years) or those
with three or more tubular adenomas (3 years); the interval for more extensive disease is best
individualized but can be as often as annually in unusual cases.

Ref: Lieberman DA, Rex DK, Winawer SJ, et al: Guidelines for colonoscopy surveillance after screening and polypectomy: A
consensus update by the US Multi-Society Task Force on Colorectal Cancer. Gastroenterology 2012;143(3):844857.

Item 45

ANSWER: A

Pain involving the big toe is a common problem. The first metatarsophalangeal (MTP) joint has two
sesamoid bones, and injuries to these bones account for 12% of big-toe injuries. Overuse, a sharp blow,
and sudden dorsiflexion are the most common mechanisms of injury.

Gout often involves the first MTP joint, but the onset is sudden, with warmth, redness, and swelling, and
pain on movement of the joint is common. Mortons neuroma typically causes numbness involving the
digital nerve in the area, and usually is caused by the nerve being pinched between metatarsal heads in the
center of the foot. Cellulitis of the foot is common, and can result from inoculation through a subtle crack
in the skin. However, there would be redness and swelling, and the process is usually more generalized.

16
Sesamoiditis is often hard to differentiate from a true sesamoid fracture. Radiographs should be obtained,
but at times they are nondiagnostic. Fortunately, treatment is similar for both conditions, unless the
fracture is open or widely displaced. Limiting weight bearing and flexion to control discomfort is the first
step. More complex treatments may be needed if the problem does not resolve in 46 weeks.

Ref: Browner BD, Jupiter JB, Levine AM, et al (eds): Skeletal Trauma: Basic Science, Management, and Reconstruction, ed
4. WB Saunders Co, 2009, pp 2721-2722. 2) Brukner P, Khan K: Clinical Sports Medicine, ed 4. McGraw-Hill, 2012,
pp 869-870.

Item 46

ANSWER: B

There is no currently approved treatment for Raynauds disease. However, patients with this disorder
reportedly experience subjective symptomatic improvement with dihydropyridine calcium channel
antagonists, with nifedipine being the calcium channel blocker of choice. "1-Antagonists such as prazosin
or terazosin are also effective. $-Blockers can produce arterial insufficiency of the Raynaud type, so
propranolol and atenolol are contraindicated. Drugs such as ergotamine preparations can produce cold
sensitivity, and should therefore be avoided in patients with Raynauds disease. Cilostazol is indicated for
intermittent claudication but not for Raynauds disease.

Ref: Goldman L, Schafer AI (eds): Goldmans Cecil Medicine, ed 24. Elsevier Saunders, 2011, pp 495-496. 2) Goundry B,
Bell L, Langtree M, Moorthy A: Diagnosis and management of Raynauds phenomenon. BMJ 2012;344:e289.

Item 47

ANSWER: C

The EKG reveals P waves unrelated to the QRS complex, in that the PR interval is variable and random.
With atrial fibrillation there is no P wave. Sinus bradycardia has a P wave associated with each QRS
complex and a fixed PR interval. With 2:1 AV block there would be two P waves followed by a QRS.
Mobitz-type blocks have a consistent PR interval, often with a widened QRS.

Ref: Dubin D: Rapid Interpretation of EKGs, ed 6. Cover Publishing Company, 2000, pp 181-182, 186-189. 2) Goldberger
AL, Goldberger ZD, Shvilkin A: Goldbergers Clinical Electrocardiography: A Simplified Approach, ed 8. Elsevier
Saunders, 2013, pp 159-169.

Item 48

ANSWER: D

This patient has numerous red flags for elder abuse, including unexplained weight loss, reliance on a
caregiver, a disheveled appearance, a pressure ulcer, and bruising in locations that are not typically
associated with unintentional trauma from falls. Although the other listed causes of bruising are possible,
in this scenario the index of suspicion should be highest for elder abuse.

Ref: Hoover RM, Polson M: Detecting elder abuse and neglect: Assessment and intervention. Am Fam Physician
2014;89(6):453-460.

17
Item 49

ANSWER: E

This patient most likely has acute compartment syndrome and must be urgently evaluated by an orthopedic
surgeon. Typically, compartment pressure can be measured using a needle attached to a manometer, and
if the pressure is elevated (usually >40 mm Hg) urgent fasciotomy is necessary to prevent muscle
necrosis. If the classic Five Ps (pain, paresthesia, pallor, pulselessness, and paralysis) are all present,
the outcome will most certainly be bad, even limb-threatening. Early identification with a high index of
suspicion and urgent referral for fasciotomy is necessary to prevent tragic results.

Before the classic findings develop, patients will have tenderness out of proportion to the physical
appearance of the injury and, most importantly, severe pain in the involved compartment with passive
stretching of the involved muscles.

While rest, immobilization, nonweight bearing, and analgesia are all appropriate measures, none of these
is sufficient treatment for this urgent problem.

Ref: Sarwark JF (ed): Essentials of Musculoskeletal Care, ed 4. American Academy of Orthopaedic Surgeons, 2011, pp 669-671.

Item 50

ANSWER: A

Untreated hypothyroidism during pregnancy impairs fetal development and increases the risk of
spontaneous miscarriage, prematurity, preeclampsia, gestational hypertension, and postpartum
hemorrhage. These risks are mitigated by appropriate levothyroxine treatment. Levothyroxine/L-
triiodothyroxine combinations and desiccated thyroid preparations have the potential to correct maternal
hypothyroidism, but the T4 level may still be too low to provide the transplacental delivery necessary for
optimal fetal health. The most appropriate pregnancy planning advice is to continue the current dosage of
levothyroxine with a plan for monthly monitoring of TSH and T4 during pregnancy, with the expectation
that an increase in dosage may be required as the pregnancy progresses.

Ref: Stagnaro-Green A, Abalovich M, Alexander E, et al: Guidelines of the American Thyroid Association for the diagnosis
and management of thyroid disease during pregnancy and postpartum. Thyroid 2011;21(10):1081-1125.

18
Item 51

ANSWER: C

Whooping cough has reemerged over the past few years. The initial catarrhal stage is manifested by
nonspecific symptoms similar to those of a viral upper respiratory illness. This stage is usually 12 weeks
in duration, and the patient is highly contagious. The paroxysmal stage is manifested by severe coughing
spells that occur in paroxysms and may be followed by the inspiratory whoop (much more likely in
children). Post-tussive emesis is another classic sign. There are no characteristic findings on examination
other than signs induced by extreme coughing. The CDC recommends both a nasopharyngeal culture and
polymerase chain reaction testing to confirm the diagnosis. Serologic testing is useful only in research
settings, and direct fluorescent antibody testing is not recommended. Azithromycin should be used as initial
therapy, but this is to decrease transmission of the illness and does not improve symptoms.

Ref: Kline JM, Lewis WD, Smith EA, et al: Pertussis: A reemerging infection. Am Fam Physician 2013;88(8):507-514.

Item 52

ANSWER: C

Blood pressure in children should be measured with an appropriate size cuff. Blood pressure standards are
based on age, sex, and height, and provide a precise classification of blood pressure according to body
size. Blood pressure tables for children now include the 50th, 90th, 95th, and 99th percentiles by age, sex,
and height (SOR A).

Ref: Kliegman RM, Stanton BF, Geme JW III, et al (eds): Nelson Textbook of Pediatrics, ed 19. Elsevier Saunders, 2011, p
1534.

Item 53

ANSWER: A

The U.S. Preventive Services Task Force recommends that all men 35 years of age and older be screened
for dyslipidemia (evidence rating A). Men who are 2035 years of age should be screened only if they
have cardiovascular risk factors (evidence rating B). Prostate-specific antigen (PSA) testing should not be
performed at any age because the harms outweigh the benefits (evidence rating D). Colorectal cancer
screening should begin at age 50 for men of average risk (evidence rating A). Abdominal aortic aneurysm
(AAA) screening once by ultrasonography is recommended for men age 6575 with a family history of
AAA, and for those who have smoked at least 100 cigarettes in their lifetime (evidence rating B).
Testicular examinations, either by self-examination or clinical screening, should not be performed routinely
(evidence rating D).

Ref: Heidelbaugh JJ, Tortorello M: The adult well male examination. Am Fam Physician 2012;85(10):964-971.

19
Item 54

ANSWER: B

Unlike the other conditions listed, Crohns disease is associated with many extragastrointestinal conditions:
erythema nodosum (as with this patient), anemia, inflammatory arthropathies, uveitis, and venous
thromboembolism (level of evidence 3).

Ref: Wilkins T, Jarvis K, Patel J: Diagnosis and management of Crohns disease. Am Fam Physician 2011;84(12):1365-1375.

Item 55

ANSWER: E

While hormonal therapy was initially used to treat postmenopausal vasomotor symptoms, it was also
believed to prevent disease. Based on retrospective studies, a decrease in ischemic heart disease and
dementia was suspected. The Womens Health Initiative trials, first undertaken in the 1990s, showed that
this was unfortunately not the case, and that hormone therapy actually increases the risk for coronary heart
disease, stroke, breast cancer, gallbladder disease, dementia, and venous thrombosis, particularly in older
women. A decrease in fractures was demonstrated, however, along with some other health benefits, such
as a reduced risk of endometrial cancer.

Ref: Manson JE, Chlebowski RT, Stefanick ML, et al: Menopausal hormone therapy and health outcomes during the intervention
and extended poststopping phases of the Womens Health Initiative randomized trials. JAMA 2013;310(13):1353-1368.

Item 56

ANSWER: E

When there is a question about the cause of goiter and thyrotoxicosis, the presence of TSH receptor
immunoglobulins indicates Graves disease. The prevalence of specific forms of TSH receptor site
antibodies can distinguish Graves disease from Hashimotos disease. Both are autoimmune diseases, but
in Graves disease there is a predominance of TSH receptor antibodies. In Hashimotos disease TSH
receptorblocking antibodies are more predominant. These immunoglobulins tend to disappear with
therapy.

Ref: Goldman L, Schafer AI (eds): Goldmans Cecil Medicine, ed 24. Elsevier Saunders, 2011, pp e68-e70.

Item 57

ANSWER: B

The majority of symptoms associated with sports-related concussions resolve within 72 hours of injury.
However, some concussions result in prolonged recovery periods. Risk factors associated with a prolonged
recovery include headaches lasting 60 hours or more, self-reported fatigue or fogginess, and four or more
symptoms at the onset of injury (SOR B). Loss of consciousness and amnesia have not been found to be
related to recovery time. Convulsions associated with the injury are benign and do not affect prognosis.
Nausea is one of the symptoms of concussion, but by itself is not a risk factor for prolonged recovery.

Ref: Makdissi M, Darby D, Maruff P, et al: Natural history of concussion in sport: Markers of severity and implications for
management. Am J Sports Med 2010;38(3):464-471. 2) Scorza KA, Raleigh MF, OConnor FG: Current concepts in
concussion: Evaluation and management. Am Fam Physician 2012;85(2):123-132.

20
Item 58

ANSWER: C

In patients at intermediate to high risk for deep vein thrombosis, compression ultrasonography is the initial
diagnostic test of choice. In low-risk patients, a negative D-dimer has a high enough negative predictive
value to essentially rule out deep vein thrombosis. It is not sensitive or specific enough for evaluating
intermediate-risk patients. Initiating treatment based on pretest probability would be inappropriate.
Impedance plethysmography is not readily available, and contrast venography is too invasive to be used
routinely.

Ref: Wilbur J, Shian B: Diagnosis of deep venous thrombosis and pulmonary embolism. Am Fam Physician
2012;86(10):913-919.

Item 59

ANSWER: C

The use of medications for osteoporosis is associated with various side effects, some of which have only
recently been recognized. Denosumab and bisphosphonates have similar, albeit low, risks for jaw
osteonecrosis. Bisphosphonates should not be used in patients with a creatinine clearance <35
mL/min/1.73 m2, but denosumab is not cleared by the kidneys and is safe in patients with chronic kidney
disease. The use of bisphosphonates for more than 5 years can increase the risk of atypical fractures and
a holiday from the drug is recommended after either 3 or 5 years, depending on the drug used.

Ref: Messinger-Rapport BJ, Gammack JK, Thomas DR, Morley JE: Clinical update on nursing home medicine: 2013. J Am
Med Dir Assoc 2013;14(12):860-876.

Item 60

ANSWER: B

The most common causes of serious bacterial infection in children 336 months of age are pneumonia and
urinary tract infection. In children without an obvious source of infection, the urinalysis and culture are
key tests in the evaluation. A valid urine sample should be obtained in all children under the age of 2 with
a fever of unknown source. The sample should be obtained through catheterization or suprapubic
aspiration. If the patient is toilet trained a clean-catch urine sample is acceptable (SOR C). C-reactive
protein is currently under investigation for its utility in detecting serious infection in young children. It is
thought to have a greater predictive value than WBC counts but is not yet standardized for common use.
A CBC with differential is most useful in neonates but is not as helpful in older infants for detecting serious
infection. It is recommended for hospitalized patients but not for those managed as outpatients (SOR C).

A chest radiograph is indicated for children with an abnormal respiratory examination or respiratory
symptoms. It is also recommended for children older than 1 month of age with a fever >39C (102F)
and a WBC count >20,000/mm3. A lumbar puncture is indicated for infants with meningeal signs such
as focal neurologic findings, petechiae, or nuchal rigidity.

Ref: Hamilton JL, John SP: Evaluation of fever in infants and young children. Am Fam Physician 2013;87(4):254-260.

21
Item 61

ANSWER: E

Hypogonadism is an often underrecognized and undertreated side effect of long-term opioid therapy. It is
more often seen in men and in patients receiving larger doses of opioids, including intrathecally. Typical
symptoms include decreased libido, erectile dysfunction, amenorrhea, or fatigue.

Constipation is not uncommon in patients on chronic opioid therapy, especially if they are elderly, have
limited mobility, or are concurrently using other constipating medications. Sedation can occur in the first
few weeks after starting therapy but usually tapers off. Hyperalgesia (not hypoalgesia) and allodynia are
other side effects resulting from chronic opioid therapy. Respiratory depression is infrequent (SOR C).

Ref: Reddy RG, Aung T, Karavitaki N, Wass JA: Opioid induced hypogonadism. BMJ 2010;341:c4462.

Item 62

ANSWER: A

The American Society of Echocardiography recommends that physicians NOT order follow-up or serial
echocardiograms for surveillance after a finding of trace valvular regurgitation on an initial
echocardiogram (SOR C). Trace mitral, tricuspid, and pulmonic regurgitation can be detected in 70%90%
of normal individuals and has no adverse clinical implications. The clinical significance of a small amount
of aortic regurgitation with an otherwise normal echocardiographic study is unknown.

Ref: Bonow RO, Carabello BA, Chatterjee K, et al: 2008 focused update incorporated into the ACC/AHA 2006 guidelines for
the management of patients with valvular heart disease: A report of the American College of Cardiology/American Heart
Association Task Force on Practice Guidelines (Writing Committee to revise the 1998 guidelines for the management of
patients with valvular heart disease). J Am Coll Cardiol 2008;52(13):e1-e142. 2) American College of Cardiology
Foundation Appropriate Use Criteria Task Force; American Society of Echocardiography; American Heart Association,
et al: ACCF/ASE/AHA/ASNC/HFSA/HRS/SCAI/SCCM/SCCT/SCMR 2011 Appropriate Use Criteria for
Echocardiography. A report of the American College of Cardiology Foundation Appropriate Use Criteria Task Force,
American Society of Echocardiography, American Heart Association, American Society of Nuclear Cardiology, Heart
Failure Society of America, Heart Rhythm Society, Society for Cardiovascular Angiography and Interventions, Society of
Critical Care Medicine, Society of Cardiovascular Computed Tomography, Society for Cardiovascular Magnetic Resonance
American College of Chest Physicians. J Am Soc Echocardiogr 2011;24(3):229-267. 3) American Society of
Echocardiography: Five things physicians and patients should question. ABIM Foundation Choosing Wisely campaign,
2014.

Item 63

ANSWER: E

This patients history, along with the bulge/impulse detected on physical examination when he strained or
coughed, is most consistent with the diagnosis of inguinal hernia. A sports hernia is not a true hernia,
but rather a tearing of tissue fibers. The patient often presents with symptoms consistent with a hernia, but
without evidence on physical examination. Pain along the symphysis pubis would suggest osteitis pubis,
and pain along the adductor tendons would suggest adductor tendinopathy. Ilioinguinal nerve entrapment
syndrome is an abdominal muscular pain syndrome characterized by the clinical triad of muscle-type iliac
fossa pain with a characteristic radiation pattern, altered sensory perception in the ilioinguinal nerve
cutaneous innervation area, and a well-circumscribed trigger point medial to and below the anterosuperior
iliac spine.

22
Ref: LeBlanc KE, LeBlanc LL, LeBlanc KA: Inguinal hernias: Diagnosis and management. Am Fam Physician
2013;87(12):844-848.

Item 64

ANSWER: B

Running injuries are primarily caused by overuse due to training errors. Runners should be instructed to
increase their mileage gradually. A stress fracture causes localized tenderness and swelling in superficial
bones, and the pain can be reproduced by having the patient jump on the affected leg. Plantar fasciitis
causes burning pain in the heel and there is tenderness of the plantar fascia where it inserts onto the medial
tubercle of the calcaneus.

Ref: Hatch RL, Alsobrook JA, Clugston JR: Diagnosis and management of metatarsal fractures. Am Fam Physician
2007;76(6):817-826. 2) Miller MD, Thompson SR (eds): DeLee & Drez's Orthopaedic Sports Medicine: Principles and
Practice, ed 4. Elsevier Saunders, 2014, pp 160-165.

Item 65

ANSWER: D

This patient presents with a typical case of mild to moderate croup. This is a viral infection that results in
swelling in the larynx. It rarely is severe enough to cause respiratory collapse or require intubation and
must be differentiated from more severe conditions such as epiglottitis, retropharyngeal abscess, or
pneumonia. There is no reason to treat this viral infection with an antibiotic. The condition is usually
benign and self-limiting, with the worst symptoms occurring at night. Cool and/or humidified air has
traditionally been recommended, but studies have not confirmed any significant benefit from these
interventions. Since this child is not in respiratory distress and oxygenation is normal, supplemental oxygen
therapy is not indicated. Studies have confirmed the benefits of treating croup with a single dose of either
an oral or intramuscular corticosteroid. Specifically, dexamethasone is recommended due to its 72-hour
length of effect. Inhaled racemic epinephrine has been shown to reduce the need for intubation in cases
of moderate to severe croup. Albuterol, however, is not indicated.

Ref: Zoorob R, Sidani M, Murray J: Croup: An overview. Am Fam Physician 2011;83(9):1067-1073.

Item 66

ANSWER: C

The Rome criteria define constipation as the presence of two or more of the following: straining on
defecation, hard stools, incomplete evacuation, or less than three bowel movements per week. This patient
has multiple symptoms on this list. The presence of watery bowel movements does not rule out the
diagnosis of constipation, as it is common for liquid stool to pass an obstructive source.

Ref: Tintinalli JE, Kelen GD, Stapczynski JS (eds): Emergency Medicine: A Comprehensive Study Guide, ed 7. McGraw-Hill,
2011, pp 541-543.

23
Item 67

ANSWER: E

In patients with acute respiratory distress syndrome (ARDS), starting mechanical ventilation with lower
tidal volumes of 6 mL/kg is superior to starting with traditional tidal volumes of 1014 mL/kg (SOR A).
Conservative fluid therapy is recommended in patients with ARDS, as this is associated with a decrease
in the number of days on the ventilator and in the intensive-care unit (SOR B). Pulmonary artery catheters
are not recommended for routine management of ARDS (SOR A). Surfactant therapy does not improve
mortality in adults with ARDS (SOR A), and antibiotics are not an effective treatment.

Ref: Saguil A, Fargo M: Acute respiratory distress syndrome: Diagnosis and management. Am Fam Physician
2012;85(4):352-358.

Item 68

ANSWER: C

According to The Joint Commissions sentinel event program, communication problems are the most
common root cause of serious medical errors. Strategies such as the implementation of structured and
standardized handoffs are an example of improving communication among members of the health care
team, thereby helping to reduce errors.

Ref: Frequency of communication problems among Joint Commission sentinel events. The Joint Commission, 2011.

Item 69

ANSWER: B

In a corneal light reflex test, the patients attention is attracted to a target while a light is directed at the
eyes. In normally aligned eyes the light reflex will be located in the center of each pupil. In patients with
esotropia the reflex will be over the lateral portion of the iris in the affected eye. In exotropia the light
reflex is over the medial iris, in hypertropia it is over the inferior iris, and in hypotropia it is over the
superior iris. The finding observed in this child, hypertropia, will occur with a congenital palsy involving
the superior oblique muscle, which is innervated by the fourth cranial nerve.

Ref: Bell AL, Rodes ME, Collier Kellar L: Childhood eye examination. Am Fam Physician 2013;88(4):241-248. 2) Yanoff M,
Duker JS (eds): Ophthalmology, ed 4. Elsevier Saunders, 2014, pp 1228-1229.

Item 70

ANSWER: B

This patient likely has severe preeclampsia based on her elevated blood pressure with 4+ protein on her
urinalysis. Patients with severe preeclampsia near term should be placed on magnesium sulfate to prevent
seizures, and labor should be induced immediately. An urgent cesarean section is not necessary.
Corticosteroids have not been shown to improve neonatal outcomes when given after 34 weeks gestation.
Elevated blood pressures can be managed with hydralazine and labetalol. Normalizing blood pressure is
not recommended, but these drugs should be used when blood pressure is over 160/105 mm Hg.

Ref: American College of Obstetricians and Gynecologists: Diagnosis and Management of Preeclampsia and Eclampsia. ACOG
Practice Bulletin no 33, 2002 (reaffirmed 2012).

24
Item 71

ANSWER: E

Syncope with exercise is a manifestation of organic heart disease in which cardiac output is fixed and does
not rise (or even fall) with exertion. Syncope, commonly occurring with exertion, is reported in up to 42%
of patients with severe aortic stenosis. Vasovagal syncope is associated with unpleasant stimuli or
physiologic conditions, including sights, sounds, smells, sudden pain, sustained upright posture, heat,
hunger, and acute blood loss. Transient ischemic attacks are not related to exertion. Orthostatic
hypotension is associated with changing from a sitting or lying position to an upright position. Atrial
myxoma is associated with syncope related to changes in position, such as bending, lying down from a
seated position, or turning over in bed.

Ref: Task Force for the Diagnosis and Management of Syncope; European Society of Cardiology (ESC); European Heart Rhythm
Association (EHRA); Heart Failure Association (HFA); Heart Rhythm Society (HRS); Moya A, Sutton R, Ammirati F,
et al: Guidelines for the diagnosis and management of syncope (version 2009). Eur Heart J 2009;30(21):2631-2371. 2)
Goldman L, Schafer AI (eds): Goldmans Cecil Medicine, ed 24. Elsevier Saunders, 2011, pp 337-339.

Item 72

ANSWER: C

This patient has red flag findings of older age and weight loss with chronic vomiting and is at risk for a
gastrointestinal malignancy. He should be referred for esophagogastroduodenoscopy (EGD). Abdominal
ultrasonography or radiographs would not be necessary at this time. A proton pump inhibitor or H2-blocker
can be prescribed but should not delay referral for EGD.

Ref: Anderson WD 3rd, Strayer SM: Evaluation of nausea and vomiting in adults: A case-based approach. Am Fam Physician
2013;88(6):371-379.

Item 73

ANSWER: D

Although high-dose amoxicillin (90 mg/kg/day) is recommended as the antibiotic of choice for acute otitis
media (AOM) in the nonallergic patient, amoxicillin/clavulanate is recommended if a child has received
antibiotic therapy in the previous 30 days. Prophylactic antibiotics are not recommended, as harms
outweigh benefits. Tympanostomy tubes are an option if a child has had three episodes of AOM in the past
6 months or four episodes in the past year with at least one episode in the past 6 months.

Ref: Lieberthal AS, Carroll AE, Chonmaitree T, et al: The diagnosis and management of acute otitis media. Pediatrics
2013;131(3):e964-e999.

25
Item 74

ANSWER: A

In areas where there is at least a 20% rate of tick infection, 200 mg of doxycycline within 72 hours of tick
removal is appropriate as prophylaxis for Lyme disease if the tick is engorged or is suspected to have been
attached for at least 36 hours. The Northeast and Upper Midwest are the most endemic areas. The other
oral antibiotics listed are appropriate for cases of acute erythema migrans and suspected infection.
Watchful waiting is not recommended, and there is no evidence for treatment with topical antibiotics alone
(SOR C).

Ref: Wright WF, Riedel DJ, Talwani R, Gilliam BL: Diagnosis and management of Lyme disease. Am Fam Physician
2012;85(11):1086-1093.

Item 75

ANSWER: D

Subacute granulomatous thyroiditis is the most common cause of thyroid pain. Free T4 is elevated early
in the disease, as it is in Graves disease; however, later in the disease T4 becomes depressed and then
returns to normal as the disease resolves. Pretibial myxedema, exophthalmos, and a thyroid thrill or bruit
can all be found in Graves disease, but are not associated with subacute granulomatous thyroiditis. Multiple
nodules on ultrasonography suggests multinodular goiter rather than subacute granulomatous thyroiditis.
Patients with subacute granulomatous thyroiditis will have a low radioactive iodine uptake (RAIU) at 24
hours, but patients with Graves disease will have an elevated RAIU (SOR C).

Ref: Bindra A, Braunstein GD: Thyroiditis. Am Fam Physician 2006;73(10):1769-1776. 2) Melmed S, Polonsky KS, Larsen
PR, Kronenberg HM (eds): Williams Textbook of Endocrinology, ed 12. Elsevier Saunders, 2011, pp 397-398.

Item 76

ANSWER: A

Given the information about this patient, such as her relatively recent diagnosis, her age, and her lack of
macro- or microvascular complications, a more strict hemoglobin A1c goal is indicated. There are several
oral and injectable medicines that are reasonable choices in this case. Exenatide is an injectable GLP-1
agonist that is associated with weight loss. Pioglitazone is also effective but is associated with fluid
retention rather than weight loss. Sitagliptin is a dipeptidyl peptidase IV (DPP-IV) inhibitor that may be
a reasonable option in this case, but is not associated with weight loss. Insulin, either basal only, mixed,
or basal-bolus regimens, may also be the best option for the patient described, but it does cause weight
gain. Cost is another major consideration in treatment decisions, but more information would be needed
to address this issue.

Ref: Ismail-Beigi F: Glycemic management of type 2 diabetes mellitus. N Engl J Med 2012;366(14):1319-1327.

26
Item 77

ANSWER: E

Many types of exercise programs are beneficial for older adults, including simply walking for 30 minutes
three times a week. However, a meta-analysis of progressive resistance training programs in nursing
homes showed that there were significant improvements in muscle strength, chair-to-stand time, stair
climbing, gait speed, and balance. This is seen even in those with advanced age, disabilities, chronic
diseases, or extremely sedentary lifestyles.

Ref: Valenzuela T: Efficacy of progressive resistance training interventions in older adults in nursing homes: A systematic
review. J Am Med Dir Assoc 2012;13(5):418-428. 2) Messinger-Rapport BJ, Gammack JK, Thomas DR, Morley JE:
Clinical update on nursing home medicine: 2013. J Am Med Dir Assoc 2013;14(12):860-876.

Item 78

ANSWER: E

Acute diverticulitis can be treated using oral antibiotics on an outpatient basis in 90% of cases. In fact,
there is good evidence that those with uncomplicated diverticulitis (no signs of abscess, fistula, phlegmon,
obstruction, bleeding, or perforation) can be treated without the use of antibiotics, using only bowel rest
and close follow-up. Among patients who require hospitalization, it is estimated that <10% of cases will
require surgical intervention. Thus, the majority of patients hospitalized with this condition, even those
with complicated diverticulitis, will respond well to bowel rest and intravenous antibiotics.

Indications for surgery include generalized peritonitis, unconfined perforation, uncontrolled sepsis, an
undrainable abscess, and failure of conservative management. CT-guided percutaneous drainage of an
accessible abscess is a well-proven treatment to avoid the use of open surgery. Prevention of future
episodes of diverticulitis increasingly revolves around the use of daily oral medications. Some experts
recommend considering surgery to remove a section of bowel after a patients third admission for
diverticulitis.

Ref: Jacobs DO: Diverticulitis. N Engl J Med 2007;357(20):2057-2066. 2) Wilkins T, Embry K, George R: Diagnosis and
management of acute diverticulitis. Am Fam Physician 2013;87(9):612-620.

Item 79

ANSWER: D

SSRIs are the most commonly used medications for postpartum depression. They have fewer side effects
and are considered safer than tricyclic antidepressants, especially in depressed women who may be at
increased risk for medication overdose (SOR C). In one study, infant serum levels of sertraline and
paroxetine were undetectable. It is also recommended that a woman with postpartum depression be started
on a medication that she had taken previously with a good response, unless there is evidence of potential
harm to her infant (SOR C).

Tricyclic antidepressants are excreted into breast milk and there is some concern regarding potential
toxicity to the newborn. Phenytoin, diazepam, and zolpidem are not antidepressants. Phenytoin and
diazepam are Category D for use in pregnant women. Diazepam is potentially toxic to infants and can
accumulate in breastfed infants, and it is not recommended for lactating women (SOR C). Zolpidem is
category B in pregnancy and probably acceptable for use in lactating women if clinically indicated.

27
Ref: Weissman AM, Levy BT, Hartz AJ, et al: Pooled analysis of antidepressant levels in lactating mothers, breast milk, and
nursing infants. Am J Psychiatry 2004;161(6):1066-1078. 2) ACOG Committee on Practice BulletinsObstetrics: ACOG
Practice Bulletin: Clinical management guidelines for obstetrician-gynecologists number 92, April 2008 (replaces practice
bulletin number 87, November 2007). Use of psychiatric medications during pregnancy and lactation. Obstet Gynecol
2008;111(4):1001-1020. 3) Dennis CL, Allen K: Interventions (other than pharmacological, psychosocial or psychological)
for treating antenatal depression. Cochrane Database Syst Rev 2008;(4):CD006795. 4) Hirst KP, Moutier CY: Postpartum
major depression. Am Fam Physician 2010;82(8):926-933.

Item 80

ANSWER: A

Topical corticosteroids are the first-line treatment for atopic dermatitis flare-ups. Topical calcineuron
inhibitors such as pimecrolimus are a second-line therapy, but carry a warning of a possible link to
lymphomas and skin malignancies and are not recommended for children under 2 years of age. Oral
antihistamines are not effective for the pruritus associated with atopic dermatitis. Probiotic use is not
supported by available evidence.

Ref: Berke R, Singh A, Guralnick M: Atopic dermatitis: An overview. Am Fam Physician 2012;86(1):35-42.

Item 81

ANSWER: B

Lichen planus is an idiopathic inflammatory disease affecting the skin and oral mucosa. The characteristic
violaceous, polygonal papules may be intensely itchy. There is a significant association between lichen
planus and hepatitis C virus infection.

Ref: Le Cleach L, Chosidow O: Lichen planus. N Engl J Med 2012;366(8):723-732.

Item 82

ANSWER: C

The American Geriatrics Society consensus report for considering treatment goals for glycemia
recommends a hemoglobin A1c goal of <8.5% for individuals with very complex health problems or poor
health. This includes individuals in long-term care and those with end-stage chronic illnesses, moderate
to severe cognitive impairment, or more than two activity of daily living (ADL) dependencies. A
hemoglobin A1c <8.5% equates to an estimated average glucose level of approximately 200 mg/dL.
Looser glycemic targets than this may expose patients to acute risks from glycosuria, dehydration,
hyperglycemic hyperosmolar syndrome, and poor wound healing.

Ref: Kirkman MS, Briscoe VJ, Clark N, et al: Diabetes in older adults: A consensus report. J Am Geriatr Soc
2012;60(12):2342-2356.

28
Item 83

ANSWER: C

The likelihood ratio (LR) is the ratio of the probability of a specific test result in people who have a
particular disease to the probability in people who do not. LRs correspond to the clinical impression of how
well a test rules in or rules out a given disease. A test with an LR of 1.0 indicates that it does not change
the probability of disease. The higher above 1 the LR is, the more likely it is that the disease is present (an
LR >10 is considered good). Conversely, the lower the LR is below 1, the more likely it is that the
disease is not present (an LR <0.1 is considered good). Likelihood ratios are alternative statistics for
summarizing diagnostic accuracy, and have several particularly powerful properties that make them more
useful clinically than other statistics.

Ref: Deeks JJ, Altman DG: Diagnostic tests 4: Likelihood ratios. BMJ 2004;329(7458):168169. 2) EBM glossary: Terms used in
evidence-based medicine. Am Fam Physician website. http://www.aafp.org/journals/afp/authors/ebm-toolkit/glossary.html

Item 84

ANSWER: B

The most common inhibitor discovered during the evaluation of an elevated aPTT is an antiphospholipid
antibody. Antiphospholipid antibody syndrome is characterized by venous or arterial thromboembolism
and recurrent spontaneous abortion, often in the second trimester, due to placental infarction. Protein C
deficiency is associated with recurrent deep vein thrombophlebitis, but does not cause elevation of aPTT.
Hemophilia A is associated with an elevated aPTT which corrects with the addition of normal plasma. In
chronic liver disease one would expect an elevation of the prothrombin time also. Von Willebrand disease
is not associated with thrombophlebitis or recurrent abortion.

Ref: Miyakis S, Lockshin MD, Atsumi T, et al: International Consensus Statement on an update of the classification criteria for
definite antiphospholipid syndrome (APS). J Thromb Haemost 2006;4(2):295-306. 2) Pengo V, Ruffatti A, Iliceto S: The
diagnosis of the antiphospholipid syndrome. Pathophysiol Haemost Thromb 2006;35(1-2):175-180. 3) Longo DL, Fauci
AS, Kasper DL, et al (eds): Harrisons Principles of Internal Medicine, ed 18. McGraw-Hill, 2012, pp 2736-2737.

Item 85

ANSWER: D

The U.S. Preventive Services Task Force recommends that asymptomatic adults with sustained blood
pressure (treated or untreated) >135/80 mm Hg be screened for type 2 diabetes mellitus. Since this patient
was screened 1 year ago for cervical cancer, and has no history of an abnormality, she does not require
cytology again for another 2 years. If her Papanicolaou test 1 year ago had been combined with HPV
testing the rescreening interval could be extended to 5 years if both were negative. Routine screening for
ovarian cancer with bimanual examination, transvaginal ultrasonography, or CA-125 testing is not
recommended. Chlamydia screening is recommended for high-risk sexually active women over 25 and for
all sexually active women age younger than 25. Colorectal cancer screening should begin at age 50.

Ref: Riley M, Dobson M, Jones E, Kirst N: Health maintenance in women. Am Fam Physician 2013;87(1):30-37.

29
Item 86

ANSWER: A

Developmental dysplasia of the hip encompasses both subluxation and dislocation of the newborn hip, as
well as anatomic abnormalities. It is more common in firstborns, females, breech presentations,
oligohydramnios, and patients with a family history of developmental dysplasia.

Experts are divided as to whether hip subluxation can be merely observed during the newborn period, but
if there is any question of a hip problem on examination by 2 weeks of age, the recommendation is to refer
to a specialist for further testing and treatment. Studies show that these problems disappear by 1 week of
age in 60% of cases, and by 2 months of age in 90% of cases. Triple diapering should not be used because
it puts the hip joint in the wrong position and may aggravate the problem. Plain radiographs may be helpful
after 46 months of age, but prior to that time the ossification centers are too immature to be seen.

Because the condition can be difficult to diagnose, and can result in significant problems, the current
recommendation is to treat all children with developmental dysplasia of the hip. Closed reduction and
immobilization in a Pavlik harness, with ultrasonography of the hip to ensure proper positioning, is the
treatment of choice until 6 months of age. The American Academy of Pediatrics recommends ultrasound
screening at 6 weeks for breech females, breech males (optional), and females with a positive family
history of developmental dysplasia of the hip. Other countries have recommended universal screening, but
a review of the literature has not shown that the benefits of early diagnosis through universal screening
outweigh the risks and potential problems of overtreating.

Ref: Storer SK, Skaggs DL: Developmental dysplasia of the hip. Am Fam Physician 2006;74(8):1310-1316. 2) Kliegman RM,
Stanton BF, Geme JW III, et al (eds): Nelson Textbook of Pediatrics, ed 19. Elsevier Saunders, 2011, pp 2356-2360.

Item 87

ANSWER: B

Fentanyl and methadone are the preferred opioids for use in patients with end-stage renal disease (SOR
C). Fentanyl is metabolized in the liver and has no active metabolites. All of the other listed opioid
medications have active metabolites that can accumulate in patients with renal failure, leading to serious
side effects. These agents should be avoided in patients on dialysis.

Ref: OConnor NR, Corcoran AM: End-stage renal disease: Symptom management and advance care planning. Am Fam
Physician 2012;85(7):705-710.

Item 88

ANSWER: B

Clostridium difficile infection (CDI) may be transmitted by direct contact with an infected patient, by
contact with a contaminated environment, or by contact with a health care worker with transient hand
colonization. Effective prevention efforts are essential to limit the spread from one patient to another in
the hospital and other health care settings. Although alcohol-based hand antiseptics have been shown to
increase compliance with hand hygiene and reduce the incidence of MRSA and VRE infections, alcohol
does not kill the spore form of C. difficile and the use of these antiseptics does not reduce the incidence
of CDI. There is insufficient data to support the widespread use of probiotics for prevention of CDI, and
there is a potential risk of bloodstream infection with their use.

30
Health care workers rarely become colonized with C. difficile, and screening them has not been shown
to affect nosocomial transmission rates. Handwashing with soap and water removes C. difficile from the
hands of health care workers and remains the cornerstone of prevention efforts. Additional contact
precautions such as the use of gloves and gowns may also be helpful. CDI is not transmitted by the
respiratory route, so the use of respiratory isolation techniques is not helpful.

Ref: Cohen SH, Gerding DN, Johnson S, et al: Clinical practice guidelines for Clostridium difficile infection in adults: 2010
update by the Society for Healthcare Epidemiology of America (SHEA) and the Infectious Diseases Society of America
(IDSA). Infect Control Hosp Epidemiol 2010;31(5):431-455. 2) Winslow BT, Onysko M, Thompson KA, et al: Common
questions about Clostridium difficile infection. Am Fam Physician 2014;89(6):437-442.

Item 89

ANSWER: E

Norepinephrine is considered to be the vasopressor of choice in this situation. It has mainly "-adrenergic
effects with some $-adrenergic effects as well. While "-adrenergic agents increase mean arterial pressure,
they decrease cardiac output. Norepinephrines $-adrenergic properties counteract the decrease in cardiac
output, so there is an increase in blood pressure with little change in pulse rate or cardiac output.

Vasopressin may be useful in patients with cardiac arrest and ventricular fibrillation and may further
improve hypotension when used with norepinephrine. Dobutamine is the inotropic agent of choice but has
little effect on blood pressure. Epinephrine can increase the incidence of arrhythmias when compared with
norepinephrine. Dopamine also increases arrhythmias when compared with norepinephrine and is
associated with an increased 28-day death rate.

Ref: Vincent JL, De Backer D: Circulatory shock. N Engl J Med 2013;369(18):1726-1734.

Item 90

ANSWER: A

A diffusion capacity test assesses how well a tracer gas in inspired air can cross from the air into the blood.
The diffusion capacity provides a general assessment of the air-blood interface. Reduced values are seen
with severe interstitial fibrosis, or when the capillary surface has been compromised by vascular
obstruction (pulmonary embolism) or is destroyed by emphysema. Chronic pulmonary embolism causes
a low diffusing capacity of the lung for carbon monoxide (DLCO) with normal pulmonary function tests.
Emphysema causes a low DLCO and an obstructive pattern on pulmonary function testing (PFT).
Interstitial lung disease and hypersensitivity pneumonitis both cause a low DLCO with a restrictive pattern
of PFTs. Patients with asthma may have an increased DLCO with an obstructive pattern, with reversibility
after bronchodilator administration.

Ref: Nilsson KR Jr, Piccini JP: The Osler Medical Handbook, ed 2. Saunders Elsevier, 2006, pp 858-864. 2) Goldman L,
Schafer AI (eds): Goldmans Cecil Medicine, ed 24. Elsevier Saunders, 2011, pp 524-527.

31
Item 91

ANSWER: D

In 2010, opioid analgesics were implicated in 75% of pharmaceutical-related overdose deaths.


Benzodiazepines were involved in 30% of these opioid analgesicrelated deaths. Conversely, opioids were
a factor in 77% of pharmaceutical overdose deaths that involved benzodiazepines.

Antidepressants are involved in less than half as many opioid deaths as benzodiazepines. Antipsychotics,
acetaminophen, and muscle relaxants are implicated in opioid overdose deaths with far less frequency than
benzodiazepines.

Ref: Jones CM, Mack KA, Paulozzi LJ: Pharmaceutical overdose deaths, United States, 2010. JAMA 2013;309(7):657-659.

Item 92

ANSWER: C

Muscle strength is scored on a scale of 0 to 5. The inability to contract a muscle is scored as 0. Contraction
without movement constitutes grade 1 strength. Movement with the effect of gravity neutralized is grade
2 strength, while movement against gravity only is grade 3 strength. Movement against gravity plus some
additional resistance indicates grade 4 strength. Normal, or grade 5, strength is demonstrated by movement
against substantial resistance.

Ref: Ropper AH, Samuels MA, Klein JP: Adams and Victors Principles of Neurology, ed 10. McGraw-Hill, 2014, p 1410.

Item 93

ANSWER: C

Oral antibiotics are effective for the treatment of moderate to severe acne (SOR A). Combined treatment
with benzoyl peroxide is recommended to reduce the risk of bacterial resistance (SOR C). Amoxicillin and
ciprofloxacin are not recommended for acne treatment. Intralesional corticosteroid therapies have been
tried for acne treatment, but long-term use of oral corticosteroids is not recommended.

Ref: Strauss JS, Krowchuk DP, Leyden JJ, et al: Guidelines of care for acne vulgaris management. J Am Acad Dermatol
2007;56(4):651-663. 2) Thiboutot D, Gollnick H, Bettoli V, et al: New insights into the management of acne: An update
from the Global Alliance to Improve Outcomes in Acne group. J Am Acad Dermatol 2009;60(5 Suppl):S1-S50. 3) Titus
S, Hodge J: Diagnosis and treatment of acne. Am Fam Physician 2012;86(8):734-740.

Item 94

ANSWER: C

The U.S. Preventive Services Task Force recommends annual screening for lung cancer with low-dose
computed tomography (LDCT) in adults 5580 years of age who have a 30-pack-year smoking history and
currently smoke or have quit within the past 15 years. Screening should be discontinued once a person has
not smoked for 15 years or develops a health problem that substantially limits life expectancy or the ability
or willingness to have lung surgery (B recommendation).

32
The risk of lung cancer increases with age and cumulative exposure to tobacco smoke and decreases with
time since quitting smoking. The best evidence comes from the National Lung Screening Trial, which
enrolled adults age 5574 who had at least a 30-pack-year smoking history and were current smokers or
had quit in the past 15 years. Screening with LDCT resulted in a 20% reduction in lung-cancer mortality
among participants. Stratification of participants according to lung cancer risk showed that screening with
LDCT prevented the greatest number of deaths from lung cancer among participants with the highest risk
and prevented few deaths in the lowest-risk groups. Smoking cessation remains the most effective way to
decrease the mortality and morbidity associated with lung cancer, however.

Ref: Kovalchik SA, Tammemagi M, Berg CD, et al: Targeting of low-dose CT screening according to the risk of lung-cancer
death. N Engl J Med 2013;369(3):245-254.

Item 95

ANSWER: D

This patient suffers from cluster headaches. Both verapamil and lithium are the mainstays of treatment for
chronic cluster headaches, but of the options listed, only verapamil is indicated for the prevention of cluster
headaches, and it is actually the first-line prophylactic agent (SOR A). Oxygen and sumatriptan are
first-line abortive therapies for cluster headaches (SOR A).

Ref: Weaver-Agostoni J: Cluster headache. Am Fam Physician 2013;88(2):122-128.

Item 96

ANSWER: A

This patient is showing signs of delirium, which is common in hospice patients. Delirium should be
considered in anyone with disturbances of cognitive function, altered attention, fluctuating consciousness,
or acute agitation. The mainstay of management is the diagnosis and treatment of any conditions that may
cause delirium. Medications that may cause delirium should be discontinued or reduced if possible.
Antipsychotic medications are the drug of choice to improve delirium. Central nervous system depressants
such as benzodiazepines and barbiturates should be avoided because they can make delirium worse.
Nortriptyline has anticholinergic side effects and can also cause delirium. Mirtazapine would not be helpful
for treating delirium.

Ref: Walsh D, Caraceni AT, Fainsinger R, et al (eds): Palliative Medicine. Saunders Elsevier, 2008, pp 860-865. 2) Rosenberg
M, Lamba S, Misra S: Palliative medicine and geriatric emergency care: Challenges, opportunities, and basic principles.
Clin Geriatr Med 2013;29(1):1-29.

33
Item 97

ANSWER: B

Respiratory syncytial virus (RSV) bronchiolitis is responsible for approximately 2.1 million health care
encounters annually in the United States. The child in this case has a typical presentation of RSV
bronchiolitis. The diagnosis can be made clinically, although specific testing for RSV is often used in the
hospital setting to segregate RSV-infected patients from others. Management is primarily supportive,
especially including maintenance of hydration and oxygenation. Bronchodilators, corticosteroids, and
antiviral agents do not have a significant impact on symptoms or the disease course. Ribavirin is not
recommended for routine use due to its expense, conflicting data on effectiveness, and potential toxicity
to exposed health care workers. Antibiotics are of no benefit in the absence of bacterial superinfection.

Ref: Dawson-Caswell M, Muncie HL Jr: Respiratory syncytial virus infection in children. Am Fam Physician
2011;83(2):141-146.

Item 98

ANSWER: C

Mesenteric ischemia presents with pain disproportionate to the findings on examination, often with nausea,
vomiting, or diarrhea. Air within the wall of dilated loops of small bowel (pneumatosis intestinalis) and
evidence of acidosis also suggest bowel ischemia. Cholangitis most likely would be associated with a more
substantial elevation of the amylase and/or lipase levels, as well as elevated bilirubin and/or alkaline
phosphatase levels. Pancreatitis would also be associated with higher amylase and/or lipase levels.

Acute appendicitis often has a vague presentation in older patients, presenting without fever and not
localizing to the right lower quadrant as it does in younger patients. However, the leukocytosis is usually
not as dramatic as in this case, there is usually no elevation of the amylase or lipase levels, and imaging
does not show air within the small bowel.

Ref: Lyon C, Clark DC: Diagnosis of acute abdominal pain in older patients. Am Fam Physician 2006;74(9):1537-1544. 2)
Cudnik MT, Darbha S, Jones J, et al: The diagnosis of acute mesenteric ischemia: A systematic review and meta-analysis.
Acad Emerg Med 2013;20(11):1087-1100.

Item 99

ANSWER: D

All of the drugs listed are used to treat motor symptoms in patients with Parkinsons disease. However,
the best evidence supports the use of carbidopa/levodopa, non-ergot dopamine agonists such as
pramipexole or ropinirole, or monoamine oxidase-B inhibitors such as selegiline or rasagiline for initial
management of patients with early disease (SOR A).

Ref: Gazewood JD, Richards DR, Clebak K: Parkinson disease: An update. Am Fam Physician 2013;87(4):267-273.

34
Item 100

ANSWER: B

A helpful guideline for assessing normal growth of a full-term healthy infant is that birth weight should
be regained within 14 days. Other useful guidelines for healthy term infants include an average weight gain
of 30 grams (1 oz) per day for the first month of life and doubling of birth weight between 4 and 5 months
of age.

Ref: Kliegman RM, Stanton BF, Geme JW III, et al (eds): Nelson Textbook of Pediatrics, ed 19. Elsevier Saunders, 2011, p
27.

Item 101

ANSWER: B

Medical conditions that decrease responsiveness to warfarin and reduce the INR include hypothyroidism,
visceral carcinoma, increased vitamin K intake, diabetes mellitus, and hyperlipidemia. Conditions that
increase responsiveness to warfarin, the INR, and the risk of bleeding include vitamin K deficiency caused
by decreased dietary intake, malabsorption, scurvy, malnutrition, cachexia, small body size, hepatic
dysfunction, moderate to severe renal impairment, hypermetabolic states, fever, hyperthyroidism,
infectious disease, heart failure, and biliary obstruction (SOR B, SOR C).

Ref: Wigle P, Hein B, Bloomfield HE, et al: Updated guidelines on outpatient anticoagulation. Am Fam Physician
2013;87(8):556-566. 2) McEvoy GK (ed): AHFS Drug Information 2013. Warfarin Sodium Class: 20:12.04.08 Coumarin
Derivatives; CAS Number: 129-06-6. American Society of Health-System Pharmacists, 2013.

Item 102

ANSWER: E

The sulfonylureas are the oral hypoglycemic agents most likely to cause hypoglycemia, with glyburide
more likely to cause low glucose levels than glipizide, due to its longer half-life. The use of these agents
should be rare in elderly patients with diabetes mellitus.

Ref: Kirkman MS, Briscoe VJ, Clark N, et al: Diabetes in older adults: A consensus report. J Am Geriatr Soc
2012;60(12):2342-2356.

Item 103

ANSWER: C

Rivaroxaban is used to prevent stroke in nonvalvular atrial fibrillation, but has also recently been approved
for prevention of deep-vein thrombosis and pulmonary embolism after hip or knee replacement surgery.
In patients with known deep-vein thrombosis or pulmonary embolism, rivaroxaban can be taken at a dosage
of 15 mg twice daily for 3 weeks followed by 20 mg daily for at least 3 months. Neither aspirin nor
clopidogrel is indicated. Subcutaneous heparin every 12 hours is not a therapeutic dosage and would be
difficult to manage in a home environment. Intravenous thrombolytic therapy may be appropriate in the
setting of a large pulmonary embolus, but it would be contraindicated in this case because of the patients
recent surgery.

35
Ref: Erlich DR: Rivaroxaban (Xarelto) for prevention of thromboembolic events. Am Fam Physician 2012;86(8):768-770. 2)
EINSTEINPE Investigators, Bller HR, Prins MH, Lensin AW, et al: Oral rivaroxaban for the treatment of symptomatic
pulmonary embolism. N Engl J Med 2012;366(14):1287-1297. 3) Graber MA, Dachs R, Endres J: Is rivaroxaban
noninferior to standard warfarin therapy in preventing recurrent PE and DVT? Am Fam Physician 2013;87(12):872-873.

Item 104

ANSWER: E

Most Hispanic immigrants have received the bacille Calmette-Gurin (BCG) vaccine. Although past
practice has been to interpret skin test results without regard to BCG status, false-positive tests in this
population are common. Interferon-( release assays are preferred to tuberculin skin testing in immigrants
who have been vaccinated with BCG.

Ref: Juckett G: Caring for Latino patients. Am Fam Physician 2013;87(1):48-54.

Item 105

ANSWER: B

Among the constellation of history and physical findings that can be found in patients with heart failure,
none provides a proof-positive diagnosis alone, as most are found in other disease states as well. Each of
the options listed raises the possibility of heart failure but the only one that has a specificity >90% is the
third heart sound, which is 99% specific for the diagnosis of heart failure. Other findings with >90%
sensitivity include a displaced point of maximal impulse, interstitial edema or venous congestion on a chest
radiograph, jugular vein distention, and hepatojugular reflux. The other options listed here have
specificities for heart failure that fall within the range of 65%80%.

Ref: King M, Kingery J, Casey B: Diagnosis and evaluation of heart failure. Am Fam Physician 2012;85(12):1161-1168.

Item 106

ANSWER: C

Fractures in children can be different from those in adults for several reasons, including the elasticity of
immature bone, the possibility of child abuse, and the presence of growth plates. This radiograph shows
a fracture from the growth plate through the metaphysis, known as a Salter-Harris type II fracture.
Approximately 6%7% of such fractures will cause a restriction of growth.

The Salter-Harris classification system was developed to classify five types of fractures into the growth
plate and can be used to estimate the risk of growth restriction. Type I fractures disrupt the physis only,
type III fractures are intra-articular fractures through the epiphysis into the physis, and type IV fractures
cross the epiphysis, physis, and metaphysis. Type V fractures involve a compression or crush injury of
the physis. The higher the classification, the greater the risk of complications.

Ref: Rakel RE, Rakel DP (eds): Textbook of Family Medicine, ed 8. Elsevier Saunders, 2011, pp 601-602.

36
Item 107

ANSWER: B

Unstable angina patients at high risk include those with at least one of the following:

Angina at rest with dynamic ST-segment changes 1 mm


Angina with hypotension
Angina with a new or worsening mitral regurgitation murmur
Angina with an S3 or new or worsening crackles
Prolonged (>20 min) anginal pain at rest
Pulmonary edema most likely related to ischemia

Ref: Devitt M: Diagnosis of stable ischemic heart disease: Recommendations from the ACP. Am Fam Physician
2013;88(7):469-470.

Item 108

ANSWER: D

The presence of a solitary enlarged left supraclavicular lymph node (Virchows node) is associated with
a gastrointestinal system malignancy. When combined with painless jaundice and a palpable nontender
gallbladder (Courvoisiers sign), pancreatic cancer is the most likely diagnosis.

A pancreatic pseudocyst develops after repeated bouts of pancreatitis and is not directly associated with
jaundice. Biliary cirrhosis and hepatocellular carcinoma typically present with pain, fatigue, malaise,
hepatomegaly, jaundice, and eventually ascites. The jaundice of biliary cirrhosis is generally accompanied
by severe pruritus. In neither condition is a palpably enlarged gallbladder present. Ascending cholangitis
presents with a high fever, right upper quadrant pain, and an overall toxic, septic picture, often
accompanied by delirium and rigors.

Ref: Longo DL, Fauci AS, Kasper DL, et al (eds): Harrisons Principles of Internal Medicine, ed 18. McGraw-Hill, 2012, pp
786-787.

Item 109

ANSWER: C

Medical hyperbaric oxygen is considered a reimbursable treatment option by many insurers for a long list
of diagnoses. The list of conditions shown to benefit from hyperbaric oxygen is a much shorter one,
however, and includes decompression sickness and wounds caused by crush injuries. Hyperbaric oxygen
treatment has been shown to improve diabetic foot ulcers in the short term but studies have so far failed
to prove long-term benefit.

Ref: Gill AL, Bell CN: Hyperbaric oxygen: Its uses, mechanisms of action and outcomes. QJM 2004;97(7):385-395. 2) Xiao
Y, Wang J, Jiang S, Luo H: Hyperbaric oxygen therapy for vascular dementia. Cochrane Database Syst Rev
2012;(7):CD009425. 3) Bennett MH, Stanford RE, Turner R: Hyperbaric oxygen therapy for promoting fracture healing
and treating fracture non-union. Cochrane Database Syst Rev 2012;(11):CD004712. 4) Bennett MH, Kertesz T, Perleth
M, et al: Hyperbaric oxygen for idiopathic sudden sensorineural hearing loss and tinnitus. Cochrane Database Syst Rev
2012;(10):CD004739. 5) Eskes A, Vermeulen H, Lucas C, Ubbink DT: Hyperbaric oxygen therapy for treating acute
surgical and traumatic wounds. Cochrane Database Syst Rev 2013;(12):CD008059.

37
Item 110

ANSWER: D

The cause of this patients rash is difficult to determine. There are many infections that could result in a
cutaneous reaction similar to what she is experiencing. Scarlet fever is caused by a systemic reaction to
Streptococcus. In this case, however, the patient is already taking an antibiotic for streptococcal disease
so the emergence of new symptoms over a week after starting therapy is highly unlikely. A viral exanthem
could also cause a skin rash similar to the one described here. Unfortunately, differentiating between a
drug-induced rash and a viral exanthem is not clinically possible. If this differentiation is necessary, the
patient should undergo a skin biopsy and allergy testing to determine the offending agent. However, since
this approach is impractical in the ambulatory setting, it is most straightforward to discontinue the agent
she is on and list it as a potential allergy. An alternative antibiotic such as erythromycin could be used to
complete the course of treatment at the discretion of the physician.

Ref: Habif TP, Campbell JL Jr, Chapman MS, et al: Skin Disease: Diagnosis & Treatment, ed 3. Saunders Elsevier, 2011, pp
290-311. 2) Bolognia JL, Jorizzo JL, Schaffer JV: Dermatology, ed 3. Elsevier Saunders, 2012, pp 81, 1345-1365.

Item 111

ANSWER: A

The best pharmacologic agent for this patient is duloxetine, as it is indicated for both depression and
chronic pain and is unlikely to cause weight gain. The other agents listed can cause weight gain to varying
degrees, and the tricyclic antidepressant nortriptyline is on the Beers list of drugs not recommended for
elderly patients (SOR A).

Ref: Drugs associated with weight gain. Pharmacists Letter/Prescribers Letter 2007;23(3):220312. 2) Gelenberg AJ, Freeman
MP, Markowitz JC, et al: Guideline for the Treatment of Patients With Major Depressive Disorder, ed 3. American
Psychiatric Association, 2010, p 74.

Item 112

ANSWER: A

Atenolol may cause growth restriction and reduced placental weight. Because there are many
antihypertensive drugs that have a much better safety profile than atenolol, it is recommended that atenolol
NOT be used during pregnancy. Both animal and human data suggests that metformin is low risk in
pregnancy and it is therefore safe to continue this drug.

Although experience with insulin during pregnancy in humans is very limited, the available data suggests
that the risk of harm to the embryo or fetus is low, if it exists at all, as insulin does not cross the placenta.
A primary concern is severe maternal hypoglycemia, making careful monitoring of blood glucose
necessary.

The animal and most of the human data on bupropion use in pregnancy suggests low risk. Although
increased rates of heart defects were reported in two studies, this outcome has not been confirmed by other
studies. If a woman requires bupropion she should be informed of the potential risks, but the drug should
not be withheld because of pregnancy.

38
The available animal and human experience indicates that fluoxetine is not a major teratogen. However,
SSRIs, including fluoxetine, have been associated with several developmental toxicities, including
spontaneous abortion, low birth weight, prematurity, neonatal serotonin syndrome, neonatal behavioral
syndrome (withdrawal), possibly sustained abnormal neurobehavior beyond the neonatal period, respiratory
distress, and persistent pulmonary hypertension of the newborn. Because the absolute risk is small, most
physicians who provide prenatal care will continue drugs such as fluoxetine in patients with a documented
significant mood disorder. Patients do, however, need to be aware of possible (albeit low) risks to the
fetus.

Ref: Briggs GG, Freeman RK, Yaffe SJ: Drugs in Pregnancy and Lactation: A Reference Guide to Fetal and Neonatal Risk, ed
9. Lippincott Williams & Wilkins, 2011.

Item 113

ANSWER: B

Patients with microscopic hematuria should initially be assessed for benign causes such as urinary tract
infection, vigorous exercise, menstruation, and recent urologic procedures. If none of these is found, the
next step would be assessing for renal disease using urine microscopy to look for casts or dysmorphic
blood cells, and checking renal function. If the results are negative, CT urography and cystoscopy should
be performed. CT evaluates the upper urinary tract for nephrolithiasis and renal cancer, while cystoscopy
evaluates the bladder for bladder cancer, urethral strictures, and prostatic problems.

Urine cytology is less sensitive than cystoscopy for bladder cancer. This patient has normal renal function
and no signs of renal disease on the urinalysis other than hematuria, so a nephrology consultation is not
necessary at this time. Metformin use is not associated with microscopic hematuria. There is no role for
antibiotics, given the negative urine culture.

Ref: Sharp VJ, Barnes KT, Erickson BA: Assessment of asymptomatic microscopic hematuria in adults. Am Fam Physician
2013;88(11):747-754.

Item 114

ANSWER: D

The differential diagnosis of multiple small scaling plaques includes drug eruptions, secondary syphilis,
guttate psoriasis, and erythema migrans. If the diagnosis cannot be made conclusively by clinical
examination, a test for syphilis should be ordered. The rash of secondary syphilis may be indistinguishable
from pityriasis rosea on initial examination, particularly when no herald patch is noted. The rashes
associated with hyperthyroidism, infectious mononucleosis, idiopathic thrombocytopenic purpura, and
fungal infections are not in the differential diagnosis for this patient.

Ref: Habif TP: Clinical Dermatology: A Color Guide to Diagnosis and Therapy, ed 5. Mosby Elsevier, 2010, pp 316-318.

39
Item 115

ANSWER: D

Inflammatory bowel disease is an autoimmune disorder that affects the gastrointestinal tract, usually
beginning in early adulthood. Ulcerative colitis and Crohns disease are the most common of these
conditions. Ulcerative colitis involves just the mucosa of the colon, starting at the anus and extending
proximally to a variable distance. Crohns disease, on the other hand, may involve all layers of
gastrointestinal tissue and can occur anywhere between the mouth and the anus. The diagnosis of either
of these conditions is made by endoscopy with biopsies in order to best assess the extent and depth of
inflammation.

Ref: Adams SM, Bornemann PH: Ulcerative colitis. Am Fam Physician 2013;87(10):699-705. 2) Ford AC, Moayyedi P,
Hanauer SB: Ulcerative colitis. BMJ 2013;346:f432.

Item 116

ANSWER: D

Due to the increasing incidence of pertussis, the Centers for Disease Control and Prevention recommends
that all pregnant women receive Tdap vaccine during every pregnancy regardless of when their last dose
was. It is ideally administered between 27 and 36 weeks gestation to maximize the maternal antibody
response and passive antibody transfer to the infant.

Ref: Bridges CB, Coyne-Beasley T; Advisory Committee on Immunization Practices: Advisory Committee on Immunization
Practices recommended immunization schedule for adults aged 19 years or older: United States, 2014. Ann Intern Med
2014;160(3):190-197.

Item 117

ANSWER: C

The presence of eosinophiluria in a patient with acute kidney injury (AKI) suggests acute interstitial
nephritis, which is typically an allergic reaction to medications such as penicillins, sulfa-containing
antibiotics and diuretics, NSAIDs, proton pump inhibitors, etc. Patients with acute interstitial nephritis may
also present with a rash, fever, eosinophilia, and other constitutional symptoms. The combination of
elevated levels of creatine kinase or myoglobin, a dipstick positive for blood but negative for RBCs, and
a history of muscle trauma would suggest rhabdomyolysis. An elevated uric acid level along with a history
of rapidly proliferating tumors or recent chemotherapy suggests tumor lysis syndrome and malignancy.
Poisoning with ethylene glycol or methanol should be suspected in a patient with AKI and altered mental
status with an increased anion gap and osmolar gap. An elevated antistreptolysin O titer suggests
poststreptococcal glomerulonephritis when combined with a history of recent pharyngitis.

Ref: Rahman M, Shad F, Smith MC: Acute kidney injury: A guide to diagnosis and management. Am Fam Physician
2012;86(7):631-639.

40
Item 118

ANSWER: C

Atrial fibrillation is a risk factor for stroke, and most patients benefit from anticoagulation for stroke
prevention, but benefits must be balanced against bleeding risks. For some patients with no additional risk
factors for stroke the balance is in favor of aspirin rather than warfarin. The CHA2DS2-VAS score is a
validated tool for identifying these low-risk patients. Those with a score of 0 are most appropriately
managed with aspirin. The score is calculated as follows:

Points
C Congestive heart failure (or left ventricular systolic dysfunction) 1
H Hypertension: Blood pressure consistently above 140/90 mm Hg
(or hypertension controlled with medication) 1
A2 Age 75 years 2
D Diabetes mellitus 1
S2 Prior Stroke or TIA or thromboembolism 2
V Vascular disease (e.g., peripheral artery disease, myocardial infarction,
aortic plaque) 1
A Age 6574 years 1
Sc Sex category (sex = female) 1

Ref: Lip GY, Nieuwlaat R, Pisters R, et al: Refining clinical risk stratification for predicting stroke and thromboembolism in
atrial fibrillation using a novel risk factor-based approach: The Euro Heart Survey on Atrial Fibrillation. Chest
2010;137(2):263-272.

Item 119

ANSWER: B

Not all hospitalized patients need stress ulcer prophylaxis. Routine acid-suppression therapy to prevent
stress ulcers has no benefit in hospitalized patients outside of the critical care setting. Only critically ill
patients who meet specific criteria should receive this therapy. One indication for stress ulcer prophylaxis
is prolonged mechanical ventilation for more than 48 hours. Hemodynamically stable patients admitted to
general care floors should not receive stress ulcer prophylaxis, as it only decreases the rate of
gastrointestinal bleeding from 0.33% to 0.22%. Furthermore, long-term proton pump inhibitor therapy
has been associated with complications such as Clostridium difficile diarrhea and community-acquired
pneumonia. Discontinuation of stress ulcer prophylaxis should be considered for this patient when she
moves out of the critical care unit. It could also be considered when the patient is removed from the
ventilator.

Ref: Grube RR, May DB: Stress ulcer prophylaxis in hospitalized patients not in intensive care units. Am J Health Syst Pharm
2007;64(13):1396-1400. 2) Eisa N, Bazerbachi F, Alraiyes AH, Alraies MC: Q: Do all hospitalized patients need stress
ulcer prophylaxis? Cleve Clin J Med 2014;81(1):23-25.

41
Item 120

ANSWER: C

The rash described in this patient with significant thrombocytopenia is consistent with purpura. Purpura
from vasculitic causes such as meningococcal infection, disseminated intravascular coagulation, or
Henoch-Schnlein purpura (also known as IgA nephropathy) is typically palpable rather than macular as
in this case. Immune thrombocytopenic purpura is a relatively common cause of isolated
thrombocytopenia. The lack of systemic symptoms or other abnormal laboratory findings make acute
lymphoproliferative disorders such as leukemia unlikely. Likewise anemia, neurologic changes, fever, and
renal failure are seen with thrombotic thrombocytopenic purpura. The acute onset of purpura and heavy
periods makes congenital thrombocytopenia unlikely.

Ref: Gauer RL, Braun MM: Thrombocytopenia. Am Fam Physician 2012;85(6):612-622.

Item 121

ANSWER: C

Uveitis is inflammation of the uveal tract and can affect any or all of its components, including the iris.
It is the most common extra-articular manifestation of ankylosing spondylitis (AS), seen in up to 60% of
patients with AS. Iritis presents with a painful red eye with conjunctival injection, photophobia, and a
sluggishly reacting pupil. A hazy-appearing anterior chamber results from the iris producing an
inflammatory exudate. Treatment includes topical corticosteroids, but oral or parenteral corticosteroids and
NSAIDs are also effective. Reduced anterior spine flexion (a positive modified Schober test) results from
the skeletal manifestations of AS. A bamboo spine is classically seen on lumbar radiographs. Oral or
ocular antibiotics, artificial tears, ophthalmic olopatadine, and oral acetazolamide are ineffective.
Ophthalmology referral is recommended (SOR B).

Ref: Longo DL, Fauci AS, Kasper DL, et al (eds): Harrisons Principles of Internal Medicine, ed 18. McGraw-Hill, 2012, pp
229, 2774-2775.

Item 122

ANSWER: B

For chronic midsubstance Achilles tendinopathy (symptoms lasting longer than 6 weeks), the preferred
first-line treatment is an intense eccentric strengthening program of the gastrocnemius/soleus complex
(SOR A). In randomized, controlled trials, eccentric strengthening programs have provided 60%90%
improvement in pain and function. Therapeutic modalities such as ultrasonography, electrical stimulation,
iontophoresis, and massage and stretching have shown inconsistent results for helping patients achieve a
long-term return to function. Surgical techniques are a last resort for severe or recalcitrant cases, but these
techniques have not been consistently successful and carry additional risk.

To perform eccentric strengthening for Achilles tendinopathy the patient should stand on the ball of the
injured foot with the calcaneal area of the foot over the edge of a stair step. The patient begins with a
straight leg and the ankle in flexion. The ankle is then lowered to full dorsiflexion with the heel below the
level of the step and then returned to flexion with the assistance of the uninjured leg.

Ref: Childress MA, Beutler A: Management of chronic tendon injuries. Am Fam Physician 2013;87(7):486-490.

42
Item 123

ANSWER: E

Posttraumatic stress disorder (PTSD) occurs in approximately 20% of women and 8% of men exposed to
traumatic events. Symptoms of PTSD include reexperiencing the event, depression, anxiety, changes in
behavior, restlessness, social withdrawal, hypervigilance, poor attention, irritability, and fear. Many
people with PTSD suffer from anxiety, depression, and substance abuse, and as many as one in five
attempt suicide. Treatment with a combination of trauma-focused therapy and medications is
recommended. SSRIs and SNRIs are considered first-line treatment. While paroxetine and sertraline are
the only ones FDA-approved for PTSD, any of these drugs may be used. Other antidepressant medications
can be used but are considered second-line treatment. Benzodiazepines have been used to treat the
symptoms of hyperarousal but can worsen other PTSD symptoms and should be avoided. Studies of mood
stabilizers in the treatment of PTSD have been mixed and many guidelines discourage their use.
Antipsychotic medications are also not recommended. A large multi-site trial of risperidone reported no
benefit over placebo.

Ref: Warner CH, Warner CM, Appenzeller GN, Hoge CW: Identifying and managing posttraumatic stress disorder. Am Fam
Physician 2013;88(12):827-834.

Item 124

ANSWER: C

Cerebrospinal fluid (CSF) rhinorrhea is not that rare, and has both surgical and nonsurgical causes. It
results from a direct communication between the subarachnoid space and the paranasal sinuses. Accidental
trauma causes 70%80% of CSF rhinorrhea cases, with 2%4% of acute head injuries resulting in CSF
rhinorrhea. Nontraumatic CSF rhinorrhea includes high-pressure and normopressure leaks from causes
including tumors, processes including boney erosion, empty sella syndrome, and congenital defects
including meningoceles. The rhinorrhea is clear and often has a sweet or salty taste. The drainage can be
continuous or intermittent, and is often associated with a gush when changing from a recumbent to an
upright position. CSF rhinorrhea can lead to meningitis or other infections by serving as a pathway for
bacteria.

Ref: Kerr JT, Chu FW, Bayles SW: Cerebrospinal fluid rhinorrhea: Diagnosis and management. Otolaryngol Clin North Am
2005;38(4):597-611. 2) Flint PW, Haughey BH, Lund VJ, et al (eds): Cummings Otolaryngology: Head and Neck Surgery,
ed 5. Mosby Elsevier, 2010, pp 785-796.

Item 125

ANSWER: C

Prolactin levels can be elevated because of a pituitary adenoma, medication side effects, hypothyroidism,
or a mass lesion compromising normal hypothalamic inhibition. Elevated prolactin levels inhibit the
secretion and effect of gonadotropins. In almost all patients with an elevated prolactin level, MRI of the
pituitary is recommended to exclude the possibility of a pituitary adenoma (SOR C). This patient is not on
any medications, essentially ruling out a pharmacologic trigger for her elevated prolactin.

Ref: Klein DA, Poth MA: Amenorrhea: An approach to diagnosis and management. Am Fam Physician 2013;87(11):781-788.

43
Item 126

ANSWER: E

The diagnosis of plantar fasciitis is based primarily on the history and physical examination. Patients may
present with heel pain, and palpation of the medial plantar calcaneal region may elicit a sharp pain.
Discomfort in the proximal plantar fascia can be elicited by passive ankle/first toe dorsiflexion. Diagnostic
imaging is rarely needed for the initial diagnosis of plantar fasciitis. In recalcitrant plantar fasciitis plain
films may be helpful for detecting bony lesions of the foot. Ultrasonography is inexpensive and may be
useful for ruling out soft-tissue pathology of the heel in some patients. While MRI is expensive, it is a
valuable tool for assessing causes of recalcitrant heel pain.

Ref: Goff JD, Crawford R: Diagnosis and treatment of plantar fasciitis. Am Fam Physician 2011;84(6):676-682.

Item 127

ANSWER: B

The U.S. Preventive Services Task Force recommends daily aspirin for women ages 5579 when the
benefit of stroke risk reduction outweighs the risk of gastrointestinal hemorrhage. Aspirin has been shown
to be effective for the secondary prevention of cardiovascular disease in both men and women. In men,
randomized trials have shown that low-dose aspirin decreases the risk of a first myocardial infarction, but
not stroke.

Ref: Ridker PM, Cook NR, Lee IM, et al: A randomized trial of low-dose aspirin in the primary prevention of cardiovascular
disease in women. N Engl J Med 2005;352(13):1293-1304. 2) US Preventive Services Task Force: Aspirin for the
Prevention of Cardiovascular Disease: Recommendation Statement. AHRQ pub no 09-05129-EF-2, 2009. 3) Riley M,
Dobson M, Jones E, Kirst N: Health maintenance in women. Am Fam Physician 2013;87(1):30-37.

Item 128

ANSWER: C

Recent reports indicate that the risk of a serious allergic reaction to second- and third-generation
cephalosporins in patients with penicillin or amoxicillin allergy appears to be almost nil, and no greater
than the risk among patients without such allergies. While patients with a history of a serious type I
immediate or accelerated (anaphylactoid) reaction to amoxicillin can be safely treated with cefdinir,
cefuroxime, or cefpodoxime, some physicians may wish to recommend an allergy referral to determine
tolerance before initiation of therapy. Pneumococcus and Haemophilus influenzae are often resistant to
trimethoprim/sulfamethoxazole and azithromycin, and these agents are therefore not recommended for the
treatment of acute bacterial sinusitis in the penicillin-allergic patient. Doxycycline should not be used in
children younger than 8 years of age except for anthrax and some tickborne infections.
Amoxicillin/clavulanate is contraindicated in a penicillin-allergic patient.

Ref: Wald ER, Applegate KE, Bordley C, et al: Clinical practice guideline for the diagnosis and management of acute bacterial
sinusitis in children aged 1 to 18 years. Pediatrics 2013;132(1):e262-e280.

44
Item 129

ANSWER: B

Somatic complaints are seen in up to two-thirds of primary care patients with depression, and are more
likely in certain groups, including pregnant women, children, the elderly, and low-income groups.

Ref: Tylee A, Gandhi P: The importance of somatic symptoms in depression in primary care. Prim Care Companion J Clin
Psychiatry 2005;7(4):167-176. 2) Pacala J, Sullivan G (eds): Geriatrics Review Syllabus, ed 7. American Geriatrics
Society, 2010, sec 7, pp 331-332.

Item 130

ANSWER: B

Left bundle branch block makes the EKG uninterpretable during an exercise stress test, and can also
interfere with nuclear imaging performed during the test. It is associated with transient positive defects in
the anteroseptal and septal regions in the absence of a lesion within the left anterior descending coronary
artery. This leads to a high rate of false-positive tests and low specificity. Pharmacologic stress tests using
vasodilators such as adenosine with nuclear imaging have a much higher specificity and positive predictive
value for LAD lesions, and the same is true for dobutamine stress echocardiography, which is why these
are the preferred methods for evaluating patients with left bundle branch block. Pharmacologic stress
testing would not be preferred for evaluating the other EKG abnormalities listed.

Ref: Wilson JF: Stable ischemic heart disease. Ann Intern Med 2014;160(1):ITC1-1-ITC1-16.

Item 131

ANSWER: E

Poison ivy dermatitis is caused by urushiol, a resin found in poison ivy, poison oak, and poison sumac
plants. Direct contact with the leaves or vines will result in an acute dermatitis manifested initially by
erythema, and later in more severe cases by vesicles and bullae. This is a type IV T cellmediated allergic
reaction, so it typically takes at least 12 hours and often 23 days before the reaction is fully manifested.
Depending on the degree of contact (i.e., the amount of resin on the skin), the rash often progresses over
a couple of days, giving the impression that it is spreading. Also, delayed contact with resin from
contaminated clothing, gloves, or pets may result in new lesions appearing over several days. Brushing
against the leaves of the plant causes the linear streaking pattern characteristic of poison ivy dermatitis.
It has been demonstrated that the resin can be inactivated with any type of soap, thereby preventing the
reaction, but the sooner the better. Approximately 50% of the resin can be removed by soap and water
within 10 minutes of contact, but after 30 minutes only about 10% can still be removed.

Therapy depends on the severity of the reaction. Group IV topical corticosteroids are effective for limited
eruptions (less than 3%5% body surface area) but are ineffective in areas with vesicles or bullae. Group
III fluorinated agents are at the strongest end of the spectrum and are not recommended for use on the
face or intertriginous areas. Short bursts of low-potency oral corticosteroids such as a methylprednisolone
dose pack have a high rate of relapse as the taper finishes, so the expert consensus is to use a higher
dosage tapered over a longer period, generally 1014 days, in order to prevent a relapse. Most experts
recommend oral corticosteroids over intramuscular corticosteroid suspensions, which may not provide high
enough concentrations in the skin (SOR C). However, 4080 mg of intramuscular triamcinolone (or an
equivalent) is an alternative to oral treatment, especially if adherence is an issue. Pruritus can be treated

45
with oral antihistamines. Secondary infection, which is common with vesiculobullous involvement, is
treated with appropriate oral antibiotics.

Ref: Habif TP: Clinical Dermatology: A Color Guide to Diagnosis and Therapy, ed 5. Mosby Elsevier, 2010, pp 138-139.

Item 132

ANSWER: C

The most likely diagnosis is Pneumocystis pneumonia. Initially named Pneumocystis carinii, the causative
organism has been reclassified and renamed Pneumocystis jiroveci. It causes disease in
immunocompromised patients. In nonHIV-infected patients, the most significant risk factors are defects
in cell-mediated immunity, glucocorticoid therapy, use of immunosuppressive agents (especially when
dosages are being lowered), hematopoietic stem cell or solid organ transplant, cancer, primary
immunodeficiencies, and severe malnutrition.

The clinical presentation in patients without HIV/AIDS is typically an acute onset of hypoxia and
respiratory failure, associated with a dry cough and fever. Characteristic radiographic findings include
diffuse bilateral interstitial infiltrates.

Pneumococcal pneumonia typically presents with fever, chills, cough, and pleuritic chest pain. A sudden
onset of severe hypoxia is less common. Radiologic findings typically include lobar infiltrates or
bronchopneumonia (with a segmental pattern of infiltrate), whereas diffuse bilateral infiltrates are much
less common. Staphylococcal pneumonia usually has radiologic findings of focal, multiple infiltrates or
cavitary lesions.

Pulmonary tuberculosis presents most commonly with pleuritic or retrosternal chest pain. Fever is present
in about 25% of patients. Cough is actually less common, and a sudden onset of acute hypoxia would be
a very rare presentation. Radiographs typically reveal hilar adenopathy and pleural effusion. Diffuse
bilateral interstitial infiltrates would be a very rare finding.

Spontaneous pneumothorax does present with an acute onset of hypoxia, tachypnea, and respiratory
distress. However, fever would be unlikely and the radiologic findings in this patient are not consistent
with pneumothorax.

Ref: Wilkin A, Feinberg J: Pneumocystis carinii pneumonia: A clinical review. Am Fam Physician 1999;60(6):1699-1708,
1713-1714. 2) Gilroy SA, Bennett NJ: Pneumocystis pneumonia. Semin Respir Crit Care Med 2011;32(6):775-782.

46
Item 133

ANSWER: B

Benzodiazepines play a key role in the management of alcohol withdrawal syndrome (AWS), especially
as they are highly effective in the prevention and treatment of seizures associated with this syndrome. In
general, nonbenzodiazepine anticonvulsants are not effective for preventing seizures in patients with AWS.
Therefore, their use is not recommended in those at risk for seizures or those who have a CIWA-Ar score
in the moderate or severe range. The potential for abuse with these agents is much lower than with
benzodiazepines, and they are preferred over benzodiazepines for outpatient management of AWS,
especially in those with a past history of substance abuse. Carbamazepine and valproic acid may be
effective for managing the symptoms associated with AWS. Gabapentin has been shown to be as effective
as lorazepam in treating AWS and reducing alcohol use during withdrawal. Phenytoin is not effective for
the treatment or prevention of seizures associated with AWS (SOR B).

Ref: Addolorato G, Leggio L, Abenavoli L, et al: Baclofen in the treatment of alcohol withdrawal syndrome: A comparative
study vs diazepam. Am J Med 2006;119(3):276.e13-276.e18. 2) Amato L, Minozzi S, Vecchi S, Davoli M:
Benzodiazepines for alcohol withdrawal. Cochrane Database Syst Rev 2010;(3):CD005063. 3) Minozzi S, Amato L, Vecchi
S, Davoli M: Anticonvulsants for alcohol withdrawal. Cochrane Database Syst Rev 2010;(3):CD005064. 4) Muncie HL
Jr, Yasinian Y, Oge L: Outpatient management of alcohol withdrawal syndrome. Am Fam Physician 2013;88(9):589-595.

Item 134

ANSWER: E

While all of the options listed may have some value in evaluating the preoperative status of a patient with
long-standing rheumatoid arthritis, imaging of the patients cervical spine to detect atlantoaxial subluxation
would be most important for preventing a catastrophic spinal cord injury during intubation. In many cases
cervical fusion must be performed before other elective procedures can be contemplated. Although
rheumatoid arthritis may influence oxygen saturation and the erythrocyte sedimentation rate, these tests
would not alert the surgical team to the possibility of significant operative morbidity and mortality. Resting
pulse rate and a rheumatoid factor titer are unlikely to be significant factors in this preoperative scenario.

Ref: Neva MH, Hkkinen A, Mkinen H, et al: High prevalence of asymptomatic cervical spine subluxation in patients with
rheumatoid arthritis waiting for orthopaedic surgery. Ann Rheum Dis 2006;65(7):884-888. 2) Klippel JH, Stone JH,
Crofford LJ, et al (eds): Primer on the Rheumatic Diseases, ed 13. Springer, 2008, p 653. 3) Miller RD, Eriksson LI,
Fleisher LA, et al (eds): Millers Anesthesia, ed 7. Churchill Livingstone, 2010, pp 2245-2246. 4) Firestein GS, Budd RC,
Gabriel SE, et al: Kelleys Textbook of Rheumatology, ed 9. Elsevier Saunders Co, 2013, pp 115-117.

Item 135

ANSWER: D

This patient has rapidly progressive erythema and pain in her right lower extremity, along with fever,
tachycardia, and leukocytosis. Group A Streptococcus (GAS) is a common monomicrobial cause of type
II necrotizing skin infections, which are often referred to as necrotizing fasciitis and warrant immediate
attention (SOR C). Type I infections are often polymicrobial due to combinations of staphylococci
(especially Staphylococcus epidermidis in combination with $-hemolytic streptococci), enterococci,
Enterobacteriaceae species (commonly Escherichia coli, Proteus mirabilis, Klebsiella pneumoniae, and
Pseudomonas aeruginosa), streptococci, Bacteroides/Prevotella species, anaerobic gram-positive cocci,
and Clostridium species.

47
For this patient with a suspected necrotizing skin infection, aggressive treatment with a broad-spectrum
empiric antibiotic is recommended along with hemodynamic support and consideration of surgical
exploration and debridement of necrotic tissue (SOR C). Empiric antibiotic treatment of a potential
necrotizing infection should consist of broad-spectrum antimicrobial therapy with activity against
gram-positive, gram-negative, and anaerobic organisms; special consideration should be given to group
A Streptococcus, Clostridium species, and methicillin-resistant Staphylococcus aureus (MRSA).

Ref: Hasham S, Matteucci P, Stanley PR, Hart NB: Necrotising fasciitis. BMJ 2005;330(7495):830-833. 2) Stevens DL, Bisno
AL, Chambers HF, et al: Practice guidelines for the diagnosis and management of skin and soft-tissue infections. Clin Infect
Dis 2005;41(10):1373-1406. 3) Anaya DA, Dellinger EP: Necrotizing soft-tissue infection: Diagnosis and management.
Clin Infect Dis 2007;44(5):705-710. 4) Department of Health and Human Services: Group A Streptococcal (GAS) Disease
(strep throat, necrotizing fasciitis, impetigo). Centers for Disease Control and Prevention, 2008. 5) Breen JO: Skin and
soft tissue infections in immunocompetent patients. Am Fam Physician 2010;81(7):893-899.

Item 136

ANSWER: E

The U.S. Preventive Services Task Force recommends screening for Chlamydia infection in all sexually
active, nonpregnant young women under the age of 25 (grade B recommendation). Papanicolaou testing
is recommended starting at 21 years of age. Testicular cancer screening, whether by self-examination or
as part of the physical examination, is not recommended. Scoliosis screening for asymptomatic adolescents
is also not recommended. There is insufficient evidence to recommend for or against lipid screening.

Ref: Screening for chlamydial infection. US Preventive Services Task Force, 2007. 2) Ham P, Allen C: Adolescent health
screening and counseling. Am Fam Physician 2012;86(12):1109-1116.

Item 137

ANSWER: E

The use of gastric acid inhibitors, particularly when a proton pump inhibitor and H2-receptor antagonist
are combined, is significantly associated with vitamin B12 deficiency. This is more common when combined
therapy has been used for 2 years or longer. Because gastric acid is required for the liberation of vitamin
B12 bound to food protein before it is bound to intrinsic factor for absorption, suppression of gastric acid
may lead to vitamin B12 deficiency.

Ref: Lam JR, Schneider JL, Zhao W, Corley DA: Proton pump inhibitor and histamine 2 receptor antagonist use and vitamin
B12 deficiency. JAMA 2013;310(22):2435-2442.

Item 138

ANSWER: E

Frozen shoulder is an inflammatory contracture of the shoulder capsule and mostly affects the
anterosuperior and anteroinferior capsular ligaments, limiting glenohumeral movement. Diabetic patients
have a 10%20% lifetime risk of frozen shoulder. Only two other common conditions selectively limit
passive external rotation: locked posterior dislocation and osteoarthritis. Plain films of the shoulder should
reveal both conditions. Rotator cuff tears do not limit passive range of motion, and calcific tendinitis has
a characteristic radiographic appearance.

Ref: Robinson CM, Seah KT, Chee YH, et al: Frozen shoulder. J Bone Joint Surg Br 2012;94(1):1-9.

48
Item 139

ANSWER: D

Patients with HIV infection and severe immunodeficiency are at risk for certain opportunistic infections.
Susceptibility to opportunistic infections can be measured by CD4+ T lymphocyte counts. Patients with
a CD4+ count <200 cells/mm3 should receive trimethoprim/sulfamethoxazole for prevention of
Pneumocystis pneumonia, and prophylaxis against Toxoplasma gondii should also be given if the CD4+
level is <100 cells/mm3. Azithromycin is used to prevent infection with Mycobacterium
avium-intracellulare complex when CD4+ counts are <50 cells/mm3. Itraconazole is used to prevent
Histoplasma capsulatum infection when the CD4+ count is 150 cells/mm3 if the patient is at risk due to
occupational exposure or living in a community with a hyperendemic rate of histoplasmosis (>10 cases
per 100 patient years). There is no recommendation for prophylaxis against microsporidiosis.

Ref: Goldman L, Schafer AI (eds): Goldmans Cecil Medicine, ed 24. Elsevier Saunders, 2011, pp 2190-2194.

Item 140

ANSWER: C

Legg-Calv-Perthes disease results from interruption of the blood supply to the still-growing femoral head.
It occurs in children 212 years of age and presents with hip pain and an atraumatic limp. Common
physical findings include leg-length discrepancies, and limited abduction and internal rotation. Radiographs
reveal sclerosis of the proximal femur with joint space widening. MRI confirms osteonecrosis.

Septic arthritis also causes atraumatic anterior hip pain but occurs in the acutely ill, febrile patient. A CBC,
erythrocyte sedimentation rate, C-reactive protein level, and guided hip aspiration are recommended if
septic arthritis is suspected. A diagnosis of stress fracture should be considered in patients with a history
of overuse and weight-bearing exercise. These patients have pain that is worse with activity, and pain on
active leg raising. MRI can detect fractures not seen on plain films.

Iliopsoas bursitis presents with snapping or popping of the hip on extension from a flexed position. Labral
tears present with sharp anterior hip pain at times, with radiation to the thigh or buttock. Usually patients
will have mechanical symptoms such as clicking with activity. The FABER (flexion, abduction, external
rotation) and FADIR (flexion, adduction, internal rotation) impingement tests are sensitive for labral tears.

Ref: Atanda A Jr, Shah SA, OBrien K: Osteochondrosis: Common causes of pain in growing bones. Am Fam Physician
2011;83(3):285-291. 2) Wilson JJ, Furukawa M: Evaluation of the patient with hip pain. Am Fam Physician
2014;89(1):27-34.

Item 141

ANSWER: B

Hormonal contraceptives are the first-line therapy for menstrual abnormalities, hirsutism, and acne in
polycystic ovary syndrome. Clomiphene is used for infertility. Thiazolidinediones have an unfavorable
risk-benefit ratio overall. Metformin is beneficial for metabolic/glycemic abnormalities and menstrual
irregularities, but does not improve hirsutism or acne. Spironolactone may be used as an add-on to
hormonal contraceptives for treatment of hirsutism and acne.

Ref: Legro RS, Arslanian SA, Ehrmann DA, et al: Diagnosis and treatment of polycystic ovary syndrome: An Endocrine Society
clinical practice guideline. J Clin Endocrinol Metab 2013;98(12):4565-4592.

49
Item 142

ANSWER: D

Patients taking $-blockers may be resistant to treatment with epinephrine. Glucagon has positive inotropic
and chronotropic effects that are not mediated through $-receptors, and should be administered to
anaphylactic patients on $-blockers when their response to epinephrine is either poor or absent. In patients
not taking $-blockers a repeat dose of epinephrine is recommended when the response to the first dose is
either poor or absent. Intravenous methylprednisolone and diphenhydramine may also be repeated based
on clinical response (SOR C). An H2-blocker such as cimetidine may provide additional benefit in
combination with an H1 antihistamine. In an emergency situation such as this, there is no benefit to using
a long-acting corticosteroid.

Ref: Thomas M, Crawford I: Glucagon infusion in refractory anaphylactic shock in patients on $-blockers. Emerg Med J
2005;22(4):272-273. 2) Arnold JJ, Williams PM: Anaphylaxis: Recognition and management. Am Fam Physician
2011;84(10):1111-1118.

Item 143

ANSWER: B

Of the choices listed, prematurity is the greatest risk factor for group B streptococcal infection. The most
important risk would be signs or symptoms of sepsis in a neonate. The other conditions listed are not risk
factors for early-onset GBS in neonates.

Ref: Verani JR, McGee L, Schrag SJ: Prevention of perinatal group B streptococcal diseaseRevised guidelines from CDC,
2010. MMWR Recomm Rep 2010;59(RR-10):1-36.

Item 144

ANSWER: C

The American Geriatrics Society (AGS) position statement on feeding tubes states that percutaneous
feeding tubes are not recommended for older adults with advanced dementia, and that careful hand feeding
should be offered instead. This is the first recommendation by the AGS in the Choosing Wisely campaign.

Careful hand feeding for patients with severe dementia is at least as good as tube feeding with regard to
the outcomes of death, aspiration pneumonia, functional status, and patient comfort. Regular food is
preferred. Tube feeding is associated with agitation, increased use of physical and chemical restraints, and
worsening pressure ulcers.

The preponderance of evidence does not support the use of tube feedings, based upon expert opinion and
extensive observational data. Published empirical work using observational data is highly consistent
regarding the lack of efficacy for tube feeding in this population.

Ref: Feeding tubes in advanced dementia position statement. American Geriatrics Society, 2013. 2) American Geriatrics Society:
Five things physicians and patients should question. ABIM Foundation Choosing Wisely campaign, 2014.

50
Item 145

ANSWER: A

Individuals on the National Weight Control Registry typically eat a low-fat diet rich in complex
carbohydrates, eat breakfast daily, weigh themselves at least once a week, and are physically active for
6090 minutes a day.

Ref: McKinney L, Skolnik N, Chrusch A: Diagnosis and Management of Obesity. AAFP Monograph, 2013, pp 1-24.

Item 146

ANSWER: D

The Ottawa Ankle Rules should be used to rule out fracture and prevent unnecessary radiographs.
According to these guidelines, ankle radiographs are needed if there is pain over the malleolus plus bony
tenderness over potential fracture areas, or an inability to bear weight and walk four steps immediately
after the injury and in the emergency department or physicians office (SOR A).

Ref: Tiemstra JD: Update on acute ankle sprains. Am Fam Physician 2012;85(12):1170-1176.

Item 147

ANSWER: A

Benign nocturnal limb pains of childhood (growing pains) occur in as many as one-third of children, most
often between 4 and 6 years of age. The etiology is unknown, but the course does not parallel pubescent
growth, as would be expected if bone growth were the source of the pain. Pain often awakens the child
within hours of falling asleep following an active day. It is generally localized around the knees, most often
in the shins and calves, but also may affect the thighs and the upper extremities. A characteristic history
coupled with a normal physical examination will confirm the diagnosis. Reassurance that no additional tests
or treatments are necessary and that the condition is self-limiting is the most appropriate response.

Ref: Junnila JL, Cartwright VW: Chronic musculoskeletal pain in children: Part I: Initial evaluation. Am Fam Physician
2006;74(1):115-122. 2) Kliegman RM, Stanton BF, Geme JW III, et al (eds): Nelson Textbook of Pediatrics, ed 19.
Elsevier Saunders, 2011, p 878.

Item 148

ANSWER: A

This patients presentation is consistent with community-acquired pneumonia (CAP). Pathogens commonly
involved include viruses such as influenza, as well as Mycoplasma pneumoniae and Streptococcus
pneumoniae. This patients history and findings are most consistent with early lobar pneumonia, given the
sputum production, presence of rales, and radiographic findings, and empiric antibiotic treatment is most
appropriate. His premorbid history of good health and the lack of findings such as confusion, tachypnea,
hypotension, or multilobar infiltrates that would indicate severe CAP make outpatient antibiotic treatment
the most appropriate option. He is outside of the time frame when anti-influenza treatments would be
expected to be effective, even if influenza seemed likely.

51
For previously healthy individuals who have not taken antibiotics in the previous 3 months the most
appropriate treatment for CAP is empiric treatment with an oral macrolide such as azithromycin,
clarithromycin, or erythromycin (level I evidence) or doxycycline (level III evidence). In the presence of
comorbidities such as diabetes, alcoholism, or chronic heart, lung, liver, or renal diseases, the treatment
of CAP should provide broader coverage with dual antibiotic treatment regimens including combinations
of fluoroquinolones, $-lactam drugs, and macrolide options, and hospitalization is often indicated.

Ref: Mandell LA, Wunderink RG, Anzueto A, et al: Infectious Diseases Society of America/American Thoracic Society
consensus guidelines on the management of community-acquired pneumonia in adults. Clin Infect Dis 2007;44(Suppl
2):S27-S72. 2) Watkins RR, Lemonovich TL: Diagnosis and management of community-acquired pneumonia in adults.
Am Fam Physician 2011;83(11):1299-1306.

Item 149

ANSWER: C

This patient presents with marked hyperglycemia but no evidence of ketoacidosis or nonketotic coma.
Differentiating between type 1 and type 2 diabetes mellitus is important for guiding therapy. The gradual
onset of symptoms is more consistent with type 2 diabetes mellitus, whereas type 1 diabetes typically has
a more rapid onset. Patients with type 1 diabetes typically need lower doses of insulin to correct
hyperglycemia, as they lack the insulin insensitivity that is the hallmark of type 2 diabetes. Positive
anti-GAD antibodies and low C-peptide at the time of the initial diagnosis are also consistent with type 1
diabetes, although C-peptide levels can also be low in long-standing type 2 diabetes. Weight loss occurs
in both types of diabetes mellitus when glucose is profoundly elevated.

Ref: Goldman L, Schafer AI (eds): Goldmans Cecil Medicine, ed 24. Elsevier Saunders, 2011, pp 236, 1475-1489.

Item 150

ANSWER: C

The American College of Chest Physicians and the American Thoracic Society recommend that for patients
discharged on supplemental home oxygen following hospitalization for an acute illness, the prescription
for home oxygen should not be renewed without assessing the patient for ongoing hypoxemia (SOR C).
The rationale for this recommendation is that hypoxemia often resolves after recovery from an acute
illness. The guidelines recommend that a plan be established to reassess the patient no later than 90 days
after discharge and that Medicare guidelines and evidence-based criteria should be followed to determine
whether the patient meets the criteria for supplemental oxygen.

Continuous oxygen therapy is indicated in patients with COPD and severe hypoxemia. There is good
evidence that the addition of home long-term continuous oxygen therapy for COPD increases survival rates
in patients with severe hypoxemia, defined as an oxygen saturation <90% or a PaO2 <8 kPa (60 mm Hg),
but not in patients with moderate hypoxemia or nocturnal desaturation.

Continuous supplemental oxygen should be used to improve exercise performance and survival in patients
with moderate to severe COPD who have severe daytime hypoxemia. The Centers for Medicare and
Medicaid Services (CMS) provides guidelines for supplemental oxygen therapy and sets the standard for
nearly all adult oxygen prescriptions. According to these standards, oxygen therapy is covered for patients
with a documented PaO2 55 mm Hg or an oxygen saturation 88% on room air at rest.

52
Ref: Bailey RE: Home oxygen therapy for treatment of patients with chronic obstructive pulmonary disease. Am Fam Physician
2004;70(5):864-865. 2) Grimes GC, Manning JL, Patel P, Via RM: Medications for COPD: A review of effectiveness.
Am Fam Physician 2007;76(8):1141-1148. 3) American College of Chest Physicians and American Thoracic Society: Five
things physicians and patients should question. ABIM Foundation Choosing Wisely campaign, 2014.

Item 151

ANSWER: B

Atypical antipsychotics may reduce the duration of delirium in adult intensive-care patients, and are
recommended by the American College of Critical Care Medicine in their clinical practice guidelines for
the management of pain, agitation, and delirium in adult patients in the intensive-care unit.

The American Geriatrics Society and the American Psychiatric Association (APA) recommend not using
antipsychotics as a first choice to treat the behavioral and psychological symptoms associated with
dementia, such as aggression and resistance to care. These drugs have limited benefit and can cause serious
harm, including stroke and premature death. The APA also recommends against routinely prescribing two
or more antipsychotic medications concurrently, and against routinely prescribing antipsychotic
medications as a first-line intervention for insomnia in adults.

Ref: Barr J, Fraser GL, Puntillo K, et al: Clinical practice guidelines for the management of pain, agitation, and delirium in adult
patients in the Intensive Care Unit: Executive summary. Am J Health Syst Pharm 2013;70(1):53-58. 2) American
Psychiatric Association: Five things physicians and patients should question. ABIM Foundation Choosing Wisely campaign,
2014. 3) American Geriatrics Society: Ten things physicians and patients should question. ABIM Foundation Choosing
Wisely campaign, 2014.

Item 152

ANSWER: A

The presence of palpable purpura along with a history of recent or current medication use is highly
suggestive of drug-induced vasculitis. Skin is the organ most often affected by drug-induced vasculitis,
with lesions predominantly affecting the lower extremities. Although clinical findings may be limited to
the skin, systemic manifestations such as fever, fatigue, and joint pains can be present. Offending drugs
include sulfonamides, allopurinol, thiazides, phenytoin, and penicillins. A skin biopsy will reveal the
presence of eosinophils. The first step in management includes discontinuation of the offending agent.
Patients with mild and nonlife-threatening small-vessel vasculitis should be treated with supportive care,
while those with more severe features should receive corticosteroids. There is no role for antihistamines,
plasmapheresis, or platelet transfusion in drug-induced vasculitis (SOR B).

Ref: Longo DL, Fauci AS, Kasper DL, et al (eds): Harrisons Principles of Internal Medicine, ed 18. McGraw-Hill, 2012, pp
2785-2801.

53
Item 153

ANSWER: B

Spironolactone is an aldosterone antagonist. This class of drugs has been found to reduce all-cause
mortality and cardiac death when initiated after a myocardial infarction in patients with a low left
ventricular ejection fraction (LVEF) and signs of heart failure. Guidelines from the American College of
Cardiology and the American Heart Association recommend the use of aldosterone blockers in patients
who have heart failure or diabetes mellitus, have an LVEF 40%, are receiving ACE inhibitors and
$-blockers, and have a serum potassium level <5.0 mEq/L (5.0 mmol/L) and a creatinine level >2.5
mg/dL in men or >2.0 mg/dL in women. None of the other medications listed has this level of evidence
to support its use.

Ref: Pitt B, White H, Nicolau J, et al; EPHESUS Investigators: Eplerenone reduces mortality 30 days after randomization
following acute myocardial infarction in patients with left ventricular systolic dysfunction and heart failure. J Am Coll
Cardiol 2005;46(3):425431. 2) Mercado MG, Smith DK, McConnon ML: Myocardial infarction: Management of the
subacute period. Am Fam Physician 2013;88(9):581-588. 3) Amsterdam EA, Wenger NK, Brindis RG, et al: 2014
AHA/ACC guideline for the management of patients with nonST-elevation acute coronary syndromes: A report of the
American College of Cardiology/American Heart Association Task Force on Practice Guidelines. Circulation 2014;Sep
23[Epub ahead of print].

Item 154

ANSWER: E

The patient described has an inflammatory myopathy of the polymyositis/dermatomyositis group. Proximal
muscle involvement and elevation of serum muscle enzymes such as creatine kinase and aldolase are
characteristic. Corticosteroids are the accepted treatment of choice.

It is extremely unlikely that Duchennes muscular dystrophy would present after age 30. In amyotrophic
lateral sclerosis, an abnormal neurologic examination with findings of upper motor neuron dysfunction is
characteristic. Patients with myasthenia gravis typically have optic involvement, often presenting as
diplopia. The predominant symptom of aseptic necrosis of the femoral head is pain rather than proximal
muscle weakness. Elevated muscle enzymes are not characteristic.

Ref: Longo DL, Fauci AS, Kasper DL, et al (eds): Harrisons Principles of Internal Medicine, ed 18. McGraw-Hill, 2012, pp
3509-3518.

Item 155

ANSWER: B

Uterotonics are the first-line treatment for postpartum hemorrhage in patients with decreased uterine tone.
While all of the uterotonic agents listed are options for the management of postpartum hemorrhage,
methylergonovine should be avoided if the patient is hypertensive. It is an ergot alkaloid that causes
generalized smooth muscle contraction and can raise blood pressure. Oxytocin and misoprostol do not have
any contraindications. Carboprost tromethamine should be avoided in asthmatic patients and is relatively
contraindicated if the patient has hepatic, renal, or cardiac disease.

Ref: Gabbe SG, Niebyl JR, Simpson JL, et al (eds): Obstetrics: Normal and Problem Pregnancies, ed 6. Saunders Elsevier,
2012, pp 427-428.

54
Item 156

ANSWER: D

Ulcerative colitis (UC) is a lifelong condition that results in a waxing and waning autoimmune
inflammation of the colon. Clinical symptoms are inadequate for assessing the need for ongoing therapy.
For this reason, once a patient with UC has achieved remission with a specific medication, that medication
should be continued indefinitely unless the disease resurfaces. Sulfasalazine is one of the most effective
agents for this purpose, is usually well tolerated, and is considered first-line therapy for ulcerative colitis.
There is no apparent reason to consider a higher order of therapy (azathioprine) in this patient or to refer
her for colectomy. Patients with UC who have had a history of moderate or extensive involvement of the
colon, however, are at markedly increased risk for the development of colorectal cancer. Current
guidelines recommend beginning screening colonoscopy 10 years after the initial diagnosis and continuing
every 25 years, with the interval based on the findings.

Ref: Adams SM, Bornemann PH: Ulcerative colitis. Am Fam Physician 2013;87(10):699-705. 2) Ford AC, Moayyedi P,
Hanauer SB: Ulcerative colitis. BMJ 2013;346:f432.

Item 157

ANSWER: A

Tramadol lowers the seizure threshold and should be avoided in patients with seizures. It is considered a
second-line treatment for mild to moderate pain (SOR B). A history of heart failure, ventricular
dysrhythmias, or hypertension is not a contraindication to its use.

Ref: Blondell RD, Azadfard M, Wisniewski AM: Pharmacologic therapy for acute pain. Am Fam Physician
2013;87(11):766-772.

Item 158

ANSWER: B

Suspected COPD should be confirmed by spirometry in stable patients, based on a postbronchodilator


FEV1/FVC ratio <70% of predicted (SOR C). While guidelines for the treatment of COPD differ slightly
among the leading national and international organizations, most have come to a consensus on using this
criterion for the diagnosis (SOR C).

Ref: Stephens MB, Yew KS: Diagnosis of chronic obstructive pulmonary disease. Am Fam Physician 2008;78(1):87-92. 2)
Qaseem A, Wilt TJ, Weinberger SE, et al: Diagnosis and management of stable chronic obstructive pulmonary disease:
A clinical practice guideline update from the American College of Physicians, American College of Chest Physicians,
American Thoracic Society, and European Respiratory Society. Ann Intern Med 2011;155(3):179-191. 3) Gruffydd-Jones
K, Loveridge C: The 2010 NICE COPD guidelines: How do they compare with the GOLD guidelines? Prim Care Respir
J 2011;20(2):199-204. 4) Lee H, Kim J, Tagmazyan K: Treatment of stable chronic obstructive pulmonary disease: The
GOLD guidelines. Am Fam Physician 2013;88(10):655-663. 5) Global Strategy for the Diagnosis, Management and
Prevention of COPD. Global Initiative for Chronic Obstructive Lung Disease (GOLD), 2014.

55
Item 159

ANSWER: E

The concentration of lithium into the thyroid gland inhibits iodine uptake, iodotyrosine coupling, and
thyroid hormone secretion. Thyroglobulin structure is also affected by lithium. The effect can be
significant enough to produce a state of hypothyroidism and/or goiter, and studies have shown that as many
as two-thirds of patients develop hypothyroidism within 10 years of beginning lithium treatment. Routine
monitoring of TSH and T4 every 612 months is a recommended standard for all patients receiving lithium
treatment. Lithium administration would not be expected to directly affect any of the other blood levels
listed, so the indications for obtaining these tests are the same as for other patients.

Ref: Bocchetta A, Loviselli A: Lithium treatment and thyroid abnormalities. Clin Pract Epidemiol Ment Health 2006;2:23. 2)
The Management of Bipolar Disorder Working Group: VA/DoD Clinical Practice Guideline for management of bipolar
disorder in adults. Department of Veteran Affairs, 2010.

Item 160

ANSWER: A

Guidelines from the Infectious Diseases Society of America recommend treatment of acute uncomplicated
cystitis based on the patients symptoms rather than documented evidence of infection (SOR C). In healthy
premenopausal women with no history of a urinary tract functional abnormality, current pregnancy, or
another underlying condition that may increase the risk of treatment failure, infection can be presumed
based on symptoms. Patients with acute uncomplicated cystitis are not febrile and have no vaginal
symptoms. Men and children, as well as women who do not meet these criteria, require in-person
evaluation.

A urine dipstick has relatively low sensitivity and specificity for urinary tract infection (SOR A). Negative
dipstick results do not reliably rule out infection in a patient with strongly positive symptoms. A midstream
culture is as good as or better than a urinary catheterobtained specimen (SOR B). However, neither of
these is required for the diagnosis or treatment of uncomplicated acute cystitis.

Ref: Hooton TM: Uncomplicated urinary tract infection. N Engl J Med 2012;366(11):1028-1037. 2) Hooton TM, Roberts PL,
Cox ME, Stapleton AE: Voided midstream urine culture and acute cystitis in premenopausal women. N Engl J Med
2013;369(20):1883-1891.

Item 161

ANSWER: D

Thyroid nodules are often detected either during a clinical examination or incidentally on an imaging study.
The first step in the evaluation of a thyroid nodule is to order a TSH level. If the TSH level is suppressed,
radionuclide scintigraphy should be ordered to rule out a hyperfunctioning nodule. If the TSH level is
either normal or high, the current recommendation is to biopsy only nodules >1 cm. Clinical follow-up
is recommended for nodules 1 cm.

Ref: Knox MA: Thyroid nodules. Am Fam Physician 2013;88(3):193-196.

56
Item 162

ANSWER: C

This patient has heavy menstrual bleeding, associated anemia, and morbid obesity, all of which need to
be taken into consideration when choosing contraception. Medroxyprogesterone acetate can contribute to
weight gain and thus should not be the first choice in this individual. The norelgestromin/ethinyl estradiol
transdermal system is not recommended in patients with a weight over 90 kg and thus is not an option for
this patient. The levonorgestrel-releasing intrauterine system would be the best option, given the associated
significant decrease in menstrual blood loss after the first 3 months of insertion and equal effectiveness in
obese and non-obese patients. Although a diaphragm is an option, it will not decrease her menstrual blood
loss.

Ref: Mansour D, Korver T, Marintcheva-Petrova M, Fraser IS: The effects of Implanon on menstrual bleeding patterns. Eur
J Contracept Reprod Health Care 2008;13(Suppl 1):13-28. 2) Kaneshiro B, Edelman A: Contraceptive considerations in
overweight teens. Curr Opin Obstet Gynecol 2011;23(5):344-349. 3) Xu H, Wade JA, Peipert JF, et al: Contraceptive
failure rates of etonogestrel subdermal implants in overweight and obese women. Obstet Gynecol 2012;120(1):21-26.

Item 163

ANSWER: E

Most watery diarrhea is self-limited and testing is not indicated. A diagnostic workup is usually reserved
for patients with severe dehydration or illness, diarrhea persisting for more than 37 days, fever, bloody
stool, immunosuppression, or a history suggesting nosocomial infection or an outbreak. Indiscriminate use
of laboratory testing is inefficient and not cost-effective.

Ref: Barr W, Smith A: Acute diarrhea. Am Fam Physician 2014;89(3):180-189.

Item 164

ANSWER: C

Statins are the drugs of choice to reduce perioperative cardiovascular risks (level of evidence 1). In
addition to lowering cholesterol, they also reduce vascular inflammation, improve endothelial function,
and stabilize atherosclerotic plaques. For the most protection, statins should be started 4 weeks prior to
the procedure and continued after surgery (SOR A). $-Blockers and aspirin are beneficial, but less so than
statins.

Ref: Holt NF: Perioperative cardiac risk reduction. Am Fam Physician 2012;85(3):239-246.

Item 165

ANSWER: A

Acetaminophen is the analgesic of choice for short-term treatment of mild to moderate pain in patients with
stage 35 chronic kidney disease. Chronic nonterminal pain requires initial treatment with nonopioid
analgesics. NSAIDs should be avoided because of the risk of nephrotoxicity.

Ref: Baumgarten M, Gehr T: Chronic kidney disease: Detection and evaluation. Am Fam Physician 2011;84(10):1138-1148.
2) Berland D, Rodgers P: Rational use of opioids for management of chronic nonterminal pain. Am Fam Physician
2012;86(3):252-258.

57
Item 166

ANSWER: C

Anxiety, shortness of breath, paresthesia, and carpopedal spasm are characteristic of hyperventilation.
Respiratory alkalosis secondary to hyperventilation is diagnosed when arterial pH is elevated and pCO2
is depressed. Low pH is characteristic of acidosis, either respiratory or metabolic, and elevated pH with
elevated pCO2 is characteristic of metabolic alkalosis with respiratory compensation.

Ref: Longo DL, Fauci AS, Kasper DL, et al (eds): Harrisons Principles of Internal Medicine, ed 18. McGraw-Hill, 2012, p
2185.

Item 167

ANSWER: B

This patient suffers from secondary amenorrhea (defined as the cessation of regular menses for 3 months
or irregular menses for 6 months). The most common causes of secondary amenorrhea are polycystic
ovary syndrome, primary ovarian failure, hypothalamic amenorrhea, and hyperprolactinemia. With a
normal physical examination, negative pregnancy test, and no history of chronic disease, a hormonal
workup is indicated, including TSH, LH, and FSH levels (SOR C).

A hormonal challenge with medroxyprogesterone to provoke withdrawal bleeding is used to assess


functional anatomy and estrogen levels (SOR C). However, it has poor specificity and sensitivity for
ovarian function and a poor correlation with estrogen levels.

Pelvic ultrasonography is indicated in the workup of primary amenorrhea to confirm the presence of a
uterus and detect structural abnormalities of the reproductive organs. Likewise, karyotyping can be used
for patients with primary amenorrhea, as conditions such as Turners syndrome and androgen insensitivity
syndrome are due to chromosomal abnormalities.

A CBC and metabolic panel would not be initial considerations in the workup of amenorrhea unless the
patient has a known chronic disease which may affect the results.

Ref: Klein DA, Poth MA: Amenorrhea: An approach to diagnosis and management. Am Fam Physician 2013;87(11):781-788.

Item 168

ANSWER: D

In the frail elderly, it may be difficult to distinguish relative starvation due to decreased or inadequate
caloric intake from cachexia, which is due to an inflammatory response with elevated cytokines. Appetite
is decreased early in cases of cachexia but remains normal in the early stages of starvation. Likewise,
albumin decreases early in cases of cachexia and later in starvation. Due to the inflammatory changes,
cachexia is resistant to refeeding.

Ref: Thomas DR: Distinguishing starvation from cachexia. Clin Geriatr Med 2002;18(4):883-891. 2) Messinger-Rapport BJ,
Gammack JK, Thomas DR, Morley JE: Clinical update on nursing home medicine: 2013. J Am Med Dir Assoc
2013;14(12):860-876.

58
Item 169

ANSWER: A

This patients symptoms are most consistent with a viral upper respiratory infection. There is no curative
treatment so management should be focused on symptoms. Most over-the-counter cough and cold
preparations, including the ingredients dextromethorphan and diphenhydramine, have no evidence of
benefit and carry a risk of harm in children and should not be recommended. Albuterol is only helpful for
cough in patients with wheezing. Buckwheat honey has limited evidence of effectiveness but appears to
carry no risk of harm and may be recommended for symptom management.

Ref: Fashner J, Ericson K, Werner S: Treatment of the common cold in children and adults. Am Fam Physician
2012;86(2):153-159.

Item 170

ANSWER: E

Localized or generalized lymphadenopathy is a common complaint in children. Most cases are benign and
related to infections or connective tissue disorders. Initial management involves watchful waiting for up
to 4 weeks (SOR C). Evidence to support treatment with antibiotics is lacking and should be reserved for
patients who show evidence of local inflammation. Malignancy should be excluded if lymphadenopathy
persists beyond 4 weeks, or if other symptoms develop, such as fever, night sweats, weight loss,
hepatosplenomegaly, or orthopnea. In this situation, the evaluation should include a CBC, blood smear,
erythrocyte sedimentation rate, and chest radiography (SOR C).

Ref: Fragkandrea I, Nixon JA, Panagopoulou P: Signs and symptoms of childhood cancer: A guide for early recognition. Am
Fam Physician 2013;88(3):185-192.

Item 171

ANSWER: B

In Southeast Asia, there is growing resistance of Campylobacter to fluoroquinolones, making azithromycin


the treatment of choice. Typical treatment for adults is 500 mg daily for 3 days and in children 10
mg/kg/day for 35 days. Rifaximin can be used for noninvasive travelers diarrhea but this patients
symptoms are most consistent with an invasive travelers diarrhea. Metronidazole is used for Giardia and
Entamoeba histolytica but not for Campylobacter. Ciprofloxacin can be used for Salmonella and Shigella.

Ref: Ross AG, Olds GR, Cripps AW, et al: Enteropathogens and chronic illness in returning travelers. N Engl J Med
2013;368(19):1817-1825. 2) Nair D: Travelers diarrhea: Prevention, treatment, and post-trip evaluation. J Fam Pract
2013;62(7):356-361.

59
Item 172

ANSWER: E

All oral NSAIDs increase the risk of myocardial infarction (relative risk versus placebo from 1.5 for
ibuprofen to 1.7 for celecoxib), with the exception of naproxen. Cardiac risks are greater in older patients,
those with a history of cardiac events, and with higher dosages.

Ref: Fogleman CD: Analgesics for osteoarthritis. Am Fam Physician 2013;87(5):354-356.

Item 173

ANSWER: C

Gonorrhea continues to be a major cause of reproductive complications in women. Effective treatment is


critical to control the spread of disease. Unfortunately, treatment has been complicated by the ability of
Neisseria gonorrhoeae to develop resistance. The emergence of fluoroquinolone-resistant gonorrhea has
led the Centers for Disease Control and Prevention to no longer recommend the use of fluoroquinolones
for treatment of gonorrhea. Cephalosporins are currently the only recommended antibiotic, with
ceftriaxone being the preferred agent.

Patients with gonorrhea often have a Chlamydia infection as well. It is recommended that they be treated
for both, even with a negative test for Chlamydia. The recommended regimen for treatment of gonorrhea
is ceftriaxone, 250 mg intramuscularly, and azithromycin, 1 g orally, or doxycycline, 100 mg twice daily
for 7 days. All sexual partners in the previous 60 days should be treated as well.

Ref: Centers for Disease Control and Prevention (CDC): Update to CDCs Sexually Transmitted Diseases Treatment Guidelines,
2010: Oral cephalosporins no longer a recommended treatment for gonococcal infections. MMWR Morb Mortal Wkly Rep
2012;61(31):590-594.

Item 174

ANSWER: A

Oral antibiotics are preferred for community-acquired pneumonia if the patient is able to take them. The
antibiotic of choice is amoxicillin for children 60 days to 5 years of age because of its activity against
Streptococcus pneumoniae. Azithromycin is the antibiotic of choice for children 516 years old because
of its activity against Mycoplasma pneumoniae and Chlamydia pneumoniae, and it can be used in children
between the ages of 60 days and 5 years who are allergic to penicillin. Ceftriaxone is not a preferred
antibiotic.

Ref: Stuckey-Schrock K, Hayes BL, George CM: Community-acquired pneumonia in children. Am Fam Physician
2012;86(7):661-667.

60
Item 175

ANSWER: A

This patient has a diagnosis of laryngitis (a viral infection causing inflammation of the vocal cords lasting
less than 3 weeks). Symptoms of laryngitis can include loss/muffling of the voice, as well as other classic
symptoms of an upper respiratory tract infection. A Cochrane study has shown that antibiotic therapy does
not decrease the duration of laryngitis symptoms or hasten the return of vocal patency (SOR A). The U.S.
Food and Drug Administration no longer recommends the use of over-the-counter cough and cold
medications for children under the age of 2 years, because of their serious and potentially life-threatening
adverse effects, including respiratory arrest due to hypopnea (SOR B).

Ref: Salisbury-Afshar E: Oral antihistamine/decongestant/analgesic combinations for the common cold. Am Fam Physician
2012;86(9):812-813. 2) Zoorob R, Sidani MA, Fremont RD, Kihlberg C: Antibiotic use in acute upper respiratory tract
infections. Am Fam Physician 2012;86(9):817-822. 3) Lambert M: Practice guidelines: IDSA releases guidelines for
management of acute bacterial rhinosinusitis. Am Fam Physician 2013;87(6):445-449.

Item 176

ANSWER: C

The signs of retinal vein occlusion typically include sudden painless loss of vision or distortion of vision.
Redness is not typical and should cause the clinician to suspect an alternate diagnosis. Tortuous and dilated
retinal veins are the most common finding on funduscopic examination. Patients also often have multiple
cotton-wool spots, although these are not specific to retinal vein occlusion. An afferent pupillary defect
often occurs on the affected side. Diabetes mellitus and hypertension are both risk factors for retinal vein
occlusion, increasing the likelihood in this patient.

Ref: Tran KT, Qualm AS, Shannon MA: Retinal changes and visual impairment. Am Fam Physician 2010;81(1):73. 2) Kiire
CA, Chong NV: Managing retinal vein occlusion. BMJ 2012;344:e499.

Item 177

ANSWER: C

The patients history and physical findings are all consistent with a keloid, which is a benign overgrowth
of scar tissue at sites of trauma to the skin, such as acne, burns, surgery, ear piercing, tattoos, and
infections. Common locations include the earlobes, jawline, nape of the neck, scalp, chest, and back.
Lesions are sometimes asymptomatic, but often are associated with hypersensitivity, pain, and pruritus.
The incidence is higher in blacks, Hispanics, and Asians. Intralesional corticosteroid injections are
first-line therapy. Silicone gel sheeting, topical imiquimod, and intralesional fluorouracil can be used when
first-line therapy fails, but these methods are more often associated with recurrence. Laser therapy and
surgical excision are associated with a high rate of recurrence when used as monotherapy.

Ref: Kundu RV, Patterson S: Dermatologic conditions in skin of color: Part II. Disorders occurring predominately in skin of
color. Am Fam Physician 2013;87(12):859-865.

61
Item 178

ANSWER: A

Dextromethorphan, diphenhydramine, ibuprofen, and even fluoroquinolones are among the many agents
that can cause a false-positive urine drug screen for opioids. Pseudoephedrine can cause a false-positive
test for amphetamines (SOR A).

Ref: Tessier J, Downen M, Engel-Brower J, et al: Pitfalls & pearls for 8 common lab tests. J Fam Pract 2014;63(4):198-205.

Item 179

ANSWER: C

Bartonella henselae is the organism that causes cat-scratch disease. IgG titers over 1:256 strongly suggest
active or recent infection. IgM elevation suggests acute disease but production of IgM is brief. Lymph
node biopsy is reserved for cases where node swelling fails to resolve or the diagnosis is uncertain. The
organism is difficult to culture and cultures are not recommended. Nontuberculous mycobacteria do not
cause cat-scratch disease (SOR C).

Ref: Klotz SA, Ianas V, Elliott SP: Cat-scratch disease. Am Fam Physician 2011;83(2):152-155.

Item 180

ANSWER: D

The National Kidney Foundation defines chronic kidney disease (CKD) as a glomerular filtration rate
(GFR) <60 mL/min/1.73 m2, or evidence of kidney damage with or without a decreased GFR, for 3 or
more months (SOR C). Individuals with CKD are at increased risk for serious cardiovascular disease
events, especially if they have other comorbid conditions such as hypertension or diabetes mellitus.

It is important to have a multifactorial strategy for this patient that includes controlling his blood glucose
and blood pressure in addition to continuing his antiplatelet therapy (SOR A). According to the U.S. Food
and Drug Administration, metformin is contraindicated when a patients serum creatinine is >1.5 mg/dL
in men or >1.4 mg/dL in women (SOR C). This recommendation is based on the fact that metformin is
cleared by the kidneys, and when the GFR is <60 mL/min/1.73 m2, the clearance of metformin decreases
by 75%. Because lactic acidosis caused by metformin is rare, some experts advocate for continued
metformin use with higher serum creatinine levels. However, most recommendations agree it should be
stopped when the GFR falls below 30 mL/min/1.73 m2 (SOR C). This patient should stop his metformin,
and his insulin should be adjusted appropriately to achieve optimal glycemic control. It is also important
that he continue his antihypertensive and antiplatelet medications.

Ref: Salpeter SR, Greyber E, Pasternak GA, Salpeter EE: Risk of fatal and nonfatal lactic acidosis with metformin use in type
2 diabetes mellitus. Cochrane Database Syst Rev 2010;(4):CD002967. 2) Lipska KJ, Bailey CJ, Inzucchi SE: Use of
metformin in the setting of mild-to-moderate renal insufficiency. Diabetes Care 2011;34(6):1431-1437. 3) Nye HJ,
Herrington WG: Metformin: The safest hypoglycaemic agent in chronic kidney disease? Nephron Clin Pract
2011;118(4):c380-c383. 4) National Kidney Foundation: KDOQI clinical practice guideline for diabetes and CKD: 2012
update. Am J Kidney Dis 2012;60(5):850-886. 5) Rivera JA, OHare AM, Harper GM: Update on the management of
chronic kidney disease. Am Fam Physician 2012;86(8):749-754.

62
Item 181

ANSWER: C

The radiograph shows a distal radial fracture (Colles fracture) with a minimal amount of displacement or
impaction. Such fractures can be treated with immobilization for 48 weeks. According to a Cochrane
review, there is insufficient evidence from randomized trials to determine which methods of conservative
treatment are most appropriate for the more common types of distal radial fractures in adults.The use of
a single sugar-tong splint or a short arm cast is recommended for these fractures. Volar or dorsal splints
are generally not recommended, as they do not restrict pronation and supination. Orthopedic referral is
recommended in the presence of intra-articular fractures (radiocarpal, distal radioulnar), carpal bone
injuries, and dislocation of the distal radioulnar joint. The other splints listed are not recommended for
immobilizing distal radial fractures (SOR B).

Ref: Handoll HH, Madhok R: Conservative interventions for treating distal radial fractures in adults. Cochrane Database Syst
Rev 2003;(2):CD000314. 2) Black WS, Becker JA: Common forearm fractures in adults. Am Fam Physician
2009;80(10):1096-1102. 3) Boyd AS, Benjamin HJ, Asplund C: Splints and casts: Indications and methods. Am Fam
Physician 2009;80(5):491-499.

Item 182

ANSWER: A

Fiber is ineffective in the treatment of adult irritable bowel syndrome (IBS) (SOR A). Symptoms do
improve, however, with several different medications and alternative therapies. Exercise, probiotics,
antibiotics, antispasmodics, antidepressants, psychological treatments, and peppermint oil all have evidence
that they may improve IBS symptoms (SOR B). A Cochrane review of 15 studies involving 922 patients
found a beneficial effect from antidepressants with regard to improvement in pain and overall symptom
scores compared to placebo. SSRIs used in these trials included citalopram, fluoxetine, and paroxetine,
and tricyclic antidepressants included amitriptyline, desipramine, and imipramine. Buspirone, clonazepam,
divalproex sodium, and risperidone have not been shown to be effective for symptom relief in IBS patients.

Ref: Wilkins T, Pepitone C, Alex B, Schade RR: Diagnosis and management of IBS in adults. Am Fam Physician
2012;86(5):419-426.

Item 183

ANSWER: A

The diagnosis of pelvic inflammatory disease (PID) is based primarily on the clinical evaluation.
Significant consequences can occur if treatment is delayed. Physicians should therefore treat on the basis
of clinical judgment without waiting for confirmation from laboratory or imaging tests (SOR B). No single
symptom, physical finding, or laboratory test is sensitive or specific enough to definitively diagnose PID
(SOR C). Clinical diagnosis alone based on the history, physical examination, and office laboratory results
is 87% sensitive, transvaginal ultrasonography is 30% sensitive, and laparoscopy is 81% sensitive but
unnecessarily invasive and not cost-effective. A study examining the diagnostic performance of CT in acute
PID concluded that the overall sensitivity of CT is poor.

Ref: Jung SI, Kim YJ, Park HS, et al: Acute pelvic inflammatory disease: Diagnostic performance of CT. J Obstet Gynaecol
Res 2011;37(3):228-235. 2) Gradison M: Pelvic inflammatory disease. Am Fam Physician 2012;85(8):791-796.

63
Item 184

ANSWER: E

Stimulants are preferred over nonstimulant medications for adults with attention-deficit disorder. Stimulant
medications can aggravate psychosis, tics, or hypertension and are therefore contraindicated in patients
with these problems. The main side effects of these drugs include insomnia, dry mouth, weight loss,
headaches, and anxiety. They are classified as schedule II drugs due to their potential for abuse. The risk
for serious adverse cardiovascular events is very low, although these drugs can increase resting heart rate
and elevate both systolic and diastolic blood pressure.

Ref: Volkow ND, Swanson JM: Adult attention deficit-hyperactivity disorder. N Engl J Med 2013;369(20):1935-1944.

Item 185

ANSWER: E

Elevated blood pressure may have a protective effect in the initial period after an ischemic stroke, and
studies have shown adverse outcomes when it is lowered in the acute period. Blood pressure usually will
spontaneously decrease without treatment in the first several hours after presentation, and antihypertensive
treatment should not be started in the first 24 hours after an acute stroke unless blood pressure exceeds
220/120 mm Hg, or treatment is warranted because of another medical condition such as acute myocardial
infarction. Tighter blood pressure control becomes more important after the first 24 hours.

Ref: Bernheisel CR, Schlaudecker JD, Leopold K: Subacute management of ischemic stroke. Am Fam Physician
2011;84(12):1383-1388.

Item 186

ANSWER: B

Fortunately, emergency tracheotomy is not often necessary, but should one be necessary the best site for
the incision is directly above the cricoid cartilage, through the cricothyroid membrane. Strictly speaking,
this is not a tracheotomy, because it is actually above the trachea. However, it is below the vocal cords
and bypasses any laryngeal obstruction. The thyrohyoid membrane lies well above the vocal cords, making
this an impractical site. The area directly below the cricoid cartilagewhich includes the second, third,
and fourth tracheal rings, as well as the thyroid isthmusis the preferred tracheotomy site under controlled
circumstances, but excessive bleeding and difficulty finding the trachea may significantly impede the
procedure in an emergency.

Ref: Tintinalli JE, Kelen GD, Stapczynski JS (eds): Emergency Medicine: A Comprehensive Study Guide, ed 7. McGraw-Hill,
2011, pp 209-215. 2) Marx JA, Hockberger RS, Walls RM (eds): Rosens Emergency Medicine: Concepts and Clinical
Practice, ed 8. Elsevier Saunders, 2013, pp 8-10.

64
Item 187

ANSWER: B

Thiazide diuretics have proven efficacy in the treatment of hypertension in all age groups and sexes. When
used as antihypertensive agents, the reduction in adverse cardiovascular outcomes equals that of
$-blockers, calcium channel blockers, and ACE inhibitors. Successful thiazide treatment of hypertension
is especially effective in preventing heart failure or strokes. Unlike the other options listed, thiazide
diuretics have also been shown to slow cortical bone loss in postmenopausal females and to reduce the
incidence of osteoporosis and hip fractures in those who take it continuously. This protective beneficial
side effect disappears within 4 months following discontinuation of thiazide therapy. As with all
medications there are potential disadvantages of thiazide use, including excessive urinary losses of
potassium and sodium and possible increases in serum glucose levels.

Ref: Grossman E, Verdecchia P, Shamiss A, et al: Diuretic treatment of hypertension. Diabetes Care 2011;34(Suppl
2):S313-S319.

Item 188

ANSWER: A

This presentation is typical of either transient synovitis or septic arthritis of the hip. Because the conditions
have very different treatment regimens and outcomes, it is important to differentiate the two. It is
recommended that after plain films, the first studies to be performed should be a CBC and an erythrocyte
sedimentation rate (ESR). Studies have shown that septic arthritis should be considered highly likely in a
child who has a fever >38.7C (101.7F), refuses to bear weight on the affected leg, has a WBC count
>12,000 cells/mm3, and has an ESR >40 mm/hr. If several or all of these conditions exist, aspiration
of the hip guided by ultrasonography or fluoroscopy should be performed by an experienced practitioner.
MRI may be helpful when the diagnosis is unclear based on the initial evaluation, or if other etiologies
need to be excluded.

Ref: Sawyer JR, Kapoor M: The limping child: A systematic approach to diagnosis. Am Fam Physician 2009;79(3):215-224.

Item 189

ANSWER: C

Improving quality and efficiency in medicine is best done using a systems approach. One of the tools for
improving a system of care is to utilize the Plan-Do-Check-Act cycle of continuous quality improvement.
This cycle includes a planning stage to identify an improvement strategy. An implementation stage is
followed by a time of evaluation for effectiveness. Finally, a decision is made to adopt or abandon the
initial strategy. The cycle is then repeated as many times as necessary to attain the desired results.

Ref: Longo DL, Fauci AS, Kasper DL, et al (eds): Harrisons Principles of Internal Medicine, ed 18. McGraw-Hill, 2012, pp
85-90.

65
Item 190

ANSWER: B

The American College of Gastroenterology recommends transabdominal ultrasonography for all patients
with acute pancreatitis (strong recommendation, low quality evidence). Contrast-enhanced CT and MRI
should be reserved for patients who have an unclear diagnosis, are not clinically improving after 4872
hours, or develop complications.

Ref: Panebianco NL, Jahnes K, Mills AM: Imaging and laboratory testing in acute abdominal pain. Emerg Med Clin North Am
2011;29(2):175-193. 2) Tenner S, Baillie J, DeWitt J, et al: American College of Gastroenterology guideline: Management
of acute pancreatitis. Am J Gastroenterol 2013;108(9):1400-1415.

Item 191

ANSWER: A

Nausea and vomiting are common in early pregnancy and can affect the health of both the pregnant woman
and the fetus. Mild cases of nausea and vomiting can be controlled with lifestyle and dietary changes. It
is recommended that patients eat frequent small meals consisting of dry and bland foods, avoid spicy and
fatty foods, eat high-protein snacks, and eat crackers in the morning before rising. If the nausea and
vomiting are not controlled with dietary modifications, the first-line treatment is vitamin B6, 1025 mg,
and doxylamine, 12.5 mg, 34 times per day. The other antiemetics can also be used in pregnancy, but
vitamin B6 and doxylamine should be tried first because of the balance of safety and efficacy.

Ref: American College of Obstetricians and Gynecologists: Nausea and vomiting of pregnancy. ACOG Practice Bulletin no 52,
2004 (reaffirmed 2013).

Item 192

ANSWER: E

Women over the age of 50 require at least 1200 mg of calcium a day. Inadequate calcium intake is
common, particularly in older women, and is associated with increased bone loss and an increased fracture
risk. Supplements should be considered when dietary intake is inadequate. Calcium carbonate is the least
expensive and most commonly used supplement, but it is constipating and stomach acid improves its
absorption. Calcium gluconate and calcium lactate are rarely used for fracture prevention or calcium
supplementation. Bone meal and oyster shell calcium are primarily composed of calcium carbonate but may
contain lead or other contaminants since they are derived from natural sources. In addition, their absorption
without stomach acid is erratic.

Calcium citrate is less dependent on stomach acidity for absorption and it may be used with long-term
gastric acid suppression agents. It may be taken without regard to food or meals.

Ref: Bauer DC: Calcium supplements and fracture prevention. N Engl J Med 2013;369(16):1537-1543.

66
Item 193

ANSWER: D

This patient has severe hyponatremia manifested by confusion and seizures, a life-threatening situation
warranting urgent treatment with hypertonic (3%) saline. The serum sodium level should be raised by only
12 mEq/dL per hour, to prevent serious neurologic complications. Saline should be used only until the
seizures stop. Some authorities recommend concomitant use of furosemide, especially in patients who are
likely to be volume overloaded, as this patient is, but it should not be used alone. The arginine vasopressin
antagonist conivaptan is approved for the treatment of euvolemic or hypervolemic hyponatremia, but not
in patients who are obtunded or in a coma, or who are having seizures.

Ref: Ellison DH, Berl T: The syndrome of inappropriate antidiuresis. N Engl J Med 2007;356(20):2064-2072. 2) Longo DL,
Fauci AS, Kasper DL, et al (eds): Harrisons Principles of Internal Medicine, ed 18. McGraw-Hill, 2012, pp 348-349,
2910-2911.

Item 194

ANSWER: E

Bupropion should not be used with MAO inhibitors or in patients with seizure or eating disorders. The
other medical conditions listed are not contraindications, and bupropion can be used for the treatment of
depression.

Ref: Fiore MC, Baker TB: Treating smokers in the health care setting. N Engl J Med 2011;365(13):1222-1231.

Item 195

ANSWER: B

Falls are the leading cause of injury in adults age 65 and older. Between 30% and 40% of community-
dwelling adults in this age group fall at least once per year.

Ref: Moyer VA; US Preventive Services Task Force: Prevention of falls in community-dwelling older adults: US Preventive
Services Task Force recommendation statement. Ann Intern Med 2012;157(3):197-204.

Item 196

ANSWER: C

It is estimated that particulate air pollution caused 100,000 deaths in the United States in 2010. The United
States Environmental Protection Agency (EPA) publishes air quality data in the form of the Air Quality
Index (AQI), which is calculated based on measured levels of the five major regulated air pollutants
(surface ozone, particulate matter, sulfur dioxide, carbon monoxide, and nitrogen dioxide). AQI is
reported as a number from 1 to 300, with corresponding colors ranging from green (150 or good) to
maroon (200300 or very unhealthy). Levels of particulate air pollution are consistently associated with
exacerbations of cardiovascular and pulmonary disease, as well as other illnesses. Physicians should
counsel people with chronic heart and lung disease like this patient to avoid heavy or prolonged exertion
when AFI levels are >100 (SOR C).

67
Ozone production is increased by heat, sunlight, and humidity and is generally worse in the warmer
months. Motorized vehicles are known to produce carbon monoxide, nitrogen dioxide, and particulate
matter, and these pollutants are known to be present at higher concentrations near busy roads. California
has responded to this concern by banning construction of schools within 500 feet of major highways.
Recirculating air conditioning is likely to reduce indoor exposure to ambient air pollution when levels are
unsafe outside.

Ref: Laumbach RJ: Outdoor air pollutants and patient health. Am Fam Physician 2010;81(2):175-180. 2) Abelsohn A, Stieb
DM: Health effects of outdoor air pollution: Approach to counseling patients using the Air Quality Health Index. Can Fam
Physician 2011;57(8):881-887. 3) US Burden of Disease Collaborators: The state of US health, 19902010: Burden of
diseases, injuries, and risk factors. JAMA 2013;310(6):591-608.

Item 197

ANSWER: C

Naltrexone is given in 380-mg monthly injections for the chronic treatment of alcohol and opioid
dependence. It is an antagonist of :-opioid receptors, and blocks the pain relief properties of opioid
agonists. Regional anesthesia and/or nonopioid analgesics are indicated when urgent pain relief is needed
in a patient on long-term naltrexone therapy. Ketorolac is the only medication listed that is not an opioid
agonist.

Ref: Shapiro B, Coffa D, McCance-Katz EF: A primary care approach to substance misuse. Am Fam Physician
2013;88(2):113-121.

Item 198

ANSWER: D

This patient has symptoms and signs consistent with a spinal cord lesion. With rapidly progressing
symptoms of paraparesis and sensory abnormalities in a diabetic patient, an epidural abscess is likely. This
is considered a neurologic emergency, because a permanent deficit can result if the problem is not
addressed promptly. MRI of the thoracic spine, which is the level of her deficits, is the best next step in
the evaluation of this patient. Intravenous antibiotics are a necessary treatment, along with surgical
debridement, if the MRI demonstrates the underlying problem. Thoracic spinal films are not helpful in this
situation. Neither antiplatelet therapy nor corticosteroids are indicated for a spinal epidural abscess.

Ref: Daroff RB, Fenichel GM, Jankovic J, Mazziotta JC (eds): Bradleys Neurology in Clinical Practice, ed 6. Elsevier
Saunders, 2012, pp 1263-1265. 2) Longo DL, Fauci AS, Kasper DL, et al (eds): Harrisons Principles of Internal
Medicine, ed 18. McGraw-Hill, 2012, p 3370.

Item 199

ANSWER: A

Echocardiography is the best study for detecting right ventricular (RV) dysfunction and also allows for
estimation of pulmonary artery pressure. Abnormal findings on the physical examination or EKG may be
the earliest indicators of RV dysfunction, but these may also appear normal. CT will show RV
enlargement, which is predictive of 30- and 90-day mortality, although it is not as quantitative as the EKG
(SOR A).

Ref: Piazza G: Submassive pulmonary embolism. JAMA 2013;309(2):171-180.

68
Item 200

ANSWER: D

Persons being evaluated for chronic fatigue should be assessed for comorbid depression, sleep disturbance,
and pain (SOR C). According to the Centers for Disease Control and Prevention, the initial evaluation
should include a urinalysis, a CBC, a metabolic panel, and measurement of TSH, antinuclear antibody,
C-reactive protein, rheumatoid factor, and phosphorus (SOR C). In addition, Britains National Institute
for Health and Clinical Excellence (NICE) guidelines recommend immunoglobulin A endomysial
antibodies to screen for celiac disease. The etiology of chronic fatigue syndrome is not clearly defined and
is likely multifactorial.

The initial treatment should be cognitive-behavioral therapy (CBT) or graded exercise therapy, or both
(SOR A). CBT has been shown to decrease anxiety, improve work and social adjustment, decrease
postexertional malaise, and improve overall fatigue (SOR A). There is not good evidence to support the
use of hydrocortisone, fludrocortisone, or antiviral medications in the treatment of chronic fatigue
syndrome. Other treatments that do not appear effective include melatonin, citalopram, methylphenidate,
and galantamine.

White fluorescent light therapy has been shown to be effective for treating seasonal affective disorder, but
not for chronic fatigue syndrome.

Ref: Price JR, Mitchell E, Tidy E, Hunot V: Cognitive behaviour therapy for chronic fatigue syndrome in adults. Cochrane
Database Syst Rev 2008;(3):CD001027. 2) Yancey JR, Thomas SM: Chronic fatigue syndrome: Diagnosis and treatment.
Am Fam Physician 2012;86(8):741-746.

Item 201

ANSWER: A

This patient has mild plaque psoriasis. Topical corticosteroids, vitamin D analogs, and tazarotene are
effective treatments for mild psoriasis (SOR A). Antimicrobials, anitfungals, diclofenac, and salicylic acid
are not effective treatments.

Ref: Weigle N, McBane S: Psoriasis. Am Fam Physician 2013;87(9):626-633.

Item 202

ANSWER: B

Increased intake of dietary fiber and increased exercise have been shown to prevent recurrences of
diverticulitis (SOR B). Weight loss has been shown to be effective in persons with a body mass index of
30 kg/m2 or higher but this patient is underweight and should not be counseled to lose weight (SOR B).
There is no evidence that avoiding nuts, corn, or popcorn decreases the risk of diverticulitis (SOR B). Risk
factors for diverticulitis include the use of NSAIDs, but not acetaminophen.

Ref: Strate LL, Liu YL, Aldoori WH, Giovannucci EL: Physical activity decreases diverticular complications. Am J
Gastroenterol 2009;104(5):1221-1230. 2) Strate LL, Liu YL, Aldoori WH, et al: Obesity increases the risks of
diverticulitis and diverticular bleeding. Gastroenterology 2009;136(1):115-122.e1. 3) Wilkins T, Embry K, George R:
Diagnosis and management of acute diverticulitis. Am Fam Physician 2013;87(9):612-620.

69
Item 203

ANSWER: C

Acute epididymitis is often the result of descending infection caused by urinary tract pathogens. When the
infection involves the epididymis and testis (epididymo-orchitis), sonography will frequently show an
enlarged heterogeneous testis with increased color flow.

In sexually active men under age 35, acute epididymitis is caused most frequently by Chlamydia
trachomatis and less commonly by Neisseria gonorrhoeae. Clinical features suggestive of urethritis may
be absent (subclinical urethritis). Epididymitis in men who have practiced unprotected insertive rectal
intercourse is often caused by Enterobacteriaceae. These men usually do not have urethritis but do have
bacteriuria. Treatment of acute epididymo-orchitis consists of administering appropriate antibiotics for the
treatment of both gonorrhea and Chlamydia infections. Additional antibiotic coverage may be indicated
based on the patients sexual history.

Unilateral absent flow on color and spectral Doppler sonography is a highly sensitive and specific finding
in acute testicular torsion and emergent urology referral is indicated. Heterogeneous echotexture of the
testis is a common finding in sonograms performed to evaluate acute scrotal pain, regardless of the cause.
There is no role for repeat ultrasonography or watchful waiting in patients with acute epididymo-orchitis
(SOR C).

Ref: Stengel JW, Remer EM: Sonography of the scrotum: Case-based review. AJR Am J Roentgenol 2008;190(6
Suppl):S35-S41. 2) Longo DL, Fauci AS, Kasper DL, et al (eds): Harrisons Principles of Internal Medicine, ed 18.
McGraw-Hill, 2012, pp 1095-1111.

Item 204

ANSWER: C

This patient has symptoms consistent with dengue fever, which is endemic in more than 100 tropical
countries. Dengue fever should always be suspected in patients returning from an endemic area with
spiking fevers and leukopenia. The dengue virus is an arbovirus that is transmitted by the Aedes mosquito.
The incubation period is typically 48 days (range 314). Some patients with dengue fever may be
asymptomatic or have a mild febrile illness, but it can also be very severe and manifest as dengue
hemorrhagic fever or dengue shock syndrome. Dengue hemorrhagic fever is defined by thrombocytopenia,
hemorrhagic manifestations, and plasma leakage.

Patients with dengue fever may have leukopenia, thrombocytopenia, hemoconcentration, elevated CPK,
and elevated liver enzymes. Treatment consists of supportive care and avoidance of NSAIDs and aspirin
due to their anticoagulant properties. There is no vaccine. The best way to prevent dengue is to wear
protective clothing, use bed nets, and apply repellents containing 30%50% DEET.

70
The presentation of yellow fever is very similar to dengue, but it is usually associated with elevated
bilirubin levels and jaundice. Typhoid fever has a presentation similar to that of a number of diseases. It
has a gradual onset of fever that rises in a stepwise fashion. Other symptoms include headache, arthralgias,
pharyngitis, constipation, anorexia, and abdominal pain. As the disease progresses a number of other
symptoms may develop, including skin lesions. Influenza can have a clinical presentation similar to dengue
fever but does not cause thrombocytopenia and hemorrhagic manifestations. Hepatitis A presents with
anorexia, nausea and vomiting, fatigue, malaise, myalgias, and a low-grade fever. It then usually
progresses to an icteric phase characterized by dark urine, pale stools, and jaundice.

Ref: Feder HM Jr, Mansilla-Rivera K: Fever in returning travelers: A case-based approach. Am Fam Physician
2013;88(8):524-530.

Item 205

ANSWER: E

Patients with mild asthma are often undertreated. Constant inhaled corticosteroids improve both asthma
control and quality of life. Inhaled albuterol is useful as a quick treatment for acute symptoms in patients
with mild asthma. Oral prednisone causes many side effects and is best for chronic use in patients whose
symptoms are not controlled by other means. Cromolyn sodium has a good side-effect profile, but is not
as effective as inhaled corticosteroids. Inhaled salmeterol, when used chronically, increases the risk of
asthma-related death.

Ref: Bel EH: Mild asthma. N Engl J Med 2013;369(6):549-557.

Item 206

ANSWER: C

Stretching is often included in comprehensive treatment programs for musculoskeletal injuries and chronic
conditions, making the determination of how much of the benefit is derived specifically from the stretching
component difficult. Trials using different stretching techniques have demonstrated measurable benefit
from a tailored stretching program for each of the options listed, with the exception of joint contracture.
This mobility-impairing condition results from post-healing shortness of noncontractile tissues that are not
easily released with stretching.

When applied to healing tissues, stretching is thought to increase muscle length and align collagen fibers,
thereby increasing range of motion and flexibility post healing. A program that includes static, dynamic,
and proprioceptive neuromuscular facilitation (PNF) stretching increases range of motion post knee
replacement. Static and PNF stretching increases range of motion in osteoarthritis of the knee. A program
of precontraction and static stretching increases hamstring flexibility following a strain. A yearlong
stretching program for relief of chronic neck pain has been shown to have a benefit equal to that of
strengthening exercises or manual therapy.

Ref: Page P: Current concepts in muscle stretching for exercise and rehabilitation. Int J Sports Phys Ther 2012;7(1):109-119.

71
Item 207

ANSWER: A

Any child younger than 29 days with a fever should undergo a complete sepsis workup and be admitted
for observation until culture results are obtained or the source of the fever is found and treated (SOR A).
The most common bacterial organisms in this age group are group B Streptococcus and Escherichia coli.
However, many other pathogens have been known to cause sepsis; therefore, broad empiric coverage with
ampicillin and cefotaxime is recommended (SOR B). Gentamicin is commonly used, but should be used
in combination with ampicillin. Vancomycin is not recommended as first-line treatment unless the child
has evidence of a soft-tissue infection suspected to be methicillin resistant (SOR C). Ciprofloxacin and
clindamycin are not indicated treatments in this case.

Ref: Sur DK, Bukont EL: Evaluating fever of unidentifiable source in young children. Am Fam Physician
2007;75(12):1805-1811. 2) Byington CL, Reynolds CC, Korgenski K, et al: Costs and infant outcomes after
implementation of a care process model for febrile infants. Pediatrics 2012;130(1):e16-e24. 3) Pantell RH: Febrile infants:
Aligning science, guidelines, and cost reduction with quality of individualized care. Pediatrics 2012;130(1):e199-e200.

Item 208

ANSWER: E

Because of an increased risk of infection, simple non-tense areas of clear blistering in a frostbitten patient
are best left intact. Tense or hemorrhagic blisters may be carefully aspirated, but only under sterile
conditions. Rapid rewarming of affected areas is best done in a whirlpool bath containing a mild antiseptic
at 40C41C. Freezing injuries are extremely painful and analgesics are indicated. Vigorous rubbing of
affected tissue is contraindicated, as it may lead to further tissue loss.

Ref: Hallam MJ, Cubison T, Dheansa B, Imray C: Managing frostbite. BMJ 2010;341:c5864.

Item 209

ANSWER: D

This patients symptoms and examination meet the diagnostic criteria for generalized anxiety disorder.
Based on the best evidence, pharmacologic agents that are beneficial for treating generalized anxiety
disorder in adults include antidepressants such as imipramine, duloxetine, paroxetine, sertraline,
escitalopram, and venlafaxine. Buspirone and hydroxyzine are likely to be beneficial but are supported by
less convincing evidence and can have unpleasant adverse effects. The antipsychotics and benzodiazepines
must be evaluated in terms of trade-offs between benefits and harms. Quetiapine is approved for use in
adults as add-on treatment in major depressive disorder for patients who do not have an adequate response
to antidepressant therapy alone, for acute depressive episodes in bipolar disorder, for acute manic or mixed
episodes in bipolar disorder either alone or with lithium or divalproex, for long-term treatment of bipolar
disorder with lithium or divalproex, and for schizophrenia.

Ref: Gale CK, Millichamp J: Generalized anxiety disorder. Am Fam Physician 2013;87(2):122-124.

72
Item 210

ANSWER: B

Children who reach the weight or height limit of their forward-facing child safety seat should use a
belt-positioning booster seat until the seat belt fits properly, typically when the child is 145 cm (57 in) tall
and between 8 and 12 years of age (SOR C). All children younger than 13 years should ride in the rear
seats of vehicles for optimal protection (SOR C). All infants and toddlers should ride in a rear-facing child
safety seat until they are 2 years of age or until they reach the weight or height limit recommended by the
seats manufacturer (SOR C).

Ref: Durbin DR: New recommendations on motor vehicle safety for child passengers. Am Fam Physician 2013;87(7):472-474.
2) Theurer WM, Bhavsar AK: Prevention of unintentional childhood injury. Am Fam Physician 2013;87(7):502-509.

Item 211

ANSWER: A

Stevens-Johnson syndrome is a rare, potentially life-threatening condition. There are many potential
causes, including infection, vaccines, systemic disease, physical agents, food, and drugs. Allopurinol is
the only drug listed that is a potential causative agent. Other drugs commonly cited as causes include
antibiotics, antiepileptics, and NSAIDs. Of these, antibiotics are the most common alleged cause of
Stevens-Johnson syndrome.

Ref: Roujeau JC, Stern RS: Severe adverse cutaneous reactions to drugs. N Engl J Med 1994;331(19);1272-1285. 2) Letko E,
Papaliodis DN, Papaliodis GN, et al: Stevens-Johnson syndrome and toxic epidermal necrolysis: A review of the literature.
Ann Allergy Asthma Immunol 2005;94(4):419-436. 3) Longo DL, Fauci AS, Kasper DL, et al (eds): Harrisons Principles
of Internal Medicine, ed 18. McGraw-Hill, 2012, p 153.

Item 212

ANSWER: A

Level A evidence shows that supplementation with fish oil (omega-3 fatty acids) decreases triglycerides
and very lowdensity lipoprotein (VLDL cholesterol) but can increase LDL-cholesterol. Treatment with
fish oil has not been shown to decrease cardiovascular events, total mortality, or cancer incidence.

Ref: Hooper L, Thompson RL, Harrison RA, et al: Omega-3 fatty acids for prevention and treatment of cardiovascular disease.
Cochrane Database Syst Rev 2004;(4):CD003177. 2) Hartweg J, Perera R, Montori V, et al: Omega-3 polyunsaturated
fatty acids (PUFA) for type 2 diabetes mellitus. Cochrane Database Syst Rev 2008;(1):CD003205. 3) Narla R, Peck SB,
Qiu KM: Fish oil for treatment of dyslipidemia. Am Fam Physician 2014;89(4):288, 290.

Item 213

ANSWER: D

This patient has polymyalgia rheumatica, based on her history and elevated erythrocyte sedimentation rate.
The initial treatment is prednisone, 15 mg per day with a slow taper over 12 years (SOR C). Alternative
treatment includes intramuscular methylprednisolone, 120 mg every 3 weeks.

Ref: Caylor TL, Perkins A: Recognition and management of polymyalgia rheumatica and giant cell arteritis. Am Fam Physician
2013;88(10):676-684.

73
Item 214

ANSWER: D

Zoster vaccine, a live attenuated virus vaccine, is contraindicated in this patient due to her
immunocompromised state. The other vaccines listed are safe and particularly recommended for patients
with inflammatory bowel disease, given their increased susceptibility to infections.

Immunosuppression is defined as:

Treatment with glucocorticoids (treatment with the equivalent of 20 mg/day of prednisone for 2
weeks or more, and discontinuation within the previous 3 months)
Ongoing treatment with effective doses of 6-MP/azathioprine or discontinuation within the
previous 3 months
Treatment with methotrexate or discontinuation within the previous 3 months
Treatment with infliximab or discontinuation within the previous 3 months
Significant protein-calorie malnutrition

Ref: Wasan SK, Baker SE, Skolnik PR, Farraye FA: A practical guide to vaccinating the inflammatory bowel disease patient.
Am J Gastroenterol 2010;105(6):12311238.

Item 215

ANSWER: B

The diagnosis of acute kidney injury (AKI) is based on elevated serum creatinine levels and is often
associated with a reduction in urine output (SOR C). The causes of AKI are commonly divided into three
categories: prerenal, intrinsic renal, and postrenal (SOR C). Prerenal AKI is most commonly due to
decreased renal perfusion, often because of volume depletion. In addition to vomiting and diarrhea,
overuse of diuretics can lead to prerenal AKI. Intrinsic renal AKI is caused by a process within the
kidneys. Glomerulonephritis and acute tubular necrosis are types of intrinsic AKI. Postrenal AKI refers
to a process distal to the kidneys and is most often caused by inadequate drainage of urine. Neurogenic
bladder and prostate hypertrophy contribute to extrarenal obstruction.

Ref: Mehta RL, Kellum JA, Shah SV, et al: Acute Kidney Injury Network: Report of an initiative to improve outcomes in acute
kidney injury. Crit Care 2007;11(2):R31. 2) Hoste EA, Schurgers M: Epidemiology of acute kidney injury: How big is
the problem? Crit Care Med 2008;36(4 Suppl):S146-S151. 3) Rahman M, Shad F, Smith MC: Acute kidney injury: A
guide to diagnosis and management. Am Fam Physician 2012;86(7):631-639.

Item 216

ANSWER: A

This infant has the typical findings of chlamydial pneumonia, which usually develops 13 months after
birth and should be suspected in a young infant who has tachypnea, a staccato cough, and no fever (SOR
A). Radiographs often show hyperinflation and infiltrates, and a CBC will reveal eosinophilia.

Ref: Mishori R, McClaskey EL, WinklerPrins VJ: Chlamydia trachomatis infections: Screening, diagnosis, and management.
Am Fam Physician 2012;86(12):1127-1132.

74
Item 217

ANSWER: A

In mentally handicapped patients it is important to avoid the use of psychotropic medications for managing
new behaviors until an attempt has been made to rule out potential medical and environmental causes,
except in patients with a high potential for harm to themselves or others. The underlying cause of behaviors
may be an undiagnosed medical condition, such as tooth pain, a urinary tract infection, or an electrolyte
disturbance. It is also important to consider environmental factors such as loss of a regular staff member,
a change in living environment, or family dynamics. If pharmacologic intervention is deemed necessary
the treatment should be aimed at minimizing emotional trauma and maximizing community integration.
Medication to restrict behaviors should not be used on a long-term basis. The history in this patient does
not suggest that imaging is needed.

Ref: Prater CD, Zylstra RG: Medical care of adults with mental retardation. Am Fam Physician 2006;73(12):2175-2183. 2)
Lacy B: Treatment of aggression in patients with mental retardation. Am Fam Physician 2007;75(5):622-624.

Item 218

ANSWER: D

In pregnant women not known to have diabetes mellitus, screening for gestational diabetes mellitus should
be done at 2428 weeks gestation. There are two acceptable screening strategies. The one-step 2-hour 75-g
oral glucose tolerance test should be performed in the morning after a minimum 8-hour fast. The diagnosis
of gestational diabetes mellitus is made if the fasting plasma glucose level is 92 mg/dL, if the level at 1
hour is 180 mg/dL, or if the level at 2 hours is 153 mg/dL.

A two-step approach may also be used, consisting of a nonfasting 1-hour 50-g oral glucose tolerance test.
If the 1-hour glucose level is 140 mg/dL, a 3-hour 100-mg glucose tolerance test should be performed.
(ACOG recommends 135 mg/dL in ethnic minorities with a higher risk of diabetes mellitus, and some
experts recommend 130 mg/dL.) The 3-hour test should be performed when the patient is fasting. A
diagnosis of gestational diabetes is made when at least two of the following four plasma glucose levels are
met or exceeded:

Carpenter/Coustan National Diabetes Group


Fasting 95 mg/dL 105 mg/dL
1h 180 mg/dL 190 mg/dL
2h 155 mg/dL 165 mg/dL
3h 140 mg/dL 145 mg/dL

Women with risk factors for diabetes mellitus should also be screened for undiagnosed diabetes at their
first prenatal visit.

Ref: American Diabetes Association: Standards of medical care in diabetes2014. Diabetes Care 2014;37(Suppl 1):S14-S80.

75
Item 219

ANSWER: D

Common indications for CT without contrast include suspected stroke within the first 3 hours of symptom
onset; closed head injury; diffuse lung disease; chronic dyspnea; soft-tissue swelling, infection, or trauma
of the extremities; suspected kidney stone; and suspected spinal trauma. Evaluation using
contrast-enhanced CT is indicated in the following common scenarios: acute appendicitis, cancer staging,
diverticulitis, suspected complications of inflammatory bowel disease, pancreatitis, and suspected
pulmonary embolism.

Ref: Rawson JV, Pelletier AL: When to order a contrast-enhanced CT. Am Fam Physician 2013;88(5):312-316.

Item 220

ANSWER: D

Vasomotor symptoms associated with menopause are best controlled with oral or topical estrogens.
However, one of the known risks of systemic estrogen treatment is an increased rate of developing deep
vein thrombosis (DVT). This risk is not lessened by the addition of progestin. Bio-identical hormones are
not FDA-regulated and are highly variable in their hormonal potency. For this reason their efficacy and
safety cannot be determined. Vaginal estrogen treatment results in very little circulating estrogen. Its use
has not been associated with venous thrombosis, but it does not provide relief from vasomotor symptoms.
A Cochrane meta-analysis reviewed multiple small studies using phytoestrogens and found no benefit for
control of menopausal symptoms.

Oral SSRIs and SNRIs, including venlafaxine, are effective for menopausal vasomotor symptoms, and
paroxetine is FDA-approved for this purpose without an associated risk for developing a DVT. Other
nonhormonal treatments that have evidence of benefit include gabapentin and clonidine.

Ref: Taylor HS, Manson JE: Update in hormone therapy use in menopause. J Clin Endocrinol Metab 2011;96(2):255-264. 2)
ACOG Practice Bulletin No. 141: Management of menopausal symptoms. Obstet Gynecol 2014;123(1):202-216.

Item 221

ANSWER: B

Diastolic dysfunction is now recognized as an important cause of heart failure. It is due to left ventricular
hypertrophy as a response to chronic systolic hypertension. The ventricle becomes stiff and unable to relax
or fill adequately, thus limiting its forward output. The typical patient is an elderly person who has systolic
hypertension, left ventricular hypertrophy, and a normal ejection fraction (50%55%).

Ref: Tzanetos K, Leong D, Wu RC: Office management of patients with diastolic heart failure. CMAJ 2009;180(5):520-527.
2) King M, Kingery J, Casey B: Diagnosis and evaluation of heart failure. Am Fam Physician 2012;85(12):1161-1168.

76
Item 222

ANSWER: E

This patient has carpal tunnel syndrome. Initial conservative approaches for mild to moderate symptom
relief include full-time splinting for 8 weeks (SOR B) and oral corticosteroids. However, studies suggest
that local corticosteroid injections offer symptom relief for 1 month longer than oral corticosteroid therapy
and some individuals experience relief for up to 1 year. Severe or chronic symptoms usually require
surgical intervention for nerve decompression. Physical therapy is not recommended, and full rest is
unlikely in a person in a high-risk occupation for overuse syndromes.

Ref: Stephens MB, Beutler AI, OConnor FG: Musculoskeletal injections: A review of the evidence. Am Fam Physician
2008;78(8):971-976. 2) LeBlanc KE, Cestia W: Carpal tunnel syndrome. Am Fam Physician 2011;83(8):952-958.

Item 223

ANSWER: E

Morphine is the best first choice for chronic potent opioid therapy (SOR B). It is reliable and inexpensive,
and equivalent doses can be easily calculated if the patient must later be switched to another medication.
Transdermal fentanyl and hydromorphone are reasonable second-line choices; however, they are not
recommended as first-line therapy because they are expensive and can produce tolerance relatively quickly
(SOR B). Methadone is another second-line option and tolerance is usually less of a problem. It is
inexpensive and long-acting but also has unique pharmacokinetics. It has a very long elimination half-life,
and its morphine-equivalent equianalgesic conversion ratio increases as dosages increase. Methadone can
prolong the QT interval, especially in patients who are taking other QT-prolonging medications (SOR B).
Buprenorphine is a partial opioid agonist that is usually used for treatment of patients with opioid
addictions. Although it can be effective for treatment of pain, it is expensive and requires special prescriber
training, so it is currently not recommended as a first-line agent for treatment of chronic pain (SOR C).

Ref: Berland D, Rodgers P: Rational use of opioids for management of chronic nonterminal pain. Am Fam Physician
2012;86(3):252-258.

Item 224

ANSWER: E

The main use of atropine in cases of cardiac arrest is for symptomatic bradycardia. It has little effect with
complete heart block and Mobitz type II atrioventricular block. It is not recommended or effective for
cardiac arrest with pulseless electrical activity or in cases of asystole. It has been removed from these
algorithms by the ACLS committee. During an acute myocardial infarction or acute cardiac ischemia, an
increase in heart rate may increase the amount of ischemia.

Ref: Sinz E, Navarro K, Soderberg ES (eds): Advanced Cardiovascular Life Support: Provider Manual. American Heart
Association, 2011.

77
Item 225

ANSWER: E

This patient most likely has schizotypal personality disorder. These patients have problems with social and
interpersonal relationships, which are marked by significant anxiety and discomfort, and they also exhibit
odd thinking, speech, and perceptions. This disorder is classified as being in the cluster A personality
disorder group. Patients with disorders in this group exhibit odd or eccentric personalities, and the group
includes paranoid, schizoid, and schizotypal personality disorders.

Cluster B disorders are characterized by dramatic, emotional, or erratic personalities, and include
antisocial, borderline, histrionic, and narcissistic personality disorders. Cluster C disorders include
avoidant, obsessive-compulsive, and dependent personality disorders. Patients with disorders in this group
exhibit mainly anxious or fearful behaviors.

Ref: Angstman KB, Rasmussen NH: Personality disorders: Review and clinical application in daily practice. Am Fam Physician
2011;84(11):1253-1260.

Item 226

ANSWER: E

This patient has a restrictive pattern on pulmonary function testing as evidenced by an FEV1/FVC ratio
>70% and an FVC below the lower limits of normal. Of the diagnostic options listed, idiopathic
pulmonary fibrosis is the only restrictive cause of lung disease. All of the other conditions listed are
obstructive causes of lung disease.

Ref: Johnson JD, Theurer WM: A stepwise approach to the interpretation of pulmonary function tests. Am Fam Physician
2014;89(5):359-366.

Item 227

ANSWER: D

Metronidazole, vancomycin, and fidaxomicin are the three medications recommended for treatment of
Clostridium difficile colitis infections. Only metronidazole is effective intravenously, because its biliary
excretion and possibly exudation through the colonic mucosa allows it to reach the colon via the
bloodstream. Treatment for this condition with vancomycin and fidaxomicin is oral. Imipenem/cilastatin,
ciprofloxacin, and meropenem have not been shown to be effective for C. difficile infection.

Ref: Knight CL, Surawicz CM: Clostridium difficile infection. Med Clin North Am 2013;97(4):523-536.

Item 228

ANSWER: D

Risk factors for stillbirth include advanced maternal age, smoking > pack of cigarettes a day, congenital
anomalies, and a BMI >30 kg/m2. Excessive exercise has not been shown to increase the risk for
stillbirth.

Ref: Van Dinter MC, Graves L: Managing adverse birth outcomes: Helping parents and families cope. Am Fam Physician
2012;85(9):900-904.

78
Item 229

ANSWER: D

This patient suffers from urge urinary incontinence, defined as the loss of urine accompanied or preceded
by a strong impulse to void. It may be accompanied by frequency and nocturia, and is common in older
adults. Conservative therapies such as behavioral therapy, including bladder training and lifestyle
modification, should be the first-line treatment for both stress and urge urinary incontinence (SOR C).
Pharmacologic interventions should be used as an adjunct to behavioral therapies for refractory urge
incontinence (SOR C). Vaginal inserts, such as pessaries, can be used for treating stress incontinence but
not urge incontinence.

Ref: Hersh L, Salzman B: Clinical management of urinary incontinence in women. Am Fam Physician 2013;87(9):634-640.

Item 230

ANSWER: C

Slipped capital femoral epiphysis (SCFE) typically occurs in young adolescents during the growth spurt,
when the femoral head is displaced posteriorly through the growth plate. Physical activity, obesity, and
male sex are predisposing factors for the development of this condition. There is pain with physical
activity, most commonly in the upper thigh anteriorly, but one-third of patients present with referred lower
thigh or knee pain, which can make accurate and timely diagnosis more difficult.

The hallmark of SCFE on examination is limited internal rotation of the hip. Specific to SCFE is the even
greater limitation of internal rotation when the hip is flexed to 90. No other pediatric condition has this
physical finding, which makes the maneuver very useful in children with lower extremity pain. Orthopedic
consultation is advised if SCFE is suspected.

Hip extension and abduction are also limited in SCFE, but these findings are nonspecific. Displacement
of the patella is not associated with SCFE.

Ref: Sawyer JR, Kapoor M: The limping child: A systematic approach to diagnosis. Am Fam Physician 2009;79(3):215-224.

Item 231

ANSWER: B

Hearing loss and tinnitus are both common and typically benign complaints in primary care. If both are
present in only one ear, the diagnosis of acoustic neuroma, also known as vestibular schwannoma, should
be considered. Acoustic neuroma is a slow-growing benign tumor of the Schwann cells surrounding the
vestibular cochlear (8th cranial) nerve. Hearing loss associated with acoustic neuroma is typically slow in
onset. The presence of vertigo on the affected side is another symptom of abnormal function of the
vestibular cochlear nerve and should further raise suspicion of acoustic neuroma or another process
affecting that nerve. MRI is the preferred imaging study for diagnosing acoustic neuroma (SOR A).
Bilateral hearing loss is more common and is less likely to be caused by an intracranial mass. Exposure
to loud sounds can cause hearing loss unrelated to an intracranial mass. Pain and otorrhea suggest infection
rather than an intracranial tumor.

Ref: McDonald R: Acoustic neuroma: What the evidence says about evaluation and treatment. J Fam Pract 2011;60(6):E1-E4.

79
Item 232

ANSWER: E

A trial of cilostazol is recommended by the American College of Cardiology and the American Heart
Association as initial treatment for peripheral arterial disease (PAD) that limits the lifestyle of patients
without heart failure (SOR A). Cilostazol has been shown to increase walking distance and improve
health-related quality of life (level of evidence 2). Although neither aspirin nor clopidogrel improves
claudication symptoms, antiplatelet therapy is recommended to reduce the risk of myocardial infarction,
stroke, or vascular death in patients with symptomatic PAD. Warfarin has not been shown to improve
cardiovascular outcomes in patients with PAD, but may increase bleeding without clinical benefit. The
available evidence indicates that the benefit of pentoxifylline is marginal, and it is, at best, a second-line
alternative to cilostazol.

Ref: Hennion DR, Siano KA: Diagnosis and treatment of peripheral arterial disease. Am Fam Physician 2013;88(5):306-310.

Item 233

ANSWER: C

In older adults, coprescription of clarithromycin or erythromycin with a statin that is metabolized by CYP
3A4 (atorvastatin, simvastatin, lovastatin) increases the risk of statin toxicity. The other antibiotics listed
do not interact with statins.

Ref: Patel AM, Shariff S, Bailey DG, et al: Statin toxicity from macrolide antibiotic coprescription: A population-based cohort
study. Ann Intern Med 2013;158(12):869-876.

Item 234

ANSWER: D

Patients with persistent supraventricular tachycardias require immediate medical attention. A patient who
has no underlying heart disease and a regular, narrow complex tachycardia should be treated with
adenosine. If the patient does not respond to this treatment, cardioversion should be considered.
Vasopressin would be useful if the patient were unstable with a ventricular tachycardia.

Ref: Link MS: Evaluation and initial treatment of supraventricular tachycardia. N Engl J Med 2012;367(15):1438-1448.

Item 235

ANSWER: D

The diagnosis of diabetic ketoacidosis (DKA) is based on an elevated serum glucose level (>250 mg/dL),
an elevated serum ketone level, a pH <7.3, and a serum bicarbonate level <18 mEq/L. The severity of
DKA is determined by the arterial pH, bicarbonate level, anion gap, and mental status of the patient.
Elevation of BUN and serum creatinine levels reflects intravascular volume loss. The measured serum
sodium is reduced as a result of the hyperglycemia, as serum sodium is reduced by 1.6 mEq/L for each
100 mg/dL rise in serum glucose. The degree of hyperglycemia does not necessarily correlate closely with
the degree of DKA since a variety of factors determine the level of hyperglycemia, such as oral intake and
urinary glucose loss (SOR C).

80
Ref: Longo DL, Fauci AS, Kasper DL, et al (eds): Harrisons Principles of Internal Medicine, ed 18. McGraw-Hill, 2012, pp
2968-3003. 2) Westerberg DP: Diabetic ketoacidosis: Evaluation and treatment. Am Fam Physician 2013;87(5):337-346.

Item 236

ANSWER: B

Primary hyperaldosteronism is a relatively common cause of resistant hypertension. Because there are
effective treatments, it is reasonable to consider testing for hyperaldosteronism in patients with resistant
hypertension. This is true even for patients with a normal potassium level. The preferred initial test is a
morning renin to aldosterone ratio. A ratio <20 (when plasma aldosterone is reported in ng/dL and plasma
renin activity is in ng/mL/hr) effectively rules out primary hyperaldosteronism. A ratio 20 with a serum
aldosterone level >15 ng/dL suggests aldosteronism, but a salt suppression test must be done for
confirmation. Although abdominal MRI may detect an adrenal mass, it is not recommended as a test for
hyperaldosteronism. Urinary potassium levels do not play a role in the diagnosis of primary
hyperaldosteronism.

Ref: Viera AJ: Resistant hypertension. J Am Board Fam Med 2012;25(4):487-495.

Item 237

ANSWER: D

According to the American Society for Colposcopy and Cervical Pathology, a Papanicolaou test with co-
testing for HPV is the preferred cervical cancer screening strategy for women age 3064. This is because
despite negative cytology, women with oncogenic HPV are at higher risk for later CIN 3+ than women
with negative HPV tests. The risk of CIN 3+ in HPV-positive but cytology-negative women is sufficient
to justify an earlier return for retesting. However, most HPV infections are cleared spontaneously, which
reduces the risk of CIN 3+, so observing patients to allow time for this to happen is an attractive option.

Guidelines must balance the risks arising from interventions for HPV that may clear spontaneously against
the risks of disease. Women with HPV 16 are at particular risk for CIN 3+. HPV 18 merits special
consideration because of its association with cervical adenocarcinomas, which are less efficiently detected
by cytology than squamous cell cancers. The patient described here should be advised to return for co-
testing in 1 year. If her cytology remains negative but her HPV test remains positive, she should be advised
to have colposcopy at that time regardless of the serotype of the HPV. If her current test had shown
evidence of either strain 16 or 18 immediate colposcopy would be indicated.

Ref: Massad LS, Einstein MH, Huh WK, et al: 2012 updated consensus guidelines for the management of abnormal cervical
cancer screening tests and cancer precursors. J Low Genit Tract Dis 2013;17(5 Suppl 1):S1-S27.

81
Item 238

ANSWER: C

For patients with a history of egg allergy who have experienced only hives, the Advisory Committee on
Immunization Practices recommends influenza vaccination with inactivated vaccine rather than
live-attenuated vaccine. The vaccine should be administered by a health care professional proficient in
potential manifestations of egg allergy, and the patient observed for at least 30 minutes afterward. Persons
who have had allergic reactions to egg proteins that include angioedema, respiratory distress,
lightheadedness, or recurrent emesis, or who required epinephrine or other emergency medical
interventions, are more likely to have a systemic or anaphylactic reaction to the vaccine. A previous severe
reaction to influenza vaccine is a contraindication to future vaccination. Prednisone is not appropriate as
a preventive measure.

Ref: Cayley WE Jr: Are neuraminidase inhibitors effective for preventing and treating influenza in healthy adults and children?
Am Fam Physician 2012;86(7):624-626.

Item 239

ANSWER: B

Late in 2012, the Advisory Committee on Immunization Practices of the Centers for Disease Control and
Prevention recommended hepatitis B vaccine for all previously unvaccinated adults between the ages of
19 and 59 with diabetes mellitus, as soon as possible after the diagnosis of diabetes is made. Vaccination
should be considered for patients age 60, after assessing their risk and the likelihood of an adequate
immune response.

Ref: American Diabetes Association: Standards of medical care in diabetes2013. Diabetes Care 2013;36(Suppl 1):S11-S66.

Item 240

ANSWER: C

Historically, patients with acute pancreatitis were kept NPO to rest the pancreas. Evidence now shows that
bowel rest is associated with intestinal mucosal atrophy and increased infectious complications because of
bacterial translocation from the gut. Multiple studies have shown that patients who are provided oral
feeding early in the course of acute pancreatitis have a shorter hospital stay, decreased infectious
complications, decreased morbidity, and decreased mortality. Starting with a low-fat solid diet has been
shown to be safe compared with clear liquids, providing more calories and shortening hospital stays.

Total parenteral nutrition should be avoided in patients with mild or severe acute pancreatitis. There have
been multiple randomized trials showing that total parenteral nutrition is associated with infectious and
other line-related complications.

Ref: Tenner S, Baillie J, DeWitt J, et al: American College of Gastroenterology guideline: Management of acute pancreatitis.
Am J Gastroenterol 2013;108(9):1400-1415.

82
American Board of Family Medicine

2013 IN-TRAINING EXAMINATION

CRITIQUE BOOK

This book contains the answers to each question in the In-Training Examination, as well as a critique that
provides a rationale for the correct answer. Bibliographic references are included at the end of each
critique to facilitate any further study you may wish to do in a particular area.

Copyright 2013 The American Board of Family Medicine, Inc. All rights reserved.
Item 1

ANSWER: E

This patient with acute kidney injury (AKI) has clinical symptoms and signs consistent with
rhabdomyolysis, a known cause of AKI. Furthermore, she is taking a medication known to cause
rhabdomyolysis. The urinalysis with a positive dipstick for blood and no RBCs on the microscopic
examination is indicative of either hemolysis or rhabdomyolysis. Darkened, pigmented serum would be
expected with hemolysis, while rhabdomyolysis is associated with clear serum. Urine abnormalities found
in glomerulonephritis include proteinuria and RBC casts, while patients with allergic interstitial nephritis
may have eosinophils and possibly WBC casts. Pyelonephritis is associated with WBCs in the urine, and
if the dipstick is positive for blood there will be RBCs on the microscopic examination.

Ref: Rahman M, Shad F, Smith MC: Acute kidney injury: A guide to diagnosis and management. Am Fam Physician
2012;86(7):631-639.

Item 2

ANSWER: A

Tumor necrosis factor (TNF) inhibitors are currently approved by the U.S. Food and Drug Administration
(FDA) for the treatment of rheumatic diseases such as rheumatoid arthritis, ankylosing spondylitis,
psoriatic arthritis, and juvenile idiopathic arthritis. All drugs in this class carry an FDA black-box warning
about the potential for developing primary tuberculosis or reactivating latent tuberculosis. These drugs are
also associated with an increased risk for invasive fungal infections and opportunistic bacterial and viral
diseases. The FDA also warns of reports of lymphomas and other malignancies in children and adolescents
taking these drugs.

A PPD skin test should be performed prior to initiating PNF-inhibitor therapy. An induration of 5 mm or
greater with tuberculin skin testing should be considered a positive test result when assessing whether
treatment for latent tuberculosis is necessary prior to PNF-inhibitor use, even for patients previously
vaccinated with bacille Calmette-Gurin (BCG) (SOR B).

Ankylosing spondylitis patients may develop fibrosis of the upper lung fields with long-standing disease,
but esomeprazole, ondansetron, and meloxicam do not cause reactivation of tuberculosis. Amiodarone is
associated with a subacute cough and progressive dyspnea due to pulmonary toxicity (patchy interstitial
infiltrates).

Ref: Hauck FR, Neese BH, Panchal AS, El-Amin W: Identification and management of latent tuberculosis infection. Am Fam
Physician 2009;79(10):879-886. 2) Information for healthcare professionals: Tumor necrosis factor (TNF) blockers
(marketed as Remicade, Enbrel, Humira, Cimzia, and Simponi). US Food and Drug Administration, 2009. 3) Miller AV,
Ranatunga SK: Immunotherapies in rheumatologic disorders. Med Clin North Am 2012;96(3):475-496.

Item 3

ANSWER: D

To ensure that patients from other cultures understand instructions, it is helpful to ask them to repeat the
instructions in their own words. A website would probably not be specific or culturally sensitive to the
patients condition. The physician should speak in a normal tone to the patient, and not to the interpreter.
Family members may be used as convenient translators if necessary, but to maintain confidentiality and
reduce miscommunication it is best to use a trained medical interpreter.

1
Ref: Juckett G: Caring for Latino patients. Am Fam Physician 2013;87(1):48-54.

Item 4

ANSWER: B

An annular rash with a bright red outer border and partial central clearing is characteristic of erythema
migrans. It is important to remember that not all lesions associated with Lyme disease look this way, and
that some patients with Lyme disease may not have any skin lesions at all. Rocky Mountain spotted fever
causes scattered individual purple macules on the ankles and wrists. A dry, scaling, dark red rash in the
groin, with an active border and central clearing, is seen with tinea cruris. A diffuse eruption with clear
vesicles surrounded by reddish macules is found in chickenpox. A migratory pruritic, erythematous,
papular eruption is most consistent with urticaria.

Ref: Habif TP: Clinical Dermatology: A Color Guide to Diagnosis and Therapy, ed 5. Mosby Elsevier, 2010, pp 600-604.

Item 5

ANSWER: B

The recommendation for switching to warfarin in a patient treated with dabigatran is to start warfarin 3
days prior to stopping dabigatran. Bridging with a parenteral agent is not necessary. Dabigatran is known
to increase the INR, so the INR will not reflect warfarins effect until dabigatran has been withheld for at
least 2 days.

Ref: Spinler SA, Shafir V: New oral anticoagulants for atrial fibrillation. Circulation 2012;126(1):133-137.

Item 6

ANSWER: B

Average-risk adults should be screened for colon cancer starting at 50 years of age, and high-risk adults
either at age 40 or 10 years before the age at which colorectal cancer was diagnosed in the youngest
affected relative.

Ref: ACP releases best practice advice on colorectal cancer screening. Am Fam Physician 2012;86(12):1153-1154. 2) Qaseem
A, Denberg TD, Hopkins RH Jr; Clinical Guidelines Committee of the American College of Physicians: Screening for
colorectal cancer: A guidance statement from the American College of Physicians. Ann Intern Med 2012;156(5):378-386.

Item 7

ANSWER: B

Metformin has multiple mechanisms of action, but its main effect on serum glucose results from inhibition
of gluconeogenesis in the liver. Sulfonylureas and meglitinides stimulate insulin release from the pancreas,
and thiazolidinediones sensitize peripheral tissues to insulin. Carbohydrate absorption in the small intestine
is inhibited by the !-glucosidase inhibitors.

Ref: Goldman L, Schafer AI (eds): Goldmans Cecil Medicine, ed 24. Elsevier Saunders, 2011, p e102. 2) Longo DL, Fauci
AS, Kasper DL, et al (eds): Harrisons Principles of Internal Medicine, ed 18. McGraw-Hill, 2012, p 2996.

2
Item 8

ANSWER: D

Morphine effectively decreases the feeling of shortness of breath in hospice patients. Randomized,
controlled trials have shown significant improvements in symptoms without a significant change in oxygen
saturation. Haloperidol can be used for nausea and vomiting (SOR B) and delirium, but is not helpful in
the treatment of shortness of breath. Scopolamine is used to decrease the production of secretions but is
not helpful for treating dyspnea. Corticosteroids will not manage the sensation of shortness of breath in
a dying patient.

Ref: Reville B, Axelrod D, Maury R: Palliative care for the cancer patient. Prim Care 2009;36(4):781-810.

Item 9

ANSWER: B

Thyroid hormone requirements increase during pregnancy. Most women with hypothyroidism who become
pregnant require an increased levothyroxine dosage (SOR A). A common recommendation is to have
women on fixed daily doses of levothyroxine begin taking nine doses weekly (one extra dose on 2 days
of the week) as soon as the pregnancy is confirmed (SOR B). Thyroid function tests should be repeated
regularly throughout the pregnancy to guide additional dosage adjustments.

Ref: Alexander EK, Marqusee E, Lawrence J, et al: Timing and magnitude of increases in levothyroxine requirements during
pregnancy in women with hypothyroidism. N Engl J Med 2004;351(3):241-249. 2) Gaitonde DY, Rowley KD, Sweeney
LB: Hypothyroidism: An update. Am Fam Physician 2012;86(3):244-251.

Item 10

ANSWER: D

Family medicine journals, including American Family Physician, The Journal of Family Practice, and The
Journal of the American Board of Family Medicine utilize the Strength of Recommendation Taxonomy
(SORT) to label key recommendations in clinical review articles. These grades are assigned on the basis
of the quality and consistency of available evidence. The Cochrane Collaboration is an extensive database
of systematic reviews and clinical trials. A Cochrane review with a clear recommendation warrants a
strength of recommendation rating of A. This indicates consistent, good quality, patient-oriented evidence.
Consistent findings from at least two randomized, controlled studies or a systematic review/meta-analysis
of randomized, controlled trials are also assigned a level A strength of recommendation. Expert opinion
and consensus guidelines are assigned a level C strength of recommendation. SORT also includes a grade
of 1 to 3 for levels of evidence. Retrospective cohort studies are considered level 2.

Ref: Ebell MH: SORT: The strength-of-recommendation taxonomy. Am Fam Physician 2007;76(8):1095-1096. 2) Ebell MH:
How to find answers to clinical questions. Am Fam Physician 2009;79(4):293-296.

3
Item 11

ANSWER: A

Hypercalcemia due to malignancy has a poor prognosis. Up to 80% of cases are due to secretion of
parathyroid hormonerelated protein. This is most common with squamous cell carcinomas. Breast cancer,
lymphomas, and multiple myeloma may cause hypercalcemia as a result of osteolytic activity at the site
of the metastasis.

Small cell carcinoma of the lung is a major cause of the syndrome of inappropriate secretion of antidiuretic
hormone (SIADH) and may also cause Cushing syndrome. Prostate cancer can also cause SIADH, and
thyroid cancer can cause Cushing syndrome.

Ref: Pelosof LC, Gerber DE: Paraneoplastic syndromes: An approach to diagnosis and treatment. Mayo Clin Proc
2010;85(9):838-854.

Item 12

ANSWER: C

Recent reports of epidemics of gastroenteritis on cruise ships are consistent with Norovirus infections due
to waterborne or foodborne spread. In the United States these viruses are responsible for about 90% of all
epidemics of nonbacterial gastroenteritis. The noroviruses are common causes of waterborne epidemics
of gastroenteritis, and have been shown to be responsible for outbreaks in nursing homes, on cruise ships,
at summer camps, and in schools. Symptomatic treatment by itself is usually appropriate.

Ref: Glass RI, Parashar UD, Estes MK: Norovirus gastroenteritis. N Engl J Med 2009;361(18):1776-1785. 2) Longo DL, Fauci
AS, Kasper DL, et al (eds): Harrisons Principles of Internal Medicine, ed 18. McGraw-Hill, 2012, pp 1588-1590.

Item 13

ANSWER: E

Hyperparathyroidism is usually caused by a single adenoma of one of the four parathyroid glands. A
minority of cases (10%15%) are associated with four-gland hyperplasia. Studies that localize the glands,
such as a technetium scan or ultrasonography, help surgeons who are familiar with this condition achieve
a cure rate of 95%98%, with an estimated complication rate of 1%3%. For patients <50 years old or
symptomatic patients, such as those with a fragility fracture, parathyroidectomy is the treatment of choice.
If a patient is older, is a poor surgical candidate, or has asymptomatic disease, long-term monitoring with
treatment focused on reducing bony complications can be considered (SOR C).

Ref: Marcocci C, Cetani F: Clinical practice. Primary hyperparathyroidism. N Engl J Med 2011;365(25):2389-2397. 2) Pallan
S, Rahman MO, Khan AA: Diagnosis and management of primary hyperparathyroidism. BMJ 2012;344:e1013.

4
Item 14

ANSWER: D

Venous thrombosis, both acute and recurrent, is associated with several hematologic abnormalities, in
addition to the well-known factors of trauma, surgery, malignancy, sepsis, and oral contraceptive use.
Notably, activated protein C resistance (factor V Leiden) has been found to be one of the most common
hereditary causes of thrombophilia.

Ref: Goldman L, Schafer AI (eds): Goldmans Cecil Medicine, ed 24. Elsevier Saunders, 2011, pp 1149-1150.

Item 15

ANSWER: A

Asthma is typically associated with an obstructive impairment that is reversible with short-acting
bronchodilators. A reduced FEV1 and a decreased FEV1/FVC ratio indicates airflow obstruction. A
reduced FVC with a normal or increased FEV1/FVC ratio is consistent with a restrictive pattern of lung
function.

Ref: Expert Panel Report 3: Guidelines for the Diagnosis and Management of Asthma. National Asthma Education and
Prevention Program, 2007, pp 43-45. Available at http://www.nhlbi.nih.gov/guidelines/asthma/asthgdln.pdf.

Item 16

ANSWER: E

While some studies have shown mixed results, there is good evidence that oral acyclovir reduces the
incidence of herpetic neuralgia when given within 72 hours of the onset of the rash, and that it reduces the
duration of symptoms (SOR A). Acyclovir, valacyclovir, and famciclovir have also been shown to reduce
the formation of new lesions, reduce viral shedding, and hasten the resolution of lesions. The effect of
acyclovir on preventing neuralgia appears to be strongest in the first month. Oral or topical corticosteroids
can reduce the duration of the rash and pain in the acute phase. Tricyclic antidepressants and gabapentin
can be used to treat the pain of postherpetic neuralgia if it does develop (SOR A).

Ref: Li O, Chen N, Yang J, et al: Antiviral treatment for preventing postherpetic neuralgia. Cochrane Database Syst Rev
2009;(2):CD006866. 2) Fashner J, Bell AL: Herpes zoster and postherpetic neuralgia: Prevention and management. Am
Fam Physician 2011;83(12):1432-1437.

Item 17

ANSWER: C

When children show signs of dehydration from diarrhea, the first step is to assess its extent. In one study,
four factors predicted dehydration: a capillary refill time >2 seconds, the absence of tears, dry mucous
membranes, and an ill general appearance; the presence of two or more of these signs indicates a fluid
deficit of at least 5%. This child has two of the signs, but does not require intravenous fluids at this point.
Early oral rehydration therapy is recommended and can be started at home. This should be done using an
oral rehydration solution that is designed for children (SOR C). Adult oral rehydration solutions should
not be used in children.

5
Water and other clear liquids, even those with sodium, such as chicken broth, should not replace an oral
rehydration solution because they are hyperosmolar. These fluids do not adequately replace potassium,
bicarbonate, or sodium, and can sometimes cause hyponatremia. Antidiarrheal medications are usually not
recommended for use in children with acute gastroenteritis because they delay the elimination of infectious
agents from the intestines.

Ref: Canavan A, Arant BS Jr: Diagnosis and management of dehydration in children. Am Fam Physician 2009;80(7):692-696.
2) Churgay CA, Aftab Z: Gastroenteritis in children: Part II. Prevention and management. Am Fam Physician
2012;85(11):1066-1070.

Item 18

ANSWER: C

Acute kidney injury (AKI) is currently defined as either a rise in serum creatinine or a reduction in urine
output. Creatinine must increase by at least 0.3 mg/dL, or to 50% above baseline within a 2448 hour
period. A reduction in urine output to 0.5 mL/kg/hr for longer than 6 hours also meets the criteria. Acute
interstitial nephritis is an intrinsic renal cause of AKI. These patients are often nonoliguric. A history of
recent medication use is key to the diagnosis, as cephalosporins and penicillin analogues are the most
common causes. Approximately one-third of patients present with a maculopapular rash, fever, and
arthralgias. Eosinophilia and sterile pyuria may also be seen in addition to eosinophiluria. Discontinuation
of the offending drug is the cornerstone of management.

Although up to 30% of patients with chronic hepatitis C infection have some kidney involvement, acute
interstitial nephritis is uncommon. Measuring postvoid residual urine volume is indicated if an obstructive
cause for the AKI is suspected. Starting an antibiotic to cover methicillin-resistant Staphylococcus aureus
(MRSA) is not indicated.

Ref: Rahman M, Shad F, Smith MC: Acute kidney injury: A guide to diagnosis and management. Am Fam Physician
2012;86(7):631-639. 2) Longo DL, Fauci AS, Kasper DL, et al (eds): Harrisons Principles of Internal Medicine, ed 18.
McGraw-Hill, 2012, pp 2293-2308.

Item 19

ANSWER: A

"-Agonists activate potassium uptake by the cells. This includes bronchodilators and tocolytic agents. Other
agents that can induce hypokalemia include pseudoephedrine and insulin. Diuretics, particularly thiazides,
can also cause hypokalemia as a result of the renal loss of potassium.

Ref: Longo DL, Fauci AS, Kasper DL, et al (eds): Harrisons Principles of Internal Medicine, ed 18. McGraw-Hill, 2012, pp
351-352.

6
Item 20

ANSWER: E

Infantile gonococcal infection is usually the result of exposure to infected cervical exudate during delivery
and manifests 25 days after birth. Ophthalmia neonatorum and sepsis are the most severe gonococcal
infections in newborns and immediate treatment is warranted based on the presumptive diagnosis. Topical
antibiotics are appropriate for prophylaxis, but not for treatment. Silver was used for prophylaxis at one
time, but is no longer available. Povidone-iodine has not been studied for prevention. A single dose of
2550 mg/kg of ceftriaxone administered intravenously or intramuscularly is the recommended treatment.

Ref: Workowski KA, Berman S, CDC: Sexually transmitted diseases treatment guidelines, 2010. MMWR Recomm Rep
2010;59(RR-12):1-110. 2) Update to CDCs sexually transmitted diseases treatment guidelines, 2010; Oral cephalosporins
no longer a recommended treatment for gonococcal infections. MMWR Morbid Mortal Wkly Rep 2012;61(31):590-594.

Item 21

ANSWER: B

Ipratropium is the only nasally inhaled anticholinergic recommended by the American College of Chest
Physicians for a cough caused by the common cold. One study showed that the nasal formulation decreases
rhinorrhea and sneezing, and a Cochrane review found that ipratropium bromide nasal spray improved
rhinorrhea but did not help nasal stuffiness (SOR B). Antihistamine monotherapy (either sedating or
nonsedating) such as diphenhydramine was no more effective than placebo (SOR A). Corticosteroids have
not been found to be effective for the symptoms of a common cold. Intranasal zinc should not be used
because it may result in the permanent loss of smell.

Ref: Albalawi ZH, Othman SS, Alfaleh K: Intranasal ipratropium bromide for the common cold. Cochrane Database Syst Rev
2011;(7):CD008231. 2) Fashner J, Ericson K, Werner S: Treatment of the common cold in children and adults. Am Fam
Physician 2012;86(2):153-159.

Item 22

ANSWER: C

The CT scan shows a subcapsular hematoma of the kidney. This is considered a grade I injury and does
not require surgical treatment.

Ref: Brunicardi FC (ed): Schwartzs Principles of Surgery, ed 9. McGraw Hill Medical, 2010, pp 1465-1466. 2) Marx JA (ed):
Rosens Emergency Medicine: Concepts and Clinical Practice, ed 7. Mosby Elsevier, 2010, pp 446-450. 3) Tintinalli JE,
Kelen GD, Stapczynski JS (eds): Emergency Medicine: A Comprehensive Study Guide, ed 7. McGraw-Hill, 2011, pp
1773-1774.

Item 23

ANSWER: C

Rubella has been directly responsible for inestimable pregnancy wastage, as well as for severe congenital
malformations. Identification and vaccination of unimmunized women immediately after childbirth or
abortion is recommended. The use of #-globulin to prevent viremia in nonimmune subjects exposed to
rubella is not recommended. The vaccine should be avoided shortly before or during pregnancy since it
is an attenuated live virus. Because of herd immunity there is a very low likelihood that this patient will
be exposed to rubella.

7
Ref: National Center for Immunization and Respiratory Diseases: General recommendations on immunizationRecommendations
of the Advisory Committee on Immunization Practices. MMWR Recomm Rep 2011;60(2):26-27.

Item 24

ANSWER: E

An adult patient with a confirmed systolic blood pressure >139 mm Hg or a diastolic blood pressure >89
mm Hg is hypertensive. JNC-7 guidelines recommend the adoption of healthy lifestyles for all patients,
especially those with hypertension, and the addition of pharmacologic treatment as necessary to reach a
goal blood pressure <140/90 mm Hg. This goal blood pressure is further reduced to <130/80 mm Hg
for patients who also have diabetes mellitus or renal disease. The same guidelines note that elevated
systolic blood pressure is a much more important cardiovascular disease risk factor than diastolic blood
pressure in persons older than age 50.

Medication is commonly required to reach the goal blood pressure, and most patients will often require
two or more drugs. In the absence of compelling indications for use of a specific class of drugs,
thiazide-type diuretics are recommended for initial treatment. Examples of compelling indications include
ACE inhibitors for patients with heart failure, diabetes mellitus, or high coronary disease risk, or
"-blockers post myocardial infarction.

The selection of an agent with favorable side benefits is recommended. Thiazide-type diuretics are useful
in slowing demineralization from osteoporosis, making this the most appropriate choice for this patient.
Other examples of choosing drugs based on side benefits include "-blockers for patients with a history of
migraine or tachycardia, calcium channel blockers for patients with Raynauds syndrome, and !-blockers
for patients with benign prostatic hyperplasia.

Ref: JNC 7 Express: The Seventh Report of the Joint National Committee on Prevention, Detection, Evaluation, and Treatment
of High Blood Pressure. National Heart, Lung, and Blood Institute, National High Blood Pressure Education Program,
2003, NIH pub no 03-5233.

Item 25

ANSWER: A

This presentation is consistent with bronchiolitis, which is a response to a viral respiratory infection.
American Academy of Pediatrics guidelines for the management of bronchiolitis do not recommend routine
use of any treatment, recommending instead that the choice be based on the specific needs of the child.
If the child responds to a trial of albuterol, then treatment can be continued; otherwise, evidence shows
no benefit. Antibiotics are indicated for signs of bacterial infection. Oxygen is indicated if the O2 saturation
is <90%. Corticosteroids have not been shown to be of benefit.

Ref: Zorc JJ, Hall CB: Bronchiolitis: Recent evidence on diagnosis and management. Pediatrics 2010;125(2):342-349.

8
Item 26

ANSWER: C

Secondary prevention of fractures is an important component of care following a hip fracture. Options to
consider include bisphosphonates, calcium supplementation, and vitamin D supplementation.
Bisphosphonates, including zoledronic acid, can reduce rates of clinical fractures among patients who have
had a hip fracture (SOR A). While long-term use of bisphosphonates may increase the risk of jaw
osteonecrosis and anemia, a CBC is not necessary before initiating therapy. Contraindications to zoledronic
acid include hypocalcemia and a creatinine clearance <35 mL/min or other evidence of acute renal
impairment.

Ref: FDA Drug Safety Communication: New contraindication and updated warning on kidney impairment for Reclast (zoledronic
acid). US Food and Drug Administration, 2011. 2) Hung WW, Egol KA, Zuckerman JD, Siu AL: Hip fracture
management: Tailoring care for the older patient. JAMA 2012;307(20):2185-2194.

Item 27

ANSWER: A

ACE inhibitors such as lisinopril do not need to be discontinued unless baseline creatinine increases by
>30%. (This patients creatinine increased by 20%.) The current dosage of lisinopril is appropriate, as
the blood pressure meets the diabetic goal of <130/80 mm Hg. Small increases in creatinine have been
associated with long-term preservation of renal function, and may be a marker of changes in
intraglomerular pressure.

Ref: Bakris GL, Weir MR: Angiotensin-converting enzyme inhibitorassociated elevations in serum creatinine: Is this a cause
for concern? Arch Intern Med 2000;160(5):685-693. 2) Roett MA, Liegl S, Jabbarpour Y: Diabetic nephropathyThe
family physicians role. Am Fam Physician 2012;85(9):883-889.

Item 28

ANSWER: E

Overly rapid correction of hyponatremia may cause osmotic demyelination syndrome, or central pontine
myelinolysis, sometimes resulting in permanent neurologic deficits after a brief improvement in neurologic
status. Signs and symptoms may include dysarthria, dysphagia, paresis, coma, and seizures. It is believed
that brain volume shrinks because it cannot assimilate the new electrolytes fast enough and water is lost
from the cells. Rapid correction of hypernatremia that has been present for a short time is relatively safe.
Hyperkalemia is a life-threatening condition that should be corrected promptly. Rapid correction of
hypoglycemia is not an issue. Overly rapid correction of hyperglycemia and subsequent cerebral edema
is unusual and is primarily seen in children.

Ref: Verbalis JG, Goldsmith SR, Greenberg A, et al: Hyponatremia treatment guidelines 2007: Expert panel recommendations.
Am J Med 2007;120(11 Suppl 1):S1-S21. 2) Goldman L, Schafer AI (eds): Goldmans Cecil Medicine, ed 24. Elsevier
Saunders, 2011, p 731.

9
Item 29

ANSWER: A

Melasma is a very common condition in pregnancy and is due to hyperpigmentation related to normal
hormonal changes that accompany pregnancy. It can also be caused by oral contraceptives and is more
common in dark-skinned persons.

High-potency broad-spectrum sunscreens may help prevent melasma, or at least prevent worsening of the
condition (SOR C). Topical retinoids, hydroquinone, and corticosteroids can also be helpful, but are
usually reserved for postpartum use and require months of treatment. Other treatments include azelaic acid,
chemical peels, kojic acid, cryosurgery, and laser treatment (SOR B). Melasma usually improves
spontaneously after delivery, but it may be prolonged or worsened by oral contraceptive use. It does not
increase the risk of developing skin malignancies.

Ref: Tunzi M, Gray G: Common skin conditions during pregnancy. Am Fam Physician 2007;75(2):211-218. 2) Habif TP:
Clinical Dermatology: A Color Guide to Diagnosis and Therapy, ed 5. Mosby Elsevier, 2010, pp 772-774.

Item 30

ANSWER: D

Hypersensitivity pneumonitis can present in acute, subacute, or chronic forms. The case described includes
two episodes of the acute form. The patient was exposed to mold antigens in his flooded home. Within 48
hours of exposure, chills, cough, and shortness of breath will be noted, and at times will be dramatic. A
chest film can be normal, even with significant hypoxia. Pulmonary function tests will show restrictive
changes, as compared to the reversible obstructive changes of acute asthma. Blood tests often show an
elevated erythrocyte sedimentation rate. Serum IgG tests for the probable antigen confirm the diagnosis.

Symptoms of acute hypersensitivity pneumonitis resolve over several days, but will suddenly and violently
recur with repeated exposure to the offending antigen. The subacute form begins gradually over weeks or
months, causing a cough and increasing shortness of breath. The chronic form develops over years of
exposure, causing fibrotic changes to the lungs that will be evident on radiographs, as well as chronic
crackles on auscultation.

Asthma would be an unlikely diagnosis in this case, with the pulmonary function tests showing restrictive
changes rather than obstructive changes, and little improvement with albuterol. Also, the lack of a previous
history of asthma makes it less likely. Legionnaires disease is always possible, but is unlikely in this case
given the sudden onset, quick recovery over several days, and sudden recurrence. Pulmonary embolism
is ruled out by the negative D-dimer test.

Ref: Goldman L, Schafer AI (eds): Goldmans Cecil Medicine, ed 24. Elsevier Saunders, 2011, pp 564-565.

10
Item 31

ANSWER: B

Hyperparathyroidism is present in more than half of patients who have a glomerular filtration rate <60
mL/min, and is independently associated with increased mortality and an increased prevalence of
cardiovascular disease. In patients with stage 4 chronic kidney disease, current guidelines recommend
monitoring of serum calcium and phosphate levels every 36 months and bone-specific alkaline
phosphatase activity every 612 months with the goal of normalizing these values. The other metabolic
abnormalities listed are less common than hyperparathyroidism.

Ref: Abboud H, Henrick WL: Stage IV chronic kidney disease. N Engl J Med 2010;362(1):56-65.

Item 32

ANSWER: C

Up through July of 2013, the Centers for Disease Control and Prevention had logged 624 cases of
hantavirus pulmonary syndrome in residents of 34 states. The infection killed more than a third of the
victims. The virus is usually spread by inhalation of dried aerosolized deer mouse urine or droppings.
Infected deer mice usually have few outward signs. Other hosts include the white-footed mouse, the rice
rat, and the cotton rat. Other rodents have not been shown to carry the virus.

Ref: Hantavirus. Centers for Disease Control website, 2012. http://www.cdc.gov/hantavirus 2) Centers for Disease Control
and Prevention: Facts about Hantaviruses: What you need to know to prevent the disease Hantavirus Pulmonary Syndrome
(HP). US Dept of Health and Human Services, nd.

Item 33

ANSWER: B

Erythema multiforme (EM) is an acute, often recurrent, inflammatory condition. While it is associated with
many different causes such as drugs, infections, and physical agents, it is most commonly associated with
acute upper respiratory infections, herpes simplex virus, and Mycoplasma pneumoniae.

EM typically occurs in persons 2040 years of age, with urticarial papules or the classic bulls-eye or
target lesions (as shown in this case). The distribution is primarily on the hands (both the dorsi and palms),
soles, and extensor aspects of the arms and legs. Lesions may also occur on mucosal surfaces, but in
typical EM these are milder than in the more severe Stevens-Johnson syndrome, which is more commonly
associated with drug exposure than with infectious agents.

The lesions of EM are usually pruritic, but not universally so. They evolve to the typical target forms over
2448 hr. The central area is often dusky and may be superficially necrotic or vesicular. Partial target
lesions may resemble urticaria. The lesions usually heal without scarring over a 10- to 14-day period.

An id reaction is a papulovesicular eruption, classically on the sides of fingers, that occurs in response to
an intense inflammatory process such as a fungal infection or acute dermatitis in another anatomic area of
the body. When that condition resolves, so does the id reaction.

11
Discoid lupus lesions are irregular but roughly round in shape, sharply demarcated, and most commonly
found on the face or scalp. They usually develop an adherent thick scale. The duration may be months or
years.

Granuloma annulare manifests as a ring of firm, flesh-colored or red papules with a more prominent outer
ring forming due to central involution. These lesions may last for months to years. The distribution is
usually on the dorsolateral aspect of the hands or feet.

Pyoderma gangrenosum begins as a tender papule, usually on the lower leg, and evolves to a painful,
necrotic, inflammatory lesion that gradually enlarges up to 10 cm. It usually is a manifestation of an
underlying systemic inflammatory condition such as inflammatory bowel disease. The lesions last for
months to years and heal with scarring.

Ref: Habif TP: Clinical Dermatology: A Color Guide to Diagnosis and Therapy, ed 5. Mosby Elsevier, 2010, pp 109, 682,
710-714, 737-740, 976-977.

Item 34

ANSWER: E

The results of this urine test were that 10 people had the disease and tested positive (true positives); 8
people had the disease but tested negative (false negatives); 17 people did not have the disease (27 10)
but tested positive (false positive); 69 people did not have the disease (77 8) and tested negative (true
negative). The negative predictive value is determined by dividing the true negatives (69) by the total
number who tested negative (true plus false negatives = 77). The result is 89.6%, which rounded to a
whole number is 90%.

Ref: Gordis L: Epidemiology, ed 4. Saunders Elsevier, 2008, pp 85-108.

Item 35

ANSWER: A

This patient should be provided with a regular diet, which may promote weight gain in nursing-home
residents with unintentional weight loss. Malnutrition and unintentional weight loss are significant problems
in nursing-home residents and lead to multiple complications, including pressure ulcers and infections. The
American Dietetic Association recommends liberalizing diets to improve nutritional status and quality of
life in older adults. A small study demonstrated equivalent glycemic control in nursing-home residents who
ate a regular diet compared to those who ate a restricted American Diabetes Association diet (SOR C).
Low-salt and low-cholesterol diets are unpalatable and are often associated with protein-energy malnutrition
and postural hypotension in older persons. Special diets should be avoided whenever possible in
nursing-home patients.

Ref: Morley JE, Silver AJ: Nutritional issues in nursing home care. Ann Intern Med 1995;123(11):850-859. 2) Niedert KC;
American Dietetic Association: Position of the American Dietetic Association: Liberalization of the diet prescription
improves quality of life for older adults in long-term care. J Am Diet Assoc 2005;105(12):1955-1965. 3) Unwin BK,
Porvaznik M, Spoelhof GD: Nursing home care: Part II. Clinical aspects. Am Fam Physician 2010;81(10):1229-1237.

12
Item 36

ANSWER: D

Several disorders that are common in women with polycystic ovary syndrome are associated with an
increased risk for endometrial carcinoma, including obesity, hyperinsulinemia, diabetes mellitus,
anovulatory cycles, and high androgen levels.

Ref: Wilson JF: In the clinic. The polycystic ovary syndrome. Ann Intern Med 2011;154(3):ITC2-2-ITC2-15.

Item 37

ANSWER: C

Sinus bradycardia is almost universally present in patients with anorexia nervosa. It is hypothesized that
this is due to vagal hyperactivity resulting from an attempt to decrease the amount of cardiac work by
reducing cardiac output. It is also possible that the bradycardia can be accounted for by low serum T3
levels, a common finding in persons with chronic malnutrition. Sinus tachycardia may occur with refeeding
in patients with anorexia. Other arrhythmias may also occur but are less frequent.

Ref: Casiero D, Frishman WH: Cardiovascular complications of eating disorders. Cardiol Rev 2006;14(5):227231. 2) Williams
PM, Goodie J, Motsinger CD: Treating eating disorders in primary care. Am Fam Physician 2008;77(2):187-195.

Item 38

ANSWER: C

The recommended treatment regimen for gonorrhea is ceftriaxone, 250 mg intramuscularly. The 125-mg
regimen is no longer recommended because of treatment failures and limited effectiveness in pharyngeal
infections. In addition, the patient should be given azithromycin, 1 g orally, because of the high incidence
of coinfection with Chlamydia, even if testing is negative, and to decrease the risk for cephalosporin
resistance.

Ref: Workowski KA, Berman S; Centers for Disease Control and Prevention (CDC): Sexually transmitted diseases treatment
guidelines, 2010. MMWR Recomm Rep 2010;59(RR-12):1-110. 2) Centers for Disease Control and Prevention (CDC):
Update to CDCs sexually transmitted diseases treatment guidelines, 2010: Oral cephalosporins no longer a recommended
treatment for gonococcal infections. MMWR Morb Mortal Wkly Rep 2012;61(31):590-594.

Item 39

ANSWER: A

The herpes zoster vaccine is the only live-attenuated virus vaccine listed, and is therefore the one
contraindicated in immunodeficient patients. Human papillomavirus, Tdap, meningococcal, and
pneumococcal polysaccharide vaccines are not live or live-attenuated vaccines, and may be given to
immunocompromised patients.

Ref: Vaughn JA, Miller RA: Update on immunization in adults. Am Fam Physician 2011;84(9):1015-1020.

13
Item 40

ANSWER: B

Polypharmacy is common in the elderly population, but the use of numerous medications is necessary in
some elderly patients. However, some medications have been identified as having a considerably higher
potential to cause problems when prescribed to elderly patients.

In the case described, acetaminophen/diphenhydramine would be an appropriate medication to stop


initially. The older antihistamines cause many adverse CNS effects such as cognitive slowing and delirium
in older patients. These effects are more pronounced in elderly patients with some degree of preexisting
cognitive impairment. The anticholinergic properties of older antihistamines produce effects such as dry
mouth, constipation, blurred vision, and drowsiness. The sedative effect of older antihistamines also
increases the risk of falls. Hip fracture and subsequent death have been reported in patients who use older
antihistamines such as diphenhydramine.

Sertraline is an SSRI, a preferred class for the treatment of depression in the elderly compared to the
tricyclic antidepressants, which are associated with several side effects. Dipyridamole is associated with
hypotension in elderly patients, but it benefits some individuals by preventing strokes. It can be used in
the elderly, but patients should be monitored for side effects. Therefore, until further information is
obtained, it is appropriate to continue the dipyridamole/aspirin in this patient.

When used in elderly patients with heart failure, digoxin should be given in a dosage no greater than 0.125
mg daily; the low dosage used in this individual should not be considered inappropriate until the reason
for its use is clarified. While omeprazole can cause problems in the elderly with long-term use, 20 mg/day
is a relatively low dose and the decision to discontinue its use should be delayed until more history is
available.

Ref: Ramakrishnan K, Scheid DC: Treatment options for insomnia. Am Fam Physician 2007;76(4):517-526. 2) American
Geriatrics Society 2012 Beers Criteria Update Expert Panel: American Geriatrics Society updated Beers Criteria for
potentially inappropriate medication use in older adults. J Am Geriatric Soc 2012;60(4):616-631. 3) Pretorius RW, Gataric
G, Swedlund SK, Miller JR: Reducing the risk of adverse drug events in older adults. Am Fam Physician
2013;87(5):331-336.

Item 41

ANSWER: B

Regional analgesia in laboring patients increases the risk of vacuum- or forceps-assisted delivery (relative
risk = 1.42; 95% confidence interval, 1.281.57; 23 trials; n = 735). Multiple randomized, controlled
trials have compared regional analgesia with no analgesia. In a meta-analysis, no statistically significant
impact was found on the risk of cesarean delivery, maternal satisfaction with pain relief, long-term
backache, or immediate effect on neonatal status as determined by Apgar scores. Regional analgesia
provides better pain relief than opioid analgesia.

Ref: Schrock SD, Harraway-Smith C: Labor analgesia. Am Fam Physician 2012;85(5):447-454.

14
Item 42

ANSWER: B

This patient suffers from diarrhea-predominant irritable bowel syndrome (IBS). Her symptoms meet the
Rome III criteria for diagnosis with no alarm features. A CBC, serum chemistries, thyroid function
studies, stool testing for ova and parasites, and abdominal imaging are all low-yield tests that are not
recommended for routine diagnostic evaluation of IBS (SOR C). The association between IBS and bacterial
overgrowth in the small intestine is not clear, so routine hydrogen breath testing is not recommended.
Routine testing for celiac disease, however, should be considered in patients with diarrhea-predominant
or mixed-presentation IBS (SOR C). A systematic review that included more than 4000 patients found that
4% of those with diarrhea-predominant or mixed-presentation IBS had biopsy-proven celiac disease.

Ref: Wilkins T, Pepitone C, Biju A, Schade RR: Diagnosis and management of IBS in adults. Am Fam Physician
2012;86(5):419-426.

Item 43

ANSWER: A

The World Health Organization (WHO) publishes the medical eligibility criteria for contraceptive use as
a guideline for the appropriate use of contraceptives. There are four categories that define the
appropriateness of contraceptive use in women with certain medical problems:

Category 1: A condition for which there is no restriction for the use of the contraceptive method.

Category 2: A condition for which the advantages of using the method generally outweigh the theoretical
or proven risks.

Category 3: A condition for which the theoretical risk or proven risks usually outweigh the advantages of
using the method.

Category 4: A condition that represents an unacceptable health risk if the contraceptive method is used.

A history of migraine with aura is classified as category 4 for oral contraceptives. Women with a history
of migraines are 24 times as likely to have a stroke compared to women without migraines, and women
who have an aura associated with their migraines are at even higher risk. Migraine without aura is
classified as category 2 in women younger than 35 and category 3 in women 35 or older. Nonmigrainous
headaches are category 1, as is chronic hepatitis C.

Combined oral contraceptive use does not appear to increase the rate or severity of cirrhotic fibrosis and
there is no increased risk for hepatocellular carcinoma. Combined oral contraceptives are not
recommended for use in women with acute hepatitis C. Breastfeeding is considered category 2 by the CDC
and category 3 by WHO. There is conflicting evidence about the effects on the volume of breast milk in
women who are on combined oral contraceptives, but the concerns are mainly during the first month of
the postpartum period. There have not been any demonstrated adverse health effects in infants exposed to
combined oral contraceptives through breast milk. Laparoscopic banding weight-loss surgery is category
1 for combined oral contraceptive use. Evidence shows no significant decrease in the effectiveness of oral
contraceptives in women who have had this surgery. Combined oral contraceptive use in patients with
endometrial hyperplasia is category 1. Combined oral contraceptives have been used to decrease the risk
for endometrial cancer.

15
Ref: Medical Eligibility Criteria for Contraceptive Use, ed 4. World Health Organization, 2009, pp 11-12, 16-27. 2) US
Medical Eligibility Criteria for Contraceptive Use, 2010, June 18, 2010. MMWR Recomm Rep 2010;59(RR-4):11-18.

Item 44

ANSWER: C

This patient has secondary hyperparathyroidism, a common cause of hypercalcemia in patients with chronic
renal insufficiency. Sevelamer is a newer synthetic agent in the therapeutic class of phosphate binders,
which includes calcium acetate. Decreasing serum phosphate lowers the feedback stimulation of
parathyroid hormone secretion by the parathyroid gland, which is often excessive in chronic renal
insufficiency. Normalizing parathyroid levels improves serum calcium levels.

Ref: Longo DL, Fauci AS, Kasper DL, et al (eds): Harrisons Principles of Internal Medicine, ed 18. McGraw-Hill, 2012, pp
3096-3120.

Item 45

ANSWER: D

Acute otitis externa is treated with topical antibiotics. Although no preparation has been shown to be most
effective, a fluoroquinolone does not create a risk of ototoxicity if the tympanic membrane is not intact.
Topical corticosteroids may hasten symptom reduction. Oral antibiotics are not appropriate unless the
infection has spread beyond the ear canal, or if the patient has diabetes mellitus or is immunocompromised.

Ref: Schaefer P, Baugh RF: Acute otitis externa: An update. Am Fam Physician 2012;86(11):1055-1061.

Item 46

ANSWER: D

Childhood obstructive sleep apnea syndrome has a prevalence rate of 5.7%. It is associated with growth,
cardiovascular, and neurobehavioral abnormalities. Adenotonsillectomy is the treatment of choice.
Although CPAP can be effective, compliance is poor and it is therefore not a first-line treatment. Intranasal
corticosteroids may also be helpful, but the benefit appears small.

Ref: Marcus CL, Brooks LJ, Draper KA, et al: Diagnosis and management of childhood obstructive sleep apnea syndrome.
Pediatrics 2012;130(3):e714-e755.

Item 47

ANSWER: D

Urine pH is an important factor in the production of kidney stones. Uric acid, cystine, and calcium oxalate
stones tend to form in acidic urine, whereas struvite (magnesium ammonium phosphate) and calcium
phosphate stones form in alkaline urine. Urine should be acidified for prevention of calcium phosphate and
struvite stones. Cranberry juice or betaine can lower urine pH.

Ref: Frassetto L, Kohlstadt I: Treatment and prevention of kidney stones: An update. Am Fam Physician
2011;84(11):1234-1242.

16
Item 48

ANSWER: B

Calcaneal apophysitis, also known as Sever disease, is the most common etiology of heel pain in children,
usually occurring between 5 and 11 years of age. It is thought that in these children the bones grow faster
than the muscles and tendons. A tight Achilles tendon then pulls on its insertion site at the posterior
calcaneus with repetitive running or jumping activities, causing microtrauma to the area. There may be
swelling and tenderness in this area and passive dorsiflexion may increase the pain. Radiography is usually
normal and therefore does not often aid in the diagnosis, but it may reveal a fragmented or sclerotic
calcaneal apophysis. Treatment involves decreasing pain-inducing activities, anti-inflammatory or analgesic
medication if needed, ice, stretching and strengthening of the gastrocnemius-soleus complex, and the use
of orthotic devices.

Plantar fasciitis and heel pad syndrome cause pain on the plantar surface of the heel rather than posteriorly.
Achilles tendinopathy causes tenderness to palpation of the Achilles tendon. Tarsal tunnel syndrome related
to compression of the posterior tibial nerve causes neuropathic pain and numbness in the posteromedial
ankle and heel.

Ref: Tu P, Bytomski JR: Diagnosis of heel pain. Am Fam Physician 2011;84(8):909-916.

Item 49

ANSWER: D

This patients age and his history of hypertension give him a CHADS2 score of 2, which implies that the
patient is at a higher risk of thromboembolism and should be anticoagulated with warfarin and not aspirin.
The patient should be fully anticoagulated for a minimum of 3 weeks prior to any attempt at cardioversion.

Ref: Gutierrez C, Blanchard DG: Atrial fibrillation: Diagnosis and treatment. Am Fam Physician 2011;83(1):61-68.

Item 50

ANSWER: B

This patient has a Revised Cardiac Risk Index (Goldman Index) score of 1, placing him in a low-risk group
for perioperative cardiac complications. Low-risk patients who are able to walk for 2 blocks or climb a
flight of stairs without stopping to rest (4 METS) do not need noninvasive cardiac testing. Patients in this
risk group who are already on a "-blocker should continue it, but adding one preoperatively may increase
risk. Stopping aspirin therapy in patients with coronary stents places them at risk for perioperative cardiac
events. Surgical bleeding is somewhat increased in patients on aspirin, but differences in the severity of
bleeding events and mortality in surgical patients on low-dose aspirin versus controls are minimal. Stopping
clopidogrel in patients who have recently undergone coronary stent placement (6 weeks for bare-metal
stents, 1 year for drug-eluting stents) markedly increases risk, but there is no need to start clopidogrel in
other patients. Perioperative statin therapy should be continued for all patients undergoing surgery. For
patients undergoing vascular therapy, statins have been associated with an improvement in postoperative
cardiac outcomes.

Ref: Holt NF: Perioperative cardiac risk reduction. Am Fam Physician 2012;85(3):239-246. 2) Drugs for lipids. Treat Guidel
Med Lett 2012;9(103):13-20.

17
Item 51

ANSWER: B

Comedones are noninflammatory acne lesions. Inflammatory lesions include papules, pustules, and
nodules. Grading acne based on the type of lesion and severity helps guide therapy. Topical retinoids
prevent the formation of comedones and reduce their number, and are indicated as monotherapy for
noninflammatory acne. Topical antibiotics are used primarily for the treatment of mild to moderate
inflammatory or mixed acne. Oral antibiotics are effective for the treatment of moderate to severe acne.
Oral isotretinoin is reserved for treatment of severe, recalcitrant acne.

Ref: Titus S, Hodge J: Diagnosis and treatment of acne. Am Fam Physician 2012;86(8):734-740.

Item 52

ANSWER: E

The most likely diagnosis for this patient is stable slipped capital femoral epiphysis (SCFE). This is more
frequent in males than in females, and is more common in African-Americans and Pacific Islanders than
in whites. Although some patients present with pain, many present with a painless limp or vague pain. The
average age of onset is 13.5 years for males and 12 years for females. Obesity is strongly associated with
SCFE.

The lack of systemic symptoms makes osteomyelitis, abscess, or a septic joint much less likely.
Malignancy is a possibility, but night pain would be more likely. Sacroiliitis is much less likely given a
negative FABER test. The patients age makes transient synovitis or Legg-Calv-Perthes disease less
likely. Although muscle strain is a possibility, the physical examination findings of external rotation
deformity and limited internal rotation are more specific for SCFE.

Once the diagnosis of SCFE is made, the patient should not bear weight and should be referred promptly
for surgery to prevent complications.

Ref: Gholve PA, Cameron DB, Millis MB: Slipped capital femoral epiphysis update. Curr Opin Pediatr 2009;21(1):39-45. 2)
Sawyer JR, Kapoor M: The limping child: A systematic approach to diagnosis. Am Fam Physician 2009;79(3):215-224.
3) Peck D: Slipped capital femoral epiphysis: Diagnosis and management. Am Fam Physician 2010;82(3):258-262.

Item 53

ANSWER: C

Methimazole and propylthiouracil (PTU) are the two oral antithyroid medications available. However,
because of reports of severe hepatocellular damage, methimazole should be used instead of PTU unless
it is contraindicated. Radioactive iodine treatment (131I) is an option, especially for patients who do not
achieve remission with antithyroid medications. However, worsening of preexisting orbitopathy is a
well-recognized potential complication of 131I treatment, as well as a transient increase in thyroid hormone
levels that can precipitate thyroid storm. Thus, patients with elevated free T3 or free T4 levels should be
treated with methimazole prior to 131I administration. Thyroidectomy is most often recommended for
patients with thyroid nodules and those who are suspected of having cancer or who do not tolerate or refuse
alternative forms of therapy. However, antithyroid medication should be given to achieve a euthyroid state
prior to surgery in most patients.

18
Ref: McDermott MT: Hyperthyroidism. Ann Intern Med 2012;157(1):ITC1-ITC16.

Item 54

ANSWER: E

Obesity increases the risk of a variety of medical conditions, including type 2 diabetes mellitus,
hypertension, hyperlipidemia, pulmonary disease, coronary artery disease, gallstones, fatty liver disease,
obstructive sleep apnea, GERD, osteoarthritis, and a variety of forms of cancer. A weight loss of at least
10% for greater than 1 year leads to statistically significant improvement in lipid ratios, blood glucose
homeostasis, and coronary artery disease risk reduction.

The AAFP recommends screening for obesity and intensive counseling (more than 1 session per month
for more than 3 months) with behavior modification for obese patients. Counseling is ineffective by itself
and must be combined with lifestyle modification.

Dietary modification, increased physical activity, and behavior modification are effective for maintaining
modest weight loss for greater than 1 year (SOR B). However, there are few large, randomized, controlled
trials with subjects maintaining weight reductions of 10% for over 1 year, even when combining therapy,
exercise, and dietary restriction. Long-term pharmacotherapy can lead to weight loss, but regaining some
weight is typical.

Bariatric surgery leads to the most effective weight reduction and long-term maintenance in patients who
are morbidly obese (SOR A). Gastric bypass is effective, with a mean weight loss of 71.2% at 3 years;
with laparoscopic gastric banding the mean weight loss is 55.2% at 3 years. In one study, 94% of gastric
bypass patients maintained at least a 20% weight loss at 6 years. Bariatric surgery has also been shown
to significantly reduce fasting blood glucose, with resolution of diabetes mellitus in 31%77% of lap band
patients and 72%100% of gastric bypass patients. Bariatric surgery is a safe and effective means for
long-term weight loss and should be considered in adults with a BMI >40, or >35 with obesity-related
comorbidities.

Ref: Grief SN, Miranda RL: Weight loss maintenance. Am Fam Physician 2010;82(6):630-634. 2) Rao G: Office-based
strategies for the management of obesity. Am Fam Physician 2010;81(12):1449-1456. 3) Adams TD, Davidson LE, Litwin
SE, et al: Health benefits of gastric bypass surgery after 6 years. JAMA 2012;308(11):1122-1131.

Item 55

ANSWER: D

This patient has signs and symptoms of lateral epicondylitis, also known as tennis elbow, or alternatively
as lateral epicondylalgia to reflect the noninflammatory nature of the condition. This is an overuse
tendinopathy of the common extensor tendon origin of the lateral elbow. Conservative care that includes
offloading the involved tendons is the key to improving outcomes at 1 year, which would mean modifying
this patients work. Physical therapy can improve pain and function in the short term, but has not been
shown to improve long-term outcomes at 1 year in randomized trials (SOR A). The evidence is weaker
for bracing, with some studies showing improved pain and function at 36 weeks (SOR B). Recent
randomized, controlled trials have made it clear that while corticosteroid injections reduce acute pain for
up to 6 weeks, their use increases rates of poor long-term outcomes (SOR A).

19
Ref: Johnson GW, Cadwallader K, Scheffel SB, Epperly TD: Treatment of lateral epicondylitis. Am Fam Physician
2007;76(6):843-848. 2) Orchard J, Kountouris A: The management of tennis elbow. BMJ 2011;342:d2687. 3) Coombes
BK, Bisset L, Brooks P, et al: Effect of corticosteroid injection, physiotherapy, or both on clinical outcomes in patients with
unilateral lateral epicondylalgia: A randomized controlled trial. JAMA 2013;309(5):461-469.

Item 56

ANSWER: A

Many elderly patients with heart failure are hospitalized repeatedly at short intervals. As described in a
2012 Cochrane review of 25 randomized, controlled trials, there is now good evidence that case
management interventions led by a heart failure specialist nurse reduce heart failure readmissions, all-cause
readmissions, and all-cause mortality. Case management interventions include home care, telephone calls,
patient education, self-management, and face-to-face visits. It is not possible to say which specific
interventions were optimal, but telephone follow-up was a common component in most of these trials.
Multidisciplinary interventions may also be effective. For patients treated with lisinopril, 40 mg/day is the
maximum amount recommended for heart failure. While "-adrenergic blockers and aldosterone antagonists
are used in the treatment of heart failure, prophylactic amiodarone would not be expected to help and may
be harmful. Admission to a nursing home may reduce hospitalization, but it would also increase overall
costs of care.

Ref: Bonow RO, Mann DL, Zipes DP, Libby P (eds): Braunwalds Heart Disease: A Textbook of Cardiovascular Medicine,
ed 9. Elsevier Saunders, 2011, pp 1039-1041. 2) Takeda A, Taylor SJ, Taylor RS, et al: Clinical service organisation for
heart failure. Cochrane Database Syst Rev 2012;9:CD002752.

Item 57

ANSWER: A

B-type natriuretic peptide (BNP) is a 32amino acid polypeptide secreted from the cardiac ventricles in
response to ventricular volume expansion and pressure overload. The major source of BNP is the cardiac
ventricles, and because of the minimal presence of BNP in storage granules, its release is directly
proportional to ventricular dysfunction. A BNP test is simple and time efficient, and reliably predicts the
presence or absence of left ventricular dysfunction on an echocardiogram.

Ref: Krishnaswamy P, Lubien E, Clopton P, et al: Utility of B-natriuretic peptide levels in identifying patients with left
ventricular systolic or diastolic function. Am J Med 2001;111(4):274-279. 2) Bonow RO, Mann DL, Zipes DP, Libby P
(eds): Braunwalds Heart Disease: A Textbook of Cardiovascular Medicine, ed 9. Elsevier Saunders, 2011, pp 505-516.

Item 58

ANSWER: D

Nonbenzodiazepine muscle relaxants such as cyclobenzaprine are beneficial for the relief of acute low back
pain for the first 714 days after the onset of symptoms. Patient education, physical therapy, and the
application of ice or heat may also help. Unsupported treatment options for acute low back pain include
oral corticosteroids, acupuncture, lumbar support, massage, chiropractic spinal manipulation, and traction.
Bed rest for acute low back pain is inadvisable.

Ref: Casazza BA: Diagnosis and treatment of acute low back pain. Am Fam Physician 2012;85(4):343-350.

20
Item 59

ANSWER: B

Many infections of the respiratory tract have a viral etiology, and when this is the case early antibiotic
treatment offers little to no benefit. Once the clinical course of a respiratory illness exceeds the expected
length for a viral illness, it may be proper to initiate antibiotic treatment for a suspected atypical or
secondary bacterial infection. Epiglottitis is one exception to this approach because of the possibility of a
bacterial infection, particularly with Haemophilus influenzae type b, that can produce a rapidly worsening,
potentially fatal airway compromise. When epiglottitis is suspected based on findings such as hoarseness,
dysphagia, stridor, drooling, fever, chills, and respiratory distress, intravenous antibiotic treatment should
be instituted immediately, ideally with a "-lactam drug that exhibits activity against methicillin-resistant
Staphylococcus aureus.

Ref: Zoorob R, Sidani M, Fremont RD, Kihlberg C: Antibiotic use in acute upper respiratory tract infections. Am Fam
Physician 2012;86(9):817-822.

Item 60

ANSWER: C

Some experts recommend inpatient observation for all children with apparent life-threatening events such
as this. Hospital admission is not always necessary, however, for a short, self-correcting episode associated
with feeding. Given the history of not breathing for 2025 seconds, having a blue tongue and lips, and
being limp, admitting the child for observation is appropriate.

Although child abuse is a concern, referral to child protective services is not mandatory. Laryngoscopy
would not be routine, but might be appropriate in some cases depending on the history and physical
findings. Many groups recommend home apnea monitoring after discharge for patients with more severe
or undiagnosed cases.

Ref: Warren J, Biagioli F, Hamilton A, Smith PC: FPINs clinical inquiries: Evaluation of apparent life-threatening events in
infants. Am Fam Physician 2007;76(1):124-126. 2) Fu LY, Moon RY: Apparent life-threatening events: An update. Pediatr
Rev 2012;33(8):361-368.

Item 61

ANSWER: E

Cefazolin is appropriate for intrapartum prophylaxis against group B Streptococcus (GBS) in


penicillin-allergic patients who do not have a history of anaphylaxis, urticaria, angioedema, or respiratory
distress. Depending on the antibiotic sensitivity of the GBS organism, either vancomycin or clindamycin
is recommended for patients at higher risk for anaphylaxis.

Ref: Verani JR, McGee L, Schrag SJ, et al: Prevention of perinatal group B streptococcal diseaseRevised guidelines from
CDC, 2010. MMWR Recomm Rep 2010;59(RR-10):1-36.

21
Item 62

ANSWER: D

The U.S. Preventive Services Task Force has concluded that the net benefit of daily aspirin is substantial
in women 5579 years of age for whom the benefit of ischemic stroke prevention exceeds the harm of an
increased risk for gastrointestinal bleeding (SOR A). Aspirin use is recommended in men 4579 years of
age for prevention of myocardial infarction when the potential benefit outweighs the potential harm of
gastrointestinal hemorrhage (SOR A).

Ref: Aspirin for the prevention of cardiovascular disease: Recommendation statement. US Preventive Services Task Force, 2009.
2) Aspirin for the prevention of cardiovascular disease: Recommendation statement. Am Fam Physician
2011;83(12):1464-1468.

Item 63

ANSWER: E

Pretibial myxedema is a complication of Graves disease, whether it presents as hypo- or hyperthyroidism.


It is a dermopathy that most often occurs in the lower legs and results from increased deposition of mucin
due to the endocrine abnormality. Diabetes mellitus can cause necrobiosis lipoidica, a lesion on the lower
extremities; hyperlipidemia can cause waxy papules; and collagen vascular and ischemic disease can cause
urticaria and/or ulceration.

Ref: Goldman L, Schafer AI (eds): Goldmans Cecil Medicine, ed 24. Elsevier Saunders, 2011, pp e69-e70.

Item 64

ANSWER: B

Laryngoscopy should be performed to visualize the larynx and evaluate for vocal cord pathology in a
patient whose hoarseness does not resolve within 3 months (SOR C). If a serious condition is suspected
for some other reason, laryngoscopy should be performed regardless of the duration of symptoms. If there
is a recent history of upper respiratory infection or vocal abuse, then it would be appropriate to recommend
voice rest for 2 weeks. Laryngoscopy would then be indicated if the hoarseness did not improve or
recurred after voice rest. For patients with symptoms of gastroesophageal reflux, a trial of a proton pump
inhibitor is recommended (SOR B). Inhaled corticosteroids, especially fluticasone, may cause hoarseness.
Oral corticosteroids do not have a role in the management of hoarseness.

Ref: Feierabend RH, Shahram MN: Hoarseness in adults. Am Fam Physician 2009;80(4):363-370. 2) Schwartz SR, Cohen SM,
Dailey SH, et al: Clinical practice guideline: Hoarseness (dysphonia). Otolaryngol Head Neck Surg 2009;141(3 Suppl
2):S1-S31.

Item 65

ANSWER: D

The U.S. Preventive Services Task Force (USPSTF) recommends one-time screening for abdominal aortic
aneurysm (AAA) by ultrasonography in men age 6575 who have ever smoked (Grade B
recommendation). The USPSTF recommends against routine screening for AAA in women (Grade D
recommendation).

22
Ref: US Preventive Services Task Force: Screening for Abdominal Aortic Aneurysm: Recommendation Statement. AHRQ pub
no 05-0569-A, 2005.

Item 66

ANSWER: B

Pneumonia caused by Legionella pneumophila is commonly preceded by nonspecific systemic symptoms


that may lead a clinician to consider other diagnoses. Symptoms may include high-grade fever, malaise,
myalgias, anorexia, and headache. Gastrointestinal and neurologic symptoms are also common and include
nausea, vomiting, abdominal pain, diarrhea, and confusion. Focal neurologic signs are less common, but
have been reported. Localizing respiratory symptoms will typically develop later, most often a dry cough
and dyspnea. From this point on the illness resembles a typical pneumonia with fever, productive cough,
pleuritic pain, and breathlessness.

Ref: Darby J, Buising K: Could it be Legionella? Aust Fam Physician 2008;37(10):812-815.

Item 67

ANSWER: D

"-Blockers are first-line antihypertensive medications for patients with coronary artery disease (CAD) and
have been shown to reduce the risk of death by 23% at 2 years. They should also be given to normotensive
patients with CAD if tolerated. Cardioselective ("1) "-blockers such as metoprolol and atenolol are
preferred, as they cause fewer adverse effects.

Ref: Pflieger M, Winslow BT, Mills K, Dauber IM: Medical management of stable coronary artery disease. Am Fam Physician
2011;83(7):819-826.

Item 68

ANSWER: B

Bronchiectasis is an illness of the bronchi and bronchioles involving obstructive and infectious processes
that injure airways and cause luminal dilation. In addition to daily viscid, often purulent sputum production
with occasional hemoptysis, wheezing and dyspnea occur in 75% of patients. Emphysema and chronic
bronchitis, forms of COPD, also cause a decreased FEV1/FVC ratio, but the sputum is generally mucoid
and luminal dilation of bronchi is not characteristically present. Bronchiolitis is usually secondary to
respiratory syncytial virus infection in young children. Asthma is not characterized by the sputum and CT
findings seen in this patient.

Ref: Barker AF: Bronchiectasis. N Engl J Med 2002;346(18):1383-1393. 2) Holmes RL, Fadden CT: Evaluation of the patient
with chronic cough. Am Fam Physician 2004;69(9):2159-2166, 2169. 3) Benich JJ, Carek PJ: Evaluation of the patient
with chronic cough. Am Fam Physician 2011;84(8):887-892. 4) Goldman L, Schafer AI (eds): Goldmans Cecil Medicine,
ed 24. Elsevier Saunders, 2011, pp 548-550.

Item 69

ANSWER: B

Multiple myeloma can be asymptomatic, but it becomes symptomatic when there is organ damage or other
abnormalities, including renal insufficiency, elevated calcium, anemia, and bone disease.

23
The majority of patients have bone pain, but hypocalcemia is not common. Hypokalemia almost never
occurs, and both hepatic failure and insomnia are not usual signs of multiple myeloma. Anemia typically
occurs either because of renal failure or infiltration of the bone marrow by myeloma cells. Polycythemia
does not occur.

Ref: Palumbo A, Anderson K: Multiple myeloma. N Engl J Med 2011;363(11):1046-1058.

Item 70

ANSWER: C

For patients undergoing major orthopedic surgery, the American College of Chest Physicians recommends
outpatient thromboprophylaxis for a duration of up to 35 days. Older recommendations for 1014 days of
prophylaxis were based on studies performed when this was the usual hospital stay. This is still
recommended as the minimum length for prophylaxis, but a longer period of time is preferred.

Ref: Guyatt GH, Akl EA, Crowther M, et al: Executive summary: Antithrombotic therapy and prevention of thrombosis, 9th
ed: American College of Chest Physicians Evidence-Based Clinical Practice Guidelines. Chest 2012;141(2 Suppl):7S-47S.

Item 71

ANSWER: D

Based on this patients history and physical examination, COPD is the most likely cause of his dyspnea.
Initial testing should include spirometry to diagnose airflow obstruction (SOR C). CT, a BNP level, a
D-dimer level, and arterial blood gas measurements would not be the best initial tests in the evaluation of
this patients dyspnea.

Ref: Wahls SA: Causes and evaluation of chronic dyspnea. Am Fam Physician 2012;86(2):173-180.

Item 72

ANSWER: A

During the treatment of severe preeclampsia with intravenous magnesium, the occurrence of apnea and
areflexia is most consistent with magnesium toxicity. In addition to hemodynamic support, calcium infusion
is recommended as an antidote. Calcium chloride can be used if a central line has been established.
Calcium gluconate would be safer with a peripheral intravenous site.

Lorazepam, phenytoin, and fosphenytoin are less useful in preventing eclamptic seizures than magnesium.
Labetalol is not indicated given the patients current blood pressure level. Dopamine, a pressor agent, is
not indicated in this scenario, and could aggravate the patients preeclampsia.

Ref: Vanden Hoek TL, Morrison LJ, Shuster M, et al: Part 12: Cardiac arrest in special situations: 2010 American Heart
Association Guidelines for Cardiopulmonary Resuscitation and Emergency Cardiovascular Care. Circulation 2010;122(18
Suppl 3):S829-S861. 2) Deak TM, Moskovitz JB: Hypertension and pregnancy. Emerg Med Clin North Am 2012;30(4):
903917.

24
Item 73

ANSWER: E

This patient has cluster headaches. Most people with cluster headaches are male. These headaches typically
present with severe unilateral pain that lasts from 15 minutes to 3 hours. The pain is generally extremely
sharp, continuous, and incapacitating. In addition to the pain, the headaches are associated with at least
one of the following ipsilateral signs: conjunctival injection, lacrimation, nasal congestion, miosis or ptosis,
eye edema, and forehead and facial sweating. Patients may also have a sense of restlessness or agitation.
The headaches occur anywhere from every other day up to 8 times a day, often in cycles for 412 weeks.
Cluster headaches respond to most of the same medications as migraine headaches (DHE, ergotamines,
triptans). They also respond well to 100% oxygen therapy.

Paroxysmal hemicranias are very unusual and present with a similar type of pain, but the attacks are
usually short and they are more common in women. Medication rebound headaches tend to be diffuse,
bilateral, almost daily headaches. These occur in people who are overusing medications, and they tend to
get worse with physical or mental exertion. Temporal arteritis usually occurs in older adults. Migraines
are also often unilateral but they are usually pulsatile, and are associated with nausea and vomiting or
photophobia and phonophobia.

Ref: Rakel RE, Rakel DP (eds): Textbook of Family Medicine, ed 8. Elsevier Saunders, 2011, pp 962-965.

Item 74

ANSWER: C

The most common form of advance directive is a patients conversations with relatives and friends, and
these carry the same ethical and legal weight as written directives. Neither the hospital attorney nor the
patients personal attorney, in the absence of a previous discussion with the patient, has the ethical or legal
authority to make the decision. Since the patient has previously expressed his wishes, it is unnecessary to
have the court appoint a surrogate decision maker. Care should be transferred to another physician only
if the original physician has a philosophical or religious objection to carrying out the patients wishes.

Ref: Halter JB, Ouslander JG, Tinetti ME, et al (eds): Hazzards Geriatric Medicine and Gerontology, ed 6. McGraw-Hill
Medical, 2009, pp 388-389.

Item 75

ANSWER: A

Patients who require inhalation therapy with "2-adrenergic-receptor agonists more than twice weekly but
not daily have mild persistent asthma. Long-term control with inhaled corticosteroids is recommended for
adults with persistent asthma.

Ref: Expert Panel Report 3: Guidelines for the Diagnosis and Management of Asthma. National Asthma Education and
Prevention Program, NIH pub no 07-4051, 2007, pp 211-217.

25
Item 76

ANSWER: C

Inflammatory bowel disease is divided into two categories: Crohns disease and ulcerative colitis.
Noncontiguous or skip lesions that are transmural in nature and are found throughout the gastrointestinal
tract make a diagnosis of Crohns disease likely in this patient. Corticosteroids are more effective in
inducing remission than placebo and 5-ASA products (SOR A). A Cochrane review revealed no difference
between elemental and nonelemental diets with regard to symptom remission (SOR A). Anti-TNF agents
such as infliximab should be considered in patients with moderate to severe Crohns disease who do not
respond to initial corticosteroid or immunosuppressive therapy, but these are not recommended for initial
treatment. While antibiotics such as metronidazole are widely used for both their anti-inflammatory and
anti-infectious properties, controlled trials have not demonstrated their effectiveness. Surgical intervention
should be considered in patients with ulcerative colitis, but surgery is not indicated for Crohns disease.

Ref: Langan RC, Gotsch PB, Krafczyk MA, Skillinge DD: Ulcerative colitis: Diagnosis and treatment. Am Fam Physician
2007;76(9):1323-1330. 2) Wilkins T, Jarvis K, Patel J: Diagnosis and management of Crohns disease. Am Fam Physician
2011;84(12):1365-1375.

Item 77

ANSWER: A

Pharyngitis is a common complaint, and usually has a viral cause. The key factors in diagnosing
streptococcal pharyngitis are a fever over 100.4F, tonsillar exudates, anterior cervical lymphadenopathy,
and absence of cough. The scenario described is consistent with a viral infection, with no risk factors to
make streptococcal infection likely; therefore, this patient should be offered symptomatic treatment only.
Testing for other infections is not indicated unless the patient worsens or does not improve.

Ref: Chan TV: The patient with sore throat. Med Clin North Am 2010;94(5):923-943.

Item 78

ANSWER: B

Many minority populations in the United States, including Hispanics, suffer health care disparities. Some
medical problems are also more common in certain populations. In 2009, Hispanics made up
approximately 16% of the U.S. population and were the largest minority. This group is projected to make
up 30% of the U.S. population by 2050. The rates of obesity and diabetes mellitus in Hispanics are
disproportionately higher than those of non-Hispanic whites. Hypertension is closely linked to obesity, and
Hispanics have higher rates of hypertension as well. Neurocysticercosis is the most common cause of
seizures in Hispanic immigrants. Risk factors for osteoporosis include female gender, non-Hispanic white
ethnicity, smoking, and low BMI.

Ref: Juckett G: Caring for Latino patients. Am Fam Physician 2013;87(1):48-54.

26
Item 79

ANSWER: B

While there is significant overlap in the symptoms of each condition, there are some signs and symptoms
that help the family physician determine whether a terminally ill patient is experiencing grief or has major
depression. This distinction is important because the terminally ill patient with depression would likely
benefit from antidepressant medication, whereas a patient with end-of-life grief is generally best managed
without psychotherapeutic medications.

The key clinical feature in distinguishing the two conditions is in the pervasiveness of symptoms in
depression, particularly the loss of pleasure or interest in all activities. Episodic feelings of guilt, anxiety,
and helplessness, and even thoughts of wanting to die can and do occur with grief reactions, but these
feelings are not constant and over time the symptoms gradually wane. Terminally ill patients with major
depression feel helplessly hopeless all the time, but they often respond to and significantly benefit from
antidepressant medication (SOR A).

Ref: Widera EW, Block SD: Managing grief and depression at the end of life. Am Fam Physician 2012;86(3):259-264.

Item 80

ANSWER: B

This patient has aortic stenosis, which is most likely of the degenerative, age-related type. An
echocardiogram should be included in this patients evaluation, but the diagnosis can still be made on the
basis of the auscultatory findings. The second heart sound (closure of the aortic valve then the pulmonic
valve) is normally split, consisting of two distinct components during inspiration. As the aortic valve
becomes more rigid with the progression of aortic stenosis, the valve closures occur simultaneously or even
become paradoxically split during expiration.

The murmur of aortic stenosis is a midsystolic ejection-type murmur, heard loudest at the right second
intercostal space parasternally. It may radiate to the right carotid, and to a lesser degree may also be heard
in the fourth intercostal space at the left lower sternal border. An S3 gallop is heard only late in the
progression of aortic stenosis and is associated with left ventricular dilatation and heart failure.

The murmur of aortic regurgitation is a diastolic descrescendo blowing-type murmur best heard at the left
fourth intercostal space at the lower left sternal border, with the patient sitting and leaning forward. Mitral
valve murmurs are typically located at the heart apex or even in the left infra-axillary area. Mitral valve
prolapse is characterized by a prominent systolic click, often with a brief mitral regurgitation murmur.

Ref: Maganti K, Rigolin VH, Serano ME, Bonow RO: Valvular heart disease: Diagnosis and management. Mayo Clin Proc
2010;85(5):483-500.

Item 81

ANSWER: C

Although patients with chronic mild liver disease may take NSAIDs, they should be avoided in all patients
with cirrhosis, due to the risk of precipitating hepatorenal syndrome. Pregabalin and gabapentin are not
metabolized by the liver and can be quite helpful. Acetaminophen, while toxic in high doses, can be used
safely in dosages of 23 g/day. Tramadol is also safe in patients with cirrhosis.

27
Ref: Chandok N, Watt KD: Pain management in the cirrhotic patient: The clinical challenge. Mayo Clin Proc
2010;85(5):451-458.

Item 82

ANSWER: B

Coral-red fluorescence on Woods light examination is typical of infection with Corynebacterium


minutissimum, a condition known as erythrasma. This organism commonly complicates intertrigo, often
in the groin or interdigital spaces. Erythromycin is the most effective treatment for this bacterial infection.

Ref: Janniger CK, Schwartz RA, Szepietowski JC, Reich A: Intertrigo and common secondary skin infections. Am Fam
Physician 2005;72(5):833-838. 2) Binic I, Jankovic A: Coral-red fluorescence. N Engl J Med 2011;364(13):e25.

Item 83

ANSWER: A

The U.S. Preventive Services Task Force (USPSTF) and the Centers for Disease Control and Prevention
(CDC) recommend that all adults age 65 years and under be screened for HIV regardless of risk factors.
The USPSTF does not recommend routine screening for ovarian cancer with a bimanual examination,
transvaginal ultrasonography, or CA-125 testing. The USPSTF recommends that women age 65 and older
be screened for osteoporosis with a DXA scan. Women younger than age 65 should be screened only if
their risk of fracture is equal to or greater than a 65-year-old white female with no additional risk factors.
The USPSTF recommends that women age 5075 be screened for colorectal cancer with colonoscopy
every 10 years OR with flexible sigmoidoscopy every 5 years plus fecal occult blood testing (FOBT) every
3 years OR with FOBT annually.

Ref: Riley M, Dobson M, Jones E, Kirst N: Health maintenance in women. Am Fam Physician 2013;87(1):30-37.

Item 84

ANSWER: A

Acute stress disorder (ASD) is a condition that develops within 4 weeks of a traumatic experience. The
individual with ASD suffers dissociative symptoms including amnesia, a reduction in awareness of his or
her surroundings, numbness, and detachment. Patients who suffer from ASD are at higher risk for
developing PTSD, but appropriate treatment can reduce this risk. Cognitive-behavioral management
(muscle relaxation techniques, gradual introduction to avoided situations, restructuring beliefs about the
trauma) aids in the treatment of ASD (SOR A). Cognitive-behavioral therapy has been found to be more
effective than supportive counseling.

Although pharmacologic management can be helpful, the evidence does not show it to be broadly
beneficial. Imipramine, fluoxetine, risperidone, and propranolol have all been used for ASD, but are not
recommended for routine use (SOR C).

Critical incident stress debriefing is a group activity usually provided shortly after a traumatic event.
During the sessions, patients are invited to share their feelings and education is provided on coping with
the trauma. There has been no evidence of a positive effect of stress debriefing and it may actually be
harmful by interfering with the natural course of recovery (SOR C).

28
Ref: Kavan MG, Elsasser GN, Barone EJ: The physicians role in managing acute stress disorder. Am Fam Physician
2012;86(7):643-649.

Item 85

ANSWER: B

The Centers for Disease Control and Prevention (CDC) recommends treatment for persons at higher risk
for complications from influenza. Pregnant women and postpartum women within 2 weeks of delivery are
considered to be at higher risk. Some data has shown that early antiviral treatment may shorten the duration
of symptoms and reduce the risk of complications, especially in these higher risk populations.

The CDC recommends against using adamantines due to increased resistance of influenza viruses. Other
antivirals such as acyclovir and famciclovir are not active against influenza A and B. The recommended
treatment of influenza in pregnancy is one of the neuraminidase inhibitors.

Ref: Fiore AE, Fry A, Shay D, et al: Antiviral agents for the treatment and chemoprophylaxis of influenzaRecommendations
of the Advisory Committee on Immunization Practices (ACIP). MMWR Recomm Rep 2011;60(1):1-24. 2) Influenza
antiviral medications: Summary for clinicians. Centers for Disease Control and Prevention, 2013.

Item 86

ANSWER: D

Patients with bone pain caused by Pagets disease usually describe the pain as continuous. Unlike
osteoarthritis, the bone pain of Pagets disease usually increases with rest, when the limbs are warmed,
and at night. A variety of deformities may occur, including bowing of the tibia, and alkaline phosphatase
is elevated. The case presented is not typical for osteoarthritis or osteoporosis, and the patient does not
have a fever, elevated WBC count, or other findings suggestive of osteomyelitis.

Seronegative spondyloarthritis (or spondyloarthropathy) is a family of inflammatory rheumatic diseases


that cause arthritis. The most common is ankylosing spondylitis, which affects mainly the spine. Others
include axial spondyloarthritis, which affects mainly the spine and pelvic joints; peripheral
spondyloarthritis, affecting mostly the arms and legs; reactive arthritis (formerly known as Reiters
syndrome); psoriatic arthritis; and enteropathic arthritis/spondylitis associated with inflammatory bowel
diseases (ulcerative colitis and Crohns disease). The main symptom in most patients with spondyloarthritis
is low back pain, which is most common in axial spondyloarthritis. In a minority of patients, the major
symptom is pain and swelling in the arms and legs. This type is known as peripheral spondyloarthritis.

Ref: Schneider D, Hofmann MT, Peterson JA: Diagnosis and treatment of Pagets disease of bone. Am Fam Physician
2002;65(10):2069-2072. 2) Whyte MP: Pagets disease of bone. N Engl J Med 2006;355(6):593-600. 3) Reveille JD:
Spondyloarthritis (spondyloarthropathy). American College of Rheumatology patient fact sheet, 2012. 4) Longo DL, Fauci
AS, Kasper DL, et al (eds): Harrisons Principles of Internal Medicine, ed 18. McGraw-Hill, 2012, pp 3136-3139.

Item 87

ANSWER: D

The National Health and Nutrition Examination Survey (NHANES) found that 1.4% of adults over 40 have
an ankle-brachial index (ABI) >1.4; this group accounts for approximately 20% of all adults with
peripheral artery disease. An ABI >1.4 indicates noncompressible arteries (calcified vessels). In patients
with arterial calcification, such as diabetic patients, more reliable information is often obtained by using
toe pressures to calculate a toe-brachial index and from pulse volume recordings.

29
Vascular imaging should be used to confirm peripheral vascular disease. MR or CT arteriography, duplex
scanning, and hemodynamic localization are noninvasive methods for lesion localization and may be
helpful when symptoms or findings do not correlate with the ABI. Contrast arteriography is used for
definitive localization before intervention.

Ref: Resnick HE, Foster GL: Prevalence of elevated ankle-brachial index in the United States 1999 to 2002. Am J Med
2005;118(6):676-679. 2) Sontheimer DL: Peripheral vascular disease: Diagnosis and treatment. Am Fam Physician
2006;73(11):1971-1976. 3) Met R, Bipat S, Legemate DA, et al: Diagnostic performance of computed tomography
angiography in peripheral arterial disease: A systematic review and meta-analysis. JAMA 2009;301(4):415-424. 4) Hauk
L: ACCF/AHA update peripheral artery disease management guideline. Am Fam Physician 2012;85(10):1000-1001.

Item 88

ANSWER: A

The inhalation of asbestos fibers may lead to a number of respiratory diseases, including lung cancer,
asbestosis, pleural plaques, benign pleural effusion, and malignant mesothelioma. High-risk populations
for asbestos exposure include individuals who worked in construction trades or as boilermakers, shipyard
workers, or railroad workers, as well as U.S. Navy veterans. The occupational history helps to guide
clinical suspicion in these high-risk populations. This patient is a retired U.S. Navy veteran who spent his
entire career in ship maintenance and repair.

The patient history is not consistent with berylliosis, silicosis, or uranium exposure. Berylliosis is an
occupational disease related to mining and manufacturing. Silicosis is seen in sandblasters, miners, persons
who have worked with abrasives, and several other occupations. Uranium exposure occurs after nuclear
reactor leaks or blasts. Uranium compounds are also used in photography and as dyes or fixatures. The
chemical toxicity involves nonmalignant damage to alveolar cells. Iron oxide exposure is not known to be
related to lung disease.

Ref: OReilly KM, Mclaughlin AM, Beckett WS, Sime PJ: Asbestos-related lung disease. Am Fam Physician
2007;75(5):683-688. 2) State of Lung Disease in Diverse Communities, 2010. American Lung Association, 2010, pp 63-69.
3) Schraufnagel DE (ed): Breathing in America: Diseases, Progress, and Hope. American Thoracic Society, 2010, pp
131-144. 4) The National Institute for Occupational Safety and Health (NIOSH): Work-Related Lung Disease Surveillance
System (eWoRLD). Centers for Disease Control and Prevention, 2013.

Item 89

ANSWER: C

A lace-up ankle support reduces pain and recovery time after an ankle sprain (SOR B). The Ottawa Rules
state that radiography is required only if there is pain in the malleolar or midfoot zone and either bony
tenderness over an area of potential fracture (i.e., distal fibula or tibia, lateral or medial malleolus, base
of the fifth metatarsal, or navicular bone) or an inability to bear weight immediately after the injury and
when evaluated by a physician. This patient did not have those findings, and therefore would not need a
radiograph (SOR A). A cast is not necessary for an ankle sprain. An elastic compression wrap alone is not
as effective as a lace-up support. Taping of the ankle for future sports participation can reduce the risk of
ankle sprains during sports, but would not be appropriate for an acute injury.

Ref: Tiemstra JD: Update on acute ankle sprains. Am Fam Physician 2012;85(12):1170-1176.

30
Item 90

ANSWER: A

Acute mountain sickness is common in people traveling to altitudes higher than 8200 ft. Symptoms include
headache and at least one of the following: nausea or vomiting, anorexia, dizziness or lightheadedness,
fatigue or weakness, and difficulty sleeping. Slow ascent is the most effective way to prevent acute
mountain sickness. Acetazolamide or dexamethasone can be used for both prevention and treatment. Ataxia
and altered mental status are signs of cerebral edema and occur with end-stage acute mountain sickness.
This can progress to coma and death and requires prompt treatment and descent. High-altitude pulmonary
edema can occur without acute mountain sickness. Alcohol consumption on the first day at altitude can
exacerbate acute mountain sickness.

Ref: Fiore DC, Hall S, Shoja P: Altitude illness: Risk factors, prevention, presentation, and treatment. Am Fam Physician
2010;82(9):1103-1110.

Item 91

ANSWER: D

The hallmark of acute mesenteric artery ischemia is severe abdominal pain that is out of proportion to
physical findings. This is a life-threatening event, which often follows a myocardial infarction when a
mural thrombus occludes a superior mesenteric artery. Patients rapidly become acidotic and hypotensive,
and experience a high mortality rate.

Alcoholic withdrawal syndrome has a much more insidious onset, with tremors, agitation, and anxiety
being the prominent features. Abdominal pain is not a common prominent symptom or finding. Although
pulmonary embolism is possible in the patient described, the major symptoms are dyspnea and chest pain,
and abdominal pain and tenderness would not be a typical finding. A normal serum lipase level should
point to a nonpancreatic origin for this patients problem. A perforated gastric ulcer would typically have
a more insidious onset and the abdominal examination would usually demonstrate marked focal tenderness.

Ref: Oldenburg WA, Lau LL, Rodenberg TJ, et al: Acute mesenteric ischemia. Arch Intern Med 2004;164(10):1054-1062. 2)
Marx JA (ed): Rosens Emergency Medicine: Concepts and Clinical Practice, ed 7. Mosby Elsevier, 2010, pp 1188-1192.

Item 92

ANSWER: A

Maximal heart rate with exercise generally decreases with age. A frequently used formula for predicting
maximal heart rate is 220 minus age, with a correction factor of 0.85 often applied for females, who have
a lower peak heart rate and a more gradual decline.

Myocardial collagen and mass both increase with age. The increase in collagen may play a role in
decreasing left ventricular compliance. The resting heart rate, like the maximal exercising heart rate,
decreases with normal aging. Tachycardia at rest may suggest a pathologic state.

Ref: Halter JB, Ouslander JG, Tinetti ME, et al (eds): Hazzards Geriatric Medicine and Gerontology, ed 6. McGraw-Hill
Medical, 2009, pp 883-895. 2) Fillit HM, Rockwood K, Woodhouse K (eds): Brocklehursts Textbook of Geriatric
Medicine and Gerontology, ed 7. Saunders Elsevier, 2010, pp 91-97.

31
Item 93

ANSWER: E

ACE inhibitors and "-blockers improve mortality in heart failure (HF). Digoxin and furosemide improve
symptoms and reduce hospitalizations in systolic HF, and furosemide may decrease mortality.
Spironolactone, an aldosterone antagonist, reduces all-cause mortality and improves ejection fractions in
systolic HF. Verapamil, due to its negative inotropic effect, is associated with worsening heart failure and
an increased risk of adverse cardiovascular events.

Ref: Jessup M, Abraham WT, Casey DE, et al: 2009 focused update: ACCF/AHA Guidelines for the Diagnosis and Management
of Heart Failure in Adults: A report of the American College of Cardiology Foundation/American Heart Association Task
Force on Practice Guidelines. Circulation 2009;119(14):1977-2016.

Item 94

ANSWER: A

The diagnostic criteria for depression are the same for children and adults, although the manner in which
these symptoms present may be different. Adolescents with depression are more likely to experience
anhedonia, boredom, hopelessness, hypersomnia, weight change, alcohol or drug use, and suicide
attempts. Psychotherapy should always be included as part of a treatment plan for depression in
adolescents.

Cognitive-behavioral therapy and interpersonal therapy are two modalities that have been proven effective
in the treatment of adolescent depression. Medications should be considered for more severe depression
or depression failing to respond to psychotherapy. A Cochrane review found that fluoxetine was the only
agent with consistent evidence of effectiveness in decreasing depressive symptoms in adolescents.
Consensus guidelines recommend fluoxetine, citalopram, or sertraline as first-line treatments for moderate
to severe depression in children and adolescents. Escitalopram is also licensed for the treatment of
depression in adolescents age 12 or older. All antidepressants have a boxed warning regarding an increased
risk of suicide; therefore, close monitoring is recommended to assess for suicidality and other adverse
effects, such as gastrointestinal effects, nervousness, headache, and restlessness. Tricyclic antidepressants
were previously used to treat depression in children, but studies have shown little to no benefit in
adolescents and children.

Ref: Clark MS, Jansen KL, Cloy JA: Treatment of childhood and adolescent depression. Am Fam Physician 2012;86(5):442-448.

Item 95

ANSWER: A

While all of the medications listed can be used to treat osteoporosis, only calcitonin-salmon is useful in the
management of pain associated with acute or chronic vertebral fractures. Calcitonin is an antiresorptive
agent that has been shown to decrease the risk of vertebral fractures, but it is not considered a first-line
treatment for osteoporosis because there are more effective agents. However, it does have modest analgesic
properties that make it useful in the treatment of the pain associated with vertebral fractures.

Ref: Sweet MG, Sweet JM, Jeremiah MP, Galazka SS: Diagnosis and treatment of osteoporosis. Am Fam Physician
2009;79(3):193-200.

32
Item 96

ANSWER: C

This patient most likely has pseudogynecomastia due to increases in subareolar fat secondary to his obesity.
This is based upon clinical findings of symmetric adipose tissue in the breast region bilaterally and a lack
of firm, palpable glandular tissue in the nipple and areolar region. In gynecomastia, there is palpable, firm
glandular tissue in a concentric mass around the nipple-areola complex. Hard, immobile masses, masses
associated with skin changes, nipple retraction, nipple discharge, or enlarged lymph nodes would suggest
possible malignancy. Fat necrosis would involve a history of breast region trauma and would generally be
asymmetric. Mastitis would cause clinical signs of infection.

Ref: Dickson G: Gynecomastia. Am Fam Physician 2012;85(7):716-722.

Item 97

ANSWER: E

Among the oral antiglycemic drugs, the sulfonylurea agents are the most likely to cause hypoglycemia,
and glyburide is more likely to cause hypoglycemia than glipizide. Glyburide should rarely be used in the
elderly.

Ref: American Diabetes Association: Standards of medical care in diabetes2013. Diabetes Care 2013;36(Suppl 1):S45.

Item 98

ANSWER: E

This patient meets the criteria for hypoactive sexual desire disorder (HSDD). The incidence of this
condition is variable based on the age, life stage, and culture of the patient, but is estimated to be present
in about 5%15% of the adult female population. This diagnosis includes two components: (1) recurrent
deficiency or absence of sexual desire or receptivity to sexual activity, and (2) distress about such a
deficiency. In menstruating women, oral estrogen and oral sildenafil have not been shown to be superior
to placebo. Cognitive-behavioral therapy has been shown to be helpful for other sexual dysfunctions, but
not with HSDD. Topical testosterone, in either patch or gel form, has shown consistent improvements in
arousal, desire, fantasy, orgasm, and overall satisfaction in cases of HSDD.

Ref: Parish SJ: From whence comes HSDD? J Fam Pract 2009;58(7 Suppl Hypoactive):S16-S21. 2) American College of
Obstetricians and Gynecologists Committee on Practice BulletinsGynecology: ACOG Practice Bulletin No. 119: Female
sexual dysfunction. Obstet Gynecol 2011;117(4):996-1007. 3) Simon JA. Opportunities for intervention in HSDD. J Fam
Pract. 2009;58(7 Suppl Hypoactive):S26-30. Review.

Item 99

ANSWER: C

The appropriate management of a thrombosed hemorrhoid presenting within 72 hours of the onset of
symptoms is elliptical excision of the hemorrhoid and overlying skin under local anesthesia, such as 0.5%
bupivacaine hydrochloride in 1:200,000 epinephrine, infiltrated slowly with a 27-gauge needle.

33
Incision and clot removal may provide inadequate drainage, resulting in rehemorrhage and clot
reaccumulation. Most thrombosed hemorrhoids contain multilocular clots that may not be accessible
through a simple incision. Rubber band ligation is an excellent technique for management of internal
hemorrhoids, and infrared coagulation is also used for this purpose. Banding an external hemorrhoid would
cause exquisite pain.

If the pain is already subsiding or more time has elapsed, and if there is no necrosis or ulceration,
measures such as sitz baths, bulk laxatives, stool softeners, and local analgesia may be helpful. Some local
anesthetics carry the risk of sensitization. Counseling to avoid precipitating factors such as prolonged
standing/sitting, constipation, and delay of defecation is also appropriate.

Ref: Mounsey AL, Henry SL: Clinical inquiries. Which treatment works best for hemorrhoids? J Fam Pract 2009;58(9):492-493.
2) Bope ET, Kellerman RD (eds): Conns Current Therapy 2013. Elsevier Saunders, 2013, pp 525-528.

Item 100

ANSWER: C

A Cochrane review of 15 studies involving 922 patients showed that antidepressants had a beneficial effect
on the symptoms of irritable bowel syndrome (IBS). Both SSRIs and tricyclic antidepressants have shown
benefit. Another Cochrane review of 12 randomized, controlled trials did not show any benefit from the
use of fiber in any type of IBS. Antibiotics have been shown to have some beneficial effects, but neomycin
is used only in constipation-predominant IBS. Lubiprostone is a selective C-2 chloride channel activator
and can be used for patients with chronic constipation. Alosetron is a 5-hydroxytryptamine 3 antagonist
and is FDA approved to treat severe diarrhea-predominant IBS only in women who have not improved with
conventional therapy. Alosetron is associated with uncommon but serious adverse events (ischemic colitis,
constipation, death) and its use is restricted in the United States. Other potentially beneficial therapies for
IBS include peppermint oil, psychological treatments, exercise, and probiotics.

Ref: Wilkins T, Pepiton C, Alex B, Schade RR: Diagnosis and management of IBS in adults. Am Fam Physician
2012;86(5):419-426.

Item 101

ANSWER: E

This patient is experiencing the Jarisch-Herxheimer reactionan acute, transient, febrile reaction that
occurs within the first few hours after treatment for syphilis. The condition peaks at 68 hours and
disappears within 1224 hours after therapy. The temperature elevation is usually low grade, and there is
often associated myalgia, headache, and malaise. It is usually of no clinical significance and may be treated
with salicylates in most cases. The pathogenesis of the reaction is unclear, but it may be due to liberation
of antigens from the spirochetes.

Ref: Goldman L, Schafer AI (eds): Goldmans Cecil Medicine, ed 24. Elsevier Saunders, 2011, pp e162-e163. 2) Mattei PL,
Beachkofsky TM, Gilson RT, Wisco OJ: Syphilis: A reemerging infection. Am Fam Physician 2012;86(5):433-440.

34
Item 102

ANSWER: E

Testing for diabetes mellitus should be considered in all asymptomatic adults who have a BMI !25 kg/m2
and have one or more additional risk factors such as physical inactivity, a first degree relative with
diabetes, a high-risk ethnicity, hypertension, hyperlipidemia, or polycystic ovary syndrome. In
asymptomatic patients with no risk factors, screening should begin at age 45.

Ref: Ismail-Beigi F: Glycemic management of type 2 diabetes mellitus. N Engl J Med 2012;366(14):1319-1327. 2) American
Diabetes Association: Standards of medical care in diabetes2013. Diabetes Care 2013;36(Suppl 1):S13.

Item 103

ANSWER: D

Myalgias and noninflammatory arthralgias are more likely with aromatase inhibitors. Venous
thromboembolism rarely occurs with these drugs. Endometrial cancer may occur with long-term use of
tamoxifen.

Ref: Khan QJ, ODea AP, Sharma P: Musculoskeletal adverse events associated with adjuvant aromatase inhibitors. J Oncol
2010:65438.

Item 104

ANSWER: D

Myelodysplastic syndrome is a hematologic malignancy with a predisposition to leukemic transformation.


It can present with findings of anemia, thrombocytopenia, neutropenia, or any combination of these.
Anemia occurs in 80%85% of patients and is typically macrocytic.

Ref: Foran JM, Shammo JM: Clinical presentation, diagnosis, and prognosis of myelodysplastic syndromes. Am J Med
2012;125(7 Suppl):S6-S13.

Item 105

ANSWER: B

Education of asthmatic patients is critically important in their follow-up care. This includes informing
patients about the severity of their asthma in addition to instruction about appropriate treatment modalities.
The National Heart, Lung, and Blood Institutes National Asthma Education and Prevention Program uses
the following definitions for asthma severity:

Intermittent: Symptoms less than or equal to twice weekly, nighttime awakenings "2 times/month,
short-acting "-agonist usage "2 days/week, no interference with daily activities, and normal FEV1 and
FEV1/FVC ratio at baseline

Mild Persistent: Symptoms >2 days/week but not daily, nighttime awakenings 34 times/month,
short-acting "-agonist usage >2 days/week but not more than once daily, minor limitation to daily
activities, FEV1 !80% predicted, and normal FEV1/FVC ratio

35
Moderate Persistent: Daily symptoms, nighttime awakenings greater than once weekly but not nightly,
daily use of a short-acting "-agonist, some limitation to daily activity, FEV1 >60% but <80% of
predicted, and FEV1/FVC ratio reduced by 5%

Severe Persistent: Symptoms throughout the day, nighttime awakenings nightly, short-acting "-agonist
usage several times daily, extremely limited daily activities, FEV1 <60% of predicted, and FEV1/FVC
ratio reduced by >5%

Status asthmaticus is a medical emergency and requires emergent treatment in a hospital setting.

Ref: Expert Panel Report 3: Guidelines for the Diagnosis and Management of Asthma. National Asthma Education and
Prevention Program, NIH pub no 07-4051, 2007. 2) Pollart SM, Elward KS: Overview of changes to asthma guidelines:
Diagnosis and screening. Am Fam Physician 2009;79(9):761-767. 3) Pollart SM, Compton RM, Elward KS: Management
of acute asthma exacerbations. Am Fam Physicians 2011;84(1):40-47.

Item 106

ANSWER: B

The treatment of depression in pregnancy is determined by the severity of the symptoms and any past
history of treatment response. For women who have a new onset of mild or moderate depression, it may
be best to start with nonpharmacologic treatments such as supportive psychotherapy or cognitive-behavioral
therapy. These interventions may improve the depression enough that the patient will not need medications.
However, in situations where pharmacologic treatment is clearly indicated, SSRIs are thought to have the
best safety profile. Fluoxetine, sertraline, and citalopram have extensive data to support their safety in
pregnancy and should be considered first line. Paroxetine is the one SSRI that is thought to carry an
increased risk of congenital malformations with first-trimester exposure and should be avoided.

Tricyclic antidepressants are class D in pregnancy. SNRIs do not have as much safety data as SSRIs to
support their use in pregnancy and would be considered a second-line choice. MAOIs are known teratogens
and should be avoided in pregnancy. Stimulants are not first-line agents and should be avoided in
pregnancy.

Ref: Cohen LS, Wang B, Nonacs R, et al: Treatment of mood disorders during pregnancy and postpartum. Psychiatr Clin North
Am 2010;33(2):273-293. 2) Stewart DE: Depression during pregnancy. N Engl J Med 2011;365:1605-1611.

Item 107

ANSWER: D

The concept of Accountable Care Organizations (ACOs) was introduced in 2009 by the Centers for
Medicare and Medicaid Services (CMS) to encourage doctors, hospitals, and other health care providers
to work together to deliver high-quality care and spend health care dollars more wisely. The ACO concept,
together with a shared savings program, has had difficulty penetrating smaller practices and more rural
regions of the country. There is also concern that ACOs may allow larger systems to work as a monopoly
as an unintended consequence. For this reason the Department of Justice and the Federal Trade
Commission are monitoring these organizations as they develop.

36
This new strategy of shared savings through coordinated health care is an alternative to the Sustainable
Growth Rate (SGR) formula that CMS had previously hoped would contain health-care costs. The SGR
was created to prevent Medicare rates from growing faster than the GDP. The high-profile topic of
requiring individuals to carry health insurance has also been part of governmental reform initiatives but
is not directly related to ACOs.

Ref: Dove JT, Weaver WD, Lewin J: Health care delivery system reform: Accountable care organizations. J Am Coll Cardiol
2009;54(11):985-988. 2) Accountable Care Organizations. Centers for Medicare and Medicaid Services, 2013.

Item 108

ANSWER: E

Blood transfusions should be administered to patients with upper gastrointestinal bleeding who have a
hemoglobin level "7.0 g/dL (SOR C). According to the Rockall risk scoring system, this patients
mortality risk from gastrointestinal bleeding is low, based on the following: age <60, systolic blood
pressure !100 mm Hg, heart rate <100 beats/min, no shock, and no major comorbidities. The
Mallory-Weiss tear adds no points to his total score, and the only stigmata of recent hemorrhage is a dark
spot in an otherwise clean ulcer base, which also adds no points. His only scored finding is the presence
of the ulcer, which adds a single point to his score.

Patients with low-risk peptic ulcer bleeding based on clinical and endoscopic criteria can be discharged
from the hospital on the same day as endoscopy (SOR C). Routine second-look endoscopy is not
recommended in patients with upper gastrointestinal bleeding who are not considered to be at high risk for
rebleeding (SOR C). Arteriography with embolization is indicated only in patients with persistent bleeding.

Ref: Wilkins T, Khan N, Nabh A, Schade RR: Diagnosis and management of upper gastrointestinal bleeding. Am Fam Physician
2012;85(5):469-476.

Item 109

ANSWER: A

Plain radiography should be the initial imaging modality to diagnose stress fractures (SOR C). One
algorithm advocates radiography 2 weeks after the onset of symptoms (if symptoms persist), with repeat
radiography the following week before performing more advanced imaging. An expert panel of the
American College of Radiology recommends that MRI be considered next if plain radiography is negative.

Ref: Patel DS, Roth M, Kapil N: Stress fractures: Diagnosis, treatment , and prevention. Am Fam Physician 2011;83(1):39-46.

Item 110

ANSWER: B

In general, when young children are found to be behind schedule in receiving recommended
immunizations, catch-up immunization is important. However, the rotavirus series should not be started
past 15 weeks of age, or continued past 8 months of age. This child should have received hepatitis B
vaccine at 6 months of age, and should be given a catch-up dose. The MMR and varicella vaccines are
recommended at the 12-month visit.

Ref: Loehr J: ACIP releases 2013 immunization schedules. Am Fam Physician 2013;87(3):204-210.

37
Item 111

ANSWER: D

In patients with type 2 diabetes mellitus, the single most important predictor of severe hypoglycemia is a
previous history of severe hypoglycemia that required external assistance. It is thought that hypoglycemia
reduces the bodys protective responses (glucagon and epinephrine) to subsequent episodes of
hypoglycemia (SOR C). Increased blood glucose level goals and increased self-monitoring of blood glucose
are the most important measures for avoiding further episodes.

Less significant risk factors for hypoglycemia include advanced age, use of five or more medications,
African-American ethnicity, and recent hospital discharge (SOR B). Diabetic autonomic neuropathy may
be a risk factor but this has not been definitively established.

Ref: McCall AL: Insulin therapy and hypoglycemia. Endocrinol Metab Clin North Am 2012;41(1):57-87.

Item 112

ANSWER: A

A persistent occiput posterior position is associated with a higher risk of cesarean delivery and assisted
vaginal delivery, and a lower chance of spontaneous vaginal delivery. Assisted vaginal deliveries are
associated with a higher rate of third- and fourth-degree perineal lacerations and postpartum hemorrhage.

Ref: Cunningham FG, Leveno KJ, Bloom SL, et al (eds): Williams Obstetrics, ed 23. McGraw-Hill Medical, 2010, p 480.

Item 113

ANSWER: E

The primary care physician can assess decision-making capacity based on the patients ability to reason,
communicate, understand the proposed treatment, and grasp the consequences of accepting or declining
the suggested treatment.

Formal mental status testing and determination of capacity are different functions. Accurate mental status
testing is helpful for assessing the capacity to make decisions, but there is not a specific score that
determines capacity. However, there is a certain level of cognitive impairment where a patient simply lacks
any ability to receive and process health information. At somewhat higher levels of cognition a patient
might lack specific mental abilities, but still be able to satisfy the requirements for making treatment
decisions. A recent meta-analysis showed that Mini-Mental State Examination (MMSE) scores below 20
increase the likelihood of incapacity (LR 6.3), scores of 2024 have no effect (LR 0.87), and scores
greater than 24 significantly lower the likelihood of incapacity (LR 0.17).

Determination of capacity does not require legal intervention or psychiatric expertise. While there is no
specific test for decision-making capacity, there are instruments available to assist physicians with making
these assessments. The best validated of these is the Aid to Capacity Evaluation (ACE), which is free and
available online (http://www.jointcentreforbioethics.ca/tools/documents/ace.pdf). It can be administered
in 1020 minutes.

38
Competence is a legal term in this situation. Decisions regarding competence are judicial determinations
of the capacity to make nonmedical decisions such as financial decisions. Under the law, adults are
presumed to be competent until a specific action of the appropriate court declares otherwise.

Ref: Sessums LL, Zembrzuska H, Jackson JL: Does this patient have medical decision-making capacity? JAMA
2011;306(4):420-427.

Item 114

ANSWER: C

This patient has a history and physical findings that are consistent with a rotator cuff tear. Most commonly
the mechanism of injury in an acute rotator cuff tear is forced abduction of the arm with significant
resistance. Often this will occur when a person attempts to break a fall with an outstretched hand. There
is usually a sudden sensation of tearing pain in the shoulder. Pain and muscle spasm will limit shoulder
motion. Patients with a large tear cannot initiate shoulder abduction and will have a discrepancy between
active and passive motion. Patients with significant tears will also have a positive drop arm test. This test
is performed by passively abducting the arm to 90 and asking the patient to hold the arm in that position
while the examiner applies pressure on the distal forearm or wrist. The test is positive if the pressure
causes the arm to drop suddenly.

Acute tears are generally managed with a splint and orthopedic referral for surgical repair. Chronic tears
may be managed with shoulder rehabilitation but may ultimately require surgical repair as well.

Bicipital tendinitis is not generally caused by acute trauma, but by irritation and microtrauma due to
repetitive elevation or abduction of the shoulder, causing an inflammatory reaction in the synovial sheath.
Patients generally present with a complaint of pain in the anterior shoulder that radiates into the upper arm.
It is more painful with activity and is worse at night. Abduction and external rotation of the arm
exacerbates the pain. On examination there should be point tenderness in the bicipital groove. Active range
of motion will be limited by pain but passive range of motion will be intact. There should not be any
weakness.

Acromioclavicular separation is usually caused by a fall or a direct blow to the point of the shoulder with
the shoulder abducted. The pain associated with this injury is over the acromioclavicular joint margin and
there may be swelling. Depending on the severity of the injury there may be full range of motion but it
may be restricted due to pain. There should not be any weakness associated with this injury.

A fracture of the humeral head generally occurs with a fall on an outstretched arm or direct blow to the
lateral side of the arm. Generally there is pain or bruising over the fracture site. Movement will be
restricted by pain, but there should not be any weakness.

A tear of the labrum can occur with acute trauma or from repetitive shoulder motion. Acute trauma may
occur from a dislocation of the shoulder, falling on an outstretched arm, or direct blows to the shoulder.
Generally, people with a tear of the labrum will have increased pain with overhead activity, popping or
grinding, loss of strength, and trouble locating a specific point of pain.

Ref: House J, Mooradian A: Evaluation and management of shoulder pain in primary care clinics. South Med J
2010;103(11):1129-1135. 2) Marx JA (ed): Rosens Emergency Medicine: Concepts and Clinical Practice, ed 7. Mosby
Elsevier, 2010, pp 586-587.

39
Item 115

ANSWER: E

Dermatitis herpetiformis is an immune-mediated vesicular disease that usually occurs in the young to
middle-aged. The skin lesions are extremely pruritic grouped vesicles and erosions located on the scalp,
posterior neck, and extensor surfaces of the elbows, knees, and buttocks. Most patients have a subclinical
gluten-sensitive enteropathy that is reversible with a gluten-free diet, which can sometimes control the skin
disease as well.

Erythema multiforme is an acute blistering eruption that occurs in all age groups. Porphyria cutanea tarda
patients develop erosions and bullae on sun-exposed skin. Herpes gestationis is a rare autoimmune
dermatosis of pregnancy, and bullous pemphigoid is an autoimmune blistering disorder seen in the elderly.
Tense blisters and urticarial plaques occur on the flexor surfaces of the arms, legs, axillae, groin, and
abdomen.

Ref: Goldman L, Schafer AI (eds): Goldmans Cecil Medicine, ed 24. Elsevier Saunders, 2011, pp 2525-2529.

Item 116

ANSWER: B

A dorsiflexion injury will typically cause a scaphoid fracture in a young adult, resulting in tenderness to
palpation over the anatomic snuffbox. A plain posterior-anterior wrist radiograph is often normal.
However, a special view with the wrist prone in ulnar deviation elongates the scaphoid, often
demonstrating subtle fractures. Hook of the hamate fractures cause tenderness at the proximal hypothenar
area 1 cm distal to the flexion crease of the wrist. When this fracture is suspected, carpal tunnel and
supinated oblique view radiographs should be obtained. A scapholunate dislocation can be identified with
a clenched-fist view and a supinated view with the wrist in ulnar deviation.

Ref: Forman TA, Forman SK, Rose NE: A clinical approach to diagnosing wrist pain. Am Fam Physician
2005;72(9):1753-1758. 2) DeLee JC, Drez D Jr, Miller MD (eds): DeLee & Drez's Orthopaedic Sports Medicine:
Principles and Practice, ed 3. Elsevier Saunders, 2010, pp 1335-1336.

Item 117

ANSWER: B

This patient has a mallet fracture. These fractures are caused by an axial load to the tip of an extended
finger that causes forced flexion at the distal interphalangeal (DIP) joint. This leads to a fracture at the
dorsal surface of the proximal distal phalanx where the terminal finger extensor mechanism inserts. The
most appropriate treatment of a mallet fracture is to splint the DIP joint in extension for 8 weeks. The joint
should remain in full extension for optimal healing. Any flexion of the finger may affect healing and extend
the treatment time.

Surgical management has been recommended for fractures that involve more than 30% of the joint space,
but a small study showed there was no difference in outcomes compared to treatment with extension
splints. Buddy taping would not offer enough support to maintain the finger in extension at all times.

Ref: Borchers JR, Best TM: Common finger fractures and dislocations. Am Fam Physician 2012;85(8):805-810.

40
Item 118

ANSWER: E

In women 3065 years old, screening for cervical cancer with cervical cytology and HPV testing is
recommended every 5 years. An alternative screening recommendation is to perform cervical cytology
only, at 3-year intervals. A population study of 331,818 women demonstrated a 0.016% risk of cancer in
the 5 years after having a negative result on both cervical cytology and an HPV test.

Ref: American College of Obstetricians and Gynecologists: Screening for cervical cancer. ACOG Practice Bulletin, no 131,
2012. 2) US Preventive Services Task Force: Screening for cervical cancer. April 2012.

Item 119

ANSWER: C

The most likely diagnosis for this patients knee pain is osteoarthritis. While he is likely to have disease
in both knees, it is common for patients to have unilateral symptoms, especially early on. Although
osteoarthritis is mainly a clinical diagnosis, plain radiography is the diagnostic study of choice if there is
concern about other diagnostic possibilities. Narrowing of the medial compartment of the knee joint is
typically the first radiographic finding; osteophytes are also commonly seen on plain films. In the scenario
presented here, there is no need for laboratory testing at this time.

Ref: Sinusas K: Osteoarthritis: Diagnosis and treatment. Am Fam Physician 2012;85(1):49-56.

Item 120

ANSWER: B

There is good evidence that psychostimulants reduce symptoms of depression within days, making
methylphenidate a good choice for this patient (SOR B). Electroconvulsive therapy is contraindicated due
to her brain lesions. Mirtazapine, fluoxetine, and nortriptyline all take at least 34 weeks to have any
antidepressant effects, and would not be appropriate given the patients life expectancy (SOR B).

Ref: Widera EW, Block JD: Managing grief and depression at the end of life. Am Fam Physician 2012;86(3):259-264.

Item 121

ANSWER: A

This patient has Bells palsy. Only corticosteroids have been shown to improve the outcome. Antiviral
agents have little value in the treatment of Bells palsy. Thrombolytic therapy may be useful for a patient
with central facial nerve weakness if it is due to a vascular event (level of evidence 3; SOR A).

Ref: Holland J, Bernstein J: Bell palsy. Am Fam Physician 2011;84(8):947-948.

41
Item 122

ANSWER: B

Marfan syndrome is an autosomal dominant disease manifested by skeletal, ophthalmologic, and


cardiovascular abnormalities. Men taller than 72 inches and women taller than 70 inches who have two
or more manifestations of Marfan syndrome should be screened by echocardiography for associated cardiac
abnormalities. These signs and symptoms include cardiac murmurs or clicks, kyphoscoliosis, anterior
thoracic deformity, arm span greater than height, upper to lower body ratio more than 1 standard deviation
below the mean, myopia, and an ectopic lens.

Athletes with a family history of Marfan syndrome should also be screened, whether they have
manifestations themselves or not. Patients with Marfan syndrome who have echocardiographic evidence
of aortic abnormalities should be placed on "-blockers and monitored with echocardiography every 6
months.

Ref: Bader RS, Goldberg L, Sahn DJ: Risk of sudden cardiac death in young athletes: Which screening strategies are
appropriate? Pediatr Clin North Am 2004;51(5):1421-1441. 2) Kliegman RM, Stanton BF, Geme JW III, et al (eds):
Nelson Textbook of Pediatrics, ed 19. Elsevier Saunders, 2011, pp 2440-2446.

Item 123

ANSWER: B

This patient most likely has an anaerobic bacterial infection. Penicillin was used to treat these infections
in the past, but because of the emergence of "-lactamaseproducing organisms, clindamycin is now the
drug of choice. Clindamycin has broader coverage against both pulmonary anaerobes and facultative
aerobes such as Staphylococcus aureus and Klebsiella, which are often seen with lung abscesses.
Metronidazole has anaerobic coverage, but not for the anaerobic species often involved in pulmonary
infections, and is therefore associated with a high failure rate when used to treat lung abscesses.
Doxycycline does not cover anaerobes. Trimethoprim/sulfamethoxazole is also not considered a good
anaerobic antibiotic.

Ref: Longo DL, Fauci AS, Kasper DL, et al (eds): Harrisons Principles of Internal Medicine, ed 18. McGraw-Hill, 2012, pp
2142-2147.

Item 124

ANSWER: A

The diagnosis of delirium is based entirely on the history and physical examination. No laboratory tests,
imaging studies, or other tests are more accurate than clinical assessment.

Ref: Marcantonio ER: In the clinic. Delirium. Ann Intern Med 2011;154(11):ITC6 1-14.

42
Item 125

ANSWER: C

While it is certainly appropriate for the nurse practitioner or physician who ordered the test to notify the
patient of mammography results, the facility performing the test is legally responsible. This is specified
by the federal Mammography Quality Standards Act, first passed by Congress in 1992.

Ref: Mammography Quality Standards Act Regulations. US Food and Drug Administration, 2009, sec 900.12(c)(2).

Item 126

ANSWER: E

Plantar fasciitis is characterized by pain that is worse with the first few steps in the morning or after a
prolonged rest. NSAIDs may help with the discomfort, but prompt relief of the pain by any modality is
not common. The pain is typically in the medial heel. While 50% of people with plantar fasciitis have heel
spurs on radiographs, this finding is not causative or diagnostic. The diagnosis is made clinically.

Ref: Tu P, Bytomski JR: Diagnosis of heel pain. Am Fam Physician 2011;84(8):909-916.

Item 127

ANSWER: A

Heat exhaustion and heatstroke are both on the continuum of heat-related illness. Heatstroke is a much
more severe condition than heat exhaustion. Evidence of central nervous system dysfunction is evidence
of heatstroke rather than heat exhaustion, even if other symptoms are not severe and point to heat
exhaustion. Heatstroke is a medical emergency.

Ref: Glazer JL: Management of heatstroke and heat exhaustion. Am Fam Physician 2005;71(11):2133-2140, 2141-2142. 2)
Marx JA (ed): Rosens Emergency Medicine: Concepts and Clinical Practice, ed 7. Mosby Elsevier, 2010, pp 1887-1890.

Item 128

ANSWER: E

Bupropion can lower the seizure threshold and should not be used in patients who have a history of a
seizure disorder or who drink heavily. A history of the other medical conditions listed does not
contraindicate the use of bupropion.

Ref: Fiore MC, Baker TB: Treating smokers in the health care setting. N Engl J Med 2011;365(13):1222-1231.

43
Item 129

ANSWER: C

This case illustrates the classic history and physical findings of iliotibial band syndrome. Pain occurs most
frequently at the site where the tendon crosses over the lateral femoral epicondyle. With osteoarthritis or
a meniscal tear there would be pain in the joint space with palpation. Osgood-Schlatter disease is more
common in younger adolescents and is characterized by tenderness of the tibial tubercle at the distal
insertion of the patellar ligament. Pes anserine bursitis is characterized by pain in the medial knee distal
to the joint space, at the conjoined tendon of the sartorius, gracilis, and semitendinosus.

Ref: Khaund R, Flynn SH: Iliotibial band syndrome: A common source of knee pain. Am Fam Physician 2005;71(8):1545-1550.
2) Longo DL, Fauci AS, Kasper DL, et al (eds): Harrisons Principles of Internal Medicine, ed 18. McGraw-Hill, 2012,
pp 2860-2863.

Item 130

ANSWER: C

Metformin is contraindicated in males with creatinine levels >1.5 mg/dL (SOR C) and should be stopped
in this patient. The recent episode of diarrhea may have exacerbated renal disease associated with
long-standing hypertension and diabetes mellitus. Dose modifications for sitagliptin and simvastatin may
be needed with renal dysfunction, but these medications would not have to be stopped. Carvedilol and
clopidogrel do not require a change in dosage with renal dysfunction because their metabolism is largely
hepatic.

Ref: Ripsin CM, Kang H, Urban RJ: Management of blood glucose in type 2 diabetes mellitus. Am Fam Physician
2009;79(1):29-36, 42.

Item 131

ANSWER: D

Constipation is a very common side effect of opioids that does not resolve with time, unlike many other
adverse effects. Constipation is easier to prevent than to treat, so it is important to start an appropriate
bowel regimen with the initiation of opioid therapy. Fiber supplements and detergents (such as docusate)
are inadequate for the prevention of opioid-induced constipation. Metoclopramide is used for nausea and
increases gastric motility, but is not indicated in the treatment of constipation. Polyethylene glycol,
lactulose, magnesium hydroxide, and senna with docusate are all appropriate in this situation.

Ref: Clary PL, Lawson P: Pharmacologic pearls for end-of-life care. Am Fam Physician 2009;79(12):1059-1065.

44
Item 132

ANSWER: C

Patients diagnosed with pharyngeal gonorrhea may be asymptomatic, or they may have oropharyngeal
erythema and an exudate, along with cervical lymphadenopathy. Gonorrheal pharyngitis can coexist with
cervical gonorrhea. First-line treatment for gonorrheal pharyngitis is ceftriaxone, 250 mg intramuscularly
once (SOR C). Although chlamydial pharyngitis is uncommon, patients with pharyngitis due to gonorrhea
are often coinfected with genital Chlamydia and should therefore be treated empirically for chlamydial
infection. There is no need to perform a pelvic examination or order additional testing. All patients who
are diagnosed with gonorrhea should be retested at 36 months due to a high rate of reinfection (SOR C).
Partner treatment is highly recommended.

Ref: Mayor MT, Roett MA, Uduhiri KA: Diagnosis and management of gonococcal infections. Am Fam Physician
2012;86(10):931-938.

Item 133

ANSWER: E

Severe hypophosphatemia is a medical emergency. In poorly nourished patients, refeeding syndrome can
occur. Symptoms usually develop by the second or third day of improved nutrition, and are often
multisystemic. Findings may include weakness, confusion, dysrhythmias, respiratory failure, heart failure,
hypotension, ileus, metabolic acidosis, seizures, coma, and sudden death. This constellation of problems
results from decreased insulin secretion as stores of intracellular phosphate become depleted. Providing
carbohydrates through intravenous fluids or refeeding increases insulin secretion, which stimulates cells
to take up phosphate, causing severe hypophosphatemia. In this setting cells are unable to produce enough
2,3 diphosphoglycerate and adenosine triphosphate to meet metabolic demands.

While hypoglycemia is also a medical emergency, this patients glucose level is not low enough to cause
these symptoms. Similarly, renal failure of some type is present, as is an elevated creatine kinase
suggesting rhabdomyolysis; however, neither of these problems would be expected to cause this patients
symptoms. Hypocalcemia can cause multisystemic problems, including weakness and seizures, but the
patients calcium level is not critically low and hypocalcemia is not associated with hypotension.

Ref: Judge BS, Eisenga BH: Disorders of fuel metabolism: Medical complications associated with starvation, eating disorders,
dietary fads, and supplements. Emerg Med Clin North Am 2005;23(3):789-813. 2) Goldman L, Schafer AI (eds):
Goldmans Cecil Medicine, ed 24. Elsevier Saunders, 2011, pp 755-756.

Item 134

ANSWER: C

There are many causes of wheezing in infants and children. Wheezing associated with feeding is most
commonly due to gastroesophageal reflux disease (level of evidence 3). Tracheoesophageal fistula and
laryngeal cleft also cause wheezing associated with feeding, but are rare. Foreign body aspiration is most
common between 8 months and 4 years of age and the child is most likely to have a history of the sudden
onset of wheezing associated with choking. The wheezing present with tracheomalacia is position related.

Ref: Weiss LN: The diagnosis of wheezing in children. Am Fam Physician 2008;77(8):1109-1114.

45
Item 135

ANSWER: E

Rhus dermatitis is an allergic phytodermatitis caused by poison ivy and other members of the
Anacardiaceae plant family, including poison oak and poison sumac. Urushiol, an oleoresin found in these
plants, is one of the most common sensitizers in the United States. Contact with these plants results in the
characteristic pruritic erythematous linear lesions with papules and edematous plaques, vesicles, and/or
bullae. The lesions typically appear 48 hours to a few days after exposure and occur on exposed areas.
However, lesions can also occur on remote sites such as the face and penis due to transfer of the oleoresin,
and may not have the characteristic linear appearance. Nickel, which is contained in jewelry and metals
in clothing, produces an eczematous eruption, often with lichenification over the affected areas. Balsam
of Peru (present in topical medications) and neomycin cause allergic contact dermatitis in the area of
application and are nonlinear in appearance. Bedbug bites appear as pruritic erythematous macules with
central hemorrhagic puncta, often in a grouped distribution (SOR C).

Ref: Habif TP: Clinical Dermatology: A Color Guide to Diagnosis and Therapy, ed 5. Mosby Elsevier, 2010, pp 12-15, 34-39.
2) Studdiford JS, Conniff KM, Trayes KP, Tully AS: Bedbug infestation. Am Fam Physician 2012;86(7):653-658. 3)
Longo DL, Fauci AS, Kasper DL, et al (eds): Harrisons Principles of Internal Medicine, ed 18. McGraw-Hill, 2012, pp
395-404.

Item 136

ANSWER: E

Basic activities of daily living, such as dressing, eating, toileting, and grooming, are generally intact in
early dementia. In contrast, instrumental activities of daily living, such as managing money and
medications, shopping, cooking, housekeeping, and transportation, which often require calculation or
planning, are frequently impaired in early dementia.

Ref: Simmons BB, Hartmann B, Dejoseph D: Evaluation of suspected dementia. Am Fam Physician 2011;84(8):895-902.

Item 137

ANSWER: A

Intoeing, as described in this patient, is usually caused by internal tibial torsion. This problem is believed
to be caused by sleeping in the prone position and sitting on the feet. In 90% of cases internal tibial torsion
gradually resolves without intervention by the age of 8. Avoiding sleeping in a prone position enhances
resolution of the problem. Night splints, orthotics, and shoe wedges are ineffective. Surgery (osteotomy)
has been associated with a high complication rate, and is therefore not recommended in mild cases before
the age of 8.

Ref: Sass P, Hassan G: Lower extremity abnormalities in children. Am Fam Physician 2003;68(3):461-468. 2) Kliegman RM,
Stanton BF, Geme JW III, et al (eds): Nelson Textbook of Pediatrics, ed 19. Elsevier Saunders, 2011, p 2347.

46
Item 138

ANSWER: C

Studies show that few residents have an advance directive at the time of admission to a nursing home.
Studies also show that residents with advance directives are more likely to die in a nursing home with
hospice care, are less likely to have a feeding tube or ventilator in the last month of life, require fewer
resources, and are hospitalized less (SOR B).

Ref: Mollow DW, Guyatt GH, Russo R, et al: Systematic implementation of an advance directive program in nursing homes:
A randomized controlled trial. JAMA 2000;283(11):1437-1444. 2) Unwin BK, Porvaznik M, Spoelhof GD: Nursing home
care: Part I. Principles and pitfalls of practice. Am Fam Physician 2010;81(10):1219-1227.

Item 139

ANSWER: A

The major indication for joint replacement is severe joint pain. Loss of joint function and radiographic
evidence of severe destruction of the joint may also be considered in the decision. The appearance of the
joint and the status of the contralateral joint may be minor considerations. Surgical insertion of a foreign
body into an infected joint is contraindicated.

Ref: Zhang W, Moskowitz RW, Nuki G, et al: OARSI recommendations for the management of hip and knee osteoarthritis, Part
II: OARSI evidence-based, expert consensus guidelines. Osteoarthritis Cartilage 2008;16(2):137-162.

Item 140

ANSWER: C

This audiogram depicts a sensorineural loss in a notch pattern, with the greatest hearing loss at 3500 Hz.
This pattern is most consistent with a noise-induced hearing loss that typically develops gradually as a
result of chronic exposure to excessive sound levels. A vestibular schwannoma most commonly results in
a high-frequency sensorineural hearing loss without the notch. Early on, Menieres disease causes a
low-frequency hearing loss, but later it may cause a peaked audiogram with both low- and high-frequency
loss. Complete occlusion of the ear canal with cerumen results in a flat conductive hearing loss across all
frequencies. Tympanic membrane perforation causes a low- to mid-frequency conductive hearing loss.

Ref: Walker JJ, Cleveland LM, Davis JL, Seales JS: Audiometry screening and interpretation. Am Fam Physician
2013;87(1):41-47.

Item 141

ANSWER: E

Community-acquired pneumonia in children over the age of 5 is most commonly due to Mycoplasma
pneumoniae, Chlamydophila pneumoniae, and Streptococcus pneumoniae. Less common bacterial
infections include Haemophilus influenzae, Staphylococcus aureus, and group A Streptococcus. Initial
treatment with antibiotics is empiric, as the pathogen is usually unknown at the time of diagnosis. The
choice in children is based on age, severity of illness, and local patterns of resistance. Children age 516
years who can be treated as outpatients are usually treated with oral azithromycin. For patients requiring
inpatient management, intravenous cefuroxime plus either intravenous erythromycin or azithromycin is
recommended.

47
Ref: Stuckey-Schrock K, Hayes BL, George CM: Community-acquired pneumonia in children. Am Fam Physician
2012;86(7):661-667.

Item 142

ANSWER: C

In 50%90% of patients, pityriasis rosea starts with an erythematous, scaly, oval patch a few centimeters
in diameter. This is usually followed within a few days by smaller patches on the trunk and sometimes the
proximal extremities. Pityriasis rubra pilaris is a rare disease with five types. The classic adult type begins
with a small red plaque on the face or upper body that gradually spreads to become a generalized eruption.
The other conditions listed typically begin with multiple lesions.

Ref: Habif TP: Clinical Dermatology: A Color Guide to Diagnosis and Therapy, ed 5. Mosby Elsevier, 2010, pp 309-311,
316-319, 332-333, 537-540, 771. 2) Goldman L, Schafer AI (eds): Goldmans Cecil Medicine, ed 24. Elsevier Saunders,
2011, pp 2518-2520.

Item 143

ANSWER: B

This patient is experiencing delirium, which is common in the last weeks of life, occurring in 26%44%
of persons hospitalized with advanced cancer and in up to 88% of persons with a terminal illness. In
studies of a palliative care population it was possible to determine a cause for delirium in less than 50%
of cases. There is a consensus based on observational evidence and experience that antipsychotic agents
such as haloperidol are effective for the management of delirium, and they are widely used. However,
there have been few randomized, controlled trials to assess their effectiveness.

While benzodiazepines are used extensively in persons with delirium who are terminally ill, there is no
evidence from well-conducted trials that they are beneficial. Trazodone is an antidepressant that is
sometimes used for insomnia. Scopolamine is an anticholinergic that is used to reduce respiratory
secretions in hospice patients, but its anticholinergic side effects would increase delirium severity.
Amitriptyline also has significant anticholinergic properties.

Ref: Keeley PW: Delirium at the end of life. Clinal Evidence Handbook, 2009, pp 595-596. 2) Keeley PW: Delirium at the end
of life. Am Fam Physician 2010;81(10):1260-1261.

Item 144

ANSWER: E

Patients with diabetes mellitus, atherosclerosis, and end-organ damage benefit from ACE inhibitors and
angiotensin receptor blockers (ARBs) equally when they are used to prevent progression of diabetic
nephropathy. Combining an ACE inhibitor with an ARB is not recommended, as it provides no additional
benefit and leads to higher creatinine levels, along with an increased likelihood that dialysis will become
necessary.

Ref: Roett MA, Liegl S, Jabbarpour Y: Diabetic nephropathyThe family physicians role. Am Fam Physician
2012;85(9):883-889.

48
Item 145

ANSWER: E

The initial treatment of mild to moderate allergic rhinitis should be an intranasal corticosteroid alone, with
the use of second-line therapies for moderate to severe disease (SOR A). The adverse effects and higher
cost of intranasal antihistamines, as well as their decreased effectiveness compared with intranasal
corticosteroids, limit their use as first- or second-line therapy for allergic rhinitis. Moderate to severe
disease not responsive to intranasal corticosteroids should be treated with second-line therapies, including
antihistamines, decongestants, cromolyn, leukotriene receptor antagonists, and nonpharmacologic therapies
such as nasal irrigation.

Ref: Sur DK, Scandale S: Treatment of allergic rhinitis. Am Fam Physician 2010;81(12):1440-1446.

Item 146

ANSWER: B

Synthetic opioid medications are generally not discovered when screening urine for opioids using an
immunoassay method. These synthetic opioids include fentanyl, methadone, and oxymorphone. Opioid
reversal should still be considered in this patient, and a search for a fentanyl patch is indicated.

Ref: Moeller KE, Lee KC, Kissack JC: Urine drug screening: Practical guide for clinicians. Mayo Clin Proc 2008;83(1):66-76.

Item 147

ANSWER: B

Primary hyperaldosteronism is the most common cause of secondary hypertension in the middle-aged
population, and can be diagnosed from a renin/aldosterone ratio. This diagnosis is further suggested by
the finding of hypokalemia, which suggests hyperaldosteronism even though it is not present in the
majority of cases.

An echocardiogram would help make a diagnosis of coarctation of the aorta, but this is more common in
younger patients. Renal MRA may demonstrate renal artery stenosis, but this condition is more common
in older patients. Sleep apnea is increasing in prevalence along with the rise in obesity, but it is not
suggested by this case. A 24-hour urine catecholamine test is used to diagnose pheochromocytoma, which
is not suggested by this patients findings. Pheochromocytoma is also less common than aldosteronism
(SOR C).

Ref: Viera AJ, Neutze DM: Diagnosis of secondary hypertension: An age-based approach. Am Fam Physician
2010;82(12):1471-1478.

49
Item 148

ANSWER: A

People taking carbamazepine have a five- to eightfold increased risk of developing agranulocytosis.
Baseline values including a CBC, serum electrolytes, and liver enzymes should be obtained before the drug
is started, and the patient should be monitored with periodic hematologic testing. The other medications
listed are not associated with agranulocytosis. Aripiprazole and olanzapine carry black box warnings for
an increased risk of death in the elderly. Lithium is associated with lithium toxicity and thyroid
dysfunction. Imipramine carries a warning for cardiac toxicity, and EKG monitoring is recommended.

Ref: Stem TA, Rosenbaum JF, Fava M, et al (eds): Massachusetts General Hospital Comprehensive Clinical Psychiatry. Mosby,
2008, pp 40-42.

Item 149

ANSWER: A

Anticholinesterase inhibitors such as donepezil are considered first-line therapy for patients with mild to
moderate Alzheimers disease (SOR A). Memantine is an NMDA receptor antagonist and is often used in
combination with anticholinesterase inhibitors for moderate to severe Alzheimers disease, but it has not
been shown to be effective as a single agent for patients with mild to moderate disease. There is not enough
evidence to support the use of selegiline, a monoamine oxidase type B inhibitor, in the treatment of
Alzheimers disease. Risperidone and other antipsychotic medications are not approved by the Food and
Drug Administration for treatment of Alzheimers disease, but can sometimes be helpful in controlling
associated behavioral symptoms. Studies of ginkgo biloba extract have not shown a consistent, clinically
significant benefit in persons with Alzheimers disease.

Ref: Winslow BT, Onysko MK, Stob CM, Hazlewood KA: Treatment of Alzheimer disease. Am Fam Physician
2011;83(12):1403-1412.

Item 150

ANSWER: D

Acute subacromial bursitis is common and is often associated with calcific deposits in the supraspinatus
tendon, pain on abduction, and local tenderness. Bicipital tendinitis results in tenderness on palpation of
the tendon of the long head of the biceps. A rotator cuff tear usually results from an injury, and affects
range of motion. Osteoarthritis seldom causes acute, severe pain. A frozen shoulder may result from
subacromial bursitis and presents with limitation of shoulder motion.

Ref: Marx JA (ed): Rosens Emergency Medicine: Concepts and Clinical Practice, ed 7. Mosby Elsevier, 2010, pp 585-590.
2) Roberts JR, Hedges JR (eds): Clinical Procedures in Emergency Medicine, ed 5. Saunders Elsevier, 2010, pp 950-951.

50
Item 151

ANSWER: E

Once considered generally beneficial to the running athlete, preparticipation static stretching has been
found lacking in terms of benefit and even detrimental when subjected to scientific study. There is strong
evidence that static stretching significantly slows performance in sprints up to 100 meters. Studies have
failed to demonstrate that static stretching before running significantly decreases the likelihood of muscular
injury of the lower limbs or results in a measurable reduction of delayed-onset muscle soreness. Limited
evidence suggests that preparticipation static stretching, when performed alone, adversely affects both
strength and endurance in elite athletes but has little measurable effect on amateur and casual athletes.
Based on current understanding of sports performance, static stretching is of most benefit when performed
during the cool-down period following exercise, which has been found to increase flexibility, and is best
avoided immediately before athletic endeavors. A preparatory aerobic warm-up combined with dynamic
range-of-motion exercises may be of some benefit for runners.

Ref: Herbert RD, de Noronha M, Kamper SJ: Stretching to prevent or reduce muscle soreness after exercise. Cochrane Database
Syst Rev 2011;(7):CD004577. 2) Yeung SS, Yeung EW, Gillespie LD: Interventions for preventing lower limb soft-tissue
running injuries. Cochrane Database Syst Rev 2011;(7):CD001256.

Item 152

ANSWER: B

A positive Lachman test indicates that the anterior cruciate ligament may be torn. The posterior drawer
test evaluates posterior cruciate ligament stability. The McMurray and Thessaly assessments test for
meniscal tears. The ballottement test is for detecting intra-articular knee effusion.

Ref: Grover M: Evaluating acutely injured patients for internal derangement of the knee. Am Fam Physician
2012;85(3):247-252.

Item 153

ANSWER: B

HPV vaccine is given as a three-dose series, so this patient is due for her third dose. The recommended
interval between the first and third doses is 6 months, with approximately 4 months recommended between
the second and third doses; however, the series can safely be completed at longer intervals (SOR C). The
patient received her second dose at age 12 and she is now 14 years of age, so it has been over 4 months.
She was up to date on all immunizations at age 8, so it can be assumed that she has received her rubella,
measles, and polio vaccinations. There is currently not a vaccine approved for hepatitis C.

Ref: 2012 Recommended immunizations for children from 7 through 18 years old. Centers for Disease Control and Prevention,
American Academy of Pediatrics, American Academy of Family Physicians, 2012.

51
Item 154

ANSWER: A

Testosterone replacement can induce polycythemia, so baseline hematocrit/hemoglobin levels should be


obtained prior to treatment and repeated approximately every 6 months. FSH is not relevant in the workup
or treatment of hypogonadism. Testosterone treatment does not directly affect glucose tolerance or
electrolytes, so baseline studies and follow-up are not necessary.

Ref: Bassil N, Morley JE: Late-life onset hypogonadism: A review. Clin Geriatr Med 2010;26(2):197-222.

Item 155

ANSWER: E

Emerging data on low-carbohydrate diets is mostly encouraging, in that these diets do not seem to cause
the expected increases in blood pressure, LDL-cholesterol levels, or triglyceride levels that the medical
community had first assumed. Although low-carbohydrate diets have been shown to result in clinically
meaningful weight loss, reduced-calorie diets appear to result in similar weight loss regardless of which
macronutrients they emphasize. This patient has symptoms of metabolic syndrome and has a higher risk
of glucose intolerance or diabetes mellitus. Low-carbohydrate diets have been shown to reduce insulin
resistance at least as well as, if not better than, traditional diet plans.

Ref: Nordmann AJ, Nordmann A, Briel M, et al: Effects of low-carbohydrate vs low-fat diets on weight loss and cardiovascular
risk factors: A meta-analysis of randomized controlled trials. Arch Intern Med 2006;166(3):285-293. 2) Barker LR, Burton
JR, Zieve PD (eds): Principles of Ambulatory Medicine, ed 7. Lippincott Williams & Wilkins, 2007, p 1429. 3) Gardner
CD, Kiazand A, Alhassan S, et al: Comparison of the Atkins, Zone, Ornish, and LEARN diets for change in weight and
related risk factors among overweight premenopausal women: The A TO Z Weight Loss Study: A randomized trial. JAMA
2007;297(9):969-977. 4) Sacks FM, Bray GA, Carey VJ, et al: Comparison of weight-loss diets with different
compositions of fat, protein, and carbohydrates. N Engl J Med 2009;360(9):859-873. 5) Longo DL, Fauci AS, Kasper DL,
et al (eds): Harrisons Principles of Internal Medicine, ed 18. McGraw-Hill, 2012, p 633.

Item 156

ANSWER: E

Certain antiepileptic drugs induce hepatic metabolism of estrogen and progestin (carbamazepine,
oxcarbazepine, phenobarbital, phenytoin, and topiramate). This can potentially lead to failure of any
contraceptive that contains estrogen and progestin. Progestin-only pills are most effective in women who
are exclusively breastfeeding. They are not as effective in pregnancy prevention in other circumstances.
Another effective option for women taking antiepileptic medications would be an intrauterine device. The
levonorgestrel (progestin only) IUD and copper IUD are acceptable choices even for a nulligravida. The
single-rod implantable progestin system also would be an acceptable choice for this patient.

Ref: Bonnema RA, McNamara MC, Spencer AL: Contraception choices in women with underlying medical conditions. Am Fam
Physician 2010;82(6):621-628.

52
Item 157

ANSWER: B

Patients receiving dual antiplatelet therapy who require bypass surgery should continue taking aspirin.
Clopidogrel or prasugrel should be stopped 5 days before the surgery due to the increased risk of major
bleeding during surgery.

Ref: Guyatt GH, Akl EA, Crowther M, et al: Executive summary: Antithrombotic therapy and prevention of thrombosis, 9th
ed: American College of Chest Physicians Evidence-Based Clinical Practice Guidelines. Chest 2012;141(2 Suppl):7S-47S.

Item 158

ANSWER: C

The EKG shows atrial fibrillation with a rapid ventricular rate. The patient is stable, so initial treatment
should focus on rate control. Intravenous "-blockers or nondihydropyridine calcium channel antagonists
are preferred for initial therapy to control the rate. Amiodarone may be used for rhythm control but would
not be the initial treatment of choice in this case. Cardioversion is not indicated unless the patient becomes
unstable. Adenosine is not a recommended treatment for atrial fibrillation.

Ref: Fuster V, Rydn LE, Cannom DS, et al: ACC/AHA/ESC 2006 Guidelines for the Management of Patients with Atrial
Fibrillation: A report of the American College of Cardiology/American Heart Association Task Force on Practice
Guidelines and the European Society of Cardiology Committee for Practice Guidelines (Writing Committee to Revise the
2001 Guidelines for the Management of Patients With Atrial Fibrillation): Developed in collaboration with the European
Heart Rhythm Association and the Heart Rhythm Society. Circulation 2006;114(7):e257-e354. 2) Gutierrez C, Blanchard
DG: Atrial fibrillation: Diagnosis and treatment. Am Fam Physician 2011;83(1):61-68.

Item 159

ANSWER: A

A decrease in serum sodium concentration does not always indicate a decrease in osmolality of body fluids.
In cases of hyperglycemia, the main cause of the hyponatremia is the glucose-related increase in osmolality
of extracellular fluid, followed by the movement of water from intracellular to extracellular fluid
compartments and a subsequent loss of excessive extracellular fluid and electrolytes. The serum sodium
concentration is also diminished in patients with hyperlipidemia or hyperproteinuria because of the volume
occupied by the lipids or proteins. If the lipids or proteins are removed, the sodium concentration in the
remaining plasma is found to be normal. No treatment is needed for these conditions.

Ref: Rakel RE, Rakel DP (eds): Textbook of Family Medicine, ed 8. Elsevier Saunders, 2011, p 200.

Item 160

ANSWER: D

Guidelines from the American Academy of Pediatrics state that stimulant medication can be prescribed for
preschool children, but only after a thorough trial of behavior modification. Foods and additives have
never been shown to cause or aggravate ADHD. Children with ADHD often have other behavioral
problems such as depression or oppositional-defiant disorder.

53
Ref: Subcommittee on Attention-Deficit/Hyperactivity Disorder; Steering Committee on Quality Improvement and Management,
Wolraich M, et al: ADHD: Clinical practice guideline for the diagnosis, evaluation, and treatment of attention-deficit/
hyperactivity disorder in children and adolescents. Pediatrics 2011;128(5):1007-1022.

Item 161

ANSWER: B

This patient is suffering from patellofemoral pain syndrome, which causes anterior knee pain that is worse
with running downhill. The examination is often normal, although there may be apprehension when the
knee is extended with pressure over the patella and the patella will sometimes track laterally.
Patellofemoral pain syndrome can be treated with exercises to strengthen the quadriceps and hips, and by
using a knee sleeve with a doughnut-type cushion that the patella fits into. Static stretching would not
address the problem. MRI would be indicated if there were joint-line pain or an unstable knee. Pes
anserine bursitis usually causes pain and tenderness medially, below the joint line.

Ref: Bope ET, Kellerman R, Rakel RE (eds): Conns Current Therapy 2011. Elsevier Saunders, 2011, p 1026.

Item 162

ANSWER: A

Rheumatoid arthritis is most often symmetric at presentation and particularly affects the wrists and other
extremity joints that have a high ratio of synovium to articular cartilage. Rheumatoid factor is often
negative in the early months of the disease, although it may be positive later. Radiographs and laboratory
tests are helpful, but the diagnosis is primarily clinical. Osteoarthritis of the wrists usually involves the
carpal-metacarpal joint of the thumb primarily, and the joint would be red if there were an injury.
Fibromyalgia usually involves the soft tissue of the trunk, and there is no evidence of inflammation. Lyme
disease can cause a variety of joint diseases, but not chronic symmetric arthritis.

Ref: Wasserman AM: Diagnosis and management of rheumatoid arthritis. Am Fam Physician 2011;84(11):1245-1252. 2)
Goldman L, Schafer AI (eds): Goldmans Cecil Medicine, ed 24. Elsevier Saunders, 2011, pp 1685-1686.

Item 163

ANSWER: B

The parallelogram shape of this infants head is typical of positional skull deformity, also known as benign
positional molding or occipital plagiocephaly. This condition has been estimated to be present in at least
1 in 300 infants, with some studies showing milder variants in up to 48% of healthy infants. The incidence
of positional skull deformity is increased in children who sleep in the supine position, but switching to
prone sleeping is not recommended because this would increase the risk of sudden infant death syndrome.
The deformity can be prevented by routine switching of the dependent side of the infants head. Supervised
tummy time for 3060 minutes each day can also decrease the amount of flattening and can increase the
childs motor development. Children who have positional skull deformity should also be screened for
torticollis. This condition can prevent correct positioning and is remedied with physical therapy techniques.

Positional skull deformity should be differentiated from cranial synostosis, which is the result of abnormal
fusion of one or more of the sutures between the skull bones. Ipsilateral frontal bossing and ear
advancement are not seen, resulting in a trapezoid-shaped head.

54
Most infants with positional skull deformity improve within 23 months with the institution of positional
changes and tummy time. If the condition does not significantly improve after this amount of time, referral
to a pediatric neurosurgeon with expertise in craniofacial malformations would be appropriate.

Ref: Laughlin J, Luerssen TG, Dias MS; Committee on Practice and Ambulatory Medicine, Section on Neurological Surgery:
Prevention and management of positional skull deformities in infants. Pediatrics 2011;128(6):1236-1241.

Item 164

ANSWER: C

Somatoform disorders are often encountered in family medicine. Studies have documented that 5% of
patients meet the criteria for somatization disorder, while another 4% have borderline somatization
disorder. Most of these patients are female and have a low socioeconomic status. They have a high
utilization of medical services, usually reflected by a thick medical chart, and are often single parents.
Physicians tend to be less satisfied with the care rendered to these patients compared to those without the
disorder. Patients with multiple unexplained physical complaints have been described as functionally
disabled, spending an average of one week per month in bed. Many of these patients seek and ultimately
undergo surgical procedures, and it is not uncommon for them to have multiple procedures, especially
involving the pelvic area. Often there are associated psychiatric symptoms such as anxiety, depression,
suicide threats, alcohol or drug abuse, interpersonal or occupational difficulties, and antisocial behavior.
A history of a dysfunctional family unit in which one or both parents abused alcohol or drugs or were
somatically preoccupied is also quite common. These individuals often enter relationships with alcohol
abusers.

Somatization disorder should be managed by one primary physician so that an established relationship and
regular visits can help curtail the dramatic symptoms that otherwise may lead to hospitalization. The family
physician is in a position to monitor family dynamics and provide direction on such issues as alcoholism
and child abuse. Each office visit should include a physical examination, and the temptation to tell the
patient that the problem is not physical should be avoided. Knowing the patient well helps to avoid
unnecessary hospitalizations, diagnostic procedures, surgery, and laboratory tests. These measures should
be carried out only if clearly indicated. Psychotropic medications should be avoided except when clearly
indicated, as medications reinforce the sick role, may be abused, and may be used for suicide gestures.
Following these recommendations significantly decreases the cost of care for the patient.

Ref: Sadock BJ, Sadock VA, Ruiz P (eds): Kaplan & Sadocks Comprehensive Textbook of Psychiatry, ed 9. Lippincott
Williams & Wilkins, 2009, pp 1927-1940. 2) Goldman L, Schafer AI (eds): Goldmans Cecil Medicine, ed 24. Elsevier
Saunders, 2011, p 2244.

Item 165

ANSWER: E

Sarcoidosis is a disease of unknown cause characterized by the presence of noncaseating epithelioid


granulomas; it involves many different organ systems. The lungs are commonly involved; bilateral hilar
lymphadenopathy is often present and pulmonary infiltrates and fibrosis somewhat typical. Sarcoidosis may
also affect the skin, central nervous system, eyes, liver, heart, salivary glands, kidneys, muscles, or bones.
When the disease is limited to asymptomatic hilar adenopathy, it is termed stage I and no treatment has
been shown to be beneficial. The most appropriate management of stage I patients is routine follow-up.

Ref: Wu JJ, Schiff KR: Sarcoidosis. Am Fam Physician 2004;70(2):312-322. 2) Longo DL, Fauci AS, Kasper DL, et al (eds):
Harrisons Principles of Internal Medicine, ed 18. McGraw-Hill, 2012, pp 2805-2813.

55
Item 166

ANSWER: E

This patient has cor pulmonale. Patients should be assessed for chronic oxygen therapy, which has been
shown to reduce hospitalization rates and mortality (SOR A). O2 saturation is <88% in most cases.
Oxygen therapy may be justified with a slightly higher O2 saturation if cor pulmonale is well documented
in a patient with COPD. Cautious diuretic therapy may be useful for symptomatic edema.

Digoxin is not thought to be beneficial in the absence of atrial fibrillation and is more likely to cause an
arrhythmia in a hypoxic patient. !-Blockers, calcium channel blockers, and ACE inhibitors are not
recommended for cor pulmonale. Calcium channel blockers and vasodilators may have some benefit in
primary pulmonary hypertension, but they have not proven beneficial in COPD-related cor pulmonale.

Ref: National Clinical Guideline Centre: Chronic Obstructive Pulmonary Disease: Management of Chronic Obstructive
Pulmonary Disease in Adults in Primary and Secondary Care. National Institute for Health and Clinical Excellence (NICE),
2010.

Item 167

ANSWER: D

Common variable immunodeficiency is the most commonly diagnosed disorder among the primary
immunodeficiencies. It is a disorder of humoral immunity associated with reduced serum levels of IgG,
IgM, and IgA, and frequently presents as late as the third or fourth decade of life. The disorder is
associated with recurrent sinus infections, otitis media, bronchiectasis, and chronic gastrointestinal
problems. Recognition of the disorder is important, as infections may be reduced when patients are treated
with intravenous immune globulin.

Abnormalities in the other test results are compatible with less common primary immunodeficiencies.
Lymphopenia suggests a disorder of cellular immunity such as severe combined immunodeficiency,
thrombocytopenia suggests the Wiskott-Aldrich syndrome, and an abnormal nitroblue tetrazolium test
suggests a phagocytic disorder.

Ref: Cooper MA, Pommering TL, Kornyi K: Primary immunodeficiencies. Am Fam Physician 2003;68(10):2001-2009. 2)
Goldman L, Schafer AI (eds): Goldmans Cecil Medicine, ed 24. Elsevier Saunders, 2011, pp 1615-1622. 3) Reust CE:
Evaluation of primary immunodeficiency disease in children. Am Fam Physician 2013;87(11):773-778.

Item 168

ANSWER: D

The U.S. Preventive Services Task Force recommends vitamin D supplementation to prevent falls in
community-dwelling adults 65 and older who are at increased risk for falls (grade B recommendation).
Some studies suggest that low vitamin D levels are associated with an increased risk of cardiovascular
disease, multiple sclerosis, colon cancer, dementia, and even diabetes mellitus, but these studies are
epidemiologic and thus are not based on high-quality evidence (SOR C).

Ref: Bordelon P, Ghetu MV, Langan RC: Recognition and management of vitamin D deficiency. Am Fam Physician
2009;80(8):841-854. 2) Kulie T, Groff A, Redmer J, et al: Vitamin D: An evidence-based review. J Am Board Fam Med
2009;22(6):698-706. 3) Ott SM: ACP Journal Club. Review: Vitamin D with calcium reduces fractures in adults. Ann
Intern Med 2012;156(12):JC6-JC7. 4) US Preventive Services Task Force: Prevention of Falls in Community-Dwelling
Older Adults: US Preventive Services Task Force Recommendation Statement. AHRQ pub no 11-05150-EF-2, 2012.

56
Item 169

ANSWER: B

For the average adult over the age of 60, the normal time required for the Timed Up and Go test is 10
seconds. A time longer than 10 seconds may indicate weakness, a balance or gait problem, and/or an
increased fall risk.

Ref: Croswell J, Young-Rok S: Prevention of falls in community-dwelling older adults. Am Fam Physician
2012;86(12):1135-1136.

Item 170

ANSWER: A

The current management of methanol intoxication, depending on its severity, includes ethanol
administration to inhibit the metabolism of methanol, hemodialysis to remove alcohol and its toxins, and
vigorous management of metabolic acidosis with bicarbonate therapy. Ethanol is a competitive inhibitor
of toxin metabolism and slows the formation of toxic metabolites, formaldehyde, and formic acid from
methanol, permitting these products to be disposed of by ordinary metabolic or excretory pathways. It has
a similar effect in ethylene glycol poisoning, slowing the formation of glycoaldehyde and glycolic,
glyoxylic, and oxalic acids.

Ref: Marx JA (ed): Rosens Emergency Medicine: Concepts and Clinical Practice, ed 7. Mosby Elsevier, 2010, pp 2001-2007.

Item 171

ANSWER: C

Treatment for a unilateral undescended testis should be started at 612 months of age to avoid testicular
damage. It was once thought that delaying descent lowered the incidence of testicular cancer, but it is now
believed that orchiopexy allows for early cancer detection. HCG treatment may promote descent into the
distal canal, but the testicle often ascends again. Ultrasonography will not show an undescended testis in
many cases and is therefore not recommended. Hormonal treatments have been used in Europe but
randomized, controlled trials have not shown them to be effective.

Ref: Docimo SG, Silver RI, Cromie W: The undescended testicle: Diagnosis and management. Am Fam Physician
2000;62(9):2037-2044. 2) Kliegman RM, Stanton BF, Geme JW III, et al (eds): Nelson Textbook of Pediatrics, ed 19.
Elsevier Saunders, 2011, pp 1859-1860.

Item 172

ANSWER: C

Head trauma is a known cause of the syndrome of inappropriate secretion of antidiuretic hormone
(SIADH). This patients course has been very acute, with hyponatremia developing within 48 hours. Such
a precipitous drop in serum sodium may lead to cerebral and pulmonary edema. If left untreated the patient
can have seizures, become obtunded, and die from brain herniation. These dangers require immediate
treatment with hypertonic saline to correct the falling levels of sodium. This must be done cautiously so
as to not overcorrect the sodium level too quickly, which could lead to osmotic demyelination syndrome.
An increase in serum sodium levels of about 6 mEq/L should be enough to reduce symptoms and prevent
progressive cerebral edema.

57
Ref: Longo DL, Fauci AS, Kasper DL, et al (eds): Harrisons Principles of Internal Medicine, ed 18. McGraw-Hill, 2012, pp
341-359.

Item 173

ANSWER: D

Parvovirus B19 is associated with erythema infectiosum, or fifth disease. It is also associated with
nonspecific fever, arthropathy, chronic anemia, and transient aplastic crisis.

Ref: Sabella C, Goldfarb J: Parvovirus B19 infections. Am Fam Physician 1999;60(5):1455. 2) Longo DL, Fauci AS, Kasper
DL, et al (eds): Harrisons Principles of Internal Medicine, ed 18. McGraw-Hill, 2012, pp 1478-1480.

Item 174

ANSWER: C

Natural family planning (NFP) is a potentially effective method for contraception and for determining the
time of ovulation for purposes of conception. While the contraceptive effectiveness of the different NFP
methods varies significantly, the success rates for typical use are as high as 92%98% (SOR B).
Monitoring the presence and consistency of cervical mucus production allows for the determination of both
the beginning and end of a womans most fertile period. Some NFP methods use cervical mucus secretion
as the sole basis for determining fertility. The symptothermal method also incorporates calendar
calculations, basal body temperature measurement, and ovulation-related symptoms as a complement to
the cervical mucus component. The Marquette Model incorporates cervical mucus and basal body
temperature charting with electronic monitoring of urine estrogen and LH metabolites to provide additional
information to determine when ovulation has occurred.

Ref: Smoley BA, Robinson CM: Natural family planning. Am Fam Physician 2012;86(10):924-928.

Item 175

ANSWER: B

Stillbirth is defined as fetal death occurring at or after 20 weeks gestation, and affects approximately 1 in
160 pregnancies in the United States. A large study of stillbirths from 2006 to 2008 tried to establish a
cause in 663 cases, and a probable or possible cause was identified in approximately 75% of these. While
there were some significant ethnic differences, placental abnormalities and obstetric complications were
the largest category of causes in white women, and this was even more true after 32 weeks gestation. Other
important causes included infection and fetal defects. More than one cause was found in one-third of cases.

Ref: Stillbirth Collaborative Research Network Writing Group: Causes of death among stillbirths. JAMA
2011;306(22):2459-2468.

Item 176

ANSWER: E

Primidone and propranolol are the first-line drugs for essential tremor. Alprazolam is considered to be
possibly effective. The tremor described is not due to Parkinsons disease and would not respond to
carbidopa/levodopa. Levetiracetam is not effective. There is not enough evidence to recommend for or
against the use of olanzapine.

58
Ref: Zesiewicz TA, Elble RJ, Louis ED, et al: Evidence-based guideline update: Treatment of essential tremor: Report of the
Quality Standards Subcommittee of the American Academy of Neurology. Neurology 2011;77(19):1752-1755.

Item 177

ANSWER: E

This patient has pneumonia, sepsis, and suspected coinfection with influenza. Although the rapid
antigen-based nasal swab was negative, false-negative rates may be as high as 70% and this test should not
be relied upon to rule out influenza. Treatment should include both antiviral and antibacterial agents that
include coverage against methicillin-resistant Staphylococcus aureus (MRSA), the most common bacterial
pathogen isolated from critically ill patients with coinfection. Oseltamivir, ceftriaxone, azithromycin, and
vancomycin should be initiated empirically for the pneumonia and sepsis. The criteria for sepsis are
satisfied by a temperature >38.3C, a WBC count >12,000/mm3, a respiratory rate >20/min, and a
source of probable infection.

Ref: Chertow DS, Memoli MJ: Bacterial coinfection in influenza: A grand rounds review. JAMA 2013;309(3):275-282.

Item 178

ANSWER: D

Emollients are a mainstay of chronic therapy for atopic dermatitis (SOR C), but topical corticosteroids are
the first-line treatment for flare-ups (SOR A). Calcineurin inhibitors such as pimecrolimus are a
second-line treatment for moderate to severe atopic dermatitis (SOR A). Antibiotics are not useful in
reducing flare-ups of atopic dermatitis unless there is clear evidence of a secondary infection (SOR A).
Neither topical nor oral antihistamines are recommended for routine treatment of atopic dermatitis because
they are not effective in treating the associated pruritus.

Ref: Berke R, Singh A, Guralnick M: Atopic dermatitis: An overview. Am Fam Physician 2012;86(1):35-42.

Item 179

ANSWER: A

The most cost-effective intervention for type 2 diabetes mellitus is lifestyle modification. The difficulty
with this intervention, however, lies in keeping the patient motivated. Metformin is also cost-effective.

Bariatric surgery shows immediate benefits in lowering glucose levels in patients who have undergone this
treatment. Unfortunately, studies on its long-term effects are lacking. Sitagliptin, pioglitazone, and insulin
are effective secondary agents but are expensive and not as cost-effective as metformin or dietary changes
and exercise.

Ref: Ismail-Beigi F: Glycemic management of type 2 diabetes mellitus. N Engl J Med 2012;366(14):1319-1327.

59
Item 180

ANSWER: C

American Heart Association guidelines recommend treating hypertension in patients with stable heart
failure with ACE inhibitors and/or "-blockers. Other agents, such as thiazide diuretics or calcium channel
blockers, can be added if needed to achieve blood pressure goals (SOR B). "-Blockers with intrinsic
sympathomimetic activity should be avoided, as they increase myocardial oxygen demand.

While thiazide diuretics are often a first choice for uncomplicated hypertension, this is not the case for
patients with coronary artery disease. Long-acting calcium channel blockers may be used in patients who
do not tolerate "-blockers, but short-acting calcium channel blockers should be avoided because they
increase mortality. ACE inhibitors are recommended as antihypertensive agents in patients already on
"-blocker therapy (especially following myocardial infarction), in diabetics, and in patients with left
ventricular dysfunction. Although angiotensin receptor blockers have indications similar to those of ACE
inhibitors, the American Heart Association recommends using them only in patients who do not tolerate
ACE inhibitors. Long-acting nitrates are used for their anti-anginal properties and have no role in the
management of hypertension.

Ref: Pflieger M, Winslow BT, Mills K, Dauber IM: Medical management of stable coronary artery disease. Am Fam Physician
2011;83(7):819-826. 2) Fihn SD, Gardin JM, Abrams J, et al: 2012 ACCF/AHA/ACP/AATS/PCNA/SCAI/STS guideline
for the diagnosis and management of patients with stable ischemic heart disease: A report of the American College of
Cardiology Foundation/American Heart Association task force on practice guidelines, and the American College of
Physicians, American Association for Thoracic Surgery, Preventive Cardiovascular Nurses Association, Society for
Cardiovascular Angiography and Interventions, and Society of Thoracic Surgeons. Circulation 2012;126(25):e354-e471.

Item 181

ANSWER: B

This patients symptoms are compatible with Menieres disease, which is characterized by multiple
episodes of vertigo lasting for 20120 minutes, accompanied by a fluctuating hearing loss, tinnitus, and
a sense of aural fullness. Audiograms will reveal a low-frequency hearing loss with an upsloping curve,
which can become flattened over the years. Most patients develop unilateral symptoms, and many patients
will develop bilateral disease many years after the onset of the unilateral symptoms. Multiple studies have
reported the rate of bilateral Meniere's disease to be as high as 50% many years after the initial diagnosis.

Motion sickness is a common cause of nausea, but the nausea usually does not come on suddenly and is
not as pronounced as with Menieres disease. A vestibular migraine can present like a sudden Menieres
disease attack but in this patient the audiograms, tinnitus, and aural fullness suggest Menieres disease.
Benign positional vertigo is very common, and hearing loss could be an incidental finding. However, the
most common form of age-related hearing loss is seen at the higher frequencies. Positional vertigo like this
patient has is common between attacks of Menieres disease.

There is often a family history of Menieres disease, and there is frequently an association with allergies.
The condition can also get worse with caffeine use. Even though the diagnosis is clinical, MRI and blood
tests are recommended to rule out other conditions that may be putting pressure on the endolymphatic
system and thus causing the symptoms.

Ref: Sajjadi H, Paparella MM: Menieres disease. Lancet 2008;372(9636):406-414.

60
Item 182

ANSWER: C

This patient has bipolar II disorder. She has a history of hypomanic episodes as well as major depression,
with no history of a manic or mixed episode. Among the pharmacologic options listed, only divalproex
and lithium are indicated for treating bipolar depression or acute mania, and for maintenance. They should
be given as single agents, however, not in combination with other drugs. No evidence supports
combination therapy or the addition of an antidepressant in the acute phase of depression.

In a study of patients with bipolar II disorder, initially adding paroxetine or bupropion to the mood
stabilizer was no more effective than using lithium or valproate. An SSRI or bupropion can be added if
a therapeutic dosage of a mood stabilizer does not resolve symptoms and the patient is not in a mixed state.
Tricyclic antidepressants and antidepressants with dual properties, such as venlafaxine, should be avoided
because they may induce mania. Aripiprazole is indicated for acute mania but not for bipolar depression.

Ref: Price AL, Marzani-Nissen GR: Bipolar disorders: A review. Am Fam Physician 2012;85(5):483-493.

Item 183

ANSWER: E

The Future Revascularization Evaluation in Patients with Diabetes Mellitus: Optimal Management of
Multivessel Disease (FREEDOM) trial funded by the National Heart, Lung, and Blood Institute concluded
that in patients with diabetes mellitus and advanced coronary artery disease, coronary artery bypass graft
surgery was superior to percutaneous coronary intervention (PCI) in that it significantly reduced rates of
death and myocardial infarction, although stroke rates were higher in the 30-day perioperative period. The
FREEDOM trial suggested that these outcomes are similar whether PCI is performed without stents, with
bare-metal stents, or with drug-eluting stents. These results were consistent with reports from other smaller
or retrospective studies of revascularization in patients with diabetes mellitus.

Ref: Farkouh ME, Domanski M, Sleeper LA, et al: Strategies for multivessel revascularization in patients with diabetes. N Engl
J Med 2012;367(25):2375-2384. 2) Hlatky MA: Compelling evidence for coronary-bypass surgery in patients with diabetes.
N Engl J Med 2012;367(25):2437-2438.

Item 184

ANSWER: D

In young children with bilious emesis, anorexia, and lack of fever, the most likely diagnosis is intestinal
malrotation with volvulus. Abdominal ultrasonography is less sensitive and specific for malrotation than
an upper gastrointestinal series, so an upper GI series should be ordered initially if volvulus is suspected.
If appendicitis were suspected, ultrasonography would be preferred. CT is not a good choice because of
the amount of radiation it delivers, especially given efforts to decrease the use of CT in children unless
absolutely necessary. This patients presentation is not typical for testicular torsion, therefore scrotal
ultrasonography should not be the initial test of choice.

Ref: Saito JM: Beyond appendicitis: Evaluation and surgical treatment of pediatric acute abdominal pain. Curr Opin Pediatr
2012;24(3):357-364.

61
Item 185

ANSWER: B

Ingestion of grapefruit juice can increase absorption and serum levels of statins, leading to an increased
risk of muscle injury. The mechanism for this is believed to be the cytochrome p-450 pathway. Starfruit
juice and pomegranate juice can have a similar effect. These juices contain an irreversible inhibitor of
intestinal CYP3A4, and increase the bioavailability of atorvastatin, lovastatin, and simvastatin.
Rosuvastatin and fluvastatin utilize the CYP2C9 system for metabolism, so the effect on these drugs is
minimal.

Grapefruit juice reduces CYP3A4 activity by 50% within 4 hours of ingestion, and activity is reduced by
30% for as long as 24 hours after ingestion. Several studies document that consuming 600 mL of
double-strength juice for 3 days produces a more than tenfold increase in the area under the curve for
simvastatin and lovastatin, but only a 250% increase in atorvastatin.

Ref: Venero CV, Thompson PD: Managing statin myopathy. Endocrinol Metabol Clin North Am 2009;38(1):121-136. 2)
Bonow RO, Mann DL, Zipes DP, Libby P (eds): Braunwalds Heart Disease: A Textbook of Cardiovascular Medicine,
ed 9. Elsevier Saunders, 2011, p 987.

Item 186

ANSWER: A

HMG-CoA reductase inhibitors, or statins, play an important role in the management of patients with
cardiovascular disease and have an excellent safety and tolerability record. The incidence of significant
liver injury from statin drugs is about 1%, and nonalcoholic fatty liver disease or stable hepatitis B or C
infection is not a contraindication to treatment with statins. Although many patients taking statins
experience elevation of hepatic transaminases, these elevations are generally mild and asymptomatic, and
often resolve spontaneously even with no changes in treatment. Transaminase elevations up to three times
the upper limit of normal are not a contraindication to continued use of the drug at the same dosage.

Ref: Gillett RC Jr, Norrell A: Considerations for safe use of statins: Liver enzyme abnormalities and muscle toxicity. Am Fam
Physician 2011;83(6):711-716.

Item 187

ANSWER: D

A 95% confidence interval is an estimate of certainty. It means there is 95% certainty that the true value
lies within the given interval range. When a confidence interval crosses 1.00, the validity of the resulting
statistical estimate is questionable. Sensitivity is the percentage of patients with a disease who have a
positive test for the disease. Specificity is the percentage of patients without a disease who have a negative
test for the disease. Relative risk reduction is the percentage difference in risk between the treatment and
control groups. The number needed to treat is the number of patients who need to receive an intervention
instead of the alternative in order for one additional patient to benefit.

Ref: Glossary of evidence-based medicine and statistical terms. Am Fam Physician 2012;86(3):231.

62
Item 188

ANSWER: C

The Fourth Report on the Diagnosis, Evaluation, and Treatment of High Blood Pressure in Children and
Adolescents defines hypertension in children as a systolic or diastolic blood pressure above the 95th
percentile for the patients sex, age, and height on several different readings. Although it is appropriate
to have this finding confirmed in the outpatient setting, 130 mm Hg is still at the 99th percentile for systolic
blood pressures in this patient. Hypertension in a patient this young should prompt a search for secondary
causes, which are more common in young hypertensive patients than in adults with hypertension. The
recommended workup includes blood and urine testing, as well as renal ultrasonography. An evaluation
for end-organ damage is also recommended, including retinal evaluation and echocardiography.

Ref: Kliegman RM, Stanton BF, Geme JW III, et al (eds): Nelson Textbook of Pediatrics, ed 19. Elsevier Saunders, 2011, pp
1639-1647. 2) Riley M, Bluhm B: High blood pressure in children and adolescents. Am Fam Physician
2012;85(7):693-700.

Item 189

ANSWER: B

For a healthy nonsmoker with no chronic disease who is not in a high-risk group, pneumococcal vaccine
is recommended once at age 65, or as soon afterward as possible. Persons that should be immunized before
age 65 include patients with chronic lung disease, cardiovascular disease, diabetes mellitus, chronic liver
disease, cerebrospinal fluid leaks, cochlear implants, immunocompromising conditions, or asplenia, and
residents of nursing homes and long-term care facilities. The Advisory Committee on Immunization
Practices of the CDC updated the recommendations for pneumococcal vaccination in 2011 to include
immunization for persons age 5064 in the following categories: Alaska Natives, Native Americans living
in areas of increased risk, persons with asthma, and smokers.

Ref: Centers for Disease Control and Prevention (CDC); Advisory Committee on Immunization Practices: Updated
recommendations for prevention of invasive pneumococcal disease among adults using the 23-valent pneumococcal
polysaccharide vaccine (PPSV23). MMWR Morb Mortal Wkly Rep 2010;59(34):1102-1105. 2) Vaughn JA, Miller RA:
Update on immunizations in adults. Am Fam Physician 2011;84(9):1015-1017.

Item 190

ANSWER: E

NSAIDs, pyridoxine, and diuretics have been shown to be no more effective than placebo in the treatment
of patients with carpal tunnel syndrome. Splinting, physical therapy, and corticosteroid injections have all
been shown to result in short-term improvement. Patients with persistent symptoms achieve the best
long-term relief with surgery.

Ref: Daniels JM, Muller MH: Hand and Wrist Injuries. FP Essentials monograph series, no 400, 2012, pp 34-38.

63
Item 191

ANSWER: A

A number of pretest probability scoring systems are available for assessing venous thromboembolism
(VTE), which includes deep vein thrombosis (DVT) and pulmonary embolism. Although the Wells clinical
prediction rule is widely used, other tools such as the Hamilton score and the AMUSE (Amsterdam
Maastricht Utrecht Study on thromboEmbolism) score are also available. The Wells rule divides patients
suspected of having a DVT into low, intermediate, and high-risk categories, with a 5%, 17%, and 53%
prevalence of DVT, respectively. This patient has a Wells score of 0 (+1 for calf circumference increase
>3 cm, +1 for pitting edema, 2 for a likely alternative diagnosis of gastrocnemius strain) and is
therefore at low risk. A negative D-dimer assay has a high negative predictive value for DVT, so the
diagnosis can be ruled out in a patient who has a low pretest probability and a negative D-dimer result. A
negative D-dimer assay does not rule out DVT in a patient with a moderate to high pretest probability
(SOR C).

Ref: Wilbur J, Shian B: Diagnosis of deep venous thrombosis and pulmonary embolism. Am Fam Physician
2012;86(10):913-919.

Item 192

ANSWER: E

This patient presents with a classic description of supraventricular tachycardia (SVT). The initial
management of SVT centers around stopping the aberrant rhythm. In the hemodynamically stable patient
initial measures should include vagal maneuvers (SOR C), intravenous adenosine or verapamil (SOR B),
intravenous diltiazem or "-blockade, intravenous antiarrhythmics, or cardioversion in refractory cases.
While digoxin is occasionally useful in atrial fibrillation with a rapid ventricular rate, it is not
recommended for SVT. Radiofrequency ablation is fast becoming the first-line therapy for all patients with
recurrent SVT, not just those refractory to suppressive drug therapies. Observational studies have shown
that this therapy results in improved quality of life and lower cost as compared to drug therapy (SOR B).

Ref: King DE, Dickerson LM, Sack JL: Acute management of atrial fibrillation: Part 1. Rate and rhythm control. Am Fam
Physician 2002;66(2):249-256. 2) Colucci RA, Silver MJ, Shubrook J: Common types of supraventricular tachycardia:
Diagnosis and management. Am Fam Physician 2010;82(8):942-952.

Item 193

ANSWER: C

Acute respiratory distress syndrome (ARDS) may be caused by pulmonary sepsis or sepsis from another
source, or it may be due to acute pulmonary injury, including inhalation of smoke or other toxins.
Inflammatory mediators are released in response to the pulmonary infection or injury. The syndrome has
an acute onset and is manifested by rapidly developing profound hypoxia with bilateral pulmonary
infiltrates. The mortality rate in patients with ARDS may be as high as 55%.

Early recognition and prompt treatment with intubation and mechanical ventilation is necessary to improve
chances for survival. Patients with ARDS should be started at lower tidal volumes (6 mL/kg) instead of
the traditional volumes (1015 mL/kg) (SOR A). These patients also often require higher positive
end-expiratory pressure settings (SOR B).

64
Fluid management should be conservative to allow for optimal cardiorespiratory and renal function and
to avoid fluid overload. However, the routine use of central venous or pulmonary artery pressure catheters
is not recommended due to the potential complications associated with their use (SOR A). While surfactant
is commonly used in children with ARDS, it does not improve mortality in adults (SOR A).

Ref: Saguil A, Fargo M: Acute respiratory distress syndrome: Diagnosis and management. Am Fam Physician
2012;85(4):352-358.

Item 194

ANSWER: A

First-line treatment for diabetic peripheral neuropathy, according to the American Diabetes Association,
is tricyclic antidepressants. Anticonvulsants are second line and opioids are third line. Many medications
have been found to be effective, including the tricyclics, duloxetine, pregabalin, oxycodone, and tramadol
(SOR A).

Among the tricyclics, amitriptyline, imipramine, and nortriptyline have been found to be the most effective
(SOR A). For an uninsured patient, the tricyclics are also the most affordable.

Ref: Page N, Deluca J, Crowell K: What medications are best for diabetic neuropathic pain? J Fam Pract 2012;61(11):691-693.

Item 195

ANSWER: D

The child described has facial features characteristic of fetal alcohol syndrome. Fetal alcohol spectrum
disorders (FASD) are caused by the effects of maternal alcohol consumption during pregnancy. Fetal
alcohol syndrome is the most clinically recognized form of FASD and is characterized by a pattern of
minor facial anomalies, including a thin upper lip, a smooth philtrum, and a flat nasal bridge; other
physical anomalies, such as clinodactyly; prenatal and postnatal growth retardation; and functional or
structural central nervous system abnormalities.

Children with Down syndrome have hypotonia, a flat face, upward and slanted palpebral fissures and
epicanthic folds, and speckled irises (Brushfield spots); varying degrees of mental and growth retardation;
dysplasia of the pelvis; cardiac malformations; a simian crease; short, broad hands; hypoplasia of the
middle phalanx of the 5th finger; and a high, arched palate.

Marfan syndrome is characterized by pectus carinatum or pectus excavatum, an arm span to height ratio
>1.05, a positive wrist and thumb sign, limited elbow extension, pes planus, and aortic ascendens
dilatation with or without aortic regurgitation.

The bilateral renal agenesis seen with Potter syndrome leads to death shortly after birth. Other anomalies
include widely separated eyes with epicanthic folds, low-set ears, a broad and flat nose, a receding chin,
and limb anomalies.

Finally, Prader-Willi syndrome is characterized by severe hypotonia at birth, obesity, short stature
(responsive to growth hormone), small hands and feet, hypogonadism, and mental retardation.

Ref: Wattendorf DJ, Muenke M: Fetal alcohol spectrum disorders. Am Fam Physician 2005;72(2):279-282, 285. 2) Kliegman
RM, Stanton BF, Geme JW III, et al (eds): Nelson Textbook of Pediatrics, ed 19. Elsevier Saunders, 2011, pp 400-403,
412-413, 625-626, 1827, 2240-2246.

65
Item 196

ANSWER: A

While there is substantial overlap in the signs, symptoms, and physical findings for the various etiologies
of chest pain, a good history and physical examination can help determine which patients require
immediate further evaluation for a potentially serious cause. The chest pain associated with pericarditis is
typically pleuritic, and is worse with inspiration or in positions that put traction on the pleuropericardial
tissues, such as lying supine. Patients with acute pericarditis typically get relief or improvement when there
is less tension on the pericardium, such as when sitting and leaning forward. This position brings the heart
closer to the anterior chest wall, which incidentally is the best position for hearing the pericardial friction
rub associated with acute pericarditis.

Radiation of chest pain to both arms should raise concerns about myocardial ischemia or infarction.
Radiation to the right scapula is sometimes seen with cholelithiasis. Worsening pain with the Valsalva
maneuver is nonspecific and is of no particular diagnostic value.

Ref: McConaghy JR, Oza RS: Outpatient diagnosis of acute chest pain. Am Fam Physician 2013;87(3):177-182.

Item 197

ANSWER: A

Children with diabetes mellitus are at increased risk for retinopathy, nephropathy, and hypertension. They
are also more likely to have immune-mediated disorders such as celiac disease and hypothyroidism. For
all children and adolescents with type 1 diabetes mellitus, the American Diabetes Association recommends
screening for hypothyroidism, nephropathy, hypertension, celiac disease, and retinopathy. Screening for
dyslipidemia should be considered if there is a family history of hypercholesterolemia or cardiac events
before age 55.

Ref: American Diabetes Association: Standards of medical care in diabetes2013. Diabetes Care 2013;36(Suppl 1):S40-S43.

Item 198

ANSWER: B

The most likely diagnosis is septic arthritis of the hip. Ultrasonography is highly sensitive for the effusion
seen in septic arthritis, which can be aspirated to confirm the diagnosis (SOR A). It is important to
diagnose this problem as soon as possible. Clinical features of septic arthritis include an oral temperature
>38.5C (101.3F), refusal to bear weight on the affected leg, an erythrocyte sedimentation rate >40
mm/hr, a peripheral WBC count >12,000/mm3, and a C-reactive protein level >20 mg/L. If
ultrasonography is negative, a bone scan should be done. CT of the hip is indicated to visualize cortical
bone. MRI is especially valuable for osteomyelitis.

Ref: Sawyer J, Kapoor M: The limping child: A systematic approach to diagnosis. Am Fam Physician 2009;79(3):215-224.

66
Item 199

ANSWER: A

Asymptomatic gallstones are not usually an indication for prophylactic cholecystectomy, as most patients
remain asymptomatic throughout their lives, and only 1%4% develop symptoms or complications from
gallstones each year. Only 10% of patients found to have asymptomatic gallstones develop symptoms
within the first 5 years after diagnosis, and only 20% within 20 years.

In the past, cholecystectomy was recommended for diabetic patients with asymptomatic gallstones, based
on the assumption that autonomic neuropathy masked the pain and signs associated with acute cholecystitis,
and that patients would therefore develop advanced disease and more complications. More recent evidence
has shown that these patients have a lower risk of major complications than previously thought.

Prophylactic cholecystectomy is not recommended in renal transplant patients with asymptomatic


gallstones. One study found that 87% of these patients remained asymptomatic after 4 years, with only 7%
developing acute cholecystitis and requiring subsequent uncomplicated laparoscopic cholecystectomy.
Other studies have shown that the presence of gallstone disease does not negatively affect graft survival.

Patients with hemoglobinopathies are at a significantly increased risk for developing pigmented stones.
Gallstones have been reported in up to 70% of sickle cell patients, up to 85% of hereditary spherocytosis
patients, and up to 24% of thalassemia patients. In sickle cell patients, complications from asymptomatic
gallstones have been reported to be as high as 50% within 35 years of diagnosis. This has been attributed
largely to the diagnostic challenge associated with symptomatic cholelithiasis versus abdominal sickling
crisis. In the past these patients were managed expectantly because of the significant morbidity and
mortality associated with open operations. The operative risk for these patients (especially sickle cell
patients) has been lowered by laparoscopic cholecystectomy, along with improved understanding of
preoperative hydration and transfusion, improved anesthetic technique, and better postoperative care.
Prophylactic laparoscopic cholecystectomy in these patients prevents future diagnostic confusion, as well
as the mortality and morbidity risk associated with emergency surgery. Furthermore, cholecystectomy can
and should be performed at the time of splenectomy, whether open or laparoscopic.

Studies have shown no significant differences in progression to symptoms from silent gallstones in cirrhotic
patients compared with noncirrhotic patients. Expectant management is therefore recommended in patients
with cirrhosis.

Ref: Cameron JL, Cameron AM (eds): Current Surgical Therapy, ed 10. Elsevier Saunders, 2011, pp 335-336.

Item 200

ANSWER: A

The criteria for chronic fatigue syndrome include fatigue for 6 months and a minimum of four of the
following physical symptoms: impaired memory, postexertional malaise, muscle pain, polyarthralgia,
tender lymph nodes, sore throat, new headaches, and unrefreshing sleep. Both cognitive-behavioral therapy
and graded exercise therapy have been shown to improve fatigue levels, anxiety, work/social adjustment,
and postexertional malaise (SOR A). Treatments that have not been shown to be effective include
methylphenidate, melatonin, and galantamine. Citalopram has not been shown to be effective in the
absence of a comorbid diagnosis of depression.

67
Ref: Yancey JR, Thomas SM: Chronic fatigue syndrome: Diagnosis and treatment. Am Fam Physician 2012;86(8):741-746.

Item 201

ANSWER: B

There are many drugs that can induce a syndrome resembling systemic lupus erythematosus, but the most
common offenders are antiarrhythmics such as procainamide. Hydralazine is also a common cause. There
is a genetic predisposition for this drug-induced lupus, determined by drug acetylation rates. Polyarthritis
and pleuropericarditis occur in half of patients, but CNS or renal involvement is rare. While all patients
with this condition have positive antinuclear antibody titers and most have antibodies to histones, antibodies
to double-stranded DNA and decreased complement levels are rare, which distinguishes drug-induced
lupus from idiopathic lupus.

The best initial management for drug-induced lupus is to withdraw the drug, and most patients will improve
in a few weeks. For those with severe symptoms, a short course of corticosteroids is indicated. Once the
offending drug is discontinued, symptoms seldom last beyond 6 months.

Ref: Klippel JH, Stone JH, Crofford LJ, et al (eds): Primer on the Rheumatic Diseases, ed 13. Springer, 2008, pp 316-317.
2) Longo DL, Fauci AS, Kasper DL, et al (eds): Harrisons Principles of Internal Medicine, ed 18. McGraw-Hill, 2012,
p 2735.

Item 202

ANSWER: E

If a metal foreign body is present on the cornea for more than 24 hours a rust ring will often be present
in the superficial layer of the cornea. This material is toxic to the cornea and should be removed as soon
as possible, but it is not an emergency. The proper removal of a rust ring requires the use of a slit lamp
and specialized ophthalmic equipment. Referral to an eye specialist within 2448 hours is the best
management in this case.

Ref: Tintinalli JE, Kelen GD, Stapczynski JS (eds): Emergency Medicine: A Comprehensive Study Guide, ed 7. McGraw-Hill,
2011, pp 1517-1549.

Item 203

ANSWER: B

Workers and children at child care centers should receive postexposure prophylaxis if one or more cases
of hepatitis A is found in a child or worker. Hepatitis A vaccine is preferred over immunoglobulin because
of its long-lasting effect, ease of administration, and efficacy. Children younger than 1 year of age should
receive immunoglobulin. Family members should receive prophylaxis only during an outbreak and if their
child is still in diapers.

Ref: Update: Prevention of hepatitis A after exposure to hepatitis A virus and in international travelers. Updated
recommendations of the Advisory Committee on Immunization Practices (ACIP). MMWR Morb Mortal Wkly Rep
2007;56(41):1080-1084. 2) Matheny SC, Kingery JE: Hepatitis A. Am Fam Physician 2012;86(11):1027-1034.

68
Item 204

ANSWER: A

In general, recommended preoperative testing is based on the patients medical history and risk factors,
the risk associated with the planned surgery, and the patients functional capacity. In the case of cataract
surgery, however, randomized, controlled trials have established a lack of benefit from preoperative testing
for patients in their normal state of health (SOR A).

Ref: Keay L, Lindsley K, Tielsch J, et al: Routine preoperative medical testing for cataract surgery. Cochrane Database Syst
Rev 2012;(3):CD007293. 2) Feely MA, Collins CS, Daniels PR, et al: Preoperative testing before noncardiac surgery:
Guidelines and recommendations. Am Fam Physician 2013;87(6):414-418.

Item 205

ANSWER: C

Cefazolin is the recommended prophylactic antibiotic for most patients undergoing orthopedic procedures
such as total joint replacement, unless the patient has a "-lactam allergy (SOR A).

Ref: Salkind AR, Rao KC: Antibiotic prophylaxis to prevent surgical site infections. Am Fam Physician 2011;83(5):585-590.
2) Bratzler DW, Dellinger EP, Olsen KM, et al: Clinical practice guidelines for antimicrobial prophylaxis in surgery. Am
J Health Syst Pharm 2013;70(3):195-283.

Item 206

ANSWER: D

A medial meniscus tear is the most likely diagnosis in a patient older than 40 who was bearing weight when
the injury occurred, was unable to continue the activity, and has a positive Thessaly test. This test is
performed by having the patient stand on one leg and flex the knee to 20, then internally and externally
rotate the knee. The presence of swelling immediately after the injury makes an internal derangement of
the knee more likely, so osteoarthritis is less probable. This patient is able to bear weight, so a fracture
is also not likely. Either a collateral ligament tear or an anterior cruciate ligament tear is possible, but these
are not as common in this situation.

Ref: Grover M: Evaluating acutely injured patients for internal derangement of the knee. Am Fam Physician
2012;85(3):247-252.

Item 207

ANSWER: D

Orthostatic hypotension is defined as a documented drop in blood pressure of at least 20 mm Hg systolic


or 10 mm Hg diastolic that occurs within 3 minutes of standing. When symptomatic it is often described
as lightheadedness or dizziness upon standing. Etiologies to consider include iatrogenic, neurologic,
cardiac, and environmental causes, plus many others alone or in combination. Since orthostatic
hypotension may result in syncope, leading to falls and substantial injury, identifying it and taking
corrective steps can produce a significant benefit.

Ref: Bradley JG, Davis KA: Orthostatic hypotension. Am Fam Physician 2003;68(12):2393-2398. 2) Gilhus NE, Barnes M,
Brainin M (eds): European Handbook of Neurological Management, ed 2. Blackwell Publishing Ltd, 2011, pp 469-475.

69
Item 208

ANSWER: A

Peaked T waves are most commonly associated with acute myocardial infarction and hyperkalemia (SOR
A). Other causes include intracranial bleeding, left ventricular hypertrophy, and left bundle branch block.
Hypocalcemia can cause peaked T waves on rare occasions, but hypercalcemia has no effect on T waves.
Metabolic alkalosis is not associated with a particular EKG pattern, but it is associated with hypokalemia
and can lead to the patterns resulting from the potassium deficit. Hypothermia causes an elevation at the
J point, called either a J wave or an Osborn wave.

Ref: Goldberger AL: Clinical Electrocardiography: A Simplified Approach, ed 7. Mosby Elsevier, 2006, pp 129-132. 2) Bonow
RO, Mann DL, Zipes DP, Libby P (eds): Braunwalds Heart Disease: A Textbook of Cardiovascular Medicine, ed 9.
Elsevier Saunders, 2011, pp 159-161.

Item 209

ANSWER: A

Spironolactone is commonly used to treat hirsutism in women with polycystic ovary syndrome. One of its
side effects is hyperkalemia. Using an oral contraceptive at the same time as spironolactone has a
synergistic effect for treating hirsutism due to the low androgenic effects of oral contraceptives.
Contraceptives containing drospirenone should be used with caution in patients taking spironolactone,
however, because they can also lead to hyperkalemia. The other contraceptive options listed do not
increase the risk for hyperkalemia.

Ref: Radosh L: Drug treatments for polycystic ovary syndrome. Am Fam Physician 2009;79(8):671-676.

Item 210

ANSWER: C

Erysipelas is caused primarily by group A Streptococcus, with a rare case caused by group C or G. Most
cases of erysipelas involve the face, but the lesions can occur anywhere on the body. Penicillin is an
effective treatment.

Ref: Longo DL, Fauci AS, Kasper DL, et al (eds): Harrisons Principles of Internal Medicine, ed 18. McGraw-Hill, 2012, p
1067.

Item 211

ANSWER: A

Some hypothyroid patients who are treated with appropriate dosages of levothyroxine and whose TSH
levels are in the appropriate range continue to have persistent symptoms such as fatigue, depressed mood,
and weight gain. If the TSH is in the appropriate range then no adjustment is necessary and annual serum
TSH testing is recommended. Patients who remain symptomatic on an appropriate dosage of levothyroxine,
as determined by a TSH <2.5 mIU/L, are not likely to benefit from combination
triiodothyronine/thyroxine therapy (SOR A). Desiccated thyroid hormone preparations are not
recommended by the American Association of Clinical Endocrinologists for the treatment of
hypothyroidism. A meta-analysis of 11 randomized, controlled trials of combination T3/T4 therapy versus
T4 monotherapy showed no improvements in pain, depression, or quality of life (SOR A).

70
Ref: Gaitonde DY, Rowley KD, Sweeney LB: Hypothyroidism: An update. Am Fam Physician 2012;86(3):244-251. 2) Garber
JR, Cobin RH, Gharib H, et al: Clinical practice guidelines for hypothyroidism in adults: Cosponsored by the American
Association of Clinical Endocrinologists and the American Thyroid Association. Endocr Pract 2012;18(6):988-1028.

Item 212

ANSWER: A

Treating basal cell carcinoma with Mohs micrographic surgery leads to the lowest recurrence rate. Because
of its cost and limited availability, however, this procedure should be limited to tumors with a higher risk
for recurrence. Risk factors include larger size, more invasive histologic subtypes (micronodular,
infiltrative, and morpheaform), and sites associated with a higher risk of recurrence.

High-risk locations include the mask areas of the face, which include the central face, eyelids, eyebrows,
periorbital area, nose, lips (cutaneous and vermilion), chin, mandible, preauricular and postauricular
skin/sulci, temple, and ear. Other high-risk sites include the genitalia, hands, and feet. Moderate-risk
locations include the cheeks, forehead, scalp, and neck. All other areas, including the trunk and
extremities, are low-risk areas.

Even with a low-risk location, a lesion that is !20 mm in size has a high risk of recurrence. With a
moderate-risk location a lesion !10 mm in size carries a higher risk of recurrence, and a lesion !6 mm in
size is considered high risk in a high-risk location.

Ref: Firnhaber JM. Diagnosis and treatment of basal cell and squamous cell carcinoma. Am Fam Physician 2012;86(2):161-168.

Item 213

ANSWER: D

In a study of 256 asymptomatic Swedish patients with mildly elevated liver transaminase levels, alcohol
was found to be the cause in 10% of cases. An accurate history is important for making the diagnosis. A
biopsy alone cannot differentiate alcoholic liver disease from nonalcoholic fatty liver disease. An
AST/ALT ratio >2 supports a diagnosis of alcoholic liver disease. Elevated #-glutamyl transpeptidase
(GGTP) is also associated with alcohol abuse, especially in a patient with an AST/ALT ratio >2.

Ref: Oh RC, Hustead TR: Causes and evaluation of mildly elevated liver transaminase levels. Am Fam Physician
2011;84(9):1003-1008.

Item 214

ANSWER: B

Tinea capitis is an infection of the scalp caused by a variety of superficial dermatophytes. The treatment
of choice for this infection is oral griseofulvin. It has the fewest drug interactions, a good safety record,
and anti-inflammatory properties. Terbinafine has equal effectiveness and requires a significantly shorter
duration of therapy, but it is only available in tablet form. Since tinea capitis most commonly occurs in
children, tablets would have to be cut and/or crushed prior to administration. Oral itraconazole,
fluconazole, and ketoconazole have significant side effects. Topical antifungals such as ketoconazole and
miconazole are ineffective against tinea capitis. Topical acyclovir is used in the treatment of herpesvirus
infections, and oral cephalosporins are used in the treatment of bacterial skin infections.

71
Ref: Habif TP: Clinical Dermatology: A Color Guide to Diagnosis and Therapy, ed 5. Mosby Elsevier, 2010, pp 509-510.

Item 215

ANSWER: D

While it would increase the risk of urinary infection, indwelling catheter placement is most likely to
provide immediate relief of this patients urinary retention. It will minimize or prevent further
contamination of his decubitus ulcer with urine. Prostatectomy may relieve the urethral obstruction, but
this patient is likely to remain incontinent due to his vascular dementia. Doxazosin or finasteride would
likely be inadequate in this situation. Tolterodine is not indicated for overflow incontinence.

Ref: Unwin BK, Porvaznik M, Spoelhof GD: Nursing home care: Part II. Clinical aspects. Am Fam Physician
2010;81(10):1229-1237. 2) Gould CV, Umscheid CA, Agarwal RK, et al: Guideline for prevention of catheter-associated
urinary tract infections 2009. Infect Control Hosp Epidemiol 2010;31(4):319-326. 3) DuBeau CE, Kuchel GA, Johnson
T 2nd, et al: Incontinence in the frail elderly: Report from the 4th International Consultation on Incontinence. Neurourol
Urodyn 2010;29(1):165-178.

Item 216

ANSWER: A

This patient has a low pretest probability of pulmonary embolism based on the Wells criteria. She would
be a good candidate for a high-sensitivity D-dimer test, with a negative test indicating a low probability
of venous thromboembolism. In patients with a low pretest probability of venous thromboembolism,
ultrasonography or helical CT would not be the recommended initial evaluation. Neither troponin I nor an
ANA level would be part of the recommended initial evaluation.

Ref: Wilbur J, Shian B: Diagnosis of deep venous thrombosis and pulmonary embolism. Am Fam Physician
2012;87(12):913-919.

Item 217

ANSWER: A

Intertrigo is skin inflammation caused by skin-on-skin friction. It is facilitated by moisture trapped in deep
skinfolds where air circulation is limited. When intertrigo does not respond to usual conservative measures,
including keeping the skin clean and dry, evaluation for infection is recommended. A Woods light
examination, KOH preparation, and exudate culture can assist in identifying causative organisms.

The moist, damaged skin associated with intertrigo is a fertile breeding ground for various
microorganisms, and secondary cutaneous infections are commonly observed in these areas. Candida is
the organism most commonly associated with intertrigo. In the interdigital spaces dermatophytes (e.g.,
Trichophyton rubrum, Trichophyton mentagrophytes, Epidermophyton floccosum) are more common.
Staphylococcus aureus may present alone or with group A "-hemolytic Streptococcus (GABHS).
Pseudomonas aeruginosa, Proteus mirabilis, or Proteus vulgaris also may occur alone or simultaneously.

Ref: Janniger CK, Schwartz RA, Szepietowski JC, Reich A: Intertrigo and common secondary skin infections. Am Fam
Physician 2005;72(5):833-838. 2) Habif TP: Clinical Dermatology: A Color Guide to Diagnosis and Therapy, ed 5. Mosby
Elsevier, 2010, p 534.

72
Item 218

ANSWER: C

Kawasaki disease, or mucocutaneous lymph node syndrome, is a common form of vasculitis most often
seen in children under 5 years of age. It is typically self-limited, with fever and acute inflammation lasting
12 days on average without therapy. However, if untreated, this illness can result in heart failure, coronary
artery aneurysm, myocardial infarction, arrhythmias, or occlusion of peripheral arteries. This diagnosis
requires that fever be present for 5 days or more with no other explanation. In addition, at least four of
the following symptoms must be present: (1) nonexudative conjunctivitis that spares the limbus; (2)
changes in the oral membranes such as diffuse erythema, injected or fissured lips, or strawberry tongue;
(3) erythema of the palms and soles, and/or edema of the hands or feet followed by periungual
desquamation; (4) cervical adenopathy in the anterior cervical triangle with at least one node larger than
1.5 cm in diameter; and, (5) an erythematous polymorphous rash, which may be targetoid or purpuric in
20% of cases. The disease must be distinguished from toxic shock syndrome, streptococcal scarlet fever,
Stevens-Johnson syndrome, juvenile rheumatoid arthritis, measles, adenovirus infection, echovirus
infection, and drug reactions.

Treatment significantly diminishes the risk of complications. Current recommendations are to hospitalize
the patient for treatment with intravenous immune globulin. In addition, aspirin is used for both its
anti-inflammatory and antithrombotic effects. While prednisone is used to treat other forms of vasculitis,
it is considered unsafe in Kawasaki disease, as a study has shown an extraordinarily high rate of coronary
artery aneurysm with its use.

Ref: Scuccimarri R: Kawasaki disease. Pediatr Clin North Am 2012;59(2):425-445.

Item 219

ANSWER: D

This patient has classic findings for acute rectal fissure. Although patients often require an internal
sphincterotomy, nonsurgical measures that relax the sphincter have proven helpful. Botulinum toxin
injected into the internal sphincter has proven most beneficial, but topical preparations are not yet available
and have not been shown to be effective for this problem. Corticosteroid creams may decrease the pain
temporarily, but potent fluorinated corticosteroid creams such as clobetasol are not indicated in the
treatment of fissure. Capsaicin cream can be helpful for pruritus ani, but not for anal fissures. Drugs that
dilate the internal sphincter, including diltiazem, nifedipine, and nitroglycerin ointment, have proven to
be beneficial in healing acute fissures, but usually have to be compounded by a pharmacist.

Ref: Fargo MV, Latimer KM: Evaluation and management of common anorectal conditions. Am Fam Physician
2012;85(6):624-630.

73
Item 220

ANSWER: A

Children generally have a benign course after insect stings, but those who have a moderate to severe
systemic reaction have a high risk of future reactions. Local reactions may initially look like cellulitis, but
antibiotic therapy is not needed. The treatment of choice for anaphylaxis subsequent to an insect sting is
systemic epinephrine. Corticosteroids may be given as adjunctive treatment. Immunotherapy may induce
cellular suppression after 4 or 5 years. Patients who stop venom immunotherapy after 1 or 2 years continue
to be at moderate risk for systemic allergic reaction to future stings.

Ref: Golden DB: Stinging insect allergy. Am Fam Physician 2003;67(12):2541-2546. 2) Kliegman RM, Stanton BF, Geme JW
III, et al (eds): Nelson Textbook of Pediatrics, ed 19. Elsevier Saunders, 2011, pp 808-809.

Item 221

ANSWER: B

Spironolactone is now recommended for treating resistant hypertension, even when hyperaldosteronism
is not present. A longer-acting diuretic such as chlorthalidone is also recommended for treating
hypertension, particularly in resistant cases with normal renal function. There is no benefit to switching
from an ACE inhibitor to an ARB. Nitrates have some effect on blood pressure but are recommended only
for patients with coronary artery disease.

Ref: Nishizaka MK, Zaman MA, Calhoun DA: Efficacy of low-dose spironolactone in subjects with resistant hypertension. Am
J Hypertens 2003;16(11 Pt 1):925-930. 2) Myat A, Redwood SR, Qureshi AC, et al: Resistant hypertension. BMJ
2012;345:e7473. 3) Adams M, Bellone JM, Wright BM, Rutecki GW: Evaluation and pharmacologic approach to patients
with resistant hypertension. Postgrad Med 2012;124(1):74-82.

Item 222

ANSWER: A

Chronic excessive alcohol intake produces functional changes in neurotransmitter activity that can lead to
a net increase in excitatory neuroreceptor activity when the person stops drinking. Withdrawal can be
divided into four levels of severity: minor, major, seizures, and delirium tremens. Minor alcohol
withdrawal is characterized by tremor, anxiety, nausea, vomiting, and/or insomnia 624 hours after the
patients last drink. Major withdrawal occurs 1072 hours after the last drink and can include the signs and
symptoms of minor withdrawal, as well as visual and auditory hallucinations, diaphoresis, tachycardia,
and elevated blood pressure. Alcoholic seizure generally occurs within 2 days of the last drink and may
be the only sign of withdrawal, although approximately one-third of these patients will progress to delirium
tremens. The onset of delirium tremens can occur anytime within 310 days following the last drink. The
defining clinical finding is delirium, but the findings seen in milder forms of alcohol withdrawal can also
be present, and may be more severe. Fever is most often seen with delirium tremens and is less common
with less severe forms of alcohol withdrawal.

Ref: Rakel RE, Rakel DP (eds): Textbook of Family Medicine, ed 8. Elsevier Saunders, 2011, pp 1095-1097. 2) Manasco A,
Chang S, Larriviere J, et al: Alcohol withdrawal. South Med J 2012;105(11):607-612.

74
Item 223

ANSWER: E

Cyanosis and tachypnea may be a sign of transient tachypnea of the newborn (TTN), or may be the first
signs of a more serious health condition. Many of the severe conditions may be ruled out through the
history (hyaline membrane disease is unlikely in this full-term infant) and physical examination (choanal
atresia may be ruled out at the bedside). After these initial steps are taken, a chest radiograph and
laboratory studies will help rule out many urgent and life-threatening conditions, such as pneumothorax,
pneumonia, severe anemia, sepsis, etc. An EKG is less helpful in determining causes since it cannot
differentiate defects that cause cyanotic heart disease. Infusion of prostaglandin E1 should be started only
if the condition is determined to be ductal dependent; this is not the case in this scenario. Bag-mask
ventilation is not warranted in this child, who is alert and breathing and has a normal heart rate. At this
time it is reasonable to start investigations into the cause of the childs problems before resorting to more
extreme measures such as intubation.

Ref: Hermansen CL, Lorah KN: Respiratory distress in the newborn. Am Fam Physician 2007;76(7):987-994. 2) Tutdibi E,
Gries K, Bcheler M, et al: Impact of labor on outcomes in transient tachypnea of the newborn: Population-based study.
Pediatrics 2010;125(3):e577-e583.

Item 224

ANSWER: E

A peripheral blood smear revealing the presence of keratocytes, blister cells, and polychromatic
macrocytes is consistent with an oxidative hemolytic process. Hemolysis is seen in glucose-6-phosphate
dehydrogenase (G6PD) deficient persons, such as this patient, following an acute insult triggered by drugs,
infection, or fava bean ingestion. A blood smear suggests the diagnosis even if an assay for G6PD is
normal, which is frequently the case in heterozygous men of African ancestry. Peripheral blood smears
showing spherocytes suggest spherocytic hemolytic anemia. Sickle-shaped red blood cells indicate sickle
cell anemia, whereas hypochromic and teardrop-shaped red blood cells may be seen in thalassemia. Red
blood cell fragments are found in microangiopathic hemolytic anemia.

Ref: Bain BJ: Diagnosis from the blood smear. N Engl J Med 2005;353(5):498-507. 2) Frank JE: Diagnosis and management
of G6PD deficiency. Am Fam Physician 2005;72(7):1277-1282. 3) Longo DL, Fauci AS, Kasper DL, et al (eds):
Harrisons Principles of Internal Medicine, ed 18. McGraw-Hill, 2012, pp 878-880.

Item 225

ANSWER: B

The combination of the vasodilators hydralazine and isosorbide dinitrate has been shown to be effective
in the treatment of heart failure when standard treatment with diuretics, "-blockers, and an ACE inhibitor
(or ARB) is insufficient to control symptoms or cannot be tolerated. This combination is particularly
effective in African-Americans with NYHA class III or IV heart failure, with advantages including reduced
mortality rates and improvement in quality-of-life measures. Digoxin, a long-time standard for the
treatment of heart failure, is useful in reducing the symptoms of heart failure but has not been shown to
improve survival. Amlodipine and other calcium channel blockers do not have a direct role in the treatment
of heart failure.

Ref: Drugs for chronic heart failure. Treat Guidel Med Lett 2012;10(121):69-72.

75
Item 226

ANSWER: D

Until recently, the number of heroin overdoses had been in decline for the past few decades. Although
heroin still only accounts for about 1% of drug overdoses, it has become more common in the past few
years. Overdose is manifested by CNS depression and hypoventilation. Clinical clues include pupillary
miosis and a decreasing respiratory rate in the presence of a semi-wakeful state. In addition to
hypoventilation, a multifactorial acute lung injury occurs within 24 hours of the overdose and is associated
with hypoxemia and a hypersensitivity reaction, resulting in noncardiogenic pulmonary edema. Findings
include hypoxia, crackles on lung auscultation, and pink, frothy sputum. Treatment must include
respiratory support with intubation, mechanical ventilation, and oxygen, as well as opiate reversal with
naloxone, which may require repeat doses or intravenous infusion.

Arrhythmias and myocardial ischemia/infarction do not occur as direct pharmacologic effects of heroin,
although they may occur as a consequence of the pulmonary toxicity or the presence of other drugs taken
intentionally or otherwise (i.e., heroin cut with other agents). Acute renal injury, hepatic injury, and
thromboembolic events are also not a direct result of the pharmacologic effects of heroin.

Ref: Shannon MW, Borron SW, Burns M: Haddad and Winchesters Clinical Management of Poisoning and Drug Overdose,
ed 4. Saunders Elsevier, 2007, pp 644-645. 2) Li W, Gunja N: Illicit drug overdose: Prevalence and acute management.
Aust Fam Physician 2013;42(7):481-485.

Item 227

ANSWER: A

Acromegaly usually has an insidious onset, with a time to diagnosis on the order of 610 years. The classic
facial findings, which include enlargement of the supraorbital ridges and mandible and a widened nose,
are sometimes difficult to identify without a reference to the patients appearance from several years
earlier. Even close family members will often not notice the changes since they occur so gradually.
Patients with this condition sometimes present with nonspecific symptoms such as hyperhidrosis,
arthralgias, fatigue, and headache. More specific complaints such as visual field defects or increasing ring
and shoe sizes should prompt a physician to consider this diagnosis. The condition is caused by a growth
hormonesecreting pituitary adenoma in 98% of cases. In younger patients this disease results in gigantism.
Blood testing for growth hormone and IGF-1 is usually adequate for initial testing.

Ref: Reddy R, Hope S, Wass J: Acromegaly. BMJ 2010;341:c4189. 2) Longo DL, Fauci AS, Kasper DL, et al (eds):
Harrisons Principles of Internal Medicine, ed 18. McGraw-Hill, 2012, pp 2876-2902.

Item 228

ANSWER: C

Psychogenic polydipsia is voluntary excessive water intake, usually in excess of 1 L/hr, which overwhelms
the capacity of the kidneys to excrete free water in the urine. This disorder is seen most often in patients
with either mental illness or a developmental disability. It is most common in individuals with chronic
schizophrenia and middle-aged women with anxiety disorders. It is characterized by hyponatremia but
seldom to the point of causing symptoms. When plasma osmolarity is <280 mOsm/kg, the secretion of
antidiuretic hormone is suppressed. The water-seeking behavior is noted by others rather than being
reported by the patient.

76
The physical examination in these patients is usually unremarkable, and evidence of volume overload such
as demonstrable pitting edema should prompt suspicion of another etiology. Plasma and urine osmolarity
should be measured, along with plasma and urinary sodium. Urinary measurements are best done by
collecting a 24-hour urine sample. If the patients hyponatremia is noted to be euvolemic and hypotonic,
with a urine osmolarity <100 mOsm/kg, and if serum osmolarity is also low (<280 mOsm/kg), then a
metabolic panel and urinalysis would be helpful to exclude other causes of hyponatremia. Other etiologies
should also be considered if elevated glucose, elevated protein, or abnormal renal function is noted. With
regard to urine specific gravity, the urine should be dilute in patients with psychogenic polydipsia. Specific
gravity would be a predictable way of noting osmolarity, as a lower specific gravity would indicate a lower
urine osmolarity.

Ref: Marx JA (ed): Rosens Emergency Medicine: Concepts and Clinical Practice, ed 7. Mosby Elsevier, 2010, p 1725.

Item 229

ANSWER: C

Regular use of inhaled long-acting "2-agonists, inhaled long-acting anticholinergic agents, or inhaled
corticosteroids has been shown to reduce the risk of COPD exacerbations, with combinations of these
agents producing additional benefit compared with monotherapy. The other agents listed are helpful for
relief of symptoms of COPD but do not reduce the incidence of exacerbations.

Ref: Niewoehner DE: Outpatient management of severe COPD. N Engl J Med 2010;362(15):1407-1416.

Item 230

ANSWER: E

Clostridium difficile infections are associated with a high rate of recurrence. Approximately 20% of
patients successfully treated for C. difficile will have a relapse. For the first relapse, a 10- to 14-day course
of oral metronidazole is recommended if symptoms are moderate. If symptoms are severe, a 10- to 14-day
course of oral vancomycin is indicated. If a second relapse is confirmed, an oral vancomycin taper over
approximately 15 weeks is recommended. The regimen is 125 mg every 6 hours for 1014 days, then 125
mg every 12 hours for 7 days, then 125 mg daily for 7 days, then 125 mg every other day for 8 days,
followed by 125 mg every 3 days for 15 days.

Intravenous vancomycin is not effective for C. difficile infections. Clindamycin is a common cause of C.
difficile infection and has no role in its treatment.

Ref: Friedman LS, Brandt LJ (eds): Sleisenger & Fordtrans Gastrointestinal and Liver Disease, ed 9. Saunders, 2010, p 1901.

Item 231

ANSWER: E

The family physician or a consulting psychiatrist can make recommendations regarding driving, and the
patients family or the person designated as having power of attorney can withhold access to a vehicle, but
the state motor vehicle department reserves final judgment in these situations, and may require a driving
test.

77
Ref: Carr DB, Ott BR: The older driver with cognitive impairment: Its a very frustrating life. JAMA
2010;303(16):1632-1641.

Item 232

ANSWER: C

Signs that an asthma exacerbation may be life threatening include altered mental status, absence of
wheezing, and paradoxical chest or abdominal movement. A PaCO2 >42 mm Hg may indicate impending
respiratory failure; levels <40 mm Hg would be expected with hyperventilation of any cause. An FEV1
<40% of expected, especially after initial treatment in the emergency department, is an indication for
admission. Systemic corticosteroids are frequently used for even moderate asthma exacerbations.

Ref: Lazarus SC: Emergency treatment of asthma. N Engl J Med 2010:363(8):755-764.

Item 233

ANSWER: C

The picture shows a very localized erythematous rash that appears to be allergic dermatitis in the healing
stage. The location is very atypical of eczema, and the chronicity would rule out herpes zoster. The lesion
does not have any typical features of tinea corporis, which tends to be annular or macular with scaling.
Squamous cell cancer is exceedingly rare in children and would not be suggested by a rash that completely
disappears only to reappear again. The history and appearance are most suggestive of nickel sensitivity,
likely associated with a metal clasp or snap on the childs pants. Placing athletic tape over this clasp usually
results in resolution of the condition.

Ref: Kliegman RM, Stanton BF, Geme JW III, et al (eds): Nelson Textbook of Pediatrics, ed 19. Elsevier Saunders, 2011, pp
2250-2252.

Item 234

ANSWER: A

This patients injury is commonly referred to as jersey finger, a flexor digitorum profundus avulsion
fracture that results from forced hyperextension of a flexed DIP joint. On examination the patient will be
unable to flex the finger at the DIP joint. Radiographs will show a bony fragment at the volar surface of
the proximal distal phalanx. Because the risk of tendon retraction is high, patients with these fractures
should be referred to a hand surgeon as soon as the diagnosis is made.

Ref: Borchers JR, Best TM: Common finger fractures and dislocations. Am Fam Physician 2012;85(8):805-810.

78
Item 235

ANSWER: C

Most persons who have come in close contact with individuals confirmed to have pertussis should receive
postexposure prophylaxis, regardless of immunization status (SOR C). The decision about whether to give
postexposure chemoprophylaxis should take into account several factors, including infectiousness, degree
of exposure, potential consequences of severe pertussis in the exposed individual, and the possibility of
secondary exposure of persons at high risk, such as infants younger than 12 months of age. If there is no
contraindication, a macrolide given to close contacts, and started within 21 days of the onset of the original
patients cough, can prevent symptomatic infection. Early macrolide administration can also reduce the
duration and severity of symptoms and shorten the time the patient is contagious.

The prophylactic dosage of azithromycin is the same as the treatment dosage, which is a daily dose for 5
days. A one-dose regimen is not recommended. Prophylaxis is indicated in this patient because he was in
close contact with the infected individual and also has a 4-month-old sister, who is at high risk from
possible secondary exposure. He is up to date on all vaccinations, and DTaP vaccine is not appropriate for
use as prophylaxis. Tetracycline is contraindicated in young children.

Ref: Tiwari T, Murphy TV, Moran J, et al: Recommended antimicrobial agents for the treatment and postexposure prophylaxis
of pertussis: 2005 CDC guidelines. MMWR Recomm Rep 2005;54(RR-14):1-16. 2) Graham L: CDC releases guidelines
on antimicrobial agents for the treatment and postexposure prophylaxis of pertussis. Am Fam Physician 2006;74(2):333-336.
3) Bader MS, McKinsey DS: Postexposure prophylaxis for common infectious diseases. Am Fam Physician 2013;88(1):
25-32.

Item 236

ANSWER: B

An association with vitamin B12 deficiency has been noted for metformin, proton pump inhibitors, and
H2-blockers. It is not known whether supplementation in patients taking these drugs will prevent this, but
monitoring vitamin B12 levels may be useful. Prednisone, insulin glargine, gabapentin, and risperidone
have not been associated with vitamin B12 deficiency.

Ref: Langan RC, Zawistoski KJ: Update on vitamin B12 deficiency. Am Fam Physician 2011;83(12):1425-1430.

Item 237

ANSWER: A

A pregnancy test is an appropriate first step in the evaluation of a woman with amenorrhea and
galactorrhea who is not taking medications. If the test is negative, hyperprolactinemia should be suspected.
Provided that the sample is obtained without excessive venipuncture stress, measurement of a single
prolactin level above the upper limits of normal confirms the diagnosis of hyperprolactinemia. In the
absence of other breast signs or symptoms, breast imaging is not necessary. A brain MRI focusing on the
pituitary sella is not needed until the diagnosis of hyperprolactinemia is confirmed. Dynamic tests of the
hypothalamic-pituitary-adrenal axis, such as a dexamethasone suppression test, are generally not indicated
in the evaluation of amenorrhea/galactorrhea.

Ref: Melmed S, Casanueva FF, Hoffman AR, et al: Diagnosis and treatment of hyperprolactinemia: An Endocrine Society
clinical practice guideline. J Clin Endocrinol Metab 2011;96(2):273-288. 2) Huang W, Molitch ME: Evaluation and
management of galactorrhea. Am Fam Physician 2012;85(11):1073-1080.

79
Item 238

ANSWER: D

SSRIs are the first-line medications in the treatment of general anxiety disorder (SOR A). SSRIs can be
used long-term without concerns about abuse or tolerance. They are both effective and well tolerated, and
help treat the concomitant depression that exists in many patients with generalized anxiety disorder (GAD).
Cognitive-behavioral therapy has also been shown to be an effective treatment for GAD and has lower
attrition rates and more durable effects compared to medications (SOR A). SNRIs have also been shown
to have some benefit. Buspirone is effective in the treatment of anxiety, but does not help with depression.
The same is true for benzodiazepines, which are effective for treating GAD but do not help with depression
that might be present. Pregabalin has been used in Europe for treatment of GAD, but the FDA has not
approved this use.

Ref: Kavan MG, Elsasser G, Barone EJ: Generalized anxiety disorder: Practical assessment and management. Am Fam Physician
2009;79(9):785-791.

Item 239

ANSWER: C

Antimitochondrial antibodies are positive in 90% of patients with primary biliary cirrhosis and this test is
the first step for ruling out the disease. A liver biopsy is the most important test in antimitochondrial
antibodynegative primary biliary cirrhosis, but it is not the initial test of choice. Antinuclear antibody
testing, antismooth muscle antibody testing, and magnetic resonance cholangiopancreatography are not
appropriate.

Ref: Goldman L, Schafer AI (eds): Goldmans Cecil Medicine, ed 24. Elsevier Saunders, 2011, pp 1014-1015.

Item 240

ANSWER: A

Cold medications, including analgesic/decongestant combinations, provide modest symptom relief in adults
and older children, but are not effective and are associated with an increased risk of adverse effects in
those younger than 2 years of age (SOR A). The number needed to harm is 14.

Ref: Salisbury-Afshar E: Oral antihistamine/decongestant/analgesic combinations for the common cold. Cochrane for Clinicians
2012;86(9):812.

80
American Board of Family Medicine

2012 IN-TRAINING EXAMINATION

CRITIQUE BOOK

This book contains the answers to each question in the In-Training Examination, as well as a critique that
provides a rationale for the correct answer. Bibliographic references are included at the end of each
critique to facilitate any further study you may wish to do in a particular area.

Copyright 2012 The American Board of Family Medicine, Inc. All rights reserved.
Item 1

ANSWER: C

Keratoacanthoma is a relatively common lesion in the elderly, but is difficult to distinguish from squamous
cell carcinoma. However, it is easily distinguished from Bowens disease, basal cell carcinoma, Kaposis
sarcoma, and seborrheic keratosis. Most keratoacanthomas undergo a benign self-healing course but may
leave a large, unsightly scar. Treatment is almost always preferred, both for cosmetic reasons and to
prevent the rare case of malignant transformation. Proper treatment for a lesion with this appearance is
excisional biopsy in order to distinguish between keratoacanthoma and squamous cell carcinoma.

Ref: Habif TP: Clinical Dermatology: A Color Guide to Diagnosis and Therapy, ed 5. Mosby Elsevier, 2010, pp 790-791.

Item 2

ANSWER: D

Repeated doses of a short-acting !2-agonist and correction of hypoxia are the main elements of initial
emergency department treatment for acute asthma exacerbations in children. Nebulizer treatments are no
better than a metered-dose inhaler with a spacer (SOR A). High-dose nebulized albuterol every 20 minutes
for 1 hour has not been shown to be beneficial. In children already receiving standard treatment with
albuterol and corticosteroids the addition of intravenous magnesium sulfate has been shown to improve
lung function and reduce the need for hospitalization (SOR A). Oral administration of corticosteroids is
as effective as the intravenous route for reducing the need for hospital admission (SOR A). Mucolytics and
chest physiotherapy have not been shown to be effective in children with acute asthma attacks.

Ref: Pollart SM, Compton RM, Elward KS: Management of acute asthma exacerbations. Am Fam Physician 2011;84(1):40-47.

Item 3

ANSWER: C

Metformin has been found to reduce cardiovascular risk in patients with type 2 diabetes mellitus. It also
decreases the risk of weight gain, and unlike some oral agents it does not significantly increase the risk of
hypoglycemia. It should be continued when insulin is initiated in patients with no renal impairment (SOR
B).

Ref: Unger J: Diagnosis and management of type 2 diabetes and prediabetes. Prim Care 2007;34(4):731-759. 2) Petznick A:
Insulin management of type 2 diabetes mellitus. Am Fam Physician 2011;84(2):183-190.

Item 4

ANSWER: D

Urinary retention is a common problem in hospitalized patients, especially following certain types of
surgery. Starting an "-blocker at the time of insertion of the urethral catheter has been shown to increase
the success of a voiding trial (SOR A). Voiding trial success rates have not been shown to be improved
by leaving the catheter in for 2 weeks, immediate removal of the catheter, using a specialized catheter, or
antibiotic prophylaxis.

Ref: Selius BA, Subedi R: Urinary retention in adults: Diagnosis and initial management. Am Fam Physician
2008;77(5):643-650.

1
Item 5

ANSWER: A (Note: An article on this subject appeared in AFP after the exam was sent to the
printer, and came to our attention after the exam was scored. B is now considered a
better answer, and the critique and references have been updated to reflect this.)

This patients symptoms are consistent with irritable bowel syndrome (IBS). Her history, physical
examination, and laboratory evaluation did not show any evidence of peptic ulcer disease, celiac disease,
thyroid disease, or inflammatory bowel disease. Red flags include unintentional and unexplained weight
loss, rectal bleeding, a family history of bowel or ovarian cancer, and a change in bowel habits to looser
and/or more frequent stools persisting for more than 6 weeks in a person over 60. The patient does not
have any of these findings and therefore does not require any additional testing to confirm the diagnosis
of IBS.

Patients should be given information that explains the importance of self-help in effectively managing their
IBS. This should include information on general lifestyle, physical activity, and dietary intake. A Cochrane
review showed that soluble fiber such as psyllium is not effective for IBS. Lubiprostone is effective for
constipation-predominant IBS.
Ref: Wilkins T, Pepitone C, Alex B, Schade RR: Diagnosis and management of IBS in adults. Am Fam Physician
2012;86(5):419-426. 2) Ruepert L, Quartero AO, de Wit NJ, et al: Bulking agents, antispasmodic and antidepressants for
the treatment of irritable bowel syndrome. Cochrane Database Syst Rev 2011;(8):CD003460.

Item 6

ANSWER: C

Several prospective trials have shown significant benefits from the use of sports-specific training of the hips
and legs in preventing anterior cruciate ligament (ACL) injuries. These programs focus on plyometrics
(repetitive actions that rapidly load and contract a targeted muscle group), strength training, and balance
exercises, along with consistent feedback about proper landing technique. One such study was able to
reduce the frequency of ACL injuries in female high-school soccer players by 88%. Although the risk of
ACL injuries in female athletes is up to 10 times that of males, there is no data to show that restricting their
participation in male-dominated sports is a successful strategy to prevent injuries.
Ref: Cimino F, Volk BS, Setter D: Anterior cruciate ligament injury: Diagnosis, management, and prevention. Am Fam
Physician 2010;82(8):917-922.

Item 7

ANSWER: B

The symptoms of this patient fit the criteria for bronchiectasis, and the gold standard for diagnosis is
high-resolution chest CT. The chest film does not suggest pulmonary tuberculosis, so a PPD would not
be appropriate initially (although tuberculosis can be a cause of bronchiectasis). Bronchoscopy may
eventually be necessary, but not at this point in the investigation. This patients age, the absence of findings
of emphysema, and the lack of a family history of emphysema or liver disease make the diagnosis of
"1-antitrypsin deficiency unlikely.
Ref: Barker AF: Bronchiectasis. N Engl J Med 2002;346(18):1383-1393. 2) Goldman L, Schafer AI (eds): Goldmans Cecil
Medicine, ed 24. Elsevier Saunders, 2011, pp 548-549.

2
Item 8

ANSWER: D

While there are several guidelines for the clinical diagnosis of acute bacterial sinusitis (ABS), there is
general agreement that patients with a duration of symptoms of at least 10 days without improvement
should be treated with antibiotics, including both children and adults (SOR C). Signs and symptoms may
include nasal drainage and congestion, facial pressure and/or pain, sinus tenderness, and headache.
Recommendations for the duration of treatment vary.

One set of guidelines calls for empiric treatment with amoxicillin alone; another recommends going directly
to amoxicillin/clavulanate. Suggested alternatives include a respiratory quinolone or the combination of
a third-generation cephalosporin and clindamycin, particularly in patients with penicillin allergy. Due to
the increasing emergence of resistant Streptococcus and Haemophilus species, neither trimethoprim/sulfa-
methoxazole nor macrolides are now recommended for empiric treatment of ABS.

Data regarding the efficacy of other measures such as nasal irrigation and the use of decongestants is
limited and variable. The most recent guidelines do not recommend the use of decongestants, whether oral
or topical.

Ref: Aring AM, Chan MM: Acute rhinosinusitis in adults. Am Fam Physician 2011;83(9):1057-1063. 2) Chow AW, Benninger
MS, Brook I, et al; Infectious Diseases Society of America: IDSA clinical practice guideline for acute bacterial
rhinosinusitis in children and adults. Clin Infect Dis 2012;54(8):e72-e112.

Item 9

ANSWER: A

This patient has classic lichen planus, with pruritic, symmetrically distributed papular lesions. The
violaceous flat-topped papules, usually 36 mm in size, are distinct and so characteristic in appearance that
a biopsy is usually not necessary to make the diagnosis. First-line treatment is with high-potency topical
corticosteroids such as clobetasol, as mid-potency topical agents such as triamcinolone are ineffective.
Topical calcineurin inhibitors, including tacrolimus, can be used in cases not responding to topical
corticosteroids. While scabies can masquerade as a variety of other dermatoses, retreatment with a
scabicide is not indicated in this patient.

Ref: Usatine RP, Tinitigan M: Diagnosis and treatment of lichen planus. Am Fam Physician 2011;84(1):53-60.

Item 10

ANSWER: E

Because speech-language therapy is effective for primary expressive language disorders, referral as early
as possible is critical (SOR A). Red flags suggesting the need for immediate evaluation include no babbling
in a 12-month-old, not saying mama or dada at 18 months, a vocabulary of less than 25 words at age
2, and using less than 200 words at age 3. Children should be able to follow two-step commands by 2
years of age (SOR A).

Ref: McLaughlin MR: Speech and language delay in children. Am Fam Physician 2011;83(10):1183-1188.

3
Item 11

ANSWER: D

Pulmonary arteriovenous malformations are found in 15%30% of patients with hereditary hemorrhagic
telangiectasia (HHT), also known as Osler-Weber-Rendu syndrome. All patients with possible or
confirmed HHT should be screened for pulmonary arteriovenous malformations with contrast
echocardiography (SOR C). While contrast echocardiography is used to detect atrioseptal and ventricular
septal defects, neither of these conditions is particularly prevalent in HHT. Aortic aneurysms and
myocardial perfusion defects are also not associated with HHT.

Ref: Olitsky SE: Hereditary hemorrhagic telangiectasia: Diagnosis and management. Am Fam Physician 2010;82(7):785-790.

Item 12

ANSWER: E

When a patient presents with sudden hearing loss it is important to distinguish between sensorineural and
conductive hearing loss. Patients should be asked about previous episodes, and the workup should include
both an assessment for bilateral hearing loss and a neurologic examination. Sudden sensorineural hearing
loss is diagnosed by audiometry demonstrating a 30-decibel hearing loss at three consecutive frequencies,
with no other cause indicated from the physical examination.

Evaluation for retrocochlear pathology may include auditory brainstem response, MRI, or follow-up
audiometry. Routinely prescribing antiviral agents, thrombolytics, vasodilators, vasoactive substances, or
antioxidants is not recommended. Oral corticosteroids may be offered as initial therapy, and hyperbaric
oxygen therapy may be helpful within 3 months of diagnosis. The guidelines also strongly recommend
against routine laboratory tests or CT of the head as part of the initial evaluation.

Ref: Stachler RJ, Chandrasekhar SS, Archer SM, et al: Clinical practice guideline: Sudden hearing loss. Otolaryngol Head Neck
Surg 2012;146(3 Suppl):S1-S35.

Item 13

ANSWER: B

Hemoglobin A1c (HbA1c) levels of 6.0%6.5% indicate an increased risk for diabetes mellitus, and levels
>6.5% can be used to diagnose diabetes. Hemoglobinopathies and conditions causing hemolysis can cause
HbA1c measurements to be falsely low. The opposite effect is seen in African-Americans, who have higher
HbA1c levels than whites along the continuum of glycemia. Other tests should be used in patients with
conditions that affect HbA1c, including pregnancy.

Ref: Buysschaert M, Bergman M: Definition of prediabetes. Med Clin North Am 2011;95(2):289-297.

4
Item 14

ANSWER: A

Vocal cord dysfunction is a disorder in which the vocal cords move toward midline during inspiration or
expiration, leading to varying degrees of obstruction. It is commonly misdiagnosed as exercise-induced
asthma. There are a number of precipitating factors, including exercise, psychological conditions, irritants,
rhinosinusitis, and gastroesophageal reflux disease. Spirometry generally will show a normal expiratory
loop with a flattened inspiratory loop. In asthma and COPD the FEV1/FVC ratio is decreased, resulting
in a concave shape in the expiratory portion of the flow-volume curve. The inspiratory loops are generally
normal. Patients with restrictive lung disease have a normal FEV1/FVC ratio with a reduced FVC.

Ref: Deckert J, Deckert L: Vocal cord dysfunction. Am Fam Physician 2010;81(2):156-159. 2) Mason RJ, Broaddus VC,
Martin TR, et al: Murray & Nadels Textbook of Respiratory Medicine, ed 5. Saunders Elsevier, 2010, pp 1059-1062.

Item 15

ANSWER: B

Most cases of uncomplicated acute pyelonephritis, including the one described here, can be managed in
the outpatient setting. Findings that might prompt consideration of inpatient management include comorbid
conditions (e.g., renal dysfunction, urologic disorders, diabetes mellitus, advanced liver or cardiac
disease), hemodynamic instability, male sex, metabolic derangements, pregnancy, severe pain, a toxic
appearance, an inability to take liquids by mouth, or a temperature >39.4C (103.0F).

Nitrofurantoin for 5 days is an appropriate treatment for an uncomplicated urinary tract infection, but not
for pyelonephritis. Amoxicillin is generally not considered first-line treatment for pyelonephritis because
of a high prevalence of resistance to oral !-lactam antibiotics, and it should only be chosen if susceptibility
results for the urinary isolate are known and indicate likely activity. Fluoroquinolones, such as
ciprofloxacin, are the preferred empiric antibiotic treatment for outpatient treatment of pyelonephritis, as
long as the local prevalence of resistance to community-acquired Escherichia coli is !10%.

Ref: Colgan R, Williams M, Johnson JR: Diagnosis and treatment of acute pyelonephritis in women. Am Fam Physician
2011;84(5):519-526.

Item 16

ANSWER: B

The attending physician is responsible for determining capacity and incapacity for decision making. The
extent, cause, and probable duration of any incapacity should be documented in the clinical record.

Ref: Olick RS: Defining features of advance directives in law and clinical practice. Chest 2012;141(1):232-238.

5
Item 17

ANSWER: D

Certain drugs can affect taste more than smell, but this does not include the bisphosphonates. Olfactory
disorders may be associated with deficiencies of vitamins A, B6, B12, and trace metals, but not with vitamin
D deficiency. Celiac disease is not known to cause a decreased ability to smell. Rare tumors involving the
olfactory region of the brain can affect smell, and are best detected by MRI.

Ref: Bromley SM: Smell and taste disorders: A primary care approach. Am Fam Physician 2000;61(2):427-436. 2) Daniels C,
Gottwald B, Pause BM, et al: Olfactory event-related potentials in patients with brain tumors. Clin Neurophysiol
2001;112(8):1523-1530. 3) Mainland JD, Johnson BN, Khan R, et al: Olfactory impairments in patients with unilateral
cerebellar lesions are selective to inputs from the contralesional nostril. J Neurosci 2005;25(27):6362-6371.

Item 18

ANSWER: E

The American Heart Association recommends a goal blood pressure of 130/80 mm Hg or less for the
treatment of hypertension in patients with diabetes mellitus, chronic kidney disease, or coronary artery
disease.

Ref: Pflieger M, Winslow BT, Mills K, Dauber IM: Medical management of stable coronary artery disease. Am Fam Physician
2011;83(7):819-826.

Item 19

ANSWER: B

Effective treatments for chronic orthostatic hypotension include fludrocortisone, midodrine, and
physostigmine (SOR B). Clonidine, pseudoephedrine, terbutaline, and theophylline are not appropriate
therapies.

Ref: Lanier JB, Mote MB, Clay EC: Evaluation and management of orthostatic hypotension. Am Fam Physician
2011;84(5):527-536.

Item 20

ANSWER: B

Dysthymic disorder is characterized by depressed mood for at least 2 years in addition to at least two of
the following: change in appetite, alteration in sleep, low energy, low self-esteem, poor concentration,
or feelings of hopelessness. There must be no history of a manic or hypomanic episode, substance abuse,
a chronic psychotic disorder, or an organic cause.

Symptoms of major depression are similar to those of dysthymia and can occasionally be difficult to
distinguish from dysthymia. This patients lifelong history of a depressed mood not triggered by any
particular depressing event, and the predominance of patient complaints as opposed to objective signs,
indicate that major depression is not the diagnosis in this case.

6
Bipolar disorder is characterized by major depression with periods of mania. Cyclothymia is characterized
by dysthymia with periods of hypomania. Adjustment disorder with depressed mood is characterized by
impaired social or occupational functioning or abnormal symptoms within 3 months of a stressor.

Ref: Sadock BJ, Sadock VA, Ruiz P (eds): Kaplan & Sadocks Comprehensive Textbook of Psychiatry, ed 9. Lippincott Williams
& Wilkins, 2009, pp 1711-1713.

Item 21

ANSWER: A

This patient has the classic pulmonary manifestations of Ascaris infection, which develop during the
transpulmonary passage of Ascaris larvae (SOR C). The larvae produce a syndrome of transient
eosinophilic pulmonary infiltrates, commonly referred to as Lffler syndrome. Ascaris infection is the most
common worldwide cause of this syndrome. Symptoms develop when larvae are within the lungs,
approximately 912 days after ingestion of Ascaris eggs. Patients may develop the following symptoms
and signs:

an irritating, nonproductive cough and burning substernal discomfort


dyspnea and blood-tinged sputum
urticaria during the first few days of the illness (15% of patients)
fever, which infrequently exceeds 38.3C (101.0F)
crackles and wheezing, with no signs of consolidation
hepatomegaly

The acute symptoms generally subside within 510 days, depending upon the severity of the illness. The
chest radiograph may show round or oval infiltrates ranging in size from several millimeters to several
centimeters in both lung fields; these lesions are more likely to be present when blood eosinophilia exceeds
10%. The infiltrates are migratory and may become confluent in perihilar areas, and usually clear
completely after several weeks.

Taenia does not infiltrate the lungs, but forms cysts in the muscles. Diphyllobothrium latum, the fish
tapeworm, does not cause pulmonary problems. Enterobius (pinworm) does not migrate from the
gastrointestinal tract into other organs.

Ref: Parasites - Ascariasis. Centers for Disease Control and Prevention, 2010. 2) Eckstein B: Primary care for refugees. Am
Fam Physician 2011;83(4):429-436.

Item 22

ANSWER: E

Nipple pain with breastfeeding is extremely common, with some studies reporting a prevalence of up to
96%. Preventing or alleviating nipple pain is important for comfort, but also for promoting breastfeeding
in general. The best intervention for nipple pain is education on proper positioning and attachment of the
infant. Topical remedies may also be effective, although no one topical agent has been shown to be clearly
superior, and none is as effective as education on positioning and latch-on.

Ref: Lochner JE, Livingston CJ, Judkins DZ: Clinical inquiries: Which interventions are best for alleviating nipple pain in
nursing mothers? J Fam Pract 2009;58(11):612a-612c.

7
Item 23

ANSWER: C

Also known as erythema infectiosum or fifth disease, parvovirus B19 infection is a fairly common cause
of an exanthematous rash and arthritis in younger women. This infection should be particularly suspected
in health-care workers who have frequent contact with children. The specific characteristics of the rash,
the pattern of joint involvement, and the place of employment in an otherwise healthy person all offer clues
suggesting parvovirus B19 as the infecting agent. Measles virus, adenovirus, and HIV rarely cause
arthritis, although HIV infection can cause a musculoskeletal syndrome later in the disease.
Varicella-zoster virus may cause large-joint arthritis, but the rash is distinctively vesicular and pruritic.

Ref: Longo DL, Fauci AS, Kasper DL, et al (eds): Harrisons Principles of Internal Medicine, ed 18. McGraw-Hill, 2012, pp
1478-1480.

Item 24

ANSWER: C

Diagnostic paracentesis is recommended for patients with ascites of recent onset, as well as for those with
chronic ascites who present with new clinical findings such as fever or abdominal pain. A neutrophil count
>250/mL is diagnostic for peritonitis. Once peritonitis is diagnosed, antibiotic therapy should be started
immediately without waiting for culture results. Bloody ascites with abnormal cytology may be seen with
hepatoma but is not typical of peritonitis. The ascitic fluid pH does not become abnormal until well after
the neutrophil count has risen, so it is a less reliable finding for treatment purposes. A protein level >1
g/dL is actually evidence against spontaneous bacterial peritonitis.

Ref: Longo DL, Fauci AS, Kasper DL, et al (eds): Harrisons Principles of Internal Medicine, ed 18. McGraw-Hill, 2012, pp
1077-1078, 2518-2519.

Item 25

ANSWER: D

There are a number of alternatives to the bisphosphonates. Unfortunately, efficacy data is not encouraging
for most of them. Intravenous zoledronic acid has been shown to reduce both hip fracture risk and
vertebral fracture risk. Teriparatide reduces vertebral fracture risk but not hip fracture risk. The same is
true for raloxifene and calcitonin salmon.

Ref: Favus MJ: Bisphosphonates for osteoporosis. N Engl J Med 2010;363(21):2027-2035.

Item 26

ANSWER: B

The syndrome of inappropriate secretion of antidiuretic hormone should be suspected in any patient who
has hyponatremia and excretes urine that is hypertonic relative to plasma. A urine sodium concentration
>20 mEq/L combined with a low BUN level provides further support for the diagnosis. Additional
findings may include weakness, lethargy, mental confusion, and weight gain.

8
Sodium depletion usually causes clinical features of dehydration, an elevated BUN level, and a urine
sodium concentration <20 mEq/L. Primary polydipsia almost invariably results in dilute urine with low
osmolality when compared to serum. Renal failure is unlikely with a BUN level of 4 mg/dL. Adrenal
insufficiency is also unlikely, as most patients will have skin pigmentation, weight loss, and hypotension.
A normal serum potassium level is also inconsistent with the diagnosis of adrenal insufficiency.

Ref: Goldman L, Schafer AI (eds): Goldmans Cecil Medicine, ed 24. Elsevier Saunders, 2011, pp 730-732. 2) Taal MW,
Chertow GM, Marsden PA, et al: Brenner & Rectors The Kidney, ed 9. Elsevier Saunders, 2011, pp 576-580.

Item 27

ANSWER: D

Endotracheal suctioning of vigorous infants born through meconium-stained amniotic fluid is not
recommended (SOR C). Although infants born through thick meconium are more likely to develop
aspiration syndrome, endotracheal suctioning does not provide any benefit over expectant management in
preventing this condition or other respiratory problems. Endotracheal suctioning may be useful if the
infant is not vigorous or shows signs of respiratory depression.

Suctioning of the infants stomach can be done electively but is not required for immediate management.
Positive pressure ventilation is indicated for ventilatory support of newborns with respiratory depression
who are not born through meconium-stained amniotic fluid.

Ref: Wiswell TE, Cannon CM, Jacob J, et al: Delivery room management of the apparently vigorous meconium-stained neonate:
Results of the multicenter, international collaborative trial. Pediatrics 2000;105(1 Pt 1):1-7. 2) Raghuveer TS, Cox AJ:
Neonatal resuscitation: An update. Am Fam Physician 2011;83(8):911-918.

Item 28

ANSWER: B

When family dynamics lead to conflict during an office visit, it is best for the physician to attempt to
remain neutral by avoiding triangulation, which occurs when the two sides in conflict each attempt to align
with a third party. Priority should be given to the patients right to privacy and confidentiality, and the
physician should ask permission from the patient to discuss his or her health issues with other people.
Physicians should always remember who they are primarily responsible to.

Ref: Omole FS, Sow CM, Fresh E, et al: Interacting with patients family members during the office visit. Am Fam Physician
2011;84(7):780-784.

Item 29

ANSWER: E

DRESS is an acronym for Drug Reaction with Eosinophilia and Systemic Symptoms. The hallmark of
DRESS syndrome is erythroderma accompanied by fever, lymphadenopathy, elevation of liver enzymes,
and eosinophilia. The offending medication should be discontinued immediately and treatment with
corticosteroids should be initiated. Seizure medications such as carbamazepine, phenytoin, lamotrigine,
and phenobarbital are responsible for approximately one-third of cases. Allopurinol-associated DRESS
syndrome has the highest mortality rate.

9
Toxic shock syndrome should be suspected in patients with erythroderma, hypotension, and laboratory
evidence of end-organ involvement (elevated liver enzymes or kidney function studies, anemia,
thrombocytopenia, or elevation of creatine kinase). Treatment with intravenous clindamycin, which inhibits
toxin synthesis, should be undertaken immediately.

Stevens-Johnson syndrome is characterized by a vesiculobullous rash with mucocutaneous involvement,


and erysipelas is a painful localized rash with well-demarcated borders. Red man syndrome is associated
with vancomycin.

Ref: Cacoub P, Musette P, Descamps V, et al: The DRESS syndrome: A literature review. Am J Med 2011;124(7):588-597.
2) McQueen A, Martin SA, Lio PA: Derm emergencies: Detecting early signs of trouble. J Fam Pract 2012;61(2):71-78.

Item 30

ANSWER: C

In cases of drug overdose, several critical physical findings must be evaluated. The most important is the
size of the pupils. Tolerance rarely reduces the miotic effects of narcotic medications. A patient who is
comatose, with decreased breathing, a slow pulse, and small pupils should be strongly suspected of having
overdosed on a narcotic. Naloxone should be administered to reverse these effects. The response to
treatment with naloxone is irregular. Cerebral infarction in the pontine angle, organophosphate poisoning,
phenothiazine overdose, and treatment for glaucoma can also cause constricted pupils, but these
associations are seen much less frequently than narcotics overdose.

Ref: Longo DL, Fauci AS, Kasper DL, et al (eds): Harrisons Principles of Internal Medicine, ed 18. McGraw-Hill, 2012, p
3554.

Item 31

ANSWER: C

Alcoholic ketoacidosis generally occurs in a patient who has been drinking heavily without eating. Blood
glucose levels are usually low or normal, and volume depletion associated with nausea, vomiting, and
abdominal pain is the norm. Patients typically have high osmolal and anion gaps. Treatment of alcoholic
ketoacidosis includes vigorous volume repletion with normal saline, along with administration of thiamine
and glucose. Only in the rare presence of marked acidemia (pH <7.10) is the administration of
bicarbonate thought to be necessary. Though insulin levels may be low, hyperglycemia is seldom found.
N-acetylcysteine and pyridoxine are not used for the treatment of alcoholic ketoacidosis. Levels of
glucagon and hydrocortisone are typically elevated in patients with alcoholic ketoacidosis.

Ref: Marx JA (ed): Rosens Emergency Medicine: Concepts and Clinical Practice, ed 7. Mosby Elsevier, 2010, pp 2375-2392.
2) Melmed S, Polonsky KS, Larsen PR, Kronenberg HM (eds): Williams Textbook of Endocrinology, ed 12. Elsevier
Saunders, 2011, pp 1436-1461.

10
Item 32

ANSWER: C

Magnesium sulfate has a long history of use for preventing seizures in preeclampsia and eclampsia, and
a recent Cochrane review confirmed that it is the preferred agent. Benzodiazepines and fosphenytoin are
secondary agents that can be used if magnesium sulfate fails, but they are not as effective. Nimodipine was
also shown to be less effective than magnesium sulfate. Delivery is indicated, but magnesium sulfate must
also be administered (SOR A).

Ref: Duley L, Glmezoglu AM, Henderson-Smart DJ, Chou D: Magnesium sulphate and other anticonvulsants for women with
pre-eclampsia. Cochrane Database Syst Rev 2010;(11):CD000025. 2) Fogleman CD: Magnesium sulfate and other
anticonvulsants for women with preeclampsia. Am Fam Physician 2011;83(11):1269-1270.

Item 33

ANSWER: A

Medicare eligibility for home oxygen therapy is based on oxygen saturation. To qualify for continuous
long-term oxygen therapy the patient must have a PaO2 !55 mm Hg or an SaO2 !88 mm Hg.

Ref: Oxygen therapy supplies: Complying with documentation & coverage requirements. Centers for Medicare & Medicaid
Services, Medicare Learning Network, 2011.

Item 34

ANSWER: A

Sialadenosis, bilateral noninflammatory enlargement of the parotid gland, is associated with diabetes
mellitus. Periodontal bleeding and inflammation, candidiasis, and delayed wound healing also are
associated with diabetes mellitus.

Tooth erosion can be an oral manifestation of gastroesophageal reflux disease or bulimia. Cobblestone oral
mucosa is seen in Crohns disease. Diffuse melanin pigmentation is an oral finding of Addisons disease.
Painful oral ulcers occur in several conditions, including Behet syndrome, aphthous ulcers, pemphigus,
and pemphigoid.

Ref: Chi AC, Neville BW, Krayer JE, Gonsalves WC: Oral manifestations of systemic disease. Am Fam Physician
2010;82(11):1381-1388.

11
Item 35

ANSWER: C

This lesion is suspicious for melanoma, based on the asymmetry, irregular border, color variegation, and
size larger than 6 mm. In addition, a history of evolution of the lesion, with changes in size, shape, or
color, has been shown in some studies to be the most specific clinical finding for melanoma. The preferred
method of biopsy for any lesion suspicious for melanoma is complete elliptical excision with a small
margin of normal-appearing skin. The depth of the lesion is crucial to staging and prognosis, so shave
biopsies are inadequate. A punch biopsy of the most suspicious-appearing area is appropriate if the location
or size of the lesion makes full excision inappropriate or impractical, but a single punch biopsy is unlikely
to capture the entire malignant portion in larger lesions. Electrodesiccation and curettage is not an
appropriate treatment for melanoma. Mohs surgery is sometimes used to treat melanomas, but is not used
for the initial diagnosis.

Ref: Shenenberger DW: Cutaneous malignant melanoma: A primary care perspective. Am Fam Physician 2012;85(2):161-168.

Item 36

ANSWER: A

Most episodes of pharyngitis are caused by viral rather than bacterial infections. The use of clinical
decision rules for diagnosing group A !-hemolytic streptococcal pharyngitis improves quality of care while
reducing unwarranted treatment and overall cost (SOR A). The original Centor score used four signs and
symptoms to estimate the probability of acute streptococcal pharyngitis in adults with a sore throat, and
was later modified by adding age as a fifth criterion. One point each is assigned for (1) absence of cough,
(2) swollen, tender anterior cervical nodes, (3) temperature >38.0C (100.4F), and (4) a tonsillar
exudate and swelling. One point is added for patients between the ages of 3 and 14 years, and a point is
subtracted for patients over the age of 45. The cumulative score determines the likelihood of streptococcal
pharyngitis and the need for antibiotics, and guides testing strategies. Patients with a score of zero or 1 are
at very low risk for streptococcal pharyngitis and do not require testing or antibiotic therapy. Patients with
a score of 23 should be tested using a rapid antigen test or throat culture, and a positive result warrants
antibiotic therapy. Patients with a score of 4 or higher are at high risk for streptococcal pharyngitis, and
empiric treatment may be considered. This patients score is zero, and no testing or treatment is warranted.

Ref: Choby BA: Diagnosis and treatment of streptococcal pharyngitis. Am Fam Physician 2009;79(5):383-390. 2) Wessels MR:
Streptococcal pharyngitis. N Engl J Med 2011;364(7):648-655.

Item 37

ANSWER: C

The understanding of fibromyalgia has been rapidly expanding in recent years, primarily due to the use
of functional magnetic resonance imaging (fMRI). The hallmark of fibromyalgia is an exaggerated
response to painful stimuli, or an attribution of pain to a stimulus that is normally not painful. Early
research focused on peripheral tissues as the source of this condition. However, the cause has now been
recognized as an abnormality in the central nervous system, which can be seen on fMRI. This physical
brain abnormality differentiates fibromyalgia from psychogenic conditions such as conversion disorder and
malingering.

12
Any evidence of an actual abnormality of the peripheral tissue on physical examination, blood tests, or an
imaging study in a patient suspected to have fibromyalgia should raise the suspicion that another diagnosis
is also present. It is estimated that up to 25% of patients who have a definable rheumatologic condition
such as lupus or rheumatoid arthritis also suffer from fibromyalgia. Many patients who develop
fibromyalgia started having symptoms in the wake of a viral infection (especially Epstein-Barr virus).
However, these viral illnesses are believed to trigger a genetic predisposition rather than being a necessary
cause of this condition.

Ref: Clauw DJ: Fibromyalgia: An overview. Am J Med 2009;122(12 Suppl):S3-S13. 2) Clauw DJ, Arnold LM, McCarberg
BH; FibroCollaborative: The science of fibromyalgia. Mayo Clin Proc 2011;86(9):907-911.

Item 38

ANSWER: E

Slipped capital femoral epiphysis is often misdiagnosed, as the symptoms are frequently vague. It is the
most common hip disorder in adolescents, with the age range being 9 to 15 years. It occurs when the
proximal femoral epiphysis slips posteriorly and inferiorly on the femoral neck through the growth plate.
The typical presentation is a limping child who may have pain in the groin, hip, thigh, or knee. Very often
the pain is vague and poorly localized. It occurs more often in boys, with African-Americans and Pacific
Islanders having a higher rate of involvement, possibly due to increased levels of obesity in these
population groups.

Physical findings vary, depending on the severity of the slippage. A child with a severe slip may not be
able to bear weight. Obligatory external rotation of the involved hip is noted when the hip is passively
flexed to 90. Radiographs are needed to diagnose unstable slipped capital epiphysis, and should include
frog-leg lateral views and anteroposterior views of both hips.

Another cause of hip pain in adolescent patients is apophyseal avulsion fractures. Clinical features include
pain after a sudden, forceful movement. Hip apophysitis presents as activity-related hip pain with a history
of overuse and negative radiographs. In children under the age of 10 years, transient synovitis is also a
common cause of hip pain. It occurs after a viral illness and is associated with negative radiographs but
positive laboratory tests. Fractures may be seen in children on occasion, but there will be a history of
trauma. Septic arthritis is an infrequent cause of hip pain in children, but patients have a history of fever
with elevation of the WBC count and inflammatory joints. The diagnosis would be confirmed by joint
aspiration. Legg-Calv-Perthes disease is also infrequent, and features include vague hip pain with
decreased internal rotation of the hip. The diagnosis is based on findings from radiographs or MRI.

Ref: Sawyer JR, Kapoor M: The limping child: A systemic approach to diagnosis. Am Fam Physician 2009;79(3):215-224. 2)
Peck D: Slipped capital femoral epiphysis: Diagnosis and management. Am Fam Physician 2010;82(3):258-262.

13
Item 39

ANSWER: E

It can be difficult to determine the point at which changes of normal aging are more appropriately
considered disease processes. Although the direction of expected change is generally well understood,
variables such as the level of fitness and overall health of an individual affect the degree of change. As the
body ages, the measured left ventricular ejection fraction, heart rate variability, and maximum heart rate
trend downward, the walls of the major aorta and major arteries stiffen, and the vasodilator capacity of
most smaller vessels is reduced (SOR A). The arterial wall changes increase peripheral resistance and
result in an increase in blood pressure. Positive adaptive changes have been shown in older adults who
engage in regular aerobic exercise, however, and these changes can be measured after only 3 months of
moderate-intensity exercise (SOR A).

Ref: American College of Sports Medicine, Chodzko-Zajko WJ, Proctor DN, et al: American College of Sports Medicine
position stand: Exercise and physical activity for older adults. Med Sci Sports Exerc 2009;41(7):1510-1530.

Item 40

ANSWER: C

Older patients are at higher risk for hypoglycemia caused by oral antidiabetic agents. Glyburide is
associated with a significantly greater risk of symptomatic hypoglycemia than other second-generation
sulfonylurea hypoglycemic agents.

Metformin decreases liver production of glucose and is not associated with hypoglycemia. Even so, this
patients creatinine elevation is a contraindication to metformin use, as it increases the risk of lactic
acidosis.

Glimepiride, glipizide, and repaglinide stimulate insulin release and increase the risk of hypoglycemia.
However, the risk of symptomatic hypoglycemia is substantially lower compared to the risk associated with
glyburide in patients with similar hemoglobin A1c values (SOR B).

Ref: Gangji AS, Cukierman T, Gerstein HC, et al: A systematic review and meta-analysis of hypoglycemia and cardiovascular
events: A comparison of glyburide with other secretagogues and with insulin. Diabetes Care 2007;30(2):389-394. 2)
Nathan DM, Buse JB, Davidson MB, et al: Medical management of hyperglycemia in type 2 diabetes: A consensus
algorithm for the initiation and adjustment of therapy: A consensus statement of the American Diabetes Association and
the European Association for the Study of Diabetes. Diabetes Care 2009;32(1):193-203. 3) Rogers M, Sands C, Self TH:
Sulfonylurea-induced hypoglycemia: The case against glyburide. Consultant 2011;51(2):97-99.

Item 41

ANSWER: E

Acute low back pain is one of the most common presenting symptoms in family medicine practices. In the
absence of red flags such as fever, a history of cancer, or neurologic deficits, patients can be successfully
treated with conservative therapy. Interventions that have been shown to be beneficial include
non-benzodiazepine muscle relaxers (SOR A). They are most effective in the first 12 weeks but can be
used for up to 4 weeks. Additional beneficial treatments include physical therapy, acetaminophen, and
NSAIDs. Bed rest is inadvisable for patients with low back pain (SOR A). Patients who stay active have
better outcomes than those who stay at rest.

14
There is no good evidence that oral corticosteroids are beneficial for acute back pain, and insufficient
evidence that massage therapy is effective. Lumbar traction provides no benefit in acute low back pain
(SOR B).

Ref: Casazza BA: Diagnosis and treatment of acute low back pain. Am Fam Physician 2012;85(4):343-350.

Item 42

ANSWER: E

Gastroesophageal reflux disease is one of the most common causes of chronic cough. Patients with silent
gastroesophageal reflux may not have the classic symptoms of heartburn and regurgitation. The diagnosis
is based on resolution of the cough with an empiric trial of a proton pump inhibitor, although a chest
radiograph should be obtained in all patients with a chronic cough to exclude bronchiectasis, tuberculosis,
and sarcoidosis. Asthma is another frequent cause of chronic cough, but it can be ruled out with normal
pulmonary function tests.

Ref: Benich JJ III, Carek PJ: Evaluation of the patient with chronic cough. Am Fam Physician 2011;84(8):887-892.

Item 43

ANSWER: E

The American College of Cardiology Foundation/American Heart Association guidelines for early
cardiovascular assessment do not recommend lipoprotein and apolipoprotein levels. A C-reactive protein
level can help to determine the need for statin therapy in men 50 and older and women 60 and older whose
LDL-cholesterol levels are <130 mg/dL and who are not on lipid-lowering medication, hormone therapy,
or immunosuppressive therapy, and who do not have clinical coronary heart disease, diabetes mellitus,
chronic kidney disease, severe inflammatory disease, or contraindications to statins. A C-reactive protein
level may also be reasonable in younger patients with intermediate, but not low, cardiovascular risk.

Measurement of cardiac calcium levels is reasonable in patients whose cardiovascular risk is intermediate
(10-year risk 10%20%) or low-to-intermediate (10-year risk 6%10%). An ankle-brachial index is
reasonable for intermediate-risk, but not low-risk, patients. At this point in time, the patient described here
does not meet any recommended criteria for further testing.

Ref: Randel A: ACCF/AHA release guidelines for early cardiovascular risk assessment. Am Fam Physician 2011;84(2):234-235.
2) Greenland P, Alpert JS, Beller GA, et al: 2010 ACCF/AHA guideline for assessment of cardiovascular risk in
asymptomatic adults: Executive summary: A report of the American College of Cardiology Foundation/American Heart
Association Task Force on Practice Guidelines. Circulation 2010;122(25):2748-2764.

Item 44

ANSWER: B

The threshold for transfusion of red blood cells should be a hemoglobin level of 7 g/dL in adults and most
children.

Ref: Sharma S, Sharma P, Tyler LN: Transfusion of blood and blood products: Indications and complications. Am Fam Physician
2011;83(6):719-724.

15
Item 45

ANSWER: C

This patient demonstrates classic findings for acute respiratory distress syndrome (ARDS). In many cases
ARDS must be differentiated from heart failure. Heart failure is characterized by fluid overload (edema),
jugular venous distention, a third heart sound, an elevated BNP level, and a salutary response to diuretics.
A BNP level <100 pg/mL can help rule out heart failure (SOR A). In addition, a patient with ARDS
would not have signs of left atrial hypertension and overt volume overload.

Hypersensitivity pneumonitis is usually preceded by exposure to an inciting organic antigen such as bird
feathers, mold, or dust. Pulmonary embolus, while certainly in the differential, is unlikely to cause such
dramatic radiographic findings. Pneumothorax would be seen on the chest radiograph.

Ref: Cullan A, Grover M, Hitchcock K: FPINs clinical inquiries: Brain natriuretic peptide for ruling out heart failure. Am Fam
Physician 2011;83(11):1333-1334. 2) Saguil A, Fargo M: Acute respiratory distress syndrome: Diagnosis and management.
Am Fam Physician 2012;85(4):352-358.

Item 46

ANSWER: A

Aggressive neurosurgical intervention is not indicated to evacuate clots in patients with intracerebral
hemorrhage except in those with a cerebellar hemorrhage, which is always an indication for neurosurgical
consultation. Guidelines have been developed by the American Heart Association for lowering blood
pressure in patients with a systolic blood pressure >180 mm Hg, or a mean arterial pressure >130 mm
Hg. The use of various forms of osmotherapy, including mannitol, to prevent the development of cerebral
edema has not been shown to improve outcomes. The data regarding hypothermia induction is unclear.
Patients with an INR >1.5 should receive therapy to replace vitamin Kdependent factors and have their
warfarin withheld.

Ref: Elliott J, Smith M: The acute management of intracerebral hemorrhage: A clinical review. Anesth Analg
2010;110(5):1419-1427. 2) Nyquist P: Management of acute intracranial and intraventricular hemorrhage. Crit Care Med
2010;38(3):946-953.

Item 47

ANSWER: B

Blood flow to the kidney is autoregulated so as to sustain pressure within the glomerulus. This is influenced
by angiotensin IIrelated vasoconstriction. ACE inhibitors can impair the kidneys autoregulatory function,
resulting in a decreased glomerular filtration rate and possibly acute renal injury. This is usually reversible
if it is recognized and the offending agent stopped. NSAIDs can exert a similar effect, but they can also
cause glomerulonephritis and interstitial nephritis. Statins, haloperidol, and drugs of abuse (cocaine,
heroin) can cause rhabdomyolysis with the release of myoglobin, which causes acute renal injury.
Thrombotic microangiopathy is a rare mechanism of injury to the kidney, and may be caused by
clopidogrel, quinine, or certain chemotherapeutic agents.

Ref: Naughton CA: Drug-induced nephrotoxicity. Am Fam Physician 2008;78(6):743-750.

16
Item 48

ANSWER: E

Medical therapy with progestational drugs is the treatment of choice for menorrhagia due to endometrial
hyperplasia without atypia. Progestins convert the proliferative endometrium to a secretory one, causing
withdrawal bleeding and the regression of hyperplasia. The most commonly used form is cyclic oral
medroxyprogesterone, given 14 days per month, but implanted intrauterine levonorgestrel is the most
effective (SOR A) and also provides contraception.

High-dose estrogen supplementation would further stimulate the endometrium. Estrogen is useful in cases
where minimal estrogen stimulation is associated with breakthrough bleeding. The anti-fibrinolytic agent
tranexamic acid prevents the activation of plasminogen and is given at the beginning of the cycle to
decrease bleeding. Side effects and cost limit this treatment option, however. It may be most useful in
women with bleeding disorders or with contraindications to hormonal therapy.

NSAIDs, which decrease prostaglandin levels, reduce menstrual bleeding but not as effectively as
progestins. While mefenamic acid is marketed for menstrual cramps and bleeding, all NSAIDs have a
similar effect in this regard.

If medical management fails, hysteroscopic endometrial ablation is an option for reducing uterine bleeding
but is considered permanent and obviously will impair fertility. Hysterectomy is reserved for severe and
chronic bleeding that is not relieved by other measures.

Ref: Sweet MG, Schmidt-Dalton TA, Weiss PM, Madsen KP: Evaluation and management of abnormal uterine bleeding in
premenopausal women. Am Fam Physician 2012;85(1):35-43.

Item 49

ANSWER: A

Osgood-Schlatter disease is an inflammatory condition that is a common cause of knee pain in children and
adolescents. The diagnosis is usually based on clinical findings, although radiographs may be necessary
to rule out fractures or other problems if findings are not typical. MRI, ultrasonography, and orthopedic
referral are not usually needed. The problem is typically self-limited and responds to activity modification,
over-the-counter analgesics, stretching, and physical therapy.

Ref: Atanda A Jr, Shah SA, OBrien K: Osteochondrosis: Common causes of pain in growing bones. Am Fam Physician
2011;83(3):285-291.

Item 50

ANSWER: E

The result of neonatal varicella infection can be catastrophic, with a fatality rate approaching 30%.
Maternal immunity is ideal, but since varicella vaccination is contraindicated during pregnancy the best
alternative is advising the patient to avoid contact with infected individuals until safe postpartum
immunization is possible. Maternal varicella infection is particularly problematic during weeks 1320 of
pregnancy (resulting in a 2% risk of congenital varicella in the newborn) and when the onset of maternal
symptoms occurs from 5 days before until 2 days after delivery.

17
Administration of varicella immune globulin to the expectant mother has not been shown to benefit the
fetus or infant, but because pregnancy can increase the risk of serious complications in the mother the
Advisory Committee on Immunization Practices (ACIP) recommends that administration to pregnant
women be considered following known exposure. The ACIP also recommends that term infants born within
the 7-day window described above, as well as all preterm infants, receive varicella immune globulin, and
that those who develop any signs of varicella infection also be given intravenous acyclovir. Term infants
delivered more than 5 days after the onset of maternal varicella are thought to have adequate passive
immunity for protection and the expected benign course generally requires only observation.

Ref: Duff P: Diagnosis and management of varicella infection in pregnancy. Perinatology 2010;1(1):6-12. 2) Atkinson W,
Hambersky J, Stanton A, Wolfe C (eds): Epidemiology and Prevention of Vaccine-Preventable Diseases, ed 12. Centers
for Disease Control and Prevention, 2011, pp 301-324.

Item 51

ANSWER: D

Fluoroquinolones such as ciprofloxacin have been shown to significantly reduce the duration and severity
of travelers diarrhea when given for 13 days. Sulfacetamide is available only in a topical form for use
in the eye. Penicillin and erythromycin are not effective against the most common cause of travelers
diarrhea, enterotoxigenic Escherichia coli.

Ref: Mandell GL, Bennett JE, Dolin R (eds): Mandell, Douglas, and Bennetts Principles and Practice of Infectious Diseases,
ed 7. Churchill Livingstone, 2009, pp 1365-1367, 4025-4026.

Item 52

ANSWER: A

The primary indication for joint replacement surgery in patients with osteoarthritis is intractable pain,
which is almost always relieved by the surgery. Joint replacement may also be appropriate for patients
with significant limitations of joint function or with altered limb alignment. Range of motion, joint laxity,
and recurrent subluxation relate to musculotendinous function, and are not reliably improved by joint
replacement.

Ref: Goldman L, Schafer AI (eds): Goldmans Cecil Medicine, ed 24. Elsevier Saunders, 2011, p 1675. 2) Longo DL, Fauci
AS, Kasper DL, et al (eds): Harrisons Principles of Internal Medicine, ed 18. McGraw-Hill, 2012, p 2836.

Item 53

ANSWER: A

Heart murmurs are common in children and adolescents. Often the murmur is innocent, but it may also
be the only finding in an asymptomatic child with structural heart disease. Physical findings that should
lead one to consider evaluation for structural heart disease include increased intensity with standing, a
holosystolic murmur, a grade of 3 or higher, a harsh quality, an abnormal S2, maximal intensity at the
upper left sternal border, a diastolic murmur, or a systolic click.

18
Characteristics that are more likely to be associated with innocent murmurs include a systolic murmur, a
soft sound, a short duration, a musical or low pitch, intensity that varies with phases of respiration,
increased loudness in the supine position, and increased loudness with exercise, anxiety, or fear. If the
diagnosis of an innocent murmur cannot be made from physical findings, an echocardiogram is the most
appropriate study. A chest radiograph and EKG rarely assist in the diagnosis of heart murmurs in children
(SOR B) and should not routinely be ordered.

Ref: Frank JE, Jacobe KM: Evaluation and management of heart murmurs in children. Am Fam Physician 2011;84(7):793-800.

Item 54

ANSWER: C

Dog and cat bite wounds may appear trivial, but if they are not managed appropriately they can become
infected and may result in functional impairment. Cultures are recommended for wounds that are clinically
infected. Because it can be toxic to tissue, povidone-iodine surgical scrub should not be used. Irrigation
with either normal saline or Ringers lactate solution may reduce the rate of infection by up to twentyfold.
Tetanus immune globulin is not needed, and DTaP is not given to children 7 years of age or older.

Ref: Tintinalli JE, Kelen GD, Stapczynski JS (eds): Emergency Medicine: A Comprehensive Study Guide, ed 7. McGraw-Hill,
2011, pp 353-355.

Item 55

ANSWER: C

Intravenous antibiotics, especially imipenem, have been shown to be beneficial in patients with pancreatitis.
Patients with pancreatitis who are not vomiting do not require nasogastric tube placement. Corticosteroids
are not indicated in the management of acute pancreatitis, and pseudocysts can be managed initially with
percutaneous aspiration.

Ref: Bassi C, Larvin M, Villatoro E: Antibiotic therapy for prophylaxis against infection of pancreatic necrosis in acute
pancreatitis. Cochrane Database Syst Rev 2003;(4):CD002941. 2) Al-Omran M, Albalawi ZH, Tashkandi MF, Al-Ansary
LA: Enteral versus parenteral nutrition for acute pancreatitis. Cochrane Database Syst Rev 2010;(1):CD002837.

Item 56

ANSWER: D

Conservative treatment of grade 1 and 2 ankle sprains in athletes, consisting of the use of leg casts for 2
weeks followed by progressive increases in activity, has been found to lead to a loss of playing time of 46
weeks. Treatment consisting of an elastic wrap and use of a crutch until pain resolves produces similar
results. Early mobilization after aggressive control of inflammation is recommended. Typical treatment
includes extensive icing, compression, and elevation, followed by the application of air or gel splints. In
the first 48 hours, physical therapy begins with early mobilization, strengthening, and proprioception
retraining. In one study utilizing this more aggressive approach, athletes were able to return to functional
status in 9 days after grade 1 sprains and in 12 days after grade 2 injuries.

Ref: Marx JA (ed): Rosens Emergency Medicine: Concepts and Clinical Practice, ed 7. Mosby Elsevier, 2010, pp 677-679.
2) Bope ET, Kellerman RD (eds): Conns Current Therapy 2012. Elsevier Saunders, 2012, pp 595-596.

19
Item 57

ANSWER: C

Cognitive problems are often a feature of depression in older patients, which can make it difficult to
distinguish depression from dementia. Congruence of mood with delusions is more typical of depression.
A longer duration of cognitive problems, slow progression of cognitive problems, and no past history of
psychiatric problems are more typical of dementia.

Ref: Michels TC, Tiu AY, Graver CJ: Neuropsychological evaluation in primary care. Am Fam Physician 2010;82(5):495-502.

Item 58

ANSWER: B

Sitagliptin is a DPP-4 inhibitor. These agents slow the inactivation of incretin hormones, prolonging their
action and thereby increasing insulin release and decreasing glucagon. Sitagliptin decreases hemoglobin
A1c levels by 0.7%, but there is no data on patient-oriented outcomes or long-term safety with this
medication.

Ref: Richter B, Bandeira-Echtler E, Bergerhoff K, Lerch CL: Dipeptidyl peptidase-4 (DPP-4) inhibitors for type 2 diabetes
mellitus. Cochrane Database Syst Rev 2008;(2):CD006739. 2) Kripke C: Dipeptidyl-peptidase-4 inhibitors for treatment
of type 2 diabetes. Am Fam Physician 2009;79(5):372.

Item 59

ANSWER: B

Pleural fluid is associated with a dull-to-flat percussion note, decreased-to-absent tactile fremitus, and
decreased-to-absent breath sounds. A consolidation would be indicated by bronchial breath sounds and
increased fremitus. Emphysematous blebs and pneumothorax are hyperresonant to percussion.

Ref: Goldman L, Schafer AI (eds): Goldmans Cecil Medicine, ed 24. Elsevier Saunders, 2011, pp 607-611.

Item 60

ANSWER: D

Intranasal corticosteroids are the most effective treatment for mild to moderate allergic rhinitis and should
be first-line therapy. Second-line therapies that can be used for symptoms that do not respond to initial
treatment include antihistamines, decongestants, cromolyn, and leukotriene receptor antagonists.
Nonpharmacologic measures that may be helpful include nasal irrigation and avoiding irritants.

Ref: Sur DK, Scandale S: Treatment of allergic rhinitis. Am Fam Physician 2010;81(12):1440-1446.

20
Item 61

ANSWER: D

Drugs such as lithium, thiazide diuretics, sex hormones, and vitamins A and D can increase the serum
ionized calcium level. The gastrointestinal symptoms associated with lithium toxicity are also the most
common presenting symptoms of hypercalcemia. Hypercalcemic patients may also complain of
constipation, fatigue, lethargy, polyuria, and weakness, all the result of an increased serum level of ionized
calcium (roughly calculated to be 40% of the total serum calcium level plus 0.8 g/dL for each 1 g/dL
decrease in serum albumin below 4 g/dL). The most common causes of hypercalcemia are malignancy and
hyperparathyroidism, together accounting for over 80% of all cases. Excessive ingestion of antacids can
result in milk-alkali syndrome, another cause of hypercalcemia. Granulomatous disease and renal diseases
are other possible causes.

Although additional details are required in this case to determine the cause, primary hyperparathyroidism
augmented by medications is highly likely given the patients age and sex, and measurement of her
parathyroid hormone level must be included in the workup. No matter the cause, the treatment of
symptomatic hypercalcemia should be immediate and directed at lowering the serum calcium level. The
safest and most effective way to accomplish this is with intravenous normal saline volume replacement,
reducing the need for reabsorption of salt, water, and, coincidentally, calcium in the proximal tubules.
Because hypercalcemia often results in volume depletion, aggressive fluid replacement is often ideal,
provided there is no contraindication to doing so. Once the volume depletion is corrected the addition of
loop diuretics such as furosemide can facilitate excretion of calcium. Each of the other options has a place
in the longer term treatment of hypercalcemia in appropriate situations: bisphosphonates for malignancy,
glucocorticoids for granulomatous disease, and cinacalcet for hyperparathyroidism.

Ref: Taniegra ED: Hyperparathyroidism. Am Fam Physician 2004;69(2):333-339. 2) Moe SM: Disorders involving calcium,
phosphorus, and magnesium. Prim Care 2008;35(2):215-237.

Item 62

ANSWER: D

Metformin is well tolerated and there is good data to show it helps prevent type 2 diabetes mellitus in
high-risk patients. Pioglitazone has been shown to slow the progression from prediabetes to diabetes, but
it has more side effects and is more expensive than metformin. Neither glipizide nor exenatide is currently
recommended as a treatment for prediabetes. Acarbose has a high discontinuation rate due to side effects.

Ref: Ratner RE, Sathasivam A: Treatment recommendations for prediabetes. Med Clin North Am 2011;95(2):385-395. 2)
American Diabetes Association: Standards of medical care in diabetes2012. Diabetes Care 2012;35(Suppl 1):S11-S63.

Item 63

ANSWER: C

While spondylolysis occurs in 6% of the general population, it may be the cause of 50% of back pain in
young adults. This unilateral or bilateral vertebral defect of the pars interarticularis is likely due to
repetitive hyperextension of the posterior spine that results in a fracture or stress injury. This usually
occurs at L4-L5. Sports that put increased demands on the spine include football, gymnastics,
weightlifting, soccer, volleyball, and ballet.

21
The recommended initial study for athletes with back pain of more than 3 weeks duration is lumbar spine
radiographs, including anterior/posterior, lateral, and oblique views bilaterally. The Scotty dog with a
collar sign can be noted on the oblique view. This may not be present in early spondylolysis, so a SPECT
scan may be appropriate.

Treatment for spondylolysis consists of discontinuing the offending activity, medication for pain, physical
therapy, and possibly bracing. Healing may take 912 months.

Ref: Cassas KJ, Cassettari-Wayhs A: Childhood and adolescent sports-related overuse injuries. Am Fam Physician
2006;73(6):1014-1022.

Item 64

ANSWER: C

This childs presentation is highly suspicious for pertussis, given the severe coughing paroxysms and the
possibility of inadequate immunization. Two weeks of oral erythromycin is recommended for children with
mild to moderate illness, principally to halt the spread of the infection. Ribavirin is used for respiratory
syncytial virus infection, which is generally seen in much younger children with more respiratory distress.
The cough of pertussis often lasts several weeks. Immune globulin is not recommended.

Ref: Kliegman RM, Stanton BF, Geme JW III, et al (eds): Nelson Textbook of Pediatrics, ed 19. Elsevier Saunders, 2011, pp
944-948.

Item 65

ANSWER: A

Crystalloids are the essential component of fluid resuscitation in patients with severe burn injuries, with
lactated Ringers solution being the most commonly used. Substantial early loss of blood is unusual, and
transfusions are not often required. The use of colloids in these patients has not been shown to be helpful
and may be harmful. Hypertonic saline solution may be useful in selected patients but requires careful
monitoring and may be detrimental.

Ref: Brunicardi FC, Anderson D, Billiar T, et al (eds): Schwartzs Principles of Surgery, ed 9. McGraw-Hill, 2009, pp 200-201.

Item 66

ANSWER: B

Patients with hoarseness lasting longer than 2 weeks with risk factors for dysplasia or carcinoma, such as
smoking, heavy alcohol use, or long-standing gastroesophageal reflux disease, should be evaluated with
laryngoscopy. Inhaled corticosteroids can contribute to hoarseness.

Ref: Feierabend RH, Shahram MN: Hoarseness in adults. Am Fam Physician 2009;80(4):363-370.

22
Item 67

ANSWER: E

Otitis media is a major health problem in the United States; it is the number one reason children visit
doctors and accounts for one-fourth of all antibiotic prescriptions. With appropriate antibiotics most
patients will improve in 23 days. Persistence or worsening of symptoms requires immediate reevaluation,
since complications such as bacterial resistance or meningitis may be developing. Occasionally a persistent
middle ear effusion will be found on reexamination 1014 days after initial treatment. Inflation of the
eustachian tube using the method of Politzer or employing the Valsalva maneuver has been shown to be
ineffective, as have antihistamines and systemic steroids. Most asymptomatic effusions with mild hearing
loss will clear in 90 days if left alone.

Ref: Kliegman RM, Stanton BF, Geme JW III, et al (eds): Nelson Textbook of Pediatrics, ed 19. Elsevier Saunders, 2011, pp
2207-2209.

Item 68

ANSWER: E

The diagnosis of occupational asthma can be made when both bronchospasm and its relationship to the
work environment can be demonstrated. A history of cough, wheezing, chest tightness, or episodic
dyspnea in varying combinations or singly should lead one to suspect bronchospasm. Relating
bronchospasm to the work environment can be done in several ways. A history of exposure to a known
sensitizer is helpful, as is a pattern of symptoms occurring after exposure. With many agents the onset of
symptoms may be delayed up to several hours. A 10% decrease in FEV1 measured before and after a work
shift supports the diagnosis. Improvement of bronchospasm with removal from exposure also suggests the
diagnosis. Treatment includes both standard pharmacologic therapy and removal from exposure as soon
as possible.

Hypersensitivity pneumonitis is an immune-mediated syndrome that is not as common as occupational


asthma. It begins with malaise, fever, and myalgias 46 hours after exposure to an antigen to which the
person has become sensitized. Byssinosis is due to exposure to the dust of hemp, flax, or cotton.
Symptoms vary from reversible chest tightness on one or more days early in the work week to chronic
bronchitis and permanent obstructive lung disease. Toxic pneumonitis or pulmonary edema is the result
of very high exposure to irritant gases, metal dust, or metal fumes, usually associated with unusual
circumstances such as a fire, explosion, or spill. Benign pleural effusions are the most common sequela
during the first 20 years after asbestos exposure. The diagnosis is one of exclusion, made by ruling out
other causes of exudative effusions in workers with known asbestos exposure.

Ref: Goldman L, Schafer AI (eds): Goldmans Cecil Medicine, ed 24. Elsevier Saunders, 2011, pp 567-569.

23
Item 69

ANSWER: A

When self-management and standard management of anticoagulation therapy are compared,


self-management improves the rate of minor hemorrhage, with no difference in the rate of major
hemorrhage. Self-monitoring also improves the rate of thromboembolism. Both self-monitoring and
self-management improve the rate of all-cause mortality. When studied, patients who self-managed their
anticoagulation therapy perceived greater self-efficacy compared to patients receiving standard care, and
self-management did not increase their levels of anxiety. When all factors are considered, self-monitoring
and self-management have outcomes superior to those of standard monitoring and management.

Ref: Cayley WE Jr: Self-monitoring and self-management of anticoagulation therapy. Am Fam Physician 2011;84(3):266-268.

Item 70

ANSWER: C

The serum 25-hydroxyvitamin D level is the best indicator of overall vitamin D status because it reflects
total vitamin D from dietary intake and sunlight, as well as conversion from adipose stores in the liver.
Measurement of 1,25-dihydroxyvitamin D, the active form of vitamin D formed in the kidney, may be
necessary in advanced chronic kidney disease. 24,25-Dihydroxyvitamin D is not biologically active.
Phosphate and parathyroid hormone are involved in the regulation of vitamin D levels, but are not helpful
in determining overall vitamin D status.

Ref: Rosen CJ: Vitamin D insufficiency. N Engl J Med 2011;364(3):248-254.

Item 71

ANSWER: E

Currently, nonalcoholic fatty liver disease is the leading cause of transaminase elevations, and is becoming
increasingly common as obesity becomes more prevalent. It is estimated that some 30% of adults in the
United States have this disease. Patients with metabolic syndrome, diabetes mellitus, or elevated
triglycerides are at the highest risk. If the AST/ALT ratio is >2, especially if #-glutamyl transpeptidase
is elevated, alcoholic liver disease should be suspected.

It is well known that severe hepatotoxicity can occur with acetaminophen overdoses, and dosages of even
4 g/day for 510 days will cause enzyme elevations in more than half of healthy nondrinkers. Herbal
preparations associated with elevated liver enzymes include kava and germander. Hepatitis C can cause
transient enzyme elevations, typically of ALT.

If liver enzymes remain elevated on a repeat test 24 weeks later, the patient should be tested for hepatitis
B and C, and iron, iron binding capacity, and ferritin levels should be ordered to check for
hemochromatosis. A lipid profile and glucose level should be ordered as well, and abdominal
ultrasonography considered to look for evidence of fatty infiltration of the liver.

Ref: Oh RC, Hustead TR: Causes and evaluation of mildly elevated liver transaminase levels. Am Fam Physician
2011;84(9):1003-1008.

24
Item 72

ANSWER: D

Of the common herbal supplements, St. Johns wort interacts with the most drugs, including statins,
warfarin, and antidepressants. The other herbal supplements listed do not interact with statins. Ginkgo
biloba and ginseng may interact with warfarin.

Ref: Gardiner P, Phillips R, Shaughnessy AF: Herbal and dietary supplementDrug interactions in patients with chronic
illnesses. Am Fam Physician 2008;77(1):73-78.

Item 73

ANSWER: C

An elevated level of parathyroid hormone (or a level that is in an unexpected normal range) in a patient
with an elevated calcium level generally indicates a diagnosis of primary hyperparathyroidism. However,
these laboratory findings may also occur with lithium or thiazide use, tertiary hyperparathyroidism
associated with end-stage renal failure, or familial hypocalciuric hypercalcemia, and a medical and family
history should be obtained to assess these possibilities. The other medications listed do not cause
hypercalcemia.

Ref: Marcocci C, Cetani F: Primary hyperparathyroidism. N Engl J Med 2011;365(25):2389-2397.

Item 74

ANSWER: D

According to the 2011 update of the American Heart Association/American College of Cardiology
Foundation guidelines on secondary prevention of coronary artery disease, metoprolol succinate has the
best evidence for mortality reduction when compared to the other medications listed. The other medications
have utility, but in more specialized circumstances: losartan for those intolerant of ACE inhibitors,
clopidogrel for those intolerant of aspirin, and ezetimibe for those intolerant of statins. Spironolactone has
evidence of benefit post myocardial infarction when added to a regimen that includes an ACE inhibitor and
a !-blocker.

Ref: Smith SC Jr, Benjamin EJ, Bonow RO, et al: AHA/ACCF secondary prevention and risk reduction therapy for patients with
coronary and other atherosclerotic vascular disease: 2011 update: A guideline from the American Heart Association and
American College of Cardiology Foundation. Circulation 2011;124(22):24582473.

Item 75

ANSWER: E

The FDA states that antipsychotics are not indicated for treating dementia-related psychosis. The reason
for this is that the efficacy for antipsychotics has not been consistently shown in clinical trials and, in fact,
patients treated with olanzapine functioned worse after treatment than did those who received a placebo.
There is also evidence that these drugs may increase mortality from infection or heart-related conditions.
Practice guidelines recommend the use of antipsychotics only after other options have been exhausted and
symptoms are severe, persistent, and not responsive to nonpharmacologic interventions (SOR B).

Ref: DEmpaire I, Macalaso M: Psychoses. FP Essentials monograph series, no 385, 2011, pp 27-31.

25
Item 76

ANSWER: C

It is thought that easier access to prescription medications leads to a higher incidence of misuse by
physicians. The drugs most commonly abused are benzodiazepines and opioids. Most studies suggest that
alcoholism rates among physicians approximate those of the general population when adjusted for
socioeconomic status. The lifetime prevalence of depression also is similar for physicians and the general
population (12.8% for men and 19.5% for women). Physicians are less likely to abuse illicit drugs,
probably because of their access to prescription drugs.

Ref: OConnor PG, Spickard A Jr: Physician impairment by substance abuse. Med Clin North Am 1997;81(4):1037-1052. 2)
Center C, Davis M, Detre T, et al: Confronting depression and suicide in physicians: A consensus statement. JAMA
2003;289(23):3161-3166. 3) Baldisseri MR: Impaired healthcare professional. Crit Care Med 2007;35(2 Suppl):S106-S116.

Item 77

ANSWER: A

Campylobacter enterocolitis in children is generally a mild, self-limiting disease. However, in patients who
are sick enough to require hospitalization or who remain symptomatic by the time a bacteriologic diagnosis
has been made, antibiotic therapy is indicated. The preferred drug is oral erythromycin, which clinical
trials indicate may produce clinical improvement. Ciprofloxacin may be an effective alternative to
erythromycin in the treatment of Campylobacter, but it is contraindicated in young children. There is no
evidence that ampicillin, trimethoprim, or metronidazole is effective for this disease.

Ref: Mandell GL, Bennett JE, Dolin R (eds): Mandell, Douglas, and Bennetts Principles and Practice of Infectious Diseases,
ed 7. Churchill Livingstone, 2009, pp 2798-2799.

Item 78

ANSWER: C

Protamine sulfate is the treatment of choice for heparin overdose or significant bleeding secondary to
heparin therapy. Vitamin K is used for reversal of anticoagulation from warfarin. Vasopressin is a pressor
agent used to treat hypotensive episodes. Dabigatran is an anticoagulant used in nonvalvular atrial
fibrillation. Cryoprecipitate is a blood product used for replacement of von Willebrands factor, factor
XIII, fibrinogen, and fibronectin.

Ref: Wachter RM, Goldman L, Hollander H (eds): Hospital Medicine, ed 2. Lippincott Williams & Wilkins, 2005, p 987. 2)
Burnett B, Kopecky S, Morton C, et al: Health Care Guideline: Antithrombotic Therapy Supplement, ed 10. Institute for
Clinical Systems Improvement, 2011.

Item 79

ANSWER: D

Adults with a chronic cough lasting 2 months or longer who are nonsmokers and are not taking ACE
inhibitors should have plain radiographs to rule out specific causes prior to initiating empiric therapy (SOR
C). Any treatment should be targeted to the most likely cause. The three most common causes of chronic
cough in adults, other than ACE inhibitors, are gastroesophageal reflux disease, asthma, and upper airway
cough syndrome.

26
Patients who are taking an ACE inhibitor should be switched to another class of antihypertensive drugs.
Metoprolol does not cause a cough per se, although it may unmask preexisting asthma or COPD, resulting
in a cough. !-Blockers should not be discontinued abruptly, however. Formal spirometry and advanced
radiographic imaging have eventual roles in the evaluation of chronic cough but are expensive tests and
are not the best initial steps for evaluating a chronic cough.

Ref: Benich JJ III, Carek PJ: Evaluation of the patient with chronic cough. Am Fam Physician 2011;84(8):887-892.

Item 80

ANSWER: D

Chest compressions are recommended for a heart rate below 60 beats/min in a neonate.

Ref: Raghuveer TS, Cox AJ: Neonatal resuscitation: An update. Am Fam Physician 2011;83(8):911-918.

Item 81

ANSWER: C

At one point, the American Diabetes Association (ADA) recommended aspirin for all patients with diabetes
mellitus. They have since revised their guidelines and advise that aspirin not be used for primary
prevention of cardiovascular events unless a patients cardiovascular risk is >10% over 10 years. The
reason the ADA revised their guidelines on use of low-dose aspirin is because of the results from two
studies: the Prevention of Progression of Arterial Disease and Diabetes (POPADAD) study and the
Japanese Primary Prevention of Atherosclerosis with Aspirin for Diabetes (JPAD) study.

The POPADAD study compared aspirin versus placebo in patients with diabetes and found that death rates
from coronary heart disease and stroke were similar for the two groups, as were rates of nonfatal
myocardial infarction and nonfatal stroke. The JPAD study also compared aspirin vs. placebo in diabetic
patients, with similar rates of sudden death, nonfatal myocardial infarction, nonfatal stroke, unstable
angina, TIA, and peripheral vascular disease. Rates of fatal myocardial infarction and fatal stroke were
lower in the aspirin group.

Ref: Jackson AN, Hume AL: Aspirin for CV preventionFor which patients? J Fam Pract 2011;60(9):518-523.

Item 82

ANSWER: C

The flick sign has the highest sensitivity (93%) and specificity (96%) among the clinical findings of carpal
tunnel syndrome. This sign is defined as a history of shaking the hand or flicking the wrist in an attempt
to alleviate discomfort after being awakened with nighttime pain. Tinels sign and the Phalen maneuver
have a sensitivity of 36% and 57%, and a specificity of 75% and 58%, respectively. Thenar atrophy is
usually seen in severe and chronic cases of carpal tunnel syndrome and has a sensitivity of 16% and a
specificity of 90% (SOR B).

Ref: LeBlanc KE, Cestia W: Carpal tunnel syndrome. Am Fam Physician 2011;83(8):952-958.

27
Item 83

ANSWER: B

Evidence shows that !-blockers reduce mortality and hospitalization rates for patients with systolic heart
failure (SOR A). They should be started at a low dosage and increased to target dosages (SOR A).
!-Blockers should be considered even in patients with COPD and asthma, given their benefits. The benefit
of !-blockers is proportional to the degree of reduction in heart rate (SOR A).

Of the listed !-blockers, carvedilol has been shown to reduce the rates of death and hospitalization in heart
failure patients. Other !-blockers that have been established through randomized, controlled trials to
benefit heart failure patients are bisoprolol and metoprolol succinate. The effect of nebivolol on mortality
has not been adequately studied.

Ref: Ong HT, Kow FP: Beta-blockers for heart failure: Why you should use them more. J Fam Pract 2011;60(8):472-477.

Item 84

ANSWER: C

Cervical spondylotic myelopathy (CSM) is the most common cause of spinal cord dysfunction in the
elderly. Degenerative changes in the cervical spine, such as osteophyte formation, stiffened and
hypertrophied ligamentum flava, and spinal stenosis, can result in spinal cord compression. Symptoms
usually develop insidiously and may include neck stiffness, pain in the arm(s), tingling or numbness in the
hands, and weakness of the hands or legs. Flexion of the neck may produce a shock-like sensation down
the back, known as Lhermittes sign.

Sensory abnormalities may vary. Hyperreflexia is a characteristic physical finding. The gait may be stiff
or spastic, and atrophy of the intrinsic muscles of the hands is common. CSM can be differentiated from
amyotrophic lateral sclerosis (ALS) by the fasciculations and leg atrophy seen in ALS. Other conditions
that produce similar findings include multiple sclerosis and masses such as a metastatic tumor.

The primary diagnostic test is MRI of the cervical spine. Plain films are of little use as an initial diagnostic
procedure. Electromyography is usually not helpful, although it is occasionally needed to exclude
peripheral neuropathy.

Nonsurgical treatment such as cervical bracing may be used in mild cases of CSM, but once a frank
myelopathy occurs surgical intervention is the only option. Studies on bracing show variable results,
although it is reported that symptomatic patients may deteriorate neurologically during bracing.

Ref: Young WF: Cervical spondylotic myelopathy: A common cause of spinal cord dysfunction in older persons. Am Fam
Physician 2000;62(5):1064-1070. 2) Longo DL, Fauci AS, Kasper DL, et al (eds): Harrisons Principles of Internal
Medicine, ed 18. McGraw-Hill, 2012, pp 3372-3373.

28
Item 85

ANSWER: C

Normal newborns may lose up to 10% of their weight following birth, and should return to their birth
weight by the end of the first week of life. The steady addition of 47 oz of weight per week should result
in a doubling of birth weight by 46 months of age. During the second half of the first year of life an
addition of 35 oz/week is more the norm, resulting in a tripling of the birth weight by 1 year of age.
Breastfed infants tend to gain weight more quickly during the first 6 months of life, while formula-fed
infants do so from 612 months, with both groups having virtually equal weight gains by the end of the
first year.

Ref: Hagan JF Jr, Shaw JS, Duncan PM (eds): Bright Futures Guidelines for Health Supervision of Infants, Children, and
Adolescents, ed 3. American Academy of Pediatrics, 2008, pp 125-126.

Item 86

ANSWER: B

A Cochrane review found that melatonin was effective for reducing jet lag, especially when crossing 5 or
more time zones in an easterly direction. The drug can also be effective when crossing 24 time zones.
The most effective dosage seems to be 0.55 mg taken at bedtime starting on the day of arrival, with
higher doses being more effective. Taking it before departure does not help, and taking it earlier in the day
could make jet lag worse. There does not seem to be any benefit from taking melatonin prior to departure,
and melatonin is not recommended when flying westward.

Ref: Herxheimer A, Petrie KJ: Melatonin for the prevention and treatment of jet lag. Cochrane Database Syst Rev
2002;(2):CD001520. 2) Bope ET, Kellerman RD (eds): Conns Current Therapy 2012. Elsevier Saunders, 2012, p 665.

Item 87

ANSWER: C

Inhaled corticosteroids are the most potent and consistently effective long-term daily controller medications
for monotherapy of mild persistent asthma (SOR A). They can be successfully used in combination with
intermittent short-acting !-agonists. Oral systemic corticosteroids are recommended for moderate to severe
asthma exacerbations and usually for a very limited time period (SOR A). Daily long-acting !-agonists are
often used in combination with inhaled corticosteroids; however, long-acting !-agonists are not
recommended for use as daily monotherapy for long-term control of persistent asthma, or for intermittent
use (SOR A). Immunomodulators such as omalizumab prevent binding of immunoglobulin E to the
high-affinity receptors on basophils and mast cells. These are used as an additive therapy for patients age
12 years and older with severe persistent asthma, and are not recommended for routine use as
monotherapeutic agents.

Ref: American Lung Association Asthma Clinical Research Centers, Peters SP, Anthonisen N, et al: Randomized comparison
of strategies for reducing treatment in mild persistent asthma. N Engl J Med 2007;356(20):20272039. 2) Expert Panel
Report 3: Guidelines for the Diagnosis and Management of Asthma. National Asthma Education and Prevention Program,
NIH pub no 07-4051, 2007. 3) Elward KS, Pollart SM: Medical therapy for asthma: Updates from the NAEPP guidelines.
Am Fam Physician 2010;82(10):1242-1251.

29
Item 88

ANSWER: A

Celiac sprue (gluten-sensitive enteropathy) classically presents as a malabsorption syndrome associated


with dermatitis herpetiformis. This dermatitis usually appears as excoriated papules, as it is extremely
pruritic. The rash may be misdiagnosed as atypical psoriasis or nonspecific dermatitis. With the
development and use of better diagnostic tests, it now appears that this disorder has been underdiagnosed.
Symptoms include fatigue, weight loss, diarrhea, abdominal pain, anemia, bone pain, aphthous ulcers,
stomatitis, infertility, impotence, alopecia areata, dental enamel defects, seizures, ataxia, and dermatitis.
Serologic tests are now available to aid in confirming the diagnosis of celiac sprue, including IgA
antigliadin antibody, IgG antigliadin antibody, IgA antiendomysial antibody, and IgA antitransglutaminase.
Cystic fibrosis, Crohns disease, and anorexia nervosa can cause weight loss but not dermatitis. Sprue
affects the small intestine; a biopsy of the colon would be inappropriate given this presentation.

Ref: Nelsen DA Jr: Gluten-sensitive enteropathy (celiac disease): More common than you think. Am Fam Physician
2002;66(12):2259-2266. 2) Rossi T: Celiac disease. Adolesc Med Clin 2004;15(1):91-103. 3) Longo DL, Fauci AS,
Kasper DL, et al (eds): Harrisons Principles of Internal Medicine, ed 18. McGraw-Hill, 2012, pp 2469-2471.

Item 89

ANSWER: E

The patient described has a history compatible with gallbladder disease. In a patient with such a typical
history, abdominal ultrasonography is likely to show gallstones and thus provide support for the diagnosis.
Serum bilirubin and AST levels are usually normal except at the time of an attack. A HIDA scan may be
useful if performed during an attack, since the scan assesses the patency of the cystic duct. A plain
abdominal film will detect only 10%15% of cases of cholelithiasis.

Ref: Friedman LS, Brandt LJ (eds): Sleisenger & Fordtrans Gastrointestinal and Liver Disease, ed 9. Saunders, 2010, pp
1105-1113. 2) Townsend CM Jr, Beauchamp RD, Evers BM, et al: Sabiston Textbook of Surgery: The Biological Basis
of Modern Surgical Practice, ed 19. Saunders, 2012, pp 1485-1487.

Item 90

ANSWER: C

In preschool-age children, lower respiratory infections such as pneumonia are most commonly viral
illnesses. Antibiotics may be withheld in young children who are mildly ill and are suspected of having a
viral disease, but antibiotic therapy should be started if their clinical status worsens. In the preschool-age
child with pneumonia, amoxicillin remains the first-line antibiotic of choice, as it provides coverage for
Streptococcus pneumoniae and Haemophilus influenzae, which are the predominant bacterial causes of
pneumonia in this age group. The pharmacokinetics of azithromycin do not preclude its use in children,
but it is not the first-line choice for this patient.

Viruses are also the most frequent cause of pneumonia in the older child, although after the age of 5 years
atypical pneumonia becomes more common. This requires antibiotic coverage for organisms such as
Mycoplasma. For these patients, empiric treatment with a macrolide antibiotic such as azithromycin is
appropriate.

30
Ref: Chang AB, Chang CC, OGrady K, Torzillo PJ: Lower respiratory tract infections. Pediatr Clin North Am
2009;56(6):1303-1321. 2) Ebell MH: Clinical diagnosis of pneumonia in children. Am Fam Physician 2010;82(2):192-193.
3) Bradley JS, Byington CL, Shah SS, et al: The management of community-acquired pneumonia in infants and children
older than 3 months of age: Clinical practice guidelines by the Pediatric Infectious Diseases Society and the Infectious
Diseases Society of America. Clin Infect Dis 2011;53(7):e25-e76.

Item 91

ANSWER: B

ACE inhibitors improve the quality of life and the prognosis for patients with myocarditis-induced dilated
cardiomyopathy, just as they do for other patients with heart failure. Neither antiviral therapy nor
immunosuppression has been shown to improve this type of cardiomyopathy when tested in controlled
trials. NSAIDs actually increase mortality by worsening sodium retention.

Ref: Cooper LT Jr: Myocarditis. N Engl J Med 2009;360(15):1526-1538.

Item 92

ANSWER: A

Treatment of enterohemorrhagic Escherichia coli infection consists of supportive measures only.


Antibiotics are contraindicated because they can trigger the release of Shiga toxins, which may lead to
hemolytic-uremic syndrome in children.

Ref: Mandell GL, Bennett JE, Dolin R (eds): Mandell, Douglas, and Bennetts Principles and Practice of Infectious Diseases,
ed 7. Churchill Livingstone, 2009, p 2825.

Item 93

ANSWER: D

Repair of symptomatic pectus excavatum should be postponed until adolescence, if possible, as this
approach allows for completion of growth and reduces the chance of recurrence. Younger children with
severe cardiopulmonary problems may also be candidates for surgery, but repair at too early an age can
result in improper growth of the chest wall and increases the risk of recurrence of the deformity. Adult
repair is also feasible.

Ref: Jaroszewski D, Notrica D, McMahon L, et al: Current management of pectus excavatum: A review and update of therapy
and treatment recommendations. J Am Board Fam Med 2010;23(2):230-239.

Item 94

ANSWER: D

More than 50% of U.S. hospitals have palliative care programs, which focus on pain and symptom
management. These programs decrease both overall hospital costs and ICU use. Because palliative care
requires a team approach, the number of nonphysician personnel is not decreased. The length of patient
hospital stays is also not decreased (level of evidence 2, SOR A).

Ref: Penrod JD, Deb P, Dellenbaugh C, et al: Hospital-based palliative care consultation: Effects on hospital cost. J Palliat Med
2010;13(8):973-979.

31
Item 95

ANSWER: B

All patients who present with palpitations should be evaluated for a cardiac cause, since this is the etiology
in 43% of cases. A standard 12-lead EKG is the initial test of choice and, along with a history and physical
examination, can determine the cause in 40% of cases. A normal resting EKG does not exclude a cardiac
arrhythmia. Therefore, if the EKG is normal, palpitations of suspected arrhythmic etiology may require
further investigation with ambulatory EKG monitoring.

Echocardiography is helpful in evaluating patients for structural heart disease and should be performed
when the initial history, physical examination, and EKG are unrevealing, or in patients with a history of
cardiac disease or more complex signs and symptoms. This patients family history, along with the fact
that she takes no medications, suggests the possibility of familial long QT syndrome, which often can be
diagnosed from a resting EKG. Diagnosing long QT syndrome is important, since it is associated with an
increased risk of sudden cardiac death. Based on the patients clinical presentation and evaluation, hospital
admission is not warranted prior to obtaining a standard 12-lead EKG.

Ref: Wexler RK, Pleister A, Raman S: Outpatient approach to palpitations. Am Fam Physician 2011;84(1):63-69.

Item 96

ANSWER: D

Medicare has an established definition of what constitutes a homebound patient. The definition includes
patients who require the use of a cane or other supportive device in order to leave the home (not just
occasional use) or require the help of another person to leave the home. Participation in a state-licensed
adult day care program or regularly attending religious services does not disqualify a person from being
considered confined to the home.

Ref: Unwin BK, Tatum PE III: House calls. Am Fam Physician 2011;83(8):925-931.

Item 97

ANSWER: A

Breast milk is considered potentially infectious in patients with HIV infection, along with vaginal
secretions, semen, and blood. Contact with saliva, sweat, urine, or feces does not require postexposure
prophylaxis.

Ref: Tolle MA, Schwarzwald HL: Postexposure prophylaxis against human immunodeficiency virus. Am Fam Physician
2010;82(2):161-166.

32
Item 98

ANSWER: C

Placenta previa is incidentally found on approximately 4% of sonograms performed between 20 and 24


weeks gestation. It often will resolve, and the incidence at term is approximately 0.4%. Symptomatic
placenta previa usually manifests as painless bleeding in the late second or third trimester. It can be painful
bleeding if it is associated with labor or abruption. Most patients with symptomatic placenta previa will
be admitted to the hospital for evaluation. Most neonatal morbidity and mortality associated with placenta
previa is due to the risks associated with preterm birth.

Corticosteroids should be given to women who present with bleeding from a placenta previa between 24
and 34 weeks gestation (SOR A). Tocolytic agents such as magnesium or calcium channel blockers would
be appropriate in patients who have vaginal bleeding associated with preterm contractions.

The goal with tocolytic treatment would be to prolong the pregnancy until fetal lung maturity is achieved.
This patient is not having preterm contractions so tocolytics would not be appropriate. The fetal heart rate
is stable and the mother is hemodynamically stable, so there is no indication for an urgent cesarean section.
Antibiotics do not have a role in the management of symptomatic placenta previa.

Ref: Sakornbut E, Leeman L, Fontaine P: Late pregnancy bleeding. Am Fam Physician 2007;75(8):1199-1206.

Item 99

ANSWER: D

The response to epinephrine may be limited in patients with anaphylaxis who have been taking !-blockers.
Such individuals may have persistent hypotension, bradycardia, and prolonged symptoms. Since glucagon
exerts positive inotropic and chronotropic effects on the heart without depending on catecholamines, an
intravenous bolus followed by an infusion would be a good choice to treat the refractory hypotension.

The use of corticosteroids in this setting is common, but their effectiveness has not been established. Their
benefit is not realized for at least 6 hours, however, so they may aid in the prevention of recurrent
anaphylaxis. Diphenhydramine sometimes provides dramatic symptom relief, but it would not improve the
hypotension. !-Agonists such as albuterol and aminophylline can be used for bronchospasm, but are not
helpful for hypotension.

Ref: Tang AW: A practical guide to anaphylaxis. Am Fam Physician 2003;68(7):1325-1332. 2) Thomas M, Crawford I:
Glucagon infusion in refractory anaphylactic shock in patients on beta-blockers. Emerg Med J 2005;22(4):272-273. 3) Marx
JA (ed): Rosens Emergency Medicine: Concepts and Clinical Practice, ed 7. Mosby Elsevier, 2010, pp 1521-1525.

33
Item 100

ANSWER: D

The U.S. Preventive Services Task Force has concluded that current evidence is insufficient to assess the
balance of benefits and harms of whole-body skin examination by a primary care physician or by patient
skin self-examination for the early detection of cutaneous melanoma, basal cell cancer, or squamous cell
skin cancer in the adult general population. Due to the lack of studies, the evidence is insufficient to
determine whether early detection of skin cancer reduces mortality or morbidity from skin cancer. The
same is true regarding the magnitude of harms from screening for skin cancer. Benefits from screening
are uncertain, even in high-risk patients.

Ref: US Preventive Services Task Force: Screening for skin cancer: Recommendation statement. Am Fam Physician
2010;81(12):1433-1434.

Item 101

ANSWER: D

This patient has several red flags that require complete colon evaluation with endoscopy: age >50, a
change in stool caliber, and obstructive symptoms. Other red flags include heme-positive stools, anemia
consistent with iron deficiency, and rectal bleeding. Malignancy should be eliminated as a possible
diagnosis prior to initiating any treatment. Biofeedback training is used to manage pelvic floor dysfunction
caused by incoordination of pelvic floor muscles during attempted evacuation. Common symptoms include
prolonged or excessive straining, soft stools that are difficult to pass, and rectal discomfort. The other
options are appropriate management strategies once malignancy has been eliminated as a possibility.

Ref: Jamshed N, Lee Z, Olden KW: Diagnostic approach to chronic constipation in adults. Am Fam Physician
2011;84(3):299-306.

Item 102

ANSWER: E

Onychomycosis is a difficult condition to treat successfully. If symptoms are minimal, treatment is often
deferred. Cellulitis of the involved extremity may be related to the onychomycosis and is an accepted
reason to consider eradication treatment. Oral terbinafine is the best treatment in terms of cure rate and
tolerability (SOR A). Significant liver disease is a contraindication.

Itraconazole is less effective and more toxic, and griseofulvin is significantly less effective. Topical
ciclopirox lacquer is also less effective than terbinafine, although it eliminates the risk for systemic
toxicity. Fluconazole is not indicated for onychomycosis.

Ref: Hinojosa JR, Hitchcock K, Rodriguez JE: Clinical inquiries. Which oral antifungal is best for toenail onychomycosis? J
Fam Pract 2007;56(7):581-582.

34
Item 103

ANSWER: A

Providing quality health care to individuals from diverse sociocultural backgrounds requires effective
communication. Low health literacy in almost half of the U.S. population makes communication more
difficult. When a language barrier exists it is better to have a professional interpreter than a family
member, and children should be used as interpreters only in cases of emergency when no other source is
available.

The typical approach to medical care in the United States assumes that patients want to make their own
decisions based on guidance from their health care providers. However, there are cultures in which patient
autonomy is not the norm. There may be a specific authority figure in the family that is regarded as the
decision maker.

Effective communication also involves knowledge of communication styles within various cultures.
Nonverbal communication can be through touch, eye contact, and personal space. For example, there are
cultures in which direct eye contact is avoided, but in other cultures it is considered a sign of respect.

Ref: Blackhall LJ, Murphy ST, Frank G, et al: Ethnicity and attitudes toward patient autonomy. JAMA 1995;274(10):820-825.
2) Green AR, Betancourt JR, Carrillo JE: Integrating social factors into cross-cultural medical education. Acad Med
2002;77(3):193-197. 3) Juckett G: Cross-cultural medicine. Am Fam Physician 2005;72(11):2267-2274. 4) Schyve PM:
Language differences as a barrier to quality and safety in health care: The Joint Commission perspective. J Gen Intern Med
2007;22(Suppl 2):360-361. 5) Schenker Y, Lo B, Ettinger KM, Fernandez A: Navigating language barriers under difficult
circumstances. Ann Intern Med 2008;149(4):264-269.

Item 104

ANSWER: D

Any patient with risk factors for infection who presents with acute joint swelling, pain, erythema, warmth,
and joint immobility should be evaluated for septic arthritis. Risk factors for septic arthritis in this patient
include a cutaneous ulcer and diabetes mellitus. Serum markers such as the WBC count, erythrocyte
sedimentation rate, and C-reactive protein levels are often used to determine the presence of infection or
inflammatory response. However, patients with confirmed septic arthritis may have normal erythrocyte
sedimentation rates and C-reactive protein levels.

Because the clinical presentation of septic arthritis may overlap with that of other causes of acute arthritis,
arthrocentesis is needed to differentiate between the various causes and, in the case of septic arthritis, to
identify the causative agent and determine appropriate therapy. No findings on imaging studies are
pathognomonic for septic arthritis. Antinuclear antibody studies may be indicated later in the course of
management if synovial fluid analysis is not consistent with infection, and if synovial fluid cultures are
negative.

Ref: Horowitz DL, Katzap E, Horowitz S, Barilla-LaBarca ML: Approach to septic arthritis. Am Fam Physician
2011;84(6):653-660.

35
Item 105

ANSWER: D

Norepinephrine is the recommended first-line vasopressor agent to correct hypotension in patients with
sepsis (SOR A). Vasopressor therapy is required to sustain life and maintain perfusion in the face of
life-threatening hypotension, even when hypovolemia has not yet been resolved. Maintaining a mean
arterial pressure of at least 65 mm Hg is critical for tissue perfusion. Dopamine is recommended as an
alternate first-line agent to elevate arterial pressure, but it is less potent compared to norepinephrine.
Dobutamine is recommended as the first-line agent for managing hypotension in cardiogenic shock.
Phenylephrine is recommended as the second-line agent for managing hypotension in patients with septic
shock who also have tachycardia or dysrhythmias. Albumin and epinephrine are not recommended as
first-line agents for managing hypotension in patients with sepsis.

Ref: Dellinger RP, Levy MM, Carlet JM, et al: Surviving Sepsis Campaign: International guidelines for management of severe
sepsis and septic shock: 2008. Crit Care Med 2008;36(1):296-327.

Item 106

ANSWER: B

A prolonged grief reaction, postpartum depression, and iron deficiency anemia could all cause fatigue,
apathy, and decreased libido, but none of these conditions is characterized by failure to lactate and
amenorrhea. This patient most likely has postpartum pituitary necrosis (Sheehans syndrome), a
complication of childbirth in which hemorrhagic shock leads to pituitary necrosis. The syndrome is caused
by the lack of hormonal influence from the anterior pituitary gland on other endocrine glands, resulting
in failure to lactate, breast atrophy, mental apathy, low blood pressure, absence or deficiency of sweating,
loss of secondary hair characteristics and libido, and loss of ovarian function, resulting in amenorrhea.

Ref: Master-Hunter T, Heiman DL: Amenorrhea: Evaluation and treatment. Am Fam Physician 2006;73(8):1374-1382. 2) Fritz
MA, Speroff L: Clinical Gynecologic Endocrinology and Infertility, ed 8. Lippincott Williams & Wilkins, 2011, pp
483-484.

Item 107

ANSWER: D

This patient should have an endometrial biopsy (SOR C). Approximately 7% of postmenopausal women
with benign endometrial cells on a Papanicolaou smear will have significant endometrial pathology. None
of the other options listed evaluate the endometrium for pathology. An asymptomatic premenopausal
woman with benign endometrial cells would not need an endometrial evaluation because underlying
endometrial pathology is rare in this group.

Ref: Apgar BS, Kittendorf AL, Bettcher CM, et al: Update on ASCCP consensus guidelines for abnormal cervical screening
tests and cervical histology. Am Fam Physician 2009;80(2):147-155. 2) Buchanan EM, Weinstein LC, Hillson C:
Endometrial cancer. Am Fam Physician 2009;80(10):1075-1080, 1087-1088.

36
Item 108

ANSWER: D

Repaglinide is a non-sulfonylurea agent that interacts with a different portion of the sulfonylurea receptor
to stimulate insulin secretion. It has a relatively short duration of action, and while it may cause
hypoglycemia this is less likely than with a sulfonylurea agent. Pioglitazone reduces insulin resistance and
has no hypoglycemic effect. Acarbose delays absorption of carbohydrates such as starch, sucrose, and
maltose, but does not affect the absorption of glucose and other monosaccharides. Sitagliptin inhibits the
enzyme responsible for the breakdown of the naturally occurring incretins, and its major advantage is the
absence of side effects. Exenatide stimulates insulin secretion in a glucose-dependent fashion, inhibits
glucagon secretion, slows gastric emptying, and may have a central satiety effect. It does not cause
hypoglycemia when used as monotherapy, but may increase the risk when used with an insulin
secretagogue such as glyburide or glipizide.

Ref: Goldman L, Schafer AI (eds): Goldmans Cecil Medicine, ed 24. Elsevier Saunders, 2011, pp e100-e104.

Item 109

ANSWER: C

Although the registration of death is a state function and the details may vary based on the laws and
regulations of each state, recorded data is contractually shared with the National Vital Statistics System.
To ensure consistency of reporting, the National Center for Health Certificates coordinates collection of
the data points by providing a standard form which most state certificates are modeled from. The standard
format includes a section titled Cause of Death, which is subdivided into two parts. In part 1, the
immediate cause of death is to be recorded on the top line (labeled a). This is defined as the final
disease, injury, or complication directly causing the death, and the directions clearly state that terminal
events such as cardiac arrest, respiratory arrest, or ventricular fibrillation are not to be entered without
showing the etiology.

Additional lines are provided to list conditions leading to the cause of death, including a final line for
entering the disease or injury that initiated the process leading to death. In this case, the proximate cause
of death was the upper gastrointestinal hemorrhage. The source of the bleeding was most likely from
esophageal varices resulting from hepatic cirrhosis, so those conditions should be entered respectively in
the next two lines. The appropriate entry for the final line in part 1 would be chronic alcoholism.

Space is provided in part 2 to include significant conditions contributing to death, such as other chronic
illness and tobacco use.

Ref: Physicians Handbook on Medical Certification of Death. Centers for Disease Control and Prevention, National Center for
Health and Statistics, DHHS pub no (PHS) 2003-1108, 2003, pp 20-23. 2) Hanzlick R: Writing cause of death
statementsBasic principles. National Association of Medical Examiners, 2005.

37
Item 110

ANSWER: A

Patients with a life expectancy of 6 months or less are eligible for the Medicare hospice benefit. This
benefit allows patients to receive hospice care in either the home or hospital setting. In addition to patients
with terminal cancer, patients with end-stage cardiac, pulmonary, and chronic debilitating diseases are
eligible. Approximately two-thirds of patients enrolled in hospice die from noncancer-related diagnoses,
and approximately 60% of Medicare patients are not enrolled in hospice at the time of their death.

Ref: Kaprow MG: Use of hospice care for patients without cancer. Am Fam Physician 2010;82(10):1196.

Item 111

ANSWER: B

Ipratropium bromide is an anticholinergic agent. When nebulized it can sometimes cause inadvertent ocular
effects. Blurred vision and pupil inequality may occur. Ipratropium has been shown to decrease the rate
of hospital admissions in severe asthmatic attacks.

Corticosteroids may elevate glucose levels in diabetic patients. The onset would be more gradual, however.
Adrenergic agents used for acute asthma do not commonly produce adverse ocular effects (level of
evidence 3).

Ref: Lazarus SC: Emergency treatment of asthma. N Engl J Med 2010;363(8):755-764.

Item 112

ANSWER: E

While the U.S. Preventive Services Task Force (USPSTF) recommends against screening for prostate
cancer using prostate-specific antigen testing, other screening methods have not been evaluated in
controlled studies. For men who have smoked, one-time ultrasonography is recommended as a screen for
aortic aneurysm between the ages of 65 and 75. The USPSTF has no recommendation for men who have
never smoked.

The USPSTF states that no evidence supports routine colorectal cancer screening in patients age 7685,
but that there may be some individuals with specific considerations for whom colorectal cancer screening
would be recommended.

At present, there is no evidence to support screening of older adults for dementia, but it is recommended
that all adults be screened for depression when staff support is in place to ensure adequate diagnosis,
treatment, and follow-up. In most instances, the elderly population will present to a primary care provider
with somatic complaints (level of evidence 1b).

Ref: Screening for dementia: Recommendations and Rationale. US Preventive Services Task Force, 2003. 2) Screening for
abdominal aortic aneurysm: Recommendation statement. US Preventive Services Task Force, 2005. 3) Screening for
colorectal cancer: Recommendation statement. US Preventive Services Task Force, 2008. 4) Screening for depression in
adults: Recommendation statement. US Preventive Services Task Force, 2009. 5) Screening for prostate cancer: US
Preventive Services Task Force recommendation statement. US Preventive Services Task Force, 2012.

38
Item 113

ANSWER: C

Amyloidosis is defined as the extracellular deposition of the fibrous protein amyloid at one or more sites.
It may remain undiagnosed for years. Features that should alert the clinician to the diagnosis of primary
amyloidosis include unexplained proteinuria, peripheral neuropathy, enlargement of the tongue,
cardiomegaly, intestinal malabsorption, bilateral carpal tunnel syndrome, or orthostatic hypotension.
Amyloidosis occurs both as a primary idiopathic disorder and in association with other diseases such as
multiple myeloma.

Ref: Klippel JH, Stone JH, Crofford LJ, et al (eds): Primer on the Rheumatic Diseases, ed 13. Springer, 2008, pp 533-541.
2) Longo DL, Fauci AS, Kasper DL, et al (eds): Harrisons Principles of Internal Medicine, ed 18. McGraw-Hill, 2012,
pp 945-950.

Item 114

ANSWER: A

Most uninsured people in the United States are members of a family with at least one working adult. Most
uninsured people who are employed work for small companies or work part-time. Most uninsured people
who work part-time with incomes below the poverty line are not eligible for Medicaid. On average,
uninsured people have less access to care and have poorer health outcomes.

Ref: Tunzi M: The uninsured. Am Fam Physician 2004;69(6):1357-1360. 2) The Henry J. Kaiser Family Foundation: The
Uninsured: A Primer. Key Facts About Americans Without Health Insurance. October 2011, pp 6-8.

Item 115

ANSWER: D

If semen analysis suggests hypogonadism (e.g., severe oligospermia or azoospermia), it is important to


distinguish between primary and secondary causes (SOR C). Evaluation of morning FSH and total serum
testosterone levels can help make this determination. Low testosterone levels correlate with hypogonadism.
High levels of FSH in the presence of low testosterone levels correlate with primary hypogonadism (SOR
B). Low levels of both hormones suggest secondary hypogonadism (SOR B). High testosterone levels are
unlikely to be associated with hypogonadism.

Ref: Jose-Miller AB, Boyden JW, Frey KA: Infertility. Am Fam Physician 2007;75(6):849-856. 2) American Academy of
Family Physicians: Information from your family doctor. Infertility: What you should know. Am Fam Physician
2007;75(6):857-858.

39
Item 116

ANSWER: E

Several medications from different classes are recommended as first-line abortive therapies to treat acute
migraine. Because relatively few trials have directly compared the different medication classes, there are
no definitive algorithms as to which class works best. NSAIDs and acetaminophen/aspirin/caffeine are
recommended as first-line therapies and can be obtained over the counter (SOR A). Triptans are effective
and safe for treatment of acute migraine and are recommended as first-line therapy (SOR A) but require
a prescription. Opiates and barbiturates are not recommended because of their potential for abuse (SOR
C). Acetaminophen alone is not effective, and the same is true of oral corticosteroids.

Ref: Silberstein SD: Practice parameter: Evidence-based guidelines for migraine headache (an evidence-based review): Report
of the Quality Standards Subcommittee of the American Academy of Neurology. Neurology 2000;55(6):754-762. 2)
Tfelt-Hansen P, De Vries P, Saxena PR: Triptans in migraine: A comparative review of pharmacology, pharmacokinetics
and efficacy. Drugs 2000;60(6):1259-1287. 3) Ferrari MD, Roon KI, Lipton RB, Goadsby PJ: Oral triptans (serotonin
5-HT(1B/1D) agonists) in acute migraine treatment: A meta-analysis of 53 trials. Lancet 2001;358(9294):1668-1675. 4)
McCrory DC, Gray RN: Oral sumatriptan for acute migraine. Cochrane Database Syst Rev 2003;(3):CD002915. 5)
Gilmore B, Michael M: Treatment of acute migraine headache. Am Fam Physician 2011;83(3):271-280.

Item 117

ANSWER: C

Proton pump inhibitors (PPIs) have a powerful effect on inhibiting the production of acid in the stomach.
This dramatically reduces symptoms of acid-mediated gastritis, peptic ulcer disease, and gastroesophageal
reflux. However, a significant reduction in stomach acidity may cause unintended consequences involving
processes that are physiologically dependent on low pH in the gastrointestinal tract. These theoretical risks
include decreased levels of vitamin B12, iron, and/or magnesium; decreased bone density; an increase in
gut infections or pneumonia; an increase in gastrointestinal neoplasms; and changes in absorption of other
medications.

The evidence has been conflicting on some of these risks. Currently, consensus is emerging that chronic
use of PPIs increases the risk for pneumonia and gut infections, primarily Clostridium difficile colitis (SOR
B). PPIs may also decrease bone density in subsets of patients. These risks need to be weighed against the
benefits that these medicines provide before prescribing them on a long-term basis.

Ref: Sheen E, Triadafilopoulos G: Adverse effects of long-term proton pump inhibitor therapy. Dig Dis Sci 2011;56(4):931-950.
2) Eom CS, Jeon CY, Lim JW, et al: Use of acid-suppressive drugs and risk of pneumonia: A systematic review and
meta-analysis. CMAJ 2011;183(3):310-319. 3) Kwok CS, Arthur AK, Anibueze CI, et al: Risk of Clostridium difficile
infection with acid suppressing drugs and antibiotics: Meta-analysis. Am J Gastroenterol 2012;107(7):1011-1019.

Item 118

ANSWER: A

The diagnosis of idiopathic scoliosis is based on a coronal plane curvature >10. It is a diagnosis of
exclusion after congenital, neuromuscular, and myopathic diseases and conditions have been ruled out.
Adolescent scoliosis is most common, and occurs in about 2%3% of adolescents. More marked curvature
(>30) occurs in about 0.3% of adolescents, as measured on posterior-anterior and lateral radiographs
using the Cobb method. For mild degrees of curvature there is an even distribution between girls and boys,
but girls have a tenfold greater risk for more severe curvature.

40
Screening for scoliosis in the asymptomatic adolescent is controversial; the U.S. Preventive Services Task
Force recommends against routine screening in its most recent update in 2004 (D recommendation).
However, if idiopathic scoliosis is discovered incidentally or when the adolescent or parent expresses
concern about scoliosis, options for further evaluation and treatment include observation for curvatures
of less than 20 and consideration for bracing and/or surgery for more severe curvatures. The risk of
progression depends on the amount of growth remaining, the magnitude of the curve, and the patients
gender.

Ref: Screening for idiopathic scoliosis in adolescents: Brief evidence update. US Preventive Services Task Force, 2004. 2)
Kliegman RM, Stanton BF, Geme JW III, et al (eds): Nelson Textbook of Pediatrics, ed 19. Elsevier Saunders, 2011, p
2367.

Item 119

ANSWER: C

This patient suffers from chronic low back pain, defined as pain, muscle tension, or stiffness localized
below the costal margin and above the inferior gluteal folds, with or without sciatica, that has persisted for
12 weeks or more and is not attributed to a recognizable pathology. Among all the listed treatment options
for chronic low back pain, only back exercises are given the beneficial recommendation in a systematic
review. Acupuncture and spinal manipulation are in the likely to be beneficial category. Back school and
epidural corticosteroid injections are of unknown effectiveness.

Ref: Chou R: Low back pain (chronic). Am Fam Physician 2011;84(4):437-438.

Item 120

ANSWER: E

Myasthenia gravis is a neuromuscular illness with an underlying immune-related cause. Corticosteroids


and anticholinesterase medications such as oral pyridostigmine can be helpful, but thymectomy may be
appropriate for patients with generalized disease not responding to medication. Thymectomy increases the
remission rate and improves the clinical course.

Ref: Goldman L, Schafer AI (eds): Goldmans Cecil Medicine, ed 24. Elsevier Saunders, 2011, pp 2418-2422.

Item 121

ANSWER: E

According to the 2010 American Heart Association scientific statement regarding acute heart failure
syndrome, levels of natriuretic peptides such as BNP lack the specificity necessary to function as absolute
indicators of acute heart failure syndrome even when they exceed established thresholds for the diagnosis.
BNP levels vary with age, sex, body habitus, renal function, and abruptness of symptom onset.

Elevated BNP levels also have been associated with renal failure (because of reduced clearance),
pulmonary embolism, pulmonary hypertension, and chronic hypoxia. BNP measures are not a substitute
for a comprehensive assessment for signs and symptoms of heart failure, and a laboratory test by itself
cannot be used to determine the diagnosis or management of heart failure. Clinical evaluation and
follow-up are essential to assure proper care for patients with heart failure or any other cardiac problem.

41
Ref: Doust J, Lehman R, Glasziou P: The role of BNP testing in heart failure. Am Fam Physician 2006;74(11):1893-1898. 2)
Weintraub NL, Collins SP, Pang PS, et al: Acute heart failure syndromes: Emergency department presentation, treatment,
and disposition: Current approaches and future aims: A scientific statement from the American Heart Association.
Circulation 2010;122(19):1975-1996.

Item 122

ANSWER: D

Erythema, redness, and pain in the first metatarsal-phalangeal joint are typical symptoms of gout. The uric
acid level can be normal at various times in gout. Acute synovitis is occasionally caused by apatite
deposition disease, but it is usually associated with long-standing osteoarthritis, and the joints involved are
most commonly the shoulder, hip, and knee. Mortons neuroma is an entrapment neuropathy of the
interdigital nerve, usually occurring between the third and fourth toes, not associated with erythema and
redness. Acute arthritis in systemic lupus erythematosus typically involves the wrists, the small joints of
the hands, and the knees.

Ref: Klippel JH, Stone JH, Crofford LJ, et al (eds): Primer on the Rheumatic Diseases, ed 13. Springer, 2008, pp 83-84,
241-249, 267-269, 307-308.

Item 123

ANSWER: A

Studies have shown that the use of intravenous tissue plasminogen activator offers sustained patient benefit
at 6 and 12 months if given within 3 hours of symptom onset (SOR B). All other listed interventions have
not been shown to be efficacious.

Ref: Wahlgren N, Ahmed N, Dvalos A, et al: Thrombolysis with alteplase for acute ischaemic stroke in the Safe
Implementation of Thrombolysis in Stroke-Monitoring Study (SITS-MOST): An observational study. Lancet
2007;369(9558):275-282. 2) Hacke W, Kaste M, Bluhmki E, et al: Thrombolysis with alteplase 3 to 4.5 hours after acute
ischemic stroke. N Engl J Med 2008;359(13):1317-1329. 3) Ferri FF (ed): Ferris Clinical Advisor 2012: 5 Books in 1.
Elsevier Mosby, 2012, pp 969-971.

Item 124

ANSWER: C

Term infants with an onset of jaundice before 24 hours of age, jaundice persisting beyond 3 weeks of age,
or a bilirubin level requiring intensive phototherapy should not be considered healthy, and require further
evaluation. A 2-day-old term infant with a total bilirubin of 10 mg/dL may be followed expectantly.

Ref: Kliegman RM, Stanton BF, Geme JW III, et al (eds): Nelson Textbook of Pediatrics, ed 19. Elsevier Saunders, 2011, pp
603-608.

42
Item 125

ANSWER: B

Vitamin B12 and folate deficiencies typically cause macrocytic anemias. When the serum vitamin B12 level
is borderline low, an elevated methylmalonic acid level can be used to confirm a vitamin B12 deficiency.
An elevated homocysteine level plays a similar role for folate deficiency anemia. Hemolysis can be
associated with an elevated LDH level, and serum ferritin is useful for diagnosing iron deficiency anemia.

Ref: Bross MH, Soch K, Smith-Knuppel T: Anemia in older persons. Am Fam Physician 2010;82(5):480-487.

Item 126

ANSWER: E

Noninferiority trials compare an active control group with a new therapy. The use of a placebo group
would be unethical, since the present therapy is either lifesaving or prevents serious injury. The new
therapy may prove superior to or slightly less effective than the standard therapy.

Ref: Non-inferiority trials. Med Lett Drugs Ther 2011;53:1.

Item 127

ANSWER: A

Much of the psychological distress caused by the diagnosis of mitral valve prolapse is related to a lack of
information and a fear of heart disease, which may be reinforced by the death of a friend or relative. A
clear explanation of mitral valve prolapse, along with printed material, is a powerful aid in relieving the
patients emotional distress. The American Heart Association publishes a helpful booklet about this
condition which can be given to these patients. It is important to avoid reinforcing illness behavior with
unnecessary testing, medications, or referrals to specialists.

Ref: Bonow RO, Mann DL, Zipes DP, Libby P (eds): Braunwalds Heart Disease: A Textbook of Cardiovascular Medicine, ed
9. Elsevier Saunders, 2011, pp 1510-1514. 2) Goldman L, Schafer AI (eds): Goldmans Cecil Medicine, ed 24. Elsevier
Saunders, 2011, pp 460-461.

Item 128

ANSWER: B

All of the treatment options listed may improve the patients depression, but it is unnecessary to add a
second agent until the initial drug is at the maximum recommended dosage. Citalopram can be increased
to a dosage of 40 mg/day.

Ref: Little A: Treatment-resistant depression. Am Fam Physician 2009;80(2):167-172.

43
Item 129

ANSWER: B

Patients with rheumatoid arthritis (RA) are at increased risk for various extra-articular manifestations of
the inflammatory disease, as well as side effects of the medications used to manage it. The leading cause
of death in RA patients is cardiovascular, related to accelerated atherosclerosis (SOR C). Patients with RA
should be screened for cardiovascular risk factors and managed appropriately to lower their risk.

Patients with RA are also at increased risk for other problems that are not leading causes of mortality.
Their risk for infection is increased, which can be related to either the RA itself or to the use of
immunosuppressive agents. Patients with RA also have a twofold increase in their risk for lymphoma. This
is independent of whether or not they are on immunosuppressive agents. Their risk for lung cancer related
to interstitial lung disease is also increased, and smoking increases this risk further.

Ref: Friedewald VE, Ganz P, Kremer JM, et al: AJC editors consensus: Rheumatoid arthritis and atherosclerotic cardiovascular
disease. Am J Cardiol 2010;106(3):442-447. 2) Wasserman AM: Diagnosis and management of rheumatoid arthritis. Am
Fam Physician 2011;84(11):1245-1252.

Item 130

ANSWER: A

This patient likely has meningitis due to Neisseria meningitidis. Ceftriaxone is recommended as first-line
therapy and should not be delayed once the diagnosis is suspected (SOR B). Ciprofloxacin and rifampin
are not recommended as first-line therapy for infected individuals, but are recommended as prophylaxis
for close contacts (SOR B). Doxycycline and amoxicillin are not proven to be effective for treatment or
prophylaxis.

Ref: Bamberger DM: Diagnosis, initial management, and prevention of meningitis. Am Fam Physician 2010;82(12):1491-1498.

Item 131

ANSWER: D

The 2010 American Heart Association guidelines for resuscitation emphasize the importance of chest
compression in CPR. Compression-Airway-Breathing (C-A-B) is now recommended over Airway-
Breathing-Compression (A-B-C). Individualization of this sequence is recommended, however, and in
drowning victims the A-B-C approach is preferred because of the hypoxic nature of the cardiac arrest.

Emergency medical services (EMS) should be activated when the victim is found flaccid and unresponsive.
In certain situations CPR may be performed for up to 2 minutes before calling 911, but a 10-minute
interval is excessive. The Heimlich maneuver and attempts to positionally drain the airway may be harmful
and delay effective CPR.

The reported incidence of cervical spine injury in drowning victims is 0.009%. Attempts at cervical spine
immobilization are not necessary and may impede airway maintenance (SOR C).

44
Ref: Berg RA, Hemphill R, Abella BS, et al: Part 5: Adult basic life support: 2010 American Heart Association Guidelines for
Cardiopulmonary Resuscitation and Emergency Cardiovascular Care. Circulation 2010;122(18 Suppl 3):S685-S705.
Vanden Hoek TL, Morrison LJ, Shuster M, et al: Part 12: Cardiac arrest in special situations: 2010 American Heart
Association Guidelines for Cardiopulmonary Resuscitation and Emergency Cardiovascular Care. Circulation 2010;122(18
Suppl 3):S829-S861.

Item 132

ANSWER: B

Because of increased resistance to fluoroquinolones, ciprofloxacin is no longer recommended for the


treatment of gonorrhea. In addition, there appears to be emerging resistance to cephalosporins, as
evidenced by an increase in the minimum inhibitory concentrations of cephalosporins between 2000 and
2010. Unfortunately, no other well-studied and effective alternative antibiotic treatment regimens are
currently available. It also appears that gonococcal resistance to cefixime might develop before resistance
to ceftriaxone. As a result, in 2011 the Centers for Disease Control and Prevention recommended dual
treatment with ceftriaxone, 250 mg intramuscularly, and azithromycin, 1 g orally, as the most effective
treatment for uncomplicated gonorrhea.

Ref: Centers for Disease Control and Prevention (CDC): Cephalosporin susceptibility among Neisseria gonorrhoeae
isolatesUnited States, 20002010. MMWR Morb Mortal Wkly Rep 2011;60(26):873-877.

Item 133

ANSWER: A

Smoking cessation slows the decline of lung function in COPD. Long-acting !2-agonists, anticholinergic
agents, and inhaled corticosteroids are useful for improving the symptoms of COPD. They improve
exercise tolerance and quality of life, and can reduce the frequency of exacerbations. However, they do
not slow the progression of COPD. Oral corticosteroids, along with antibiotics, are useful in treating acute
exacerbations of COPD, but long-term treatment is not recommended.

Ref: Drugs for chronic obstructive pulmonary disease. Treat Guidel Med Lett 2010;8(99):83-88.

Item 134

ANSWER: E

This patient most likely has a psychogenic tremor, given its abrupt onset, spontaneous remission, changing
tremor characteristics, and extinction with distraction (SOR C). Other characteristics of this case that
suggest psychogenic tremor are the associated stressful life event, the patients employment in a health care
setting, and no evidence of disease by laboratory or radiologic investigations. In addition, the tremor
increases with attention and has been unresponsive to anti-tremor medications (SOR C).

Dystonic tremor is a rare tremor found in less than 1% of the population, and other signs of dystonia, such
as abnormal flexion of the wrists, are usually present. Essential tremor is an action tremor and is usually
postural; however, persons with essential tremor typically have no other neurologic findings. Essential
tremor typically improves with alcohol consumption (2 drinks/day). A cerebellar tremor is usually
associated with other neurologic signs, such as dysmetria (overshoot on finger-to-nose testing), dyssynergia
(abnormal heel-to-shin testing and/or ataxia), and hypotonia. A parkinsonian tremor is most often a resting
tremor, and although it may become less prominent with voluntary movement, it usually does not
spontaneously remit.

45
Ref: Gupta A, Lang AE: Psychogenic movement disorders. Curr Opin Neurol 2009;22(4):430-436. 2) Crawford P, Zimmerman
EE: Differentiation and diagnosis of tremor. Am Fam Physician 2011;83(6):697-702.

Item 135

ANSWER: E

Testosterone replacement therapy can improve many of the effects of hypogonadism. Beneficial effects
include improvements in mood, energy level, sexual functioning, sense of well-being, lean body mass and
muscle strength, erythropoiesis, bone mineral density, and cognition. However, there are also some risks
associated with testosterone use, including an increased risk for prostate cancer, worsening of symptoms
of benign prostatic hyperplasia, liver toxicity and tumor, worsening of sleep apnea and heart failure,
gynecomastia, infertility, and skin diseases. Testosterone replacement therapy is not appropriate in men
who are interested in maintaining fertility, as exogenous testosterone will suppress the
hypothalamic-pituitary-thyroid axis.

Ref: Bassil N, Morley JE: Late-life onset hypogonadism: A review. Clin Geriatr Med 2010;26(2):197-222.

Item 136

ANSWER: E

In addition to their lipid-lowering effects, statins have been shown to have plaque-stabilizing and vascular
anti-inflammatory effects. There is strong clinical evidence that perioperative statin therapy, even when
initiated within days of the procedure and without regard to lipid levels, significantly reduces
cardiovascular risk for patients undergoing vascular surgery (SOR A).

There is strong clinical evidence of benefit in perioperative cardiovascular risk reduction for continuation
of !-blockers before, during, and after vascular surgery in patients who have been on them for at least 4
weeks preoperatively (SOR A). However, in patients who have not been on a !-blocker for at least 14
weeks preoperatively, initiation prior to surgery may be harmful (SOR B).

Traditionally, aspirin has been discontinued prior to surgery for fear of increased surgical bleeding
complications. However, studies have shown that in most cases it is safe to continue low-dose aspirin in
the perioperative period, and doing so reduces cardiovascular complications. This is especially true for
patients with a past history of myocardial infarction or with coronary stents (SOR B).

Preoperative cardiac stress testing is of little value in patients with low or medium cardiovascular risk
status, such as the patient described here. Enoxaparin would not be indicated preoperatively in this patient.

Ref: Holt NF: Perioperative cardiac risk reduction. Am Fam Physician 2012;85(3):239-246.

46
Item 137

ANSWER: C

At every office visit, family physicians should encourage smokers to quit (SOR A). Patients who are ready
to quit may be helped by various pharmacologic treatments. Sustained-release bupropion and nicotine
replacement, especially gum and lozenges, may delay the weight gain often associated with smoking
cessation. Varenicline has a variety of side effects, including an increased risk for cardiovascular events
and a multitude of neuropsychiatric symptoms. It is also the agent most commonly associated with
postsmoking cessation weight gain. Clonidine, considered a second-line, off-label alternative for smoking
cessation, is not associated with weight gain.

Ref: Larzelere MM, Williams DE: Promoting smoking cessation. Am Fam Physician 2012;85(6):591-598.

Item 138

ANSWER: A

For the initial treatment of polymyalgia rheumatica, current evidence suggests using prednisone, 15 mg
daily, or its equivalent, with slow tapering. Relapses are more common with an initial dosage of 10 mg
daily, and slow tapering is associated with fewer relapses. Few patients require a dosage greater than 15
mg/day, which increases the risk for adverse effects.

Ref: Hernndez-Rodriguez J, Cid MC, Lpez-Soto A, et al: Treatment of polymyalgia rheumatica: A systematic review. Arch
Intern Med 2009;169(20):1839-1850. 2) Moon KT: What is the best treatment for polymyalgia rheumatica? Am Fam
Physician 2010;81(6):788.

Item 139

ANSWER: B

This patient has a high probability of pulmonary embolism, given his clinical presentation and recent
hospitalization with bed rest. Multidetector CT is the best initial test to confirm pulmonary embolism in
this situation. D-dimer testing is of limited value in patients with a high probability of pulmonary
embolism. If positive for deep-vein thrombosis, venous ultrasonography of the lower limbs can eliminate
the need for CT or lung scans, but this occurs in only about 10% of patients. Pulmonary angiography is
currently reserved for the rare case in which catheter-based treatment is indicated.

Ref: Agnelli G, Becattini C: Acute pulmonary embolism. N Engl J Med 2010;363(3):266-274.

Item 140

ANSWER: A

Paroxetine has been shown to cause the highest rate of sexual dysfunction among the SSRIs and other
antidepressants. The fewest sexual side effects occur with bupropion.

Ref: Gartlehner G, Hansen RA, Morgan LC, et al: Comparative benefits and harms of second-generation antidepressants for
treating major depressive disorder: An updated meta-analysis. Ann Intern Med 2011;155(11):772-785.

47
Item 141

ANSWER: D

Contrast-induced nephropathy is a concern in patients undergoing contrast studies, and can lead to
decreased renal function. Theoretically, this can cause an increased risk of lactic acidosis in patients taking
metformin. Current guidelines recommend stopping metformin use before imaging procedures that use
contrast, and restarting it 48 hours after the procedure if renal function is unchanged. The other drugs
listed do not carry this risk, although they can cause other problems in hospitalized patients, such as
hypoglycemia, depending on the situation.

Ref: Sawin G, Shaughnessy AF: Glucose control in hospitalized patients. Am Fam Physician 2010;81(9):1121-1124.

Item 142

ANSWER: E

When gynecomastia persists for a prolonged period, the initial glandular hyperplasia is transformed to a
progressive fibrosis and hyalinization. Surgery remains the mainstay of therapy. Medical management is
most useful when the onset is recent or to prevent the initial development of the problem. All the drugs
listed have been tried with varying success in this context, but their clinical usefulness is not established.

Ref: Bembo SA, Carlson HE: Gynecomastia: Its features, and when and how to treat it. Cleve Clin J Med 2004;71(6):511-517.
2) Melmed S, Polonsky KS, Larsen PR, Kronenberg HM (eds): Williams Textbook of Endocrinology, ed 12. Elsevier
Saunders, 2011, pp 719-721.

Item 143

ANSWER: C

This patients presentation is characteristic of vestibular neuronitis, a common condition affecting the
vestibular apparatus. The exact location and cause of the derangement is uncertain, although a viral or
post-viral cause has been postulated.

Benign positional vertigo is characterized by brief attacks of vertigo. Menieres disease is associated with
tinnitus and hearing loss. Migraines have a more gradual onset, and the symptoms of eustachian tube
dysfunction would be milder.

Ref: Marx JA (ed): Rosens Emergency Medicine: Concepts and Clinical Practice, ed 7. Mosby Elsevier, 2010, pp 93-100.

Item 144

ANSWER: C

Many patients are concerned about the risks associated with estrogen replacement therapy, and alternative
options should be addressed. Escitalopram has been shown to be effective for hot flashes in
postmenopausal women. This would be a reasonable choice for this patient, who also likely has depression.

Ref: Freeman EW, Guthrie KA, Caan B, et al: Efficacy of escitalopram for hot flashes in healthy menopausal women: A
randomized controlled trial. JAMA 2011;305(3):267-274.

48
Item 145

ANSWER: B

The range of normal hemoglobin values for healthy individuals varies with age, sex, pregnancy, smoking,
altitude, and ethnicity to an extent that an adjustment derived from population-based studies is appropriate
in each of these situations. A healthy individual should have their lifetime highest hemoglobin
concentrations at full-term birth, exclusive of any later changes from altitude or smoking. Hemoglobin
levels fall during the first 2 months of life and thereafter gradually increase until stabilizing at
approximately 6 months of age.

Studies of ethnic groups in the U.S. demonstrate no significant differences in normal hemoglobin values
among East Asians, Hispanics, Japanese, Native Americans, and non-Hispanic whites; hemoglobin values
of African-Americans tend to be 1 g/dL lower compared to the other U.S. ethnic groups studied. Living
at higher altitudes has a direct effect on hemoglobin levels, and a hemoglobin reference range adjustment
of +1 g/dL at 1000 meters up to +5.5 g/dL at 5000 meters is appropriate. Similarly, smoking increases
hemoglobin levels by 3 g/dL for a 1 pack/day smoker to as high as +7 g/dL for individuals smoking more
than 2 packs/day. The plasma volume expansion that occurs during pregnancy results in a 1.01.5 g/dL
reduction in normal hemoglobin levels.

Ref: Nestel P: Adjusting Hemoglobin Values in Program Surveys. The International Nutritional Anemia Consultative Group,
2002. 2) Jopling J, Henry E, Wiedmeier SE, Christensen RD: Reference ranges for hematocrit and blood hemoglobin
concentration during the neonatal period: Data from a multihospital health care system. Pediatrics 2009;123(2):e333-e337.

Item 146

ANSWER: B

Sudden cardiac death affects 500,000 people in the United States each year, causing more deaths than lung
cancer, breast cancer, and stroke combined. The most common final pathway is ventricular tachycardia
degenerating into ventricular fibrillation. The best predictor of sudden cardiac death is an ejection fraction
!35%. Thus, it is critical for family physicians to evaluate the ejection fraction of patients with heart
disease.

Ref: Turakhia MP: Sudden cardiac death and implantable cardioverter-defibrillators. Am Fam Physician 2010;82(11):1357-1366.

Item 147

ANSWER: A

Clostridium difficile infection is a common cause of diarrhea in hospitalized patients, and recent antibiotic
use is a risk factor for infection. The bacteria can be spread in a hospital setting by contact, and contact
precautions with gown and gloves are indicated in addition to hand washing with soap and water to ensure
removal of spores (SOR A). Hand sanitizer is inadequate, as it does not kill the spores. Respiratory
precautions are not necessary.

Ref: Cohen SH, Gerding DN, Johnson S, et al: Clinical practice guidelines for Clostridium difficile infection in adults: 2010
update by the Society for Healthcare Epidemiology of America (SHEA) and the Infectious Diseases Society of America
(IDSA). Infect Control Hosp Epidemiol 2010;31(5):431-455.

49
Item 148

ANSWER: E

It is estimated that about one-third of patients with bipolar disorder seek medical care within a year of the
onset of symptoms, but that nearly 70% do not receive an accurate diagnosis. The symptoms can often be
subtle and may be attributed to other causes by patients or their loved ones.

A diagnosis of attention-deficit disorder requires that a patients symptoms be present since early
childhood, although they are sometimes not recognized at the time. This patient and his girlfriend have
both acknowledged that he is not his usual self. He presents with increased self-esteem, a decreased need
for sleep, pressured/tangential speech, and irritability, which point to the possibility of a manic or
hypomanic episode. Together these symptoms suggest bipolar disease (SOR C). Patients with full-blown
mania are often out of touch with reality and easy to identify. However, patients with hypomania consider
themselves to have increased well-being and productivity, and will not always seek attention or consider
themselves to have a problem.

Other symptoms that should alert the physician to this diagnosis include substance abuse (present in over
70% of cases) and involvement in other pleasurable but destructive activities such as overspending or
hypersexuality. If substance abuse is present, however, it must be addressed before making a diagnosis
of bipolar disorder. Bipolar disorder is highly genetic, and asking about affected first degree family
members can often assist in making the diagnosis.

Ref: Loganathan M, Lohano K, Roberts RJ, et al: When to suspect bipolar disorder. J Fam Pract 2010;59(12):682-688.

Item 149

ANSWER: A

This patient has asymptomatic hyperthyroidism, which is more common in the elderly. Elevated T4 and
markedly suppressed TSH are diagnostic. Common causes include Graves disease, toxic adenoma,
multinodular goiter, thyroiditis, and use of iodine-containing medications such as amiodarone.
Amiodarone-associated hyperthyroidism may be related to either iodine excess or a toxic effect on the
gland, causing thyroiditis (level of evidence 3). Lithium is associated with hypothyroidism.

Ref: Donangelo I, Braunstein GD: Update on subclinical hyperthyroidism. Am Fam Physician 2011;83(8):933-938.

Item 150

ANSWER: C

Allergic reactions to medications have four primary mechanisms, referred to as Gell and Coombs
classifications. The most frequent forms are type I reactions, which are immediate and mediated through
IgE, and type IV reactions, which are delayed and mediated through T-cell hypersensitization. Severe type
I reactions are often referred to as anaphylaxis and are the most likely to be life threatening with very little
warning. Recognition of the early signs of anaphylaxis is the first step in preventing such catastrophes.

50
Anaphylactic reactions result from a massive release of histamine and start with pruritus around the mouth,
on the scalp, and on the palms and soles; flushing of the face and neck, with rhinitis and conjunctivitis;
angioedema of the oral mucosa, especially of the pharynx and larynx; severe urticaria; dyspnea and
bronchospasm (especially in known asthmatics); and hypotension. A delay in lifesaving therapy during this
phase will result in full shock, hypotension, and death. Type IV reactions usually result in benign, diffuse
erythematous macules on the trunk and proximal extremities, often referred to as a drug rash. These
reactions infrequently become more severe and rarely are life threatening. In severe cases the lesions
become painful and palpable, and may involve blistering, mucositis, and ecchymosis.

Ref: Schnyder B: Approach to the patient with drug allergy. Med Clin North Am 2010;94(4):665-679. 2) Scherer K, Bircher
AJ: Danger signs in drug hypersensitivity. Med Clin North Am 2010;94(4):681-689.

Item 151

ANSWER: C

Vaginal atrophy is a common symptom accompanying menopause. Local application of estrogen is the
most effective treatment (SOR A) and is FDA approved for this indication. Efficacy ranges from 80% to
100%. All formulations are equally effective, so patient preference should drive the choice. Potential
adverse effects of vaginal estrogen include candidal infections, bleeding, burning with application, and
breast pain.

It is not necessary to add an oral progestogen to the local estrogen treatment. Vaginal estrogen does not
lead to endometrial proliferation, so endometrial protection in a patient who still has her uterus is not
necessary.

Ref: Hill DA, Hill SR: Counseling patients about hormone therapy and alternatives for menopausal symptoms. Am Fam
Physician 2010;82(7):801-807. 2) North American Menopause Society: Estrogen and progestogen use in postmenopausal
women: 2010 position statement of The North American Menopause Society. Menopause 2010;17(2):242-255.

Item 152

ANSWER: A

The frequency of head lice infestations has increased in recent years, and resistance to permethrin is now
common. Permethrin is unlikely to be effective in this child since her siblings infestations have failed to
respond to it. Of the other choices, only benzyl alcohol lotion is approved for use in children under 2 years
of age.

Ref: Spinosad (Natroba) topical suspension for head lice. Med Lett Drugs Ther 2011;53:50-51. 2) Kliegman RM, Stanton BF,
Geme JW III, et al (eds): Nelson Textbook of Pediatrics, ed 19. Elsevier Saunders, 2011, pp 2321-2322.

51
Item 153

ANSWER: D

Premenstrual syndrome (PMS) may be diagnosed when recurrent psychological and physical symptoms
occur only during the week prior to menses. The presence of more severe affective and somatic symptoms
that cause significant dysfunction in a patients social and work life is more consistent with premenstrual
dysphoric disorder. Both pyridoxine (vitamin B6), 50100 mg/day, and chasteberry, 20 mg/day, have been
shown in randomized, controlled trials to reduce the symptoms of PMS compared with placebo. No good
evidence supports the use of vitamin E, saffron, St. Johns wort, or soy.

Ref: Biggs WS, Demuth RH: Premenstrual syndrome and premenstrual dysphoric disorder. Am Fam Physician
2011;84(8):918-924.

Item 154

ANSWER: A

The U.S. Preventive Services Task Force and the American Heart Association/American Stroke
Association recommend not performing carotid artery screening with ultrasonography or other screening
tests in patients without neurologic symptoms because the harms outweigh the benefits. In the general
population, screening tests for carotid artery stenosis would result in more false-positive results than
true-positive results. This would lead to surgical procedures that are not indicated or to confirmatory
angiography. As a result of these procedures, some patients would suffer serious harms such as death,
stroke, or myocardial infarction, which outweigh the potential benefit surgical treatment may have in
preventing stroke.

Ref: US Preventive Services Task Force: Screening for carotid artery stenosis: Recommendation statement. Am Fam Physician
2008;77(7):1006-1010.

Item 155

ANSWER: A

This patient has rheumatoid arthritis (RA) by symptoms and physical findings. A positive latex fixation
test for rheumatoid factor is not necessary for the diagnosis. A negative rheumatoid factor does not
exclude RA, and a positive rheumatoid factor is not specific. Rheumatoid factor is found in the serum of
approximately 85% of adult patients with RA; in subjects without RA, the incidence of positive rheumatoid
factor is 1%5% and increases with age.

The ANA test is positive in at least 95% of patients with systemic lupus erythematosus, but in only about
35% of patients with RA. Elevation of the erythrocyte sedimentation rate is seen in many patients with
RA, and the degree of elevation roughly parallels disease activity. At a mean of 6 months after the onset
of Lyme disease, 60% of patients in the United States have brief attacks of asymmetric, oligoarticular
arthritis, primarily in the large joints and especially in the knee.

Ref: Klippel JH, Stone JH, Crofford LJ, et al (eds): Primer on the Rheumatic Diseases, ed 13. Springer, 2008, pp 114-121.

52
Item 156

ANSWER: D

This patient has a disequilibrium type of dizziness. Causes of this include medication side effects,
Parkinsons disease, and peripheral neuropathy. In this patient the history and examination do not indicate
a specific cause. She is on several medications, and one or more could be contributing to her symptoms.
A trial of medication reduction should be considered before ordering additional studies.

Ref: Post RE, Dickerson LM: Dizziness: A diagnostic approach. Am Fam Physician 2010;82(4):361-368, 369.

Item 157

ANSWER: A

Among the causes of food poisoning, Staphylococcus aureus is associated with the shortest incubation
period (16 hours). Most cases are related to contamination of food by infected human carriers. Leaving
food to cool slowly at room temperature allows organisms that produce enterotoxins to multiply. Common
food sources include ham, poultry, potato or egg salad, cream, and pastries.

Campylobacter jejuni infections can be subclinical or symptomatic. Symptoms usually occur within 24
days of exposure to the organism in food or water. A prodrome of fever, headache, and myalgias occurs
2448 hours before the diarrheal symptoms begin. Vomiting is usually not a symptom of the infection.
Clostridium botulinum is usually associated with canned foods. Enterohemorrhagic Escherichia coli
produces a Shiga-like toxin, which kills intestinal epithelial cells. While the symptoms are like those of
food poisoning, they are much more severe and are usually associated with bloody diarrhea (SOR C).

Ref: DuPont HL: Bacterial diarrhea. N Engl J Med 2009;361(16):1560-1569. 2) Longo DL, Fauci AS, Kasper DL, et al (eds):
Harrisons Principles of Internal Medicine, ed 18. McGraw-Hill, 2012, p 1088.

Item 158

ANSWER: D

A single intra-articular injection has little or no effect on glycemic control (SOR A). Soft-tissue or
peritendinous injections can affect blood glucose levels for 521 days, however, and diabetic patients
should closely monitor blood glucose levels for 2 weeks following these injections.

Ref: Stephens MB, Beutler AI, OConnor FG: Musculoskeletal injections: A review of the evidence. Am Fam Physician
2008;78(8):971-976.

53
Item 159

ANSWER: B

Constitutional delay of growth and puberty (CDGP) tends to be inherited. Bone age is delayed, but growth
potential is often normal. LH and FSH are elevated in hypergonadotropic hypogonadism, but this is not
characteristic of CDGP. Thyrotropin is most often elevated in hypothyroidism, which can cause a
secondary delay in growth and puberty. Anosmia is characteristic of Kallmann syndrome, but not CDGP.
Puberty is also delayed in this form of hypogonadotropic hypogonadism. Prolactin is elevated in some
pituitary tumors and by dopamine-blocking agents (SOR C).

Ref: Palmert MR, Dunkel L: Delayed puberty. N Engl J Med 2012;366(5):443-453.

Item 160

ANSWER: D

The combination of elevated LDH and elevated unconjugated bilirubin with otherwise normal liver enzyme
levels suggests hemolysis. Gilberts syndrome would not explain the LDH elevation. Hepatitis is unlikely
with normal transaminase levels.

Ref: Tintinalli JE, Kelen GD, Stapczynski JS (eds): Emergency Medicine: A Comprehensive Study Guide, ed 7. McGraw-Hill,
2011, pp 571-572. 2) Longo DL, Fauci AS, Kasper DL, et al (eds): Harrisons Principles of Internal Medicine, ed 18.
McGraw-Hill, 2012, pp 2527-2531.

Item 161

ANSWER: E

Routine screening of the general population for ovarian cancer is not recommended by any professional
society.

Ref: Clarke-Pearson DL: Screening for ovarian cancer. N Engl J Med 2009;361(2):170-177.

Item 162

ANSWER: A

The preferred anticoagulant for venous thrombosis during pregnancy is low molecular weight heparin.
Unfractionated heparin requires more monitoring and may increase the risk of heparin-induced
thrombocytopenia. Warfarin should not be used during pregnancy but may be used in women who are
breastfeeding (SOR B).

Ref: Dresang LT, Fontaine P, Leeman L, King VJ: Venous thromboembolism during pregnancy. Am Fam Physician
2008;77(12):1709-1716. 2) Longo DL, Fauci AS, Kasper DL, et al (eds): Harrisons Principles of Internal Medicine, ed
18. McGraw-Hill, 2012, pp 57-58.

54
Item 163

ANSWER: D

Eruptive xanthomas like these are associated with elevated triglycerides, obesity, alcohol abuse, diabetes
mellitus, and estrogen or retinoid therapies. The lesions of molluscum contagiosum can be distinguished
from these xanthomas by the characteristic central umbilication of molluscum.

Ref: Holsinger JM, Campbell SM, Witman P: Multiple erythematous-yellow, dome-shaped papules. Am Fam Physician
2010;82(5):517.

Item 164

ANSWER: D

There is no specific therapy for anemia of chronic disease except to manage or treat the underlying
disorder. Iron therapy is of no benefit, but erythropoietin may be helpful in some patients. There is no
available data to suggest that combination therapy or prednisone is beneficial for this disorder.

Ref: Bross MH, Soch K, Smith-Knuppel T: Anemia in older persons. Am Fam Physician 2010;82(5):480-487. 2) Goldman L,
Schafer AI (eds): Goldmans Cecil Medicine, ed 24. Elsevier Saunders, 2011, pp 1042-1043.

Item 165

ANSWER: C

Enuresis alarms should be offered as initial treatment for bed-wetting, based on randomized, controlled
trials and cost-effectiveness evidence. Desmopressin can also be considered if the child or parents do not
want to try an alarm. Restriction of fluids during the day should not be recommended, as it is important
that children have enough to drink. Waking a child and carrying him or her to the toilet has not been
shown to have a long-term effect on bed-wetting. Oxybutynin and imipramine should only be considered
in cases where bed-wetting does not respond to initial treatment.

Ref: Nunes VD, OFlynn N, Evans J, et al: Management of bedwetting in children and young people: Summary of NICE
guidance. BMJ 2010;341:c5399.

Item 166

ANSWER: D

The U.S. Medical Eligibility Criteria for Contraceptive Use were created to guide health care providers
in assessing the safety of contraceptive use for patients with specific conditions. Category 1 includes
conditions for which no restrictions exist for use of the contraceptive method. Category 2 indicates that
the method generally can be used, but careful follow-up may be required. Category 3 is used to classify
conditions for which the method usually is not recommended unless more-preferred methods are not
available or acceptable. Category 4 comprises conditions that represent an unacceptable health risk if the
method is used. For combined hormonal contraceptives, migraine headaches with aura at any age are
classified as category 4 because of the increased risk of ischemic stroke. A family history of breast cancer
is category 1, rheumatoid arthritis treated by immunosuppression is category 2, a BMI "30 kg/m2 is
category 2, and ovarian cancer is category 1.

55
Ref: Centers for Disease Control and Prevention (CDC): US Medical Eligibility Criteria for Contraceptive Use, 2010. MMWR
Recomm Rep 2010;59(RR-4):1-86.

Item 167

ANSWER: B

Acute bronchiolitis is a viral illness most frequently caused by the respiratory syncytial virus. Its peak
incidence occurs at approximately 6 months of age. The illness frequently causes a few days of mild upper
respiratory symptoms, followed by increased coughing and wheezing. Examination often reveals tachypnea
and use of the accessory muscles of respiration, such as intercostal retractions. Acute asthma is uncommon
in the first year of life, and is difficult to diagnose without recurrent episodes or prior respiratory
problems. Croup usually presents with stridor, and pertussis and pneumonia do not usually present with
wheezing.

Ref: Kliegman RM, Stanton BF, Geme JW III, et al (eds): Nelson Textbook of Pediatrics, ed 19. Elsevier Saunders, 2011, pp
1456-1459.

Item 168

ANSWER: D

All antipsychotic agents can prolong ventricular repolarization, leading to a prolonged QT interval, which
can in turn lead to torsades de pointes and sudden cardiac death. Although all antipsychotics can affect
EKG intervals, the agents with the greatest propensity to prolong QTc are thioridazine, pimozide,
droperidol, and ziprasidone. The incidence of sudden cardiac death among patients taking antipsychotics
is about twice that of the general population.

Ref: Muench J, Hamer AM: Adverse effects of antipsychotic medications. Am Fam Physician 2010;81(5):617-622.

Item 169

ANSWER: E

Stress fractures are caused by repetitive loading that exceeds the bones ability to heal. They occur more
commonly in female athletes. Patients should be evaluated for risk factors such as eating disorders,
menstrual irregularities, and chronic medical conditions. Lower-extremity alignment, gait, and strength
should also be evaluated. High-risk fractures such as those of the femoral neck, anterior cortex of the tibia,
or proximal fifth metatarsal should be referred to an orthopedist, as there is a high likelihood of
fracture-related complications.

Ref: Sanderlin BW, Raspa RF: Common stress fractures. Am Fam Physician 2003;68(8):1527-1532. 2) Joy EA, Van Hala S,
Cooper L: Health-related concerns of the female athlete: A lifespan approach. Am Fam Physician 2009;79(6):489-495.

Item 170

ANSWER: A

Cardioselective !-blockers, such as metoprolol, should not be withheld from patients with COPD.
Metoprolol could be started at a low dosage in this patient. Nadolol, timolol, and sotalol are not
cardioselective.

56
Ref: Salpeter S, Ormiston T, Salpeter E: Cardioselective beta-blockers for chronic obstructive pulmonary disease. Cochrane
Database Syst Rev 2005;(4):CD003566. 2) Navas EV, Taylor DO: Q: Can patients with COPD or asthma take a
beta-blocker? Cleve Clin J Med 2010;77(8):498-499.

Item 171

ANSWER: E

While most patients with heart failure should be treated with an ACE inhibitor and a diuretic, a subset of
patients with heart failure present with only fatigue or mild dyspnea on exertion, and no evidence of
volume overload. Since ACE inhibitors alone appear to prevent or slow the development of heart failure
in patients with asymptomatic left ventricular dysfunction, it is reasonable to start an ACE inhibitor such
as lisinopril in patients with very mild symptoms and observe to see if the symptoms resolve. The other
medications listed are useful in the treatment of heart failure but would not be appropriate as initial
treatment in the patient described.

Ref: Hunt SA, Abraham WT, Chin MH, et al: 2009 focused update incorporated into the ACC/AHA 2005 Guidelines for the
Diagnosis and Management of Heart Failure in Adults: A report of the American College of Cardiology
Foundation/American Heart Association Task Force on Practice Guidelines: Developed in collaboration with the
International Society for Heart and Lung Transplantation. Circulation 2009;119(14):e391-479. 2) Bonow RO, Mann DL,
Zipes DP, Libby P (eds): Braunwalds Heart Disease: A Textbook of Cardiovascular Medicine, ed 9. Elsevier Saunders,
2011, pp 549-564.

Item 172

ANSWER: E

Pneumococcal 13-valent vaccine produces a satisfactory immune response in 1-year-old children, while
polyvalent vaccine does not cause a good antibody response in children under the age of 2 years. Neither
vaccine is available orally, and cost is not a factor. The 13-valent vaccine requires multiple doses. The
vaccine cannot be combined with MMR in a single injection, but can be administered concurrently with
routine childhood immunizations at a separate site using a separate syringe.

Ref: 23-Valent pneumococcal polysaccharide vaccine: WHO position paper. Wkly Epidemiol Rec 2008;83(42):373-384. 2)
Nuorti JP, Whitney CG; Centers for Disease Control and Prevention (CDC): Prevention of pneumococcal disease among
infants and childrenUse of 13-valent pneumococcal conjugate vaccine and 23-valent pneumococcal polysaccharide vaccine:
Recommendations of the Advisory Committee on Immunization Practices (ACIP). MMWR Recomm Rep 2010;59(RR-
11):1-18.

Item 173

ANSWER: D

This patient most likely has Charcot foot (neuropathic arthropathy). This is an uncommon condition, most
often found in patients with at least a 10-year history of type 1 or 2 diabetes. Patients have peripheral
neuropathy and typically present with painless swelling of a foot. About 50% of patients can recall minor
trauma preceding the onset of symptoms. Early radiologic findings may be normal, but as the condition
progresses plain films may reveal bony fractures, fragmentation, and bone destruction. On examination
the foot is either stiff or hypermobile, warm, erythematous, and edematous. Crepitus can be felt in later
stages of the disease. Laboratory results can be normal or show increases in the WBC count. Any patient
with long-standing diabetes mellitus and a warm, red, swollen foot should be presumed to have Charcot
foot and have the foot immobilized immediately to protect it from stress and to prevent further destruction
and disability (SOR C).

57
The differential diagnosis of Charcot foot includes osteomyelitis, which can be difficult to distinguish.
Patients with osteomyelitis can have skin ulcers that may be probed to the bone. They will often be febrile,
with higher erythrocyte sedimentation rates and WBC counts than patients with Charcot foot. Aspiration
of the joint fluid is helpful in distinguishing between the two conditions.

Aspiration can also help distinguish between acute gout and Charcot foot. Patients may have pain and
stiffness in their foot with either condition. Patients with gout may have an elevated uric acid level, and
plain films may reveal tophi.

Patients with osteoarthritis are unlikely to have warm, erythematous skin overlying the involved joint.
Symptoms often have an insidious onset rather than the more acute onset of Charcot foot.

Ref: Sommer TC, Lee TH: Charcot foot: The diagnostic dilemma. Am Fam Physician 2001;64(9):1591-1598. 2) Kronenberg
HM, Melmed S, Polonsky K, et al: Williams Textbook of Endocrinology, ed 11. Saunders, 2008, pp 1482-1483.

Item 174

ANSWER: A

General recommendations regarding prevention of recurrent nephrolithiasis include increasing fluid intake
up to 2 L of water daily (SOR B); greater volumes may lead to electrolyte disturbances and are not
recommended. More specific dietary recommendations depend on the stone type. If the stone is not
recovered, the type may be inferred from a 24-hour urine collection for calcium, phosphorus, magnesium,
uric acid, and oxalate.

Approximately 60% of all stones in adults are calcium oxalate. Uric acid stones account for up to 17% of
stones and, like cystine stones, form in acidic urine. Alkalinization of the urine to a pH of 6.57.0 may
reduce stone formation in patients with these types of stones. This includes a diet with plenty of fruits and
vegetables, and limiting acid-producing foods such as meat, grains, dairy products, and legumes. Drinking
mineral water, which is relatively alkaline with a pH of 7.07.5, is also recommended. Restriction of
dietary oxalates has not been shown to be effective in reducing stone formation in most patients.

Acidification of the urine to a pH !7.0 is recommended for patients with the less common calcium
phosphate and struvite stones. This can be accomplished by consumption of at least 16 oz of cranberry
juice per day, or by taking betaine, 650 mg three times daily.

Ref: Frassetto L, Kohlstadt I: Treatment and prevention of kidney stones: An update. Am Fam Physician 2011;84(11):1234-1242.

Item 175

ANSWER: E

The statement, It is time to consider withdrawal of care, can make patients think that the physician no
longer wants to care for them. It would be better to ask, Do you think it is time for us to consider a
different type of treatment that focuses on your symptoms? Telling the individual that you want to provide
intense coordinated care with a team that will treat symptoms and maintain comfort is a way of involving
hospice without making the patient feel hopeless.

Ref: Ngo-Metzger Q, August KJ, Srinivasan M, et al: End-of-life care: Guidelines for patient-centered communication. Am Fam
Physician 2008;77(2):167-174.

58
Item 176

ANSWER: C

De Quervains tenosynovitis is a tenosynovitis of the first dorsal compartment of the wrist, specifically a
chronic inflammation of the extensor pollicis brevis and abductor pollicis longus tendons. It is a fairly
common cause of pain in the distal forearm. The diagnosis is made by physical examination. Pain,
tenderness, and occasionally swelling are present on the radial side of the wrist. The pain is exacerbated
by passive wrist ulnar deviation while the thumb is flexed and the fingers curled around it.

These symptoms are not typical for rheumatoid disease, and cervical radiculopathy would not cause radial
tenderness. Carpal joint arthritis would be more distal. The sesamoid bones are common and of no clinical
significance. Carpal tunnel syndrome is caused by an entrapment neuropathy of the median nerve as it
traverses the carpal tunnel. Symptoms typically include pain and paresthesias in the hand. Numbness
occurs in the first two fingers, in the distribution of the median nerve. Thenar muscle weakness is a later
sign.
Ref: Frontera WR, Silver JK, Rizzo TD Jr (eds): Essentials of Physical Medicine and Rehabilitation, ed 2. Saunders Elsevier,
2008, pp 129-131.

Item 177

ANSWER: E

This patients examination is consistent with female pattern hair loss. Women with female pattern hair loss
who also have a history of abnormal menses, infertility, cystic acne, and hirsutism should have an
evaluation for hyperandrogenism. Minoxidil 2% topically is the only treatment approved by the FDA for
treating female pattern hair loss in women over 18, but a hyperandrogenic state may limit the response to
minoxidil. If the hyperandrogenism evaluation is normal, spironolactone, 100200 mg daily, may slow the
rate of hair loss. Approximately 90% of such women report a modest decrease in hair loss with this
treatment.
Ref: Mounsey AL, Reed SW: Diagnosing and treating hair loss. Am Fam Physician 2009;80(4):356-362.

Item 178

ANSWER: D

Tourettes syndrome is often associated with psychiatric comorbidities, mainly


attention-deficit/hyperactivity disorder and obsessive-compulsive disorder. The other conditions listed are
not associated with Tourettes syndrome.
Ref: Kenney C, Kuo SH, Jimenez-Shahed J: Tourettes syndrome. Am Fam Physician 2008;77(5):651-658.

Item 179

ANSWER: B

Plantar fasciitis affects more than 1 million people in the United States each year. Risk factors include
excessive pronation, running, obesity, and prolonged standing. Patients often have pain when they get out
of bed and take their first steps in the morning, or after prolonged sitting. Palpation usually causes pain
in the medial plantar calcaneal region. The pain is described as sharp and stabbing.

59
A sudden onset of ecchymosis and heel pain is more consistent with a diagnosis of plantar fascia rupture.
Pain in the region of the posterior medial ankle is more consistent with posterior tibial tendinitis. Burning
pain in the medial plantar region is more consistent with medial calcaneal and abductor digiti quinti nerve
entrapment.

Ref: Goff JD, Crawford R: Diagnosis and treatment of plantar fasciitis. Am Fam Physician 2011;84(6):676-682.

Item 180

ANSWER: D

Seborrheic dermatitis is one of the more common skin conditions affecting infants within the first few
months of life. The characteristic reddish, waxy rash most commonly involves the scalp (cradle cap), but
can also appear on the face, ears, neck, skin folds, and diaper area. While the rash is similar to that of
atopic dermatitis, seborrheic dermatitis is not associated with pruritus. Although the rash can appear
alarming to parents, reassurance that the condition can be expected to resolve within a few months is the
most appropriate management.

Ref: OConnor NR, McLaughlin MR, Ham P: Newborn skin: Part I. Common rashes. Am Fam Physician 2008;77(1):47-52.

Item 181

ANSWER: E

Somatization disorder usually begins in the teens or twenties and is characterized by multiple unexplained
physical symptoms, insistence on surgical procedures, and an imprecise or inaccurate medical history.
Abuse of alcohol, narcotics, or other drugs is also common in these patients.

Hypochondriacs are overly concerned with bodily functions, and can often provide accurate, extensive,
and detailed medical histories. Malingering is an intentional pretense of illness to obtain personal gain.
Patients with panic disorder have episodes of intense, short-lived attacks of cardiovascular, neurologic,
or gastrointestinal symptoms. Generalized anxiety disorder is characterized by unrealistic worry about life
circumstances accompanied by symptoms of motor tension, autonomic hyperactivity, or vigilance and
scanning.

Ref: American Psychiatric Association: Diagnostic and Statistical Manual of Mental Disorders, ed 4. American Psychiatric
Association, 1994, pp 448-450. 2) Sadock BJ, Sadock VA, Ruiz P (eds): Kaplan & Sadocks Comprehensive Textbook of
Psychiatry, ed 9. Lippincott Williams & Wilkins, 2009, pp 1927-1935.

Item 182

ANSWER: D

This patient has mild acute croup, also known as laryngotracheobronchitis. She has no signs of pneumonia,
epiglottitis, or bacterial tracheitis. Acute croup is almost always viral in nature, with parainfluenza viruses
being the most common etiologic agents. A single dose of oral dexamethasone has been shown to benefit
children with even mild croup, presumably by decreasing edema of the laryngeal mucosa. Inhaled
epinephrine is helpful in severe croup with signs of respiratory distress. Inhaled albuterol is used to treat
asthma. Oseltamivir would be appropriate treatment for influenza, and azithromycin for bacterial
pneumonia.

Ref: Zoorob R, Sidani M, Murray J: Croup: An overview. Am Fam Physician 2011;83(9):1067-1073.

60
Item 183

ANSWER: C

Chronic radiation proctitis develops months to years after radiation exposure and is characterized by pain
with defecation, along with diarrhea and sometimes rectal bleeding. On colonoscopy, the mucosa is pale
and friable with telangiectases which are sometimes large, multiple, and serpiginous.

Ref: Berkey FJ: Managing the adverse effects of radiation therapy. Am Fam Physician 2010;82(4):381-388, 394.

Item 184

ANSWER: B

It is recommended that asthmatics, once stabilized, be taken off long-acting !-agonists and maintained on
an inhaled corticosteroid such as fluticasone. It is not recommended to change from a combination inhaled
corticosteroid/long-acting !-agonist to a long-acting anticholinergic agent. Montelukast can be used for
maintenance, but inhaled corticosteroids are preferable.

Ref: Ducharme F, Schwartz Z, Hicks G, Kakuma R: Addition of anti-leukotriene agents to inhaled corticosteroids for chronic
asthma. Cochrane Database Syst Rev 2004;(2):CD003133. 2) Chowdhury BA, Dal Pan G: The FDA and safe use of
long-acting beta-agonists in the treatment of asthma. N Engl J Med 2010;362(13):1169-1171.

Item 185

ANSWER: A

This patient almost certainly has sarcoidosis. The diagnosis is supported by a compatible clinical and
radiographic presentation, and histologic evidence of noncaseating granulomas on a biopsy (without
organisms or particles). In patients who present with Lfgren syndrome (erythema nodosum, hilar
adenopathy, and polyarthralgias), a probable diagnosis of sarcoidosis can be made without a biopsy. In
all other cases a biopsy should be performed on the most accessible organ, such as the skin or peripheral
lymph nodes.

While the thorax is the most common site of disease, skin involvement occurs in at least 30% of patients
and is often missed. This patient was diagnosed with psoriasis, which may have been another manifestation
of sarcoidosis. Cutaneous sarcoidosis presents as single foci or crops, and is often attributed to other
causes, perhaps because of its highly variable manifestations such as macular-papular, nodular,
psoriatic-like, and hypomelanotic lesions. Careful skin examination is warranted because biopsy of a
sarcoidal lesion has a high diagnostic yield.

Ref: Iannuzzi MC, Fontana JR: Sarcoidosis: Clinical presentation, immunopathogenesis, and therapeutics. JAMA
2011;305(4):391-399.

61
Item 186

ANSWER: C

Problems with gait and balance increase in frequency with advancing age and are the result of a variety
of individual or combined disease processes. Findings may be subtle initially, making it difficult to make
an accurate diagnosis, and knowing the relative frequencies of primary causes may be useful for
management. A cautious gait (broadened base, slight forward leaning of the trunk, and reduced arm swing)
may be the first manifestation of many diseases, or it may just be somewhat physiologic if not excessive.

In the past, a problematic gait abnormality in an elderly person was generally termed a senile gait if there
was no clear diagnosis; it is more accurate, however, to describe this as an undifferentiated gait problem
secondary to subclinical disease. From the long list of potential causes, arthritic joint disease is by far the
most likely to be seen in the family physicians office, accounting for more than 40% of total cases. It most
frequently causes an antalgic gait characterized by a reduced range of motion. The patient favors affected
joints by limping or taking short, slow steps.

Ref: Salzman B: Gait and balance disorders in older adults. Am Fam Physician 2010;82(1):61-68. 2) Lam R: Office management
of gait disorders in the elderly. Can Fam Physician 2011;57(7):765-770.

Item 187

ANSWER: A

Pregnancy with an intrauterine device in place is rare but does occur. Removal of an in situ intrauterine
device in early pregnancy reduces the risks of spontaneous abortion, preterm labor, and sepsis, so gentle
removal should be accomplished as soon as the pregnancy becomes known.

Ref: Cunningham FG, Leveno KJ, Bloom SL, et al (eds): Williams Obstetrics, ed 23. McGraw-Hill Medical, 2010, p 686.

Item 188

ANSWER: C

This patient meets the criteria for antisocial personality disorder, including age over 18, evidence of
conduct disorder in childhood, a pattern of irresponsible and antisocial behavior since age 15, and absence
of schizophrenia or manic episodes. Although the patient has some features of borderline personality
disorder, such as unstable relationships, the persistently aggressive nature and lack of remorse are much
more typical of antisocial personality. While the boasting quality of the patient might appear somewhat
grandiose, there are no other features to suggest mania. Abused child reaction formation is not a
recognized diagnosis in the Diagnostic and Statistical Manual of Mental Disorders. Schizotypal personality
disorder is not usually associated with such pervasive antisocial behavior and violence.

Ref: American Psychiatric Association: Diagnostic and Statistical Manual of Mental Disorders, ed 4. American Psychiatric
Association, 1994, pp 645-650. 2) Sadock BJ, Sadock VA, Ruiz P (eds): Kaplan & Sadocks Comprehensive Textbook of
Psychiatry, ed 9. Lippincott Williams & Wilkins, 2009, pp 2220-2221.

62
Item 189

ANSWER: C

Although Medicare does not pay for an annual physical, it does provide for annual preventive screening
services, including a complete health history and an array of screening measures for depression, fall risk,
cognitive problems, and other challenges. The physical examination conducted as part of the annual
wellness visit includes measurement of blood pressure and weight, a vision check, and hearing evaluation,
as well as additional elements depending on the individuals health risks.

While questioning the patient or caregiver regarding perceived hearing difficulties may suffice when
screening for hearing loss, screening for vision loss requires use of a standard screening tool.
Documentation of visual acuity by use of the Snellen chart is an accepted means of screening for visual
acuity in the primary care setting (SOR A). Vision screening will not pick up age-related macular
degeneration or cataracts, however.

Ref: Screening for visual acuity in older adults: Recommendation statement. US Preventive Services Task Force, 2009. 2)
Hughes C: Medicare annual wellness visits made easier. Fam Pract Manag 2011;18(4):10-14.

Item 190

ANSWER: A

Depot medroxyprogesterone acetate and the combination contraceptive vaginal ring are the most effective
hormonal contraceptives for obese women because they do not appear to be affected by body weight.
Women using the combination contraceptive patch who weight "90 kg may experience decreased
contraceptive efficacy. Obese women using oral contraceptives may also have an increased risk of
pregnancy.

Ref: Gordon L, Thakur N, Atlas M, Januchowski R: What hormonal contraception is most effective for obese women? J Fam
Pract 2007;56(6):471-473.

Item 191

ANSWER: E

There are several possible causes of secondary hypertension in young adults age 1939, including
coarctation of the aorta, thyroid dysfunction, renal parenchymal disease, and fibromuscular dysplasia.
Fibromuscular dysplasia is more common in females, and has a predilection for causing stenosis of the
renal arteries. The diagnosis can be made using MRI with gadolinium contrast media, or with CT
angiography.

Middle-aged adults (age 4064) are more likely to have primary aldosteronism (evaluated with an
aldosterone/renin ratio), sleep apnea, pheochromocytoma (associated with elevated metanephrines), or
Cushings syndrome (elevated 24-hour urinary cortisol). The patient described has no signs or symptoms
of any of these problems.

Ref: Viera AJ, Neutze DM: Diagnosis of secondary hypertension: An age-based approach. Am Fam Physician
2010;82(12):1471-1478.

63
Item 192

ANSWER: D

The female athlete triad is a relatively common condition in athletes, and is characterized by amenorrhea,
disordered eating, and osteoporosis. It is more common in sports that promote lean body mass. Female
athletes should be screened for the disorder during their preparticipation evaluations. Individuals who
present with one or more components of the triad should be evaluated for the other components. This
patient evidences disordered eating (low BMI for age) and secondary amenorrhea, and should be screened
for osteoporosis using a DXA scan. The International Society for Clinical Densitometry recommends using
the Z-score, rather than the T-score, when screening children or premenopausal women. The T-score is
based on a comparison to a young adult at peak bone density, whereas the Z-score uses a comparison to
persons of the same age as the patient. A Z-score less than 2.0 indicates osteoporosis. The American
College of Sports Medicine defines low bone density as a Z score of 1.0 to 2.0.

An EKG is not required in this patient since she has normal vital signs. Pelvic ultrasonography is not
necessary unless an abnormal finding is identified on a pelvic examination. Abdominopelvic CT would be
inappropriate given the patients age and lack of abdominopelvic symptoms such as pain or a mass. A
nuclear bone scan likewise is not recommended, as it is not used to diagnose osteoporosis (SOR C).
Ref: Joy EA, Van Hala S, Cooper L: Health-related concerns of the female athlete: A lifespan approach. Am Fam Physician
2009;79(6):489-495. 2) Mendelsohn FA, Warren MP: Anorexia, bulimia, and the female athlete triad: Evaluation and
management. Endocrinol Metab Clin North Am 2010;39(1):155-167.

Item 193

ANSWER: C

To address and minimize perioperative neurologic morbidity in patients undergoing coronary artery bypass
grafting (CABG), individualized surgical management strategies are now recommended. These address
patient risk factors for postoperative stroke, such as carotid stenosis, hypertension, older age, a past history
of stroke, small-vessel disease in the brain, and diabetes mellitus. Because concomitant carotid disease is
often associated with CAD, pre-CABG carotid Doppler ultrasonography is routinely recommended.

Several approaches have been evaluated for decreasing the risk associated with carotid stenosis.
Performing both carotid and CABG surgery at the same time increases stroke risk, and no studies have
compared doing one before the other. While the use of statins has increased in patients with systemic
atherosclerosis, the roles of both postoperative statins and !-blockers are still controversial. The only
treatment that has been shown to reduce postoperative cerebrovascular events is the use of aspirin in the
first 48 hours after surgery.
Ref: Selnes O, Gottesman RF, Grega MA, et al: Cognitive and neurologic outcomes after coronary-artery bypass surgery. N
Engl J Med 2012;366(3):250-257.

Item 194

ANSWER: D

Based on a review of randomized, controlled trials, screening mammography reduces breast cancer
mortality for women between 40 and 49 years of age (SOR A). The absolute risk reduction is less for
women in this age group than for older women, however. For technical reasons, and because of greater
breast density, the false-positive and false-negative rates may be higher in this group than in older women.
Radiation risk is also greater in younger women.

64
Ref: Qaseem A, Snow V, Sherif K, et al: Screening mammography for women 40 to 49 years of age: A clinical practice
guideline from the American College of Physicians. Ann Intern Med 2007;146(7):511-515. 2) Krishnaiah PB, Nunes NL,
Safranek S: FPINs clinical inquiries: Screening mammography for reducing breast cancer mortality. Am Fam Physician
2012;85(2):176-183.

Item 195

ANSWER: D

Pneumococcal vaccine is usually given only once to individuals "65 years of age. A repeat dose may be
given 5 years later for those at higher risk. Immunization is also recommended for younger persons with
chronic medical problems, such as heart disease, diabetes mellitus, renal failure, and sickle cell anemia,
as well as those who have undergone splenectomy or who work or live with high-risk persons.

Ref: Centers for Disease Control and Prevention (CDC); Advisory Committee on Immunization Practices: Updated
recommendations for prevention of invasive pneumococcal disease among adults using the 23-valent pneumococcal
polysaccharide vaccine (PPSV23). MMWR Morb Mortal Wkly Rep 2010;59(34):1102-1106.

Item 196

ANSWER: A

All of the diagnoses listed may cause elevations of serum calcium, but malignancy, sarcoidosis, Paget's
disease, and hypervitaminosis D are all associated with suppressed levels of parathyroid hormone. While
the parathyroid levels in this patient are within the normal range, they are inappropriately high for the level
of serum calcium and suggest hyperparathyroidism, the most common cause of hypercalcemia in this age
group.

Ref: Carroll MF, Schade DS: A practical approach to hypercalcemia. Am Fam Physician 2003;67(9):1959-1966. 2) Longo DL,
Fauci AS, Kasper DL, et al (eds): Harrisons Principles of Internal Medicine, ed 18. McGraw-Hill, 2012, pp 3096-3111.

Item 197

ANSWER: E

Research has consistently confirmed that stimulant medications are the most efficacious first-line treatment
for children with attention-deficit/hyperactivity disorder (ADHD) (SOR A). No research supports the
notion that the use of a stimulant in ADHD patients will promote addiction. To the contrary, some evidence
suggests that ADHD patients who take stimulant medication have lower rates of drug abuse than those who
do not. Diversion and misuse of prescription stimulants is a growing concern, however, and the use of a
long-acting stimulant can decrease the chances for diversion.

There are a number of well-supported behavioral interventions for ADHD. Most behavioral approaches
focus on rewarding desired behavior and applying consequences for unwanted behavior to gradually
reshape the childs thinking and actions. Interventions that help reinforce parental involvement include
support groups, which connect parents who have children with similar problems, and parenting skills
training, which gives parents techniques and tools for managing their childs behavior. Psychotherapy and
cognitive-behavioral therapy have little or no documented effectiveness for the treatment of ADHD.

65
A multicenter, randomized study comparing the effectiveness of multimodal treatment (combined
behavioral interventions and pharmacotherapy) with either treatment alone showed that combination
treatment and pharmacotherapy alone yielded similar results and each was more effective than behavioral
treatment alone or standard care in reducing core ADHD symptoms. A tool kit has been developed by the
American Academy of Pediatrics and the National Initiative for Childrens Healthcare Quality to help
physicians improve the management of ADHD. While the second edition is only available in print form,
the first edition can be downloaded free at http://www.nichq.org/adhd_tools.html.

Ref: Rader R, McCauley L, Callen EC: Current strategies in the diagnosis and treatment of childhood
attention-deficit/hyperactivity disorder. Am Fam Physician 2009;79(8):657-665. 2) Antshel KM, Hargrave TM, Simonescu
M, et al: Advances in understanding and treating ADHD. BMC Med 2011;9:72.

Item 198

ANSWER: C

The low TSH level suggests hyperthyroidism. If TSH is <0.5 $U/mL, the immediate next step is to
measure free T3 and free T4 levels (SOR C), which are elevated in hyperthyroidism and normal in
subclinical hyperthyroidism. If levels of free T3 and free T4 are elevated and the underlying cause of
hyperthyroidism is unknown, then it is advisable to order a 24-hour radioactive iodine (RAI) uptake test.
With Graves disease, RAI uptake is increased and diffuse, whereas with toxic multinodular goiter it is
increased and nodular (SOR A). If RAI uptake is low, subacute thyroiditis should be suspected and could
be confirmed by measuring levels of thyroglobulin (SOR A). Levothyroxine is not indicated, as it is used
to treat hypothyroidism. Fasting does not significantly affect TSH levels.

Ref: Baskin HJ, Cobin RH, Duick DS, et al: American Association of Clinical Endocrinologists medical guidelines for clinical
practice for the evaluation and treatment of hyperthyroidism and hypothyroidism. Endocr Pract 2002;8(6):457-469. 2)
Boelaert K, Torlinska B, Holder RL, Franklyn JA: Older subjects with hyperthyroidism present with a paucity of symptoms
and signs: A large cross-sectional study. J Clin Endocrinol Metab 2010;95(6):2715-2726. 3) Ghandour A, Reust C:
Hyperthyroidism: A stepwise approach to management. J Fam Pract 2011;60(7):388-395.

Item 199

ANSWER: C

The patient has symptoms consistent with chronic bacterial prostatitis: irritative voiding symptoms;
testicular, perineal, and low back pain; recurrent urinary tract infections; and distal penile pain. His
symptoms have also been present for more than 3 months. Because chronic bacterial prostatitis is a
bacterial infection, an appropriate antibiotic with good tissue penetration in the prostate should be selected.
Fluoroquinolones have the best tissue concentration and are recommended as first-line agents.

Penicillin derivatives, commonly used to treat acute prostatitis, have not been shown to provide good
symptom relief for chronic bacterial prostatitis. "-Blockers are second-line agents for treating chronic
pelvic pain. Transrectal ultrasonography is indicated in patients whose acute prostatitis fails to resolve and
who have a persistent fever or whose maximal temperature is not trending downward after 36 hours. In
this case a prostatic abscess should be suspected, and transrectal ultrasonography can facilitate the
diagnosis. Prostate-specific antigen is a screening test for prostatic malignancy and would contribute
nothing to the management of this patients problem.

Ref: Sharp VJ, Takacs EB, Powell CR: Prostatitis: Diagnosis and treatment. Am Fam Physician 2010;82(4):397-406.

66
Item 200

ANSWER: D

Observational studies have consistently shown that alcohol use has a J-shaped curve for several health
effects. Small amounts of alcohol on a daily basis (less than 1 drink/day for women, and 12 drinks/day
for men) are associated with an 18% lower risk for all-cause mortality and a 30% decreased risk for
coronary heart disease. As the use of alcohol increases these benefits disappear and even reverse, showing
a dose-dependent increase in all-cause mortality when women consume more than 2 drinks/day or men
over 4 drinks/day. At this level of alcohol use, rates increase for hypertension, cancer, stroke, heart
failure, dementia, and diabetes mellitus.

Ref: OKeefe JH, Bybee KA, Lavie CJ: Alcohol and cardiovascular health: The razor-sharp double-edged sword. J Am Coll
Cardiol 2007;50(11):1009-1014. 2) OKeefe JH, Carter MD, Lavie CJ: Primary and secondary prevention of
cardiovascular diseases: A practical evidence-based approach. Mayo Clin Proc 2009;84(8):741-757. 3) Ronksley PE, Brien
SE, Turner BJ, et al: Association of alcohol consumption with selected cardiovascular disease outcomes: A systematic
review and meta-analysis. BMJ 2011;342:d671.

Item 201

ANSWER: D

The only FDA-approved oral medication for the treatment of interstitial cystitis is pentosan polysulfate
sodium, which is thought to repair the urothelium (SOR B). Trimethoprim/sulfamethoxazole and
nitrofurantoin are indicated for urinary tract infections (UTIs), but usually not in cases of cystitis with no
infection. In addition, this patient has already received empiric treatment for a UTI despite having multiple
negative urine cultures. Ibuprofen is an anti-inflammatory medication commonly used to treat pain but is
not specifically indicated for interstitial cystitis. While tricyclic antidepressants such as amitriptyline have
been used to treat interstitial cystitis, fluoxetine is not generally recommended.

Ref: Davis EL, El Khoudary SR, Talbott EO, et al: Safety and efficacy of the use of intravesical and oral pentosan polysulfate
sodium for interstitial cystitis: A randomized double-blind clinical trial. J Urol 2008;179(1):177185. 2) French LM,
Bhambore N: Interstitial cystitis/painful bladder syndrome. Am Fam Physician 2011;83(10):1175-1181.

Item 202

ANSWER: E

A previous history of depression is the strongest risk factor for depression during pregnancy. Other risk
factors include childhood abuse, smoking, age under 20, and low socioeconomic status, especially without
social support. A family history of hyperthyroidism is not a risk factor.

Ref: Stewart DE: Depression during pregnancy. N Engl J Med 2011;365(17):1605-1611.

67
Item 203

ANSWER: B

This patient has perlche, or angular cheilitis. Most cases are secondary to moisture from patients licking
their lips, promoting a monilial or staphylococcal infection. Other causes include contact and irritant
dermatitis. Underlying HIV infection, celiac disease, or vitamin B12 and iron deficiencies have also been
reported. Treatment may include appropriate topical creams such as mupirocin or antifungal agents, or
low-potency nonfluorinated corticosteroid creams for irritant or contact causes.

Ref: Gonsalves WC, Chi AC, Neville BW: Common oral lesions: Part I. Superficial mucosal lesions. Am Fam Physician
2007;75(4):501-507.

Item 204

ANSWER: C

Almost all patients with ocular problems should have visual acuity testing before anything else is done
(level of evidence 3, SOR A). If this is difficult, a local anesthetic may be applied. The main exception
to this rule is a chemical burn of the eye, which should be irrigated for 30 minutes before further
evaluation or treatment is undertaken..

Ref: Pokhrel PK, Loftus SA: Ocular emergencies. Am Fam Physician 2007;76(6):829-836.

Item 205

ANSWER: D

Systemic corticosteroids are recognized for their dramatic impact on both the subjective and objective
course of poison ivy dermatitis. Oral prednisone at an initial dosage of 1 mg/kg/day tapered over 1421
days is the standard regimen. Complications can result from the use of shorter prepackaged courses of
corticosteroid therapy, resulting in significant rebound flares. These products usually begin with an initial
dosage approximately half that of the recommended dosage, with the course tapering too rapidly.
Over-the-counter topical hydrocortisone is ineffective for all but the mildest cases, and once the disease
is established, superpotent topical corticosteroids do little to alter the overall course and natural history.
Antihistamines and compresses provide some symptomatic relief, but do little to alter the course of
established disease.

Ref: Habif TP: Clinical Dermatology: A Color Guide to Diagnosis and Therapy, ed 5. Mosby Elsevier, 2010, pp 138-139.

68
Item 206

ANSWER: A

The U.S. Preventive Services Task Force recommends that all adults be screened for colon cancer
beginning at age 50 and continue regular screening until age 75 (SOR A). They recommend against
continued routine screening in previously screened adults 7585 years of age and against any screening
in adults over 85 (SOR A). Most organizations do not recommend a particular screening method, but
instead list screening options, including fecal occult blood testing, flexible sigmoidoscopy, and
colonoscopy. The recommended interval for fecal occult blood testing is every year. There is new evidence
based on randomized, controlled trials that participation and detection rates for advanced adenomas and
cancer are higher for immunochemical fecal testing than for stool guaiac testing (SOR A). As long as
results are normal, screening colonoscopy is recommended at 10-year intervals and screening
sigmoidoscopy at 5-year intervals.

Ref: Hewitson P, Glasziou P, Irwig L, et al: Screening for colorectal cancer using the faecal occult blood test, Hemoccult.
Cochrane Database Syst Rev 2007;(1):CD001216. 2) US Preventive Services Task Force: Screening for colorectal cancer:
US Preventive Services Task Force recommendation statement. Ann Intern Med 2008;149(9):627-637. 3) Wilkins T,
Reynolds PL: Colorectal cancer: A summary of the evidence for screening and prevention. Am Fam Physician
2008;78(12):1385-1392, 1393-1394. 4) van Rossum LG, van Rijn AF, Laheij RJ, et al: Random comparison of guaiac and
immunochemical fecal occult blood tests for colorectal cancer in a screening population. Gastroenterology
2008;135(1):82-90.

Item 207

ANSWER: A

Teenage mortality is an important public health issue because the majority of deaths among teenagers are
caused by external causes of injury such as accidents, homicide, and suicide. The leading causes of death
for the teenage population remained constant throughout the period 19992006: accidents (48% of deaths),
homicide (13%), suicide (11%), cancer (6%), and heart disease (3%). Motor vehicle accidents accounted
for 73% of all deaths from unintentional injury.

Ref: Minino AM: Mortality among teenagers aged 1219 years: United States 19992006. National Center for Health Statistics,
NCHS data brief no 37, 2010.

Item 208

ANSWER: D

Interventions recommended for hospitalized older patients to reduce the risk of hospital-induced disability
include minimizing restricted diets. Bed rest orders should be avoided, with recommendations that the
patient ambulate 34 times/day and be out of bed and in a chair for all meals. This patient is disoriented
and probably has delirium. Restraints should be avoided if possible, and should be limited if they become
necessary. This would also apply to functional restraints, such as indwelling urinary catheters, IV poles,
nasal cannulas, continuous pulse oximetry, and telemetry, which all increase the risk of delirium.

Ref: Covinsky KE, Pierluissi E, Johnston CB: Hospitalization-associated disability: She was probably able to ambulate, but
Im not sure. JAMA 2011;306(16):1782-1793.

69
Item 209

ANSWER: C

The first step in diagnosing deep-vein thrombosis (DVT) is to complete a validated clinical prediction
inquiry such as the Wells Clinical Prediction Rule in order to estimate the pretest probability of DVT. The
Wells criteria include such factors as active cancer, calf swelling, pitting edema, prolonged inactivity, or
major surgery within the previous 12 weeks.

The next step for patients with a low pretest probability of DVT is a high-sensitivity D-dimer assay, with
a negative result indicating a low likelihood of DVT (SOR A). D-dimer is a degradation product of
cross-linked fibrin blood clots and is usually elevated in patients with DVT, although it can also be elevated
with other conditions such as recent surgery, hemorrhage, trauma, pregnancy, or cancer. If the assay is
negative, the likelihood of DVT is very small.

In this case, the patient has several factors listed in the Wells criteria, indicating a high pretest probability
of DVT. D-dimer testing would not be useful, as the next step in this patients evaluation should be
imaging. Ultrasonography is the best test for symptomatic proximal-vein thrombosis, with a sensitivity
ranging between 89% and 96% (SOR A). Although ultrasonography is the most appropriate first imaging
test, contrast venography is considered the definitive test to rule out the diagnosis of DVT if there is still
a high degree of suspicion after negative ultrasonography.

Helical CT is commonly used to detect pulmonary embolism but is not routinely recommended to diagnose
DVT. Similarly, MRI is not routinely recommended for detecting DVT.

Ref: Segal JB, Eng J, Tamariz LJ, Bass EB: Review of the evidence on diagnosis of deep venous thrombosis and pulmonary
embolism. Ann Fam Med 2007;5(1): 63-73. 2) Qaseem A, Snow V, Barry P, et al: Current diagnosis of venous
thromboembolism in primary care: A clinical practice guideline from the American Academy of Family Physicians and the
American College of Physicians. Ann Fam Med 2007;5(1):57-62.

Item 210

ANSWER: E

The extended flat pharmacokinetic curve of long-acting insulin analogues makes once-daily administration
of larger doses of insulin possible. Such treatment should, in theory, provide increased flexibility with
regard to the timing of injections and improve compliance. This should improve control of the patients
diabetes, reduce the risk of hypoglycemia, and improve overall patient satisfaction. To date, however, the
only proven benefit of treatment with insulin analogues is a reduction in the low rate of symptomatic,
nocturnal, and overt hypoglycemia experienced by patients treated with isophane insulin. Although the total
cost of treatment with insulin analogues is higher, a Cochrane review of the limited number of studies
comparing insulin treatments showed no statistically significant differences in the hemoglobin A1c levels
measured at the end of the studies in any treatment group (SOR C). Significant changes in morbidity,
mortality, or quality of life have not been demonstrated (SOR C).

Ref: Schooff MD, Gupta L: Are long-acting insulin analogues better than isophane insulin? Am Fam Physician
2008;77(4):447-449.

70
Item 211

ANSWER: B

In 2011, the American College of Physicians published new guidelines on COPD management. For
patients with COPD who are symptomatic and have an FEV1 less than 60% of predicted, the
recommendation is monotherapy with either a long-acting inhaled anticholinergic (tiotropium) or a
long-acting inhaled !-agonist such as salmeterol or formoterol. This is in addition to rescue therapy with
a short-acting inhaled bronchodilator such as albuterol. Long-acting inhaled anticholinergics and
long-acting inhaled !-agonists reduce exacerbations and improve quality of life. The evidence is
inconclusive with regard to their effect on mortality, hospitalizations, and dyspnea.

Inhaled corticosteroids have been found to be better than placebo for decreasing COPD exacerbations, but
their side-effect profile keeps them from being preferred as monotherapy. Neither inhaled mast-cell
stabilizers nor inhaled antihistamines are recommended as first-line agents for the treatment of COPD.

Ref: Qaseem A, Wilt TJ, Weinberger SE, et al: Diagnosis and management of stable chronic obstructive pulmonary disease:
A clinical practice guideline update from the American College of Physicians, American College of Chest Physicians,
American Thoracic Society, and European Respiratory Society. Ann Intern Med 2011;155(3):179-191. 2) Armstrong C:
ACP updates guideline on diagnosis and management of stable COPD. Am Fam Physician 2012;85(2):204-205.

Item 212

ANSWER: B

This patient has grade 2 internal hemorrhoids. These protrude with defecation but reduce spontaneously.
Sitz baths are commonly recommended, but a review of studies found no benefit from sitz baths for various
anorectal disorders, including hemorrhoids. A meta-analysis of seven randomized trials of patients with
symptomatic hemorrhoids showed that fiber supplementation with psyllium, sterculia, or unprocessed bran
decreased bleeding, pain, prolapse, and itching. No randomized, controlled trials support the use of
corticosteroid creams for treating hemorrhoidal disease. Topical diltiazem and topical lidocaine have been
shown to provide pain relief postoperatively following excision of external hemorrhoids.

Ref: Mounsey AL, Halladay J, Sadiq TS: Hemorrhoids. Am Fam Physician 2011;84(2):204-210.

Item 213

ANSWER: B

Sensitivity is defined as the percentage of patients with a disease who have a positive test for the disease
in question. Specificity is the percentage of patients without the disease who have a negative test. The
positive predictive value is the percentage of patients with a positive or abnormal test who have the disease
in question. The negative predictive value is the percentage of patients with a negative or normal test who
do not have the disease in question. Likelihood ratios correspond to the clinical impression of how well
a test rules in or rules out a given disease.

Ref: Flaherty RJ: A simple method for evaluating the clinical literature. Fam Pract Manag 2004;11(5):47-52. 2) Rakel RE,
Rakel DP (eds): Textbook of Family Medicine, ed 8. Elsevier Saunders, 2011, pp 112-119.

71
Item 214

ANSWER: D

Thrombotic thrombocytopenic purpura (TTP) is an emergent condition that can result in up to 30%
mortality. Prompt hospitalization with plasma exchange is the preferred treatment. Patients with TTP
present with nonspecific symptoms such as fever, abdominal pain, nausea, and weakness. Patients may
also exhibit neurologic deficits. Microangiopathic anemia is also likely to be present, as evidenced by
schistocytes on a peripheral smear and elevated levels of LDH and nucleated RBCs.

Congenital thrombocytopenia is a benign condition in which patients have long-standing low platelet counts
and/or a family history of thrombocytopenia. It is usually asymptomatic but a concomitant bleeding
diathesis may occur.

Gestational thrombocytopenia is also benign and asymptomatic. It is often confused with mild immune
thrombocytopenic purpura. Platelet counts rarely drop below 70,000/mm3. There is no associated fetal
thrombocytopenia. Preeclampsia and HELLP syndrome should also be ruled out. Platelet counts return
to normal after delivery (SOR C).

Drug-induced thrombocytopenia can be severe, but platelet counts do not usually drop below 20,000/mm3.
It is characterized by an abrupt drop in the platelet count within a week of starting the offending medication
and resolves within 2 weeks after the medication is stopped.

Lyme disease can be associated with a transient thrombocytopenia. Patients present with common
symptoms of Lyme disease, such as fever, myalgias, and rash. The thrombocytopenia resolves with
treatment of the underlying infection.

Ref: Hoffman R, Benz EJ Jr, Shattil SJ, et al: Hematology: Basic Principles and Practice, ed 5. Churchill Livingstone Elsevier,
2009, pp 2100-2101. 2) Gauer RL, Braun MM: Thrombocytopenia. Am Fam Physician 2012;85(6):612-622.

Item 215

ANSWER: A

Influenza should be diagnosed on the basis of clinical signs and symptoms rather than diagnostic testing.
Antiviral treatment is not recommended in otherwise healthy adults and children. Symptomatic treatment
should be initiated with over-the-counter antipyretics and anti-inflammatory medications, and aspirin should
be avoided due to the risk of Reyes syndrome. Antibiotics are indicated only when a bacterial coinfection
is diagnosed and not for prophylaxis.

Ref: Erlikh IV, Abraham S, Kondamudi VK: Management of influenza. Am Fam Physician 2010;82(9):1087-1095.

72
Item 216

ANSWER: C

This patient does not meet the LDL-cholesterol goals for a diabetic patient and therefore needs adjustment
of his antihyperlipidemic regimen. In June 2011, the Food and Drug Administration recommended limiting
the use of the highest dosage of simvastatin (80 mg/day) because of concerns about an increased risk of
muscle damage. This dosage should only be used in patients who have already been taking 80 mg/day for
12 months or more without evidence of muscle injury, and it should not be started in new patients. This
patient should be switched to an alternative medication that provides a greater reduction of
LDL-cholesterol, such as atorvastatin. Gemfibrozil is contraindicated for use with simvastatin because it
can raise simvastatin drug levels and increase the risk of myopathy.

Ref: FDA drug safety communication: New restrictions, contraindications, and dose limitations for Zocor (simvastatin) to reduce
the risk of muscle injury. US Food and Drug Administration, 2011. 2) PL Detail-Document: New restrictions,
contraindications, and dose limitations for Zocor (simvastatin). Pharmacists Letter/Prescribers Letter. July 2011.

Item 217

ANSWER: D

Polycystic ovary syndrome may be the most common cause of female infertility, affecting 6%8% of
women. Some patients with polycystic ovary syndrome have hyperandrogenism, elevated levels of
luteinizing hormone, and hyperinsulinemia. While early studies supported the use of metformin to increase
fertility, a more recent study has shown that only 7% of women treated with metformin were able to
conceive, whereas 22% of women treated with clomiphene citrate had a live birth. Spironolactone is useful
for treating hirsutism, but not infertility. Since levels of other hormones are already elevated, the other
measures listed would not be of benefit.

Ref: Legro RS, Barnhart HX, Schlaff WD, et al: Clomiphene, metformin, or both for infertility in the polycystic ovary
syndrome. N Engl J Med 2007;356(6):551-566. 2) Melmed S, Polonsky KS, Larsen PR, Kronenberg HM (eds): Williams
Textbook of Endocrinology, ed 12. Elsevier Saunders, 2011, pp 631-632.

Item 218

ANSWER: D

The American College of Rheumatology criteria for the diagnosis of systemic lupus erythematosus (SLE)
includes the presence of 4 of 11 criteria. One criterion is evidence of the presence of an immunologic
disorder. Evidence of an immunologic disorder includes a positive finding of antiphospholipid antibodies,
based upon one of the following: an abnormal serum level of immunoglobulin G or M anticardiolipin
antibodies, a positive lupus anticoagulant test, or a false-positive serologic test for syphilis. The other test
results listed are not criteria for the diagnosis of SLE.

Ref: Klippel JH, Stone JH, Crofford LJ, et al (eds): Primer on the Rheumatic Diseases, ed 13. Springer, 2008, pp 303-314.
2) Gibson K, Goodemote P, Johnson S: Antibody testing for systemic lupus erythematosus. Am Fam Physician
2011;84(12):1407-1409.

73
Item 219

ANSWER: A

This patient has stage 4 chronic kidney disease with a glomerular filtration rate (GFR) <30 mL/min. This
puts her at high risk for developing nephrogenic systemic fibrosis. The FDA recommends against using
gadolinium-based contrast agents in patients with acute or chronic kidney disease and a GFR <30 mL/min.

While anaphylaxis is possible, this patient would not be at a high risk for this. Dermatomyositis, focal
seizures, and hypertensive crisis are not associated with gadolinium-based contrast agents.

Ref: Schlaudecker JD, Bernheisel CR: Gadolinium-associated nephrogenic systemic fibrosis. Am Fam Physician
2009;80(7):711-714.

Item 220

ANSWER: D

Trimethoprim/sulfamethoxazole is the drug of choice for treating Pneumocystis jiroveci pneumonia.


Atovaquone has been shown to be very effective for treating mild to moderate Pneumocystis jiroveci
pneumonia and is also very well tolerated, and would be the first choice for a patient with a sulfa allergy.
Clindamycin plus primaquine is also effective therapy, but clindamycin is not effective as monotherapy.
Azithromycin and rifampin are not effective against this organism. Other treatment options include
pentamidine, dapsone plus trimethoprim, and clindamycin plus primaquine.

Ref: Mandell GL, Bennett JE, Dolin R (eds): Mandell, Douglas, and Bennetts Principles and Practice of Infectious Diseases,
ed 7. Churchill Livingstone, 2009, pp 1858-1874.

Item 221

ANSWER: E

Because of the high prevalence of reduced gastric acidity related to either endogenous causes or
medications such as proton pump inhibitors, calcium carbonate is best taken with meals to optimize
absorption. Calcium citrate, which is well absorbed regardless of gastric acidity, may be taken with or
without food. The other medications listed do not impair calcium absorption.

Ref: Lewiecki EM: Osteoporosis. Ann Intern Med 2011;155(1):ITC1-1-ITC1-15.

74
Item 222

ANSWER: B

Joint pain in the presence of fever with no apparent source indicates a possible infection, malignancy, or
rheumatologic condition and requires further workup. Laboratory evaluation, including a CBC, a
C-reactive protein level, and an erythrocyte sedimentation rate can help assess for these conditions, even
though none of the tests is sufficiently sensitive to rule out these diseases, and they are not specific to a
single disease entity. Knee joint aspiration would be indicated to rule out septic arthritis in the presence
of a joint effusion. If the hip were painful or had decreased range of motion, then ultrasonography could
help identify a hip joint effusion, which would need to be aspirated. MRI may be needed in this patient,
but it would likely require sedation and thus is more invasive. Starting with laboratory work is a good first
step toward identifying the source of his pain and fever.

Ref: Sawyer JR, Kapoor M: The limping child: A systematic approach to diagnosis. Am Fam Physician 2009;79(3):215-224.

Item 223

ANSWER: A

Extracorporeal shock wave therapy is effective for calcific tendinitis of the rotator cuff. Side effects include
bruising and pain. Needling and irrigation, physical therapy, and cortisone injections are sometimes used
in patients with acute symptoms. Endoscopic and open surgical treatments are alternatives to extracorporeal
shock wave therapy in refractory cases. Extracorporeal shock wave therapy does not have an established
role in gout, rotator cuff tear, frozen shoulder, or hooked acromion.

Ref: Gerdesmeyer L, Wagenpfeil S, Haake M, et al: Extracorporeal shock wave therapy for the treatment of chronic calcifying
tendonitis of the rotator cuff: A randomized controlled trial. JAMA 2003;290(19):2573-2580. 2) Murphy R, Carr A:
Clinical Evidence Handbook. Shoulder pain. Am Fam Physician 2011;83(2):137-138.

Item 224

ANSWER: E

This patient has acute bronchitis. The most appropriate management option is to provide reassurance that
symptoms will likely resolve on their own within 3 weeks. Approximately 90% of cases are caused by
viruses, and antibiotics do not significantly change the course of the condition. For this reason, and
because of concerns about antibiotic resistance and side effects from antibiotic use, antibiotics should not
be used routinely for the treatment of acute bronchitis (SOR B). Despite this, approximately two-thirds of
patients in the United States diagnosed with bronchitis are still treated with antibiotics. Corticosteroids and
!-agonists are not indicated in the absence of asthma or wheezing on examination. Expectorants have not
been shown to be effective in the treatment of bronchitis (SOR B).

Ref: Braman SS: Chronic cough due to acute bronchitis: ACCP evidence-based clinical practice guidelines. Chest 2006;129(1
Suppl):95S-103S. 2) Albert RH: Diagnosis and treatment of acute bronchitis. Am Fam Physician 2010;82(11):1345-1350.

75
Item 225

ANSWER: D

Recommendations for reducing ventilator-associated pneumonia (VAP) include elevation of the head to
an angle of at least 30, noninvasive mechanical ventilation rather than intubation when appropriate, oral
intubation when an endotracheal tube is necessary, orogastric rather than nasogastric tubes, minimization
of sedation, administration of a proton pump inhibitor when prophylaxis is indicated, changing ventilator
tubing every 7 days or when it becomes soiled, avoidance or elimination of endotracheal tube leaks, good
technique in removal of condensate, and excellent hand hygiene. One study favored waiting more than 5
days before initiating tube feedings, as this reduced the incidence of VAP, although further data is needed
to confirm this.

Ref: Koenig SM, Truwit JD: Ventilator-associated pneumonia: Diagnosis, treatment, and prevention. Clin Microbiol Rev
2006;19(4):637-657.

Item 226

ANSWER: D

There are no proven therapies to reduce the severity of borderline personality disorder (SOR A). The most
promising psychological therapy is dialectic behavioral therapy (DBT). DBT is a multi-faceted program
specifically designed to treat borderline personality disorder. The few, small studies of DBT found
improvement in many symptoms of borderline personality disorder, but long-term data is lacking. Another
promising therapy is psychoanalytic-oriented day hospital therapy. Again, study sizes have been small and
data cannot be extrapolated to the population as a whole.

Omega-3 fatty acids, second-generation antipsychotics, and mood stabilizers have been shown to be helpful
for some symptoms of borderline personality disorder but not for overall severity. Their benefits are based
on single-study results and side effects were not addressed in the studies. SSRIs are not recommended for
borderline personality disorder unless there is a concomitant mood disorder.

Ref: Binks CA, Fenton M, McCarthy L, et al: Psychological therapies for people with borderline personality disorder. Cochrane
Database Syst Rev 2006;(1):CD005652. 2) Sadock BJ, Sadock VA, Ruiz P (eds): Kaplan & Sadocks Comprehensive
Textbook of Psychiatry, ed 9. Lippincott Williams & Wilkins, 2009, p 2885. 3) Stoffers J, Vllm BA, Rcker G, et al:
Pharmacological interventions for borderline personality disorder. Cochrane Database Syst Rev 2010;(6):CD005653.

Item 227

ANSWER: A

The drug most often recommended as first-line therapy for hypertension in pregnancy is labetalol. Reports
of an association of metoprolol with fetal growth restriction have given rise to the recommendation to avoid
its use in pregnancy. Both ACE inhibitors and angiotensin-receptor blockers are contraindicated in
pregnancy because of the risk of birth defects and fetal or neonatal renal failure. Immediate-release
nifedipine is not recommended due to the risk of hypotension.

Ref: Leeman L, Fontaine P: Hypertensive disorders of pregnancy. Am Fam Physician 2008;78(1):93-100. 2) Seely EW, Ecker
J: Chronic hypertension in pregnancy. N Engl J Med 2011;365(5):439-446.

76
Item 228

ANSWER: D

Staphylococcus aureus is the most common cause of acute infectious endocarditis worldwide. Additionally,
the most common cause of tricuspid valve endocarditis is intravenous drug abuse, and Staphylococcus
aureus is the infecting organism in 80% of tricuspid valve infections. Streptococcus viridans is also a
frequent cause of infectious endocarditis, with Enterococcus, Pseudomonas, and Cardiobacterium being
less likely causes.

Ref: Bonow RO, Carabello BA, Chatterjee K, et al: 2008 Focused update incorporated into the ACC/AHA 2006 guidelines for
the management of patients with valvular heart disease: A report of the American College of Cardiology/American Heart
Association Task Force on Practice Guidelines (Writing Committee to Revise the 1998 Guidelines for the Management of
Patients With Valvular Heart Disease): Endorsed by the Society of Cardiovascular Anesthesiologists, Society for
Cardiovascular Angiography and Interventions, and Society of Thoracic Surgeons. Circulation 2008;118(15):e523-e661.
2) Fuster V, Walsh RA, Harrington RA (eds): Hursts The Heart, ed 13. McGraw-Hill, 2011, chap 86.

Item 229

ANSWER: E

A number of measures for managing spinal compression fractures have been evaluated. The evidence for
recommending kyphoplasty is weak, and the evidence for recommending against vertebroplasty is strong.
The data on bracing is inconclusive, as is the recommendation for bed rest. Calcitonin has been shown to
reduce the incidence of recurrent fractures and may be useful in the relief of pain.

Ref: Esses SI, McGuire R, Jenkins J, et al: The treatment of symptomatic osteoporotic spinal compression fractures. J Am Acad
Orthop Surg 2011;19(3):176-182.

Item 230

ANSWER: A

Heart disease is the leading cause of death in the U.S., and this holds true for both men and women.
Among men the only ethnicity for which heart disease is not the most common cause of death is
Asian/Pacific Islander.

Ref: Heron M: Deaths: Leading causes for 2006. Natl Vital Stat Rep 2010;58(14):12.

Item 231

ANSWER: A

A middle-aged woman with pruritus and elevated levels of alkaline phosphatase, #-glutamyltransferase
(GGT), and antimitochondrial antibody titers is likely to have primary biliary cirrhosis. Levels of
5'-nucleotidase or GGT are usually elevated in parallel with those of alkaline phosphatase in patients with
liver disease, but not in patients with bone disorders. Infiltrative liver disease, as seen with sarcoidosis,
drug-induced cholestasis, and choledocholithiasis, is not associated with elevated antimitochondrial
antibody levels. Patients with choledocholithiasis will also usually have dilated hepatic ducts on
ultrasonography. This patient should next have a liver biopsy to confirm her diagnosis.

77
Ref: Pratt DS, Kaplan MM: Evaluation of abnormal liver-enzyme results in asymptomatic patients. N Engl J Med
2000;342(17):1266-1271. 2) Longo DL, Fauci AS, Kasper DL, et al (eds): Harrisons Principles of Internal Medicine,
ed 18. McGraw-Hill, 2012, p 2595.

Item 232

ANSWER: C

According to the U.S. Preventive Services Task Force (USPSTF), there is good evidence that screening
for Chlamydia infection in women who are at increased risk can reduce the incidence of pelvic
inflammatory disease, while the harms are minimal. The evidence regarding screening for cervical cancer
with Papanicolaou testing or human papillomavirus (HPV) testing, however, shows that the harms
outweigh any possible benefits. Harms include overdiagnosis and overtreatment, including invasive
cervical procedures that can affect future pregnancy outcomes. In addition, there is adequate evidence that
screening women younger than 21 years of age (regardless of sexual history) does not reduce the incidence
of cervical cancer or mortality compared with beginning screening at age 21. The USPSTF concludes that
the evidence is insufficient to recommend for or against routine screening for lipid disorders or intimate
partner violence in women this age.

Ref: Screening for family and intimate partner violence: Recommendation statement. US Preventive Services Task Force, 2004.
2) Screening for chlamydial infection: Recommendation statement. US Preventive Services Task Force, 2007. 3) Screening
for lipid disorders in adults: Recommendation statement. US Preventive Services Task Force, 2008. 4) Screening for
cervical cancer: US Preventive Services Task Force recommendation statement. US Preventive Services Task Force, 2012.

Item 233

ANSWER: C

Interactions between the physician and patient involve the patients family and friends, as well as others
who may be part of the patients social support system. These interactions, as well as the physician-patient
relationship itself, are also strongly influenced by ethnic, cultural, and spiritual values and by beliefs about
illness and approaches to treatment and ongoing care. Involvement of family members in a patients care
is advantageous to good communication and helpful for both accurate diagnosis and appropriate treatment
(SOR C).

Patient confidentiality should be protected, and the mere presence of other people accompanying the patient
does not automatically constitute permission to discuss private health information with them. The physician
should determine not only who is present with a patient, but also each persons reason for being present
(SOR C). Speaking to relatives or friends without the patient present may be appropriate under certain
circumstances but would not be the best option in this scenario. Constructing a genogram can also be
helpful, but that would not be the first priority in this case.

Ref: Omole FS, Sow CM, Fresh E, et al: Interacting with patients family members during the office visit. Am Fam Physician
2011;84(7):780-784.

78
Item 234

ANSWER: C

Radiographs confirm the clinical diagnosis of small-bowel obstruction in most patients and more accurately
define the site of obstruction. Small-bowel obstruction typically occupies the more central portions of the
abdomen. Patients with mechanical small-bowel obstruction usually have minimal or no colonic gas. Films
taken in the upright or lateral decubitus position in patients with small-bowel obstruction usually show
multiple gas-filled levels, with the distended bowel resembling an inverted U.

Patients with small-bowel obstruction are likely to be depleted of fluids and electrolytes, and will require
intravenous fluids, electrolyte management, and surgical evaluation.

Ref: Marx JA (ed): Rosens Emergency Medicine: Concepts and Clinical Practice, ed 7. Mosby Elsevier, 2010, pp 1184-1188.
2) Townsend CM Jr, Beauchamp RD, Evers BM, et al: Sabiston Textbook of Surgery: The Biological Basis of Modern
Surgical Practice, ed 19. Saunders, 2012, pp 1236-1243.

Item 235

ANSWER: D

The salient feature of atrial fibrillation is the absence of P waves, along with normal QRS complexes that
are irregular in time (irregularly irregular) and sometimes vary in amplitude. Sinus tachycardia,
paroxysmal tachycardia, multifocal atrial tachycardia, and atrial flutter are all associated with P waves that
are constantly related to QRS complexes, although they may sometimes be abnormal and difficult to
discern.

Ref: Bonow RO, Mann DL, Zipes DP, Libby P (eds): Braunwalds Heart Disease: A Textbook of Cardiovascular Medicine, ed
9. Elsevier Saunders, 2011, pp 825-844. 2) Longo DL, Fauci AS, Kasper DL, et al (eds): Harrisons Principles of Internal
Medicine, ed 18. McGraw-Hill, 2012, pp 1880-1890.

Item 236

ANSWER: D

Scleritis is an inflammatory disorder affecting the sclera, often associated with a connective tissue disorder
such as rheumatoid arthritis, systemic lupus erythematosus, Wegeners granulomatosis, polyarteritis
nodosa, or relapsing polychondritis. In the anterior form of scleritis, inflammation results in local or
diffuse erythema and thickening of the sclera. Patients present with either diffuse or focal ocular erythema,
tenderness, and pain. When the inflammation is focal, a tender nodule may be present. The initial
treatment of scleritis is an oral NSAID to help reduce ocular inflammation. Topical or systemic
corticosteroids may be used when NSAIDs fail or are contraindicated (SOR B).

Ref: Longo DL, Fauci AS, Kasper DL, et al (eds): Harrisons Principles of Internal Medicine, ed 18. McGraw-Hill, 2012, p
229.

79
Item 237

ANSWER: C

The condition shown is representative of a simple anterior dislocation of the lunate. The semilunar shape
of the lunate bone is displaced anterior to the distal radial articular surface. Occasionally, a transnavicular
fracture may occur along with this injury and is termed a trans-scaphoid perilunate fracture-dislocation.
There is no evidence in the radiograph shown, however, of dislocations of the other areas mentioned.

Ref: DeLee JC, Drez D Jr, Miller MD (eds): DeLee & Drez's Orthopaedic Sports Medicine: Principles and Practice, ed 3.
Elsevier Saunders, 2010, pp 1321-1325.

Item 238

ANSWER: A

The findings in this patient are consistent with a lung abscess caused by anaerobic organisms, which is
usually related to aspiration. Most patients have a history of compromised consciousness, such as a seizure
disorder or drug and alcohol abuse, and many have dental or gingival disease.

Routine bacteriologic studies of expectorated sputum are hampered by mouth contamination, but are useful
for detecting mycobacteria and other potential etiologic agents. An open lung biopsy, immediate
bronchoscopy, and transtracheal aspiration are useful when the patient has not responded to initial therapy.

Since this is a typical clinical picture for anaerobic lung abscess, the treatment of choice would be large
doses of intravenous clindamycin.

Ref: Longo DL, Fauci AS, Kasper DL, et al (eds): Harrisons Principles of Internal Medicine, ed 18. McGraw-Hill, 2012, p
2145. 2) Bope ET, Kellerman RD (eds): Conns Current Therapy 2012. Elsevier Saunders, 2012, 366-367.

Item 239

ANSWER: A

Right ventricular infarction is most frequently an extension of an inferior myocardial infarction (MI).
Right-sided precordial leads may need to be evaluated to document this on the EKG. Hypotension during
the acute event, especially after nitrate administration, is characteristic. Frequently, 12 liters of normal
saline must be administered.

Acute pericarditis most frequently develops 24 days after the infarction. Free wall rupture, septal rupture,
and papillary muscle rupture typically do not occur until 15 days after the acute MI. Septal rupture is
more common with anterior MI.

Ref: Tintinalli JE, Kelen GD, Stapczynski JS (eds): Emergency Medicine: A Comprehensive Study Guide, ed 7. McGraw-Hill,
2011, pp 383-384.

80
Item 240

ANSWER: B

Eczema craquel, a common complication of aging, is due to dryness of the skin. It is best treated with
wet compresses and antibiotics to remove crusts and suppress infection, followed by topical corticosteroids
and lubricants. The primary lesions do not suggest scabies or necrobiosis lipoidica, and neurodermatitis
and lichen sclerosis are secondary responses to itching.

Ref: Habif TP: Clinical Dermatology: A Color Guide to Diagnosis and Therapy, ed 5. Mosby Elsevier, 2010, p 110.

81
American Board of Family Medicine

2011 IN-TRAINING EXAMINATION

CRITIQUE BOOK

This book contains the answers to each question in the In-Training Examination, as well as a critique that
provides a rationale for the correct answer. Bibliographic references are included at the end of each
critique to facilitate any further study you may wish to do in a particular area.

Copyright 2011 The American Board of Family Medicine, Inc. All rights reserved.
Item 1

ANSWER: D

Celiac sprue is an autoimmune disorder characterized by inflammation of the small bowel wall, blunting
of the villi, and resultant malabsorption. Symptoms commonly include diarrhea, fatigue, weight loss,
abdominal pain, and borborygmus; treatment consists of elimination of gluten proteins from the diet.
Extraintestinal manifestations are less common but may include elevated transaminases, osteopenia, and
iron deficiency anemia. Serum IgA tissue transglutaminase (TTG) antibodies are highly sensitive and
specific for celiac sprue, and a small bowel biopsy showing villous atrophy is the gold standard for
diagnosis. This patients rash is consistent with dermatitis herpetiformis, which is pathognomonic for celiac
sprue and responds well to a strict gluten-free diet.

Lactose intolerance, irritable bowel syndrome, collagenous colitis, and Crohns disease are in the
differential diagnosis for celiac sprue. However, significant weight loss is not characteristic of irritable
bowel syndrome or lactose intolerance. The diarrhea associated with Crohns disease is typically bloody.
Collagenous colitis does cause symptoms similar to those experienced by this patient, but it is not
associated with dermatitis herpetiformis.

Ref: Presutti RJ, Cangemi JR, Cassidy HD, Hill DA: Celiac disease. Am Fam Physician 2007;76(12):1795-1802.

Item 2

ANSWER: E

There are no absolute contraindications to electroconvulsive therapy (ECT), but factors that have been
associated with reduced efficacy include a prolonged episode, lack of response to medication, and
coexisting psychiatric diagnoses such as a personality disorder. Persons who may be at increased risk for
complications include those with unstable cardiac disease such as ischemia or arrhythmias, cerebrovascular
disease such as recent cerebral hemorrhage or stroke, or increased intracranial pressure. ECT can be used
safely in elderly patients and in persons with cardiac pacemakers or implantable cardioverter-defibrillators.
ECT also can be used safely during pregnancy, with proper precautions and in consultation with an
obstetrician.

Ref: Lisanby SH: Electroconvulsive therapy for depression. N Engl J Med 2007;357(19):1939-1945.

Item 3

ANSWER: E

Difficult-to-control hypertension has many possible causes, including nonadherence or the use of alcohol,
NSAIDs, certain antidepressants, or sympathomimetics. Secondary hypertension can be caused by
relatively common problems such as chronic kidney disease, obstructive sleep apnea, or primary
hyperaldosteronism, as in the case described here.

As many as 20% of patients referred to specialists for poorly controlled hypertension have primary
hyperaldosteronism. It is more common in women and often is asymptomatic. A significant number of
these individuals will not be hypokalemic. Screening can be done with a morning plasma aldosterone/renin
ratio. If the ratio is 20 or more and the aldosterone level is >15 ng/dL, then primary hyperaldosteronism
is likely and referral for confirmatory testing should be considered.

1
Ref: Viera AJ, Hinderliter AL: Education and management of the patient with difficult-to-control or resistant hypertension. Am
Fam Physician 2009;79(10):863-869.

Item 4

ANSWER: E

The diagnosis of community-acquired pneumonia is mostly based on the history and physical examination.
Pneumonia should be suspected in any child with fever, cyanosis, and any abnormal respiratory finding
in the history or physical examination. Children under 2 years of age who are in day care are at higher risk
for developing community-acquired pneumonia. Laboratory tests are rarely helpful in differentiating viral
versus bacterial etiologies and should not be routinely performed. Outpatient antibiotics are appropriate
if the child does not have a toxic appearance, hypoxemia, signs of respiratory distress, or dehydration.
Streptococcus pneumoniae is one of the most common etiologies in this age group, and high-dose
amoxicillin is the drug of choice.

Ref: Ostapchuk M, Roberts D, Haddy R: Community-acquired pneumonia in infants and children. Am Fam Physician
2004;70(5):899-908. 2) Kliegman RM, Stanton BF, Geme JW III, et al (eds): Nelson Textbook of Pediatrics, ed 19.
Elsevier Saunders, 2011, pp 1474-1479.

Item 5

ANSWER: C

In normal term infants, there is little evidence that solid foods contribute to well-being before the age of
46 months. In addition, the extrusion reflex (pushing foreign material out of the mouth with the tongue)
makes feeding of solids difficult and often forced. This reflex disappears around the age of 4 months,
making feeding easier. The introduction of solids at this age helps supply calories, iron, and vitamins, and
may prepare the infant for later dietary diversity and healthy dietary habits.

Ref: Kliegman RM, Stanton BF, Geme JW III, et al (eds): Nelson Textbook of Pediatrics, ed 19. Elsevier Saunders, 2011, pp
164-165.

Item 6

ANSWER: D

Midshaft posteromedial tibial stress fractures are common and are considered low risk. Management
consists of relative rest from running and avoiding other activities that cause pain. Once usual daily
activities are pain free, low-impact exercise can be initiated and followed by a gradual return to previous
levels of running. A pneumatic stirrup leg brace has been found to be helpful during treatment (SOR C).
Nonweight bearing is not necessary, as this patient can walk without pain. Casting is not recommended.
Ultrasonic pulse therapy has helped fracture healing in some instances, but has not been shown to be
beneficial in stress fractures.

Ref: Harrast MA, Colonno D: Stress fractures in runners. Clin Sports Med 2010;29(3):399-418.

2
Item 7

ANSWER: B

A serum total testosterone level is recommended as the initial screening test for late-onset male
hypogonadism. Due to its high cost, a free testosterone level is recommended only if the total testosterone
level is borderline and abnormalities in sex hormonebinding globulin are suspected. Follow-up LH and
FSH levels help to distinguish primary from secondary hypogonadism.

Ref: Bhasin S, Cunningham GR, Hayes FJ, et al: Testosterone therapy in men with androgen deficiency syndromes: An
Endocrine Society clinical practice guideline. J Clin Endocrinol Metab 2010;95(6):2536-2559.

Item 8

ANSWER: C

Because of the precise relationship between circulating thyroid hormone and pituitary TSH secretion,
measurement of serum TSH is essential in the management of patients receiving levothyroxine therapy.
Immunoassays can reliably distinguish between normal and suppressed concentrations of TSH. In a patient
receiving levothyroxine, a low TSH level usually indicates overreplacement. If this occurs, the dosage
should be reduced slightly and the TSH level repeated in 23 months time. There is no need to
discontinue therapy in this situation, and repeating the TSH level in 2 weeks would not be helpful. A free
T4 level would also be unnecessary, since it is not as sensitive as a TSH level for detecting mild states of
excess thyroid hormone.

Ref: Hennessey JV, Scherger JE: Evaluating and treating the patient with hypothyroid disease. J Fam Pract 2007;56(8 Suppl
Hot Topics):S31-S39. 2) Woeber KA: The year in review: The thyroid. Ann Intern Med 1999;131(12):959-962. 3) Fauci
AS, Braunwald E, Kasper DL, et al (eds): Harrisons Principles of Internal Medicine, ed 17. McGraw-Hill, 2008, pp
2228-2233.

Item 9

ANSWER: B

Plantar fasciitis is a common cause of heel pain. It may be unilateral or bilateral, and the etiology is
unknown, although it is thought to be due to cumulative overload stress. While it may be associated with
obesity or overuse, it may also occur in active or inactive patients of all ages. Typically the pain is located
in the plantar surface of the heel and is worst when the patient first stands up when getting out of bed in
the morning (first step phenomenon) or after prolonged sitting. The pain may then improve after the patient
walks around, only to worsen after prolonged walking. The diagnosis is made by history and physical
examination. Typical findings include point tenderness to palpation on the plantar surface of the heel at
the medial calcaneal tuberosity where the calcaneal aponeurosis inserts. Radiographs are not necessary
unless there is a history of trauma or if the diagnosis is unclear.

The condition may last for months or years, and resolves in most patients over time with or without
specific therapy. One long-term follow-up study showed that 80% of patients had complete resolution of
their pain after 4 years. Treatments with limited (level 2) evidence of effectiveness include off-the-shelf
insoles, custom-made insoles, stretching of the plantar fascia, corticosteroid iontophoresis, custom-made
night splints, and surgery (for those who have failed conservative therapy). NSAIDs and ice, although not
independently studied for plantar fasciitis, are included in most studies of other treatments, and are
reasonable adjuncts to first-line therapy. Magnetic insoles and extracorporeal shockwave therapy are
ineffective in treating plantar fasciitis.

3
Due to their expense, custom-made insoles, custom-made night splints, and corticosteroid iontophoresis
should be reserved as second-line treatments for patients who fail first-line treatment. Surgery may be
offered if more conservative therapies fail. Corticosteroid injection may have a short-term benefit at 1
month, but is no better than other treatments at 6 months and carries a risk of plantar fascia rupture.

Ref: Landorf KB, Keenan AM, Herbert RD: Effectiveness of foot orthoses to treat plantar fasciitis: A randomized trial. Arch
Intern Med 2006;166(12):1305-1310.

Item 10

ANSWER: D

The history and physical findings in this patient are consistent with gingivostomatitis due to a primary or
initial infection with herpes simplex virus type 1 (HSV-1). There are no additional findings to suggest other
diagnoses such as aphthous ulcers, Behets syndrome, or herpangina (coxsackievirus).

After a primary HSV-1 infection with oral involvement, the virus invades the neurons and replicates in the
trigeminal sensory ganglion, leading to recurrent herpes labialis and erythema multiforme, among other
things. Although some clinicians might choose to use oral anesthetics for symptomatic care, it is not a
specific therapy.

Antibiotics are not useful for the treatment of herpetic gingivostomatitis and could confuse the clinical
picture should this child develop erythema multiforme, which occurs with HSV-1 infections. An orally
applied corticosteroid is not specific treatment, but some might try it for symptomatic relief. An
immunosuppressant is sometimes used for the treatment of Behets syndrome, but this patients findings
are not consistent with that diagnosis. Therefore, the only specific treatment listed is acyclovir suspension,
which has been shown to lead to earlier resolution of fever, oral lesions, and difficulties with eating and
drinking. It also reduces viral shedding from 5 days to 1 day (SOR B).

Ref: Usatine RP, Tinitigan R: Nongenital herpes simplex virus. Am Fam Physician 2010;82(9):1075-1082.

Item 11

ANSWER: B

Anterior cruciate ligament (ACL) tears occur more commonly in women than in men. The intensity of play
is also a factor, with a much greater risk of ACL injuries occurring during games than during practices.
The most accurate maneuver for detecting an ACL tear is the Lachman test (sensitivity 60%100%, mean
84%), followed by the anterior drawer test (sensitivity 9%93%, mean 62%) and the pivot shift test
(sensitivity 27%95%, mean 62%) (SOR C). McMurrays test is used to detect meniscal tears.

Ref: Solomon DH, Simel DL, Bates DW, et al: The rational clinical examination. Does this patient have a torn meniscus or
ligament of the knee? Value of the physical examination. JAMA 2001;286(13):1610-1620. 2) Cimino F, Volk BS, Setter
D: Anterior cruciate ligament injury: Diagnosis, management, and prevention. Am Fam Physician 2010;82(8):917-922.

4
Item 12

ANSWER: D

The diagnosis of multiple myeloma is based on evidence of myeloma-related end-organ impairment in the
presence of M protein, monoclonal plasma cells, or both. This evidence may include hypercalcemia, renal
failure, anemia, or skeletal lesions. Monoclonal gammopathy of undetermined significance does not
progress steadily to multiple myeloma. There is a stable 1% annual risk of progression.

Ref: Landgren O, Waxman AJ: Multiple myeloma precursor disease. JAMA 2010;304(21):2397-2404.

Item 13

ANSWER: A

These findings are typical of femoral neuropathy, a mononeuropathy commonly associated with diabetes
mellitus, although it has been found to be secondary to a number of conditions that are common in
diabetics and not to the diabetes itself. Diabetic polyneuropathy is characterized by symmetric and distal
limb sensory and motor deficits. Meralgia paresthetica, or lateral femoral cutaneous neuropathy, may be
secondary to diabetes mellitus, but is manifested by numbness and paresthesia over the anterolateral thigh
with no motor dysfunction. Spinal stenosis causes pain in the legs, but is not associated with the neurologic
signs seen in this patient, nor with knee problems. Iliofemoral atherosclerosis, a relatively common
complication of diabetes mellitus, may produce intermittent claudication involving one or both calf muscles
but would not produce the motor weakness noted in this patient.

Ref: Bradley WG, Daroff RB, Fenichel GM, Jankovic J (eds): Neurology in Clinical Practice, ed 5. Butterworth Heinemann
Elsevier, 2008, pp 442-443. 2) Ropper AH, Samuels MA: Adams and Victors Principles of Neurology, ed 9.
McGraw-Hill, 2009, p 1318. 3) Melmed S, Polonsky KS, Larsen PR, Kronenberg HM (eds): Williams Textbook of
Endocrinology, ed 12. Elsevier Saunders, 2011, pp 1503-1505.

Item 14

ANSWER: E

Women who use low-dose estrogen oral contraceptives have at least a 50% lower risk of subsequent
epithelial ovarian cancer than women who have never used them. Epidemiologic data also suggests other
potential long-term benefits of oral contraceptives, including a reduced risk of postmenopausal fractures,
as well as reductions in the risk of endometrial and colorectal cancers. Oral contraceptives do not reduce
the risk of carcinoma of the breast, cervix, lung, or head and neck.

Ref: Kaunitz AM: Hormonal contraception in women of older reproductive age. N Engl J Med 2008;358(12):1262-1270.

Item 15

ANSWER: C

Polymyalgia rheumatica is an inflammatory disorder that occurs in persons over the age of 50. White
women of European ancestry are most commonly affected. The clinical hallmarks of polymyalgia
rheumatica are pain and stiffness in the shoulder and pelvic girdle. One review found that 4%13% of
patients with clinical polymyalgia rheumatica have a normal erythrocyte sedimentation rate (ESR). As
many as 5% of patients initially have a normal ESR that later rises.

5
Polymyalgia rheumatica can have a variety of systemic symptoms. Fever is common, with temperatures
as high as 39C (102F) along with night sweats. Additional symptoms include depression, fatigue,
malaise, anorexia, and weight loss.

Corticosteroids are the mainstay of therapy for polymyalgia rheumatica. Typically, a dramatic response
is seen within 4872 hours.

Ref: Michet CJ, Matteson EL: Polymyalgia rheumatica. BMJ 2008;336(7647):765-769.

Item 16

ANSWER: C

This patient meets the criteria for frontotemporal dementia (FTD), a common cause of dementia in patients
younger than 65, with an insidious onset. Unlike with Alzheimers disease, memory is often relatively
preserved, even though insight is commonly impaired.

There are three subtypes of frontotemporal dementia: behavioral variant FTD, semantic dementia, and
progressive nonfluent aphasia. This patient would be diagnosed with the behavioral variant due to his loss
of executive functioning leading to personality change (apathy) and inappropriate behavior (SOR C).
Speech output is often distorted in frontotemporal dementia, although the particular changes differ between
the three variants.

Patients with FTD often are mistakenly thought to have major depressive disorder due to their apathy and
diminished interest in activities. However, patients with depression do not usually exhibit inappropriate
behavior and lack of restraint. Dementia with Lewy bodies and Alzheimers dementia are both
characterized predominantly by memory loss. Alzheimers dementia is most common after age 65, whereas
FTD occurs most often at a younger age. Lewy body dementia is associated with parkinsonian motor
features. Patients diagnosed with schizophrenia exhibit apathy and personality changes such as those seen
in FTD. However, the age of onset is much earlier, usually in the teens and twenties in men and the
twenties and thirties in women.

Ref: Cardarelli R, Kertesz A, Knebl JA: Frontotemporal dementia: A review for primary care physicians. Am Fam Physician
2010;82(11):1372-1377.

Item 17

ANSWER: D

Refeeding syndrome can be defined as the potentially fatal shifts in fluids and electrolytes that may occur
in malnourished patients receiving artificial refeeding (whether enterally or parenterally). These shifts
result from hormonal and metabolic changes and may cause serious clinical complications. The hallmark
biochemical feature of refeeding syndrome is hypophosphatemia. However, the syndrome is complex and
may also include abnormal sodium and fluid balance; changes in glucose, protein, and fat metabolism;
thiamine deficiency; hypokalemia; and hypomagnesemia.

Ref: Mehanna HM, Moledina J, Travis J: Refeeding syndrome: What it is, and how to prevent and treat it. BMJ
2008;336(7659):1495-1498.

6
Item 18

ANSWER: A

Pharyngeal and laryngeal side effects of inhaled corticosteroids include sore throat, coughing on inhalation
of the medication, a weak or hoarse voice, and oral candidiasis. Rinsing the mouth after each
administration of the medication and using a valved holding chamber when it is delivered with a
metered-dose inhaler can minimize the risk of oral candidiasis.

Ref: Fanta CH: Asthma. N Engl J Med 2009;360(10):1002-1014.

Item 19

ANSWER: E

Hoarseness most commonly affects teachers and older adults. The cause is usually benign, but extended
symptoms or certain risk factors should prompt evaluation; specifically, laryngoscopy is recommended
when hoarseness does not resolve within 3 months or when a serious underlying cause is suspected (SOR C).
The American Academy of Otolaryngology/Head and Neck Surgery Foundation guidelines state that
antireflux medications should not be prescribed for patients with hoarseness without reflux symptoms
(SOR C). Antibiotics should not be used, as the condition is usually caused by acute laryngitis or an upper
respiratory infection, and these are most likely to be viral. Inhaled corticosteroids are a common cause
of hoarseness. Voice therapy should be reserved for patients who have undergone laryngoscopy first
(SOR A).

Ref: Schwartz SR, Cohen SM, Dailey SH, Rosenfeld RM, et al: Clinical practice guideline: Hoarseness (dysphonia). Otolaryngol
Head Neck Surg 2009;141(3 suppl 2):S1-S31. 2) Huntzinger A: Guidelines for the diagnosis and management of
hoarseness. Am Fam Physician 2010;81(10):1292-1296.

Item 20

ANSWER: B

Although myocarditis, pulmonic stenosis, and ventricular septal defects can be causes of right heart failure,
left heart failure is the most common cause of right heart failure in adults.

Ref: Goldman L, Ausiello D (eds): Cecil Medicine, ed 23. Saunders, 2008, pp 349-350.

Item 21

ANSWER: C

Results of urine drug test panels obtained in the workplace are reported by a Medical Review Officer
(MRO) as positive, negative, dilute, refusal to test, or test canceled; the drug/metabolite for which the test
is positive or the reason for refusal (e.g., the presence of an adulterant) or cancellation is also included in
the final report. The MRO interpretation is based on consideration of many factors, including the
confirmed patient medical history, specimen collection process, acceptability of the specimen submitted,
and qualified laboratory measurement of drugs or metabolites in excess of the accepted thresholds. These
thresholds are set to preclude the possibility that secondary contact with smoke, ingestion of poppy seeds,
or similar exposures will result in an undeserved positive urine drug screen report. Other findings, such
as the presence of behavioral or physical evidence of unauthorized use of opiates, may also factor into the
final report.

7
When a properly collected, acceptable specimen is found to contain drugs or metabolites that would be
expected based on a review of confirmed prescribed use of medications, the test is reported as negative.
Morphine is a metabolite of codeine that may be found in the urine of someone taking a codeine-containing
medication; morphine is not a metabolite of methadone. Oxazepam is a metabolite of diazepam but the
reverse is not true. Tetrahydrocannabinol would not be found in the urine as a result of tramadol use.

Ref: Procedures for Transportation Workplace Drug and Alcohol Testing Programs. Office of Drug & Alcohol Policy &
Compliance, US Dept of Transportation, 2010, sections 40.139, 40.141, 40.163. 2) Standridge JB, Adams SM, Zotos AP:
Urine drug screening: A valuable office procedure. Am Fam Physician 2010;81(5):635-640.

Item 22

ANSWER: E

Infectious mononucleosis presents most commonly with a sore throat, fatigue, myalgias, and
lymphadenopathy, and is most prevalent between 10 and 30 years of age. Both an atypical lymphocytosis
and a positive heterophil antibody test support the diagnosis, although false-negative heterophil testing is
common early in the disease course. The cornerstone of treatment for mononucleosis is supportive,
including hydration, NSAIDs, and throat sprays or lozenges.

In general, corticosteroids do not have a significant effect on the clinical course of infectious
mononucleosis, and they should not be used routinely unless the patient has evidence of acute airway
obstruction. Antihistamines are also not recommended as routine treatment for mononucleosis. The use
of acyclovir has shown no consistent or significant benefit, and antiviral drugs are not recommended.

There is also no evidence to support bed rest as an effective management strategy for mononucleosis.
Given the evidence from other disease states, bed rest may actually be harmful.

Although most patients will not have a palpably enlarged spleen on examination, it is likely that all, or
nearly all, patients with mononucleosis have splenomegaly. This was demonstrated in a small study in
which 100% of patients hospitalized for mononucleosis had an enlarged spleen by ultrasound examination,
whereas only 17% of patients with splenomegaly have a palpable spleen. Patients should be advised to
avoid contact- or collision-type activities for 34 weeks because of the increased risk of rupture.

Ref: Ebell MH: Epstein-Barr virus infectious mononucleosis. Am Fam Physician 2004;70(7):1279-1287. 2) Dickens KP, Nye
AM, Gilchrist V, et al: Clinical Inquiries. Should you use steroids to treat infectious mononucleosis? J Fam Pract
2008;57(11):754-755.

Item 23

ANSWER: C

The initial management of hypercalcemia of malignancy includes fluid replacement with normal saline to
correct the volume depletion that is invariably present and to enhance renal calcium excretion. The use
of loop diuretics such as furosemide should be restricted to patients in danger of fluid overload, since these
drugs can aggravate volume depletion and are not very effective alone in promoting renal calcium
excretion. Although intravenous pamidronate has become the mainstay of treatment for the hypercalcemia
of malignancy, it is considered only after the hypercalcemic patient has been rendered euvolemic by saline
repletion. The same is true for the other calcium-lowering agents listed.

Ref: Behl D, Hendrickson AW, Moynihan TJ: Oncologic emergencies. Crit Care Clin 2010;26(1):181-205.

8
Item 24

ANSWER: A

In the symptomatic patient with uterine fibroids unresponsive to medical therapy, myomectomy is
recommended over fibroid embolization for patients who wish to become pregnant in the future. Uterine
fibroid embolization requires a shorter hospitalization and less time off work. General anesthesia is not
required, and a blood transfusion is unlikely to be needed. Uterine fibroids can recur or develop after
either myomectomy or embolization.

Ref: Schorge JO, Schaffer JI, Halvorsen LM, et al (eds): Williams Gynecology. McGraw-Hill, 2008, pp 205-207. 2) Bradley
LD: Uterine fibroid embolization: A viable alternative to hysterectomy. Am J Obstet Gynecol 2009;201(2):127-135.
3) Goodwin SC, Spies JB: Uterine fibroid embolization. N Engl J Med 2009;361(7):690-697.

Item 25

ANSWER: B

This patient has melanosis coli, which is a benign condition resulting from abuse of anthraquinone laxatives
such as cascara, senna, or aloe. The condition resolves with discontinuation of the medication.

Ref: Rakel RE, Bope ET, Kellerman R (eds): Conns Current Therapy 2010. Saunders Elsevier, 2010, p 19.

Item 26

ANSWER: D

The Valsalva maneuver decreases venous return to the heart, thereby decreasing cardiac output. This
causes most murmurs to decrease in length and intensity. The murmur of hypertrophic obstructive
cardiomyopathy, however, increases in loudness. The murmur of mitral valve prolapse becomes longer,
and may also become louder.

Ref: Bonow RO, Mann DL, Zipes DP, Libby P (eds): Braunwalds Heart Disease: A Textbook of Cardiovascular Medicine, ed
9. Elsevier Saunders, 2011, pp 117-118.

Item 27

ANSWER: D

Slipped capital femoral epiphysis is the most common hip disorder in this patients age group. It usually
occurs between the ages of 8 and 15 and is more common in boys and overweight or obese children. It
presents with limping and pain, and limited internal rotation of the hip is noted on physical examination.

Septic arthritis would typically present with a fever. Juvenile rheumatoid arthritis, transient synovitis, and
Legg-Calv-Perthes disease are more common in younger children.

Ref: Peck D: Slipped capital femoral epiphysis: Diagnosis and management. Am Fam Physician 2010;82(3):258-262.

9
Item 28

ANSWER: B

When a patient or nursing-home resident is losing weight or has suffered an acute change in the ability to
perform activities of daily living, a decision must be made as to whether or not to place a PEG tube to
provide artificial nutrition. Studies have shown that PEG tubes do not improve nutritional status or quality
of life for residents with dementia, nor do they decrease the risk of aspiration pneumonia, although
aspiration risk may possibly be decreased if the feeding tube is placed below the gastroduodenal junction
(SOR B). Feeding tubes can also cause discomfort and agitation, leading to an increased use of restraints
(SOR B).
Ref: Unwin BK, Porvaznik M: Nursing home care: Part II. Clinical aspects. Am Fam Physician 2010;81(10):1229-1237.

Item 29

ANSWER: D

It is important to distinguish between COPD and asthma because of the differences in treatment. Patients
with COPD are usually in their sixties when the diagnosis is made. Symptoms of chronic cough (sometimes
for months or years), dyspnea, or sputum production are often not reported because the patient may
attribute them to smoking, aging, or poor physical condition.

Spirometry is the best test for the diagnosis of COPD. The pressure of outflow obstruction that is not fully
reversible is demonstrated by postbronchodilator spirometry showing an FEV1/FVC ratio of 70% or less.
Ref: Price DB, Yawn BP, Jones RCM: Improving the differential diagnosis of chronic obstructive pulmonary disease in primary
care. Mayo Clin Proc 2010;85(12):1122-1129. 2) Dewar M, Curry RW Jr: Chronic obstructive pulmonary disease:
Diagnostic considerations. Am Fam Physician 2006;73(4):669-676.

Item 30

ANSWER: D

Pain with eye movement suggests an orbital condition. Orbital inflammation, infection, or tumor invasion
can lead to such eye pain. Other findings suggestive of an orbital cause of eye pain include diplopia or
proptosis. If an orbital lesion is suspected, imaging studies should be performed.
Ref: Fiore DC, Pasternak AV, Radwan RM: Pain in the quiet (not red) eye. Am Fam Physician 2010;82(1):69-73.

Item 31

ANSWER: C

Patients should be risk-stratified according to their family history. Patients who have one first degree
relative diagnosed with colorectal cancer or adenomatous polyps before age 60, or at least two second
degree relatives with colorectal cancer, are in the highest risk group. They should start colon cancer
screening at age 40, or 10 years before the earliest age at which an affected relative was diagnosed
(whichever comes first) and be rescreened every 5 years. Colonoscopy is the preferred screening method
for this highest-risk group, as high-risk patients are more likely to have right-sided colon lesions that would
not be detected with sigmoidoscopy.
Ref: Wiltz SA, Nelson RM: Whats the most effective way to screen patients with a family history of colon cancer? J Fam Pract
2010;59(3):176-178.

10
Item 32

ANSWER: A

Drug-induced pleuritis is one cause of pleurisy. Several drugs are associated with drug-induced pleural
disease or drug-induced lupus pleuritis. Drugs that may cause lupus pleuritis include hydralazine,
procainamide, and quinidine. Other drugs known to cause pleural disease include amiodarone, bleomycin,
bromocriptine, cyclophosphamide, methotrexate, minoxidil, and mitomycin.

Ref: Kass SM, Williams PM, Reamy BV: Pleurisy. Am Fam Physician 2007;75(9):1357-1364.

Item 33

ANSWER: C

Adenosine, digoxin, and calcium channel antagonists act by blocking conduction through the
atrioventricular (AV) node, which may increase the ventricular rate paradoxically, initiating ventricular
fibrillation. These agents should be avoided in Wolff-Parkinson-White syndrome. Procainamide is usually
the treatment of choice in these situations, although amiodarone may also be used.

Ref: Blomstrm-Lundqvist C, Scheinman MM, Aliot EM, et al: ACC/AHA/ESC guidelines for the management of patients with
supraventricular arrhythmiasExecutive summary. A report of the American College of Cardiology/American Heart
Association Task Force on Practice Guidelines and the European Society of Cardiology Committee for Practice Guidelines
(Writing Committee to Develop Guidelines for the Management of Patients with Supraventricular Arrhythmias) developed
in collaboration with NASPE-Heart Rhythm Society. J Am Coll Cardiol 2003;42(8):1493-1531. 2) Bonow RO, Mann DL,
Zipes DP, Libby P (eds): Braunwalds Heart Disease: A Textbook of Cardiovascular Medicine, ed 9. Elsevier Saunders,
2011, pp 787, 795, 798-801.

Item 34

ANSWER: E

Doxycycline is contraindicated in the second and third trimesters of pregnancy due to the risk of permanent
discoloration of tooth enamel in the fetus. Cephalosporins such as ceftriaxone are usually considered safe
to use during pregnancy. The use of ciprofloxacin during pregnancy does not appear to increase the risk
of major congenital malformation, nor does the use of amoxicillin. Animal studies using rats and mice
treated with daily doses of azithromycin up to maternally toxic levels revealed no impairment of fertility
or harm to the fetus.

Ref: Briggs GG, Freeman RK, Yaffe SJ (eds): Drugs in Pregnancy and Lactation, ed 8. Lippincott Williams & Wilkins, 2008,
pp 86, 158, 292-293, 361, 364, 579.

Item 35

ANSWER: C

There is not a direct relation between daily calorie consumption and weight. An adult male consuming an
extra 100 calories a day above his caloric need will not continue to gain weight indefinitely; rather, his
weight will increase to a certain point and then become constant. Fat must be fed, and maintaining the
newly created tissue requires an increase in caloric expenditure. An extra 100 calories a day will result
in a weight gain of approximately 5 kg, which will then be maintained.

Ref: Katan MB, Ludwig DS: Extra calories cause weight gainBut how much? JAMA 2010;303(1):65-66.

11
Item 36

ANSWER: C

Inhaled corticosteroids improve asthma control in adults and children more effectively than any other single
long-term controller medication, and all patients should also receive a prescription for a short-acting
!-agonist (SOR A).

Ref: Pollart SM, Elward KS: Overview of changes to asthma guidelines: Diagnosis and screening. Am Fam Physician
2009;79(9):761-767.

Item 37

ANSWER: C

Tumor necrosis factor inhibitors have been associated with an increased risk of infections, including
tuberculosis. This class of agents includes monoclonal antibodies such as infliximab, adalimumab,
certolizumab pegol, and golimumab. Patients should be screened for tuberculosis and hepatitis B and C
before starting these drugs.

The other drugs listed can have adverse effects, but do not increase the risk for tuberculosis.

Ref: Scott DL, Wolfe F, Huizinga TW: Rheumatoid arthritis. Lancet 2010;376(9746):1094-1108.

Item 38

ANSWER: D

The most widely used initial assay for detecting hepatitis C virus (HCV) antibody is the enzyme
immunoassay. A positive enzyme immunoassay should be followed by a confirmatory test such as the
recombinant immunoblot assay. If negative, it indicates a false-positive antibody test. If positive, the
quantitative HCV RNA polymerase chain reaction is used to measure the amount of virus in the blood to
distinguish active from resolved HCV infection. In this case, the results of the test indicate that the patient
had a past infection with HCV that is now resolved.

Ref: Wilkins T, Malcolm JK, Raina D, Schade RR: Hepatitis C: Diagnosis and treatment. Am Fam Physician
2010;81(11):1351-1357.

Item 39

ANSWER: D

All staff, visitors, and nursing-home residents should observe strict handwashing practices in this situation.
Barrier precautions for wounds and medical devices should also be initiated. Surveillance cultures are not
warranted. Aggressive housekeeping practices play little, if any, role in preventing the spread of MRSA.
Isolating the patient is not practical or cost-effective.

Ref: Unwin BK, Porvaznik M, Spoelhof GD: Nursing home care: Part I. Principles and pitfalls of practice. Am Fam Physician
2010;81(10):1219-1227.

12
Item 40

ANSWER: C

Since even a temporary reduction in renal function, such as occurs after pyelography or angiography, can
cause lactic acidosis in patients taking metformin, the drug should be discontinued 48 hours before such
procedures (SOR C) and restarted 48 hours after the procedure if renal function is normal. The other
procedures listed are not indications for stopping metformin.

Ref: Stacul F, Adam A, Becker CR, et al: Strategies to reduce the risk of contrast-induced nephropathy. Am J Cardiol
2006;98(suppl 6A):59k-77k.

Item 41

ANSWER: C

The DSM-IV criteria for obsessive-compulsive disorder (OCD) indicate that the patient at some point
recognizes that the obsessions or compulsions are excessive or unreasonable. The impulses of OCD are
not related to excessive worry about ones problems, and the patient recognizes that they are the product
of his or her own mind. In addition, the patient experiences marked distress because of the impulses. Full
remission is rare, but treatment can provide significant relief.

Ref: Fenske JN, Schwenk TL: Obsessive-compulsive disorder: Diagnosis and management. Am Fam Physician
2009;80(3):239-245.

Item 42

ANSWER: E

Nursing homeacquired pneumonia should be suspected in patients with a new infiltrate on a chest
radiograph if it is associated with a fever, leukocytosis, purulent sputum, or hypoxia. Nursing-home
patients who are hospitalized for pneumonia should be started on intravenous antimicrobial therapy, with
empiric coverage for methicillin-resistant Staphylococcus aureus (MRSA) and Pseudomonas aeruginosa.
The 2005 American Thoracic Society/Infectious Diseases Society of America guideline recommends
combination therapy consisting of an antipseudomonal cephalosporin such as cefepime or ceftazidime, an
antipseudomonal carbapenem such as imipenem or meropenem, or an extended-spectrum
!-lactam/!-lactamase inhibitor such as piperacillin/tazobactam, PLUS an antipseudomonal fluoroquinolone
such as levofloxacin or ciprofloxacin, or an aminoglycoside such as gentamicin, tobramycin, or amikacin,
PLUS an anti-MRSA agent (vancomycin or linezolid). Ceftriaxone and azithromycin or levofloxacin alone
would be reasonable treatment options for a patient with nursing homeacquired pneumonia who does not
require hospitalization.

Ref: American Thoracic Society; Infectious Diseases Society of America: Guidelines for the management of adults with
hospital-acquired, ventilator-associated, and healthcare-associated pneumonia. Am J Respir Crit Care Med
2005;171(4):388-416. 2) Mills K, Nelson AC, Winslow BT, Springer KL: Treatment of nursing homeacquired
pneumonia. Am Fam Physician 2009;79(11):976-992.

13
Item 43

ANSWER: D

The use of psychiatric medications during pregnancy should always involve consideration of the potential
risks to the fetus in comparison to the well-being of the mother. Lithium is known to be teratogenic.
Benzodiazepines such as alprazolam are controversial due to a possible link to cleft lip/palate. Studies have
shown no significant risk of congenital anomalies from SSRI use in pregnancy, except for paroxetine.
Paroxetine is a category D medication and should be avoided in pregnant women (SOR B). There is
concern about an increased risk of congenital cardiac malformations from first-trimester exposure.
Bupropion has not been studied extensively for use in pregnancy, and in one published study of 136
patients it was linked to an increased risk of spontaneous abortion.

Ref: ACOG Committee on Practice Bulletins: ACOG Practice Bulletin: Clinical management guidelines for
obstetricians-gynecologists number 92, April 2008 (replaces practice bulletin number 87, November 2007). Use of
psychiatric medications during pregnancy and lactation. Obstet Gynecol 2008;111(4):1001-1020.

Item 44

ANSWER: D

This is a classic presentation for intussusception, which usually occurs in children under the age of 2 years
and is characterized by paroxysms of colicky abdominal pain. A mass is palpable in about two-thirds of
patients.

Pyloric stenosis presents with a palpable mass, but usually develops between 4 and 6 weeks of age. A
choledochal cyst presents with the classic triad of right upper quadrant pain, jaundice, and a palpable mass.
Meckels diverticulum usually presents in this age group with painless lower gastrointestinal bleeding.
Intestinal malrotation usually presents within the first 4 weeks of life and is characterized by bilious
vomiting.

Ref: Kliegman RM, Stanton BF, Geme JW III, et al (eds): Nelson Textbook of Pediatrics, ed 19. Elsevier Saunders, 2011, pp
1287-1289.

Item 45

ANSWER: C

Teriparatide is indicated for the treatment of severe osteoporosis, for patients with multiple osteoporosis
risk factors, or for patients with failure of bisphosphonate therapy (SOR B). Therapy with teriparatide is
currently limited to 2 years and is contraindicated in patients with a history of bone malignancy, Paget
disease, hypercalcemia, or previous treatment with skeletal radiation. Its route of administration
(subcutaneous) and high cost should be considered when prescribing teriparatide therapy. Testosterone
therapy is contraindicated in patients with a history of prostate cancer. Zoledronic acid is a parenterally
administered bisphosphonate and would not be appropriate in a patient who has already failed
bisphosphonate therapy. Likewise, raloxifene and calcitonin are not indicated in patients with severe
osteoporosis who have failed bisphosphonate therapy.

Ref: Rao S, Budhwar N, Ashfaque A: Osteoporosis in men. Am Fam Physician 2010;82(5):503-508.

14
Item 46

ANSWER: B

Women who present with symptoms of acute dysuria, frequency, and pyuria do not always have bacterial
cystitis. In fact, up to 30% will show either no growth or insignificant bacterial growth on a midstream
urine culture. Most commonly these patients represent cases of sexually transmitted urethritis caused by
Chlamydia trachomatis, Neisseria gonorrhoeae, or herpes simplex virus.

In this case, the gradual onset, absence of hematuria, and week-long duration of symptoms suggest a
sexually transmitted disease. A history of a new sexual partner or a finding of mucopurulent cervicitis
would confirm the diagnosis. Empiric treatment with a tetracycline and a search for other sexually
transmitted diseases would then be indicated.

Another possible diagnosis is urinary tract infection with Escherichia coli or Staphylococcus species;
however, the onset of these infections is usually abrupt and accompanied by other signs, such as
suprapubic pain or hematuria. Candida is unlikely because there is no accompanying discharge or itching,
and the patients symptoms predate the use of antibiotics.

Ref: Berek JS (ed): Berek & Novaks Gynecology, ed 14. Lippincott Williams & Wilkins, 2007, pp 556-557. 2) Rakel RE, Bope
ET, Kellerman R (eds): Conns Current Therapy 2010. Saunders Elsevier, 2010, pp 1251-1255.

Item 47

ANSWER: A

Gait and balance disorders are one of the most common causes of falls in older adults. Correctly
identifying gait and balance disorders helps guide management and may prevent consequences such as
injury, disability, loss of independence, or decreased quality of life. The Timed Up and Go test is a
reliable diagnostic tool for gait and balance disorders and is quick to administer. A time of <10 seconds
is considered normal, a time of >14 seconds is associated with an increased risk of falls, and a time of
>20 seconds usually suggests severe gait impairment.

This patient has the cautious gait associated with visual impairment. It is characterized by abducted arms
and legs; slow, careful, walking on ice movements; a wide-based stance; and en bloc turns. Patients
with cerebellar degeneration have an ataxic gait that is wide-based and staggering. Frontal lobe
degeneration is associated with gait apraxia that is described as magnetic, with start and turn hesitation
and freezing. Parkinsons disease patients have a typical gait that is short-stepped and shuffling, with hips,
knees, and spine flexed, and may also exhibit festination and en bloc turns. Motor neuropathy causes
a steppage gait resulting from foot drop with excessive flexion of the hips and knees when walking, short
strides, a slapping quality, and frequent tripping.

Ref: Salzman B: Gait and balance disorders in older adults. Am Fam Physician 2010;82(1):61-68.

15
Item 48

ANSWER: A

Family-based treatment for the adolescent with anorexia nervosa has been found to provide superior results
when compared with individual adolescent-focused therapy (SOR B). Antidepressants have not been
successful. They may be indicated for coexisting conditions, but this is more common with bulimia.

Ref: Walsh BT, Kaplan AS, Attia E, et al: Fluoxetine after weight restoration in anorexia nervosa: A randomized controlled
trial. JAMA 2006;295(22):2605-2612. 2) Goroll AH, Mulley AG (eds): Primary Care Medicine: Office Evaluation and
Management of the Adult Patient, ed 6. Lippincott Williams & Wilkins, 2009, pp 1507-1513. 3) Rosen DS, American
Academy of Pediatrics Committee on Adolescence: Identifying and treating eating disorders. Pediatrics
2010;126(6):1240-1253. 4) Lock J, Grange DL, Agras WS, et al: Randomized clinical trial comparing family-based
treatment with adolescent-focused individual therapy for adolescents with anorexia nervosa. Arch Gen Psychiatry
2010;67(10):1025-1032.

Item 49

ANSWER: A

Influenza is a highly contagious viral illness spread by airborne droplets. This patients symptoms are
highly suggestive of typical influenza: a sudden onset of malaise, myalgia, headache, fever, rhinitis, sore
throat, and cough. While influenza is typically uncomplicated and self-limited, it can result in severe
complications, including encephalitis, pneumonia, respiratory failure, and death.

The effectiveness of treatment for influenza is dependent on how early in the course of the illness it is
given. Because of the recent global H1N1 influenza outbreak that resulted in demand potentially
outstripping the supply of antiviral medication, the Centers for Disease Control and Prevention has
modified its recommendation as follows:

Antiviral treatment is recommended as soon as possible for patients with confirmed or suspected
influenza who have severe, complicated, or progressive illness or who require hospitalization.
Antiviral treatment is recommended as soon as possible for outpatients with confirmed or
suspected influenza who are at higher risk for influenza complications based on their age or
underlying medical conditions. Clinical judgment should be an important component of outpatient
treatment decisions.
Antiviral treatment also may be considered on the basis of clinical judgment for any outpatient with
confirmed or suspected influenza who does not have known risk factors for severe illness, if
treatment can be initiated within 48 hours of illness onset.

Many rapid influenza tests produce false-negative results, and more accurate assays can take more than
24 hours. Thus, treatment of patients with a clinical picture suggesting influenza is recommended, even
if a rapid test is negative. Delaying treatment until further test results are available is not recommended.

Ref: Fiore AE, Fry A, Shay D, et al; Centers for Disease Control and Prevention (CDC): Antiviral agents for the treatment and
chemoprophylaxis of influenzaRecommendations of the Advisory Committee on Immunization Practices (ACIP). MMWR
Recomm Rep 2011;60(1):1-24.

16
Item 50

ANSWER: B

The management of somatizing patients can be difficult. One strategy that has been shown to be effective
is to schedule regular office visits so that the patient does not need to develop new symptoms in order to
receive medical attention. Regular visits have been shown to significantly reduce the cost and chaos of
caring for patients with somatization disorder and to help progressively diminish emergency visits and
telephone calls. In addition, it is important to describe the patients diagnosis with compassion and avoid
suggesting that its all in your head.

Continued diagnostic testing and referrals in the absence of new symptoms or findings is unwarranted.
Visits to the emergency department often result in inconsistent care and mixed messages from physicians
who are seeing the patient for the first time, and unnecessary and often repetitive tests may be ordered.
Opiates have significant side effects such as constipation, sedation, impaired cognition, and risk of
addiction.

Ref: Servan-Schreiber D, Tabas G, Kolb R: Somatizing patients: Part II. Practical management. Am Fam Physician
2000;61(5):1423-1428, 1431-1432. 2) Hull SK, Broquet K: How to management difficult patient encounters. Fam Pract
Manag 2007;14(6):30-34.

Item 51

ANSWER: B

The U.S. Preventive Services Task Force (USPSTF) has summarized the evidence for the use of aspirin
in the primary prevention of cardiovascular disease as follows:

The USPSTF recommends the use of aspirin for men 4579 years of age when the potential benefit
from a reduction in myocardial infarctions outweighs the potential harm from an increase in
gastrointestinal hemorrhage (Grade A recommendation)
The USPSTF recommends the use of aspirin for women 5579 years of age when the potential
benefit of a reduction in ischemic strokes outweighs the potential harm of an increase in
gastrointestinal hemorrhage (Grade A recommendation)
The USPSTF concludes that the current evidence is insufficient to assess the balance of benefits
and harms of aspirin for cardiovascular disease prevention in men and women 80 years of age or
older (Grade I statement)
The USPSTF recommends against the use of aspirin for stroke prevention in women younger than
55 and for myocardial infarction prevention in men younger than 45 (Grade D recommendation)

In summary, consistent evidence from randomized clinical trials indicates that aspirin use reduces the risk
for cardiovascular disease events in adults without a history of cardiovascular disease. It reduces the risk
for myocardial infarction in men, and ischemic stroke in women. Consistent evidence shows that aspirin
use increases the risk for gastrointestinal bleeding, and limited evidence shows that aspirin use increases
the risk for hemorrhagic strokes. The overall benefit in the reduction of cardiovascular disease events with
aspirin use depends on baseline risk and the risk for gastrointestinal bleeding.

Ref: US Preventive Services Task Force: Aspirin for the primary prevention of cardiovascular events: An update of the evidence.
AHRQ pub no 09-05129-EF-4, 2009. 2) Aspirin for the Prevention of Cardiovascular Disease. US Preventive Services
Task Force, 2009.

17
Item 52

ANSWER: D

Iron deficiency is almost certainly the diagnosis in this child. The patients response to a therapeutic trial
of iron would be most helpful in establishing the diagnosis. Additional tests might be necessary if there
is no response.

Ref: Kliegman RM, Stanton BF, Geme JW III, et al (eds): Nelson Textbook of Pediatrics, ed 19. Elsevier Saunders, 2011, pp
1655-1658.

Item 53

ANSWER: D

Because of the risk of asthma exacerbation or asthma-related death, the FDA has added a warning against
the use of long-acting !2-agonists as monotherapy. Inhaled corticosteroids, leukotriene-receptor
antagonists, short-acting !2-agonists, and mast-cell stabilizers are approved and accepted for both
monotherapy and combination therapy in the management of asthma (SOR A).

Ref: Elward KS, Pollart SM: Medical therapy for asthma: Updates from the NAEPP guidelines. Am Fam Physician
2010;82(10):1242-1251.

Item 54

ANSWER: E

The diagnosis of inflammatory bowel disease (IBD) can be elusive but relies primarily on the patient
history, laboratory findings, and endoscopy (or double-contrast radiographs if endoscopy is not available).
Endoscopy is usually reserved for patients with more severe symptoms or in whom preliminary testing
shows the potential for significant inflammation. It is recommended that this preliminary evaluation include
a WBC count, platelet count, potassium level, and erythrocyte sedimentation rate.

Patients who have minimal symptoms and normal preliminary testing likely do not have a significant case
of IBD. Plain radiographs and CT of the abdomen may help rule out other etiologies but are not considered
adequate to diagnose or exclude IBD. Panels of serologic blood tests have recently been developed and are
being assessed as to their place in evaluating patients who may have IBD. However, this testing is
expensive, lacks sufficient predictive value, and has yet to prove its utility compared to standard testing.

Ref: Langan RC, Gotsch PB, Krafczyk MA, Skillinge DD: Ulcerative colitis: Diagnosis and treatment. Am Fam Physician
2007;76(9):1323-1330. 2) Benor S, Russell GH, Silver M, et al: Shortcomings of the Inflammatory Bowel Disease
Serology 7 panel. Pediatrics 2010;125(6):1230-1236.

18
Item 55

ANSWER: D

This infant has a typical presentation of erythema toxicum neonatorum. Staphylococcal pyoderma is
vesicular and the stain of the vesicle content shows polymorphonuclear leukocytes and clusters of
gram-positive bacteria. Because the mother is healthy and the infant shows no evidence of being otherwise
ill, systemic infections such as herpes are unlikely. Acne neonatorum consists of closed comedones on the
forehead, nose, and cheeks. Rocky Mountain spotted fever is a tickborne disease that does not need to be
considered in a child who is not at risk.

Ref: OConnor NR, McLaughlin MR, Ham P: Newborn skin: Part I. Common rashes. Am Fam Physician 2008;77(1):49-52.

Item 56

ANSWER: E

Patients with chronic kidney disease (CKD) and those at risk for CKD because of conditions such as
hypertension and diabetes have an increased risk of deterioration in renal function from NSAID use.
NSAIDs induce renal injury by acutely reducing renal blood flow and, in some patients, by causing
interstitial nephritis. Because many of these drugs are available over the counter, patients often assume
they are safe for anyone. Physicians should counsel all patients with CKD, as well as those at increased
risk for CKD, to avoid NSAIDs.

ACE inhibitors and angiotensin II receptor blockers are renoprotective and their use is recommended in
all diabetics. The use of low-dose aspirin and folic acid is recommended in all patients with diabetes, due
to the vasculoprotective properties of these drugs. High-dose aspirin should be avoided because it acts as
an NSAID.

Ref: Fauci AS, Braunwald E, Kasper DL, et al (eds): Harrisons Principles of Internal Medicine, ed 17. McGraw-Hill, 2008,
p 1771.

Item 57

ANSWER: B

The primary treatment for symptomatic mitral valve prolapse is !-blockers. Quinidine and digoxin were
used to treat this problem in the past, especially if sinus bradycardia or cardiac arrest occurred with
administration of propranolol. Procainamide and phenytoin have not been used to treat this syndrome.
Asymptomatic patients require only routine monitoring, while those with significant mitral regurgitation
may require surgery. Some patients with palpitations can be managed with lifestyle changes such as
elimination of caffeine and alcohol. Orthostatic hypotension can often be managed with volume expansion,
such as by increasing salt intake.

Ref: American College of Cardiology/American Heart Association Task Force on Practice Guidelines; Society of Cardiovascular
Anesthesiologists; Society for Cardiovascular Angiography and Intervention; Society of Thoracic Surgeons; et al:
ACC/AHA 2006 guidelines for the management of patients with valvular heart disease: A report of the American College
of Cardiology/American Heart Association Task Force on Practice Guidelines (Writing Committee to Revise the 1998
Guidelines for the Management of Patients With Valvular Heart Disease): Developed in collaboration with the Society of
Cardiovascular Anesthesiologists: Endorsed by the Society for Cardiovascular Angiography and Interventions and the
Society of Thoracic Surgeons. Circulation 2006;114(5):e84-e231. 2) Bonow RO, Mann DL, Zipes DP, Libby P (eds):
Braunwalds Heart Disease: A Textbook of Cardiovascular Medicine, ed 9. Elsevier Saunders, 2011, pp 1510-1514.

19
Item 58

ANSWER: D

Altitude illness is common, affecting 25%85% of travelers to high altitudes. The most common
manifestation is acute mountain sickness, heralded by malaise and headache. Risk factors include young
age, residence at a low altitude, rapid ascent, strenuous physical exertion, and a previous history of altitude
illness. However, activity restriction is not necessary for patients with coronary artery disease who are
traveling to high altitudes (SOR C).

Ginkgo biloba has been evaluated for both prevention and treatment of acute mountain sickness and
high-altitude cerebral edema, and it is not recommended. Acetazolamide is an effective prophylactic agent
(SOR B), but is contraindicated in patients with a sulfa allergy. If used, it should be started a minimum
of one day before ascent and continued until the patient acclimatizes at the highest planned elevation.
Dexamethasone is an effective prophylactic and treatment agent (SOR B), and it is not contraindicated for
those with a sulfa allergy. It would be the best option for this patient.

Ref: Dumont L, Mardirosoff C, Tramr MR: Efficacy and harm of pharmacological prevention of acute mountain sickness:
Quantitative systematic review. BMJ 2000;321(7256):267-272. 2) Murdoch D: Altitude sickness. Clin Evid 2005;(14):
1570-1575. 3) Fiore DC, Hall S, Shoja P: Altitude illness: Risk factors, prevention, presentation, and treatment. Am Fam
Physician 2010;82(9):1103-1110.

Item 59

ANSWER: E

Antibiotics, especially penicillins, cephalosporins, and sulfonamides, are the most common drug-related
cause of acute interstitial nephritis. Corticosteroids may be useful for treating this condition. The other
drugs listed may cause renal injury, but not acute interstitial nephritis.

Ref: Abdel-Kader K, Palevsky P: Acute kidney injury in the elderly. Clin Geriatr Med 2009;25(3):331-358.

Item 60

ANSWER: A

Horse chestnut seed extract has been shown to have some effect when used orally for symptomatic
treatment of chronic venous insufficiency, such as varicose veins. It may also be useful for relieving pain,
tiredness, tension, and swelling in the legs. It contains a number of anti-inflammatory substances, including
escin, which reduces edema and lowers fluid exudation by decreasing vascular permeability. Milk thistle
may be effective for hepatic cirrhosis. Ephedra is considered unsafe, as it can cause severe life-threatening
or disabling adverse effects in some people. St. Johns wort may be effective for treating mild to moderate
depression. Vitamin B12 is used to treat pernicious anemia.

Ref: Jones RH, Carek PJ: Management of varicose veins. Am Fam Physician 2008;78(11):1289-1294. 2) Pittler MH, Ernst E:
Horse chestnut seed extract for chronic venous insufficiency. Cochrane Database Syst Rev 2006;(1):CD003230.

20
Item 61

ANSWER: D

Breastfeeding provides such optimal nutrition for an infant that the benefits still far outweigh the risks even
when the mother smokes tobacco, tests positive for hepatitis B or C virus, or develops a simple
undifferentiated fever. Maternal seropositivity to cytomegalovirus (CMV) is not considered a
contraindication except when it has a recent onset or in mothers of low birthweight infants. When present,
the CMV load can be substantially reduced by freezing and pasteurization of the milk. All patients who
smoke should be strongly encouraged to discontinue use of tobacco, particularly in the presence of infants,
but smoking is not a contraindication to breastfeeding.

Mothers with active herpes simplex lesions on a breast should not feed their infant from the infected breast,
but may do so from the other breast if it is not infected. Breastfeeding is also contraindicated in the
presence of active maternal tuberculosis, and following administration or use of radioactive isotopes,
chemotherapeutic agents, recreational drugs, or certain prescription drugs.

Ref: American Academy of Pediatrics Section on Breastfeeding: Policy Statement. Breastfeeding and the use of human milk.
Pediatrics 2005;115(2):496-506.

Item 62

ANSWER: B

Erythema multiforme usually occurs in adults 2040 years of age, although it can occur in patients of all
ages. Herpes simplex virus (HSV) is the most commonly identified cause of this hypersensitivity reaction,
accounting for more than 50% of cases.

Ref: Lamoreux MR, Sternbach MR, Hsu WT: Erythema multiforme. Am Fam Physician 2006;74(11):1883-1888.

Item 63

ANSWER: A

This patient has pyogenic tenosynovitis. When early tenosynovitis (within 48 hours of onset) is suspected,
treatment with antibiotics and splinting may prevent the spread of the infection. However, this patients
infection is no longer in the early stages and is more severe, so it requires surgical drainage and antibiotics.
A delay in treatment of these infections can lead to ischemia of the tendons and damage to the flexor
tendon and sheath. This can lead to impaired function of the finger. Needle aspiration would not
adequately drain the infection. Antiviral medication would not be appropriate, as this is a bacterial
infection. Corticosteroid injections are contraindicated in the presence of infection.

Ref: Canale ST, Beaty JH (eds): Campbells Operative Orthopaedics, ed 11. Mosby Elsevier, 2007, pp 4351-4354.

21
Item 64

ANSWER: D

This geriatric diabetic patient should be treated with insulin. Metformin is contraindicated in patients with
renal insufficiency. Sitagliptin should not be added to a sulfonylurea drug initially, the dosage should be
lowered in patients with renal insufficiency, and given alone it would probably not result in reasonable
diabetic control. Pioglitazone can cause fluid retention and therefore would not be a good choice for a
patient with cardiomyopathy.

Ref: Rodbard HW, Jellinger PS, Davidson JA, et al: Statement by an American Association of Clinical
Endocrinologists/American College of Endocrinology consensus panel on type 2 diabetes mellitus: An algorithm for
glycemic control. Endocr Pract 2009;15(6):540-559. 2) Marcus A: Diabetes careInsulin delivery in a changing world.
Medscape J Med 2008;10(5):120.

Item 65

ANSWER: D

Only symptomatic treatment is indicated for ciguatera poisoning, as there is no specific treatment. The
same is true for shellfish poisoning, although potential respiratory distress or failure must be kept in mind.

Scombroid poisoning is a pseudoallergic condition resulting from consumption of improperly stored


scombroid fish such as tuna, mackerel, wahoo, and bonito. Nonscombroid varieties such as mahi-mahi,
amberjack, sardines, and herring can also cause this problem. The poisoning is due to high levels of
histamine and saurine resulting from bacterial catabolism of histidine. Symptoms occur within minutes to
hours, and include flushing of the skin, oral paresthesias, pruritus, urticaria, nausea, vomiting, diarrhea,
vertigo, headache, bronchospasm, dysphagia, tachycardia, and hypotension. Therapy should be the same
as for allergic reactions and anaphylaxis, and will usually lead to resolution of symptoms within several
hours.

Ref: Goldman L, Ausiello D (eds): Cecil Medicine, ed 23. Saunders, 2008, pp 2450-2451.

Item 66

ANSWER: C

Postural orthostatic tachycardia syndrome (POTS) is manifested by a rise in heart rate >30 beats/min or
by a heart rate >120 beats/min within 10 minutes of being in the upright position. Symptoms usually
include position-dependent headaches, abdominal pain, lightheadedness, palpitations, sweating, and
nausea. Most patients will not actually pass out, but some will if they are unable to lie down quickly
enough. This condition is most prevalent in white females between the ages of 15 and 50 years old. Often
these patients are hardworking, athletic, and otherwise in good health.

There is a high clinical correlation between POTS and chronic fatigue syndrome. Although no single
etiology for POTS has been found, the condition is thought to have a genetic predisposition, is often incited
after a prolonged viral illness, and has a component of deconditioning. The recommended initial
management is encouraging adequate fluid and salt intake, followed by the initiation of regular aerobic
exercise combined with lower-extremity strength training, and then the use of !-blockers.

22
Ref: Post RE, Dickerson LM: Dizziness: A diagnostic approach. Am Fam Physician 2010;82(4):361-368, 369. 2) Johnson JN,
Mack KJ, Kuntz NL, et al: Postural orthostatic tachycardia syndrome: A clinical review. Pediatr Neurol 2010;42(2):77-85.

Item 67

ANSWER: A

Cigarette smokers are five times more likely than nonsmokers to develop an abdominal aortic aneurysm
(AAA). The risk is associated with the number of years the patient has smoked, and declines with
cessation. Diabetes mellitus is protective, decreasing the risk of AAA by half. Women tend to develop
AAA in their sixties, 10 years later than men. Whites are at greater risk than African-Americans.
Hypertension is less of a risk factor than cigarette smoking (SOR A).

Ref: Schermerhorn M: A 66-year-old man with an abdominal aortic aneurysm. JAMA 2009;302(18):2015-2022.

Item 68

ANSWER: D

Subclinical hypothyroidism is defined as slightly elevated TSH (approximately 510 mIU/L) and normal
levels of thyroid hormone (free T4 or free T3) in an asymptomatic patient. There is a low rate of
progression to overt hypothyroidism manifested by symptoms, TSH levels >10 mIU/L, or reduced levels
of thyroid hormone.

Recent studies have shown that there is an increased risk for cardiovascular morbidity and mortality in
those with subclinical hypothyroidism. However, treatment with thyroid replacement hormone did not seem
to affect this risk. The decision about whether to recommend thyroid replacement therapy to patients like
the one described here should be individualized. An alternative to treating the patient with medication at
this time would be to retest her TSH annually, or sooner if she becomes symptomatic.

Ref: Hennessey JV, Scherger JE: Evaluating and treating the patient with hypothyroid disease. J Fam Pract 2007;56(8):S31-S39.
2) Vaidya B, Pearce SHS: Management of hypothyroidism in adults. BMJ 2008;337:284-289.

Item 69

ANSWER: C

Dermatophyte infections caused by aerobic fungi produce infections in many areas. Tinea capitis requires
systemic therapy to penetrate the affected hair shafts. Tinea cruris and tinea pedis rarely require systemic
therapy. Extensive outbreaks of tinea corporis and tinea versicolor benefit from both oral and topical
treatment (SOR A), but more localized infections require only topical treatment.

Ref: Andrews MD, Burns M: Common tinea infections in children. Am Fam Physician 2008;77(10):1415-1420.

23
Item 70

ANSWER: C

The ASC-US/LSIL Triage Study (ALTS) demonstrated that there are three appropriate follow-up options
for managing women with an ASC-US Papanicolaou (Pap) test result: (1) two repeat cytologic
examinations performed at 6-month intervals; (2) reflex testing for HPV; or (3) a single colposcopic
examination. This expert consensus recommendation has been confirmed in more recent clinical studies,
additional analyses of the ALTS data, and meta-analyses of published studies (SOR A).

Reflex HPV testing refers to testing either the original liquid-based cytology residual specimen or a
separate sample collected for HPV testing at the time of the initial screening visit. This approach eliminates
the need for women to return to the office or clinic for repeat testing, rapidly reassures women who do not
have a significant lesion, spares 40%60% of women from undergoing colposcopy, and has been shown
to have a favorable cost-effectiveness ratio. In this patients case, the HPV testing was negative, and there
is no need to repeat the Pap test at 6-month intervals or to perform colposcopy.

Although women in certain low-risk groups need routine cervical cancer screening only every 3 years, this
patient should have a repeat Pap test in 12 months. Immediately repeating the test or testing at 3-month
intervals is not recommended in any of the algorithms to manage ASC-US results for otherwise healthy
women.

Ref: ASCUS-LSIL Triage Study (ALTS) Group: Results of a randomized trial on the management of cytology interpretations
of atypical squamous cells of undetermined significance. Am J Obstet Gynecol 2003;188(6):1383-1392. 2) Wright TC Jr,
Massad LS, Dunton CJ, et al: 2006 consensus guidelines for the management of women with abnormal cervical cancer
screening tests. Am J Obstet Gynecol 2007;197(4):346-355.

Item 71

ANSWER: D

Family physicians are often asked to perform a preoperative evaluation prior to noncardiac surgery. This
requires an assessment of the perioperative cardiovascular risk of the procedure involved, the functional
status of the patient, and clinical factors that can increase the risk, such as diabetes mellitus, stroke, renal
insufficiency, compensated or prior heart failure, mild angina, or previous myocardial infarction.

This patient is not undergoing emergency surgery, nor does she have an active cardiac condition; however,
she is undergoing a high-risk procedure (>5% risk of perioperative myocardial infarction) with vascular
surgery. As she cannot climb a flight of stairs or do heavy housework, her functional status is <4 METs,
and she should be considered for further evaluation. The patients diabetes is an additional clinical risk
factor.

With vascular surgery being planned, appropriate recommendations include proceeding with the surgery
with heart rate control, or performing noninvasive testing if it will change the management of the patient.
Coronary angiography is indicated if the noninvasive testing is abnormal. Pulmonary function studies are
most useful in patients with underlying lung disease or those undergoing pulmonary resection. Hemoglobin
A1c is a measure of long-term diabetic control and is not particularly useful perioperatively. Carotid
angiography is not indicated in asymptomatic patients being considered for lower-extremity vascular
procedures.

24
Ref: Fleisher LA, Beckman JA, Brown KA, et al: ACC/AHA 2007 guidelines on perioperative cardiovascular evaluation and
care for noncardiac surgery: A report of the American College of Cardiology/American Heart Association Task Force on
Practice Guidelines (Writing Committee to Revise the 2002 Guidelines on Perioperative Cardiovascular Evaluation for
Noncardiac Surgery): Developed in collaboration with the American Society of Echocardiography, American Society of
Nuclear Cardiology, Heart Rhythm Society, Society of Cardiovascular Anesthesiologists, Society for Cardiovascular
Angiography and Interventions, Society for Vascular Medicine and Biology, and Society for Vascular Surgery. Circulation
2007;116 (17):e418-e499.

Item 72

ANSWER: B

Although proton pump inhibitors are the most effective treatment for patients with asymptomatic
gastroesophageal reflux disease, there are several potential problems with prolonged therapy. Omeprazole
is associated with an increased risk of community-acquired pneumonia and Clostridium difficile colitis.
Omeprazole has also been shown to acutely decrease the absorption of vitamin B12, and it decreases
calcium absorption, leading to an increased risk of hip fracture. The risk for Clostridium difficile colitis
is also increased.

Ref: Marcuard SP, Albernaz L, Khazanie PG: Omeprazole therapy causes malabsorption of cyanocobalamin (vitamin B 12). Ann
Intern Med 1994;120(3):211-215. 2) Dial S, Delaney JAC, Barkun AN, Suissa S: Use of gastric acidsuppressive agents
and the risk of community-acquired Clostridium difficileassociated disease. JAMA 2005;294(23):2989-2995. 3) Smith L:
Practice guidelines. Updated ACG guidelines for diagnosis and treatment of GERD. Am Fam Physician
2005;71(12):2376-2382. 4) Yang Y, Lewis JD, Epstein S, Metz DC: Long-term proton pump inhibitor therapy and risk
of hip fracture. JAMA 2006;296(24):2947-2953. 5) Aseeri M, Schroeder T, Kramer J, Zackula R: Gastric acid suppression
by proton pump inhibitors as a risk factor for Clostridium difficileassociated diarrhea in hospitalized patients. Am J
Gastroenterol 2008;103(9):2308-2313.

Item 73

ANSWER: B

A patient over the age of 35 who experiences abnormal vaginal bleeding must have an endometrial
assessment to exclude endometrial hyperplasia or cancer. An endometrial biopsy is currently the preferred
method for identifying endometrial disease. A laboratory evaluation for thyroid dysfunction or
hemorrhagic diathesis is appropriate if no cancer is present on an endometrial biopsy and medical therapy
fails to halt the bleeding. The other options listed can be used as medical therapy to control the bleeding
once the histopathologic diagnosis has been made.

Ref: Casablanca Y: Management of dysfunctional uterine bleeding. Obstet Gynecol Clin North Am 2008;35(2):219-234.

Item 74

ANSWER: D

Atrial flutter is not ordinarily a serious arrhythmia, but this patient has heart failure manifested by rales,
jugular venous distention, hepatojugular reflux, hypotension, and angina. Electrical cardioversion should
be performed immediately. This is generally a very easy rhythm to convert. Digoxin and verapamil are
appropriate in hemodynamically stable patients. A pacemaker for rapid atrial pacing may be beneficial
if digitalis intoxication is the cause of atrial flutter, but this is unlikely in a patient with no previous history
of cardiac problems. Amiodarone is not indicated in this clinical situation.

25
Ref: Bonow RO, Mann DL, Zipes DP, Libby P (eds): Braunwalds Heart Disease: A Textbook of Cardiovascular Medicine, ed
9. Elsevier Saunders, 2011, pp 771-795.

Item 75

ANSWER: D

This patient has stage 2 hypertension, and his history of stroke is a compelling indication to use specific
classes of antihypertensives. For patients with a history of previous stroke, JNC-7 recommends using
combination therapy with a diuretic and an ACE inhibitor to treat the hypertension, as this combination
has been clinically shown to reduce the risk of recurrent stroke. Other classes of drugs have not been
shown to be of benefit for secondary stroke prevention. Although blood pressure should not be lowered
quickly in the setting of acute ischemic stroke, this patient is not having an acute stroke, so treatment of
his hypertension is warranted.

Ref: Ressel GW; NHLBI: NHLBI releases new high blood pressure guidelines. Am Fam Physician 2003;68(2):376, 379. 2)
Chobanian AV, Bakris GL, Black HR, et al: The Seventh Report of the Joint National Committee on Prevention, Detection,
Evaluation, and Treatment of High Blood Pressure: The JNC 7 report. JAMA 2003;289(19):2560-2572.

Item 76

ANSWER: E

The sympathetic nervous system acts as a positive chronotropic (increases heart rate) and inotropic
(increases contractility) agent. This additional work by the heart will increase metabolic demand and
coronary flow rate. The increased heart rate will decrease the time intervals between electrical events
shown on an EKG.

Ref: Fauci AS, Braunwald E, Kasper DL, et al (eds): Harrisons Principles of Internal Medicine, ed 17. McGraw-Hill, 2008,
pp 1370-1373.

Item 77

ANSWER: A

Chronic plaque psoriasis is the most common type of psoriasis and is characterized by redness, thickness,
and scaling. A variety of treatments were found to be more effective than placebo, but the best results were
produced by topical vitamin D analogues and topical corticosteroids. Vitamin D and high-potency
corticosteroids were equally effective when compared head to head, but the corticosteroids produced fewer
local reactions (SOR A).

Ref: Bailey J, Whitehair B: Cochrane for clinicians. Topical treatments for plaque psoriasis. Am Fam Physician
2010;81(5):596-598.

26
Item 78

ANSWER: A

Up to one-third of patients with unipolar depression will fail to respond to treatment with a single
antidepressant, despite adequate dosing and an appropriate treatment interval. Lithium, triiodothyronine
(T3), and atypical antipsychotics can all provide clinical improvement when used in conjunction with the
ineffective antidepressant. The American Psychiatric Association and the Institute for Clinical Systems
Improvement both recommend a trial of lithium or low-dose T3 for patients who have an incomplete
response to antidepressant therapy. A meta-analysis showed that a serum lithium level !0.5 mEq/L and
a treatment duration of 2 weeks or greater resulted in a good response (SOR A).

While thyroid supplementation as adjunctive therapy is effective, the recommended dosage is no higher
than 50 "g/day (SOR B). Atypical antipsychotics can be used as add-on therapy, but are not as effective
as lithium or T3 (SOR B). Anticonvulsant medications such as gabapentin have been shown to be effective
in the management of bipolar affective disorder, but not as adjunctive therapy in the treatment of unipolar
depression resistant to single-agent antidepressants.

Ref: Chilakamarri G, Weismantel D, Weismantel A: Can nonantidepressants help treat depression? J Fam Pract
2009;58(10):550-551.

Item 79

ANSWER: B

When proteinuria is noted on a dipstick and the history, examination, full urinalysis, and serum studies
suggest no obvious underlying problem or renal insufficiency, a urine protein/creatinine ratio is
recommended. This test correlates well with 24-hour urine protein, which is particularly difficult to collect
in a younger patient. Renal ultrasonography is appropriate once renal insufficiency or nephritis is
established. If pathogenic proteinuria is confirmed, an antinuclear antibody and/or complement panel may
be indicated. A nephrology referral is not necessary until the presence of kidney disease or proteinuria
from a cause other than benign postural proteinuria is confirmed.

Ref: Leung AKC, Wong AHC: Proteinuria in children. Am Fam Physician 2010;82(6):645-651.

Item 80

ANSWER: A

Patients who present with acute cervical radiculopathy and normal radiographs can be treated
conservatively. The vast majority of patients with cervical radiculopathy improve without surgery. Of
the interventions listed, NSAIDs are the initial treatment of choice. Tricyclic antidepressants, as well as
tramadol and venlafaxine, have been shown to help with chronic neuropathic pain. Cervical MRI is not
indicated unless there are progressive neurologic defects or red flags such as fever or myelopathy.
Likewise, referral to a subspecialist should be reserved for patients who have persistent pain after 68
weeks of conservative management and for those with signs of instability. Cervical corticosteroid
injections have been found to be helpful in the management of cervical radiculopathy, but should not be
administered before MRI is performed (SOR C).

Ref: Polston DW: Cervical radiculopathy. Neurol Clin 2007;25(2):373-385. 2) Eubanks JD: Cervical radiculopathy:
Nonoperative management of neck pain and radicular symptoms. Am Fam Physician 2010;81(1):33-40.

27
Item 81

ANSWER: B

Clinical clues to the diagnosis of malaria in this case include an appropriately targeted recent travel history,
a prodrome of delirium or erratic behavior, unarousable coma following a generalized convulsion, fever,
and a lack of focal neurologic signs in the presence of a diffuse, symmetric encephalopathy. The
peripheral blood smear shows normochromic, normocytic anemia with Plasmodium falciparum
trophozoites and schizonts involving erythrocytes, diagnostic of cerebral malaria. Treatment of this true
medical emergency is intravenous quinidine gluconate.

Vitamin B12 deficiency is a predominantly peripheral neuropathy seen in older adults. Ehrlichiosis causes
thrombocytopenia but not hemolytic anemia. Sickle cell disease presents with painful vaso-occlusive crises
in multiple organs. Coma is rare.

Ref: Fauci AS, Braunwald E, Kasper DL, et al (eds): Harrisons Principles of Internal Medicine, ed 17. McGraw-Hill, 2008,
pp 1280-1294.

Item 82

ANSWER: A

Apraxia is a transmission disturbance on the output side, which interferes with skilled movements. Even
though the patient understands the request, he is unable to perform the task when asked, but may then
perform it after a time delay. Agnosia is the inability to recognize previously familiar sensory input, and
is a modality-bound deficit. For example, it results in a loss of ability to recognize objects. Aphasia is
a language disorder, and expressive aphasia is a loss of the ability to express language. The ability to
recognize objects by palpation in one hand but not the other is called astereognosis.

Ref: Fauci AS, Braunwald E, Kasper DL, et al (eds): Harrisons Principles of Internal Medicine, ed 17. McGraw-Hill, 2008,
p 157.

Item 83

ANSWER: E

Using trained, qualified interpreters for patients with limited English proficiency leads to fewer
hospitalizations, less reliance on testing, a higher likelihood of making the correct diagnosis and providing
appropriate treatment, and better patient understanding of conditions and therapies. Although the patient
may request that a family member interpret, there are many pitfalls in using untrained interpreters: a lack
of understanding of medical terminology, concerns about confidentiality, and unconscious editing by the
interpreter of what the patient has said. Additionally, the patient may be reluctant to divulge sensitive or
potentially embarrassing information to a friend or family member. The other principles listed are
important practices when working with interpreters. Pictures and diagrams can help strengthen the patients
understanding of his or her health care.

Ref: Herndon E, Joyce L: Getting the most from language interpreters. Fam Pract Manag 2004;11(6):37-40. 2) Juckett G:
Cross-cultural medicine. Am Fam Physician 2005;72(11):2267-2274.

28
Item 84

ANSWER: C

This estimated fetal weight is at the 90th percentile for a term fetus. Unfortunately, the accuracy of fetal
weight estimates declines as pregnancy proceeds, and the actual size may be as much as 15% different
from the estimate. Delivery of a large infant results in shoulder dystocia more often than delivery of a
smaller infant, but most large infants are delivered without complications. Intuitively, it would seem logical
to induce labor when the fetus seems to be getting large, but this intervention has been studied in controlled
trials and the only difference in outcome was an increase in the cesarean rate for women who underwent
elective induction for this indication.

Recently, there has been an increase in requests from patients to have an elective cesarean section near
term to avoid the risks of labor, including pain, shoulder dystocia, and pelvic relaxation. The American
Congress of Obstetricians and Gynecologists (ACOG) recommends consideration of cesarean delivery
without a trial of labor if the estimated fetal weight is 4500 g in a mother with diabetes mellitus, or 5000 g
in the absence of diabetes. Even at that size, there is not adequate data to show that cesarean section is
preferable to a trial of labor. Frequent ultrasonography is often performed to reduce anxiety for both
patient and physician, but the problem of accuracy of weight estimates remains an issue even with repeated
scans at term.

Ref: ACOG practice bulletin: Shoulder dystocia. American College of Obstetricians and Gynecologists, 2002, no 40. 2)
Gherman RB, Chauhan S, Ouzounian JG, et al: Shoulder dystocia: The unpreventable obstetric emergency with empiric
management guidelines. Am J Obstet Gynecol 2006;195(3):657-672. 3) Gabbe SG, Niebyl JR, Simpson JL (eds):
Obstetrics: Normal and Problem Pregnancies, ed 5. Churchill Livingstone, 2007, pp 219-220, 851. 4) Mozurkewich E,
Chilimigras J, Koepke E, et al: Indications for induction of labor: A best-evidence review. BJOG 2009;116(5):626-636.

Item 85

ANSWER: D

Intertrigo is inflammation of skinfolds caused by skin-on-skin friction and is common on opposing


cutaneous or mucocutaneous surfaces. Secondary cutaneous bacterial and fungal infections are common
complications. Cutaneous erythrasma may complicate intertrigo of interweb areas, intergluteal and crural
folds, axillae, or inframammary regions. Erythrasma is caused by Corynebacterium minutissimum and
presents as small reddish-brown macules that may coalesce into larger patches with sharp borders.
Intertrigo complicated by erythrasma is treated with topical or oral erythromycin.

Ref: Janniger CK, Schwartz RA, Szepietowski JC, Reich A: Intertrigo and common secondary skin infections. Am Fam
Physician 2005;72(5):833-838, 840. 2) Habif TP: Clinical Dermatology: A Color Guide to Diagnosis and Therapy, ed 5.
Mosby Elsevier, 2010, pp 501, 534.

Item 86

ANSWER: C

The CAGE-AID (CAGE Adapted to Include Drugs) questionnaire is a tool for assessing potential substance
abuse risk. In one study it had a sensitivity of 70% and a specificity of 85% for drug abuse when two or
more affirmative responses were defined as a positive result. It consists of the following four questions:

29
Have you ever felt you ought to Cut down on your drinking or drug use?
Have people Annoyed you by criticizing your drinking or drug use?
Have you ever felt bad or Guilty about your drinking or drug use?
Have you ever had a drink or used drugs first thing in the morning as an Eye opener to
steady your nerves or to get rid of a hangover?

Ref: Yong SS, Ballantyne JC, Domino FJ, Wetterau NW: Balancing clinical and risk management considerations for chronic
pain patients on opioid therapy. American Academy of Family Physicians CME monograph 2008, pp 5-7. 2) Lanier D,
Ko S: Screening in in primary care for illicit drug use: Assessment of screening instrumentsSupplemental evidence update
for the US Preventive Services Task Force. Evidence synthesis no 58.2. AHRQ Publication no 08-05108-EF-2.

Item 87

ANSWER: C

This patient has symptoms and signs consistent with chronic paronychia. This condition is often associated
with chronic immersion in water, contact with soaps or detergents, use of certain systemic drugs
(antiretrovirals, retinoids) and, as is most likely in a 6-year-old child, finger sucking.

Findings on examination are similar to those of acute paronychia, with tenderness, erythema, swelling,
and retraction of the proximal nail fold. Often the adjacent cuticle is absent. Chronic paronychia has
usually been persistent for at least 6 weeks by the time of diagnosis.

In addition to medication, basic treatment principles for the condition include avoidance of contact irritants,
avoiding immersion of the hands in water, and use of an emollient. Topical corticosteroids have higher
efficacy for treating chronic paronychia compared to oral antifungals (SOR B), particularly given the
young age of the patient. A topical antifungal can also be tried in conjunction with the corticosteroid.

Ref: Tosti A, Piraccini BM, Ghetti E, Colombo MD: Topical steroids versus systemic antifungals in the treatment of chronic
paronychia: An open, randomized double blind and double dummy study. J Am Acad Dermatol 2002;47(1):73-76. 2) Habif
TP: Clinical Dermatology: A Color Guide to Diagnosis and Therapy, ed 4. Mosby Inc, 2004, pp 871-872. 3) Rigopoulos
D, Larios G, Gregoriou S, Alevizos A: Acute and chronic paronychia. Am Fam Physician 2008;77(3):339-346.

Item 88

ANSWER: A

In the geriatric population, presbycusis is the most common cause of hearing loss. Patients typically have
the most difficulty hearing higher-frequency sounds such as consonants. Lower-frequency sounds such
as vowels are preserved.

Ref: Halter JB, Ouslander JG, Tinetti ME, et al (eds): Hazzards Geriatric Medicine and Gerontology, ed 6. McGraw-Hill
Medical, 2009, pp 525-534. 2) Elsawy B, Higgins KE: The geriatric assessment. Am Fam Physician 2011;83(1):48-56.

30
Item 89

ANSWER: E

This patient has obesity-hypoventilation syndrome, often referred to as Pickwickian syndrome. These
patients are obese (BMI >30 kg/m2 ), have sleep apnea, and suffer from chronic daytime hypoxia and
carbon dioxide retention. They are at increased risk for significant respiratory failure and death compared
to patients with otherwise similar demographics. Treatment consists of nighttime positive airway pressure
in the form of continuous (CPAP) or bi-level (BiPAP) devices, as indicated by sleep testing. The more
hours per day that patients can use this therapy, the less carbon dioxide retention and less daytime hypoxia
will ensue. Several small studies suggest that the increased mortality risk from obesity-hypoventilation
syndrome can be decreased by adhering to this therapy. The use of daytime oxygen can improve
oxygenation, but is not considered adequate to restore the chronic low respiratory drive that is
characteristic of this condition.

Ref: Goldman L, Ausiello D (eds): Cecil Medicine, ed 23. Saunders, 2008, p 611. 2) Mokhlesi B: Obesity hypoventilation
syndrome: A state-of-the-art review. Respir Care 2010;55(10):1347-1362; discussion 1363-1365.

Item 90

ANSWER: A

Pertussis, once a common disease in infants, declined to around 1000 cases in 1976 as a result of
widespread vaccination. The incidence began to rise again in the 1980s, possibly because the immunity
from vaccination rarely lasts more than 12 years.

The disease is characterized by a prodromal phase that lasts 12 weeks and is indistinguishable from a viral
upper respiratory infection. It progresses to a more severe cough after the second week. The cough is
paroxysmal and may be severe enough to cause vomiting or fracture ribs. Patients are rarely febrile, but
may have increased lacrimation and conjunctival injection. The incubation period is long compared to a
viral infection, usually 710 days.

Nonasthmatic eosinophilic bronchitis, cough-variant asthma, and gastroesophageal reflux disease cause a
severe cough not associated with a catarrhal phase. A rhinovirus infection would probably be resolving
within 23 weeks.

Ref: Cornia PB, Hersh AL, Lipsky BA, et al: Does this coughing adolescent or adult patient have pertussis? JAMA
2010;304(8):890-896.

Item 91

ANSWER: D

A thrombosed external hemorrhoid is manifested by the sudden development of a painful, tender, perirectal
lump. Because there is somatic innervation, the pain is intense, and increases with edema. Treatment
involves excision of the acutely thrombosed tissue under local anesthesia, mild pain medication, and sitz
baths. It is inappropriate to use procedures that would increase the pain, such as banding or cryotherapy.
Total hemorrhoidectomy is inappropriate and unnecessary.

Ref: Tintinalli JE, Kelen GD, Stapczynski JS (eds): Emergency Medicine: A Comprehensive Study Guide, ed 7. McGraw-Hill,
2011, pp 1245-1247.

31
Item 92

ANSWER: C

Patellofemoral pain syndrome is a common overuse injury observed in adolescent girls. The condition is
characterized by anterior knee pain associated with activity. The pain is exacerbated by going up or down
stairs or running in hilly terrain. It is associated with inadequate hip abductor and core strength; therefore,
a prescription for a rehabilitation program is recommended. Surgical intervention is rarely required.

Ref: Joy E, Van Hala S, Cooper L: Health-related concerns of the female athlete: A lifespan approach. Am Fam Physician
2009;79(6):489-495.

Item 93

ANSWER: D

Hamartomatous (or juvenile) polyps and hyperplastic polyps are benign lesions and are not considered to
be premalignant. Adenomas, on the other hand, have the potential to become malignant. Sessile adenomas
and lesions >1.0 cm have a higher risk for becoming malignant. Of the three types of adenomas (tubular,
tubulovillous, and villous), villous adenomas are the most likely to develop into an adenocarcinoma.

Ref: Fauci AS, Braunwald E, Kasper DL, et al (eds): Harrisons Principles of Internal Medicine, ed 17. McGraw-Hill, 2008,
p 574.

Item 94

ANSWER: E

Repaglinide and nateglinide are nonsulfonylureas that act on a portion of the sulfonylurea receptor to
stimulate insulin secretion. Pioglitazone is a thiazolidinedione, which reduces insulin resistance. It is
believed that the mechanism for this is activation of PPAR-Y, a receptor that affects several
insulin-responsive genes. Acarbose is a competitive inhibitor of #-glucosidases, enzymes that break down
complex carbohydrates into monosaccharides. This delays the absorption of carbohydrates such as starch,
sucrose, and maltose, but does not affect the absorption of glucose. Sitagliptin is a DPP-IV inhibitor, and
this class of drugs inhibits the enzyme responsible for the breakdown of the incretins GLP-1 and GIP.
Exenatide is an incretin mimetic that stimulates insulin secretion in a glucose-dependent fashion, slows
gastric emptying, and may promote satiety.

Ref: Goldman L, Ausiello D (eds): Cecil Medicine, ed 23. Saunders, 2008, pp 1754-1755.

Item 95

ANSWER: A

Behavioral interventions, especially those including telephone counseling and/or a dental examination, have
been shown to be helpful for promoting smokeless tobacco cessation (SOR B). Studies examining mint
snuff as a tobacco substitute, bupropion, and nicotine replacement in patch or gum form did not show any
significant benefit.

Ref: Ebbert J, Montori VM, Erwin PJ, Stead LF: Interventions for smokeless tobacco use cessation. Cochrane Database Syst
Rev 2011;(2):CD004306.

32
Item 96

ANSWER: B

Autoimmune hypothyroidism is common in elderly women. Symptoms often include fatigue, bradycardia,
dry skin, brittle hair, and a prolonged relaxation phase of the deep tendon reflexes. While replacement
therapy with levothyroxine is indicated, care must be taken in the elderly, particularly in those with
coronary artery disease, to replace the deficit slowly. Levothyroxine replacement should begin at 25 "g
daily for 6 weeks, with the dosage increased in 25-"g increments as needed, based on TSH levels.

Rapid replacement of thyroid hormone can increase the metabolic rate, and therefore myocardial oxygen
demand, too quickly. This can precipitate complications of coronary artery disease such as atrial
fibrillation, angina, and myocardial infarction. Stopping a !-blocker in this setting is likely to increase the
risk. Radioactive iodine ablation is indicated for some cases of hyperthyroidism.

Ref: Melmed S, Polonsky KS, Larsen PR, Kronenberg HM (eds): Williams Textbook of Endocrinology, ed 12. Elsevier
Saunders, 2011, pp 407-427.

Item 97

ANSWER: D

Vacuum-assisted delivery is associated with higher rates of neonatal cephalhematoma and retinal
hemorrhage compared with forceps delivery. A systematic review of 10 trials found that vacuum-assisted
deliveries are associated with less maternal soft-tissue trauma when compared to forceps delivery.
Compared with spontaneous vaginal delivery, the likelihood of a severe perineal laceration is increased
in women who have vacuum-assisted delivery without episiotomy, and the odds are even higher in
vacuum-assisted delivery with episiotomy. Operative vaginal delivery is a risk factor for shoulder dystocia,
which is more common with vacuum-assisted delivery than with forceps delivery.

Ref: Hook CD, Damos JR: Vacuum-assisted vaginal delivery. Am Fam Physician 2008;78(8):953-960.

Item 98

ANSWER: B

The best initial imaging study for acute pelvic pain in women is transvaginal ultrasonography (SOR C).
This provides the greatest level of detail regarding the uterus and adnexae, superior to transcutaneous
ultrasonography. CT of the abdomen/pelvis and hysterosalpingography may be indicated eventually in
some patients with pelvic pain, but they are not the initial studies of choice. Hysteroscopy is not routinely
used in the evaluation of pelvic pain.

Ref: Kruszka PS, Kruszka SJ: Evaluation of acute pelvic pain in women. Am Fam Physician 2010;82(2):141-147.

33
Item 99

ANSWER: C

Escherichia coli O:157 is an increasingly common cause of serious gastrointestinal illness. The usual
source is undercooked beef. The child is at risk, since at least 20% of cases result from secondary spread.
Transmission is frequent in childrens day-care facilities and nurseries. Some cases are asymptomatic, but
the great majority are symptomatic, and patients present with bloody diarrhea. Levofloxacin is not useful
for prophylaxis in contacts. This patient has a 10%15% risk of developing hemolytic uremic syndrome
secondary to her E. coli O:157 infection, making close monitoring of renal function essential.

Ref: Pennington H: Escherichia coli O157. Lancet 2010;376(9750):1428-1435.

Item 100

ANSWER: C

Upper extremity deep-vein thrombosis (UE-DVT) accounts for 4% of all cases of DVT. Catheter-related
thromboses make up the majority of these cases. Occult cancer, use of oral contraceptives, and inheritable
thrombophilia are other common explanations. Another proposed risk factor is the repetitive compression
of the axillary-subclavian vein in athletes or laborers, which is the most likely cause of this patients
UE-DVT.

Taken as a whole, UE-DVT is generally associated with fewer venous complications, including less chance
for thromboembolism, postphlebitic syndrome, and recurrence compared to lower-extremity deep-vein
thrombosis (LE-DVT). However, the rates of these complications are still high enough that most experts
recommend treatment identical to that of LE-DVT. Specifically, heparin should be given for 5 days, and
an oral vitamin-K antagonist for at least 3 months.

Ref: Kearon C, Kahn SR, Agnelli G, et al: Antithrombotic therapy for venous thromboembolic disease: American College of
Chest Physicians evidence-based clinical practice guidelines (8th edition). Chest 2008;133(6 suppl):454S-545S. 2) Muoz
FJ, Mismetti P, Poggio R, et al: Clinical outcome of patients with upper-extremity deep vein thrombosis. Chest
2008;133(1):143-148. 3) Martinelli I, Franchini M, Mannucci PM: How I treat rare venous thromboses. Blood
2008;112(13):4818-4823.

Item 101

ANSWER: B

Trazodone may be useful for insomnia, but is not recommended as a primary antidepressant because it
causes sedation and orthostatic hypotension at therapeutic doses. Bupropion would aggravate this patients
insomnia. Tricyclic antidepressants may be effective, but are no longer considered first-line treatments
because of side effects and because they can be cardiotoxic. Mirtazapine has serotonergic and
noradrenergic properties and is associated with increased appetite and weight gain. It may be particularly
useful for patients with insomnia and weight loss.

Ref: Untzer J: Late-life depression. N Engl J Med 2007;357(22):2269-2276.

34
Item 102

ANSWER: A

Diuretics such as hydrochlorothiazide are known to increase serum uric acid levels, but losartan has been
shown to decrease uric acid. Metoprolol, simvastatin, and acetaminophen have no specific effect on serum
uric acid levels.

Ref: Richette P, Bardin T: Gout. Lancet 2010;375(9711):318-328.

Item 103

ANSWER: B

The Cardiovascular risk Reduction by Early Anemia Treatment with Epoetin Beta (CREATE) trial, the
Correction of Hemoglobin and Outcomes in Renal insufficiency (CHOIR) trial, and the Trial to Reduce
Cardiovascular Events with Aranesp Therapy (TREAT) have shown that patients who had hemoglobin
levels targeted to normal ranges did worse than patients who had hemoglobin levels of 1012 g/dL. The
incidence of stroke, heart failure, and death increased in patients targeted to normal hemoglobin levels,
and there was no demonstrable decrease in cardiovascular events (SOR A).

Ref: Taliercio JJ: Anemia and chronic disease: Whats the connection? J Fam Pract 2010;59(1):14-18.

Item 104

ANSWER: B

Vocal cord dysfunction is an idiopathic disorder commonly seen in patients in their twenties and thirties
in which the vocal cords partially collapse or close on inspiration. It mimics, and is commonly mistaken
for, asthma. Symptoms include episodic tightness of the throat, a choking sensation, shortness of breath,
and coughing. A careful history and examination reveal that the symptoms are worse with inspiration than
with exhalation, and inspiratory stridor during the episode may be mistaken for the wheezing of asthma.
The sensation of throat tightening or choking also helps to differentiate it from asthma.

Pulmonary function tests (PFTs) are normal, with the exception of flattening of the inspiratory loop, which
is diagnostic of extra-thoracic airway compression. Fiberoptic laryngoscopy shows paradoxical inspiratory
and/or expiratory partial closure of the vocal cords. Vocal cord dysfunction is treated with speech therapy,
breathing techniques, reassurance, and breathing a helium-oxygen mixture (heliox).

PFTs in patients with asthma are normal between exacerbations, but when symptoms are present the
FEV1/FVC ratio is reduced, as with COPD. With anaphylaxis, there will typically be itching or urticaria
and signs of angioedema, such as lip or tongue swelling, in response to a trigger such as food or
medication; PFTs are normal when anaphylaxis symptoms are absent. Globus hystericus is a type of
conversion disorder in which emotional stress causes a subjective sensation of pain or tightness in the
throat, and/or dysphagia; diagnostic tests such as spirometry and laryngoscopy are normal.

Ref: Deckert J, Deckert L: Vocal cord dysfunction. Am Fam Physician 2010;81(2):156-159.

35
Item 105

ANSWER: B

Chest pain is common in patients with pulmonary embolism (PE). When evaluating a patient for possible
PE, the presence of orthopnea suggests heart failure, fever suggests an infectious process, wheezing
suggests asthma or COPD, and rhonchi suggest heart failure, interstitial lung disease, or infection. These
generalizations are supported by a 2008 study designed to improve the diagnosis of PE based on the
history, physical examination, EKG, and chest radiograph.

Ref: Miniati M, Bottai M, Monti S, et al: Simple and accurate prediction of the clinical probability of pulmonary embolism. Am
J Respir Crit Care Med 2008:178(3):290-294.

Item 106

ANSWER: C

Nursemaids elbow is one of the most common injuries in children under 5 years of age. It occurs when
the childs hand is suddenly jerked up, forcing the elbow into extension and causing the radial head to slip
out from the annular ligament.

Ref: Kliegman RM, Stanton BF, Geme JW III, et al (eds): Nelson Textbook of Pediatrics, ed 19. Elsevier Saunders, 2011, p
2384.

Item 107

ANSWER: D

Tetanus vaccine is indicated for adults with clean minor wounds who have received fewer than three
previous doses of tetanus toxoid, or whose immune status is unknown. Tetanus immune globulin is not
recommended if the wound is clean.

The CDC recommends that adults aged 65 years and older who have not received Tdap and are likely to
have close contact with an infant less than 12 months of age (e.g., grandparents, child-care providers, and
health-care practitioners) should receive a single dose to protect against pertussis and reduce the likelihood
of transmission. For other adults aged 65 years and older, a single dose of Tdap vaccine should be given
instead of a scheduled dose of Td vaccine if they have not previously received Tdap. Tdap can be
administered regardless of the interval since the last vaccine containing tetanus or diphtheria toxoid, and
either Tdap vaccine product may be used. After receiving Tdap, persons should continue to receive Td for
routine booster immunizations against tetanus and diphtheria, according to previously published guidelines.

Ref: Updated recommendations for use of tetanus toxoid, reduced diphtheria toxoid and acellular pertussis (Tdap) vaccine from
the Advisory Committee on Immunization Practices, 2010. MMWR Morb Mortal Wkly Rep 2011;60(1):13-15.

36
Item 108

ANSWER: B

The target blood pressure in patients with diabetes mellitus is <130/80 mm Hg (SOR A). ACE inhibitors
and angiotensin receptor blockers (ARBs) are the preferred first-line agents for the management of patients
with hypertension and diabetes mellitus (SOR A). If the target blood pressure is not achieved with an ACE
inhibitor or ARB, the addition of a thiazide diuretic is the preferred second-line therapy for most patients;
potassium-sparing and loop diuretics are not recommended (SOR B).

!-Blockers are recommended for patients with diabetes mellitus who also have a history of myocardial
infarction, heart failure, coronary artery disease, or stable angina (SOR A). Calcium channel blockers
should be reserved for patients with diabetes mellitus who cannot tolerate preferred antihypertensive
agents, or for those who need additional agents to achieve their target blood pressure (SOR A).

Ref: Whalen KL, Stewart RD: Pharmacologic management of hypertension in patients with diabetes. Am Fam Physician
2008;78(11):1277-1282.

Item 109

ANSWER: C

This patient has anemia of chronic disease secondary to her rheumatoid arthritis. This anemia is usually
mild, with hemoglobin levels of 9.011.0 g/dL, and is usually normocytic-normochromic, although it can
be microcytic. Characteristically, serum iron and total iron-binding capacity are decreased and ferritin is
increased. The best treatment of this anemia is to treat the underlying systemic disease. Neither iron nor
folic acid is effective. Since the anemia is usually mild, transfusion is not necessary.

Ref: Weiss G, Goodnough LT: Anemia of chronic disease. N Engl J Med 2005;352(10):1011-1023.

Item 110

ANSWER: B

Sudden infant death syndrome (SIDS) is the most common cause of death during the first 6 months of life
in the United States, with a peak incidence at 24 months of age and a quick dropoff by the age of 6
months. The cause of death is a retrospective diagnosis of exclusion, and is supported by a history of quiet
death during sleep in a previously healthy infant younger than 6 months of age. Evidence of terminal
activity may be present, such as clenched fists or a serosanguineous, blood-tinged, or mucoid discharge
from the mouth or nose. Lividity and mottling are frequently present in dependent areas.

The reported history and autopsy findings of deliberate suffocation may mirror the findings of SIDS, but
suffocation should be considered when there is documentation of any of the following: infant age older than
6 months, previous similar sibling deaths, simultaneous twin deaths, or evidence of pulmonary
hemorrhage. A history of recurrent apnea or cyanosis has not been causally linked to SIDS; when such
reported events have only been witnessed by one caretaker, deliberate suffocation should be suspected.

Ref: American Academy of Pediatrics, Hymel KP; Committee on Child Abuse and Neglect; National Association of Medical
Examiners: Distinguishing sudden infant death syndrome from child abuse fatalities. Pediatrics 2006;118(1):421-427.

37
Item 111

ANSWER: E

Calcium oxalate stones are the most common of all renal calculi. A low-sodium, restricted-protein diet with
increased fluid intake reduces stone formation. A low-calcium diet has been shown to be ineffective.
Oxalate restriction also reduces stone formation. Oxalate-containing foods include spinach, chocolate, tea,
and nuts, but not yellow vegetables. Potassium citrate should be taken at mealtime to increase urinary pH
and urinary citrate (SOR B).

Ref: Borghi L, Schianchi T, Meschi T, et al: Comparison of two diets for the prevention of recurrent stones in idiopathic
hypercalciuria. N Engl J Med 2002;346(2):77-84. 2) Pietrow PK, Karellas ME: Medical management of common urinary
calculi. Am Fam Physician 2006;74(1):86-94.

Item 112

ANSWER: A

The findings in this patient are consistent with a stress fracture. Plain radiographs should be the initial
imaging modality because of availability and low cost (SOR C). These are usually negative initially, but
are more likely to be positive over time. If the initial films are negative and the diagnosis is not urgently
needed, a second plain radiograph can be performed in 23 weeks.

Although CT is useful for evaluation of bone pathology, it is not commonly used as even second-line
imaging for stress fractures, due to lower sensitivity and higher radiation exposure than other modalities.
Triple-phase bone scintigraphy has a high sensitivity and was previously used as a second-line modality;
however, MRI has equal or better sensitivity than scintigraphy and higher specificity. MRI is now
recommended as the second-line imaging modality when plain radiographs are negative and clinical
suspicion of stress fracture persists (SOR C). Musculoskeletal ultrasonography has the advantage of low
cost with no radiation exposure, but additional studies are needed before it can be recommended as a
standard imaging modality.

Ref: Patel DS, Roth M, Kapil N: Stress fractures: Diagnosis, treatment, and prevention. Am Fam Physician 2011;83(1):39-46.

Item 113

ANSWER: D

Ethylene glycol poisoning should be suspected in patients with metabolic acidosis of unknown cause and
subsequent renal failure, as rapid diagnosis and treatment will limit the toxicity and decrease both
morbidity and mortality. This diagnosis should be considered in a patient who appears intoxicated but does
not have an odor of alcohol, and has anion gap acidosis, hypocalcemia, urinary crystals, and nontoxic
blood alcohol levels. Ethylene glycol is found in products such as engine coolant, de-icing solution, and
carpet and fabric cleaners. Ingestion of 100 mL of ethylene glycol by an adult can result in toxicity.

The American Academy of Clinical Toxicology criteria for treatment of ethylene glycol poisoning with an
antidote include a plasma ethylene glycol concentration >20 mg/dL, a history of ingesting toxic amounts
of ethylene glycol in the past few hours with an osmolal gap >10 mOsm/kg H 2O (N 510), and strong
clinical suspicion of ethylene glycol poisoning, plus at least two of the following: arterial pH <7.3, serum
bicarbonate <20 mmol/L, or urinary oxalate crystals.

38
Until recently, ethylene glycol poisoning was treated with sodium bicarbonate, ethanol, and hemodialysis.
Treatment with fomepizole (Antizol) has this specific indication, however, and should be initiated
immediately when ethylene glycol poisoning is suspected. If ethylene glycol poisoning is treated early,
hemodialysis may be avoided, but once severe acidosis and renal failure have occurred hemodialysis is
necessary. Ethylene glycol is rapidly absorbed, and use of ipecac or gastric lavage is therefore not
effective. Large amounts of activated charcoal will only bind to relatively small amounts of ethylene
glycol, and the therapeutic window for accomplishing this is less than 1 hour.

Ref: Scalley RD, Ferguson DR, Picarro JC, et al: Treatment of ethylene glycol poisoning. Am Fam Physician 2002;66(5):
807-812. 2) Marx JA (ed): Rosens Emergency Medicine: Concepts and Clinical Practice, ed 7. Mosby Elsevier, 2010,
pp 2003-2007.

Item 114

ANSWER: A

It is recommended that metformin be avoided in patients with a creatinine level >1.5 mg/dL for men or
>1.4 mg/dL for women. Glyburide has an active metabolite that is eliminated renally. This metabolite
can accumulate in patients with chronic kidney disease, resulting in prolonged hypoglycemia. Acarbose
should be avoided in patients with chronic kidney disease, as it has not been evaluated in these patients.
Glipizide does not have an active metabolite, and is safe in patients with chronic renal disease.

Ref: Cavanaugh KL: Diabetes management issues for patients with chronic kidney disease. Clinical Diabetes 2007;25(3):90-97.

Item 115

ANSWER: D

In patients with a drug-eluting stent, combined therapy with clopidrogel and aspirin is recommended for
12 months because of the increased risk of late stent thrombosis. After this time, aspirin at a dosage of
75165 mg/day is recommended. The minimum duration of combined therapy is 1 month for a bare metal
stent, 3 months for a sirolimus-eluting stent, and 6 months for other drug-eluting stents.

Ref: Dehmer GJ, Smith KJ: Drug-eluting coronary artery stents. Am Fam Physician 2009;80(11):1245-1251, 1252-1253.

Item 116

ANSWER: C

Parkinsons disease and essential tremor are the primary concerns in a person of this age who presents with
a new tremor. A coarse, resting, pill-rolling tremor is characteristic of Parkinsons disease. Essential
tremor is primarily an action tremor and is a common movement disorder, occurring in members of the
same family with a high degree of frequency. Alcohol intake will temporarily cause marked reduction in
the tremor. !-Adrenergic blockers have been the mainstay of treatment for these tremors, but this patient
is intolerant to these drugs. Primidone has been effective in the treatment of essential tremor, and in
head-to-head studies with propranolol has been shown to be superior after 1 year. Levodopa in
combination with carbidopa is useful in the treatment of parkinsonian tremor but not essential tremor.

Ref: Ropper AH, Samuels MA: Adams and Victors Principles of Neurology, ed 9. McGraw-Hill, 2009, pp 90-92.

39
Item 117

ANSWER: D

When a medical error has been made, patients prefer that their physician disclose the error and offer an
explanation of events. Withholding that information from a patient is not ethical and is counter to
standards set forth by various organizations such as the Joint Commission on Accreditation of Health Care
Organizations. Using the word error is acceptable and does not lead to an increase in litigation. In fact,
there is no evidence that malpractice litigation rates increase when an error is admitted, and rates often
decrease. Private-practice physicians are less likely to admit errors to patients. It is surmised that these
physicians have less access to training in disclosure than those employed by hospitals or health care
organizations.

Ref: Gallagher TH, Waterman AD, Garbutt JM, et al: US and Canadian physicians attitudes and experiences regarding
disclosing errors to patients. Arch Intern Med 2006;166(15):1605-1611.

Item 118

ANSWER: D

First-line therapies for urge urinary incontinence include behavioral therapy, such as pelvic muscle
contractions, and anticholinergic therapy. Oral estrogen is not indicated. Noninvasive treatments should
be tried initially. Urodynamic testing is indicated preoperatively. Stopping the hydrochlorothiazide would
not be helpful, as it would not address the issue of detrusor instability.

Ref: Nygaard I: Clinical practice. Idiopathic urgency urinary incontinence. N Engl J Med 2010;363(12):1156-1162.

Item 119

ANSWER: C

The ADA recommends testing to detect type 2 diabetes mellitus in asymptomatic adults with a BMI !25
kg/m2 and one or more additional risk factors. Risk factors include physical inactivity, hypertension, an
HDL-cholesterol level <35 mg/dL, a triglyceride level >250 mg/dL, a history of cardiovascular disease,
a hemoglobin A1c !5.7%, a history of gestational diabetes or delivery of an infant weighing >4 kg (9 lb),
and a history of polycystic ovary syndrome.

Diabetes mellitus can be diagnosed if the patients fasting blood glucose level is !126 mg/dL on two
separate occasions. It can also be diagnosed if a random blood glucose level is !200 mg/dL if classic
symptoms of diabetes are present. A fasting blood glucose level of 100125 mg/dL, a glucose level of
140199 mg/dL 2 hours following a 75-g glucose load, or a hemoglobin A 1c of 5.7%6.9% signifies
impaired glucose tolerance. Patients meeting these criteria have a significantly higher risk of progression
to diabetes and should be counseled about lifestyle modifications such as weight loss and exercise.

Ref: Patel P, Macerollo A: Diabetes mellitus: Diagnosis and screening. Am Fam Physician 2010;81(7):863-870.

40
Item 120

ANSWER: C

Causes of low back pain include vertebral disk herniation and spinal stenosis. Numbness and muscle
weakness may be present in both. Pain from spinal stenosis is relieved by sitting and aggravated by
standing, whereas the opposite is true for pain from a herniated disk.

Ref: Chou R, Qaseem A, Snow V, Casey D, et al; Clinical Efficacy Assessment Subcommittee of the American College of
Physicians; American College of Physicians; American Pain Society Low Back Pain Guidelines Panel: Diagnosis and
treatment of low back pain: A joint clinical practice guideline from the American College of Physicians and the American
Pain Society. Ann Intern Med 2007;147(7):478-491. 2) Haig AJ, Tomkins CC: Diagnosis and management of lumbar spinal
stenosis. JAMA 2010;303(1):71-72. 3) Abraham P, Ouedraogo N, Leftheriotis G: Diagnosing lumbar spinal stenosis. JAMA
2010;303(15):1479-1480.

Item 121

ANSWER: D

Community-acquired methicillin-resistant Staphylococcus aureus (MRSA) is resistant to !-lactam and


macrolide antibiotics, and is showing increasing resistance to fluoroquinolones. FDA-approved treatments
include clindamycin and doxycycline. Other commonly used treatments include minocycline and
trimethoprim/sulfamethoxazole.

Ref: Goldman L, Ausiello D (eds): Cecil Medicine, ed 23. Saunders, 2008, pp 2152, 2170.

Item 122

ANSWER: C

In 2008, the American Academy of Pediatrics increased its recommended daily intake of vitamin D in
infants, children, and adolescents to 400 IU/day (SOR C). Breastfeeding does not provide adequate levels
of vitamin D. Exclusive formula feeding probably provides adequate levels of vitamin D, but infants who
consume less than 1 liter of formula per day need supplementation with 400 IU of vitamin D daily. Vitamin
D supplementation should be started within the first 2 months of birth.

Ref: Wagner CL, Greer FR: American Academy of Pediatrics Section on Breastfeeding; American Academy of Pediatrics
Committee on Nutrition: Prevention of rickets and vitamin D deficiency in infants, children, and adolescents. Pediatrics
2008;122(5):1142-1152. 2) Casey CF, Slawson DC, Neal LR: Vitamin D supplementation in infants, children, and
adolescents. Am Fam Physician 2010;81(6):745-748. 3) Insititute of Medicine: Dietary Reference Intakes for Calcium and
Vitamin D. The National Academies Press, 2011.

Item 123

ANSWER: A

Although dietary management may be appropriate, a weight reduction diet is not likely to improve this
patients cardiovascular outcome. In fact, even if this person were obese, there is insufficient evidence
that weight reduction would decrease his cardiovascular mortality (SOR C). There is good evidence that
the other options, even !-blockers in a patient with normal blood pressure, are indicated. All of these
measures have evidence to support their usefulness for secondary prevention of coronary artery disease
(SOR A).

41
Ref: Walker C, Reamy BV: Diets for cardiovascular disease prevention: What is the evidence? Am Fam Physician
2009;79(7):571-578. 2) Hall SL, Lorenc T: Secondary prevention of coronary artery disease. Am Fam Physician
2010;81(3):289-296.

Item 124

ANSWER: A

Selective serotonin reuptake inhibitors (SSRIs) such as sertraline have the broadest range of efficacy in
treating posttraumatic stress disorder (PTSD) since they are able to reduce all three clusters of PTSD
symptoms. Studies on the effectiveness of tricyclic antidepressants such as amitriptyline demonstrate
modest lessening of the symptoms of reexperiencing, with minimal or no effect on avoidance or arousal
symptoms. Patients treated with monoamine oxidase inhibitors such as phenelzine have shown moderate
to good improvement in reexperiencing and avoidance symptoms, but little improvement in hyperarousal.
Benzodiazepines such as alprazolam have been used to treat PTSD, but their efficacy against the major
symptoms has not been proven in controlled studies.

Ref: Jeffereys M: Clinicians Guide to Medications for PTSD. National Center for PTSD, US Dept of Veterans Affairs, 2009.

Item 125

ANSWER: A

Students with uncontrolled stage 2 hypertension should not participate in sports associated with static
exercise, in which the blood pressure load is more significantly increased (SOR C). Rowing involves both
a high static and a high dynamic load. Soccer, tennis, fencing, and baseball have relatively few static
exercise components and blood pressure spikes are less likely.

Ref: McCambridge TM, Benjamin HJ, Brenner JS: Council on Sports Medicine and Fitness: Athletic participation by children
and adolescents who have systemic hypertension. Pediatrics 2010;125(6):1287-1294.

Item 126

ANSWER: C

Psychomotor agitation is experienced by most patients during alcohol withdrawal. Benzodiazepines are
clearly the drug class of choice. Providing medication on an as-needed basis rather than on a fixed schedule
is generally preferred. Antipsychotics and butyrophenones (including haloperidol) lower the seizure
threshold and should not be used. For short-term management of status epilepticus, anticonvulsants may
be used in conjunction with benzodiazepines. The vast majority of seizures from withdrawal are
self-limited and do not require anticonvulsant treatment. Clonidine and other # 2-agonists do reduce minor
symptoms of withdrawal, but have not been shown to prevent seizures. The effectiveness of baclofen in
acute alcohol withdrawal is unknown.

Ref: Kosten TR, OConnor PG: Management of drug and alcohol withdrawal. N Engl J Med 2003;348(18):1786-1795. 2) Bayard
M, McIntyre J, Hill KR, Woodside J Jr: Alcohol withdrawal syndrome. Am Fam Physician 2004;69(6):1443-1450. 3)
Ricks J, Replogle WH, Cook NJ: FPINs Clinical Inquiries. Management of alcohol withdrawal syndrome. Am Fam
Physician 2010;82(4):344-347.

42
Item 127

ANSWER: D

Microscopic examination of synovial fluid in a patient suffering an acute attack of pseudogout shows large
numbers of polymorphonuclear leukocytes. Calcium pyrophosphate dihydrate crystals are frequently found
extracellularly and in polymorphonuclear leukocytes. When viewed with polarized light, the crystals
appear as short, blunt rods, rhomboids, and cuboids. The diagnosis is made by finding typical crystals
under compensated polarized light and is supported by radiographic evidence of chondrocalcinosis.

Ref: Goldman L, Ausiello D (eds): Cecil Medicine, ed 23. Saunders, 2008, pp 2075-2078.

Item 128

ANSWER: A

According to the American Diabetes Association, the goal for patients with type 2 diabetes mellitus is to
achieve a hemoglobin A1c of <7.0% (SOR C). This patient has achieved this goal, and there is no
indication for changes in his management.

Ref: American Diabetes Association: Standards of medical care in diabetes2011. Diabetes Care 2011;34(Suppl 1):S19-S21.

Item 129

ANSWER: E

The diagnosis of giardiasis is suggested by its most characteristic symptoms: foul-smelling, soft, or loose
stools; foul-smelling flatus; belching; marked abdominal distention; and the virtual absence of mucus or
blood in the stool. Stools are usually mushy between exacerbations, though constipation may occur. If
eosinophilia occurs, it is more likely to be related to some other concomitant cause rather than to
giardiasis.

Ref: Fauci AS, Braunwald E, Kasper DL, et al (eds): Harrisons Principles of Internal Medicine, ed 17. McGraw-Hill, 2008,
pp 1311-1313.

Item 130

ANSWER: D

Pasteurella species are isolated from up to 50% of dog bite wounds and up to 75% of cat bite wounds, and
the hand is considered a high-risk area for infection (SOR A). Although much more rare, Capnocytophaga
canimorsus, a fastidious gram-negative rod, can cause bacteremia and fatal sepsis after animal bites,
especially in asplenic patients or those with underlying hepatic disease. Anaerobes isolated from dog and
cat bite wounds include Bacteroides, Fusobacterium, Porphyromonas, Prevotella, Propionibacterium and
Peptostreptococcus.

In addition to animal oral flora, human skin flora are also important pathogens, but are less commonly
isolated. These can include streptococci and staphylococci, including methicillin-resistant Staphylococcus
aureus (MRSA). Coverage for MRSA may be especially important if the patient has risk factors for
colonization with community-acquired MRSA. Pets can also become colonized with MRSA and transmit
it via bites and scratches.

43
Cat bites that become infected with Pasteurella multocida can be complicated by cellulitis, which may form
around the wound within 24 hours and is often accompanied by redness, tenderness, and warmth. The use
of prophylactic antibiotics is associated with a statistically significant reduction in the rate of infection in
hand bites (SOR A). If infection develops and is left untreated, the most common complications are
tenosynovitis and abscess formation; however, local complications can include septic arthritis and
osteomyelitis. Fever, regional adenopathy, and lymphangitis are also seen.

Ref: Garcia VF: Animal bites and Pasteurella infections. Pediatr Rev 1997;18(4):127-130. 2) Talan DA, Citron DM,
Abrahamian FM, et al: Bacteriologic analysis of infected dog and cat bites. Emergency Medicine Animal Bite Infection
Study Group. N Engl J Med 1999;340(2):85-92. 3) Medeiros IM, Saconato H: Antibiotic prophylaxis for mammalian bites.
Cochrane Database Syst Rev 2001;(2):CD001738. 4) Rabinowitz PM, Gordon Z, Odofin L: Pet-related infections. Am Fam
Physician 2007;76(9):1314-22. 5) Sing A, Tuschak C, Hrmansdorfer S: Methicillin-resistant Staphylococcus aureus in
a family and its pet cat. N Engl J Med 2008;358(11):1200-1201.

Item 131

ANSWER: E

The epidemiology of group A streptococcal disease of the perineum is similar to that of group A
streptococcal pharyngitis, and the two often coexist. It is theorized that either auto-inoculation from mouth
to hand to perineum occurs, or that the bacteria is transmitted through the gastrointestinal tract. In one
study, the average age of patients with this disease varied from 1 to 11 years, with a mean of 5 years. Girls
and boys were almost equally affected. The incidence is estimated to be about 1 in 200 pediatric visits and
peaks in March, April, and May in North America. The condition usually presents with itching and a
beefy redness around the anus and/or vulva and will not clear with medications used to treat candidal
infections.

Ref: Mogielnicki NP, Schwartzman JD, Elliott JA: Perineal group A streptococcal disease in a pediatric practice. Pediatrics
2000;106(2 Pt 1):276-281. 2) Habif TP: Clinical Dermatology: A Color Guide to Diagnosis and Therapy, ed 5. Mosby
Elsevier, 2010, p 348. 3) Kliegman RM, Stanton BF, Geme JW III, et al (eds): Nelson Textbook of Pediatrics, ed 19.
Elsevier Saunders, 2011, pp 916-917.

Item 132

ANSWER: C

The majority of patients presenting with spontaneous pneumothorax are tall, thin individuals under 40 years
of age. Most do not have clinically apparent lung disease, and the chest pain is sometimes minimal at the
time of onset and may resolve within 24 hours even if untreated. Patients with small pneumothoraces
involving <15% of the hemithorax may have a normal physical examination, although tachycardia is
occasionally noted. The diagnosis is confirmed by chest radiographs. When a pneumothorax is suspected
but not seen on a standard chest film, an expiratory film may be obtained to confirm the diagnosis.

Studies have found that an average of 30% of patients will have a recurrence within 6 months to 2 years.
An initial pneumothorax of <20% may be monitored if the patient has few symptoms. Follow-up should
include a chest radiograph to assess stability at 2448 hours. Indications for treatment include progression,
delayed expansion, or the development of symptoms. The majority of patients with spontaneous
pneumothoraces, and perhaps almost all of them, will have subcutaneous bullae on a CT scan.

Ref: Marx JA (ed): Rosens Emergency Medicine: Concepts and Clinical Practice, ed 7. Mosby Elsevier, 2010, pp 942-943.

44
Item 133

ANSWER: D

A systematic review is a literature review focused on a research question that tries to identify, appraise,
select, and synthesize all high-quality research evidence relevant to that question. A randomized, controlled
trial (RCT) involves a group of patients who are randomized into an experimental group and a control
group. These groups are followed for the outcomes of interest. The process of randomization minimizes
bias and is thus the individual study type that is most likely to provide accurate results about an
interventions effectiveness.

A cohort study is a nonexperimental study design that follows a group of people (a cohort), and then looks
at how events differ among people within the group. A study that examines a cohort of persons who differ
in respect to exposure to some suspected risk factor such as smoking is useful for trying to ascertain
whether exposure is likely to cause specified events such as lung cancer. This study design is less reliable
due to inherent biases that may not be accounted for and may exist in the groupings of patients.

Retrospective and prospective case-control studies compare people with a disease or specific diagnosis with
people who do not have the disease. The groups are studied to find out if other characteristics are also
different between the two groups. This type of study often overestimates the benefit of a trial and is of
lower quality than a randomized, controlled trial.

Ref: OCEBM Table of Evidence Working Group: The Oxford 2011 Table of Evidence. Oxford Centre for Evidence-Based
Medicine, 2009. 2) Shaughnessy AF: Evaluating and understanding articles about treatment. Am Fam Physician
2009;79(8):668-670.

Item 134

ANSWER: B

Acid laryngitis is a group of respiratory symptoms related to gastroesophageal reflux disease. The
symptoms of hoarseness (especially in the morning), a repeated need to clear the throat, and nocturnal or
early morning wheezing may occur singly or in varying combinations, and are believed to be caused by
gastric contents irritating the larynx and hypopharynx. Thyroid disease, sinusitis, and tracheal stenosis
can produce one or more of the symptoms described, but not all of them.

Ref: Feldman M, Friedman LS, Brandt LJ: Sleisenger and Fordtrans Gastrointestinal and Liver Disease, ed 9. Saunders, 2010,
pp 179-181.

Item 135

ANSWER: A

Latex allergy management includes preventing exposure and treating reactions. Patients with latex allergy
can reduce their risk of exposure by avoiding direct contact with common latex products. Additionally,
they should be aware of foods with crossreactive proteins. Foods that have the highest association with
latex allergy include avocados, bananas, chestnuts, and kiwi. Walnuts, shellfish, strawberries, and wheat
have low or undetermined associations.

Ref: Pollart SM, Warniment C, Mori T: Latex allergy. Am Fam Physician 2009;80(12):1413-1418.

45
Item 136

ANSWER: D

The cardiac toxicity of methadone is primarily related to QT prolongation and torsades de pointes.
Ref: Death, narcotic overdose, and serious cardiac arrhythmias. FDA Alert, 2006.

Item 137

ANSWER: D

Diagnosis and appropriate treatment of genital herpes during pregnancy is particularly important because
of the high mortality in neonates who contract herpes during delivery and then develop disseminated
infection. In those who survive, there is a very high risk of serious neurologic sequelae.

HSV is acquired by deposition of the virus on a break in the skin or mucous membranes during close
physical contact with an infected person. Neonatal infection most commonly results from transmission via
the birth canal, although transplacental transmission can occur. The risk of HSV infection in the neonate
is higher during an episode of primary genital herpes than during a recurrent episode.

DNA polymerase chain reaction testing is 95% sensitive as long as an ulcer is present, and has a specificity
of 90%. The diagnosis is established by culturing the virus from an infected lesion. A Tzanck prep and
Papanicolaou smear can detect cellular changes, but both have low sensitivity. Serologic diagnosis is
mainly an epidemiologic tool and has limited clinical usefulness. Cultures of the virus by amniocentesis
have shown both false-positive and false-negative results.
Ref: Sen P, Barton SE: Genital herpes and its management. BMJ 2007;334(7602):1048-1052. 2) Gabbe SG, Niebyl JR, Simpson
JL (eds): Obstetrics: Normal and Problem Pregnancies, ed 5. Churchill Livingstone, 2007, pp 1219-1221.

Item 138

ANSWER: B

Pseudomonas organisms have been associated with outbreaks of otitis externa, dermatitis, and folliculitis
in persons using swimming pools and hot tubs.
Ref: Goldman L, Ausiello D (eds): Cecil Medicine, ed 23. Saunders, 2008, p 2956. 2) Habif TP: Clinical Dermatology: A Color
Guide to Diagnosis and Therapy, ed 5. Mosby Elsevier, 2010, pp 363-365.

Item 139

ANSWER: E

This patients symptoms and laboratory findings suggest a significant lack of TSH despite low levels of
circulating thyroid hormone. This is diagnostic of secondary hypothyroidism. Such findings should prompt
a workup for a pituitary or hypothalamic deficiency that is causing a lack of TSH production. Primary
hypothyroidism, such as Hashimotos thyroiditis, would be evidenced by an elevated TSH and low (or
normal) T4. Graves disease is a cause of hyperthyroidism, which would be expected to increase T 4 levels,
although low TSH with a normal T4 level may be present. Some nonthyroid conditions such as malnutrition
may suppress T4. In such cases the TSH would be elevated or normal. This patient has gained weight,
which does not coincide with malnutrition. The patient does not have the thyroid gland enlargement seen
with goiter.

46
Ref: Fauci AS, Braunwald E, Kasper DL, et al (eds): Harrisons Principles of Internal Medicine, ed 17. McGraw-Hill, 2008,
pp 2229-2233. 2) McPHee SJ, Papadakis MA (eds): Current Medical Diagnosis and Treatment, ed 50. McGraw-Hill Co,
2011, pp 1061-1069.

Item 140

ANSWER: A

Atrial fibrillation is the most common arrhythmia, and its prevalence increases with age. The major risk
with atrial fibrillation is stroke, and a patients risk can be determined by the CHADS2 score. CHADS
stands for Congestive heart failure, Hypertension, Age >75, Diabetes mellitus, and previous Stroke or
transient ischemic attack. Each of these is worth 1 point except for stroke, which is worth 2 points. A
patient with 4 or more points is at high risk, and 23 points indicates moderate risk. Having "1 point
indicates low risk, and this patient has 0 points.

Low-risk patients should be treated with aspirin, 81325 mg daily (SOR B). Moderate- or high-risk
patients should be treated with warfarin. Amiodarone is used for rate control, and clopidogrel is used for
vascular events not related to atrial fibrillation.

Ref: Gutierrez C, Blanchard DG: Atrial fibrillation: Diagnosis and treatment. Am Fam Physician 2011;83(1):61-68.

Item 141

ANSWER: A

Constipation is one adverse effect of opioid treatment that does not diminish with time. Thus, this effect
should be anticipated, and recommendations for prevention and treatment of constipation should be
discussed when initiating opioids. Nausea and vomiting, mental status changes, sedation, and pruritus are
also common with the initiation of opioid treatment, but these symptoms usually diminish with time, and
can be managed expectantly.

Ref: Clary PL, Lawson P: Pharmacologic pearls for end-of-life care. Am Fam Physician 2009;79(12):1059-1065.

Item 142

ANSWER: D

Lung cancer is the leading cause of cancer-related deaths in the United States. In 2006, lung cancer caused
more deaths than colorectal, breast, and prostate cancers combined.

Ref: Collins LG, Haines C, Perkel R, et al: Lung cancer: Diagnosis and management. Am Fam Physician 2007;75(1):56-63.

47
Item 143

ANSWER: E

Aspirin was once the best initial therapy for rheumatoid arthritis and then NSAIDs became the preferred
treatment. Now, however, disease-modifying drugs such as methotrexate are the best choice for initial
therapy. Aspirin and NSAIDs are no longer considered first-line treatment because of concerns about their
limited effectiveness, inability to modify the long-term course of the disease, and gastrointestinal and
cardiotoxic effects. Glucocorticoids such as prednisone are often useful, but have significant side effects.
Biologic agents such as rituximab are expensive and have significantly more side effects than methotrexate.

Ref: Scott DL, Wolfe F, Huizinga TWJ: Rheumatoid arthritis. Lancet 2010;376(9746):1094-1108.

Item 144

ANSWER: E

Respiratory viruses appear to be the most common cause of acute bronchitis; however, the organism
responsible is rarely identified in clinical practice because viral cultures and serologic assays are not
routinely performed. Fewer than 10% of patients will have a bacterial infection diagnosed as the cause of
bronchitis. For this reason, for patients with a putative diagnosis of acute bronchitis, routine treatment
with antibiotics is not justified and should not be offered. Antitussive agents are occasionally useful and
can be offered as therapy for short-term symptomatic relief of coughing.

Ref: Braman SS: Chronic cough due to acute bronchitis: ACCP evidence-based clinical practice guidelines. Chest 2006;129(1
Suppl):95S-103S.

Item 145

ANSWER: A

Insulin and glucose intravenously will provide the fastest and most consistent early lowering of serum
potassium (SOR C). Calcium is important for arrhythmia prevention, but does not lower the potassium
level. Sodium polystyrene sulfonate given orally or rectally will only lower potassium in a delayed
fashion.

Ref: Carvalhana V, Burry L, Lapinsky SE: Management of severe hyperkalemia without hemodialysis: Case report and
literature review. J Crit Care 2006;21(4):316-321. 2) Brenner BM (ed): Brenner & Rectors The Kidney, ed 8. Saunders
Elsevier, 2007, pp 573-578. 3) Weiberg LS: Management of severe hyperkalemia. Crit Care Med 2008;36(12):3246-3251.

Item 146

ANSWER: E

Trimethoprim/sulfamethoxazole is the treatment of choice for acute Pneumocystis pneumonia. Adjunctive


corticosteroids should also be started in any patient whose initial pO2 on room air is <70 mm Hg. Three
prospective trials have shown that there is a decrease in mortality and frequency of respiratory failure when
corticosteroids are used in addition to antibiotics. All of the other medications listed are effective therapy
for Pneumocystis pneumonia, but they do not need to be given with trimethoprim/sulfamethoxazole.

Ref: Mandell GL, Bennett JE, Dolin R (eds): Mandell, Douglas, and Bennetts Principles and Practice of Infectious Diseases,
ed 7. Churchill Livingstone, 2009, pp 1858-1874.

48
Item 147

ANSWER: E

Current evidence indicates that traditional sliding-scale insulin as the only means of controlling glucose in
hospitalized patients is inadequate. For patients in a surgical intensive-care unit, using an insulin drip to
maintain tight glucose control decreases the risk of sepsis but has no mortality benefit. Metformin should
be stopped if the serum creatinine level is !1.5 mg/dL in men or !1.4 mg/dL in women, or if an imaging
procedure requiring contrast is needed. In patients who have not had their hemoglobin A1c measured in the
past 30 days, this could be done to provide a better indication of glucose control. If adequate control has
been demonstrated and no contraindications are noted, the patients usual medication regimen should be
continued (SOR B).

Ref: Nau KC, Lorenzetti RC, Cucuzella M, et al: Glycemic control in hospitalized patients not in intensive care: Beyond
sliding-scale insulin. Am Fam Physician 2010;81(9):1130-1135.

Item 148

ANSWER: E

Fracture of the scaphoid should be suspected in every sprained wrist presenting with tenderness in the
anatomic snuffbox. Radiographs may be negative initially. The scaphoid circulation enters the bone for
the most part through the distal half. Fractures through the proximal third tend to cause loss of circulation
and are slower to heal, and should be referred to an orthopedist because of the risk of nonunion and
avascular necrosis. Fractures through the middle or distal one-third can be handled by the family physician
in consultation with an orthopedist. The fracture is treated with a thumb spica cast for 1012 weeks. A
wrist splint does not provide adequate immobilization. A bone scan is unnecessary, and physical therapy
is inappropriate. If there is still no evidence of union after 10 weeks of immobilization, the patient should
be referred to an orthopedist for further care.

Ref: McKeag K, Moeller J (eds): ACSM Primary Care Sports Medicine, ed 2. Lippincott, Williams & Wilkins, 2007, p 410.

Item 149

ANSWER: B

The decision to utilize the Medicare hospice benefit is reversible, and patients may elect to return to
Medicare Part A. Individuals who reside in nursing homes and assisted-living facilities are eligible for the
Medicare hospice benefit. Patients with end-stage Alzheimers disease are eligible for the Medicare hospice
benefit if they meet criteria for hospice. If the patient lacks decision-making capacity, a family member
or guardian may elect the Medicare hospice benefit for the patient. The patient must be certified by the
hospice medical director and primary physician to have a life expectancy of less than 6 months to qualify
for hospice services. This requirement is the same whether or not the patient resides in a nursing home.

Ref: Pan CX, Russo DJ: Hospice as a care option in long-term skilled nursing care. Ann Longterm Care 2010;18(12):32-37.

49
Item 150

ANSWER: A

According to CDC guidelines, the initial workup for urethritis in men includes gonorrhea and Chlamydia
testing of the penile discharge or urine, urinalysis with microscopy if no discharge is present, VDRL or
RPR testing for syphilis, and HIV and hepatitis B testing. Empiric treatment for men with a purulent
urethral discharge or a positive urine test (positive leukocyte esterase or !10 WBCs/hpf in the first-void
urine sediment) includes azithromycin, 1 g orally as a single dose, OR doxycycline, 100 mg orally twice
a day for 7 days, PLUS ceftriaxone, 125 mg intramuscularly, OR cefixime, 400 mg orally as a single dose.

If the patient presents with the same complaint within 3 months, and does not have a new sexual partner,
the tests obtained at his first visit should be repeated, and consideration should be given to obtaining
cultures for Mycoplasma or Ureaplasma and Trichomonas from the urethra or urine. Treatment should
include azithromycin, 500 mg orally once daily for 5 days, or doxycycline, 100 mg orally twice daily for
7 days, plus metronidazole, 2 g orally as a single dose.

Ref: Brill JR: Diagnosis and treatment of urethritis in men. Am Fam Physician 2010;81(7):873-878. 2) Workowski KA, Berman
S; Centers for Disease Control and Prevention (CDC): Sexually transmitted diseases treatment guidelines, 2010. MMWR
Recomm Rep 2010;59(RR-12):42-43.

Item 151

ANSWER: B

Two doses of varicella vaccine are recommended for all children unless they are immunocompromised,
in which case they should not be immunized against varicella, or with other live-virus vaccines.

Shingles is evidence of prior varicella infection and is a reason not to vaccinate with varicella vaccine.

Ref: Zimmerman RK, Middleton DB, Burns IT, et al: Routine vaccines across the life span, 2007. J Fam Pract 2007;56(2 Suppl
Vaccines):S18-S37.

Item 152

ANSWER: D

The black box warning for thiazolidinediones specifically addresses heart failure. These agents are also
contraindicated in patients with type 1 diabetes mellitus or hepatic disease, and in premenopausal
anovulatory women.

Ref: Nathan DM, Buse JB, Davidson MB, et al: Management of hyperglycemia in type 2 diabetes: A consensus algorithm for
the initiation and adjustment of therapy. Update regarding thiazolidinediones: A consensus statement from the American
Diabetes Association and the European Association for the Study of Diabetes. Diabetes Care 2008;31(1):173-175.

50
Item 153

ANSWER: B

The treatment for otitis media is evolving. Recommendations by the American Academy of Family
Physicians and the American Academy of Pediatrics advocate a 10-day course of antibiotics for children
under the age of 2 years if the diagnosis is certain. If the diagnosis is not certain and the illness is not
severe, there is an option of observation with follow-up. For children over the age of 2 years, the
recommendation is still to treat if the diagnosis is certain, but there is an option of observation and
follow-up if the illness is not severe and follow-up can be guaranteed.

Amoxicillin is the first-line therapy; the recommended dosage is 8090 mg/kg/day in two divided doses,
which increases the concentration of amoxicillin in the middle ear fluid to help with resistant
Pneumococcus.

Azithromycin, because of a broader spectrum and potential for causing resistance, is not considered the
treatment of first choice. Treatment regimens ranging from 5 to 7 days are appropriate for selected children
over the age of 5 years.

Oral decongestants and antihistamines are not recommended for children with acute otitis media.

Ref: Neff MJ; AAP; AAFP; AAO-HNS: AAP, AAFP, AAO-HNS release guideline on diagnosis and management of otitis
media with effusion. Am Fam Physician 2004;69(12):2929-2931. 2) Coleman C, Moore M: Decongestants and
antihistamines for acute otitis media in children. Cochrane Database Syst Rev 2008;(3):CD001727. 3) Hoberman A,
Paradise JL, Rockette HE, et al: Treatment of otitis media in children under 2 years of age. N Engl J Med
2011;364(2):105-115.

Item 154

ANSWER: E

In breastfeeding women, bilateral nipple pain with and between feedings after initial soreness has resolved
is usually due to Candida. Pain from engorgement typically resolves after feeding. Mastitis is usually
unilateral and is associated with systemic symptoms and wedge-shaped erythema of the breast tissue.
Improper latch-on is painful only during feedings. Eczema isolated to the nipple, while a reasonable part
of the differential, would be much more unusual.

Ref: Jatoi I, Kaufman M (eds): Management of Breast Diseases. Springer, 2010, pp 95-96.

Item 155

ANSWER: A

Azithromycin has been shown to reduce the duration of lymphadenopathy in cat-scratch disease (SOR B).
Other antibiotics that have been used include rifampin, ciprofloxacin, trimethoprim/sulfamethoxazole, and
gentamicin. Ceftriaxone, amoxicillin/clavulanate, doxycycline, and clindamycin are not effective in the
treatment of Bartonella infection.

Ref: Klotz SA, Ianas V, Elliott SP: Cat-scratch disease. Am Fam Physician 2011;83(2):152-155.

51
Item 156

ANSWER: C

The current (2004) recommendation of the U.S. Preventive Services Task Force (USPSTF) is that children
over the age of 6 months receive oral fluoride supplementation if the primary drinking water source is
deficient in fluoride. The USPSTF cites fair evidence (B recommendation) that such supplementation
reduces the incidence of dental caries and concludes that the overall benefit outweighs the potential harm
from dental fluorosis.

Dental fluorosis is chiefly a cosmetic staining of the teeth, is uncommon with currently recommended
fluoride intake, and has no other functional or physiologic consequences. Fluoridated toothpaste can cause
fluorosis in children younger than 2 years of age, and is therefore not recommended in this age group.
Fluoridated toothpaste by itself does not reliably prevent tooth decay.

Fluoride varnish, applied by a dental or medical professional, is another treatment option to prevent caries.
It provides longer-lasting protection than fluoride rinses, but since it is less concentrated, it may carry a
lower risk of fluorosis than other forms of supplementation.

Oral fluoride supplementation for children over the age of 6 months is based not only on age but on the
concentration of fluoride in the primary source of drinking water, whether it be tap water or bottled water.
Most municipal water supplies in the United States are adequately fluoridated, but concentrations vary.
Fluoride concentrations in bottled water vary widely. If the concentration is >0.6 ppm no supplementation
is needed, and may result in fluorosis if given. Lower concentrations of fluoride may indicate the need for
partial or full-dose supplementation.
Ref: Recommendations for using fluoride to prevent and control dental caries in the United States. MMWR 2001;50(RR-14):1-42.
2) US Preventive Services Task Force: Prevention of dental caries in preschool children: Recommendations and rationale.
Am Fam Physician 2004;70(8):1529-1532. 3) Riley M, Locke AB, Skye EP: Health maintenance in school-aged children:
Part I. History, physical examination, screening, and immunizations. Am Fam Physician 2011;83(6):683-688.

Item 157

ANSWER: B

An enlarged tongue (macroglossia) may be part of a syndrome found in developmental conditions such as
Down syndrome, or may be caused by a tumor (hemangioma or lymphangioma), metabolic diseases such
as primary amyloidosis, or endocrine disturbances such as acromegaly or cretinism. A bald tongue may
be associated with xerostomia, pernicious anemia, iron deficiency anemia, pellagra, or syphilis.
Ref: Fauci AS, Braunwald E, Kasper DL, et al (eds): Harrisons Principles of Internal Medicine, ed 17. McGraw-Hill, 2008,
p 219.

Item 158

ANSWER: B

This child is experiencing simple breath-holding spells, a relatively common and benign condition that
usually begins in children between the ages of 6 months and 6 years. The cause is uncertain but seems
to be related to overactivity of the autonomic nervous system in association with emotions such as fear,
anger, and frustration. The episodes are self-limited and may be associated with pallor, cyanosis, and loss
of conciousness if prolonged. There may be an association with iron deficiency anemia, but this child had
a recent normal hemoglobin level.

52
These events are not volitional, so disciplinary methods are neither effective nor warranted. While
children may experience a loss of consciousness and even exhibit some twitching behavior, the episodes
are not seizures so neither EEG evaluation nor anticonvulsant therapy is indicated. No additional
laboratory studies are indicated. Parents should be reassured that the episodes are benign and will resolve
without treatment.

Ref: Rudolph CD, Rudolph AM, Lister G, et al (eds): Rudolphs Pediatrics, ed 22. McGraw-Hill, 2011, p 2223.

Item 159

ANSWER: D

This patient has classic symptoms of allergic rhinitis. Intranasal corticosteroids are considered the mainstay
of treatment for mild to moderate cases. In multiple studies, intranasal corticosteroid sprays have proven
to be more efficacious than the other options listed, even for ocular symptoms. Air filtration systems and
bedding covers have not been shown to reduce symptoms.

Ref: Sur DK, Scandale S: Treatment of allergic rhinitis. Am Fam Physician 2010;81(12):1440-1446.

Item 160

ANSWER: A

This woman most likely has primary hyperparathyroidism due to a parathyroid adenoma or hyperplasia.
Secondary hyperparathyroidism is unlikely with normal renal function, a normal vitamin D level, and
hypercalcemia. Likewise, tertiary hyperparathyroidism is unlikely with normal renal function. The
parathyroid hormone level is suppressed with hypercalcemia associated with bone metastases. Parathyroid
hormonerelated protein, produced by cancer cells in humoral hypercalcemia of malignancy, is not
detected by the assay for parathyroid hormone.

Ref: Bilezikian JP, Silverberg SJ: Asymptomatic primary hyperparathyroidism. N Engl J Med 2004;350(17):1746-1751. 2)
Melmed S, Polonsky KS, Larsen PR, Kronenberg HM (eds): Williams Textbook of Endocrinology, ed 12. Elsevier
Saunders, 2011, pp 1260-1269.

Item 161

ANSWER: E

Uncomplicated hypertension is frequently detected in the emergency department. Many times this is a
chronic condition, but it also may result from an acutely painful situation. Hypertensive emergencies,
defined as severe blood pressure elevations to >180/120 mm Hg complicated by evidence of impending
or worsening target organ dysfunction, warrant emergent treatment. There is no evidence, however, to
suggest that treatment of an isolated blood pressure elevation in the emergency department is linked to a
reduction in overall risk. In fact, the aggressive reduction of blood pressure with either intravenous or oral
agents is not without potential risk.

The appropriate management for the patient in this scenario is simply to discharge her and ask her to
follow up with you in the near future.

Ref: American College of Emergency Physicians Clinical Policies Subcommittee (Writing Committee) on Asymptomatic
Hypertension in the ED: Clinical policy: Critical issues in the evaluation and management of adult patients with
asymptomatic hypertension in the emergency department. Ann Emerg Med 2006;47(3):237-249.

53
Item 162

ANSWER: E

This child has typical findings of bronchiolitis. The initial infection usually occurs by the age of 2 years.
It is caused by respiratory syncytial virus (RSV). Bronchodilator treatment may be tried once and
discontinued if there is no improvement. Treatment usually consists of supportive care only, including
oxygen and intravenous fluids if indicated (SOR B). Corticosteroids, antibiotics, and decongestants are of
no benefit. RSV infection may recur, since an infection does not provide immunity. Up to 10% of infected
children will have wheezing past age 5, and bronchiolitis may predispose them to asthma.

Ref: Dawson-Caswell M, Muncie HL Jr: Respiratory syncytial virus infection in children. Am Fam Physician
2011;83(2):141-146.

Item 163

ANSWER: D

Contraindications to insertion of the levonorgestrel intrauterine system (LNG-IUS) include uterine


anomalies, postpartum endometritis, untreated cervicitis, and current pelvic inflammatory disease.
Nulliparity may increase discomfort during insertion but is not a contraindication. Levonorgestrel is a
synthetic progestin and is not associated with an increased risk of deep vein thrombosis. It also is not
associated with any adverse effect on quantity or quality of milk in breastfeeding women, and has no
adverse effects on the infant. The LNG-IUS is not contraindicated in patients with endometriosis, and there
is some evidence that it may improve symptom scores in these women.

Ref: Lockhat FB, Emembolu JO, Konje JC: The evaluation of the effectiveness of an intrauterine-administered progestogen
(levonorgestrel) in the symptomatic treatment of endometriosis and in the staging of the disease. Hum Reprod
2004;19(1):179-184. 2) Paladine HL, Blenning CE, Judkins DZ, Mittal S: What are contraindications to IUDs? J Fam
Pract 2006:55(8):726-729.

Item 164

ANSWER: C

For men older than 65 years of age with small-volume, low-grade disease and a 10- to 15-year life
expectancy, the risk of complications from treatment outweighs any decreased risk of dying from prostate
cancer. Radiation, androgen deprivation therapy, and surgical approaches have not been shown to improve
disease-free survival (SOR A).

Ref: Delbanco T, Albertsen PC: Update: A 72-year-old man with localized prostate cancer14 years later. JAMA
2009;302(10):1105-1106.

54
Item 165

ANSWER: D

The sudden onset of severe abdominal pain, vomiting, and diarrhea in a patient with a cardiac source of
emboli and evidence of a separate embolic event makes superior mesenteric artery embolization likely. In
this case, evidence of a brachial artery embolus and a cardiac rhythm indicating atrial fibrillation suggest
the diagnosis. Some patients may have a surprisingly normal abdominal examination in spite of severe
pain. Microscopic hematuria and blood in the stool may both occur with embolization, and severe
leukocytosis is present in more than two-thirds of patients with this problem. Diagnostic confirmation by
angiography is recommended. Immediate embolectomy with removal of the propagated clot can then be
accomplished and a decision made regarding whether or not the intestine should be resected. A second
procedure may be scheduled to reevaluate intestinal viability.

Ref: Fauci AS, Braunwald E, Kasper DL, et al (eds): Harrisons Principles of Internal Medicine, ed 17. McGraw-Hill, 2008,
p 1910-1912.

Item 166

ANSWER: A

The threshold for prophylactic platelet transfusion is 10,000/"L (SOR A). Platelet transfusion decreases
the risk of spontaneous bleeding in such patients. A count below 50,000/"L is an indication for platelet
transfusion in patients undergoing an invasive procedure.

Ref: Slichter SJ: Evidence-based platelet transfusion guidelines. Hematology Am Soc Hematol Educ Program 2007:172-178.
2) Fauci AS, Braunwald E, Kasper DL, et al (eds): Harrisons Principles of Internal Medicine, ed 17. McGraw-Hill, 2008,
p 709.

Item 167

ANSWER: E

Ibandronate, raloxifene, denosumab, and etidronate have been shown to reduce new vertebral fractures,
but are not proven to prevent hip fracture. Only zoledronic acid, risedronate, and alendronate have been
confirmed in sufficiently powered studies to prevent hip fracture, and these are the anti-osteoporosis drugs
of choice.

Ref: Favus MJ: Bisphosphonates for osteoporosis. N Engl J Med 2010;363(21):2027-2035.

Item 168

ANSWER: D

Sensitivity is the percentage of patients with a disease who have a positive test result. Specificity is the
percentage of patients without the disease who have a negative test result. Pretest probability is the
probability of disease before a test is performed. Posttest probability is the probability of disease after a
test is performed. Positive predictive value is the percentage of patients with a positive test result who are
confirmed to have the disease.

55
Ref: Jaeschke R, Guyatt GH, Sackett DL: Users guides to the medical literature. III. How to use an article about a diagnostic
test. B. What are the results and will they help me in caring for my patients? The Evidence-Based Medicine Working
Group. JAMA 1994;271(9):703-707. 2) Shaughnessy AF: Evaluating and understanding articles about treatment. Am Fam
Physician 2009;79(8):668-670.

Item 169

ANSWER: B

First-line agents for ovulation induction and treatment of infertility in patients with polycystic ovary
syndrome (PCOS) include metformin and clomiphene, alone or in combination, as well as rosiglitazone
(SOR A). In one study of nonobese women with PCOS, metformin was found to be more effective than
clomiphene for improving the rate of conception (level of evidence 1b). However, the treatment of infertile
women with PCOS remains controversial. One recent group of experts recommended that metformin use
for ovulation induction in PCOS be restricted to women with glucose intolerance (SOR C).

Oral contraceptives are commonly used to treat menstrual irregularities in women with PCOS; however,
there are few studies supporting their use, and they would not be appropriate for ovulation induction.
Spironolactone is a first-line agent for treatment of hirsutism (SOR A) and has shown promise in treating
menstrual irregularities, but is not commonly recommended for ovulation induction. There is a high
prevalence of insulin resistance in women with PCOS, as measured by glucose intolerance;
insulin-sensitizing agents are therefore indicated, but not insulin or sulfonylurea medications.

Ref: Palomba S, Ono F Jr, Falbo A, et al: Prospective parallel randomized, double-blind, double-dummy controlled clinical trial
comparing clomiphene citrate and metformin as the first-line treatment for ovulation induction in nonobese anovulatory
women with polycystic ovary syndrome. J Clin Endocrinol Metab 2005;90(7):4068-4074. 2) Ehrmann DA: Polycystic
ovary syndrome. N Engl J Med 2005;352(12):1223-1236. 3) Legro RS, Barnhart HX, Schlaff WD, et al: Clomiphene,
metformin, or both for infertility in the polycystic ovary syndrome. N Engl J Med 2007;356(6):551-566. 4) Thessaloniki
ESHRE/ASRM-Sponsored PCOS Consensus Workshop Group: Consensus on infertility treatment related to polycystic
ovary syndrome. Fertil Steril 2008;89(3):505-522. 5) Radosh L: Drug treatments for polycystic ovary syndrome. Am Fam
Physician 2009;79(8):671-676.

Item 170

ANSWER: C

Fluoroquinolones are associated with an increased risk of tendinopathy and tendon rupture. About 1/6000
prescriptions will cause an Achilles tendon rupture. The risk is higher in those also taking corticosteroids
or over the age of 60.

Ref: OConnor NR: FDA boxed warnings: How to prescribe drugs safely. Am Fam Physician 2010;81(3):298-303.

Item 171

ANSWER: D

Pentoxifylline is effective when used with compression therapy for venous ulcers, and may be useful as
monotherapy in patients unable to tolerate compression therapy. Aspirin has also been shown to be
effective. Other treatments that have been studied but have not been found to be effective include oral zinc
and antibiotics (SOR A).

Ref: Collins L, Seraj S: Diagnosis and treatment of venous ulcers. Am Fam Physician 2010;81(8):989-996.

56
Item 172

ANSWER: C

A pulmonary capillary wedge pressure of 8 mm Hg suggests hypovolemia. Normal saline should be given
because 5% dextrose is not a reliable volume expander.

Ref: Bonow RO, Mann DL, Zipes DP, Libby P (eds): Braunwalds Heart Disease: A Textbook of Cardiovascular Medicine, ed
9. Elsevier Saunders, 2011, pp 1141-1142. 2) Tintinalli JE, Kelen GD, Stapczynski JS (eds): Emergency Medicine: A
Comprehensive Study Guide, ed 7. McGraw-Hill, 2011, p 175.

Item 173

ANSWER: D

Acute exacerbations of COPD are very common, with most caused by superimposed infections.
Supplemental oxygen, antibiotics, and bronchodilators are used for management. Systemic corticosteroids,
either oral or parenteral, have been shown to significantly reduce treatment failures and improve lung
function and dyspnea over the first 72 hours, although there is an increased risk of adverse drug reactions.

Ref: Wood-Baker RR, Gibson PG, Hannay M, et al: Systemic corticosteroids for acute exacerbations of chronic obstructive
pulmonary disease. Cochrane Database Syst Rev 2005;(4):C00075320. 2) Evensen AE: Management of COPD
exacerbations. Am Fam Physician 2010;81(5):607-613.

Item 174

ANSWER: C

Although intravenous dextrose in normal saline can initially be used for aggressive rehydration, it does not
meet the nutritional needs of patients with acute pancreatitis. Total enteral nutrition is superior to total
parenteral nutrition in stable patients with acute pancreatitis, in both mild and severe cases (SOR A). When
compared to total parenteral nutrition in these patients, enteral nutrition is associated with reduced rates
of mortality, multiple organ failure, systemic infection, and operative interventions (SOR A). Enteral
nutrition likely contributes to better outcomes by inhibiting bacterial translocation from the gut, thereby
preventing the development of infected necrosis. This patient is awake and alert and presumably able to
protect her airway, so nasogastric tube feeding is unnecessary to provide enteral nutrition.

Ref: Carroll JK, Herrick B, Gipson T, Lee SP: Acute pancreatitis: Diagnosis, prognosis, and treatment. Am Fam Physician
2007;75(10):1513-1520. 2) Al-Omran M, Albalawi ZH, Tashkandi MF, Al-Ansary LA: Enteral versus parenteral nutrition
for acute pancreatitis. Cochrane Database Syst Rev 2010(1):CD002837. 3) Kulick D, Deen D: Specialized nutrition
support. Am Fam Physician 2011;83(2):173-183.

Item 175

ANSWER: E

In controlled studies, it has been shown that a single 200-mg dose of doxycycline given within 72 hours
after an Ixodes scapularis tick bite can prevent the development of Lyme disease.

Ref: Nadelman RB, Nowakowski J, Fish D, et al: Prophylaxis with single-dose doxycycline for the prevention of Lyme disease
after an Ixodes scapularis tick bite. N Engl J Med 2001;345(2):79-84. 2) Bratton RL, Whiteside JW, Hovan MJ, et al:
Diagnosis and treatment of Lyme disease. Mayo Clin Proc 2008;83(5):566-571.

57
Item 176

ANSWER: D

Both the CDC and the American Academy of Family Physicians recommend that all adults over the age
of 65 receive a single dose of pneumococcal polysaccharide vaccine. Immunization before the age of 65
is recommended for certain subgroups of adults, including institutionalized individuals over the age of 50;
those with chronic cardiac or pulmonary disease, diabetes mellitus, anatomic asplenia, chronic liver
disease, or kidney failure; and health-care workers. It is recommended that those receiving the vaccine
before the age of 65 receive an additional dose at age 65 or 5 years after the first dose, whichever is later.

Ref: Updated recommendations for prevention of invasive pneumococcal disease among adults using the 23-valent pneumococcal
polysaccharide vaccine (PPSV23). MMWR 2010;59(34):1102-1106.

Item 177

ANSWER: B

Solitary pulmonary nodules are common radiologic findings, and the differential diagnosis includes both
benign and malignant causes. The American College of Chest Physicians guidelines for evaluation of
pulmonary nodules are based on size and patient risk factors for cancer. Lesions !8 mm in diameter with
a ground-glass appearance, an irregular border, and a doubling time of 1 month to 1 year suggest
malignancy, but smaller lesions should also be evaluated, especially in a patient with a history of smoking.

CT is the imaging modality of choice to reevaluate pulmonary nodules seen on a radiograph (SOR C). PET
is an appropriate next step when the cancer pretest probability and imaging results are discordant (SOR
C). Patients with notable nodule growth during follow-up should undergo a biopsy (SOR C).

Ref: Albert RH, Russell JJ: Evaluation of the solitary pulmonary nodule. Am Fam Physician 2009;80(8):827-831, 834.

Item 178

ANSWER: D

It has been shown that palliative care offered early in the course of a terminal disease has many benefits.
Palliative care leads to improvement in a patients quality of life and mood, and patients who receive
palliative care often have fewer symptoms of depression than those who do not receive palliative care. In
addition, palliative care reduces aggressive end-of-life care and thus reduces health care costs. Palliative
care does not reduce the need for hospice, but in fact enables patients to enter hospice care earlier and
perhaps for longer. Palliative care has been shown to extend survival times in terminal patients (SOR B).

Ref: Temel JS, Greer JA, Muzikansky A, Gallagher ER, et al: Early palliative care for patients with metastatic non-small-cell
lung cancer. N Engl J Med 2010;363(8):733-742.

58
Item 179

ANSWER: B

Prevention traditionally has been divided into three categories: primary, secondary, and tertiary. Primary
prevention targets individuals who may be at risk to develop a medical condition and intervenes to prevent
the onset of that condition (e.g., childhood vaccination programs, water fluoridation, antismoking
programs, and education about safe sex). Secondary prevention targets individuals who have developed
an asymptomatic disease and institutes treatment to prevent complications (e.g., routine Papanicolaou tests;
screening for hypertension, diabetes, or hyperlipidemia). Tertiary prevention targets individuals with a
known disease, with the goal of limiting or preventing future complications (e.g., screening diabetics for
microalbuminuria, rigorous treatment of diabetes mellitus, and postmyocardial infarction prophylaxis with
!-blockers and aspirin).

Ref: Roadmaps for clinical practice: A primer on population-based medicine. American Medical Association, 2002, pp 28-33.
2) Rakel RE, Rakel DP (eds): Textbook of Family Medicine, ed 8. Elsevier Saunders, 2011, p 74.

Item 180

ANSWER: C

Ventricular septal defect causes overload of both ventricles, since the blood is shunted left to right. The
murmur is harsh and holosystolic, generally heard best at the lower left sternal border. As the volume of
the shunting increases, cardiac enlargement and increased pulmonary vascular markings can be seen on
a chest radiograph.

Hypoplastic left heart syndrome would be manifested by near-obliteration of the left ventricle on the EKG
and chest radiograph, and the infant would be cyanotic. Transposition of the great vessels would cause
AV conduction defects and single-sided hypertrophy on the EKG. The chest radiograph would show a
straight shoulder on the left heart border where the aorta was directed to the right. Tetralogy of Fallot
causes cyanosis and right ventricular enlargement. The murmur of patent ductus arteriosus is continuous,
best heard below the left clavicle. The EKG shows left atrial and ventricular enlargement.

Ref: Rudolph CD, Rudolph AM, Lister G, et al (eds): Rudolphs Pediatrics, ed 22. McGraw-Hill, 2011, pp 1807-1808.

Item 181

ANSWER: E

Non-alcoholic fatty liver disease (NAFLD) is the most common cause of abnormal liver tests in the
developed world. Its prevalence increases with age, body mass index, and triglyceride concentrations, and
in patients with diabetes mellitus, hypertension, or insulin resistance. There is a significant overlap between
metabolic syndrome and diabetes mellitus, and NAFLD is regarded as the liver manifestation of insulin
resistance.

Statin therapy is considered safe in such individuals and can improve liver enzyme levels and reduce
cardiovascular morbidity in patients with mild to moderately abnormal liver tests that are potentially
attributable to NAFLD.

59
Ref: Athyros VG, Tziomolos K, Gossios TD, et al: Safety and efficacy of long-term statin treatment for cardiovascular events
in patients with coronary heart disease and abnormal liver tests in the Greek Atorvastatin and Coronary Heart Disease
Evaluation (GREACE) study: A post-hoc analysis. Lancet 2010;376(9756):1916-1922.

Item 182

ANSWER: C

In patients with severe pneumonia, the urine should be tested for antigens to Legionella and pneumococcus.
Two blood cultures should also be drawn, but these are positive in only 10%20% of all patients with
community-acquired pneumonia.

Ref: Niederman M: In the clinic. Community-acquired pneumonia. Ann Intern Med 2009;151(7):ITC4-2ITC4-14.

Item 183

ANSWER: B

The history and physical examination are critical for making a diagnosis in patients with scrotal pain.
Transillumination may also be performed as part of the clinical assessment. If the diagnosis is uncertain,
ultrasonography with color Doppler imaging has become the accepted standard for evaluation of the acutely
swollen scrotum (SOR B). Ultrasonography alone can confirm the diagnosis in a number of conditions,
such as hydrocele, spermatocele, and varicocele. For other conditions such as orchitis, carcinoma, or
torsion, color Doppler ultrasonography is essential because it will show increased flow in orchitis, normal
or increased flow in carcinoma, and decreased blood flow in testicular torsion.

For testicular torsion, color Doppler ultrasonography has a sensitivity of 86%88% and a specificity of
90%100%. When testicular torsion is strongly suspected, emergent surgical consultation should be
obtained before ultrasonography is performed, because surgical exploration as soon as possible is critical
to salvaging the testis and should not be delayed for imaging unless the diagnosis is in doubt.

While radionuclide imaging would be accurate for diagnosing testicular torsion, it is not used for this
purpose because of time limits and lack of easy availability. CT or MRI may be appropriate if
ultrasonography indicates a possibility of carcinoma. Plain films are not useful in assessing scrotal swelling
or masses.

Ref: Tiemstra JD, Kapoor S: Evaluation of scrotal masses. Am Fam Physician 2008;78(10):1165-1170.

Item 184

ANSWER: D

For patients who have acute back pain without sciatic involvement, a return to normal activities as tolerated
has been shown to be more beneficial than either bed rest or a basic exercise program. Bed rest for more
than 2 or 3 days in patients with acute low back pain is ineffective and may be harmful. Patients should
be instructed to remain active. Injections should be considered only if conservative therapy fails.

Ref: Kinkade S: Evaluation and treatment of acute low back pain. Am Fam Physician 2007;75(8):1181-1188.

60
Item 185

ANSWER: C

It has been recognized that patients suffering from a critical illness with an exaggerated inflammatory
response often have a relative cortisol deficiency. Clinically, this can cause hypotension that is resistant
to intravenous fluid resuscitation, and evidence is mounting that survival is increased if these patients are
treated with intravenous corticosteroids during acute management. Cortisol levels can be assessed with
a single serum reading, or by the change in the cortisol level after stimulation with cosyntropin (referred
to as $cortisol). The other hormones listed are not important for the acute management of a critically ill
patient.

Ref: Bornstein SR: Predisposing factors for adrenal insufficiency. N Engl J Med 2009;360(22):2328-2339.

Item 186

ANSWER: B

The Medicare Hospice Benefit reimburses hospice providers for the care of terminally ill patients. In order
to be eligible for this benefit, patients must be entitled to Medicare Part A and be certified by both the
personal physician and the hospice medical director as having a life expectancy of 6 months or less.
Services covered include physician services; nursing services; social services; counseling services;
physical, occupational, and speech therapy; diagnostic testing; home health aides; homemaker services;
and medical supplies. These services may be provided in the patients home or in the hospital setting.
Malignancy, ambulatory status, caregiver availability, and do-not-resuscitate orders are not specifically
related to eligibility requirements for this benefit.

Ref: Centers for Medicare and Medicaid Services: Medicare Benefit Policy Manual (Internet only). US Dept of Health and
Human Services, 2011, rev 141, pub 100-02, chap 9.

Item 187

ANSWER: E

This patients asthma is well-controlled according to the 2007 NHLBI asthma guidelines. The rule of
twos is useful in assessing asthma control: in children under the age of 12, asthma is NOT well-controlled
if they have had symptoms or used a !-agonist for symptom relief more than twice per week, had two or
more nocturnal awakenings due to asthma symptoms in the past month, or had two or more exacerbations
requiring systemic corticosteroids in the past year. For individuals over 12 years of age, there must be
more than two nocturnal awakenings per month to classify their asthma as not well controlled.

Exercise-induced asthma is considered separately. A !-agonist used as premedication before exercise is


not a factor when assessing asthma control. Since this patient does not exceed the rule of twos, her asthma
is categorized as well-controlled and no changes to her therapy are indicated. Asthma education should be
reinforced at every visit.

Ref: Expert Panel Report 3: Guidelines for the Diagnosis and Management of Asthma. National Asthma Education and
Prevention Program, NIH pub no 07-4051, 2007. 2) Pollart SM, Elward KS: Overview of changes to asthma guidelines:
Diagnosis and screening. Am Fam Physician 2009;79(9):761-767.

61
Item 188

ANSWER: E

Intravenous magnesium sulfate reduces the risk of subsequent seizures in women with eclampsia compared
with placebo, and with fewer adverse effects for the mother and baby compared with phenytoin or
diazepam. The newer oral agents have no role in this emergency.

Ref: Duley L: Clinical evidence handbook: Preeclampsia, eclampsia, and hypertension. Am Fam Physician 2009;79(10):895-896.

Item 189

ANSWER: A

A comprehensive, systematic literature review found an association of sexual abuse with a lifelong history
of functional gastrointestinal disorders, irrespective of the age of the victim at the time of abuse. There
was no statistically significant association with obesity, headache, or syncope.

Ref: Paras ML, Murad MH, Chen LP, et al: Sexual abuse and lifetime diagnosis of somatic disorders. JAMA
2009;302(5):550-561.

Item 190

ANSWER: D

People with familial adenomatous polyposis typically develop hundreds or thousands of polyps in their
colon and rectum, usually in their teens or early adulthood. Cancer usually develops in one or more of
these polyps as early as age 20. By age 40, almost all people with this disorder will have developed cancer
if preventive colectomy is not performed.

The approximate lifetime risk of colon cancer in the general population of the United States is 6%. Most
case-control studies of cigarette exposure and adenomas have found an elevated risk for smokers. Tobacco
use raises the risk of colon cancer by approximately 50%.

Patients with ulcerative colitis are at increased risk for colon cancer. The anatomic extent and duration of
the disease correlate with the degree of risk. In one meta-analysis, investigators found that the risk of colon
cancer was 2% in the first 10 years after ulcerative colitis develops, 8% during the first 20 years, and 18%
during the first 30 years.

The evidence is still evolving regarding the level of future risk of colon cancer associated with having had
an adenomatous polyp removed in the past, but it may approach a doubling of the baseline risk of colon
cancer. Studies suggest a clear association with a history of multiple polyps or a single large (>1 cm)
polyp. The data is less clear for single small adenomas. Of the three types of adenomas (tubular,
tubulovillous, and villous), villous adenomas are most likely to develop into adenocarcinomas.

Having a family history of a first degree relative with colon cancer raises the risk approximately two- to
threefold. If that relative was younger than age 50 at the time of diagnosis the risk is three- to fourfold
higher.

62
Ref: Winawer S, Fletcher R, Rex D, et al; Gastrointestinal Consortium Panel: Colorectal cancer screening and surveillance:
Clinical guidelines and rationaleUpdate based on new evidence. Gastroenterology 2003;124(2):544-560. 2) Wilkins T,
Reynolds PL: Colorectal cancer: A summary of the evidence for screening and prevention. Am Fam Physician
2008;78(12):1385-1392.

Item 191

ANSWER: A

Topical lidocaine produces very low serum levels of active drug, resulting in very few adverse effects
(SOR C). Hydrocodone could produce any opiate-type effect. Nortriptyline and duloxetine could
aggravate this patients atrial arrhythmia and cause urinary retention. Celecoxib could aggravate his reflux
problem.
Ref: Malanga G, Paster Z: Update on managing chronic pain in the elderly. J Fam Pract 2007;56(12):S11-S16.

Item 192

ANSWER: D

HIV screening is recommended as part of routine prenatal care, even in low-risk pregnancies. Counseling
about cystic fibrosis carrier testing is recommended, but not routine testing. Hepatitis C and parvovirus
antibodies are not part of routine prenatal screening. Routine screening for bacterial vaginosis with a
vaginal smear for clue cells is not recommended.
Ref: US Preventive Services Task Force: Human Immunodeficiency Virus Infection. Agency for Healthcare Research and
Quality, 2007.

Item 193

ANSWER: D

Delayed attainment of social skill milestones is the earliest and most specific sign of autism. Delayed or
odd use of language is a common, but less specific, early sign of autism. Compared with social and
language impairments, restricted interests and repetitive behaviors are less prominent and more variable
in young children. Self-injurious behaviors are associated with autism, but not specific for it. For
example, new-onset head banging may be the way an autistic child attempts to deal with pain from a dental
abscess, headache, sinusitis, otitis media, or other source of pain.
Ref: Carbone PS, Davis T: Primary care for children with autism. Am Fam Physician 2010;81(4):453-460, 461.

Item 194

ANSWER: B

Based on the American College of Radiologys Appropriateness Criteria for chronic neck pain, a complete
cervical spine series that includes five views is the correct study in a patient of any age with chronic neck
pain and no history of trauma, malignancy, or surgery. If the radiographs are normal and the patient has
neurologic signs or symptoms, the next step would be MRI. If MRI is contraindicated, CT myelography
should be offered (SOR B). A single lateral radiograph is not sufficient. Diskography is not recommended
in patients with chronic neck pain (SOR C).
Ref: Daffner RH: Radiologic evaluation of chronic neck pain. Am Fam Physician 2010;82(8):959-964.

63
Item 195

ANSWER: E

For patients with a pulmonary embolus, American College of Chest Physicians guidelines recommend
initial treatment with low molecular weight heparin (LMWH), unfractionated heparin, or fondaparinux for
at least 5 days, and then can be stopped if the INR has been !2.0 for at least 24 hours (SOR C). Warfarin
reduces the activity of coagulation factors II, VII, IX, and X produced in the liver. Coagulation factors
produced prior to initiating warfarin remain active for their usual several-day lifespan, which is why
LMWH and warfarin must be given concomitantly for at least 5 days. The INR may reach levels >2.0
before coagulation factors II and X have reached their new plateau levels, accounting for the need for an
additional 24 hours of combined therapy before stopping LMWH.
Ref: Kearon C, Kahn S, Agnelli G, et al: Antithrombotic therapy for venous thromboembolic disease: American College of
Chest Physicians evidence-based clinical practice guidelines (8th edition). Chest 2008;133(6):454S-545S.

Item 196

ANSWER: A

Cardiovascular changes associated with aging include decreased cardiac output, maximum heart rate, and
stroke volume, as well as increased systolic and diastolic blood pressure. Respiratory changes include an
increase in residual lung volume and a decrease in vital capacity. Other changes include decreases in nerve
conduction, proprioception and balance, maximum O2 uptake, bone mass, muscle strength, and flexibility.
Most of these changes, however, can be reduced in degree by a regular aerobic and resistance training
program.
Ref: Halter JB, Ouslander JG, Tinetti ME, et al (eds): Hazzards Geriatric Medicine and Gerontology, ed 6. McGraw-Hill
Medical, 2009, pp 975-976.

Item 197

ANSWER: A

Atopic dermatitis is a pruritic, inflammatory skin disorder affecting nearly 1 in 5 children residing in
developed countries. The vast majority of those eventually afflicted experience the onset of symptoms by
the age of 5 years, and more than half will present before the age of 1 year. The etiology is not fully
understood, but it seems clear that environmental, immune, genetic, metabolic, infectious, and
neuroendocrine factors all play a role. Environmental factors that may be involved include harsh
detergents, abrasive clothing, Staphylococcus aureus skin infection, food allergens (cows milk, eggs,
peanuts, tree nuts, etc.), overheating, and psychological stress. Aeroallergens that are problematic for
asthmatics, such as animal dander, dust mites, and pollen, have not been clearly linked to atopic dermatitis.

Large, well-designed studies have found no evidence that delaying the introduction of solid foods until after
6 months of age reduces the likelihood of atopic dermatitis. Ingestion of probiotic agents during pregnancy
has also not been shown to have any effect, and studies of probiotic use in breastfeeding mothers and their
infants have yielded conflicting results. Exclusive breastfeeding for the first 4 months of life has been
shown to reduce the cumulative incidence of atopic dermatitis in the first 2 years of life for infants at high
risk of developing atopic disease; doing so beyond 4 months does not appear to provide additional benefit.
Maternal dietary restriction during pregnancy and lactation has not been associated with significant benefit.
Limited studies have demonstrated that emollients and moisturizers can reduce associated xerosis and are
thought to be helpful treatments, but the data is not convincing.

64
Ref: Krakowski AC, Eichenfield LF, Dohil MA: Management of atopic dermatitis in the pediatric population. Pediatrics
2008;122(4):812-824.

Item 198

ANSWER: D

This patient most likely is suffering from recurrent migraine headaches; at the described frequency and
intensity, he meets the criteria for prophylactic medication. Ibuprofen or acetaminophen could still be used
as rescue medications, but a daily agent is indicated and propranolol is the best choice for this patient (SOR B).
Sumatriptan is not approved for children under the age of 12 years. Carbamazepine has significant side
effects and requires monitoring. Amitriptyline is a commonly used agent, but it could worsen his
constipation.

Ref: Lewis DW: Headaches in children and adolescents. Am Fam Physician 2002;65(4):625-632. 2) Damen L, Bruijn JKJ,
Verhagen AP, et al: Symptomatic treatment of migraine in children: A systematic review of medication trials. Pediatrics
2005;116(2):e295-e302. 3) Lewis DW: Pediatric migraine. Neurol Clin 2009;27(2):481-501.

Item 199

ANSWER: D

This patient has Henoch-Schnlein purpura. This condition is associated with a palpable purpuric rash,
without thrombocytopenia. Other diagnostic criteria include bowel angina (diffuse abdominal pain or bowel
ischemia), age "20, renal involvement, and a biopsy showing predominant immunoglobulin A deposition.
The long-term prognosis depends on the severity of renal involvement. Almost all children with
Henoch-Schnlein purpura have a spontaneous resolution, but 5% may develop end-stage renal disease.
Therefore, patients with renal involvement require careful monitoring (SOR A).

Ref: Reamy BV, Lindsay TJ: Henoch-Schnlein purpura. Am Fam Physician 2009;80(7):697-704.

Item 200

ANSWER: A

The Beers criteria, a list of drugs that should generally be avoided by older patients, was developed by
expert consensus, and was last updated in 2002. Indomethacin is on the list due to its propensity to
produce more central nervous system adverse effects than other NSAIDs.

Ref: Pham CB, Dickman RL: Minimizing adverse drug events in older patients. Am Fam Physician 2007;76(12):1837-1844.

Item 201

ANSWER: A

Of the many currently available medications to treat diabetes mellitus, only metformin and incretin
mimetics such as exenatide have the additional benefit of helping the overweight or obese patient lose a
significant amount of weight. Most of the other medications, including all the insulin formulations,
unfortunately lead to weight gain or have no effect on weight.

Ref: LaRocque P, Beard JM, Mintz M: Type 2 diabetes prevention and treatment. CME Bull 2009;8(1). 2) Shyangdan DS,
Royle P, Clar C, et al: Glucagon-like peptide analogues for type 2 diabetes mellitus. Cochrane Database Syst Rev
2011;10:CD006423.

65
Item 202

ANSWER: A

This patient suffers from separation anxiety disorder, which is unique to pediatric patients and is
characterized by excessive anxiety regarding separation from the home or from people the child is attached
to, such as family members or other caregivers. The anxiety is beyond what is developmentally appropriate
for the childs age. Patients may even suffer distress from anticipation of the separation. Other
characteristics include persistent worry about harm occurring to major attachment figures, worry about an
event that may separate the patient from caregivers, reluctance to attend school due to the separation it
implies, fear of being alone, recurring nightmares with themes of separation, and physical complaints when
faced with separation. Children diagnosed with separation anxiety disorder must be under 18 years of age
and have had symptoms for at least 4 weeks.

Social phobia is a persistent fear of a specific object or situation. Exposure to the object provokes an
immediate anxiety response such as a panic attack. To meet the criteria for social phobia, patients must
suffer symptoms for at least 6 months. Generalized anxiety disorder is characterized as excessive anxiety
and worry regarding a number of events or activities. Physical symptoms include restlessness, irritability,
or sleep disturbance. Symptoms must be present for at least 6 months.

Acute stress disorder occurs after a traumatic event that the individual considers life threatening. Patients
experience dissociative symptoms, flashbacks, and increased arousal. Symptoms are present for at least
2 days, with a maximum of 4 weeks. Beyond 4 weeks, a diagnosis of posttraumatic stress disorder is
made. Panic disorder with agoraphobia is characterized by recurrent panic attacks with a fear of being in
situations in which the patient cannot escape or may be embarrassed by doing so. Symptoms must be
present for 1 month for the diagnosis to be made (SOR C).

Ref: Rockhill C, Kodish I, DiBattisto C, et al: Anxiety disorders in children and adolescents. Curr Probl Pediatr Adolesc Health
Care 2010;40(4):66-99.

Item 203

ANSWER: C

The pes anserine bursa is associated with the tendinous insertion of the sartorius, gracilis, and
semitendinosus muscles into the medial aspect of the proximal tibia. Commonly associated with early
osteoarthritis in the medial knee compartment, pes anserine bursitis can also result from overuse of the
involved muscles or from direct trauma to the area. A patient with pes anserine bursitis will generally
complain of pain in the area of insertion when flexing and extending the knee and tenderness of the area
will be noted on examination. Slight swelling may be present but no effusion is generally evident.
Treatment may include oral anti-inflammatory agents, physical therapy, and corticosteroid injection.

Ref: Calmbach WL, Hutchens M: Evaluation of patients presenting with knee pain: Part II. Differential diagnosis. Am Fam
Physician 2003;68(5):917-922. 2) Fauci AS, Braunwald E, Kasper DL, et al (eds): Harrisons Principles of Internal
Medicine, ed 17. McGraw-Hill, 2008, pp 2154, 2184.

66
Item 204

ANSWER: A

The p-value is a level of statistical significance, and characterizes the likelihood of achieving the observed
results of a study by chance alone; in this study that likelihood is 5%, although 5% or less of the results
of the study can be achieved by chance alone and still be significant. The confidence interval is a measure
of variance and is derived from the test data. The p-value in and of itself says nothing about the truth or
falsity of the null hypothesis, only that the likelihood of the observed results occurring by chance is 5%.
The # or type I error is akin to the error of false-positive assignment; the ! or type II error is analogous
to the false-negative rate, or 1 specificity, and cannot be calculated from the information given.

Ref: Merrill R: Introduction to Epidemiology, ed 5. Jones & Bartlett, 2009, pp 231-236.

Item 205

ANSWER: D

Baby blues are differentiated from postpartum depression by the severity and duration of symptoms.
Baby blues occur in 80% of postpartum women and are associated with mild dysfunction. They begin
during the first 23 days after delivery and resolve within 10 days. Symptoms include brief crying spells,
irritability, poor sleep, nervousness, and emotional reactivity. An estimated 5%7% of women develop
a postpartum major depression associated with moderate to severe dysfunction during the first 3 months
post partum. While women with baby blues are at risk for progression to major depression, no more than
8%10% will progress to a major postpartum depression.

A previous history of major depressive disorder significantly increases the risk of developing postpartum
depression (RR = 4.5), and a prior episode of postpartum depression is the strongest risk factor for
postpartum depression in subsequent pregnancies. Prenatal and obstetric complications and socioeconomic
status have not consistently been shown to be risk factors. First pregnancy is also not a significant risk
factor.

Ref: Hirst KP, Moutier CY: Postpartum major depression. Am Fam Physician 2010;82(8):926-933.

Item 206

ANSWER: E

Studies suggest that in 25% of patients, irritable bowel syndrome may be caused or aggravated by one or
more dietary components. Restriction of fermentable, poorly absorbed carbohydrates is beneficial,
including fructan (found in wheat and onions), sorbitol, and other such alcohols. Further studies are
needed, however. Despite its popularity, fiber is marginally beneficial and insoluble fiber may worsen
symptoms in patients with diarrhea. Probiotics in the form of foods such as buttermilk and live-culture
yogurt have thus far not been established as useful. Daily use of peppermint oil has been shown to relieve
symptoms.

Ref: Heizer WD, Southern S, McGovern S: The role of diet in symptoms of irritable bowel syndrome in adults: A narrative
review. J Am Diet Assoc 2009;109(7):1204-1214. 2) Graham L: Practice guidelines. ACG releases recommendations on
the management of irritable bowel syndrome. Am Fam Physician 2009;79(12):1108-1117.

67
Item 207

ANSWER: A

Loss of vision is a devastating neurologic deficit that occurs with idiopathic intracranial hypertension
(pseudotumor cerebri, benign intracranial hypertension), although it is uncommon. Sixth cranial nerve
palsies may also occur as a false localizing sign. The typical presentation is a young, obese woman with
a headache, palpable tinnitus, and nausea and vomiting. CT is usually normal or shows small ventricles.
The lumbar puncture shows elevated pressure with normal fluid examination. CSF protein levels may be
low.

Hearing loss and vertigo are not characteristic of this disorder. Long tract signs and facial nerve palsies
have been attributed to idiopathic intracranial hypertension; they are atypical and should lead to
consideration of other diagnoses.

Ref: Goldman L, Ausiello D (eds): Cecil Medicine, ed 23. Saunders, 2008, pp 1448-1449.

Item 208

ANSWER: E

The U.S. Preventive Services Task Force recommends against screening adults for COPD with spirometry.
Spirometry is indicated for patients who have symptoms suggestive of COPD, but not for healthy adults.
While tobacco use is a risk factor for COPD, routine spirometry, chest radiographs, or arterial blood gas
analysis is not recommended to screen for COPD in patients with a history of tobacco use. Peak flow
measurement is not recommended for screening for COPD.

Ref: US Preventive Services Task Force: Screening for Chronic Obstructive Pulmonary Disease Using Spirometry. Agency for
Healthcare Research and Quality, 2008.

Item 209

ANSWER: C

Postcholecystectomy pain associated with jaundice (which can cause itching) is a classic presentation for
a retained common duct stone. Acetaminophen toxicity is usually painless, and is associated with ingestion
of large amounts of the drug and/or alcohol, or other potentially hepatotoxic drugs. Viral hepatitis is
usually painless and accompanied by other systemic symptoms. Hydrocodone can cause pruritus but not
pain and jaundice.

Ref: Bellows CF, Berger DH, Crass RA: Management of gallstones. Am Fam Physician 2005;72(4):637-642. 2) Roche SP,
Kobos R: Jaundice in the adult patient. Am Fam Physician 2004;69(2):299-304. 3) Townsend CM Jr, Beauchamp RD,
Evers BM, et al: Sabiston Textbook of Surgery: The Biological Basis of Modern Surgical Practice, ed 18. Saunders, 2008,
p 1572.

68
Item 210

ANSWER: E

Most women do not regularly perform breast self-examinations (BSE). Evidence from large, well-
designed, randomized trials of adequate duration has shown that the performance of regular BSE by trained
women does not reduce breast cancerspecific mortality or all-cause mortality. The 2009 update to the
U.S. Preventive Services Task Force breast cancer screening recommendations recommended against
teaching BSE (D recommendation). The rationale for this recommendation is that there is moderate
certainty that the harms outweigh the benefits. The two available trials indicated that more additional
imaging procedures and biopsies were done for women who performed BSE than for control participants,
with no gains in breast cancer detection or reduction in breast-cancer related mortality.

Ref: US Preventive Services Task Force: Screening for breast cancer: Recommendation statement. AHRQ pub no 10-5142-EF-2,
2009.

Item 211

ANSWER: E

Actinic keratoses are precursor lesions for cutaneous squamous cell carcinoma. The conversion rate of
actinic keratoses into squamous cell carcinoma has been estimated to be 1 in 1000 per year. Thicker
lesions, cutaneous horns, and lesions that show ulceration have a higher malignant potential. Although
sun exposure is a risk factor for both melanoma and basal cell carcinoma, there are no recognized
precursor lesions for either. Actinic keratosis is not a precursor lesion to keratoacanthoma.

Ref: Habif TP: Clinical Dermatology: A Color Guide to Diagnosis and Therapy, ed 5. Mosby Elsevier, 2010, p 814.

Item 212

ANSWER: D

Factors associated with a higher risk of heat-related death include being confined to bed, not leaving home
daily, and being unable to care for oneself. Living alone during a heat wave is associated with an
increased risk of death, but this increase is not statistically significant. Among medical conditions, the
highest risk is associated with preexisting psychiatric illnesses, followed by cardiovascular disease, use of
psychotropic medications, and pulmonary disease.

A lower risk of heat-related death has been noted in those who have working air conditioning, visit
air-conditioned sites, or participate in social activities. Those who take extra showers or baths and who
use fans have a lower risk, but this difference is not statistically significant.

Ref: Bouchama A, Dehbi M, Mohamed G, et al: Prognostic factors in heat waverelated deaths: A meta-analysis. Arch Intern
Med 2007;167(20):2170-2176.

69
Item 213

ANSWER: D

The National Institutes of Health Consensus Development Conference issued recommendations for gastric
bypass surgery in 1991, and these are still considered to be basic criteria (SOR C). Indications for
laparoscopic bariatric surgery for morbid obesity include a BMI >40 kg/m2 or a BMI of 3540 kg/m2 with
significant obesity-related comorbidities. Weight loss by nonoperative means should be attempted before
surgery, and patients should be evaluated by a multidisciplinary team that includes a dietician and a mental
health professional before surgery.

Ref: Pentin PL, Nashelsky J: What are the indications for bariatric surgery? J Fam Pract 2005;54(7):633-634. 2) Cameron JL
(ed): Current Surgical Therapy, ed 9. Mosby, 2008, p 1368.

Item 214

ANSWER: A

Duplex Doppler ultrasonography is the preferred initial test for renovascular hypertension in patients with
impaired renal function. Tests involving intravenous radiographic contrast material may cause
deterioration in renal function. Captopril renography is not reliable in the setting of poor renal function.
Magnetic resonance angiography also could be considered, but the association between the use of
gadolinium contrast agents and nephrogenic systemic fibrosis in patients with renal dysfunction would be
a concern.

Ref: Hartman RP, Kawashima A: Radiologic evaluation of suspected renovascular hypertension. Am Fam Physician
2009;80(3):273-279.

Item 215

ANSWER: E

Approximately 10% of women with nausea and vomiting during pregnancy require medication.
Pharmacologic therapies that have been used include vitamin B6, antihistamines, and prokinetic agents, as
well as other medications. Randomized, placebo-controlled trials have shown that vitamin B 6 is effective
for this problem. The combination of vitamin B6 and doxylamine was studied in more than 6000 patients
and was associated with a 70% reduction in nausea and vomiting, with no evidence of teratogenicity. It
is recommended by the American Congress of Obstetricians and Gynecologists as first-line therapy for
nausea and vomiting in pregnancy. A combination pill was removed from the U.S. market in 1983 because
of unjustified concerns about teratogenicity, but the medications can be bought separately over the counter.

In rare cases, metoclopramide has been associated with tardive dyskinesia, and the FDA has issued a
black-box warning concerning the use of this drug in general. The 5-HT 3-receptor antagonists, such as
ondansetron, are being used for hyperemesis in pregnancy, but information is limited. Droperidol has been
used for this problem in the past, but it is now used infrequently because of its risks, particularly heart
arrhythmias.

Ref: Niebyl JR: Nausea and vomiting in pregnancy. N Engl J Med 2010;363(16):1544-1550.

70
Item 216

ANSWER: B

This patient has a specific situational anxiety disorder or social phobia called performance anxiety or
speech phobia, characterized by marked and sometimes disabling symptoms of catecholamine excess
during specific performance situations, such as public speaking. Rates of speech phobia may exceed 50%
in the population, but it is unclear whether such fear and avoidance of public speaking warrants a
psychiatric diagnosis.

Specific phobias such as speech phobia respond moderately well to !-blockers used prior to a performance.
These drugs block peripheral anxiety symptoms such as tachycardia and tremulousness that can escalate
subjective anxiety and impair performance. Drugs that are primarily psychotropics or antiparkinsonian
agents are much less likely to be of value in this specific anxiety disorder, and may cause undesirable
sedation and dry mouth.

Ref: Sadock BJ, Sadock VA, Ruiz P (eds): Kaplan & Sadocks Comprehensive Textbook of Psychiatry, ed 9. Lippincott Williams
& Wilkins, 2009, p 1873.

Item 217

ANSWER: D

The diagnosis of Duchenne muscular dystrophy, the most common neuromuscular disorder of childhood,
is usually not made until the affected individual presents with an established gait abnormality at the age of
45 years. By then, parents unaware of the X-linked inheritance may have had additional children who
would also be at risk.

The disease can be diagnosed earlier by testing for elevated creatine kinase in boys who are slow to walk.
The mean age for walking in affected boys is 17.2 months, whereas over 75% of developmentally normal
children in the United States walk by 13.5 months. Massive elevation of creatine kinase (CK) from 20 to
100 times normal occurs in every young infant with the disease. Early detection allows appropriate genetic
counseling regarding future pregnancies.

Hypothyroidism and phenylketonuria could present as delayed walking. However, these diseases cause
significant mental retardation and would be associated with global developmental delay. Furthermore, these
disorders are now diagnosed in the neonatal period by routine screening. Disorders of amino acid
metabolism present in the newborn period with failure to thrive, poor feeding, and lethargy. Gross
chromosomal abnormalities would usually be incompatible with a normal physical examination at 18
months of age.

Ref: Pegoraro E, Hoffman EP, Piva L, et al: Cooperative International Neuromuscular Research Group: SPP1 genotype is a
determinant of disease severity in Duchenne muscular dystrophy. Neurology 2011;76(3):219-226. 2) Dooley J, Gordon
KE, Dodds L, MacSween J: Duchenne muscular dystrophy: A 30-year population-based incidence study. Clin Pediatr
(Phila) 2010;49(2):177-179.

71
Item 218

ANSWER: B

The American College of Chest Physicians recommends a starting warfarin dosage of "5 mg/day in elderly
patients, or in patients who have conditions such as heart failure, liver disease, or a history of recent
surgery. The INR should be used to guide adjustments in the dosage.

Ref: Ansell J, Hirsch J, Hylek E, et al: Pharmacology and management of the vitamin K antagonists. American College of Chest
Physicians evidence-based clinical practice guidelines (8th edition). Chest 2008;133(6 Suppl):161S.

Item 219

ANSWER: C

Testing for androgen excess is indicated in the young woman with an acute onset of hirsutism or when it
is associated with menstrual irregularity, infertility, central obesity, acanthosis nigricans, or clitoromegaly.
It should be kept in mind that excess hair has a male pattern in women with hirsutism, whereas
hypertrichosis is characterized by excessive hair growth all over the body.

Elevated early morning total testosterone is most often associated with polycystic ovary syndrome, but
other causes of hyperandrogenism and other endocrinopathies should be eliminated. These studies should
include pregnancy testing if the patient has amenorrhea, as well as a serum prolactin level to exclude
hyperprolactinemia. DHEA-S and early morning 17-hydroxyprogesterone can detect adrenal
hyperandrogenism and congenital adrenal hyperplasia. Assessment for Cushing syndrome, thyroid disease,
or acromegaly is appropriate if associated signs or symptoms are present. Pelvic ultrasonography can be
performed to evaluate for ovarian neoplasm or polycystic ovaries, although PCOS is a clinical diagnosis
and ultrasonography has a low sensitivity.

Ref: Curran DR, Moore C, Huber T: Clinical inquiries. What is the best approach to the evaluation of hirsutism? J Fam Pract
2005;54(5):465-467. 2) Martin KA, Chang JA, Ehrmann DA, et al: Evaluation and treatment of hirsutism in
premenopausal women: An Endocrine Society Clinical Practice Guideline. J Clin Endocrinol Metab 2008;93(4):1105-1120.

Item 220

ANSWER: D

High-risk patients should have a target LDL-cholesterol level of <100 mg/dL. High risk is defined as the
presence of known coronary heart disease (CHD), diabetes mellitus, noncoronary atherosclerotic disease,
or multiple risk factors for CHD (SOR C). Patients at very high risk (known CHD and multiple additional
risk factors) have an optional target of <70 mg/dL.

Ref: Evensen A, Elliott M, Hooper-Lane C: Clinical inquiries. Which patients benefit from lowering LDL to <100 mg/dL?
J Fam Pract 2010;59(12):706-708.

72
Item 221

ANSWER: C

The patient described has symptomatic arterial vascular disease manifested by intermittent claudication.
He has already initiated the two most important changes: he has stopped smoking and started a walking
program. His LDL-cholesterol is at target levels; further lowering is not likely to improve his symptoms.
In the presence of diffuse disease, interventional treatments such as angioplasty or surgery may not be
helpful; in addition, these interventions should be reserved as a last resort. Cilostazol has been shown to
help with intermittent claudication, but additional antiplatelet agents are not likely to improve his
symptoms. Fish oil and warfarin have not been found to be helpful in the management of this condition.

Ref: Arain FA, Cooper LT Jr: Peripheral arterial disease: Diagnosis and management. Mayo Clin Proc 2008;83(8):944-950.

Item 222

ANSWER: E

More than 9500 cases of calcium channel blocker intoxication were reported to U.S. poison control centers
in 2005. Substantial toxicity can occur with one or two tablets, and all children suspected of ingesting a
calcium channel blocker should be admitted to a pediatric intensive-care unit for monitoring and
management.

The use of gastric emptying, cathartics, or adsorptive agents is unlikely to be helpful and should be
considered only in patients presenting within 1 hour of ingestion, if then. The American Academy of
Pediatrics has advised that syrup of ipecac not be kept in the home because of toxicity and dubious benefit.

Ref: Hanhan UA: The poisoned child in the pediatric intensive care unit. Pediatr Clin North Am 2008;55(3):669-686.

Item 223

ANSWER: A

The patient described in this case has polycythemia vera. Pruritus after a hot shower (aquagenic pruritus)
and the presence of splenomegaly helps to clinically distinguish polycythemia vera from other causes of
erythrocytosis (hematocrit >55%). Specific criteria for the diagnosis of polycythemia vera include an
elevated red cell mass, a normal arterial oxygen saturation (>92%), and the presence of splenomegaly.
In addition, patients usually exhibit thrombocytosis (platelet count >400,000/mm3), leukocytosis (WBC
>12,000/mm3 ), a low serum erythropoietin level, and an elevated leukocyte alkaline phosphatase score.
High carboxyhemoglobin levels are associated with secondary polycythemia.

Ref: Fauci AS, Braunwald E, Kasper DL, et al (eds): Harrisons Principles of Internal Medicine, ed 17. McGraw-Hill, 2008,
pp 672-673. 2) Goldman L, Ausiello D (eds): Cecil Medicine, ed 23. Saunders, 2008, pp 1250-1252.

73
Item 224

ANSWER: D

The syndrome of delirium is common in the postoperative setting. It is characterized by disorganized


thinking; rambling, incoherent speech; and a reduced ability to maintain and shift attention. In addition,
at least two of the following are typically present: a reduced level of consciousness with perceptual
disturbances or hallucinations; sleep disturbances or changes in psychomotor activity; disorientation to
time, place, or person; and memory impairment. This syndrome typically begins abruptly and may
fluctuate hourly. There is usually a specific etiologic factor identified, such as surgery in this case.

A patient with normal vital signs, no fever, and normal laboratory studies is unlikely to be septic. Patients
with psychosis typically maintain orientation to person and place, as well as attention. Dementia with Lewy
bodies has a more chronic onset, and the absence of focal neurologic findings makes stroke unlikely.
Alcohol withdrawal is also a consideration in the differential diagnosis.

Ref: Halter JB, Ouslander JG, Tinetti ME, et al (eds): Hazzards Geriatric Medicine and Gerontology, ed 6. McGraw-Hill
Medical, 2009, pp 231-232, 647-658. 2) Mantz J, Hemmings HC Jr, Boddaert J: Case scenario: Postoperative delirium
in elderly surgical patients. Anesthesiology 2010;112(1):189-195. 3) Miller MO: Evaluation and management of delirium
in hospitalized older patients. Am Fam Physician 2008;78(11):1265-1270.

Item 225

ANSWER: A

Clostridium difficile infection is more common with aging and can be treated with either metronidazole or
vancomycin daily. For mild recurrent disease, repeating the course of the original agent is appropriate
(SOR B). Multiple recurrences or severe disease warrants the use of both agents. The effectiveness of
probiotics such as Lactobacillus remains uncertain. Intravenous vancomycin has not been effective.
Antiperistaltic drugs should be avoided.

Ref: Simor AE: Diagnosis, management, and prevention of Clostridium difficile infection in long-term care facilities: A review.
J Am Geriatr Soc 2010;58(8):1556-1564.

Item 226

ANSWER: E

An international expert committee issued a report in 2009 recommending that a hemoglobin A 1c level
!6.5% be used to diagnose diabetes mellitus. Other criteria include a fasting plasma glucose level !126
mg/dL, a random glucose level !200 mg/dL in a patient with symptoms of diabetes, or a 2-hour oral
glucose tolerance test value !200 mg/dL. While a urine dipstick may be used to screen for diabetes, it is
not a diagnostic test.

Ref: The International Expert Committee: International Expert Committee report on the role of the A1C assay in the diagnosis
of diabetes. Diabetes Care 2009;32(7):1327-1334.

74
Item 227

ANSWER: D

All women !65 (SOR A) and all men !70 (SOR C) should be screened for osteoporosis. For men and
women age 5069, the presence of factors associated with low bone density would merit screening. Risk
factors include low body weight, previous fracture, a family history of osteoporosis with fracture, a history
of falls, physical inactivity, low vitamin D or calcium intake, and the use of certain medications or the
presence of certain medical conditions.

Chronic systemic diseases that increase risk include COPD, HIV, severe liver disease, renal failure,
systemic lupus erythematosus, and rheumatoid arthritis. Endocrine disorders that increase risk include type
1 diabetes mellitus, hyperparathyroidism, hyperthyroidism, Cushings syndrome, and others. Medications
that increase risk include anticonvulsants, corticosteroids, and immunosuppressants. Nutritional risks
include celiac disease, vitamin D deficiency, anorexia nervosa, gastric bypass, and increased alcohol or
caffeine intake.

Ref: Sweet MG, Sweet JM, Jeremiah MP, Galazka SS: Diagnosis and treatment of osteoporosis. Am Fam Physician
2009;79(3):193-200.

Item 228

ANSWER: C

Treatment of hypertension reduces the risk of stroke, myocardial infarction, and heart failure. For most
patients, JNC-7 recommends a goal blood pressure of <140/90 mm Hg. However, the goal for patients
with chronic kidney disease (CKD) or diabetes mellitus is <130/80 mm Hg. Both conditions are
independent risk factors for cardiovascular disease. The National Kidney Foundation and the American
Society of Nephrology recommend treating most patients with CKD with an ACE inhibitor or angiotensin
receptor blocker (ARB), plus a diuretic, with a goal blood pressure of <130/80 mm Hg. Most patients
with CKD will require two drugs to reach this goal.

Ref: K/DOQI clinical practice guidelines for chronic kidney disease: Evaluation, classification, and stratification. Am J Kidney
Dis 2002;39(2 Suppl 1):S1-S266. 2) Chobanian AV, Bakris GL, Black HR, et al: The Seventh Report of the Joint National
Committee on Prevention, Detection, Evaluation, and Treatment of High Blood PressureThe JNC 7 Report. National
Heart Lung and Blood Institute (NHLBI), 2003.

Item 229

ANSWER: B

In patients with grade 2 ascites (visible clinically by abdominal distention, not just with ultrasonography),
the initial treatment of choice is diuretics along with salt restriction. Aldosterone antagonists such as
spironolactone are more effective than loop diuretics such as furosemide (SOR A). Chlorthalidone, a
thiazide diuretic, is not recommended. Large-volume paracentesis is the recommended treatment of grade
3 ascites (gross ascites with marked abdominal distention), and is followed by salt restriction and diuretics.

Ref: European Association for the Study of the Liver: EASL clinical practice guidelines on the management of ascites,
spontaneous bacterial peritonitis, and hepatorenal syndrome in cirrhosis. J Hepatol 2010;53(3):397-417.

75
Item 230

ANSWER: A

The indolent course of the sore described favors the diagnosis of cutaneous leishmaniasis. Neither malaria
nor schistosomiasis produces these sores. The chancres of syphilis and trypanosomiasis are more fleeting
in duration.

Ref: Goldman L, Ausiello D (eds): Cecil Medicine, ed 23. Saunders, 2008, pp 2391-2393.

Item 231

ANSWER: E

Sporulating organisms such as Clostridium difficile are not killed by alcohol products. Staphylococcus
aureus, Pseudomonas aeruginosa, and Klebsiella pneumoniae are killed by alcohol products (SOR A).

Ref: Schroeder A, Schroeder MA, DAmico F: Whats growing on your stethoscope? (And what you can do about it). J Fam
Pract 2009;58(8):404-409.

Item 232

ANSWER: D

Biceps tendon rupture is one of the most common musculotendinous ruptures. Patients typically present
with a visible lump in the upper arm following an audible, painful pop. The injury typically results from
application of an eccentric load to a flexed elbow. Risk factors for biceps tendon rupture include age >40,
deconditioning, contralateral biceps tendon rupture, a history of rotator cuff tear, rheumatoid arthritis, and
cigarette smoking. Weakness in forearm supination and elbow flexion may be present. The biceps squeeze
test and the hook test are both sensitive and specific for diagnosing the condition.

Acute anterior shoulder dislocation is typically very painful, with restricted shoulder movements. Lateral
epicondylitis results in pain and tenderness over a localized area of the proximal lateral forearm. Biceps
tendinitis results in a deep throbbing pain over the anterior shoulder, accompanied by bicipital groove
tenderness.

Ref: Churgay CA: Diagnosis and treatment of biceps tendinitis and tendinosis. Am Fam Physician 2009;80(5):470-476.

Item 233

ANSWER: C

When third-stage hookworm larvae, most commonly of the species infecting dogs and cats, penetrate the
skin and migrate through the dermis, they create the serpiginous, erythematous tracks characteristic of
cutaneous larva migrans. Although this dermatosis can occur in northern areas when conditions are ideal,
it is most often encountered in tropical and semitropical regions such as the Caribbean, Africa, Asia, and
South America.

76
Travelers to beach environments where pet feces have been previously deposited are most at risk because
of the direct contact of bare skin with the sand. As in this case, a stinging or itching sensation may be
noted upon penetration; this is followed by the development of the creeping eruption, which usually
appears 15 days later, although the onset may be delayed for up to a month. The larvae will not develop
in the human host, so the infection is self-limited, usually resolving within weeks to months. Treatment
with antihelminthic drugs can greatly reduce the clinical course. Preventive measures include treatment
of infected dogs and cats and limiting exposure to contaminated soil by wearing shoes and protective
clothing.

Ref: CDC Health Information for International Travel 2010. Centers for Disease Control and Prevention, 2010, pp 313-314.

Item 234

ANSWER: D

The EKG shown represents torsades de pointes. This special form of ventricular tachyarrhythmia is often
regarded as an intermediary between ventricular tachycardia and ventricular fibrillation. Morphologically
it is characterized by wide QRS complexes with apices that are sometimes positive and sometimes negative.
It is generally restricted to polymorphous tachycardias associated with QT prolongation.

Anything that produces or is associated with a prolonged QT interval can cause torsades de pointes,
including drugs (quinidine, procainamide, disopyramide, phenothiazines), electrolyte disturbances,
insecticide poisoning, subarachnoid hemorrhage, and congenital QT prolongation. Its great clinical
importance lies in the fact that the usual anti-arrhythmic drugs are not only useless but contraindicated,
because they can make matters worse.

Ventricular flutter is the term used by some authorities to describe a rapid ventricular tachycardia
producing a regular zigzag on EKG, without clearly formed QRS complexes. Ventricular tachycardia
consists of at least three consecutive ectopic QRS complexes recurring at a rapid rate. They are usually
regular. Ventricular fibrillation is characterized by the complete absence of properly formed ventricular
complexes; the baseline wavers unevenly, with no clear-cut QRS deflections.

Ref: Bonow RO, Mann DL, Zipes DP, Libby P (eds): Braunwalds Heart Disease: A Textbook of Cardiovascular Medicine, ed
9. Elsevier Saunders, 2011, pp 806-809.

Item 235

ANSWER: D

Fractures of the medial third of the clavicle in pediatric patients are common and are best treated by a
figure-of-8 apparatus. Open surgical reduction with intramedullary fixation will minimize angular
deformity at the fracture site but leaves a scar and may result in nonunion. With the rare exception of
neurovascular injury accompanying the fracture, there are no indications for open reduction of a clavicular
fracture in a child.

Ref: Tintinalli JE, Kelen GD, Stapczynski JS (eds): Emergency Medicine: A Comprehensive Study Guide, ed 7. McGraw-Hill,
2011, pp 570-573.

77
Item 236

ANSWER: D

The retinal findings shown are consistent with central retinal artery occlusion. The painless, unilateral,
sudden loss of vision over a period of seconds may be caused by thrombosis, embolism, or vasculitis.
Acute narrow-angle glaucoma is an abrupt, painful, monocular loss of vision often associated with a red
eye, which will lead to blindness if not treated. In persons with optic neuritis, funduscopy reveals a
blurred disc and no cherry-red spot. Occlusion of the central retinal vein causes unilateral, painless loss
of vision, but the retina will show engorged vessels and hemorrhages.

Ref: Yanoff M, Duker JS (eds): Ophthalmology, ed 3. Mosby, 2008, pp 589-592. 2) Pokhrel PK, Loftus SA: Ocular
emergencies. Am Fam Physician 2007;76(6):829-836.

Item 237

ANSWER: C

Bedbug bites are difficult to diagnose due to the variability in bite response between people and the changes
in a given individuals skin reaction over time. It is best to collect and identify bedbugs to confirm bites.
Cimex lectularius injects saliva into the bloodstream of the host to prevent coagulation. It is this saliva that
causes the intense itching and welts.

Ref: Goddard J, deShazo R: Bed bugs (Cimex lectularius) and clinical consequences of their bites. JAMA
2009;301(13):1358-1366.

Item 238

ANSWER: B

Echocardiography is the most effective imaging study for the diagnosis of pericardial effusion. It is a
simple, sensitive, specific, noninvasive test that can be used at the patients beside (SOR A). The test also
helps to quantify the amount of pericardial fluid and to detect the presence of any accompanying cardiac
tamponade. The erythrocyte sedimentation rate, WBC count, and antinuclear antibody titer are helpful
for guiding the follow-up care of patients with systemic lupus erythematosus, but not for diagnosing
precordial pain. Cardiac angiography has no role in the diagnosis of pericardial effusion.

Ref: Fauci AS, Braunwald E, Kasper DL, et al (eds): Harrisons Principles of Internal Medicine, ed 17. McGraw-Hill, 2008,
pp 1489-1490.

Item 239

ANSWER: C

In the radiograph shown, there is a fracture of the fifth metacarpal head, commonly known as a boxers
fracture. There is only slight volar angulation and no displacement. The proper treatment for this fracture
is an ulnar gutter splint, which immobilizes the wrist, hand, and fourth and fifth digits in the neutral
position. Generally, 3 or 4 weeks of continuous splinting is adequate for healing.

78
Surgical pinning is indicated in cases of significant angulation (3540 or more of volar angulation) or
in fractures with significant rotational deformity or displacement. The other options listed are not
appropriate treatments for this injury. This injury most commonly results from man-versus-wall
pugilistics, but other mechanisms of injury are possible.

Ref: Boyd AS, Benjamin HJ, Asplund C: Splints and casts: Indications and methods. Am Fam Physician 2009;80(5):491-499.

Item 240

ANSWER: E

This presentation is typical of pityriasis rosea. There was a mild prodrome, thought to be jet lag by this
patient, followed by the development of an ovoid salmon-colored, slightly raised herald patch, most
commonly seen on the trunk. This was followed by an outbreak of multiple smaller, similar lesions that
trend along Langers lines. In this case, clear evidence of the herald patch remains visible in the left
interscapular region, which is helpful in confirming the diagnosis.

Guttate psoriasis shares some features with pityriasis rosea in that it can appear suddenly and often follows
a triggering incident such as a streptococcal infection, which could be confused with a prodromal phase;
however, the absence of a herald patch and the smaller but thicker erythematous lesions differentiate
psoriasis from pityriasis rosea. Tinea versicolor often involves the upper trunk and may appear as a lightly
erythematous, scaling rash, but the onset is more gradual than in this case. Although this patient may be
exposed to low levels of radiation in her job, radiation dermatitis requires doses such as those administered
in cancer treatment protocols and would generally be limited to the field of exposure. Cutaneous T-cell
lymphoma usually presents as a nonspecific dermatitis, most commonly in men over the age of 50.

An infectious etiology for pityriasis rosea is strongly suspected, although none has been identified. There
is some evidence that the agent may be human herpesvirus 6. The illness generally resolves within 2
months, leaving no residual signs other than postinflammatory hyperpigmentation.

Ref: Stulberg DL, Wolfrey J: Pityriasis rosea. Am Fam Physician 2004;69(1):87-91, 94. 2) Habif TP: Clinical Dermatology:
A Color Guide to Diagnosis and Therapy, ed 5. Mosby Elsevier, 2010, pp 316-319.

79

S-ar putea să vă placă și